You are on page 1of 600

WWW.IIT-NEET.

XYZ

WWW.IIT-NEET.XYZ
EBD_7443
WWW.IIT-NEET.XYZ

WWW.IIT-NEET.XYZ
www.jeeneetbooks.in

WWW.IIT-NEET.XYZ
EBD_7443
WWW.IIT-NEET.XYZ

WWW.IIT-NEET.XYZ
WWW.IIT-NEET.XYZ

WWW.IIT-NEET.XYZ
EBD_7443
WWW.IIT-NEET.XYZ

WWW.IIT-NEET.XYZ
www.jeeneetbooks.in

WWW.IIT-NEET.XYZ
EBD_7443
WWW.IIT-NEET

WWW.IIT-NEET.XYZ
www.jeeneetbooks.in

WWW.IIT-NEET.XYZ
EBD_7443
www.jeeneetbooks.in

WWW.IIT-NEET.XYZ
www.jeeneetbooks.in

WWW.IIT-NEET.XYZ
EBD_7443
www.jeeneetbooks.in

WWW.IIT-NEET.XYZ
www.jeeneetbooks.in

WWW.IIT-NEET.XYZ
EBD_7443
www.jeeneetbooks.in

WWW.IIT-NEET.XYZ
www.jeeneetbooks.in

WWW.IIT-NEET.XYZ
EBD_7443
www.jeeneetbooks.in

WWW.IIT-NEET.XYZ
www.jeeneetbooks.in

WWW.IIT-NEET.XYZ
EBD_7443
www.jeeneetbooks.in

WWW.IIT-NEET.XYZ
www.jeeneetbooks.in

WWW.IIT-NEET.XYZ
EBD_7443
www.jeeneetbooks.in

WWW.IIT-NEET.XYZ
www.jeeneetbooks.in

WWW.IIT-NEET.XYZ
EBD_7443
www.jeeneetbooks.in

WWW.IIT-NEET.XYZ
www.jeeneetbooks.in

WWW.IIT-NEET.XYZ
EBD_7443
www.jeeneetbooks.in

WWW.IIT-NEET.XYZ
www.jeeneetbooks.in

WWW.IIT-NEET.XYZ
EBD_7443
www.jeeneetbooks.in

WWW.IIT-NEET.XYZ
www.jeeneetbooks.in

WWW.IIT-NEET.XYZ
EBD_7443
www.jeeneetbooks.in

WWW.IIT-NEET.XYZ
www.jeeneetbooks.in

WWW.IIT-NEET.XYZ

VITEEE
SOLVED PAPER 2017
(memory based)
GENERAL INSTRUCTIONS
• This question paper contains total 125 questions divided into four parts :
Part I : Physics Q. No - 1 to 40
Part II : Chemistry Q. No - 41 to 80
Part III : Mathematics Q. No - 81 to 120
Part IV : English Q. No - 121 to 125
• All questions are multiple choice questions with four options, only one of them is correct.
• For each correct response, the candidate will get 1 mark.
• There is no negative marking for the wrong answer.
• The test is of 2½ hours duration.

PART - I (PHYSICS) 4. Two bodies A and B having masses in the ratio


of 3 : 1 possess the same kinetic energy. The
1. A 5000 kg rocket is set for vertical firing. The ratio of linear momentum of B to A is
exhaust speed is 800 m/s. To give an initial (a) 1 : 3 (b) 3 : 1
upward acceleration of 20 m/s2, the amount of
(c) 1: 3 (d) 3 :1
gas ejected per second to supply the needed
thrust will be (Take g = 10 m/s2) 5. In which sequence the radioactive radiations are
emitted in the following nuclear reaction?
(a) 127.5 kg/s (b) 137.5 kg/s
A ¾¾
® A ¾¾
® A–4
(c) 155.5 kg/s (d) 187.5 kg/s ZX Z + 1Y Z–1K
2. The power dissipated in the circuit shown in the
figure is 30 Watts. The value of R is ¾¾ ®Z– 1 KA–4
(a) g, a, b (b) a, b, g
R
(c) b, g, a (d) b, a, g
6. Which of the following does not support the
wave nature of light?
5W (a) Interference (b) Diffraction
(c) Polarisation
10V
(d) Photoelectric effect
7. Six identical conducting rods are joined as
(a) 20 W (b) 15 W shown in figure. Points A and D are maintained
(c) 10 W (d) 30 W at 200°C and 20°C respectively. The temperature
3. If the kinetic energy of a moving particle is E, of junction B will be
then the de-Broglie wavelength is
2mE
(a) l = h 2mE (b) l =
h
h hE A B C D
(c) l= (d) l =
2mE 2mE
EBD_7443
www.jeeneetbooks.in

WWW.IIT-NEET.XYZ
2017-2 Target VITEEE
(a) 120°C (b) 100°C 13. Light of wavelength 500 nm is incident on a metal
(c) 140°C (d) 80°C with work function 2.28 eV. The de Broglie
8. A hydrogen atom is in ground state. Then to get wavelength of the emitted electron is:
six lines in emission spectrum, wavelength of (a) < 2.8 × 10-9 m (b) ³ 2.8 × 10-9 m
incident radiation should be (c) £ 2.8 × 10-12 m (d) < 2.8 × 10-10 m
(a) 800 Å (b) 825 Å 14. Kerosene oil rises up in a wick of a lantern
(c) 975 Å (d) 1025 Å because of
9. A conducting circular loop of radius r carries a
(a) diffusion of the oil through the wick
constant current i. It is placed in a uniform
r r (b) capillary action
magnetic field B0 such that B0 is perpendicular (c) buoyant force of air
to the plane of the loop. The magnetic force (d) the gravitational pull of the wick
acting on the loop is 15. The current in a coil of L = 40 mH is to be
(a) ir B0 (b) 2p ir B0 increased uniformly from 1A to 11A in 4 milli sec.
(c) zero (d) p ir B0 The induced e.m.f. will be
10. A vessel of depth 2d cm is half filled with a liquid (a) 100 V (b) 0.4 V
of refractive index m1 and the upper half with a (c) 440 V (d) 40 V
liquid of refractive index m2. The apparent depth
16. An alternating voltage of 220 V, 50 Hz frequency
of the vessel seen perpendicularly is
is applied across a capacitor of capacitance 2
æ m1 m 2 ö æ 1 1 ö µF. The impedence of the circuit is
(a) ç ÷d (b) çç m + m ÷÷ d
çm +m ÷ p 1000
è 1 2 ø è 1 2ø
(a) (b)
5000 p
æ 1 1 ö æ 1 ö
ç ÷ ç ÷
ç m + m ÷ 2d (d) ç m m ÷ 2d
(c) 5000
è 1 2ø è 1 2ø (c) 500 p (d)
p
11. A smooth sphere of mass M moving with 17. The combination of gates shown below yields
velocity u directly collides elastically with
another sphere of mass m at rest. After collision,
their final velocities are V and v respectively. A
The value of v is
2uM 2um X
(a) (b)
m M
2u 2u B
(c) (d)
m M
1+ 1+
M m
(a) OR gate (b) NOT gate
12. Two capacitors C1 and C2 in a circuit are joined (c) XOR gate (d) NAND gate
as shown in figure. The potentials of points A
18. A hollow insulated conduction sphere is given
and B are V1 and V2 respectively. Then the
a positive charge of 10 mC. What will be the
potential of point D will be
electric field at the centre of the sphere if its
A B radius is 2 metres?
V1 D V2 (a) Zero (b) 5 mCm–2
C1 C2 –2
(c) 20 mCm (d) 8 mCm–2
(V1 + V2 ) C 2 V1 + C1V2 19. Two mercury drops (each of radius r) merge to
(a) (b) form a bigger drop. The surface energy of the
2 C1 + C 2
bigger drop, if T is the surface tension, is
C1V1 + C 2 V2
(c) (d) C 2 V1 + C1V2 (a) 25/3 pr2T (b) 4 pr2T
C1 + C 2 C1 + C 2 (c) 2 pr T
2 (d) 28/3 pr2T
www.jeeneetbooks.in

WWW.IIT-NEET.XYZ
Solved Paper 2017 2017-3

20. Resistances 1 W, 2 W and 3 W are connected to 25. A milli voltmeter of 25 milli volt range is to be
form a triangle. If a 1.5 V cell of negligible internal converted into an ammeter of 25 ampere range.
resistance is connected across the 3 W resistor, The value (in ohm) of necessary shunt will be
the current flowing through this resistor will be (a) 0.001 (b) 0.01
(a) 0.25 A (b) 0.5 A (c) 1 (d) 0.05
(c) 1.0 A (d) 1.5 A 26. In young’s double-slit experiment, the intensity
21. A current carrying coil is subjected to a uniform of light at a point on the screen where the path
magnetic field. The coil will orient so that its difference is l is I, l being the wavelength of
plane becomes light used. The intensity at a point where the
l
(a) inclined at 45° to the magnetic field path difference is will be
(b) inclined at any arbitrary angle to the 4
I I
magnetic field (a) (b)
4 2
(c) parallel to the magnetic field
(c) I (d) zero
(d) perpendicular to the magnetic field
27. Which of the following is a self adjusting force?
22. The value of tan (90° – q) in the graph gives (a) Static friction (b) Limiting friction
(c) Dynamic friction (d) Sliding friction
28. Which of the following are not electromagnetic
waves?
(a) Cosmic rays (b) Gamma rays
(c) b-rays (d) X-rays
Strain

29. Graph of specific heat at constant volume for a


monatomic gas is
q
Y Y
Stress
3R
(a) Young's modulus of elasticity (a) cv (b) cv
(b) compressibility X
O O X
(c) shear strain T T
(d) tensile strength Y 3 Y
—R
23. An electron makes a transition from an excited 2
state to the ground state of a hydrogen - like (c) cv (d) cv
atom. Then O X O X
T T
(a) kinetic energy decreases, potential energy 30. A charge +q is at a distance L/2 above a square
increases but total energy remains same of side L. Then what is the flux linked with the
(b) kinetic energy and total energy decrease surface?
but potential energy increases q 2q
(c) its kinetic energy increases but potential (a) 4e 0 (b) 3e
0
energy and total energy decrease
(d) kinetic energy, potential energy and total q 6q
(c) 6e 0 (d) e
energy decrease 0
24. An A.C. source is connected to a resistive circuit. 31. The potential energy of a system increases if
Which of the following is true? work is done
(a) Current leads ahead of voltage in phase (a) upon the system by a non conservative
(b) Current lags behind voltage in phase force
(b) by the system against a conservative force
(c) Current and voltage are in same phase
(c) by the system against a non conservative
(d) Any of the above may be true depending force
upon the value of resistance. (d) upon the system by a conservative force
EBD_7443
www.jeeneetbooks.in

WWW.IIT-NEET.XYZ
2017-4 Target VITEEE
32. Two capacitors when connected in series have 39. Transfer characteristics [output voltage (V0) vs
a capacitance of 3 mF, and when connected in input voltage (Vi)] for a base biased transistor in
parallel have a capacitance of 16 mF. Their CE configuration is as shown in the figure. For
individual capacities are using transistor as a switch, it is used
(a) 1 mF, 2 mF (b) 6 mF, 2 mF
V0 I II
(c) 12 mF, 4 mF (d) 3 mF, 16 mF III
33. Resonance frequency of LCR series a.c. circuit
is f0. Now the capacitance is made 4 times, then
the new resonance frequency will become
(a) f0/4 (b) 2f0 Vi
(c) f0 (d) f0/2.
34. If the light is polarised by reflection, then the (a) in region (III)
(b) both in region (I) and (III)
angle between reflected and refracted light is (c) in region (II)
(a) 180º (b) 90º (d) in region (I)
(c) 45º (d) 36º 40. A bar magnet of magnetic moment M, is placed in
35. The velocity of efflux of a liquid through an a magnetic field of induction B. The torque exerted
orifice in the bottom of the tank does not depend on it is
r r r r
upon (a) M . B (b) - M . B
(a) size of orifice r r r r
(b) height of liquid (c) M ´ B (d) - B. M
(c) acceleration due to gravity
PART - II (CHEMISTRY)
(d) density of liquid
36. On a smooth plane surface (figure) two block A 41. Schottky defect in crystals is observed when
and B are accelerated up by applying a force 15 (a) unequal number of cations and anions are
missing from the lattice
N on A. If mass of B is twice that of A, the force
(b) equal number of cations and anions are
on B is
missing from the lattice
(c) an ion leaves its normal site and occupies an
15 N B interstitial site
A
(d) density of the crystal is increased
42. The cyclobutyl methylamine with nitrous acid
(a) 30 N (b) 15 N gives
(c) 10 N (d) 5 N CH2 OH
37. A potentiometer wire, 10 m long, has a resistance (a) (b)
of 40W. It is connected in series with a resistance
box and a 2 V storage cell. If the potential gradient (c) (d) All of these
along the wire 0.1 m is V/cm, the resistance
unplugged in the box is 43. The exothermic formation of CIF3 is represented
(a) 260 W (b) 760 W by the equation :
CI 2(g ) + 3F2(g ) 2ClF3 (g ) ; Δ H = – 329 kJ
(c) 960 W (d) 1060 W
Which of the following will increase the quantity
38. A prism has a refracting angle of 60º. When
of CIF3 in an equilibrium mixture of
placed in the position of minimum deviation, it
produces a deviation of 30º. The angle of CI 2 , F2 and CIF3 ?
incidence is (a) Adding F2
(a) 30º (b) 45º (b) Increasing the volume of the container
(c) Removing Cl2
(c) 15º (d) 60º
(d) Increasing the temperature
www.jeeneetbooks.in

WWW.IIT-NEET.XYZ
Solved Paper 2017 2017-5

44. For the reaction 47. What is Z in the following sequence of reactions?
2NO2(g) 2 NO(g) + O2(g) , Zn
Phenol¾¾®X¾¾ ¾
CH Cl
3¾®Y¾¾ ¾¾® Z Alkaline
dust Anhyd. AlCl3 KMnO 4
( K c = 1.8 ´ 10 -6 at 184°C ) (R = 0.0831 kJ/ (mol. K)
(a) Benzene (b) Toluene
When K p and K c are compared at 184°C, it is (c) Benzaldehyde (d) Benzoic acid
found that 48. Which of the following oxy-acids has the
(a) Whether K p is greater than, less than or maximum number of hydrogens directly attached
to phosphorus?
equal to K c depends upon the total gas
(a) H4P2O7 (b) H3PO2
pressure
(c) H3PO3 (d) H3PO4
(b) K p = K c 49. The number of geometrical isomers of
(c) K p is less than K c CH3CH=CH–CH=CH–CH=CHCl is
(a) 2 (b) 4
(d) K p is greater than K c
(c) 6 (d) 8
CH3 50. If ‘a’ stands for the edge length of the cubic
systems : simple cubic, body centred cubic and
45. (CH3 CO)2 O
A
Br2 /CH3COOH
B face centred cubic, then the ratio of radii of the
spheres in these systems will be respectively,
+
H /H2 O
X 1 3 1
NH2 (a) a: a: a
What is X? 2 4 2 2
CH3 CH3 1 1
(b) a : 3a : a
Br 2 2
(a) (b)
1 3 3
Br (c) a: a: a
2 2 2
NH2 NH2
(d) 1a : 3a : 2a
CH3 CH3 51. For a first order reaction A®P, the temperature
COCH3 (T) dependent rate constant (k) was found to
(c) (d) 1
COCH3 follow the equation log k = – (2000) + 6.0 . The
T
NH2 NH2 pre-exponential factor A and the activation energy
46. A compound MpXq has cubic close packing (ccp) Ea, respectively, are
arrangement of X. Its unit cell structure is shown (a) 1.0 × 106 s–1 and 9.2 kJ mol–1
below. The empirical formula of the compound is (b) 6.0 s–1 and 16.6 kJ mol–1
(c) 1.0 × 106 s–1 and 16.6 kJ mol–1
(d) 1.0 × 106 s–1 and 38.3 kJ mol–1
52. 1-Propanol and 2-propanol can be distinguished
M by
X (a) oxidation with alkaline KMnO4 followed by
reaction with Fehling solution
(b) oxidation with acidic dichromate followed by
reaction with Fehling solution
(c) oxidation by heating with copper followed
by reaction with Fehling solution
(a) MX (b) MX2 (d) oxidation with concentrated H2SO4 followed
(c) M2X (d) M5 X14 by reaction with Fehling solution
EBD_7443
www.jeeneetbooks.in

WWW.IIT-NEET.XYZ
2017-6 Target VITEEE
53. Which group contains coloured ions out of (a) 750 K (b) 1000 K
1. Cu 2+ 2. Ti 4+ (c) 1250 K (d) 500 K
58. An organic compound (A) on reduction gives
3. Co 2+ 4. Fe 2 + compound (B). (B) on treatment with CHCl3 and
(a) 1, 2, 3, 4 (b) 1, 3, 4 alcoholic KOH gives (C). (C) on catalytic
(c) 2, 3 (d) 1, 2 reduction gives N-methylaniline. The compound
54. The half life period of a first order chemical A is
reaction is 6.93 minutes. The time required for the
completion of 99% of the chemical reaction will (a) Methylamine (b) Nitromethane
be (log 2 = 0.301) (c) Aniline (d) Nitrobenzene
(a) 23.03 minutes (b) 46.06 minutes
59. The standard reduction potential for Cu 2 + /Cu
(c) 460.6 minutes (d) 230.03 minutes
55. A mixture of benzaldehyde and formaldehyde on is + 0.34. Calculate the reduction potential at pH
heating with aqueous NaOH solution gives = 14 for the above couple. (Ksp Cu (OH ) 2
(a) benzyl alcohol and sodium formate
= 1 × 10 -19 )
(b) sodium benzoate and methyl alcohol
(c) sodium benzoate and sodium formate (a) – 0.22 V (b) + 0.22 V
(d) benzyl alcohol and methyl alcohol (c) – 0.44 V (d) + 0.44 V
56. In the following reaction sequence, the correct 60. A substance C4H10O yields on oxidation a
structures of E, F and G are compound, C4H8O which gives an oxime and a
O O positive iodoform test. The original substance
on treatment with conc. H2SO4 gives C4H8. The
structure of the compound is
Ph * OH (a) CH3CH2CH2CH2OH
(b) CH3CHOHCH2CH3
Heat I
¾¾¾
®[E] ¾¾¾®
2
NaOH
[F] + [G] (c) (CH3)3COH
[* implies 13C labelled carbon) (d) CH3CH2–O–CH2CH3
O O 61. The emf of a particular voltaic cell with the cell
(a) E= * F= * – + reaction Hg 22 + + H 2 2Hg + 2H +
CH3 O Na G = CHI3
Ph Ph
is 0.65 V. The maximum electrical work of this cell
O O
when 0.5 g of H 2 is consumed.
(b) E= F= * – +
* O Na G = CHI3
Ph CH3 Ph (a) – 3.12 × 10 4 J (b) –1.25´105 J
O O
(c) 25.0 ´ 10 6 J (d) None
(c) E= F=
– + *
* O Na G = CHI3 62. The number of aldol reaction(s) that occurs in
Ph CH3 Ph
the given transformation is :
O O
CH3CHO + 4HCHO
(d) E= F=
– + *
* O Na G = CH3I OH
Ph CH3 Ph OH
conc. aq. NaOH
¾¾¾¾¾¾¾
®
57. Standard entropies of X2 , Y2 and XY3 are 60,
30 and 50 JK–1mol–1 respectively. For the reaction HO
OH
1 3
X 2 + Y2 ƒ XY3 , DH = – 30 kJ to be at (a) 1 (b) 2
2 2 (c) 3 (d) 4
equilibrium, the temperature should be:
www.jeeneetbooks.in

WWW.IIT-NEET.XYZ
Solved Paper 2017 2017-7

63. Which of the following is not intermediate in the of a quantum of light with frequency of 8 × 1015
acid catalysed reaction of benzaldehyde with 2 s –1 ?
equivalent of methanol to give acetal ? (a) 5 × 10–18 (b) 4 × 101
7
(c) 3 × 10 (d) 2 × 10–25
OCH3 + OH 2 68. The number of stereoisomers possible for a
HO H HO H compound of the molecular formula
(a) (b) CH3 – CH = CH – CH(OH) – Me is:
(b) 2 (c) 4
(d) 6 (d) 3
+ 69. The optically active tartaric acid is named as
Å
H - O- CH 3
H - C = O- CH 3
D - (+) - tartaric acid because it has a positive
HO H (a) optical rotation and is drived from D - glucose
(c) (d) (b) pH in organic solvent
(c) optical rotation and is derived from D - (+) -
64. Iron crystallizes in several modifications. At about glyceraldehyde
911°C, the bcc ' a ' form undergoes a trasition to (d) optical r otation when substituted by
fcc ' g ' form. If the distance between the two deuterium
nearest neighbours is the same in the two forms 70. Consider the reaction : N 2 + 3H 2 ® 2 NH 3
at the transition temperature, the ratio of the
carried out at constant temperature and pressure.
density of iron in fcc form (r2 ) to that of iron of
If DH and DU are the enthalpy and internal
bcc form (r1 ) at the transition temperature energy changes for the reaction, which of the
r1 following expressions is true ?
(a) = 0.918 (b) r1 = 0.718
r2 r2 (a) DH > DU (b) DH < DU
(c) DH = DU (d) DH = 0
r r
(c) 1 = 0.518 (d) 1 = 0.318 71. What is D in the following sequence of reactions ?
r2 r2
NaBH HBr
O¾¾ ¾¾4® ( i) Mg, Et 2O
A¾¾¾® B¾¾¾ ¾¾ ¾®
65. The half life of the first order reaction CH OH
3 ( ii ) H 2C = O
( iii ) H3O +
CH 3 .CHO ( g ) ¾
¾® CH 4 ( g ) + CO (g )
PCC
C ¾¾¾® D
If initial pressure of CH 3CHO (g) is 80 mm Hg CH 2 Cl 2

and the total pressure at the end of 20 minutes is CHO COOH


(a) (b)
120 mm Hg
(a) 80 min (b) 120 min OH CH 3
(c) 20 min (d) 40 min (c) (d)
CHO OH
66. A compound is soluble in conc. H2SO4. It does 72. Knowing that the chemistry of lanthanoids(Ln)
not decolourise bromine in carbon tetrachloride is dominated by its + 3 oxidation state, which of
but is oxidised by chromic anhydride in aqueous the following statements is incorrect?
sulphuric acid within two seconds, turning (a) The ionic size of Ln (III) decrease in general
orange solution to blue, green and then opaque. with increasing atomic number
The original compound is (b) Ln (III) compounds are generally colourless.
(a) a primary alcohol (b) a tertiary alcohol (c) Ln (III) hydroxide are mainly basic in
(c) an alkane (d) an ether character.
67. The values of Planck's constant is 6.63 × 10–34 Js. (d) Because of the large size of the Ln (III) ions
The velocity of light is 3.0 × 108 m s–1. Which the bonding in its compounds is
value is closest to the wavelength in nanometres predominantly ionic in character.
EBD_7443
www.jeeneetbooks.in

WWW.IIT-NEET.XYZ
2017-8 Target VITEEE
73. What is the R and S configuration for each stereogenic 77. Which of the following has maximum number of
centre in this sugar from top to bottom ? lone pairs associated with Xe ?
O H (a) XeF4 (b) XeF6
(c) XeF2 (d) XeO3
H OH
HO H 78. Which one of the following statements is not
H OH true regarding (+) Lactose ?
CH 2 OH (a) On hydrolysis (+) Lactose gives equal
(a) R, R, S (b) R, S, S amount of D(+) glucose and D(+) galactose.
(c) R, S, R (d) S, S, R
(b) (+) Lactose is a b-glycoside formed by the
74. Saponification of coconut oil yields glycerol and
(a) palmitic acid (b) sodium palmitate union of a molecule of D(+) glucose and a
(c) oleic acid (d) stearic acid molecule of D(+) galactose.
75. A certain reaction is non spontaneous at 298K. (c) (+) Lactose is a reducing sugar and does not
The entropy change during the reaction is 121 exhibit mutarotation.
J K -1 . Is the reaction is endothermic or (d) (+) Lactose, C12 H22 O11 contains 8-OH
groups.
exothermic ? The minimum value of DH for the 79. If one strand of DNA has the sequence
reaction is
ATGCTTGA, the sequence in the complimentary
(a) endothermic, DH = 36.06 kJ
strand would be
(b) exothermic, DH = – 36.06 kJ
(a) TACGAACT (b) TCCGAACT
(c) endothermic, DH = 60.12 kJ
(c) TACGTACT (d) TACGTAGT
(d) exothermic, DH = – 60.12 kJ 80. The starting reagents needed to make the azo
76. p -cresol reacts with chloroform in alkaline
compound shown below
medium to give the compound A which adds
hydrogen cyanide to form, the compound B. The CH 3CH 2 N=N OH
latter on acidic hydrolysis gives chiral carboxylic
acid. The structure of the carboxylic acid is NH 2

CH 3
(a) + ethylamine
(a)
CH 2 COOH OH
OH C2 H5 OH
CH 3
CH 2 COOH (b) +
(b)
NH 2
OH
NH 2 NH 2
CH 3
(c) +
(c)
CH(OH)COOH
OH C 2 H5
OH
CH 3 NH 2 OH
CH(OH)COOH
(d) (d) +

OH C 2 H5
www.jeeneetbooks.in

WWW.IIT-NEET.XYZ
Solved Paper 2017 2017-9

87. The acceleration of a sphere falling through a


PART - III (MATHEMATICS)
liquid is (30 – 3v) cm/s2 where v is its speed in cm/
s. The maximum possible velocity of the sphere
81. sin–1(sin 5) > x2 – 4x holds if
and the time when it is achieved are
(a) x = 2 – 9 – 2p (a) 10 cm/s after 10 second
(b) x = 2 + 9 – 2p (b) 10 cm/s instantly
(c) 10 cm/s, will never be achieved
(c) x > 2 + 9 – 2p (d) 30 cm/s, after 30 second
(d) x Î (2 – 9 – 2 p , 2 + 9 – 2 p ) 88. A straight line parallel to the line 2x – y + 5 = 0 is
82. A value of c for which conclusion of Mean Value also a tangent to the curve y2 = 4x + 5. Then the
Theorem holds for the function f (x) = loge x on point of contact is
the interval [1, 3] is (a) (2, l) (b) ( –1, 1)
(a) log3 e (b) loge3
(c) (1, 3) (d) (3, 4)
1
(c) 2 log3e (d) log3e p /2
2 sin x
83. Negation of the proposition : If we control
89. Value of ò sin x + cos x
dx is
0
population growth, we prosper
(a) If we do not control population growth, we
p -p
prosper (a) (b)
(b) If we control population growth, we do not 2 2
prosper
p
(c) We control population but we do not (c) (d) None of these
prosper 4
(d) We do not control population, but we
1
prosper 90. The range of the function f (x) = is
2 - cos3x
84. The equation z z + (2 - 3i) z + (2 + 3i) z + 4 = 0
represents a circle of radius (a) (-2, ¥) (b) [-2,3]
(a) 2 (b) 3
(c) 4 (d) 6 é1 ù æ1 ö
(c) êë 3 ,1úû (d) ç 2 ,1÷
85. The function f(x) = sin x – kx – c, where k and è ø
c are constants, decreases always when 1
(a) k > 1 (b) k ³ 1 91. The area bounded by y –1 = |x|, y = 0 and |x| =
2
(c) k < 1 (d) k £ 1 will be :
1 1 3 3 3
86. Equation = + cos q represents (a) (b)
r 8 8 4 2
(a) A rectangular hyperbola
(b) A hyperbola 5
(c) (d) None of these
(c) An ellipse 4
(d) A parabola
EBD_7443
www.jeeneetbooks.in

WWW.IIT-NEET.XYZ
2017-10 Target VITEEE
92. The value of x obtained from the equation 96. Let A be the centre of the circle x2 + y2 – 2x–4y –
20 = 0, and B( 1,7) and D(4,–2) are points on the
x+a b g circle then, if tangents be drawn at B and D,
g x +b a =0 which meet at C, then area of quadrilateral ABCD
will be
a b x+g is -
(a) 150 (b) 75
(a) 0 and -(a + b + g ) (c) 75/2 (d) None of these
1
(b) 0 and a + b + g 97. ò0 [f ( x)g"( x ) - f " (x ) g (x )] dx is equal to :
(c) 1 and (a - b - g)
[Given f(0) = g(0) = 0]
2 2 2 (a) f(1) g(1) – f(1)g’(1)
(d) 0 and a + b + g
(b) f(1) g’(1) + f’(1)g(1)
93. The solution of the differential equation (c) f(1) g’(1) – f’(1)g(1)
dy y (d) none of these
log x + = sin 2x is
dx x 7-i
98. If z = then z14 =
1 3 - 4i
(a) y log | x |= C - cos x (a) 27 (b) 27 i
2
1 (c) 214 i (d) - 27 i
(b) y log | x |= C + cos 2x
2 99. The difference between greatest and least value
1 é 3p ù
(c) y log | x |= C - cos 2x of f (x) = 2 sin x + sin 2x, x Î ê0, ú is –
2 ë 2û
1
(d) xylog | x |= C - cos 2x 3 3 3 3
2 (a) (b) -2
2 2
æ x2 xö
94. lim ç - ÷= 3 3
x ®¥ è 3x - 2 3 ÷ø
ç (c) +2 (d) None of these
2
1 2 100. A and B are two independent witnesses (i.e. there
(a) (b)
3 3 is no collision between them) in a case. The
-2 2 probability that A will speak the truth is x and
(c) (d) the probability that B will speak the truth is y. A
3 9
r r r r r r and B agree in a certain statement. The
95. If ((a ´ b ) ´ (c ´ d )).( a ´ d ) = 0 , then which of probability that the statement is true is
the following is always true ?
r r x–y xy
(a) ar, b , cr, d are necessarily coplanar (a) (b)
x+y 1 + x + y + xy
r r
(b) either a or d must lie in the plane of
r r x–y xy
b and c (c) (d)
1 – x – y + 2 xy 1 – x – y + 2 xy
r r
(c) either b or c must lie in the plane of
101. A and B are events such that P(A È B)=3/4,
r r
a and d P(A Ç B)=1/4, P( A ) =2/3 then P ( A Ç B) is
r r (a) 5/12 (b) 3/8
(d) either a or b must lie in the plane of
r r (c) 5/8 (d) 1/4
c and d
www.jeeneetbooks.in

WWW.IIT-NEET.XYZ
Solved Paper 2017 2017-11

102. The line which passes through the origin and


intersect the two lines X 1 2 3 4 5 6 7 8
p(X) 0.15 0.23 0.12 0.10 0.20 0.08 0.07 0.05
x - 1 y + 3 z - 5 x - 4 y + 3 z - 14
= = , = = , is
2 4 3 2 3 4 For the events E = {X is a prime number} and
x y z x y z F = {X < 4}, then P(E È F) is
(a) = = (b) = =
1 -3 5 -1 3 5 (a) 0.50 (b) 0.77
x y z x y z (c) 0.35 (d) 0.87
(c) = = (d) = =
1 3 -5 1 4 -5 1 1 7
108. The value of tan -1 + tan -1 + tan -1 is
p/4
2 3 8
103. If u n = ò 0 tan n q dq then un + un–2 is :
7
(a) tan -1 (b) cot -1 15
8
1 1
(a) (b) 15
n -1 n +1 (c) tan -1 15 (d) tan -1
24
1 1 109. The parabola having its focus at (3, 2) and
(c) (d)
2n - 1 2n + 1 directrix along the y-axis has its vertex at
104. Ten different letters of an alphabet are given,
æ3 ö
words with five letters are formed from these (a) (2, 2) (b) ç , 2÷
è2 ø
given letters. Then the number of words which
have at least one letter repeated is
æ1 ö æ2 ö
(a) 69760 (b) 30240 (c) ç , 2÷ (d) ç , 2÷
è2 ø è3 ø
(c) 99784 (d) None of these
105. The area bounded by f (x) = x2, 0 £ x £ 1, é -1 2 5 ù
g(x) = - x + 2,1£ x £ 2 and x - axis is 110. The rank of the matrix êê 2 -4 a - 4ú is
ú
ëê 1 -2 a + 1ûú
3 4
(a) (b) (a) 1 if a = 6 (b) 2 if a = 1
2 3
(c) 3 if a = 2 (d) 1 if a = 4
8
(c) (d) None of these
3 cos x 1 0
111. If f (x ) = 1 2 cos x 1 , then
x y
106. The condition that the line + = 1 be a 0 1 2 cos x
p q
normal to the parabola y2 = 4ax is] p/2

(a) 3
p = 2ap + aq2 2 (b) 3 2
p = 2aq + ap 2
ò f (x)dx is equal to
0
(c) q 3 = 2ap 2 + aq 2 (d) None of these
1 1
107. A random variable X has the probability (a) (b) –
4 3
distribution
1
(c) (d) 1
2
EBD_7443
www.jeeneetbooks.in

WWW.IIT-NEET.XYZ
2017-12 Target VITEEE
112. The distance of the point (1, –2, 3) from the plane 117. The volume V and depth x of water in a vessel
x - y + z = 5 measured parallel to the line 2
are connected by the relation V = 5x – x and
x y z -1 6
= = is
2 3 -6 the volume of water is increasing , at the rate of
5 cm3/sec, when x = 2 cm. The rate at which the
(a) 1 (b) 2
depth of water is increasing, is
(c) 4 (d) 2 3
5 1
113. The tangent lines to the curve y2 = 4ax at points (a) cm / sec (b) cm / sec
18 4
where x = a, are
(a) parallel (b) perpendicular 5
(c) cm / sec (d) None of these
(c) inclined at 60º (d) inclined at 30º 16
114. If the eccentricity of the hyperbola
11 8. If vectors aiˆ + ˆj + kˆ, iˆ + bjˆ + kˆ and iˆ + ˆj + ckˆ
2 2 2
x - y cos ec a = 25 is 5 times the (a ¹ b ¹ c ¹ 1) are coplanar, then find
1 1 1
+ + .
eccentricity of the ellipse x 2 cos ec 2 a + y 2 = 5 , 1- a 1 - b 1- c
then a is equal to : (a) 0 (b) 1
(c) –1 (d) 2
3
(a) tan -1
2 (b) sin -1
4
é3 - 2 4 ù
ê ú
119. If matrix A = ê1 2 - 1ú and
-1 2 -1 2
(c) tan (d) sin êë0 1 1 úû
5 5

115. The conditional ( p Ù q) Þ p is 1


A -=
1
adj (A) , then k is
(a) A tautology k
(b) A fallacy i.e., contradiction (a) 7 (b) – 7
(c) Neither tautology nor fallacy (c) 15 (d) – 11
(d) None of these 120. The angle between a pair of tangents drawn
116. The set of points of discontinuity of the function from a point T to the circle
x 2 + y 2 + 4 x - 6 y + 9 sin 2 a + 13 cos 2 a = 0 is 2a .
(2 sin x ) 2 n
f(x) = lim is given by The equation of the locus of the point T is
n ®¥ 3 n - (2 cos x ) 2 n
(a) x 2 + y 2 + 4x - 6y + 4 = 0
ì p ü
(a) R (b) ínp ± , n Î I ý (b) x 2 + y 2 + 4x - 6y - 9 = 0
î 3 þ
(c) x 2 + y 2 + 4x - 6y - 4 = 0
ì p ü
(c) ínp ± , n Î I ý (d) None of these
î 6 þ (d) x 2 + y 2 + 4x - 6y + 9 = 0
www.jeeneetbooks.in

WWW.IIT-NEET.XYZ
Solved Paper 2017 2017-13

PART - IV (ENGLISH) Directions (Q. 124) : In the question below a sentence


is given, a part of which is printed in bold and
Directions (Qs. 121-123): Study the paragraph and underline. This part may contain a grammatical error.
answer the questions that follow: Each sentence is followed by phrases a, b, c and d.
At this stage of civilisation, when many nations are Find out which phrase should replace the phrase given
brought into close and vital contact for good and evil, in bold/underline to correct the error, if there is any to
it is essential, as never before, that their gross make the sentence grammatically meaningful and
ignorance of one another should be diminished, that correct.
they should begin to understand a little of one another's 124. There are any number of skilled writers who can
historical experience and resulting mentality. It is the develop content and create marketing materials
fault of the English to expect the people of other with a keen eye to using proven methods, but
countries to react as they do, to political and also to developing new and innovative techniques.
international situations. Our genuine goodwill and (a) with a keen eye to using proven methods,
good intentions are often brought to nothing, because but also to developing new and innovative
we expect other people to be like us. This would be techniques.
corrected if we knew the history, not necessarily in (b) with a keen eye for using proven methods,
detail but in broad outlines, of the social and political and also to developing new and innovative
conditions which have given to each nation its present techniques.
character. (c) with a keen eye not only to using proven
121. The character of a nation is the result of its methods, but also to developing new and
(a) gross ignorance innovative techniques.
(b) cultural heritage (d) with a keen eye to using proven methods,
(c) socio-political conditions but to developing new and innovative
(d) mentality techniques.
122. According to the author Mentality' of a nation is 125. Choose the best pronunciation of the word,
mainly product of its Sorbet from the following options.
(a) present character (a) Sore-bet
(b) international position (b) Sore-bay
(c) politics (c) Sorb rhymes with orb
(d) history (d) Shore-bay
123. The need for a greater understanding between
nations
(a) is more today than ever before
(b) was always there
(c) is no longer there
(d) will always be there
EBD_7443
www.jeeneetbooks.in

WWW.IIT-NEET.XYZ
2017-14 Target VITEEE

SOLUTIONS
PART - I (PHYSICS) PB mB vB
=
1. (d) Given : Mass of rocket (m) = 5000 Kg PA mA vA
Exhaust speed (v) = 800 m/s
Acceleration of rocket (a) = 20 m/s2 mA
mB mB 1
Gravitational acceleration (g) = 10 m/s2 = = =
We know that upward force mA mB mA 3
F = m (g + a) = 5000 (10 +20)
= 5000 × 30 = 150000 N. 5. (d)
We also know that amount of gas ejected 6. (d) Photoelectric effect does not support the
wave nature of light.
æ dm ö F 150000
ç ÷= = = 187.5 kg / s 7. (c) The equivalent electrical circuit of the
è dt ø v 800
arrangement is shown in figure.
2. (c) The power dissipated in the circuit.
2R
V2
P= ...(i)
Req B
A R C R D
v = 10 volt 2R
1 1 1 5+ R Temperature difference between the end
= + =
Req R 5 5R points A and D = 200 – 20 = 180°C
As the resistances for the three parts are
æ 5R ö equal, the temperature difference must be
Req = çè ÷
5 + Rø distribuited equally in the three parts (= 180/
P = 30 W 3 = 60°C)
Substituting the values in equation (i) \ Temperature of B = 200°C – 60° = 140°C.
8. (c) Number of possible spectral lines emitted
(10) 2
30 = when an electron jumps back to ground
æ 5R ö
çè ÷ n(n - 1)
5 + Rø state from n th orbit =
2
15 R n(n - 1)
= 10 Þ 15R = 50 + 10R Here, =6Þn =4
5+ R 2
5R = 50 Þ R = 10 W Wavelength l from transition from n = 1 to
1 n = 4 is given by,
3. (c) E = mv 2 or mv = 2m E
2 1 æ1 1 ö 16
= R ç - 2÷ Þl = = 975 Å
h h l è1 4 ø 15R
so l = =
mv 2m E 9. (c) The magnetic field is perpendicular to the
plane of the paper. Let us consider two
1 2 1 2 diametrically opposite elements. By
4. (c) As mA v A = mB v B
2 2 Fleming's Left hand rule on element AB the
direction of force will be Leftwards and the
vA mB
= magnitude will be
;
vB mA dF = Idl B sin 90° = IdlB
www.jeeneetbooks.in

WWW.IIT-NEET.XYZ
Solved Paper 2017 2017-15

x x x x x x x 20
´ 10-34
3
x xB x x Cxdl x x =
2 ´ 9 ´10 -31 ´ 0.2 ´1.6 ´10 -19
dF dl dF
A D
x x x x Ix x x 25
lmin = × 10–9
x x x x x x x 9
= 2.80 × 10–9 nm \ l ³ 2.8 × 10–9 m
On element CD, the direction of force will
be towards right on the plane of the papper 14. (b) Kerosene oil rises up in wick of a lantern
and the magnitude will be dF = IdlB. because of capillary action. If the surface
10. (b) Apparent depth = d/m1 + d/m2 tension of oil is zero, then it will not rise, so
11. (c) By law of conservation of momentum, oil rises up in a wick of a lantern due to
Mu = MV + mv surface tension.
-3
....(i) LdI 40 ´10 (11 - 1)
15. (a) e= = = 100V
| v1 - v 2 | dt 4 ´10-3
Also e = Þ Mu = Mv - MV 16. (d) Impedence of a capacitor is XC = 1/wC
| u1 - u 2 |
1 1 5000
....(ii) XC = = = .
2 pfC 2 p ´ 50 ´ 2 ´ 10 - 6 p
From (i) and (ii), 2Mu = (M + m)v
17. (a) The final boolean expression is,
2uM 2u
Þv =
M+m
Þ v=
m ( )
X = A . B = A + B = A + B Þ OR gate
1+ 18. (a) Charge resides on the outer surface of a
M
conducting hollow sphere of radius R. We
12. (c) Consider the potential at D be ‘V’. consider a spherical surface of radius r < R.
Potential drop across C1 is (V – V1) and C2 By Gauss theorem
i s ++ +++
++ +
(V2 – V) +
+ +
+ +
\ q 1 = C1(V - V1 ), q 2 = C2 (V2 - V) + R +
+ O +
As q1 = q2 [capacitors are in series] + S +
+ r +
E +
\ C1 (V - V1) = C2 (V2 - V) +
+ +
+ +
C1V1 + C2 V2
++
+ + ++
V= rr
C1 + C2 1
ò
s
E.ds = ´ charge
e0
enclosed or
13. (b) Given : work function f of metal = 2.28 eV
Wavelength of light l = 500 nm = 500 × 10– 2 1
9m E ´ 4pr= ´0 ÞE =0
e0
hc i.e electric field inside a hollow sphere is
KEmax = –f zero.
l
19. (d) Let R be the radius of the bigger drop, then
6.6 ´ 10-34 ´ 3 ´ 108 Volume of bigger drop = 2 × volume of small
KEmax = – 2.82 drop
5 ´ 10-7
4 3 4
KEmax = 2.48 – 2.28 = 0.2 eV pR = 2 ´ pr 3 Þ R = 21/ 3 r
3 3
h h Surface energy of bigger drop,
lmin = =
p 2m ( KE )max E = 4pR 2T = 4 ´ 2 2 / 3 pr 2T = 28 / 3 pr 2T
EBD_7443
www.jeeneetbooks.in

WWW.IIT-NEET.XYZ
2017-16 Target VITEEE
20. (b) Equivalent resistance between A and B = 26. (b) For path difference l, phase
series combination of 1 W and 2 W in parallel
æ 2p 2p ö
with 3 W resistor.. difference = 2p ç Q = x = .l = 2p ÷
è l l ø
Þ I = I0 + I0 + 2I0 cos 2p
Þ I = 4I0 (\ cos 2p = 1)
l p
For x = , phase difference =
4 2
p
\ I' = I1 + I 2 + 2 I1 I 2 cos
2
I I
If I1 = I2 = I0 then I ' = 2I0 = 2. =
1 1 1 2 4 2
= + = or R = 1.5 W.
R 3 3 3 27. (a) Static friction is a self adjusting force in
\ Current in the circuit is I = V/R = 1.5/1.5 = magnitude and direction.
1A. 28. (a) Cosmic rays are coming from outer space,
Since the resistance in arm ACB = resistance having high energy charged particles, like
in arm AB = 3 W, the current divides equally a-particle, proton etc. b-rays are stream of
in the two arms. Hence the current through high energy electrons, coming from the
the 3 W resistor = I/2 = 0.5 A. nucleus of radioactive atoms.
21. (d)
29. (c) For a monatomic gas
stress
22. (a) tan(90° - q) = 3
strain Cv = R
2
ze 2 k ze 2 So correct graph is
23. (c) U = –K ; T.E = –
r 2 r
k ze 2 ­ 3 / 2R
K.E = . Here r decreases
2 r Cv
24. (c) When resistance is connected to A.C
source, then current & voltage are in same
phase. T®
25. (a) Galvanometer is converted into ammeter, by
connected a shunt, in parallel with it. q
30. (c)
G L/2
I q
L L/2

L L
S L
The given square of side L may be
GS VG 25 ´ 10-3 considered as one of the faces of a cube
==
G+ S I 25 with edge L. Then given charge q will be
considered to be placed at the centre of the
GS
= 0.001W cube. Then according to Gauss's theorem,
G+S the magnitude of the electric flux through
Here S << G so S = 0.001 W the faces (six) of the cube is given by
www.jeeneetbooks.in

WWW.IIT-NEET.XYZ
Solved Paper 2017 2017-17

f = q/e0 36. (c) The acceleration of both the blocks =


Hence, electric flux through one face of the
cube for the given square will be 15 5
=
1 q 3x x
f' = f =
6 6e 0 5
31. (d) When work is done upon a system by a \ Force on B = ´ 2x = 10 N
x
conservative force then its potential energy
increases. 37. (b) Potential gradient along wire
C1 C 2 potential difference along wire
32. (c) Cs = =3 =
C1 + C 2 length of wire
C p = C1 + C 2 = 16 \ C1 C 2 = 48
I ´ 40
2
or, 0.1 ´10 -3 = V / cm
C1 - C 2 = (C1 + C 2 ) - 4 C1 C 2 1000

= 16 2 - 4 ´ 48 = 64 = 8 1
or, Current in wire, I= A
C1 + C2 = 16 mF 400
C1 - C2 = 8 mF
Þ 2C1 = 24mF Þ C1 = 12mF 2 1
or, = or R = 800 - 40 = 760 W
48 40 + R 400
\ C2 = = 4mF
12
A + d m 60 + 30
33. (d) In LCR series circuit, resonance frequency 38. (b) i= = = 45 º
f0 is given by 2 2
39. (b) I ® ON
1 1 1
Lw = Þ w2 = \ w= = 2pf0 II ® OFF
Cw LC LC
In IInd state it is used as a amplifier it is
1 1 active region.
\ f0 = or f 0a
2p LC C r r
When the capacitance of the circuit is made 40. (c) t = M B sin q = M ´ B
4 times, its resonant frequency become f 0' PART - II (CHEMISTRY)
f0' C f0
\ = or f 0' = 41. (b) It is stoichiometric defect and it is observed
f0 4C 2 when equal number of cations and anions
34. (b) On polarisation by reflection, the reflected are missing from the lattice site.
and refracted waves are at 90º to each other.
35. (a) v = velocity of efflux through an orifice OH
|
= CH 2 NH 2 :N = O
:

2gH 42. (d)

v= 0
H OH
H | | HÅ
CH 2 - Å N - N - O ¾¾¾®
v |
:

H
It is independent of the size of orifice.
EBD_7443
www.jeeneetbooks.in

WWW.IIT-NEET.XYZ
2017-18 Target VITEEE
45. (b)
H OH
| |
CH 2 - Å N - N :- O ¾-¾
2 H 2O
¾ ¾® CH3 CH3
| |
H OH (CH 3CO) 2O Br2/CH3COOH

Å
CH 2 - N º N : ¾
¾® NH2 NHCOCH3

CH3 CH3
+
CH 2 + : N º N: +
Br /CH COOH H
+
-H
Br Br
| CH 2
| NHCOCH3 NH2
|
| (–NHCOCH3 is more electron-releasing than
|
| Å –CH3)
|¾¾ - H+
® ¾ ¾¾ ® 1
46. (b) No. of M atoms = ´ 4+1=1+1=2
OH - 4
1 1
OH No. of X atoms = ×6+ ×8 =3+1 =4
2 8
So, formula = M2X4 = MX2
47. (c)
43. (a) The reaction given is an exothermic reaction OH
thus according to Le chatalier’s principle CH3Cl, anhy. AlCl3
Zn
lowering of temperature, addition of F2 and dust Friedel-Craft reac.
Cl2 favour the forward direction and hence
Phenol Benzene, X
the production of ClF3.
44. (d) For the reaction:-
ˆˆ†
2NO2 ( g ) ‡ˆˆ 2NO(g) + O2 (g) CH3 COOH

Given Kc = 1.8 × 10–6 at 184 ºC CH Cl, anhy. AlCl KMnO4


Friedel-Craft reac.
R = 0.00831 kJ/mol. K
Dn g
Toluene,Y Benzoic acid, Z
K p = K c ( RT )
48. (b)
Dn g = 3 - 2 = 1
OH OH
Kp= 1.8 × 10–6 × 0.00831 × 457 (a) H4P2O7 ÞO P O P O
= 6.836 × 10–6
OH OH
Hence it is clear that Kp > Kc
Pyrophosphoric acid
www.jeeneetbooks.in

WWW.IIT-NEET.XYZ
Solved Paper 2017 2017-19

O Ea
51. (d) log k = log A - …(1)
P 2.303RT
(b) H3PO2 Þ OH H
H
1
Hypophosphorous acid Also given log k = 6.0 - (2000)
T
O …(2)
(c) H3PO3 Þ HO – P – OH On comparing equations, (1) and (2)
H log A = 6.0 Þ A = 106 s–1
Phosphorous acid Ea
and 2.303 R = 2000 ;
O
(d) H3PO4 Þ HO – P – OH Þ Ea = 2000 × 2.303 × 8.314
OH = 38.29 kJ mol–1
orthophosphoric acid 52. (c)
49. (d) The given structure has three double bonds
whose each carbon atom is differently (a)
substituted hence number of geometrical
alk. KMnO4
isomers will be 2n = 23’ = 8, where n is the CH 3CH 2CH 2OH ¾¾ ¾¾ ¾¾ ¾® CH 3CH 2CHO
or K 2Cr2O7 / H +
number of double bonds whose each carbon
atom is differently substituted. Fehling sol.
CH3CH 2 CH 2 OH ¾¾¾¾¾¾¾ ® CH3CH 2 CHO ¾¾ ® CH3CH 2 COOH ¾¾¾¾¾® No reaction
50. (a) Following generalization can be easily derived
for various types of lattice arrangements in (b)
cubic cells between the edge length (a) of
alk. KMnO4
the cell and r the radius of the sphere. CH3CHOHCH3 ¾¾¾¾¾¾¾ ® CH3COCH3
or K 2Cr2O7 /H +
a
For simple cubic : a = 2r or r =
2 ¾¾ ® CH 3COOH ¾¾¾¾
Feh sol.
® No reaction
For body centred cubic :
(c) CH 3CH 2 CH 2OH ¾¾® Cu
CH 3CH 2 CHO
4 3
a= r or r = a
3 4 Fehling
¾¾¾¾ ® Cu 2O ¯
solution (red)
For face centred cubic :
1
a = 2 2r or r = a Cu
2 2 CH 3CHOHCH 3 ¾¾® CH 3COCH3
Thus the ratio of radii of spheres for these
Fehling
will be ¾¾¾¾
® No reaction
solution
simple : bcc : fcc
(d) CH3CH 2CH 2OH or
a 3 1
= : a: a i.e. Conc. H2SO4
2 4 2 2 CH3CHOHCH3 ¾¾¾¾¾¾ ® CH 3CH = CH 2
option (a) is correct answer.
EBD_7443
www.jeeneetbooks.in

WWW.IIT-NEET.XYZ
2017-20 Target VITEEE

53. (b) Cu 2 + [ Ar ]3d 9 , Ti 4 + [ Ar ]3d 0 1 3


57. (a) ΔS for the reaction X 2 + Y2 ƒ XY3
2 2
Co 2 + [ Ar ]3d 7 , Fe2 + [ Ar ]3d 6
ΔS = 50 – (30 + 60) = – 40 J
1,3,4 are coloured ions hence the answer is b.
54. (b) For first order reaction, For equilibrium DG = 0 = DH – T DS

0.693 0.693 DH -30000


k= = T= = = 750 K
t1/ 2 6.93 DS -40
58. (d)
2.303 100
k= log
t 100 - 99
NO2 NH2
0.693 2.303 100
= log
6.93 t 1 reduction
¾¾ ¾ ¾®
CHCl
¾¾ ¾
¾3® C
alc. KOH
0.693 2.303 ´ 2
=
6.93 t NC NHCH3
t = 46.06 min
H / Catalyst
55. (a) Benzaldyde and formaldehyde, both do not ¾¾2 ¾¾ ¾
¾®
have a - hydrogen atom, so both will undergo
Cannizzaro reaction; here formaldehyde will N-Methylaniline
always be oxidised to formate while the other
aldehyde (C6H5CHO or any other aldehyde
not having a-H, viz- Me3CCHO) will always 59. (a) When pH = 14 [H + ] = 10-14
be reduced to correspondin g alcohol
(crossed Cannizzaro reaction) and [OH - ] = 1 M

a NaOH
Ksp = [Cu 2 + ] [OH - ]2 = 10 -19
CHO + HCHO ¾¾ ¾
¾®
10 -19
\ [Cu 2+ ] = = 10 -19
Benzaldehyde Formaldehyde [OH - ]2
a - + The half cell reaction
CH2OH + HCOO Na
Cu 2+ + 2e - ¾
¾® Cu
Benzyl alcohol sod. formate 0 .059 1
E = E° – log
56. (c) O
O O 2 [ Cu 2+ ]
C Heat C
¾¾¾® * 0.059 1
Ph * OH - CO 2 CH3 = 0.34 – log -19 = – 0.22 V
Ph 2
(b - keto acid) 10
(E)
Conc.H SO Oxi
O 60. (b) C4H8 ¬¾¾¾¾¾
2 4¾ C H O ¾¾¾
4 10 ®
(-H 2O)
I /NaOH
¾¾¾¾
2
® (–) (+)
*
Ph ONa + CHI3 C4H8O (R- COCH3 )
(F) (G) Thus C4H8O should be CH3CH2COCH3,
hence C4H10O should be CH3CH2CHOHCH3
www.jeeneetbooks.in

WWW.IIT-NEET.XYZ
Solved Paper 2017 2017-21

61. (a) Wmax = – n.FE; H


|
Wmax = – 2 × 96500 × 0.65 = – 1.25 ´ 10 5 J Ph - C O CH 3 -H 2 O

: :
0.5g H 2 = 0.25 mole.
|
Å OH 2
Hence,
H
Wmax = 1.25 ´ 10 5 ´ 0.25 = –3.12 ´ 10 4 J CH 3
| Å

: :
– j
Ph O
a C O CH 3
62. (c) CH3 – CHO + HCHO ¾¾¾¾
®
OH H

:
1st aldol
condensation
H
a CH2 OH
|
CH2 CHO –
a

: :
OH / HCHO Ph - C O CH 3
2nd aldol
CH CHO | - H+
CH2 OH condensation O
CH2 OH Å
CH 3 H
CH2OH

OH / HCHO
3rd aldol
HOCH2 C CHO H
condensation |
CH2OH

: :
Ph - C O CH 3
|
O
: :
CH2OH |
– CH 3
OH / HCHO
HOCH2 C CH2OH
Cannizzaro
reaction 64. (a) In a – form distance between nearest
CH2OH

H 3a1
neighbour atom is .
| 2
63. (b) Ph - C = O + H +
In g form distance between nearest
H
| CH 3 neighbour atom is a 2 .
j - CÅ O
: :

Ph 2
| H
OH
3 a1 a 2
\ = (given)
H 2 2
| Å
CH 3
j-C O - H+
Ph a2 3
| =
:

H or
OH a1 2

H 3 3
| r1 z1 æ a2 ö 1 æ 3 ö÷
+ H+ = ç ÷ = ç = 0.918
j-C ça ÷
: :

Ph O CH 3 r2 z2 è 1ø 2 çè 2 ÷ø
|
OH
EBD_7443
www.jeeneetbooks.in

WWW.IIT-NEET.XYZ
2017-22 Target VITEEE
*
65. (c) CH 3CHO (g ) ¾
¾® CH 4 (g) + CO (g) 68. (b) CH 3 – CH = CH – CHCH 3
|
OH
When t = 0 p 0 0 0
exhibits both geometrical as well as optical
When t = t p - p0 p p isomerism.
cis - R cis - S
\ p 0 - p + p + p = 120 mm Hg trans - R trans - S
or, p 0 + p = 120 mm Hg; 69. (c) Positive sign is for optical rotation (dextro
rotatory) and D - is for configuration. It is
p = 120 – 80 = 40 mm Hg
derived from
1 p0 1 80 1 CHO
k= ln = ln = ln 2
0
t p -p 20 80 - 40 20 |
H– C – OH
D (+) glyceraldehyde
|
ln2 CH2OH
Again, t1 / 2 =
k
CHO
ln2 |
\ t1 / 2 = ´ 20 = 20 min. L (–)glyceraldehyde is HO– C – H
ln2
|
66. (a) Solubility of the compound in conc. H2SO4 CH2OH
indicates that it can be an alkene, alcohol or
70. (b) DH = DU + DnRT for
an ether. The inability to discharge bromine
colour indicates absence of an alkene. Hence N 2 + 3H 2 ¾
¾® 2 NH 3
the compound is an alcohol which should be Dng = 2 – 4 = – 2
primary because it is readily (within 2
seconds) oxidised by CrO3 in sulphuric acid. \D H = D U - 2 RT or DU = D H+ 2R T \D U > D H

ch c NaBH
67. (b) E = hu = ;and u = 71. (a) O ¾¾ ¾¾

OH
l l CH 3OH

HBr (i) Mg. Et O


¾¾
¾® Br ¾¾¾¾¾
2¾®
3.0 ´108
8 ´1015 = (ii ) H 2C = O
l
CH 2 OH
3.0 ´ 108
\ l= = 0.37 ´ 10-7 PCC
8 ´ 1015 ¾¾ ¾¾® CHO
CH 2Cl 2

72. (b) Most of the Ln 3+ compounds except La3+


= 37.5 ´ 10-9 m = 4 ´ 101 nm
and Lu3+ are coloured due to the presence of
f-electrons.
www.jeeneetbooks.in

WWW.IIT-NEET.XYZ
Solved Paper 2017 2017-23

O H O F
C 2
H OH O - C - H
73. (c) HO H 4 H - C - OH 1
H OH
CH 2 OH HO - C - H
77. (c) XeF2 : Xe
H - C - OH 3
CH 2 OH

2 1 F
3lp
4 C 1 º 3 C 2 R configurations

3 4
Arrange the groups in order of priority by
following the text. F F
74. (b) Saponification (alkaline hydrolysis) of oils
and fats gives glycerol and sodium salt of Xe
fatty acids, which is sodium palmitate in the
present question XeF4 : F F
75. (a) For non spontaneous reaction
DG = + ve 2lp
DG = DH – T DS and
DS = 121 J K -1
F
For DG = + ve F
DH has to be positive. Hence the reaction is
endothermic.
XeF6 : F Xe F
The minimum value of DH can be obtained
by putting DG = 0
F
DH = TDS = 298 × 121 J F
= 36.06 kJ
76. (c)
CH3 CH3
CHCl3 + NaOH HCN
Reimer Tiemman reaction C=O Xe
OH OH H
XeO3 : O O
CH3 CH3 O
OH HOH
1lp
C CN CH(OH).COOH

OH H OH Hence XeF2 has maximum no. of lone pairs


Cyanohydrin
of electrons.
EBD_7443
www.jeeneetbooks.in

WWW.IIT-NEET.XYZ
2017-24 Target VITEEE
82. (c) Using Lagrange's Mean Value Theorem
78. (c) Let f(x) be a function defined on [a, b]
H OH
CH2OH f (b) - f (a)
HO
O then, f '(c) = ....(i)
OH H H b-a
H O
OH H H
H H H O OH c Î [a, b]
H OH CH2OH
1
\ Given f(x) = logex \ f'(x) =
x
(Lactose) \ equation (i) become
All reducing sugar shows mutarotation.
1 f (3) - f (1)
79. (a) On the basis of structure of guanine and =
c 3 -1
complementary bases present in them, we
can say that if the sequence of bases in one 1 log e 3 - log e 1 log e3 2
Þ = = Þ c=
strand of DNA is I, then the sequence in the c 2 2 log e 3
second strand should be II
Þ c = 2 log3e
A:T : G : C :T :T : G :A I 83. (c) p : we control population, q : we prosper
T : A : C : G : A : A : C : T II \ we have p Þ q
Its negation is ~ (p Þ q) i.e. p Ù ~ q
80. (b) C2H5 — NH 2
i.e., we control population but we do not
NaNO - HCl prosper.
¾¾ ¾ ¾
2 ¾¾®
C2 H5 N = N – Cl 84. (b) Consider the equation
0 - 5°C
zz + (2 - 3i ) z + (2 + 3i ) z + 4 = 0 ...(1)
C2H5 N = N – Cl + OH Let z = x + iy and z = x – iy, z z = x2 + y2
Put value of z, z and z z in equation (1),
® C2H5 N=N OH. we get
(x2 + y2) + (2 – 3i) (x + iy) + (2 + 3i) (x – iy)
PART - III (MATHEMATICS) +4=0
3p 5p Þ 4x + 6y + 4 + x2 + y2 = 0
81. (d) <5<
2 2 Now, we make it perfect square
Þ x2 + y2 + 4x + 6y + 4 + 4 + 9 = 4 + 9
Þ sin–1 (sin 5) = 5 – 2p
Þ (x + 2)2 + (y + 3)2 = 9
Given sin–1 (sin 5) > x2 – 4x
This represents a circle of radius 3.
Þ x2 – 4x + 4 < 9 – 2p 85. (b) Let f (x) = sin x – kx – c where k and c
Þ (x – 2)2 < 9 – 2p are constants
f '(x) = cos x – k
Þ – 9 – 2p < x – 2 < 9 – 2 p
\ f decreases if cos x £ k
Þ 2 – 9 – 2p < x < 2 + 9 – 2p Thus, f (x) = sin x – kx – c decrease
always when
k > 1.
www.jeeneetbooks.in

WWW.IIT-NEET.XYZ
Solved Paper 2017 2017-25

86. (b) Given, equation is Given line is 2x – y + 5 = 0


Þ y = 2x + 5
1 1 3 8
= + cos θ or = 1 + 3cos θ slope of line is 2. Therefore,
r 8 8 r
2
l = 2 Þ y =1
which is the form of = 1 + e cos θ y
r
Q e = 3 > 0, put y = 1 in the equation of curve, we get
\ Given equation is a hyperbola. 1 = 4x + 5
87. (c) The differential equation of the motion is x=–1
Hence, point of contact is (– 1, 1)
dv
= 30 - 3v ....(i)
dt p /2
sin x
dv
89. (c) Let I = ò sin x + cos x
dx … (i)
0
Þ = dt .
30 - 3v
Then, I =
Integrating we get
p /2
sin(p / 2 - x)
1
- log (30 - 3v) = t + C ò sin(p / 2 - x) + cos( p / 2 - x)
dx
3 0

Þ log (30 - 3v) = -3(t + c) p /2


cos x
Þ 30 - 3v = e -3t -3c
= Ae -3 t
, A =e -3c Þ I= ò cos x + sin x
dx … (ii)
0

Þ 3v = 30 - Ae -3 t ....(ii) Adding (i) and (ii), we get


2I
dv
For maximum velocity = 0
dt p /2 p /2
sin x cos x
Þ 30 - 3v = 0 from (i) ò =
cos x + sin x
dx + ò sin x + cos x
dx
0 0
30
\v = = 10 cm / s p/2 p/2
3 sin x + cosx p /2 p
which is the maximum velocity ò =
sin x + cosx
dx = ò 1.dx =[x]0 = -0
2
0 0
dv
However from (ii) = 3Ae -3t p /2
dt p sin x p
Þ I=
4
Þ ò sin x + cos x
dx =
4
dv 0
Clearly = 0 if t ® ¥
dt 90. (c) We have
\ The maximum velocity will be achieved after
– 1 £ cos 3x £ 1 Þ – 1 £ – cos 3x £ 1
infinite time in other words, the maximum
velocity will hever be reached. Add ‘2’ on both side
88. (b) Given curve is y2 = 4x + 5 1 £ 2 – cos 3x £ 3
on differentiating, we get
1 1
dy dy 2 Þ 1³ ³ .
2y =4Þ = 2 - cos 3x 3
dx dx y
EBD_7443
www.jeeneetbooks.in

WWW.IIT-NEET.XYZ
2017-26 Target VITEEE
91. (c) The given lines are,
1 β γ 1 β γ
y – 1 = x, x ³ 0; y – 1 = – x, x < 0
1 x +β α =0 Þ 0 x α - γ =0
1 1
y = 0; x=- , x < 0; x = , x ³ 0 1 β γ 0 0 x
2 2
so that the area bounded is as shown in the
Þ x2 = 0 Þ x = 0
figure.
\ Solutions of the equation are x = 0,
-(a + b + g )

dy y sin 2x
93. (c) + =
dx x log x log x

dx
ò x log x
I.F. = e

1
ò dt
\ I.F. = e t = elog t = t = log | x |
12
Required area = 2 ò0 (1 + x ) dx solution is given by

12
y (I.F.) = ò Q.(I.F.) dx + C
æ x 2 ö÷ æ1 1ö 5
= 2ç x + = 2ç + ÷ = sin 2x
ç 2 ÷ è 2 8ø 4
è ø0 y log | x | = ò log | x |
(log | x |) dx + C

x +α β γ cos 2x
92. (a) Given γ x +β α =0 =- +C
2
α β x+γ
é x2 xù
94. (d) Consider lim ê - ú
Operate C1 ® C1 + C 2 + C 3 x ®¥ êë 3x - 2 3 úû

x +α+β+γ β γ
é 3x 2 - x (3x - 2) ù
x +α+β+γ x +β α =0 = x ®¥ ê 3(3x - 2) ú
lim
x +α+β+γ β x+γ êë úû

2x 2x
1 β γ = lim = lim
x ®¥ 3(3 x - 2) x ®¥ é 2ù
= ( x + α + β + γ) 1 x + β α =0 3 x ê3 - ú
ë xû
1 β x+γ

2 1 2 1 2
Þ x + α + β + γ = 0 Þ x = -(α + β + γ) = lim = ´ =
x ®¥ 3 æ 2ö 3 3-0 9
Again if ç3- ÷
è x ø
www.jeeneetbooks.in

WWW.IIT-NEET.XYZ
Solved Paper 2017 2017-27

r r r r r r
95. (c) ((a ´ b ) ´ (c ´ d )).( a ´ d ) = 0 7 - i 3 + 4i
98. (d) z= ´
rrr r rrr r r r 3 - 4i 3 + 4 i
([a c d ] b - [b c d ] a ).(a ´ d ) = 0
rr r rr r 21 + 25i + 4 25(1 + i)
[a c d ][b a d ] = 0 = = = (1 + i )
16 + 9 25
r r r
Either c or b must lie in the plane of a z14 = (1 + i )14 = [(1 + i )2 ]7 = (2i)7
r
and d .
= 2 7 i 7 = -2 7 i
96. (b)
99. (c) f (x) = 2 sin x + sin 2x
f ' (x) = 2 cos x + 2 cos 2x = 2 (cos x + cos 2x)
B (1, 7) \ f ' (x) = 0 Þ 2cos2x + cos x – 1 = 0
-1 ± 3 1 p
cos x = = -1, \ x = p,
(1, 2) C 4 2 3
A
æ 3p ö
Now, f (0) = 0, f çè ÷ø = -2
2
D (4, – 2)
æ pö 3 3 3 3
f (p) = 0, f ç ÷ = 2 + =
Here, centre is A (1,2), and Tangent at B è 3ø 2 2 2
(1,7) is \ difference between greatest value and
x.1 + y.7 – 1 (x + 1) – 2 (y + 7) – 20 = 0 least value
or y = 7 ...(1)
3 3
Tangent at D (4,–2) is = +2
3x – 4y – 20 = 0 ...(2) 2
Solving (1) and (2), we get C is (16, 7) 100. (d) A and B will agree in a certain statement if
Area ABCD = 2 (Area of D ABC) both speak truth or both tell a lie. We define
1
following events
= 2× AB × BC E1 = A and B both speak truth Þ P(E1) = xy
2
AB × BC = 5 × 15 = 75 units E2 = A and B both tell a lie
97. (c) Integrating by parts. Þ P (E2) = (1 – x ) (1 – y)
E = A and B agree in a certain statement
ò f (x )g"(x ) dx - ò f "(x ) g(x ) dx Clearly, P(E / E1 ) = 1 and P(E / E 2 ) = 1
= f ( x )g ' ( x ) - ò f ' ( x ) g ' ( x ) dx The required probability is P(E1 / E ) .

- f ' ( x ) g ( x ) + ò f ' ( x ) g ' ( x )dx Using Baye’s theorem


P(E1 / E )
= f ( x ) g' ( x ) - f ' ( x ) g( x )
1 1 P(E1 )P(E / E1 )
=
Hence, ò0 f (x ) g" ( x) dx - ò0 f " (x )g ( x ) dx P(E1 )P(E / E1 ) + P(E 2 )P(E / E 2 )

= f (1)g' (1) - f ' (1)g(1) - f (0)g' (0) + f ' (0)g(0) xy.1 xy


= =
xy.1 + (1 – x )(1 – y).1 1 – x – y + 2 xy
= f (1)g' (1) - f ' (1)g(1)
EBD_7443
www.jeeneetbooks.in

WWW.IIT-NEET.XYZ
2017-28 Target VITEEE
101. (a) P (A È B) = P (A) + P (B) – P (A Ç B);
p/4

ò sec q tan n -2 q d q
2
3 1 =
Þ =1 – P( A ) + P(B) –
4 4 0

2 2 p/4
Þ 1=1– + P(B) Þ P(B) = ;
3 3 – ò tan n -2 q dq
0
2
Now, P( A Ç B ) = P(B) – P ( A Ç B ) = –
3 p/4
= ò sec 2 q tan n - 2 q dq - u n - 2
1 5 0
= .
4 12
p/4
x y z Þ un + un – 2 = ò sec 2 q tan n - 2 q dq
102. (a) Let the line be = = … (i)
a b c 0

If line (i) intersects with the line p/4


tan n -1 q 1
x -1 y + 3 z - 5 = n -1 =
= = , then n -1
2 4 3 0

104. (a) Total number of words that can be formed


a b c = 105. Number of words in which no letter
2 4 3 =0 is repeated = 10P5. So, number of words in
4 -3 14 which at least one letter is repeated = 105 –
10P = 69760.
5
Þ 9a – 7b – 10c = 0 …(ii) 105. (d) Required area = Area of OAB + Area of ABC
from (i) and (ii) , we have

a b c g (x) = – x + 2
= =
1 -3 5
3 f (x) = x2
x y z 2
\ The line is = =
1 -3 5 1 A
C
p/4 O B1 2 3
103. (a) Given: un = ò tan n q dq
0

p/4
1 2
= ò tan 2 q tan n -2 q dq
0 ò
Now, Area of OAB = f (x) dx + g(x)dx ò
0 1
p/4

ò (sec q - 1) tan n -2 q dq
2 1 2
=
0 ò 2
ò
= x dx + (- x + 2) dx
0 1
www.jeeneetbooks.in

WWW.IIT-NEET.XYZ
Solved Paper 2017 2017-29

1 2 1 1 7
x3 é -x 2 ù 108. (c) tan -1 + tan -1 + tan -1
= +ê + 2x ú 2 3 8
3 2
0 ëê ûú1
é 1 1 7 1 1 7 ù
1 é æ -4 ö æ -1 ö ù ê + + - ´ ´ ú
= + êç + 4÷ - ç + 2÷ ú =ê 2 3 8 2 3 8 ú
3 ëè 2 ø è 2 øû 1 1 1 7 7 1
ê1 - ´ - ´ - ´ ú
ë 2 3 3 8 8 2û
1 é æ 3ö ù
= + ê(-2 + 4) - ç ÷ ú
3 ë è 2ø û
é -1 -1 -1
êQ tan x + tan y + tan z
1 1 5 ë
= + = sq unit
3 2 6
æ x + y + z - xyz ö ù
= tan -1 ç ú
106. (a) The line
x y
+ = 1 will be a normal to the è 1 - xy - yz - zx ø÷ û
p q
parabola y2 = 4ax if, for some value of m, it é 41 7 ù
-
-1 ê 24 48 ú
is identical with
= tan ê 1 7
y = mx – 2am – am3 i.e. mx – y = (2am +
ê1 - - 7ú
am3) - ú
ë 6 24 16 û
Comparing coefficients, we get
m - 1 2am + am 3
= = Þ mp = – q, \ é 75 ù
1/ p 1/ q 1 ê ú æ 75 ö
= tan -1 ê 48 ú = tan -1 ç
ê1 - 43 ú è 48 - 43 ÷ø
q
m=– and mp = m (2a + am2) ë 48 û
p

2
æ -q ö é 75 ù
or P = 2a + am 2 = 2a + a çç ÷÷ = tan -1 ê ú = tan -1 15
è p ø ë5û
109. (b) Vertex of the parabola is a point which lies
aq 2
or p = 2a + on the axis of the parabola, which is a line
p2 ^ to the directrix through the focus, i.e., y =
or p3 = 2ap2 + aq2, 2 and equidistant from the focus and
Which is the required condition.
107. (b) P(E) = P ( 2 or 3 or 5 or 7) æ3 ö
directrix x = 0, so that the vertex is ç , 2 ÷ .
= 0.23 + 0.12 + 0.20 + 0.07 = 0.62 è2 ø

P ( F ) = P (1 or 2 or 3) Y
= 0.15 + 0.23 + 0.12 = 0.50
P ( E Ç F ) = P(2 or 3) y=2
A S (3,2)
= 0.23 + 0.12 = 0.35
O X
\ P( EUF ) = P( E ) + P ( F ) - P ( E Ç F )
= 0.62 + 0.50 - 0.35 = 0.77
EBD_7443
www.jeeneetbooks.in

WWW.IIT-NEET.XYZ
2017-30 Target VITEEE
110. (b) Let 113. (b) The given equation of the curve is y2 = 4ax
....(1)
é -1 2 5 ù é -1 2 5 ù
A = ê 2 -4 a - 4 ú ~ ê 0 0 a + 6 ú Differentiating both sides of (1) with respect
ê ú ê ú
êë 1 -2 a + 1úû êë 0 0 a + 6úû to x, we get

[R 2 ® R 2 + 2R1, R 3 ® R 3 + R1 ] dy dy 4a 2a
2y = 4a ; Þ = = ...(2)
dx dx 2 y y
Clearly rank of A is 1 if a = –6

-1 2 5 If y be the angle which the tangent to the


Also, for a = 1, | A |= 2 -4 -3 = 0 curve at (x, y) makes with the positive
1 -2 2
dy
direction of x-axis then tan y = or
dx
2 5
and = -6 + 20 = 14 ¹ 0
-4 -3 2a
tan y = ....(3), [using (2)]
\ rank of A is 2 if a = 1 y
111. (b) f ( x ) = 4 cos 3 x - cos x - 2 cos x = cos 3x At x = a, then from (1), y2 = 4a.a = 4a2 Þ y
[Expansion of determinant] = ± 2a.
p/2 p/2 Hence, we get two points (a, 2a) and (a, –
sin 3x ù 1
\ ò f ( x )dx =
3 úû 0
=-
3 2a) on the curve.
0
112. (a) Equation of the line through (1, –2, 3) At (a, 2a) x = a, y = 2a and let y = y1 .

x y z -1 2a
parallel to the line = = is \ from (3), tan y1 = = 1 = tan 45°;
2 3 -6 2a
x -1 y + 2 z - 3
= = = r (say) ...(1) Þ y1 = 45°.
2 3 -6
Then any point on (1) is (2r + 1, 3r – 2, –6r At (a, - 2a), x = a, y = -2a and let y = y2.
+ 3)
If this point lies on the plane x – y + z = 5
2a
then \ from (3), tan y 2 = = -1 = tan135°;
-2a
1
(2r + 1) – (3r – 2) + (–6r + 3) = 5 Þ r =
7 or y2 = 135°.

æ9 11 15 ö Hence the required angle between tangents


Hence the point is ç , - , ÷
è7 7 7ø
to (1) at (a, 2a) and (a, –2a) = y 2 – y1 = 135°–
Distance between (1, –2, 3) an d
45° = 90°.
æ9 11 15 ö This shows that the tangent lines to (1) at
ç ,- , ÷
è7 7 7ø (a, 2a) and (a, –2a) are perpendicular to each
other.
æ 4 9 36 ö æ 49 ö
= ç + + ÷= ç ÷= 1
è 49 49 49 ø è 49 ø
www.jeeneetbooks.in

WWW.IIT-NEET.XYZ
Solved Paper 2017 2017-31

x2 y2 dV
114. (a) Eccentrcity of - =1 is dx
25 25 sin 2 a Þ = dt
dt æ xö
çè 5 – ÷ø
1 + sin 2 a . 3

x2 y2 æ dx ö 5 15
Eccentricity of + =1 is Þç ÷ = = cm / sec .
è dt ø x = 2 5 – 2 13
5 sin 2 a 5
3
1 - sin 2 a 118. (b) Since vectors are coplanar

Given, 1 + sin 2 a = 5 1 - sin 2 a a 1 1


1 b 1
\ =0
2 2 1 1 c
Þ sin a =
3

2 a 1 1
Þ a = sin -1 = tan -1 2 1- a b -1 0
3 Þ = 0 [Using R2 – R1,
0 1 - b c -1
115. (a)
p q p Ù q ( p Ù q) Þ p R3 – R2]
T T T T Þ a (b – 1)(c – 1) – (1 – a) {(c – 1) – (1 –b)} = 0
T F F T Þ a (1 – b)(1 – c) + (1 – a) (1 – c) + (1 – a) (1 – b)
F T F T =0
F F F T Þ (a – 1 + 1) (1 – b) (1 – c) + (1 – a) (1 – c)
+ (1 – a) (1 – b)= 0
Þ (1 – b) (1 – c) + (1 – a) (1 – c) + (1 – a) (1 – b)
\ ( p Ù q) Þ p is a tautology..
= (1 – a) (1 – b) (1 – c)
(2 sin x ) 2n 1 1 1
116. (c) We have, f ( x ) = lim Þ + + =1
n ® ¥ 3n - (2 cos x ) 2 n 1- a 1 - b 1- c

(2sin x)2n é 3 -2 4 ù
= lim 119. (c) If A = ê1 2 -1ú
n ®¥ ( 3)2n - (2 cos x)2n ëê0 1 1 úû
f(x) is discontinuous when
1
and A -1 = adj(A) ......(i)
( 3 ) 2 n - (2 cos x ) 2 n = 0 k

3 p adj(A)
i.e. cos x = ± Þ x = np ± Also, we know A -1 = .......(ii)
2 6 |A|
\ By comparing (i) and (ii)
x2 dV dx x dx
117. (d) V = 5x – Þ =5 × |A| = k
6 dt dt 3 dt
EBD_7443
www.jeeneetbooks.in

WWW.IIT-NEET.XYZ
2017-32 Target VITEEE

3 -2 4 Þ h 2 + k 2 + 4 h - 6k + 9 sin 2 a
Þ | A |= 1 2 -1
0 1 1 +13cos 2 a = 4 cos 2 a

= 3 (2 + 1) + 2 (1 + 0) + 4 (1 – 0) Þ h 2 + k 2 + 4 h - 6k + 9 = 0
= 9 + 2 + 4 = 15 \ Locus of T is x 2 + y 2 + 4x - 6 y + 9 = 0
120. (d) Radius of circle
PART - IV (ENGLISH)
= 4 + 9 - 9 sin a - 13 cos a = 2 | sin a |
2 2
121. (c)
If T be (h, k) then as in Q. 44 122. (d)
2 | sin a | 123. (a)
tana =
124. (c)
h 2 + k 2 + 4h - 6k + 9 sin2 a +13cos2 a
125. (b) The best pronunciation of the word 'sorbet'
is sore-bay.
www.jeeneetbooks.in

WWW.IIT-NEET.XYZ

VITEEE
SOLVED PAPER 2016
(memory based)
GENERAL INSTRUCTIONS
• This question paper contains total 125 questions divided into four parts :
Part I : Physics Q. No - 1 to 40
Part II : Chemistry Q. No - 41 to 80
Part III : Mathematics Q. No - 81 to 120
Part IV : English Q. No - 121 to 125
• All questions are multiple choice questions with four options, only one of them is correct.
• For each correct response, the candidate will get 1 mark.
• There is no negative marking for the wrong answer.
• The test is of 2½ hours duration.

PART - I (PHYSICS) (a) 5 resistance is connected in series with Q


1. The potential energy of a system increases if (b) 4 resistance is connected parallel to S
work is done (c) 10 resistance is connected in series with P
(a) upon the system by a non conservative (d) 4 resistance is connected in series with R
force 4. In given circuit, C1 = C2 = C3 = C initially. Now, a
(b) by the system against a conservative force 3
(c) by the system against a non conservative dielectric slab of dielectric constant K is
2
force
inserted in C2.
(d) upon the system by a conservative force
2. In photoelectric effect, initially when energy of 12V C1
electrons emitted is Eo, de-Broglie wavelength
associated with them is o. Now, energy is
doubled then associated de-Broglie wavelength C2 C3
is

o
(a) (b) 2 o
2
The equivalent capacitance become
o
(c) o (d)
2 5C 7C
(a) (b)
3. In Wheatstone bridge, 4 resistors P = 10 , 7 5
Q = 5 , R = 4 , S = 4 are connected in cyclic
order. To ensure no current through galvanometer 2C C
(c) (d)
3 2
EBD_7443
www.jeeneetbooks.in

WWW.IIT-NEET.XYZ

2016-2 Target VITEEE


5. If the terminal speed of a sphere of gold (density (a) 3.6 A (b) 1.8 A
= 19.5 kg/m3) is 0.2 m/s in a viscous liquid (density (c) 2 A (d) 1 A
= 1.5 kg/m3), find the terminal speed of a sphere 11. The wavelength of an electron for transition from
of silver (density = 10.5 kg/m3) of the same size 9
in the same liquid a state n1 to n2 is . Which of the following
8R
(a) 0.4 m/s (b) 0.133 m/s wavelengths is possible for a transition from n 2
(c) 0.1 m/s (d) 0.2 m/s to n1
6. In shown fig, the circular loop of wire is moved
16 4
with velocity towards the infinite current carrying (a) (b)
wire. Then 15R 3R
9 36
(c) (d)
v 8R 5R
12. Two solenoids are given – 1st has 1 turn per unit
length and 2nd has n turns per unit length. Ratio
of magnetic fields at their centres is
I (a) n : 1 (b) 1 : n
(a) no current is induced in loop (c) 1 : n2 (d) n2 : 1
(b) current is induced in loop clockwise 13. Which statement is correct for the given circuit?
(c) current is induced in loop anticlockwise R1
(d) extra charges are induced on the wire loop
7. For a current carrying inductor, emf associated
is 20mV. Now, current through it changes from R2
6A to 2A in 2s. The coefficient of mutual
inductance is
(a) 20 mH (b) 10 mH R3
(c) 1mH (d) 2 mH (a) I through R1 > I through R2
8. A square current carrying loop is changed to a (b) I through R3 > I through R2 and R1
circular loop in time t1. Then (c) I through R2 > I through R3 and R1
(a) emf is induced in loop for time t < t1 (d) I is same in R1, R2 and R3
(b) emf is induced in loop for time t > t1 14. A (+)vely charged particle is placed near an
(c) no emf is induced in loop during whole infinitely long straight conductor where there is
process zero gravity. Then
(d) emf is induced due to change in magnetic (a) the charged particle will not move
field (b) it will move parallel to the straight conductor
9. Hologram is based on phenomenon of (c) it will move perpendicular to the straight
(a) diffraction conductor
(b) polarisation (d) it will move with constant acceleration
(c) interference 15. A metallic bar is heated from 0ºC to 100ºC. The
(d) total internal reflection coeficient of linear expansion is 10–5 K–1. What
10. In given circuit, all resistances are of 10 . Current will be the percentage increase in length?
flowing through ammeter is (a) 0.01% (b) 0.1%
(c) 1% (d) 10%
16. If the wavelength is brought down from 6000 Å
to 4000 Å in a photoelectric experiment then what
will happen?
(a) The work function of the metal will increase
(b) The threshold frequency will decrease
A
(c) No change will take place
(d) Cut off voltage will increase
12V
www.jeeneetbooks.in

WWW.IIT-NEET.XYZ

Solved Paper 2016 2016-3

17. For what value of A, B and C, the output Y = 1 act as a single resistance in a circuit. The
temperature coefficient of their single resistance
A
will be
B Y
C
(a) (b) 1 2
1 2
(a) 0 0 1 (b) 1 0 1 1 2
(c) 1 0 0 (d) 0 1 0 1 2 1 2
(c) (d)
2 2
18. Let the energy of an emitted photoelectron be E
23. The current density varies with radial distance r
and thewave-length of incident light be . What as J = a r2, in a cylindrical wire of radius R. The
will be the change in E if is doubled? current passing through the wire between radial
(a) E (b) E / 2 distance R/3 and R/2 is
(c) 2 E (d) E / 4
19. A solid sphere of radius R carries a uniform 65 a R 4 25 a R 4
(a) (b)
volume charge density . The magnitude of 2592 72
electric field inside the sphere at a distance r
65 a 2 R3 81 a 2 R 4
from the centre is (c) (d)
2938 144
r R
(a) (b) 24. A potentiometer circuit shown in the figure is
3 0 3 0 set up to measure emf of cell E. As the point P
moves from X to Y, the galvanometer G shows
R2 R3
(c) (d) deflection always in one direction, but the
r 0 r2 0 deflection decreases continuously until Y is
p reached. The balance point between X and Y
20. Two point dipoles pkˆ and kˆ are located at may be obtained by
2
(0, 0, 0) and (1m, 0, 2m) respectively. The resultant V R
electric field due to the two dipoles at the point
(1m, 0, 0) is
9p ˆ 7p ˆ P Y
(a) 32 k (b) k X
0 32 0
7p
(c) kˆ (d) none of these G
32 0 E
21. An iron rod of length 2m and cross-sectional (a) decreasing the resistance R and decreasing V
area of 50 mm2 stretched by 0.5 mm, when a mass (b) decreasing the resistance R and increasing V
of 250 kg is hung from its lower end. Young’s (c) increasing the resistance R and increasing V
modulus of iron rod is (d) increasing the resistance R and decreasing V.
25. A current I flows in the anticlockwise direction
(a) 19.6 10 20 N / m 2
through a square loop of side a lying in the xoy
18 2 plane with its center at the origin. The magnetic
(b) 19.6 10 N / m
induction at the center of the square loop is
(c) 19.6 1010 N / m 2 2 2 0I 2 2 0I
(a) eˆx (b) eˆz
a a
(d) 19.6 1015 N / m 2
22. Two resistances equal at 0° C with temperature 2 2 0I 2 2 0I
coefficient of resistance 1 and 2 joined in series (c) eˆz (d) eˆx
2 2
a a
EBD_7443
www.jeeneetbooks.in

WWW.IIT-NEET.XYZ

2016-4 Target VITEEE


26. A particle of charge q and mass m moves in a the refractive index of water is 4/3 and the fish is
circular orbit of radius r with angular speed . 12 cm below the surface, the radius of this circle
The ratio of the magnitude of its magnetic (in cm) is
moment to that of its angular momentum depends (a) 36 5 (b) 4 5
on
(c) 36 7 (d) 36 / 7
(a) and q (b) , q and m
33. A metal ball of mass 2 kg moving with a velocity
(c) q and m (d) and m
of 36 km/h has a head on collision with a
27. A long straight wire of radius R carries current i.
stationary ball of mass 3 kg. If after the collision,
The magnetic field inside the wire at distance r the two balls move together, the loss in kinetic
from its centre is expressed as : energy due to collision is
µ0 i 2µ0 i (a) 140 J (b) 100 J
(a) .r (b) .r (c) 60 J (d) 40 J
R 2 R2
34. Two lenses of focal length f1 = 10 cm and f2 = –
µ0 i µ0i 20 cm are kept as shown. The resultant power of
(c) .r (d) .r combination will be
2 R2 2 R
f2
28. If i1 = 3 sin t and i2 = 4 cos t, then i3 is f1
Silvered
i1 i2

i3
(a) –10D (b) 5 D
(c) 0 (d) 10 D
(a) 5 sin ( t + 53°) (b) 5 sin ( t + 37°)
(c) 5 sin ( t + 45°) (d) 5 cos ( t + 53°) 35. When a plastic thin film of refractive index 1.45
29. The equation of AC voltage is E = 220 sin is placed in the path of one of the interfering
waves then the central fringe is displaced
t + / 6 and the A.C. current is I = 10 sin
through width of five fringes. The thickness of
t / 6 . The average power dissipated is the film, if the wavelength of light is 5890Å, will
(a) 150 W (b) 550 W be
(c) 250 W (d) 50 W (a) 6.544 × 10–4 cm (b) 6.544 × 10–4 m
30. The current in an L–R circuit builds up to (3/4)th (c) 6.54 × 10–4 cm (d) 6.5 × 10–4 cm
of its steady state value in 4 seconds. The time 36. An unpolarised beam of intensity I0 is incident
constant of this circuit is on a pair of nicols making an angle of 60° with
1 2 each other. The intensity of light emerging from
(a) sec (b) sec the pair is
ln 2 ln 2
(a) I0 (b) I0/2
3 4 (c) I0/4 (d) I0/8
(c) sec (d) sec
ln 2 ln 2 37. The half life of radioactive Radon is 3.8 days.
31. The magnetic flux in a closed circuit of resistance 1
10 varies with time as = (2t –4t2 +1). The th of the radon
The time at the end of which
20
current in the loop will change its direction after
a time of sample will remain undecayed is (given
(a) 0.25 sec (b) 0.5 sec log10 e 0.4343 )
(c) 1 sec (d) none
(a) 3.8 days (b) 16.5 days
32. A fish looking up through the water sees the
outside world contained in a circular horizon. If (c) 33 days (d) 76 days.
www.jeeneetbooks.in

WWW.IIT-NEET.XYZ

Solved Paper 2016 2016-5

38. If the nuclear radius of 27Al is 3.6 Fermi, the 45. Which of these undergo polymerisation?
(a) CH3OH (b) C2H5OH
approximate nuclear radius of 64Cu in Fermi is
O
(a) 4.8 (b) 3.6 (c) (d) CH3CHO
(c) 2.4 (d) 1.2 CH3 C CH3
39. A hydrogen atom is in an excited state of principal 46. Which of the following graph represents
quantum number (n), it emits a photon of variation of 2p-orbital wave function with
wavelength ( ), when it returns to the ground distance from the nucleus?
state. The value of n is
R ( R 1)
(a) (b)
R 1 R
R (a) (b)
(c) ( R 1) (d)
R 1 r r
40. A marble block of mass 2 kg lying on ice when
given a velocity of 6 m/s is stopped by friction in
10 s. Then the coefficient of friction is (Take g =
10 ms–2)
(c) (d)
(a) 0.06 (b) 0.03
(c) 0.04 (d) 0.01 r r
47. Name the catalyst used to bring down the
PART - II (CHEMISTRY) reaction
NO2 NH2
41. IUPAC name of valeric acid is
(a) Propanoic acid (b) Butanoic acid ?
(c) Ethanoic acid (d) Pentanoic acid
NH2
(a) Sn/HCl (b) CuCl/HCl
HNO3, H2SO4 (c) Cu2Cl2/HCl (d) Zn-Hg/HCl
42. ?
48. The correct set of quantum numbers for Rb
The product P for the above given reaction will (atomic no. 37) is
be
1 1
(a) m-nitroaniline (a) 5, 0, 0, (b) 5,1, 0,
(b) o-nitroaniline 2 2
(c) p-nitroaniline
1 1
(d) both o & p nitroaniline (c) 6, 0,1, (d) 5,1,1,
2 2
43. Coordination number of Co in [Co(NH3 )6 ] Cl3
49. XeF4 disproportionate in water to give
(a) + 2 (b) + 3
(c) + 5 (d) + 8 (a) Xe + HF (b) Xe and XeO3
44. Which of the following complex will show fac & (c) XeOF4 and HF (d) XeO2Fe and HF
mer isomerism? 50. An ionic compound has a unit cell consisting of
(a) Co(NH3 )3 (NO2 )3 A ions at the corners of a cube and B ions on the
centres of the faces of the cube. The empirical
(b) CoCl2 (en)2 formula for this compound would be
2 (a) A 3B (b) AB3
(c) (Co(NH3 ) Cl (en) 2
(c) A 2B (d) AB
(d) Co(NH3 )2 Cl2 (en)
EBD_7443
www.jeeneetbooks.in

WWW.IIT-NEET.XYZ

2016-6 Target VITEEE


51. Among the following the incorrect statement is
(a) R – NH2 (b) R – HN – CH3
(a) Density of crystals remains unaffected due
to Frenkel defect. (c) R – CH3 (d) R – N – (CH3)2
(b) In BCC unit cell the void space is 32%. 59. The protein present in the hair is
(c) Density of crystals decreases due to (a) Lysine (b) Myosine
Schottky defect. (c) Keratin (d) Alanine
(d) Electrical conductivity of semiconductors 60. One mole of an ideal gas at 300 K is expanded
and metals increases with increase in isothermally from an initial volume of 1 litre to
temperature. 10 litres. Then S (cal deg–1 mol–1) for this
process is : (R = 2 cal K–1 mol–1)
NO2 (a) 7.12 (b) 8.314
NH2
(c) 4.6 (d) 3.95
61. For a reaction A B; H = 20 kJ mol–1 the
52.
activation energy of the forward reaction is
(A) (B) (C) 85 kJ/mol. The activation energy of the backward
The correct order of electrophilic substitution reaction will be
for the compounds given above will be (a) 105 kJ/mol (b) 65 kJ/mol
(a) A > B > C (b) C > B > A (c) 45 kJ/mol (d) 75 kJ/mol
(c) B > C > A (d) B > A > C 62. If the reaction N2 + 3H2 = 2NH3 occurs at 200°C
53. For mesotartaric acid, the correct configuration and 1000 atm then the graph showing the correct
for chiral carbon is equilibrium yield at 400°C is
(a) 2R, 3S (b) 2R, 3R
(c) 2S, 3R (d) 1D, 2L 400°C
54. Which of the two acids form anhydrides?
(a) [NH3]
(I) Oxalic acid (II) Succinic acid 200°C
(III) Benzoic acid (IV) Phthalic acid
(a) I & III (b) II & IV
T(°C)
(c) II & III (d) III & IV
55. By which reaction ketal is formed? 200°C
(a) Glycol with acetone 400°C
(b) Hydration of glycol (b) [NH3]
(c) Condensation of glycol
(d) Glycol with acetaldehyde
T(°C)
56. Wh ich one of the following show
stereoisomerism?
(a) 2-Butene 400°C
(b) 3-Methyl but-1-ene (c) [NH3] 200°C
(c) 2-Methyl butene
(d) Butanol
T(°C)
57. Acetophenone and Benzophenone can be
distinguished by which of the following test 200°C
(a) Knoeveangel reaction 400°C
(b) Canizzaro’s reaction (d) [NH3]
(c) Aldol condensation
(d) HVZ Reaction
T(°C)
LiAlH4 63. Group 15 elements have more electron gain
58. R NC P
enthalpy than group 16-elements. The correct
The product P in this reaction is
reason for this is
www.jeeneetbooks.in

WWW.IIT-NEET.XYZ

Solved Paper 2016 2016-7

(a) Half -filled stability of gp. 15 elements. 69. The enthalpy change for a given reaction at 298
(b) Poor shielding in gp. 15 K is – x J mol –1 . For the reaction to be
(c) Poor shielding in gp. 16 spontaneous at 298 K, the entropy change at
that temperature
(d) Half -filled stability of gp. 16 elements
(a) can be negative, but numerically greater
64. t-butyl CH CH C OH can’t give x
|| than Jk 1
O 298
(b) can be negative, but numerically smaller
decarboxylation while normally - unsaturated
x
acid give this reaction because than Jk 1 .
298
(a) t-butyl gp. has large size and does not let
(c) can not be negative
the COOH group to leave. (d) can not be positive
(b) t-butyl gp. can’t extract H from COOH. 70. a moles of PCl5 is heated in a closed container
(c) t-butyl gp. stabilise carbanion formed. to equilibriate PCl5(g) PCl3(g) + Cl2(g) at a
pressure of P atm. If x moles of PCl5 dissociate
(d) t-butyl gp. does not allow this composition at equilibrium, then
to convert to - -unsaturated acid 1/2
65. Which type of carbocation is/are formed when x Kp x Kp P
(a) a Kp P (b)
a Kp

OH is treated with an acid? Kp


1/2
Kp
1/2
x x
(c) (d)
(a) 1º (b) 2º a P a Kp p
(c) 3º (d) All the three 1
71. A plot of ln K against (abscissa) is expected
66. For hydrogen-oxygen fuel cell, the cell reaction is T
2H 2 ( g ) O 2 ( g ) 2H 2 O( ) to be a straight line with intercept on ordinate axis
equal to
If G f (H 2O) = – 237.2 kJ mol–1, then emf of
S S
this cell is (a) (b)
2.303 R R
(a) + 2.46 V (b) – 2.46 V
S
(c) + 1.23 V (d) – 1.23 V (c) (d) R S
R
67. At 298 K, the conductivity of a saturated solution 72. In a reaction A Products, when start is made
of AgCl in water is 2.6 × 10–6 S cm–1. Its solubility from 8.0 × 10–2 M of A, half-life is found to be 120
product at 298 K. minute. For the initial concentration 4.0 × 10–2
M, the half-life of the reaction becomes 240
Given : (Ag ) 63.0 S cm 2 mol 1 ,
minute. The order of the reaction is :
(Cl ) 67.0 S cm 2 mol 1 (a) zero (b) one
(a) 2.0 × 10–5 M 2 (b) 4.0 × 10–10 M 2 (c) two (d) 0.5
73. A reaction : A2 + B Products, involves the
(c) 4.0 × 10–16 M 2 (d) 2 × 10–8 M 2 following mechanism :
68. Standard entropy of X2, Y2 and X Y3 are 60, 40 A2 2A (fast)
and 50 JK–1 mol–1, respectively. For the reaction, (A being the intermediate)
1 3 A B Products (slow). The rate law
X2 Y2 XY3 , H –30kJ , to be at k 2
2 2 consistent to this mechanism is :
equilibrium, the temperature will be
(a) 1250 K (b) 500 K (a) rate = k[ A2 ][ B] (b) rate = k[ A2 ]2 [ B ]
(c) 750 K (d) 1000 K (c) rate = k[ A2 ]½ [ B ] (d) rate = k[ A2 ][ B ]2
EBD_7443
www.jeeneetbooks.in

WWW.IIT-NEET.XYZ

2016-8 Target VITEEE


74. The following data were obtained for a given 79. Which of the following name reaction is not
reaction at 300 K. used for introducing a – COOH group?
Reaction Energy of activation (a) Cannizzaro reaction
(kJ mol–1) (b) Benzilic acid rearrangement
(i) uncatalysed 76 (c) Baeyer – Villiger oxidation
(ii) catalysed 57 (d) Iodoform reaction
The factor by which rate of catalysed reaction is 80. Esterification of acid chloride with ethanol is
increased, is usually carried out in the presence of pyridine.
(a) 21 (b) 2100 The function of pyridine is
(c) 2000 (d) 1200 (a) to remove HCl formed in the reaction
75. The wave number of the first emission line in the (b) to react with acid chloride to form an
Balmer series of H-Spectrum is : acylpyridinium ion
(R = Rydberg constant) : (c) both (a) and (b)
5 9 (d) as a catalyst
(a) R (b) R
36 400
PART - III (MATHEMATICS)
7 3
(c) R (d) R
6 4 81. The solution of the differential equation
76. Which one of the following reactions of xenon 1 dy
1 y2 x e tan y 0
compounds is not feasible? dx
(a) 3Xe F4 + 6H 2 O
tan 1 y
2 Xe + Xe O3 +12HF +1.5O 2 (a) (x 2) ke
(b) 2Xe F2 + 2H 2 O 2 Xe + 4HF + O 2 1 1
(b) 2xe tan y
e 2 tan y
k
(c) Xe F6 + RbF Rb[Xe F7 ] 1
(d) Xe O3 + 6HF Xe F6 + 3H 2 O (c) xe tan y
tan 1
y k
77. Anisole is treated with HI under two different 1 1
conditions. (d) xe 2 tan y e tan y k
82. A tetrahedron has vertices at O (0, 0, 0), A (1, 2, 1)
HI ( g ) conc . HI
C D C 6 H 5 OCH 3 A B B (2, 1, 3) and C (–1, 1, 2). Then the angle between
The nature of A to D will be the faces OAB and ABC will be
(a) A and B are CH3I and C6H5OH, while C and 1 17
(a) 120° (b) cos
D are CH3OH and C6H5I 31
(b) A and B are CH3OH and C6H5I, while C and (c) 30° (d) 90°
D are CH3I and C6H5OH
(c) Both A and B as well as both C and D are x2 y2
83. The foci of the ellipse 1 and the
CH3I and C6H5OH 16 b2
(d) A and B are CH3I and C6H5OH, while there
is no reaction in the second case. x2 y2 1
hyperbola coincide then value
78. Phenol undergoes electrophilic substitution 144 81 25
more easily than benzene because of b2 is
(a) –OH group exhibits +M effect and hence (a) 1 (b) 5
increases the electron density on the o- and (c) 7 (d) 9
p-positions. 84. If the tangent to the function y = f ( x) at (3,4)
(b) oxocation is more stable than the 3
carbocation makes an angle of with the positive direction
4
(c) both (a) and (b)
(d) –OH group exhibits acidic character of x-axis in anticlockwise direction then f '(3) is
www.jeeneetbooks.in

WWW.IIT-NEET.XYZ

Solved Paper 2016 2016-9

(a) –1 (b) 1
93. If e x y 1 y2 , then the value of y is
1
(c) (d) 3 1 x x 1 x x
3 (a) (e e ) (b) (e e )
85. The probability of India winning a test match 2 2
x x
1 x
against Australia is assuming independence (c) e x e 2 (d) e e 2
2
94. What is the area of a loop of the curve r = asin3 ?
from match to match. The probability that in a
match series India’s second win occurs at third a2 a2
test match is (a) (b)
6 8
1 1 1 2
(a) (b) (c) (d) a2 a2
8 4 2 3 (c) (d)
12 24
86. If | a | 3, | b | 2, | c | 1 then the value of 95. Convert the hexadecimal numeral ABCD into
binary numeral
| a. b b . c c . a | is (given that a b c 0) (a) (1010101111001101)2
(a) –7 (b) 7 (b) (1001000011111111)2
(c) 14 (d) –14 (c) (1111110000010001)2
(d) (1000100100111100)2
87. If f ( x) x 2 , g ( x) 2 x, 0 x 2 then the 96. The normal at the point (at 12, 2at 1) on the
2 parabola, cuts the parabola again at the point
value of I( x ) max f ( x ), g ( x) is whose parameter is
0 2 2
(a) t2 t1 (b) t2 t1
10 1 11 t1 t1
(a) (b) (c) (d) 32
3 3 3 2
88. If A and B are matrices and B = ABA–1 then the (c) t2 t1 (d) None of these
t1
value of (A + B) (A – B) is
(a) A2 + B2 (b) A2 – B2 97. The distance moved by the particle in time t is
(c) A + B (d) A – B given by s = t 3 12 t 2 6 t 8 . At the instant,
89. The value of (1 + – 2)7 is when its acceleration is zero the velocity is :
(a) 128 2 (b) –128 2 (a) 42 (b) –42
(c) 128 (d) –128 (c) 48 (d) –48
90. The moment about the point ˆi 2ˆj 3kˆ of a force 98. The logical expression X, in its simplest form for
the truth table
represented by ˆi ˆj kˆ acting through the a b X
point 2i + 3j + k is 1 0 0
(a) 3iˆ 3jˆ is
(b) 3iˆ ˆj 1 1 1
0 1 0
(c) ˆi ˆj (d) 3iˆ 3jˆ 0 0 0
91. If g (x) is a polynomial satisfying (a) X = a . b (b) X = a + b
g (x) g(y) = g(x) + g(y) + g(xy) – 2 (c) X = a ' . b (d) X = a . b'
for all real x and y and g (2) = 5 then Lt g(x) is 1 1
x 3
99. The value of cos cos 1 is equal to
(a) 9 (b) 10 2 8
(c) 25 (d) 20 (a) – 3/4 (b) 3/4
92. The equation of one of the common tangents to (c) 1/16 (d) 1/4
100. Consider the objective function Z = 40x + 50y.
the parabola y2 = 8x and x 2 y 2 12x 4 0
The minimum number of constraints that are
is required to maximize Z are
(a) y = –x + 2 (b) y = x – 2 (a) 4 (b) 2
(c) y = x + 2 (d) None of these (c) 3 (d) 1
EBD_7443
www.jeeneetbooks.in

WWW.IIT-NEET.XYZ

2016-10 Target VITEEE


101. In a culture the bacteria count is 1,00,000. The
(a) x 2 y 2 z 2 9p 2
number is increased by 10% in 2 hours. In how
many hours will the count reach 2,00,000 if the (b) x 3 y 3 z 3 9p 3
rate of growth of bacteria is proportional to the
number present. (c) x2 y2 z2 9p 2
2 2 log 2 (d) x 3 y3 z3 9p 3
(a) (b)
11 11 108. While shuffling a pack of 52 playing cards, 2 are
log log
10 10 accidentally dropped. The probability that the
missing cards to be of different colours is
log 2 log 2 29 1 26 27
(c) (d) (a) (b) (c) (d)
log11 11 52 2 51 51
log
10 109. Which of the following is INCORRECT for the

1 hyperbola x 2 2y2 2x 8 y 1 0
1
102. The value of sin –1 cot (3) is
5 (a) Its eccentricity is 2

(a) (b) (b) Length of the transverse axis is 2 3


6 4
(c) Length of the conjugate axis is 2 6
(c) (d)
3 2 (d) Latus rectum is 4 3
103. If a = cos 2 + i sin 2 , b = cos 2 + i sin 2 ,
110. A box contains 20 identical balls of which 10 are
c = cos 2 + i sin 2 and d = cos 2 + i sin 2 , then blue and 10 are green. The balls are drawn at
1 random from the box one at a time with
abcd = replacement. The probability that a blue ball is
abcd drawn 4th time on the 7th draw is
(a) 2 cos ( + + + ) 27 5 5 1
(a) (b) (c) (d)
(b) 2 cos ( + + + ) 32 64 32 2
(c) cos ( + + + ) 111. The number of common tangents to the circles
(d) None of these x2 + y2 – 6x – 14y + 48 = 0 and x2 + y2 – 6x = 0 is
104. If the mean of a binomial distribution is 25, then (a) 1 (b) 2
(c) 0 (d) 4
its standard deviation lies in the interval
112. The solution of the equation
(a) [0, 5) (b) (0, 5]
cos2 + sin + 1 = 0, lies in the interval
(c) [0, 25) (d) (0,25]
105. Number of ways of selecting three squares on a 3
(a) – , (b) ,
chessboard so that all the three be on a diagonal 4 4 4 4
line of the board or parallel to it is
3 5 5 7
(a) 196 (b) 126 (c) , (d) ,
(c) 252 (d) 392 4 4 4 4
106. If A and B are two matrices such than rank of A = 113. If f(x) = (1 + x)2/x for x 0 and f(0) = e2 is
m and rank of B = n, then (a) left continuous only at x = 0
(a) rank (AB) = mn (b) right continuous only at x = 0
(b) rank (AB) rank (A) (c) continuous at x = 0
(c) rank (AB) rank (B) (d) discontinuous at x = 0
(d) rank (AB) min (rank A, rank B) dy
107. A variable plane remains at constant distance p 114. If y = 2x/Inx then at x = e is
from the origin.If it meets coordinate axes at dx
points A, B, C then the locus of the centroid of (a) e (b) 2e log 2
ABC is (c) log 2 (d) 0
www.jeeneetbooks.in

WWW.IIT-NEET.XYZ

Solved Paper 2016 2016-11

1 PART - IV (ENGLISH)
115. dx is equal to
2 4 3/ 4
x (x 1) Direction (Qs. 121-123) Read the passage carefully
and anser the questions given below.
1/ 4
1 Laws of nature are not commands but statements of
(a) 1 C (b) ( x 4 1)1 / 4 C
acts. The use of the word "law" in this context is rather
x4
unfortunate. It would be better to speak of uniformities
1/ 4 1/ 4 in nature. This would do away with the elementary
1 1
(c) 1 C (d) C 1 fallacy that a law implies a law giver. If a piece of matter
x4 x4 does not obey a law of nature it is punished. On the
116. If the letters of the word KRISNA are arranged contrary, we say that the law has been incorrectly
in all possible ways and these words are started.
written out as in a dictionary, then the rank of 121. If a piece of matter violates nature's law, it is not
the word KRISNA is punished because
(a) 324 (b) 341 (a) it is not binding to obey it
(c) 359 (d) None of these (b) there is no superior being to enforce the
law of nature
117. The shortest distance between the lines (c) it cannot be punished
x = y + 2 = 6z – 6 and x + 1 = 2y = – 12z is (d) it simply means that the facts have not been
1 correctly stated by law
(a) (b) 2 122. Laws of nature differ from man-made laws
2
because
3 (a) the former state facts of Nature
(c) 1 (d) (b) they must be obeyed
2
(c) they are natural
118. The domain and range of the function f given (d) unlike human laws, they are systematic
by f(x) = 2 – |x – 5| is 123. The laws of nature based on observation are
(a) Domain = R+, Range = (– , 1] (a) conclusion about the nature of the universe.
(b) Domain = R, Range = (– , 2] (b) true and unfalsifiable.
(c) Domain = R, Range = (– , 2) (c) figments of the observer imagination.
(d) Domain = R+, Range = (– , 2] (d) subject to change in the light of new facts.
124. Direction: This question presents a sentence,
119. The number of surjective functions from A to B
part of which or all of which is underlined.
where A = {1, 2, 3, 4} and B = {a, b} is Beneath the sentence you will find four ways of
(a) 14 (b) 12 phrasing the underlined part. The first of these
(c) 2 (d) 15 repeats the original; the other three are
b different. If you think the original is best, choose
120. If f (a b – x ) f ( x) , then x f ( x) dx is the first answer; otherwise choose one of the
a
others.
The administration discussed whether the
equal to
number of studen ts studying European
a b
b languages was likely to decline when the senior
(a) f (b – x ) dx lecturer retired.
2 a (a) whether the number of students studying
b European languages was likely
a b (b) whether the number of students studying
(b) f (b x) dx
2 a European languages were likely
(c) if th e students studying European
b languages were likely
b a
(c) f ( x) dx (d) if the number of European language
2 students were likely
a
b
125. Choose the best pronunciation of the word,
a b Restaurant, from the following options.
(d) f ( x ) dx (a) res-trawnt (b) res-tuh-rawnt
2
a (c) rest-rant (d) resto-raunt
EBD_7443
www.jeeneetbooks.in

WWW.IIT-NEET.XYZ

2016-12 Target VITEEE

SOLUTIONS
PART - I (PHYSICS) with Q, the equivalent resistance in the
P-arm becomes 10 .
1. (d) When work is done upon a system by a P 10
conservative force then its potential energy 1
Q 10
increases.
2. (a) de-Broglie wavelength is given by S 4
and 1
h R 4
= , where h = Planck’s constant and
p
P S
p = momentum .
Q R
Also, energy (E) and momentum are related
as 4. (a) When a dielectric slab of dielectric constant
p2 K
3
E is inserted between the plates of C 2,
2m 2
p 2mE
its new capacitance C 2 becomes
h 1
as h and m are 3
2mE E C2 C
constants 2

o E' 2E Equivalent capacitance of C 2 and C3 is


Hence, 2
E E
3 5C
Ceq C2 C3 C C
. o 2 2
2
3. (a) For no current through the galvanometer, 12V C1
the wheatstone bridge should be balanced.
For this, we must have
P S C2 C3
Q R
This condition is satisfied with only option
(a).

Now, Ceq and C1 are in series. Therefore,


P Q their equivalent capacitance is
(10 ) (5 )
5C
C
G Ceq C1 2
Ceq 5C
Ceq C1 C
2
S R
(4 ) (4 )
5C2 5C
.
7C 7
When a 5 resistor is connected in series
www.jeeneetbooks.in

WWW.IIT-NEET.XYZ

Solved Paper 2016 2016-13

5. (c) Terminal velocity, 6 3


2 20 10
2r (d1 d 2 )g
vT
9 10
vT2 (10.5 1.5) 9 Rp
vT2 0.2 3
0.2 (19.5 1.5) 18 Hence, current flowing through ammeter is
vT2 0.1 m / s
V 12
6. (b) Mganetic field at the centre of the circular I 3.6A.
loop increases as it moves towards the Rp 10
current carrying wire as we know it varies 3
inversely with the distance. Therefore,
11. (c) Energy released in transition from state n1
induced current in the loop should be such
that it can reduce the increased magnetic to n2 is equal to the energy absorbed in
field. This is possible only when the induced transistion from state n2 to n 1.
current is clockwise. 12. (b) Magnetic field no. of turns per unit
length
dI
7. (b) e L Required ratio = 1 : n.
dt
–3 13. (d) R1, R2 and R3 are in series. Therefore, same
Here, e = 20 mV = 20 × 10 V current will flow through all the resistors.
dI 6 2 14. (a) As the charged particle is at rest, no force
2 A/s
dt 2 will act on it due to the magnetic field
L= ? produced by the conductor at the site of
the charge. Hence, it will remain at rest.
20 10 3 L 2

L 10 10 3 H = 10 mH. 15. (b) T 10 5


100 10 3

8. (a) For the time t < t1, area of loop changes.


Hence, magnetic flux linked with it changes
during this time and emf is induced thereby. 100% 10 3 100
9. (c) Hologram is based on the phenomenon of
interference. 10 1 0.1%
10. (a) An equivalent of the given network is as 16. (d) When wavelength decreases, frequency
shown in the figure. increases. Also, we know that cut-off
If Rp be the net resistance, then voltage (or stopping potential) increases
10 when frequency increases.
10 10 Hence, option (d) is correct. Note that work
function and threshold frequency are
10 10 constant for a given metal.
10 17. (b) Clearly,
Y = (A + B) . C = (A.C) + (B.C)
For
A A = 0, B = 0 & C = 1, Y = (0.1) + (0.1) = 0
12V A = 1, B = 0 & C = 1, Y = (1.1) + (0.1) = 1
A = 1, B = 0 & C = 0, Y = (0.1) + (0.0) = 0
1 1 1 1 1 A = 0, B = 1 & C = 0, Y = (0.0) + (1.0) = 0
Rp 10 10 10 10 10 10 So, option (b) is correct.
18. (b) We have
1 1 1 1 h = Wo + E, where E is the energy of emitted
10 20 20 10 photoelectron
EBD_7443
www.jeeneetbooks.in

WWW.IIT-NEET.XYZ

2016-14 Target VITEEE


2
23. (a) Given ; J = ar .
hc
Wo E 2 R/2
i= J 2 r dr ar 2 2 r dr
As hc and Wo are constant, 1 R/3
R/2
E
1 R/2
3 r4
=2 a r dr 2 a
R /3 4
R/3
Therefore, as is doubled, E will become
4 4
half. a R R
q r =
1 qr r 2 2 3
19. (a) E 3 = 4 =
4 0 R 3 3 0 3
R 0
3 aR 4 65 65 aR 4
=
p 2 81 16 2592
20. (b) The given point is on axis of dipole and 24. (b) Decreasing R increases current in XY and
2
there by increases the potential drop across
at equatorial line of p dipole so that field at XP and the balance point may be obtained.
The current may be increased also by
given point is ( E1 E2 ) increasing V.
25. (b) Field due to one side of loop at O
0I
= (2 sin 45 )
a
4
2
Field at O due to all four sides is along unit
vector eˆz
I a

2K ( p / 2) Kp ˆ 45° 45°
E1 ( k) I I
23 8
O
Kp ˆ
E2 ( k)
1
I
7 7p
E1 E2 Kp( kˆ) = kˆ Total field
8 32 0
0I 2 2 0I
250 9.8 = 4. (2sin 45 )
a a
4
21. (c) Y F/A 50 10 6 2
/ 0.5 10 3 26. (c) The angular momentum L of the particle is
2 given by
250 9.8 2 L = mr2 where = 2 n.
6 3 19.6 1010 N / m 2
50 10 0.5 10
Frequency n = ;
22. (d) R1 = R0(1 + 2
1t) + R0(1 + 2t)
q
= 2 R0 1 1 2
t Further i = q × n =
2
2
Comparing with R = R0 (1+ t) q
Magnetic moment, M = iA = r2;
2
1 2
2
www.jeeneetbooks.in

WWW.IIT-NEET.XYZ

Solved Paper 2016 2016-15

qr 2 d
M= 31. (a) e = [ 2 8t 0]
2 dt
M qr 2 q 1
So, 8t =2 t= sec = 0.25 sec
L 2mr 2 2m 4
27. (c) Using Ampere's law, we have
1 r
B.d µ0iin r 32. (d) sin ic
2
R r h2
i
or B × 2 r = µ0 r2 Using h = 12 cm, µ = 4/3
R2
µ0 i r r
B=. 2
2 R
28. (a) From Kirchhoff’s current law, ic
h
i3 i1 i2 3sin t 4sin ( t 90 ) ic

Fish
= 32 42 2(3)(4) cos 90 sin ( t )
36
4sin 90 4 We get r = cm.
where tan 7
3 4 cos 90 3 33. (c) Apply conservation of momentum,
i3 = 5 sin ( t + 53°) m1v1 = (m1 + m2)v
m1 v1
E0 v=
29. (b) We know that, Z = I (m1 m 2 )
0
Given, E0 = 220V and I0 = 10 A Here v1 = 36 km/hr = 10 m/s,
m1 = 2 kg, m2 = 3 kg
220
so Z = = 22 ohm
10 10 2
v 4 m/s
5
=
6 6 3 1
K.E. (initial) = 2 (10) 2 100 J
E0 I0 2
pa cos
2 2 1
K.E. (Final) = (3 2 ) ( 4 ) 2 40 J
220 10 2
= cos = 550 W
2 2 3 Loss in K.E. = 100 – 40 = 60 J
30. (b) I I 0 (1 e ) t/ Alternatively use the formula
where time constant 1 m1m2 2
Ek u1 u 2
3 t/ 2 m1 m 2
I0 I0 (1 e )
4
3 1 100 100
l e t/
e t/ 34. (d) P = 2 0
4 4 10 20

t 1 4 P = 10 dioptre.
ln e ln 2 ln 2
4
(0.45)t
35. (a) X0 ( 1)t 5 10
2 5890 10
ln 2
EBD_7443
www.jeeneetbooks.in

WWW.IIT-NEET.XYZ

2016-16 Target VITEEE

5 5890 10 10 –4 4
t = 6.544 × 10 cm rCu 3.6 4.8 Fermi
0.45 3
36. (c) According to2Malus’ law2
I = I0 cos = I0 (cos 60º) 1 1 1
39. (d) As R
2
I n12 n22
1
= I0 × = 0 1 1 1
2 4 R
37. (b) t1/2 = 3.8 day 12 n 2
Multiply both sides by
0.693 0.693
0.182 1 1 1
t1/ 2 3.8 1 R 1 2 or 1 2
n R n
If the initial number of atom is a = A0 then
after time t the number of atoms is a/20 = A. 1 1 R 1
or 1
We have to find t. n2 R R
2.303 A0 2.303 a R
t log log or n
A 0.182 a / 20 R 1
2.303 6
log 20 = 16.46 days 40. (a) a g [using v = u + at]
0.182 10
6 6
38. (a) Nuclear radius, r A1/ 3 = 0.06
where A is mass number 10 g 10 10
1/ 3 1/ 3
rCu ACu 64
rAl AAl 27
+
PART - II (CHEMISTRY) : NH2 NH3
41. (d) The common name of Pentanoic acid is
conc. HNO3
valeric acid.
conc. H2SO4
42. (d) Nitric acid not only nitrates, but also +
—NH2 gp. o, p-director —NH3 gp. m-director
oxidizes the highly reactive ring as well,
with loss of much material as dark-coloured However, all these difficulties are overcome
tar. Furthermore, in the strongly acidic by protecting the amino group by
nitration medium, the amine is converted acetylation, with either acetyl chloride or
into anilinium ion (—NH3+) ; substitution acetic anhydride. Acetylation (—NH2
is thus controlled not by the —NH2 group NHCOCH3 ) converts —NH2 group to
but by the —NH3+ group which, because acetamido (—NHCOCH3) group which is o,
of its positive charge, directs the entering p-directing but lesser activating toward
group to the meta- position instead of electrophilic aromatic substitution than the
ortho, and para. parent —NH2 group.

H H
+
: NH2 O
:N N
C—CH3 C—CH3
CH3 CCl
O O–
Aniline Acetanilide Resonance in acetanilid due to amide group
(electron pair can delocalize (note that electron pair on N can also delocalize to
only to benzene ring making amide group, hence —NHCOCH3 gp. becomes
o, p-positions highly reactive) weak activator than the —NH2 group)
www.jeeneetbooks.in

WWW.IIT-NEET.XYZ

Solved Paper 2016 2016-17

NH2 NHCOCH3 NHCOCH3 NHCOCH3


NO2
(CH3CO)2O HNO3, H2SO4, 15°C
+

NO2
NH2 NH2
+ NO2
(i) H2 O, H , heat
+

(ii ) OH

NO2
o-Nitroaniline p-Nitroaniline

43. (c) The number of atoms of the ligands that are 46. (a)
directly bound to the central metal atom or 47. (a) The most widely used method for preparing
ion by coordinate bond is known as the aromatic amines is the reduction of the nitro
coordination number of the metal atom or group to the amino group. This reduction
ion. Hence the coordination no. of the given can be achieved by catalytic
compound will be 6. hydrogenation, or most frequently with an
44. (a) Complexes of the type MA B exist in two acid and a metal (Fe, Zn, Sn) or a metal salt
3 3 like SnCl2.
geometrical forms which are named as facial
(fac-) and meridonal (mer-isomers). H2, catalyst
C6H5 NO2 C6H5NH2
Co(NH3 )3 (NO2 )3 may be represented or (i) Sn, HCl, (ii) OH
Nitrobenzene Aniline
in fac- and mer-isomeric forms as follows.
NO2 NH3 48. (a) The electronic configuration for Rb (37) is
NH3 NO2 NO2 Rb (37)
NO2
= 1s2 2s2 2p6 3s2 3p6 4s2 3d10 4p6 5s1
Co Co 1
For 5s1, n = 5, = 0, m = 0, s
NH3 NO2 NH3 NO2 2
49. (b) XeF4 disproportionates in water giving
NH3 NH3
solid XeO3 on evaporation.
fac-isomer mer-isomer
6XeF4 12H 2 O 2XeO3 24HF 4Xe 3O 2
45. (c, d) Acetone on polymerisation give mesitylene
6XeF4 12H 2 O 2XeO3 24HF 4Xe 3O 2
50. (b) Number of A ions in the unit cell
CH3
1
= 8 1
conc H2SO4 8
3CH3COCH3
Number of B ions in the unit cell
H3C CH3 1
= 6 3
3 molecules of acetaldehyde produce 2
paraldehyde (CH3CHO)3 and 4 molecules of it Hence empirical formula of the compound
produce metaldehyde (CH3CHO)4. = AB3
EBD_7443
www.jeeneetbooks.in

WWW.IIT-NEET.XYZ

2016-18 Target VITEEE


51. (d) The electrical resistance of metals
CH 2 COOH heat
CH2 CO
depends upon temperature. Electrical 54. (b) | | O
resistance decreases with decrease in H 2O
CH 2 COOH CH2 CO
temperature and becomes zero near the
Succinic anhydride
absolute temperature. Material in this state
is said to possess super conductivity. COOH CO
heat
52. (a) An electron releasing group stablilises the H 2O
O
carbocation by dispersing its positive COOH CO
charge and thus activates the ring while an Phthalic anhydride
electron-withdrawing group destabilises Oxalic on heating produces formic acid.
the carbocation by intensifying its positive 55. (a)
charge and thus deactivates the ring. CH2OH CH 3 CH 2 O R
| +O=C | O
–NH2 being electron releasing group CH2OH CH 3 CH 2 O R
releases electron and thus tend to neutralise
positive charge of the ring and itself CH3 CH3 H CH3
C=C C=C
becomes somewhat positive. The dispersal 56. (a) H H H3C H
of positive charge of the ring stabilises cis-2-butene trans-2-butene
carbocation and hence facilitates its 57. (c) Aldehydes and Ketones containing
formation which ultimately results an -hydrogen undergo aldol condensation,
increased rate of reaction. Now since such since benzophenone does not have
factor is not present in benzene, its -hydrogen hence do not undergo aldol
condensation whereas acetophenone show
carbocation is less stable than (A). Hence
this reaction due to presence of -H atom.
aniline undergoes electrophilic substitution 58. (b) Isonitriles on reduction with LiAlH4 give
at a faster rate than benzene. 2° amines
On the other hand in (C) the –NO2 group R N C
LiAlH
4 RNHCH 3
(an electron withdrawing group) intensifies 59. (c) Keratin
positive charge on the ring and thus 60. (c) For an isothermal process
destabilises the carbocation and hence its V2
formation becomes difficult which ultimately S = 2.303 nR log
V1
results a slower reaction. Now since
substituent is not present in benzene, its 10
= 2.303 × 1 × 2 log
carbocation (B) is relatively more stable 1
than (C). Hence nitrobenzene undergoes = 4.6 cal deg–1 mol–1
electrophilic substitution at a slower rate 61. (b) Energy of activation for forward reaction
than benzene. (Ea) = 85 kJ/mol
Energy of activation for backward reaction
53. (c) COOH = Ea – H = 85 – 20 = 65 kJ mol–1
4
62. (a) Since the formation of amnonia is an
3 exothermic reaction hence on increasing
H OH
temperature, reaction will proceed in
backward direction i.e. formation of NH3
2
H OH decreases.
63. (a) The atoms having half-filled and fully filled
1 orbitals are comparatively more stable,
COOH
2S, 3R hence more energy is required to remove
www.jeeneetbooks.in

WWW.IIT-NEET.XYZ

Solved Paper 2016 2016-19

the electron from such atoms. Therefore [ G 0]


group 15 have more I.E. than gp. 16
elements. T (–40) –30 1000 [ 1kJ 1000J]
64. (c) Because of +I effect, t-butyl group –30 1000
or T or 750 K
destabilises the carbanion. –40
65. (d) 69. (b)
H+ + 70. (d) PCl5(g) PCl3(g) + Cl2(g)
OH OH2
1–
x
+ = ( degree of dissociation )
–H2O CH2 a
Total moles =1– + + = 1 +
1º carbocation
(least stable) PPCl3 PCl2 p .p
1 1
Kp
PPCl5 1
p
1, 2 – alkyl 1, 2 – alkyl + 1
shift + shift
1/ 2
2 Kp
2º carbocation 3° carbocation p
(more stable) (most stable) 2
1 Kp P
66. (c) We know G° = – nFE°
71. (b) RT ln K = - G° = T S° - H° ;
G
E S H
nF ln K
R RT
237.2 1000 J
= 1.23 V [ n = 2] Thus, a plot of ln K versus 1 T (abscissa)
2 96500
will be straight line with slope equal to
1000k 1000 2.6 10 6 H S
67. (b) Solubility S and intercept .
º º º R R
AgCl Ag Cl
72. (c)
3 n 1 n 1
2.6 10
2 10 5 mol L 1 ; Ksp = S2 (t1/ 2 )1 a2 120 4 10 2
; ;n 2
63 67 (t1/ 2 )2 a1 240 8 10 2
68. (c) For a reaction to be at equilibrium G = 0.
73. (c) From slow step :
Since G H – T S so at equilibrium rate = k2[A][B] .........(i)
H–T S 0 From fast step :
or H T S [ A]2
For the reaction Ke or [ A] Ke½ [ A]½ .........(ii)
[ A2 ]
1 3
X2 Y2 XY3 ; H –30kJ From (i) and (ii)
2 2
(given) rate k 2 ke½ [ A2 ]½ [ B ] k [ A2 ]½ [ B ]
Calculating S for the above reaction, we 74. (c) Using Arrhenius equation,
get Ea

1 3 1
K A.e Rt , we get
S 50 60 40 JK
2 2 Ea
log k log A
=50 – (30 + 60) JK–1 = – 40 JK–1 2.303 RT
At equilibrium, T S H
EBD_7443
www.jeeneetbooks.in

WWW.IIT-NEET.XYZ

2016-20 Target VITEEE

Ea (1) +
....(i) H
log k1 log A E
+
2.303 RT E
Ea(2)
and log k2 log A ...(ii) Carbonium ion
2.303 RT (less stable)

k2 1 High stability of oxonium ion (oxocation) is


or log [Ea (1) Ea(2) ] because here every atom (except H) has a
k1 2.303RT
complete octet of electrons, while in
(from (i) and (ii)) carbocations, carbon bearing positive
charge is having six electrons.
1 79. (c) Baeyer - Villiger oxidation involves the
(76000 57000) conversion of a cyclic ketone to a lactone,
2.303 8.314 300
or an acyclic ketone into ester.
k2 19000
or log 80. (c)
k1 2.303 8.314 300 RCOCl +C2H5OH +
N
190
6.9 8.314 RCOOC2H5 +
N+ –
k HCl
or 2 2000 [taking antilog]
k1

1 1 RCOCl +
75. (a) v RZ2 N +
22 32 N –
COR Cl
1 1 5R a better electrophile than RCOCl
= R =
4 9 36
C 2H 5OH
76. (d) The products of the concerned reaction RCOOC2H5 +
react each other forming back the reactants. +
N –
XeF6 3H 2 O XeO3 6HF HCl
77. (c) Although in both cases products are CH3I
and C 6 H5 OH; the two reactions follow PART - III (MATHEMATICS)
different mechanism. –1 dy
2
HI (g ) 81. (b) 1 y ( x e tan y
) 0
C6 H5 O CH 3 CH 3 I C 6 H 5 OH dx
S 2
N 1
conc . HI
(1 y 2 )dx (e tan y
x )dy
C6H5 O CH 3 CH 3 I C 6 H 5 OH
S 1
N1 dx e tan y x
Remember that during S 1 reaction, CH 3 dy 1 y 2
N
is formed because it is more stable than
1
C6H5 . dx 1
. x e tan y
dy 1 y 2 = 1 y 2
: OH + OH
Which is the linear different equation of the
H
78. (c) E
+
dx
E ; form Rx S , where R and S are
dy
functions of y or constant (s)
+ M effect in phenol Oxonium ion
activates benzene ring (more stable)
www.jeeneetbooks.in

WWW.IIT-NEET.XYZ

Solved Paper 2016 2016-21

1 Now b2 = a2 (1 – e2)
.dy 1
I.F = = e tan y
1 y2
e b2
b2 = 16 (1 – e2), 1 e2
Hence required solution is 16
1
e tan y b2 16 b2 16 b2
x.(I.F.) (I.F) dy e2 1 e=
1 y2 16 16 4
1
tan 1 y e tan 1y
Foci = ( ae, 0) = 16 b 2 ,0
x.e 2
(e tan y )dy
1 y
x2 y2 1
Given hyperbola :
2tan 1 y 144 81 25
1 e
x.e tan y dy ...(1)
1 y2 x2 y2
2 2
1
1 12 9
Put t tan y
5 5
dt 1 1
2
dt .dy Now, b2 = a2 (e2 – 1)
dy 1 y 1 y2
2 2
2 tan 1 y 9 12
e e 2t (e 2 1)
2
dy e 2t .dt K 5 5
1 y 2
2
Hence equation (1) becomes, 9
e2 1,
1 1 2t 12
x e tan y e K
2 81 144 81
e2 1
1 1 2tan 1 y 144 144
xetan y e K
2 15 5
e
tan 1 y 2 tan 1 y 12 4
2x e e K
82. (a) AO iˆ 2 ˆj kˆ Foci = ae, o = ( 3,0)

AC –2iˆ ˆj kˆ Since foci of the given ellipse and hyperbola


Angle between faces OAB and ABC coincide, therefore
= Angle between AO and AC 16 – b2 = 9
16 b2 3
If Q be the angle between AO and AC ,
b2 = 7
then
3
AO. AC 84. (a) f '(3) tan tan 1
cos 4 4
AO AC
85. (b) Required probability
1 ( 2) 2 ( 1) 1 1 3 = P (First win) × P(First win) × P(Second
= win)
1 4 1. 4 1 1 6
+ P(First Defeat) × P(First win) × P(Second
1
= cos120 win)
2
= 120° 1 1 1 1 1 1 1
=
x2 y2 2 2 2 2 2 2 4
83. (c) Given ellipse : 1
16 b2
EBD_7443
www.jeeneetbooks.in

WWW.IIT-NEET.XYZ

2016-22 Target VITEEE

86. (b) a b c 0 1
Put y in equation (1) , we get
x
(a b c )2 0
1 1
g(x).g g(x) g g(1) 2
2 2 2 x x
a b c 2(a .b b .c c .a ) 0
1 1
g(x).g g(x) g 2 2
9 + 4 + 1 + 2 (a.b b .c c.a ) 0 x x
[ g(1) 2 ]
a.b b .c c .a 7
This is valid only for the polynomial
87. (d) Let r (x) = f (x) . g (x) g (x) = 1 + xn ... (2)
= x2 . 2x = 2x3 Now g (2) = 5 (Given)
r (x) = 6x2 x 0 2 1 + 2n = 5 [Using equation (2)]
Put 6x2 = 0, x = 0 r ( x ) 0 16 + 2n = 4, 2n = 4, –4
Max r(x) = 2(2)3 = 16 Since the value of 2n cannot be –Ve.
or Max (f (x), g(x)) = 16 So, 2n = 4, n = 2
Now, put n = 2 in equation (2), we get
2
g (x) = 1 + x2
I(x) = 16 dx
0 Lt g(x) = Lt (1 x 2 ) = 1 + (3)2
x 3 x 3
2
I(x) = 16x 0 = 32 – 0 = 32 = 1 + 9 = 10, – 8
92. (c) Any tangent to parabola y2 = 8x is y =
88. (b) B = ABA–1 (Given)
But B = BAA–A 2
mx ...(i)
ABA–1 = BAA–1 AB = BA m
Now (A + B) (A – B) = A2 – AB + BA – B2 It touches the circle x 2 y 2 12x 4 0 ,
= A2 – AB + AB – B2 [ AB = BA]
if the length of perpendicular from the centre
= A2 – B2
89. (b) (1 + – 2)7 = (– 2 – 2)7 = (–2 2)7 (6, 0) is equal to radius 32 .
= –128 ( 4)3 2 = – 128 2
2
6m
90. (d) Here, r 2iˆ 3jˆ kˆ i 2ˆj 3kˆ m 32
m2 1
r i ˆj 2kˆ and F i ˆj kˆ
2
Then, the required moment is given by 1
3m 8(m 2 1)
r F i ˆj 2kˆ i ˆj kˆ m

(3m 2 1) 2 8(m 4 m2 )
ˆi ˆj kˆ
= 1 1 2 3iˆ 3jˆ m 4 2m 2 1 0 m 1
Hence, the required tangents are y = x + 2
1 1 1
and y = –x – 2.
Moment about given point = 3iˆ 3jˆ 93. (b) Given e x y 1 y2
91. (b) g (x). g(y) = g(x) + g (y) + g (x y) – 2 ...(1)
Put x = 1, y = 2, then ex y 1 y2
g (1). g(2) = g (1) + g (2) + g (2) – 2 Squaring both side, we have
5g (1) = g (1) + 5 + 5 – 2 e2x + y2 – 2exy = 1 + y2
4g (1) = 8 g(1) = 2 2ex y = e2x – 1
www.jeeneetbooks.in

WWW.IIT-NEET.XYZ

Solved Paper 2016 2016-23

e 2x 1 1 a2 sin 2 2
y y ex e x .
x 2 12 2
2e 0
2
94. (d) If curve r = a sin 3 a a2
. sin
To trace the curve, we consider the 12 2 24
following table : 95. (a) Replacing each hexadecimal digit by the
corresponding 4-digit binary numeral, we
3 5 have
3 0 2 3
2 2 2 (ABCD)16 = (1010 1011 1100 1101)2
2 5 96. (c) Let thenormal at ‘t1’ cuts the parabola again
0
6 3 2 3 6 at the point ‘t2’. the equation of the normal
r 0 a 0 a 0 a 0 at (at12, 2at1) is y + t1x = 2at1 + at13
Since it passes through the point ‘t2’ i.e
(at22, 2at2)
Thus there is a loop between 0&
3 2at2 + at1t22 = 2at1 + at13
2a(t1 – t2) + at1(t12 – t22) = 0
as r varies from r = 0 to r = 0.
2 + t1(t1 + t2) = 0 ( t1 t 2 0)
2 + t12 + t1t2 = 0
2
t1t2 = –(t12 + 2) t2 t1
t1
97. (b) s = t3 – 12t2 + 6t + 8
X ds
3t 2 24t 6
dt
d 2s
6t 24
dt 2
Acceleration = 0
Hence, the area of the loop lying in the 6t – 24 = 0
t=4
ds
13 2 Required velocity =
positive quadrant r d dt t 4
2 2
0 = 3 × (4) – 24 × 4 + 6
= 48 – 96 + 6 = 42 units
98. (a) X = a . b
1 3 2 1
sin . d 11
2
0
3 99. (b) Let cos , where 0 < < .
8 2
1 1 1 1
[On putting, 3 d d ] cos 1
3 2 8 2
1 1 1
cos cos 1 cos
a2 2
2 8 2
sin 2 d
6 1 1
0
Now , cos 1 = cos =
8 8
a 2 2 1 cos 2 1
. d cos 2 1 2sin 2 cos 2 1
6 2 2 8
0
EBD_7443
www.jeeneetbooks.in

WWW.IIT-NEET.XYZ

2016-24 Target VITEEE


9 3 1 1
cos 2 cos
2 16 2 4 1 3 2 5
3 = tan 1 1
1
[ 0< , so cos ] 1 .
2 4 2 4 2 3
100. (c) Two constraints are x 0, y 0 and the
1 5/ 6 2
third one will be of the type ax + by c. = tan tan 1 1 =
101. (b) Let y denote the number of bacteria at any 6 1 4
instant t then according to the question 6
dy dy 103. (b) We have,
y = k dt ... (i) abcd = cos (2 + 2 + 2 +2 ) + i sin (2 + 2
dt y
+ 2 +2 )
k is the constant of proportionality, taken
to be + ve on integrating (i), we get abcd = [cos (2 + 2 + 2 +2 )
log y = kt + c ... (ii) + i sin (2 + 2 + 2 +2 )]1/2
c is a parameter. let y0 be the initial number
or abcd
of bacteria
i.e., at t = 0 using this in (ii), c = log y0 = cos ( + + + ) + i sin ( + + + ).... (1)
log y = kt + log y0 [De Moivre's Theorem]
y 1
log = kt ...(iii) = cos ( + + + ) – i sin ( +
y0 abcd
+ + )
10 11y0
y = y0 y , when t = 2 .... (2)
100 0 10 Adding (1) and (2), we obtain
11y 0 1
abcd = 2 cos ( + + + )
So, from (iii), we get log 10 k (2) abcd
y0
104. (a) Standard deviation npq 0
1 11
k = log ... (iv) Now mean = np =25 and q < 1
2 10
y 1 11 So npq np 5
Using (iv) in (iii) log log t
y0 2 10 0 5
... (v) 105. (d) Number of ways
let the number of bacteria become 1, 00,
= [( 3C3 4C3 5C3 6C3 7C3 ) 2 8C 3 ] 2
000 to 2,00,000 in t1 hours. i.e., y = 2y0
when t = t1 hours. from (v) = 392
106. (d) rank (A B) rank (A)
2y0 1 11 2 log 2
log log t t1 = 11 and rank (A B) rank (B)
y0 2 10 1 log Therefore rank (A B) min (rank A, rank B)
10
107. (a) Let A (a, 0, 0), B (0, b, 0), C (0, 0, c), then
2 log 2
Hence, the reqd. no. of hours = 11 x y z
log equation of the plane is =1
10 a b c
Its distance from the origin,
1 1 1
102. (b) Consider sin cot 3 ...(i) 1 1 1 1
5 ... (i)
2 2 2
a b c p2
1 1 1
We have, sin 2 cot If (x, y, z) be centroid of ABC, then
5
From equation (i), we have a b c
x= ,y= ,z= ... (ii)
11 1 3 3 3
cos–1 2 + cot–1 3 = tan tan 1
2 3
www.jeeneetbooks.in

WWW.IIT-NEET.XYZ

Solved Paper 2016 2016-25

Eliminating a,b,c from (i) and (ii) required


3 3
locus is 6 1 1 1
= C3 .
x–2 + y–2 + z–2 = 9p–2 2 2 2
108. (c) There are 26 red cards and 26 black cards
7
i.e., total number of cards = 52 6 5 4 1 1 5
P(both cards of different colours) = 20
1 2 3 2 32 4 32
= P(B) P(R) + P(R) P(B)
111. (d) For the first circle centre = (3, 7)
26 26 26 26 26 26 26
= 2 Radius r1 32 72 – 48 10
52 51 52 51 52 51 51
109. (a) The equation of the hyperbola is For the second circle, centre (3, 0); radius
r2 = 3
x2 2y 2 2x 8y 1 0 So, r1 + r2 < d (distance between the centres)
or ( x 1) 2 2( y 2) 2 6 0 Circle don’t cut and hence the number
of common tangents = 4.
( x 1) 2 ( y 2) 2 112. (d) We have,
or 1 cos2 + sin + 1 = 0 1 – sin2 + sin + 1
6 3
=0
( y 2) 2 ( x 1) 2 sin = – 1 ( sin 2) =3 /2
or 1 ...(1)
3 6
5 7
2 2
,
Y X 4 4
or 1
3 6
113. (c) lim f (x) lim [(1 x)1/ x ]2 e2 f (0)
where X = x – 1 and Y = y – 2 ...(2) x 0 x 0
The centre = (0, 0) in the X-Y co-ordinates.
The centre = (1, 2) in the x-y co-ordinates, x
114. (d) log y log 2
using (2). log x
If the transverse axis be of length 2a, then a
dy
= 3 , since in the equation (1) the 0
dx x e
transverse axis is parallel to the y-axis.
If the conjugate axis is of length 2b, then
dx dx
b= 115. (d)
6. x (x2 4
1) 3/ 4 3/ 4
1
x5 1
But b 2 a 2 (e 2 1) x4
6 3(e 2 1) , e2 3 or e 3. 1 4
Put 1 t dx dt
4
The length of the transverse axis = 2 3 . x x5
So, integral is
The length of the conjugate axis = 2 6 .
1 1/ 4
1 dt 1
2b 2 2 6 I c t4 c 1
Latus rectum = 4 3 t3/ 4 4 x4
a 3
116. (a) The number of words starting from A are
110. (c) Probability of getting a blue ball at any draw 5! = 120
10 1 The number of words starting from I are 5!
= p = 120
20 2 The number of words starting from KA are
P [getting a blue ball 4th time in 7th draw] 4! = 24
= P [getting 3 blue balls in 6 draw] × P [a The number of words starting from KI are
blue ball in the 7th draw]. 4! = 24
EBD_7443
www.jeeneetbooks.in

WWW.IIT-NEET.XYZ

2016-26 Target VITEEE


The number of words starting from KN are 119. (a) If A and B are two sets having m and n
4! = 24 elements such that
The number of words starting from KRA n
are 3! = 6 ( 1) n r n
Cr r m
The number of words starting from KRIA 1 n m=
r 1
are 2! = 2
The number of words starting from KRIN Number of surjection from A to B
are 2! = 2 n
The number of words starting from KRISA = ( 1) 2 r 2 Cr (r) 4
are 1! = 1 r 1
The number of words starting from KRISNA = (–1)2–1 2C1 (1)4 + (–1)2–2 2C2 (2)4 = – 2 + 16
are 1! = 1 = 14
Hence, rank of word ‘KRISNA b
= 2(120) + 3(24) + 6 + 2(2) + 2(1) = 324 120. (d) Let I = x f ( x) dx
x y 2 z 1 a
117. (b) The lines are Let a + b – x = z – dx = dz
6 6 1
When x = a, z = b and when x = b, z = a
x 1 y z a
and I ( a b – z ) f ( z ) dz
12 6 1
b
Here, a b

a1 2ˆj k,
ˆ b 6iˆ 6jˆ k,
ˆ a2 ˆi, I ( a b ) f ( x ) dx – x f ( x ) dx
1
b a
a
b2 12iˆ 6jˆ kˆ
I ( a b ) f ( x ) dx – I ;
bb
ˆi ˆj k
2I ( a b ) f ( x ) dx
b1 b 2 6 6 1 12iˆ 18ˆj 36kˆ a
b
12 6 1 a b
Hence, I f ( x ) dx
2
a
a2 a1 . b1 b2
Shortest distance = PART - IV (ENGLISH)
b1 b2
121 (b)
ˆi 2ˆj kˆ . 12i 18ˆj 36kˆ 122 (a)
=
2 2 2 123 (d)
12 18 36
124 (a) "Whether" is correct because the question
12 36 36 84 concerns a choice not a condition. With the
= 2 expression "the number of" a singular verb
1764 42
is needed and hence "was" is correct.
118. (b) Given f(x) = 2 – |x – 5| "Liable" is used in expressions such as
Domain of f(x) is defined for all real values "liable to prosecution " and not for
of x. expressions of possibility.
Since, |x – 5| 0 – |x – 5| 0 125 (b)
2 – |x – 5| 2 f(x) 2
Hence, range of f(x) is (– , 2].
www.jeeneetbooks.in

WWW.IIT-NEET.XYZ

VITEEE
SOLVED PAPER 2015

(a) electrons can be emitted with any energy


PART - I (PHYSICS)
with a maximum of E0
1. When a hydrogen atom is raised from ground (b) no electrons will be emitted as only photons
energy level to excited energy level, then can emit electrons
(a) potential energy increases and kinetic (c) electrons can be emitted but all with an
energy decreases energy E0
(b) kinetic energy increases and potential
(d) electrons can be emitted with any energy
energy decreases
(c) Both KE and PE increase with a maximum of E0 – f , where being
(d) Both KE and PE decrease work function
2. The half life for -decay of uranium 92U228 is 7. An n-type semiconductor is
4.47 × 108 yr. If a rock contains 60% of original (a) neutral
228 atoms, then its age is
92U (b) positively charged
[take log 6 = 0.778, log 2 = 0.3] (c) negatively charged
(a) 1.2 × 107 yr (b) 3.3 × 108 yr (d) negatively or positively charged
9
(c) 4.2 × 10 yr (d) 6.5 × 109 yr depending on the amount of impurity added
3. A nuclear transformation is given by 8. In the half wave rectifier circuit operating with
Y ( n, ) ® 3 Li 7 . The nucleus of element Y is 50 Hz mains frequency. The fundamental
frequency in the ripple will be
(a) 5Be 11 (b) 5B10
9 (a) 100 Hz (b) 20 Hz
(c) 5B (d) 6C12
(c) 50 Hz (d) 25 Hz
4. The angular momentum of an electron in Bohr’s
hydrogen atom whose energy is –3.4 eV, is 9. The input resistance of a common emitter
amplifier is 330 and the load resistance is 5
5h h
(a) (b) k . A change of base current is 15 µA results in
2p 2p
the change of collector current by 1mA. The
h 2h voltage gain of amplifier is
(c) (d)
p 3p (a) 1000 (b) 10001
5. When the momentum of a photon is changed by (c) 1010 (d) 1100
an amount p’ then the corresponding change 10. To get an output y = 0 from the circuit shown in
in the de-Broglie wavelength is found to be the figure, the input C must be
0.20%. Then, the original momentum of the
photon was A
(a) 300 p’ (b) 500 p’ B y
(c) 400 p’ (d) 100 p’ C
6. Suppose a beam of electrons with each electron (a) 0 (b) 1
having energy E0 incident on a metal surface
(c) either 0 or 1 (d) None of these
kept in an evacuated chamber. Then ,
EBD_7443
www.jeeneetbooks.in

WWW.IIT-NEET.XYZ

2015-2 Target VITEEE


11. Equal charges q each are placed at the vertices is
of an equilateral triangle of side r. The magnitude
(a) 10 (b) 30
of electric field intensity at any vertex is
(c) 20 (d) 40
2q q 17. In a potentiometer experiment, the balancing
(a) (b)
4pe0 r 2 4pe 0 r 2 length of a cell is 560 cm. When an external
resistance of 10 is connected in parallel to
3q 2q the cell, the balancing length changes by 60
(c) (d)
4pe0 r 2 4pe 0 r 2 cm.The internal resistance of a cell is
12. Two points masses, m each carrying charges -q (a) 1.4 (b) 1.6
and +q are attached to the ends of a massless (c) 0.12 (d) 1.2
rigid non-conducting wire of length ‘L’. When 18. Two sources of equal emf are connected to a
this arrangement is placed in a uniform electric resistance R. The internal resistance of these
field, then it deflects through an angle i. The sources are r1 and r2 (r1 > r2 ). If the potential
minimum time needed by rod to align itself along difference across the source having internal
the field is resistance r2 is zero, then
mL p mL ær + r ö
2p R = r2 ççç 1 2 ÷÷÷
r1r2
(a) (b) (a) R= (b)
qE 2 2qE r2 - r1 è r2 - r1 ø÷

p
2mL
2p
3mL æ rr ö
(c) (d) (c) R = çç 1 2 ÷÷÷ (d) R = r2 – r1
qE qE çè r2 + r1 ÷ø
13. A condenser of capacitance C is fully charged 19. An electron of mass 9.0 × 10–31kg under the
by a 200V supply. It is then discharged through action of a magnetic field moves in a circle of
a small coil of resistance wire embedded in thermally radius 2 cm at a speed of 3 × 106 m/s. If a proton
insulated block of specific heat 250 J/kg-K and of mass 1.8 × 1027 kg was to move in a circle of
of mass 100 g. If the temperature of the block same radius in the same magnetic field, then its
rises by 0.4 K, then the value of C is speed will become
(a) 300µF (b) 200µF (a) 1.5 × 103 m/s (b) 3 × 106 m/s
(c) 400µF (d) 500µF 4
(c) 6 × 10 m/s (d) 2 ×106 m/s
14. The capacitance of a parallel plate capacitor with 20. A horizontal rod of mass 0.01kg and length 10
air as medium is 3 µF. As a dielectric is introduced cm is placed on a frictionless plane inclined at
between the plates, the capacitance becomes 15 an angle 60° with the horizontal and with the
µF. The permittivity of the medium in length of rod parallel to the edge of the inclined
C2N–1m–2 is plane. A uniform magnetic field is applied
(a) 8.15 × 10–11 (b) 0.44 × 10–10 ‘Vertically downwards. If the current through the
(c) 15.2 × 1012 (d) 1.6 × 10–14 rod is 1.73 A, then the value of magnetic field
15. The masses of three copper wires are in the induction B for which the rod remains stationary
ratio 2 : 3 : 5 and their lengths are in the ratio on the inclined plane is
5 : 3 : 2.Then, the ratio of their electrical (a) 1 T (c) 3 T
resistances is (b) 2.5 T (d) 4 T
(a) 1 : 9 : 15 (c) 2 : 3 : 5 21. A current of 2 A is flowing in the sides of an
(b) 5 : 3 : 2 (d) 125 : 30 : 8 equilateral triangle of side 9 cm. The magnetic
16. A 30V-90W lamp is operated on a 120 V DC line. field at the centroid of the triangle is
A resistor is connected in series with the lamp in (a) l.66 ×10–5 T (b) 1.22 × 10–4 T
order to glow it properly. The value of resistance (c) 1.33 × 10 T –5 (d) 1.44 × 10–4 T
www.jeeneetbooks.in

WWW.IIT-NEET.XYZ

Solved Paper 2015 2015-3

22. The direction of magnetic field dB due to current 28. A coil of area 5 cm2 having 20 turns is placed in
element dl at a distance r is the direction of a uniform magnetic field of 103gauss. The normal
(a) r × dl (b) dl × r to the plane of coil makes an angle 30° with the
(c) (rdl)r (d) dl magnetic field. The flux through the coil is
23. A galvanometer with a scale divided into 100 (a) 6.67 × 10–4 wb (b) 3.2 × 10–5 Wb
equal divisions has a current sensitivity of 10 (c) 5.9 × 10–4 wb (d) 8.65 × 10–4 wb
divisions per milliampere and a voltage 29. The current graph for resonance in LC circuit
sensitivity of 2 divisions per millivolt. The is
galvanometer resistance will be
(a) 4 (b) 5
I
(c) 3 (d) 7
24. The earth is considered as a short magnet with (a)
its centre coinciding with the geometric centre
of earth. The angle of dip related to the
magnetic latitude as
1
(a) tanf = (b) tan = 2tan f
2 tan Z

(c) tan = 2tan f (d) tanf = 2tan


(b)
25. Which of the following statement related to
hysteresis loop is incorrect?
(a) The curve of B against H for a
ferromagnetic material is called hysteresis
loop I
(b) The area of B-H curve is a measure of power
dissipated per cycle per unit area of the
(c)
specimen
(c) Coercitivity is a measure of the magnetic
field required to destroy the residual
magnetism of ferromagnetic material
(d) The retentivity of a specimen is the measure Z
of magnetic field remaining in the specimen
when the magnetising field is removed
26. A magnetic needle lying parallel to the magnetic (d)
field requires W units of work to turn it through
an angle 45°. The torque required to maintain
the needle in this position will be 30. The value of inductance L for which the current
is maximum in series LCR circuit with C =10 µF
1 and =1000 rad/s
(a) 2W (b)
3W (a) 10 mH (b) 50 mH
(c) 200 mH (d) 100 mH
W
(c) ( 2 -1)W (d) 31. A ray of light is incident on a plane mirror at an
( 2 -1) angle of 30°. At what angle with the horizontal
27. An induced emf has must a plane mirror be placed so that the
(a) a direction same as field direction reflected ray becomes vertically upwards?
(b) a direction opposite to the field direction (a) 40° (b) 20°
(c) no direction of its own (c) 30° (d) 60°
(d) None of the above
EBD_7443
www.jeeneetbooks.in

WWW.IIT-NEET.XYZ

2015-4 Target VITEEE


32. A compound microscope having magnifying 38. Pick out the wrong statement.
power 35 with its eye-piece of focal length 10 (a) Gauss’s law of magnetism is given by
cm. Assume that the final image is at least
distance of distinct vision then the magnification ò B · ds = 0
produced by the objective is (b) An EM wave is a wave radiated by a charge
(a) –4 (b) 5 at rest and propagates through electric field
only
(c) 10 (d) –10
(c) A time varying electric field is a source
33. The refractive index for a prism is given as
of changing magnetic field
A (d) Faraday’s law of EM induction is
m = cot . Then, angle of minimum deviation
2 d fB
in terms of angle of prism is ò E.dl = – dt
(a) 90°–A (b) 2A 39. When sunlight is scattered by atmospheric
(c) 180°–A (d) 180°–2A atoms and molecules the amount of scattering
34. Two convex lenses of power 2D and 5D are of light of wavelength 880nm is A. Then, the
amount of scattering· of light of wavelength
1 330 nm is approximately
separated by a distance m. The power of
3 (a) 10 A (b) 20 A
optical system formed is (c) 40 A (d) 50.5 A
(a) + 2 D (b) –2 D 40. The ratio of volume occupied by an atom to the
(c) – 3 D (d) + 3 D volume of the nucleus is
35. Two light rays having the same wavelength in (a) 105:1 (b) 1020 :1
15
(c) 10 :1 (d) 1:1015
vacuum are in phase initially. Then, the first ray
travels a path L1through a medium of refractive
index µ1 while the second ray travels a path L2 PART - II (CHEMISTRY)
through a medium of refractive index µ2. The
41. When copper is treated with a certain
two waves are then combined to observe
concentration of nitric acid, nitric oxide and
interference. The phase difference between the
nitrogen dioxide are liberated in equal volumes
two waves is
according to the equation,
2p æç L1 L2 ö÷ 2p xCu + yHNO3 ¾¾ ® Cu(NO3)2 + NO
ç - ÷÷ ( L2 - L1 )
èç m1 m 2 ø÷
(a) (b) + NO2 + H2O
The coefficients of x and y are respectively
2p 2p (a) 2 and 3 (b) 2 and 6
(c) (m 2 L1 -m1L2 ) (d) (m1L1 -m 2 L2 )
(c) 1 and 3 (d) 3 and 8
36. Two polaroids are kept crossed to each other. If 42. A saturated solution of H2S in 0.1 M HCl at 25°C
one of them is rotated an angle 60°, the contains S2– ion concentration of 10–23 mol L–1.
percentage of incident light now transmitted The solubility product of some sulphides are
through the system is CuS=10–44, FeS = 10–14, MnS = 10–15, CdS =
10–25. If 0.01 M solution of these salts in 1M
(a) 10% (b) 20% HCl are saturated with H2S, which of these will
(c) 25% (d) 12.5% be precipitated?
37. An electromagnetic wave propagating along (a) All
north lies its electric field vertically upward. The (b) All except MnS
magnetic field vector points towards
(c) AU except MnS and FeS
(a) downward (b) east
(d) Only-CuS
(c) north (d) south
www.jeeneetbooks.in

WWW.IIT-NEET.XYZ

Solved Paper 2015 2015-5

43. Consider the water gas equilibrium reaction,


B ¬¾¾¾¾ ¾ H3 O+
(i) B H /THF
C(s) + H2O (g) CO (g) + H2(g) 49. CH2
2 6
(ii) H O /OH –
2 2
A
Which of the following statements is true at A and B respectively are
equilibrium?
(a) If the amount of C(s) is increased, less CH2OH
(a) Both
water would be formed
(b) If the amount of C(s) is increased, more CO
CH3
and H2 would be formed (b) Both
(c) If the pressure on the system is increased OH
by halving the volume, more water would CH3
be formed (c) CH2OH,
(d) If the pressure on the system is increased OH
by halving the volume, more CO and H2 CH3
would be formed (d) CH2OH
44. The chemical composition of slag formed during OH,
the smelting process in the extraction of copper 50. A certain metal when irradiated by light (r = 3.2 ×
is 1016Hz) emits photoelectrons with twice kinetic
(a) Cu2O + FeS (b) FeSiO3 energy as did photoelectrons when the same
(c) CuFeS2 (d) Cu2S + FeO metal is irradiated by light (r =2. 0 × 1016Hz).
The v0 of metal is
45. X Cl2 (excess) + Y Cl2 ¾¾
® X Cl4 + Y ¯
(a) 1.2 ×1014 Hz (b) 8 × 1015 Hz
1 16
(c) 1.2 ×10 Hz (d) 4 × 1012 Hz
Y O ¾¾¾ D
>400ºC
® O2 +Y
2 51. Gaseous benzene reacts with hydrogen gas in
Ore of Y would be, presence of a nickel catalyst to form gaseous
(a) siderite (b) malachite cyclohexane according to the reaction,
(c) hornsilver (d) cinnabar C6H6(g) + 3H2(g) ¾¾ ® C6H12(g)
46. For the given reaction, A mixture of C6H6 and excess H2 has a pressure
H2 (g) + Cl2 (g) ¾¾
® 2H+ (aq) + 2Cl– (aq); of 60 mm of Hg in an unknown volume. After the
gas had been passed over a nickel catalyst and
D Gº = –262.4kJ
all the benzene converted to cyclohexane, the
The value of free energy of formation
pressure of the gas was 30 mm of Hg in the same
( D Gºf ) for the ion Cl–1 (aq), therefore will be volume at the same temperature. The fraction
(a) –131.2 kJ mol–1 (b) + 131.2 kJ mol–1 of C6H6 (by volume) present in the original
(c) –262.4 kJ mol–1 (d) + 262.4 kJ mol–1 volume is
47. The molarity of NO3– in the solution after 2L of (a) 1/3 (b) 1/4
3M AgNO3 is mixed with 3L of 1M BaCl2 is (c) 1/5 (d) 1/6
(a) 1.2 M (b) 1.8 M 52. An alloy of copper, silver and gold is found to
(c) 0.5 M (d) 0.4 M have copper atom constituting the ccp lattice. If
48. Amongest silver atom occupy the edge centres and gold
NO3– , AsO3– 2– – 2– 3–
3 , CO 3 , ClO 3 , SO 3 and BO 3
, atom is present at body centred, the alloy has a
the non-planar species are formula
(a) Cu4 Ag2 Au (b) Cu4 Ag4 Au
(a) CO32– , SO32– and BO32–
(c) Cu4 Ag3 Au (d) CuAgAu
AsO3– Given, D Gº = – nFE° cell and D Gº = –RT ln k.
– 2–
(b) 3 , ClO 3 and SO 3 53.
The value of n = 2 will be given by the slope of
(c) NO3– , CO32– and BO3–
3
which line in the figure
(d) SO 32– , NO 3– and BO 3–
3
EBD_7443
www.jeeneetbooks.in

WWW.IIT-NEET.XYZ

2015-6 Target VITEEE


58. A compound X on heating gives a colourless
1.0 B gas. The residue is dissolved in water to obtain
Y. Excess CO2 is passed through aqueous
0.5 A solution of Y when Z is formed. Z on gentle
heating gives back X. The compound X is
O In k (a) Ca(HCO3)2 (b) CaCO3
E°cell

–0.5 C (c) NaHCO3 (d) Na2 CO3


59. Which two sets of reactants best represents
D the amphoteric character of Zn(OH)2?
–1.0 Set I Zn(OH)2(s) and OH (aq)
(a) OA (b) OB Set II Zn(OH)2(s) and H2O (I)
Set III Zn(OH)2(s) and H+ (aq)
(c) OC (d) OD
Set IV Zn(OH)2(s) and NH3 (aq)
54. The false statements among the following are
I. A primary carbocation is less stable than a (a) III and II (b) I and III
tertiary carbocation. (c) IV and I (d) II and IV
II. A secondary propyl carbocation is less C6H5—NO2 ¾¾ ¾® A ¾¾ ¾¾ ® B.
Zn dust cold
60. NH 4Cl conc. HCl
stable than allyl carbocation.
A and B respectively are
III. A tertiary free radical is more stable than a
primary free radical. (a) NHOH, HO NH2
IV. Isopropyl carbanion is more stable than
ethyl carbanion.
(a) I and II (b) II and Ill (b) NH NH

(c) I and IV (d) II and IV


55. A colourless water soluble solid A on heating H2N NH2
gives equimolar quantities of B and C. B gives
dense white fumes with HCl and C does so with
NH3. B gives brown precipitate with Nessler’s (c) N N
reagent and C gives white precipitate with
nitrates of Ag+, Pb+ and Hg+. A is O
(a) NH4Cl (b) NH4CO3
N N NH2
(c) NH4NO2 (d) FeSO4
(d) None of the above
61. Point out incorrect stability order
56. The IUPAC name of (a) [Cu(NH3 ) 4 ] 2+ < [Cu(en) 2 ] 2+ < [Cu
(trien)]2+
is (b) [Fe(H2O)6]3+ < [Fe(NO2)6]3– < [Fe (NH3)6]3+
(a) 4-ethyl-5,6,7,9-tetramethyldeca-2, 9-diene (c) [Co(H2O)6]3+ < [Rh(H2O)6 ]3+ < [Ir (H2O)6]3+
(b) 7-ethyl-2,4,5,6-tetramethyldeca-1, 8-diene (d) [Cr(NH3)6 ]+ < [Cr(NH3)6 ]2+ < [Cr (NH3)6]3+
(c) 7-ethyl-2,4,5,6-tetramethyldeca-1, 7-diene 62. Consider the following changes
(d) 7-(1-propenyl)-2,3,4,5-tetramethyl non-1-ene M(s) ¾¾
® M(g) ...(1)
57. Caffeine has a molecular weight of 194 u. If it M(g) ¾¾
® M2+(g) + 2e– ...(2)
contains 28.9% by mass of nitrogen, number of
atom of nitrogen in one molecule of caffeine is M(g) ¾¾
® M+(g) + e– ...(3)
(a) 4 (b) 6 M+ (g) ¾¾
® M2+(g) + e– ...(4)
(c) 2 (d) 3 M(g) ¾¾
® M2+(g) + 2e– ...(5)
www.jeeneetbooks.in

WWW.IIT-NEET.XYZ

Solved Paper 2015 2015-7

The second ionisation energy of M could be 68. The ratio of slopes of Kmax vs V and V0 vs v
determined from the energy values associated curves in the photoelectric effects gives
with (v = frequency, Kmax = maximum kinetic energy,
(a) 1 + 2 + 4 (c) 1 + 5 – 3 v0 = stopping potential)
(b) 2 + 3 – 4 (d) 5–3 (a) the ratio of Planck’s constant of electronic
63. In benzene, the triple bond consists of charge
(a) one sp-sp sigma bond and two p-p pi bonds (b) work function
(b) two sp-sp sigma bonds and one p-p pi bond (c) Planck’s constant
(d) charge of electron
(c) one sp2-sp2 sigma bond, one p-p pi bond
69. With excess of water, both P2O5 and PCl5 give
(d) one sp2-sp2 sigma bond, one sp2-sp2 pi
(a) H3PO3 (b) H3PO2
bond and one p-p pi bond
64. In keto-enol tautomerism of dicarbonyl (c) H3PO4 (d) H4P2O7
compounds; the enol-form is preferred in contrast 70. The dissolution of AI(OH)3 by a solution of
to the keto-form, this is due to NaOH results in the formation of
(a) presence of carbonyl group on each side (a) [AI(H2O)4 (OH)2]+
of —CH2 — group (b) [AI(H2O)3(OH)3]
(b) resonance stabilisation of enol form (c) [AI(H2O)2 (OH)4 ]–
(c) presence of methylene group (d) [AI(H2O)6 (OH)3]
71. Which of the following does not exist?
(d) rapid chemical exchange
65. An organic compound having carbon, (a) Kl + l2 ¾¾
® Kl3
hydrogen and sulphur contains 4% of sulphur.
The minimum molecular weight of the compound (b) KF + F2 ¾¾
® KF3
is (c) KBr + ICl2 ¾¾
® K[BrlCl]
(a) 200 (b) 400
(d) KF+ BrF3 ¾¾ ® K[BrF4]
(c) 600 (d) 800 72. If the ionisation energy and electron affinity of
66. Which one of the following is a case of negative an element are 275 and 86 kcal mol-1 respectively,
adsorption? then the electronegativity of the element on the
(a) Acetic acid solution in contact with animal Mulliken scale is
charcoal. (a) 2.8 (b) 0.0
(b) Dilute KCl solution in contact with blood (c) 4.0 (d) 2.6
charcoal. 73. Which of the following sets of reactants is used
(c) Concentration KCl solution in contact with for preparation of paracetamol from phenol?
blood charcoal. (a) HNO3 , H2 /Pd,(CH3CO)2O
(d) H2 gas in contact with charcoal at 300 K. (b) H2SO4 ,H2 /Pd, (CH3CO)2O
67. The concentrations of the reactant A in the (c) C6H5 N2 Cl, SnCl2/HCl, (CH3CO)2O
reacntion A ® B at different times are given (d) Br2/H2O, Zn / HCl, (CH3CO)2O
below
74. A certain compound gives negative test with
Concentration (M) Time (Minutes)
ninhydrin and positive test with Benedict’s
0.069 0 solution. The compound is
0.052 17 (a) a protein (b) a monosaccharide
0.035 34 (c) a lipid (d) an amino acid
0.018 51 75. Super glue or crazy glue is
The rate constant of the reaction according to (a) poly (methyl methacrylate)
the correct order of reaction is (b) poly (ethyl acrylate)
(a) 0.001 M/min (b) 0.001 min-1 (c) poly (methyl -cyanoacrylate)
(c) 0.001 min/M (d) 0.001 M-1 min-1 (d) poly (ethyl methacrylate)
EBD_7443
www.jeeneetbooks.in

WWW.IIT-NEET.XYZ

2015-8 Target VITEEE

OH
80 Which of the following reactions is used to
HNO3 prepare isobutane?
COOH X (a) Wurtz reaction of C2H5Br
76.
Br 2, water (b) Hydrolysis of n-butylmagnesium iodide
Y
(c) Reduction of propanol with red phosphorus
X and Y respectively are and HI
(a) picric acid, 2, 4, 6-tribromophenol
(d) Decarboxylation of 3-methylbutanoic acid
(b) 5-nitrophenol acid, 5-bromosalicylic acid
(c) o-nitrophenol, O-bromophenol
(d) 3,5-dinitrosalicylic acid, 3, 5-dibromosalicylic PART - III (MATHEMATICS)
acid
77. In the cannizzaro reaction given below 81. The differential equation
(3x + 4y + 1)dx + (4x + 5y + 1)dy =0
2Ph—CHO ¾¾¾ ®

OH
represents a family of
(a) circles (b) parabolas
Ph—CH 2 — OH + PhCO-2
(c) ellipses (d) hyperbolas
the slowest step is
å D(r )
n
82. If D(r ) = r r3 , then is
(a) the attack of OH at the carbonyl group 1 n(n + 1) r =1
(b) the transfer of hydride ion to the carbonyl equal to
group
år år
n n
(c) the abstraction of a proton from the (a)
2 3
(b)
carboxylic acid r =1 r =1
(d) the deprotonation of Ph-CH2OH
år år
n n
4
78. The reaction of 1-bromo-3-chlorocyclobutane (c) (d)
with metallic sodium in dioxane under reflux r =1 r =1

conditions gives 83. If A, B, C are three events associated with a


random experiment, then

(a) Cl Cl B C
P ( A) P P B is
A A
(a) P (A B C) (b) P (A B C)
(b) Br Cl
C B
(c) P B (d) P
A A
(c) Br Br é1 3 1 ù
84. If A = êê 2 1 –1úú , then rank (A) is equal to
êë3 0 1 úû
(d) (a) 4 (b) 1
(c) 2 (d) 3
79. Identify Z in the following reaction sequence 85. The probability of atleast one double six being
thrown in n throws with two ordinary dice is
CH3CH2 CH2OH ¾¾¾¾®
Conc.H2 SO4
160 –180°C
X greater than 99%.
Then, the least numerical value of n is
Y
(a) 100 (b) 164
Z
(i) Alc. KOH
(ii) NaNH 2
(c) 170 (d) 184
86. Find the value of k for which the simultaneous
(a) CH3 — CH(NH2 ) — CN2 NH2 equations x + y + z = 3; x + 2y + 3z = 4 and x + 4y
(b) CH3 — CHOH — CH2OH + kz = 6 will not have a unique solution.
(c) CH3 — C(OH) = CH2 (a) 0 (b) 5
(d) CH3 — C º CH (c) 6 (d) 7
www.jeeneetbooks.in

WWW.IIT-NEET.XYZ

Solved Paper 2015 2015-9

87. If the complex number z lies on a circle with centre 93. If N is a set of natural numbers, then under binary
1 operation a ·b =a + b, (N, ·) is
at the origin and radius , then the complex (a) quasi-group (b) semi-group
4
(c) monoid (d) group
number –1+ 8z lies on a circle with radius
dx
ò cos x +
(a) 4 (b) 1
(c) 3 (d) 2 94. equals
3sin x
88. If line y = 2x + c is a normal to the ellipse
1 æx p ö
x2 y 2 log tan çç + ÷÷÷ + C
+ = 1 , then (a)
2 èç 2 12 ø
9 16
1 æx p ö
c=
2
c=
73 log tan çç - ÷÷÷ + C
(a)
3
(b)
5
(b)
3 èç 2 12 ø
æ x pö
c=
14
c=
5 log tan çç + ÷÷÷ + C
(c)
73
(d)
7
(c)
èç 2 6 ø

æ 1 ö÷ 1 æ x pö
log tan çç - ÷÷÷ + C
2

å èççç x
6
n
+ ÷ is
89. If x2 + x + 1 = 0, then the value of
n =1 x n ø÷
(d)
2 èç 2 6 ø
(a) 13 (b) 12 95. If (2, 7, 3) is one end of a diameter of the sphere
(c) 9 (d) 14 x2 + y2 + z2– 6x –12y –2z + 20 = 0, then the
90. If p : It rains today, q : I go to school, r : I shall coordinates of the other end of the diameter are
meet my friends and s : I shall go for a movie, (a) (–2, 5, –1) (b) (4, 5, 1)
then which of the following is the proportion? (c) (2, –5, 1) (d) (4, 5, –1)
If it does not rain or if I do not go to school, then 96. The two lines x = my + n, z = py + q and x = m y + n ,
I shall meet my friend and go for a movie. z = p y + q are perpendicular to each other, if
(a) (~ p Ù ~ q) Þ (r Ù s ) m p
(a) mm + pp =1 (b) + = -1
(b) ~ ( p Ù q ) Þ ( r Ù s) m' p'
(c) ~ ( p Ú q) Þ (r Ú s) m p
(d) None of these (c) + =1 (d) mm + pp = –1
m' p'
é1 3 1ù
ê ú 97. A tetrahedron has vertices at O(0, 0, 0),
91. If the matrix A = ê –1 2 –3ú then adj (adj A) is A (1, –2, 1), B (– 2, 1, 1) and C (1, –1, 2). Then, the
êë 0 1 2úû
angle between the faces OAB and ABC will be
equal to
æ1ö æ -1ö
é12 36 12 ù é12 26 –12ù cos –1 çç ÷÷÷ cos –1 çç ÷÷÷
ê –12 24 –36 ú ê 24 36 –36úú
(a) çè 2 ø (b)
èç 6 ø
(a) ê ú (b) ê
êë 0 12 24 úû êë 0 12 –24úû
æ -1ö æ1ö
cos –1 çç ÷÷÷ cos –1 çç ÷÷÷
é12 -12 36ù (c)
èç 3 ø (d) çè 4 ø
ê ú
(c) êê 24 -24 –36úú (d) None of there
êë 0 12 24úû 98. If a line segment OP makes angles of and
4 3
92. Which of the following options is not the
asymptote of the curve with X-axis and Y-axis, respectively. Then, the
3x3 + 2x2y – 7xy2 + 2y3 – 14xy + 7y2 + 4x + 5y direction cosines are
= 0? 1 3 1 1 1 1
(a) , , (b) ,,
-1 5 7 2 2 2 2
(a) y= x- (b) y = x- 2 2
2 6 6
1
3 3 (c) 1, 3, 1 (d) 1, ,1
(c) y = 2x + (d) y = 3x - 3
7 2
EBD_7443
www.jeeneetbooks.in

WWW.IIT-NEET.XYZ

2015-10 Target VITEEE


99. If p, q, r are simple propositions with truth values
T, F, T, then the truth value of (~p Ú q) Ù ~ r (c) (4 2 -1) sq units
Þ p is (d) (4 2 +1) sq units
(a) true (b) false 105. If a, b and c are three non-coplanar vectors, then
(c) true, if r is false (d) true, if q is true (a + b – c) · [(a – b) × (b – c) equals
100. On the interval [0, 1], the function x25(1 – x)75 (a) 0 (b) a . b × c
takes its maximum value at the point (c) a. c × b (d) 3a. b × c
106. If there is an error of m% in measuring the edge
1
(a) 0 (b) of cube, then the percent error in estimating
4 its surface area is
1 1 (a) 2 m (b) 3 m
(c) (d) (c) 1 m (d) 4 m
2 3
107. If the rectangular hyperbola is x2 – y2 = 64. Then,
1 which of the following is not correct?
101. If |z| ³ 3, then the least value of z + is (a) The length of latus rectum is 16
4
(b) The eccentricity is 2
11 11 (c) The asymptotes are parallel to each other
(a) (b)
2 4
(d) The directrices are x = ± 4 2
1 108. The equation of tangents to the hyperbola
(c) 3 (d) 3x2 – 2y2 = 6, which is perpendicular to the line
4
x – 3y = 3, are
102. The normal at the point ( at12 , 2 at1 ) on the
(a) y = –3x ± 15 (b) y = 3x ± 6
parabola meets the parabola again in the point
(c) y = –3x ± 6 (d) y = 2x ± 15
( at22 , 2 at 2 ) , then
tan x -1
2 2 109. xlim is equal to
(a) t 2 = -t1 + (b) t 2 = -t1 - ®p / 4cos 2 x
t1 t1
(a) 1 (b) 0
(c) – 2 (d) –1
2 2
(c) t 2 = t1 - (d) t 2 = t1 + 110. The area of the region bounded by the curves
t1 t1 x2 + y2 = 9 and x + y = 3 is
103. If a = ˆi - ˆj + 2kˆ and b = 2ˆi - ˆj + kˆ , then the 9p 1 9p 1
(a) + (b) -
angle between a and b is given by 4 2 4 2
æp 1ö æp 1ö
-1 æ 1 ö 9 çç - ÷÷÷ 9 çç + ÷÷÷
(a) tan - 1 (1) (b) sin ççç ÷÷÷ (c)
èç 4 2 ø (d)
èç 4 2 ø
è 2ø
111. For any three vectors a, b and c, [a + b, b + c, c + a]
1 1 is
(c) sec–1 (1) (d) tan
3 (a) [a b c] (b) 3 [a b c]
(c) 2 [a b c] (d) 0
104. The area bounded by the curves y = cos x and
p /2

y = sin x between the ordinates x = 0 and x =


3p
is
112. ò 0
sin 2 x.log tan x dx is equal to
2 (a) 0 (b) 2
(c) 4 (d) 7
(a) (4 2 - 2) sq units 113. If the mean and variance of a binomial
distribution are 4 and 2, respectively. Then, the
(b) (4 2 + 2) sq units
probability of atleast 7 successes is
www.jeeneetbooks.in

WWW.IIT-NEET.XYZ

Solved Paper 2015 2015-11

3 4 sin t
(a) (b) 117. At t = 0, the function f (t) = has
214 173 t
9 7 (a) a minimum
(c) (d) (b) a discontinuity
256 231
114. The shortest distance between the lines (c) a point of inflexion
(d) a maximum
x -7 y + 4 z -6
= = 118. Using Rolle’s theorem, the equation
3 -16 7
a0xn + a1xn–1 + . . . + an = 0
x -10 y - 30 4 - z
and = = is has atleast one root between 0 and 1, if
3 8 5
a0 a
234 288 (a) + 1 + .... + an-1 = 0
(a) units (b) units n n -1
7 21
a0 a
221 234 (b) + 1 + .... + an-2 = 0
(c) units (d) units n -1 n - 2
3 21
115. If a plane passing through the point (2, 2, 1) and
is perpendicular to the planes 3x + 2y + 4z + 1= 0 (c) na0 + (n -1)a1 + .... + an-1 = 0
and 2x + y + 3z + 2 = 0. Then, the equation of the
plane is a0 a
(d) + 1 + .... + an = 0
(a) 2x – y – z – 1 = 0 (b) 2x + 3y + z –1= 0 n +1 n
(c) 2x + y + z + 3 = 0 (d) x – y + z –1 = 0 119. Which of the following inequality is true for
116. From a city population, the probability of x > 0?
7
selecting a male or smoker is , a male x
10 (a) log(1 + x) < <x
1+ x
2
smoker is and a male, if a smoker is already x
5 (b) < x < log (1 + x )
1+ x
2
selected, is . Then, the probability of x
3 (c) x < log (1 + x ) <
1+ x
3
(a) selecting a male is x
2 (d) < log (1 + x ) < x
1+ x
1
(b) selecting a smoker is d 2x
5 120. The solution of - x = k , where k is a
dy 2
2
(c) selecting a non-smoker is non-zero constant, vanishes when y = 0 and
5
tends of finite limit as y tends to infinity, is
(d) selecting a smoker, if a male is first selected,
(a) x =k (1 + e–y) (b) x = k(e y + e –y – 2)
8 –y
(c) x = k(e –1) (d) x = k(ey –1)
is given by
5
EBD_7443
www.jeeneetbooks.in

WWW.IIT-NEET.XYZ

2015-12 Target VITEEE

SOLUTIONS
PART - I (PHYSICS) p p
1. (a) As r increase, the potential energy p p
increases. Thus, it decreases kinetic
energy of hydrogen atom. So, when an p 0.20 1
atom jumps from one energy level to the p 100 500
higher level, its potential energy increases
and kinetic energy decreases. or, p = 500 p
2. (b) Given: T1/2 = 4.47 × 108 yr 6. (a) The emitted electrons may lie near the
surface and can have a maximum amount
n
N 60 1 10 of energy E0.
2n
N0 100 2 6 If they are from deep inside, then energy
is less than E0.
Apply logarithm on both sides
7. (a) The n-type semiconductor has excess of
n log2 = log 10 – log 6
n × 0.3 = 1 – 0.778 = 0.22 free electrons for conduction. The total
number of electrons in an atom is equal
0.222 to the total number of protons in the
n 0.74
0.3 nucleus. So, n-type semiconductor is
So, t = nT1/2 = 0.74 × 4.47 × 108 neutral.
or, t = 3.3 × 108 yr 8. (c) The output is obtained for half cycle only
3. (b) Y(n, ) the nucleus splits into -particle in half wave rectifier. Therefore, frequency
and neutrons of the ripple is same as that of the input
A 1 7 4 i.e. 50 Hz.
i.e. Z Y 0n 3 Li 2 He 9. (c) Given: IC = 1mA = 10–3 A
So A + 1 = 7 + 4 A = 10 Ib = 15 A = 15 × 10–6 A
and Z + 0 = 3 + 2 or Z = 5 RL = 5k = 5 × 103 .
Hence, the nucleus of element Y is boron Ri = 330
10 10
5Y = 5B . The voltage gain of an amplifier
4. (c) Energy of electron in nth orbit of hydrogen
atom IC R L
Ar
13.6 Ib R i
En eV
n2 10 3
5 103
= 1010
13.6 15 10 6 330
3.4 n2 =4
n2 10. (a) As we know, output of OR gate
or, n = 2 Y=A+ B
Angular momentum of electron Output of AND gate
nh 2h h Y = Y.C
L Y = (A + B).C
2 2
If C = 0 irrespective of A and B, then
5. (b) As, we know de-Broglic wavelength, output Y must be zero.
h 11. (c) Due to charge at A and B magnitude of
p intensity of electric field at point C

1 1 q
E1 E2 .
p 4 0 r2
www.jeeneetbooks.in

WWW.IIT-NEET.XYZ

Solved Paper 2015 2015-13

The rod will become parallel to the field


ER T
E2 E1 in time .
60° 4
Cq T mL
t
60° 4 2 2qE
r r 13. (d) The energy stored in the capacitor
1 1 2
U CV 2 = C 200 = 2C×104 J
2 2
A 60° 60° B This energy is used to heat up the block.
q r q
Let be the rise in temperature, then
heat energy
Net intensity at point C is
Q = ms = 0.1 × 250 × 0.4 = 10J
ER E12 E 22 2E1E 2 cos 60 Now,
2C × 104 = 10

1 3q 10 4
= E12 E12 2E12 3E1 = C 4
5 10
= 500 F
2 2 10
2 4 0r
14. (b) Capacitance of air capacitor
12. (b) Torque when the wire is brought in a
uniform field E. 0A
C0 3 F ....(i)
= qEL sin d
= qEL [ is very small] When a dielectric of permittivity r and
Moment of inertia of rod AB about O dielectric constant K is introduced
between the plates, then
2 2
L L mL2 K 0A
I m m
2 2 2 Capacitance, C 15 F ....(ii)
d
+q F+q Dividing eq. (ii) by (i), we get
B C d 15
O C0 0A 3
Eq d
–q A K =5
=I . permittivity of the medium
r= 0K
qEL = 8.85 × 10–12 × 5 = 0.44 × 10–10
I mL2 1
2 15. (d) using, R
A

2 2qEL l1 l2 l3
2 [ = 2 ] R1 : R2 : R3 = A : A : A
mL 1 2 3

2qE
2 l12 l 22 l32
= : :
mL V1 V2 V3
Time period of the wire
l12 l 22 l32
2 mL = : :
T 2 m1d m2d m 3d
2qE
EBD_7443
www.jeeneetbooks.in

WWW.IIT-NEET.XYZ

2015-14 Target VITEEE


r1 + r2 + R = 2r2
l12 l22 l32 R = r2 – r1
= : :
m1 m 2 m3 19. (a) Here, the magnetic force (Bqv) will
provide the necessary centripetal force
52 32 22
= : : = 125 : 30 : 8 mv2
2 3 5
16. (b) Resistance of lamp r
2
V2 30 mv2
R0 10 Bqv
P 90 r
Current in the lamp Bqr = mv
V 30 For electron and proton, the magnetic field
I 3A B, charge q and radius r, all same.
R 0 10 mv = constant
As the lamp is operated on 120V DC, then i.e. meve = mpvp
resistance becomes
me 9 10 31
V 120 vP ve 3 106
R 40 mp 1.8 10 27
i 3
For proper glow, a resistance R is joined = 1.5 × 103
m/s
in series with the bulb 20. (a) Here two forces acting on the rod
R = R + R0 simultaneously.
R = R – R0 = 40 – 10 = 30
17. (d) Let us Consider a cell of emf E and iB cos 60°
N
balancing length l1
E = kl1
potential difference is balanced by length l2.
V = kl2 60°
Internal resistance of the cell mg sin 60°
60° mg mg cos 60°
E V E l1
r R 1 R 1 R
V V l2 From FBD, mg sin 60 = Bil cos 60°
mg
560 56 B tan 60
= 1 10 1 10 il
560 60 50
0.01 10
6 = 3 1T
= 1.2 173 0.1
5 21. (c) Due to current through side AB Magnetic
18. (d) Let the emf of each source be E. When field at the centre O
they are connected in series, the current
in the circuit 0I
B1 sin 1 sin 2
4 a
E tot E E 2E
I =
R tot r1 r2 R r1 r2 R C
potential drop across the cell of internal
2E I I
resistance r2 , r r R r2 O
1 2
1
2E 2
Hence, E r2 0 a
r1 r2 R A D I B
www.jeeneetbooks.in

WWW.IIT-NEET.XYZ

Solved Paper 2015 2015-15

As the magnetic field due to each of the 24. For a dipole at position (R, Q)
three sides is the same in magnitude and
direction. 0 2M cos
BR . ....(i)
Total magnetic field at O is sum of all the 4 R3
fields.
0 M sin
3 0I and BQ . ....(ii)
i.e. B = 3B1
4 a
sin 1 sin 2 4 R3
BV BR
Also tan ....(iii)
AD 2 BH BQ
Here, tan 1 tan 60
OD a
BQ
BR
2
9 10
a
2 3 2 3
R
Now B S
7
4 10 2 N
= 3 2
sin 60 sin 60
9 10
4
2 3
Dividing eq. (i) by (ii)
4 3 3 3
= 10 5 BR 2 cos
9 2 2 2 cot ....(iv)
BQ sin
= 1.33 × 10–5
T
From eq. (iii) and (iv)
22. (b) The direction of dB is the direction of
tan = –2cot
vector dl × r. From right hand screw rule, From figure, = 90° +
if we place a right handed screw at the tan = – 2 cot (90 + )
point where the magnetic field is needed tan = 2 tan
to be determined and turn its handle from 25. (b) The hysteresis loop i.e. area of B-H curve
dl to r, then the direction in which the is a measure of energy dissipated per cycle
screw advances gives the direction of field per unit volume of the specimen. It
dB. depends on the nature of magnetic
23. (b) Given: current sensitivity = 10 div/mA and material.
there are 100 division on the scale. 26. (d) Work done by magnet to turn from angle
Current required for full scale deflection. 1 to 2
W = MB(cos 1 – cos 2)
1
Ig 100mA 10mA = 0.01 A = MB (cos0° – cos 45°)
10
Also voltage sensitivity = 2 div/mV 1 2 1
= MB 1 MB
voltage required for full scale deflection 2 2
1 Also torque acting on the magnet
Vg 100mV 0.05V
2 MB
= MB sin 45° =
Galvanometer resistance is given by 2
Vg 0.05 W
G 5 W 2 1.
Ig 0.01 2 1
EBD_7443
www.jeeneetbooks.in

WWW.IIT-NEET.XYZ

2015-16 Target VITEEE


27. (c) From Lentz’s law, the direction of induced
1
emf in a circuit is such that it opposes the XL = XC L
magnetic flux that produces it. C
So, if the magnetic flux linked with a
closed circuit increases the induced 1 1
L 2 = 2 6
current flows in a direction so as to C 100 10 10
develop a magnetic flux in the opposite
direction of original flux. 1
If the magnetic flux linked with a closed L= H = 0.1 H = 100 mH
10
circuit decreases then the induced current
31. (c) When a light ray falls on a mirror at an
flows in the same direction of original
flux. So, the induced emf has not direction angle 30°, then the reflected ray will make
of its own. the same angle with the plane as shown
28. (a) Given : N = 20 in Fig. (i)
B = 103 gauss
= 103 × 10–4 T = 0.1T Normal Reflected ray
A = 5 cm2 = 5 × 10–4 m2 Incident ray
= 80° 30°
30°
Flux through the coil 30°
= NBA cos
= 20 × 0.1 × 5 × 10–4 × cos 30°
30° 30° 30°
3 (i) (ii)
= 10 10 4 = 5 3 10 4
865 10 4 wb
2
In order to make the reflected ray vertical,
1 the mirror should be rotated at an angle
29. (c) In LC circuit, if XL = XC then of 60°.
LC
So, th e mirror should be tilted by
E0
I0 , so Z 0. 60
30 Fig. (ii)
I0
2
1 32. (d) For a compound microscope, magnifying
As is the natural power
LC
MP = me × m0
frequency of LC circuit, therefore for an When the final image is at least distance
LC circuit if the frequency of applied AC of distance vision then
becomes equal to the natural frequency of
an AC circuit then the amplitude of D
Me 1
current becomes infinite due to zero fe
impedance.
30. (d) Maximum current flows in the circuit in D
MP m0 1
resonance condition Current in the LCR fe
circuit
25
V 35 m0 1
i 10
2
R2 XL XC –35 = m0 × 35
For current to be maximum denominator m0 = –10
should be minimum The negative sign shows that the image
(XL – XC)2 = 0 formed by objective is inverted.
www.jeeneetbooks.in

WWW.IIT-NEET.XYZ

Solved Paper 2015 2015-17

33. (d) Using prism formula, 36. (d) Let the intensity of unpolarised light be
I0
A m I0, so the intensity of first polaroid is
.
sin 2
2
....(i) On rotating through 60°, the intensity of
A
sin light from second polaroid
2
I0 2 I0 1 I0
where, A = angle of prism I cos 60 = 0.125I0
2 2 4 8
m = angle of minimum deviation
percentage of incident light transmitted
A through the system = 12.5%.
cos 37. (b) As the electromagnetic wave is the crossed
A 2
Given, cot field of electric and magnetic waves, so
2 A
sin the direction of propagation of EM wave
2
is the direction of vector E × B. Here E is
So, from Eq. (i) upward and (E × B) is towards north.
So, from right hand thumb rule B will be
A A m along east.
cos sin 38. (c) An electromagnetic wave is the wave
2 2
A A radiated by an accelerated charge and
sin sin propagates through space as coupled
2 2
electric and magnetic field. These fields
are oscillating perpendicular to each other.
A A m 39. (d) From Rayleigh’s law of scattering,
sin sin
2 2 2 2 intensity

m= – 2A = 180° – 2A 1
I
34. (d) Given: 4
P1 = 2D; P2 = 3D
4
1 I1 2
d m
3 I2 1

1 1 1 d 4 4
I1 330 3 81
We know that
F f1 f2 f1.f2 I2 880 8 4096
Equivatent power,
4096
P = P1 + P2 – dP1. P2 I2 Å 50.557 Å
81
1 40. (c) As we knows, radius of an atom,
=2+3– × 2 × 3 = 3D
3 –10 m
A 10
35. (d) First ray optical path = 1 L1 radius of nucleus,. B 10–15 m
second ray optical path = 2 L2 So, ratio of their volumes
So, phase difference 4 3
rA 3 3
2 2 VA 3 rA 10 10
× path difference = x
VN 4 3 =
rN IN 10 15
3
2
1L1 2 L2 VA : VN = 1015 : 1
EBD_7443
www.jeeneetbooks.in

WWW.IIT-NEET.XYZ

2015-18 Target VITEEE


0
PART - II (CHEMISTRY) 264.4 = [2 G 0f (H+) + 2 G f (Cl–)]
41. (b) Balanced equations are
or 264.4 = – G 0f H 2 G 0f Cl 2
3Cu 8HNO 3 3Cu NO3 2
2NO 4H 2O
0
...(i) = 0 2 G f Cl 0 0

Cu 4HNO3 Cu NO3 2NO 2 2H 2O


2 or, 262.4 2 G 0f Cl
...(ii)
Here NO and NO2 are evolved in equal G 0f Cl
or, = –131. 2kJmol–1.
volumes, hence, on adding Eqs. (i) and
(ii) 47. (a) 2L of 3M AgNO3 will contains 6 moles of
4Cu + 12 HNO3 4Cu(NO3)2 AgNO3.
3L of 1 M BaCl2 will contain 3 moles of
+ 2NO + 2NO2 + 6H2O BaCl2.
or 2Cu + 6HNO3
2AgNO3 + BaCl2 2AgCl
2Cu(NO3)2 +NO + NO2+3H2O + Ba(NO3)2.
Hence, coefficients x and y of Cu and So, 6 moles of AgNO3 will react with 3
HNO3 are 2 and 6 respectively. moles on BaCl2 it means, two solution will
42. (c) [S2–] = 10–23 mol L–1. react completely to form 3 moles of
[M2+] = 10–2M Ba(NO3)2 6 moles of NO3 ions in 2 +
Ionic product, K1P = [M2+] [S2–] = 10–25,
3 = 5L solution
ionic product is greater than Ksp of CuS
and CdS. 6
Hence, molarity of NO3 = 1.2 M
Therefore, all except MnS and FeS are 5
precipitated.
48. (b) AsO33 , CIO33 , an d SO32 have sp 2
CO g H2 g
43. (c) K hybridisation and hence are non-planar
H2 O g
species, while NO3 , CO32 and BO33
Concentration will increase, on halving have sp2 hybridisation and hence are
the volume. There are two terms in planar species
numerator. So to keep K constant, 49. (d)
concentration of [H2O] should increase
much more. H3O+ CH3
44. (d) During smelting process of copper from CH2
Markownikoff's OH
copper pyrites reactions are addition (A)
(i) B2H6/THF Anti-Markownikoff's
Cu2O+FeS Cu2S + FeO (ii) H2O2/OH
addition

2FeS + 3O2 2FeO + 2SO2


CH2OH
45. (d) SnCl2 + HgCl2 SnCl4 + Hg
(B)
(xCl2) (yCl2) (xCl2) (y)
50. (b) (KE)1 = hv1 – hv0
1 (KE)2 = hv2 – hv0
HgO Hg O2
400 C 2 As, (KE)1 = 2 × (KE)2
y
hv1 – hv0 = 2(hv2 – hv0)
So ore of is HgS i.e., Cinnabar.
or, hv0 = 2hv2 – hv1
46. (a) ( G°)reaction = G 0f (Products) or, v0 = 2v2 – v1
= 2× (2 × 1016) – (3.2 × 1016)
– G 0f (reactants) = 0.8 × 1016 Hz = 8 × 1015 Hz
www.jeeneetbooks.in

WWW.IIT-NEET.XYZ

Solved Paper 2015 2015-19

51. (d) Let initially, pressure of C6H6 (g) is p1 H2 NHgO.Hgl + 7KI + H2O
mm and for H2(g) is p2 mm brown ppt.
p1 + p2 = 60 mm ....(i) iodine of Million’s base
After the reaction pressure of
C6H6(g) = 0 (as all C6H6 has reacted) HCl MNO3 MCl HNO3
H2(g) = p2 – 3p1 C white ppt.
So, total pressure = p2 – 3p1 + p1 = 30
(M = Ag+, Pb+, Hg+)
mm
p2 – 2p1 = 30 mm ....(ii)
On solving equation (i) and (ii) 7 5 3
p1 = 10 mm, p2 = 50 mm 6 4 2 1
8
Fraction of C 6H6 by volume = moles 56. (b)
10 1 9
fraction fraction of pressure = 10
60 6
52. (c) In the unit cell number of Cu atoms The correct IUPAC name is 7-ethyl- 2, 4, 5,
(fcc/ccp) 6-tetramethyldeca-1, 8-diene
57. (a) M.wt. of caffeine = 194 u
1 1 % of N present in one molecular of
= 8 6 4
6 2 caffeine is 28.9% of
As Ag atoms occupying edge centred 28.9
194u 194 56 u
1 100
= 12 × 3
4 Mass of one N atom = 14 m
and Au atoms are presents at the body = 1N atom ( 14 m = 14 u)
centred = 1
56
formula, Cu4Ag3Au. 56u N atom 4N atom
53. (b) As we know that –nFE°cell = –RT In k or 14
RT 58. (b) CaCO3 CaO CO 2
E cell ln k.
nF X Colourless
Plot of ln k or E°cell will have slope gas
1 RT
=
2 F CaO H 2O Ca OH 2
54. (d) Since 2° propyl carbocation is little more Re sidue Y
stable than allyl carbocation and ethyl
carbanion is more stable than isopropyl Ca OH 2 2CO2 Ca HCO3
carbanion. 2
Y Excess Z
55. (a) NH 4 Cl NH3 HCl
A B C Ca HCO 3 2 CaCO3 CO 2 H 2O
colourless Z x
equimolar
quantities
59. (b) Zn(OH) 2 2O H ZnO 22 2H 2O
Acid Base Salt Water
NH3 HCl NH 4 Cl
B dense white Zn OH 2
2H Zn 2 2H 2O
Acid Salt Water
fumes Base
The amphoteric character of Zn(OH)2 is
NH3 2K 2 HgI 4 3KOH represented by I and III
Nessler 's reagent
EBD_7443
www.jeeneetbooks.in

WWW.IIT-NEET.XYZ

2015-20 Target VITEEE


64. (b) Resonance stabilisation of enol form is
NO2 NHOH
O – H .....O
Zn dust CH3 – C = CH – C – CH3
60. (a)
NH4CI enol form
A O..........H – O
HCI CH3 – C – CH = C – CH3
keto form
HO NH2 65. (d) The minimum m. wt. must contain at least
B one S atom.
61. (b) Increasing order of stability weight of one S atom
% S 100
(a) [Cu(NH3)4]2+ < [Cu(en)2]2+ minimum m.wt.
< [Cu (trein)2+.
32
Their formation of entropy increases in 4 100
the same order. Ligand denticity is minimum m.wt.
increased. 32
(b) [Fe(H2O)6]3+ < [Fe(NO2)6]3– minimum m. wt. = 100 800
< [Fe(NH3)6]3+. 4
NH3 is weaker ligand than NO2–. 66. (b) When the concentration of the adsorbate
is less on the surface as compare to its
The correct stability order is
concentration in the bulk is called negative
[Fe(H2O)3+ < [Fe(NH3)6]3+ < [Fe(NO2)6]3–
adsorption. Add from left in this
(c) [Co(H2O)6]3+ < [Rh(H2O)6]3+ adsorption, concentration of dilute KCl
< [Ir(H2O)6]3+ solution is less on the surface of blood
Zeff value increases from Co3+ to Ir 3+. charcoal as compare to its concentration
(d) [Cr(NH3)6]1+ < [Cr(NH3)6]2+ in solution.
< [Cr(NH3)6]3+ 67. (a)
O.S of Cr atom increases from +1 to +3.
62. (d) The amount of energy required to take out Interval Conc.change Rate
an electron from the monopositive cation 0.017
0 17 min 0.069 0.052 0.017 M 0.001
is called second ionisation energy 17
0.017
M(g) M2+(g) + 2e– (v) 17 34 min 0.052 – 0.035 0.017M 0.001
17
M(g) M+ (g) + e– (iii) 0.017
34 51 min 0.035 0.018 0.017 M 0.001
17
On subtracting eq(iii) form eq. (v)
we get, Rate remains constant. So, it is
independent of concentration, the reaction
M M2+ + e–. is of zero order.
According to rate law
sp2 Rate = K(conc.)0 = 0.001M/min
68. (d) hv = hv0 + ev0.
63. (d) h h
v0 v v0
2 e e
sp
On comparing this equation with the
In benzene, the triple bond consists of one straight line equation, i.e y = mx + c
sp2–sp2 -bond, one sp2–sp2, -bond and The slope of v0 vs v is
one p–p -bond. (v0 is stopping potential)
h
(slope)1
e
www.jeeneetbooks.in

WWW.IIT-NEET.XYZ

Solved Paper 2015 2015-21

Likewise, 74. (b) A compound which gives a negative test


hv = hv0 + Kmax. with ninhydrin, it cannot be a protein or
or Kmax = hv–hv0 an amino acid. As, it gives a positive test
Thus, slope of Kmax vs v is with Benedict’s solution. So, it must be a
monosaccharide but not a lipid.
slope 2 h
slope 2
h e CN Polymerisation
slope 1
h/e
75. (c) nCH2 = C
69. (c) Both P2 O5 and PCl5 give H3 PO4 With COOCH3
excess of water. Methyl- -cyanoacrylate
P2O5 + 3H2O 2H3PO4
PCl5 + 4H2O H3PO4 + 5HCl CH2
70. (c) Al(OH)3 dissolves in NaOH solution to
give Al OH ion which is supposed to CH2 C
4
have the octahedral complex species
[Al(OH)4(H2O)2]– in aqueous solution. COOCH3 n
Poly (methyl -cyanoacrylate)
Al OH 3
NaOH aq [Al(OH)4 Super glue of crazy glue
(H2O)2 – (aq) + Na+(aq) 76. (a) Nitration and bromination of salicylic
71. (b) In the absence of d-orbitals F2 does not acid, give picric acid (X) and 2, 4, 6–
combine with F– to form F3–. ion. tribromophenol (Y) respectively.
72. (a) According to Mulliken, electronegativity 1. Decarboxylation
of an atom is average of ionization energy Bromination
and electron affinity (in eV).
IE EA OH OH
nm COOH O2N NO2
2 HNO3
If ionization energy and electron affinity
Salicylic acid
are in kcalmol–1. NO2
IE EA 275 86 Br2/Water Picric acid
n 288 (X)
125 125 OH
73. (a) For the preparation of paracetamol Br Br

OH OH Br
2, 4, 6- tribromophenol(Y)
HNO 3
77. (b) Hydride ion transfer to the carbonyl group
Phenol is the slowest or the rate determining step.
NO2 –
(Major isomer) O O
O
H2/Pd –
OH Ph – C – H
OH Ph – C – H Ph – C – H
OH slow
(CH3CO)2 O OH

O O O
NHCOCH2
Paracetamol NH2
Ph – C – OH Ph – C – OH + Ph – C – H
+
Ph – CH2OH H
EBD_7443
www.jeeneetbooks.in
=

WWW.IIT-NEET.XYZ

2015-22 Target VITEEE


78. (d) This is Wurtz reaction. Bromides have
high reactivity than chlorides in Wurtz PART - III (MATHEMATICS)
reaction therefore, reaction occurs from 81. (d) The given differential equation is
Br atoms, (3x + 4y + 1) dx + (4x + 5y + 1) dy = 0
....(i)
–Br – 2Na Comparing eq. (i) with Mdx + Ndy = 0,
Cl Br Cl + we get
M = 3x + 4y + 1
1-bromo-3-chloro cyclobutane and N = 4x + 5y + 1
M N
+ Here, 4
Cl – Na + NaCl y x
Hence, eq. (i) is exact and solution is given
Bicyclo (1, 1, 0) by
butane
79. (d) 3x 4y 1 dx 5y 1 dy C

CH 3CH 2 CH 2 OH
Conc H 2SO 4
CH 3 CH CH 2 3x 2 5y2
160 180 C 4xy x y C 0
2 2
3x2 + 8xy + 2x + 5y2 + 2y – 2C = 0
Br 2 alc. KOH
CH3CHBr CH 2 Br 3x2 + 2.4xy + 2x + 5y2 + 2y + C = 0 ...(ii)
(Y) where, C = –2C
[CH3C(Br) = CH2 + CH3CH = CHBr] On comparing eq. (ii) with standard form
(A) (B) of conic section
ax2 + 2hxy + by2 + 2gx + 2fy + C = 0
NaNH 2
CH 3C CH We get,
HBr
(Z)
a = 3, h = 4, b = 5
Alcoh olic KOH brings about Here, h 2 – ab = 16 – 15 = 1 > 0
dehydrobromination of Y and give a Hence, the solution of differential equation
mixture of vinyl bromide (A and B) while represents family of hyperbolas.
NaNH2 being a strong base than alc. r r3
KOH readily brings about 82. (b) r
dehydrobromination of less reactive vinyl 1 n n 1
bromide to give propyne CH3 C CH i.e.
n n
(Z).
80. (d) 3-methylbutanoic acid gives isobutane on
n r r3
r r 1 r 1
decarboxylation i.e., r 1
1 n n 1
NaOH/CaO
CH3 CH CH 2COOH
|
CH3 n(n 1) [n(n 1)]2
3-methylbutanoic acid = 2 2
1 n(n 1)
CH3 CH CH3 + CO2
2 2
n n 1 n n 1
CH3 =
Isobutane 2 4
While Wurtz reaction of C2H5Br gives. 2
n n 1 n
n-butane an d h ydrolysis of n-butyl
= r3
magnesium bromide gives n-butane but 2 r 1
reduction of propanol with HI/P gives
propane.
www.jeeneetbooks.in

WWW.IIT-NEET.XYZ

Solved Paper 2015 2015-23

n(log 35 – log 36) < log 0.01


B C
83. (b) P A P P B n[15441 – 15563] < –2
A A – 0.0122n < – 2
0.0122n > 2 n > 163.9
C So, the least value of n is 164.
= P A B P B
A 86. (d) The given system of equations will be
consistent with unique solution, when
P A B P C A B
= 1 1 1
P A B
1 2 3 0
= P A B C 1 4 k
1 3 1 1 3 1 1(2k – 12) + 1 (3 – k) + 1(4 – 2) 0
k – 12 + 3 + 2 0 k–7 0 k 7
84. (d) A 2 1 1 = 0 5 3
1
3 0 1 0 9 2 87. (d) Given: z
4
[Applying R2 R2 – 2R1, R3 R3 – 3R1] Let z = – 1 + 8z
z 1 |z 1|
z |z|
1 3 1 8 8
0 5 3 9 1 z 1
R3 R3 R2 z 1 2
17 5 4 8
0 0
5 z lies on a circle with centre (–1, 0) and
radius 2.
rank (A) = 3 88. (c) If the line y = mx + c is a normal to the
85. (b) The probability of getting a double six in
one throw of two dice x2 y2
ellipse 1, then
1 1 1 a2 b2
=
6 6 36 m 2 (a 2 b2 )2
c2
1 a 2 b2 m 2
p ,
36 [Here, m = 2, a2 = 9 and b2 = 16]
q 1 p 2
2
9 16
2

= 2
1 35 9 16 2
=1
36 36
4 49 4 49 196
Now, (p + q)m =
= qn + nC1qn–1p + nC2qn – 2 p2 + .... 9 64 73 73
+ nCrqn–r pr +....+ pn 14
The probability of getting atleast one c
73
double six in n throws with two dice.
= (q + p)n – qn 89. (b) Given equation is x2 + x + 1 = 0
x = and x 2
n Case I : When x =
n 35
=1 q 1 Then
36
6 2 6
1 2n 2 1
35
n xn n 2
1 0.99 n
n 1 x n 1
36
=( + 2 )2 + ( 2 + 4 )2 + ( 3 + 6 )2
n +( 4 + 8)2 + ( 5 + 10)2 +( 6+ 12)2
35
0.01
36
EBD_7443
www.jeeneetbooks.in

WWW.IIT-NEET.XYZ

2015-24 Target VITEEE


= (–1)2 + (–1)2 + (2)2 + (–1)2
1 5
+ (–1)2 + (2)2 = 12 So, when m ,c
Case II: When x = 2 2 6
Then
7
6 2 6 When m = 1, c = –
1 n 2 1 6
xn 2n

n 1 xn n 1
2
3
= 12 When m = 3, c = –
2
90. (b) Correct result is as follows:
(~ p ~ q) (r s)
1 5
or ~(p q) r s Asymptotes are y = x ,
2 6

1 3 1 7 3
y= x– and y = 3x – .
91. (a) A 1 2 3 6 2
0 1 2 93. (b) The structure (N,.) satisfies the closure
|A| = 1.(4 + 3) – 3 (–2 + 0) + 1(–1 –0) property, associativity and commutativity
= 7 + 6 – 1 = 12 but the identity element 0 does not belong
So, adj (adj A) = |A|n – 2 = A to N.
= (12)3 – 2 A = 12A
So, N is a semi-group.
1 3 1 dx
94. (a)
= 12 1 2 3 cos x 3 sin x
0 1 2 1 dx
=
2 1 3
cos x sin x
12 36 12 2 2
= 12 24 36 1 dx
0 12 24 =
2 cos cos x sin sin x
3 3
92. (c) 3(m) = 3 + 2m – + 7m2 2m3
(m) = –14m + 7 m 2 1 dx
2 =
2 2
3(m) = 2 – 14m + 6m cos x
Now, putting 3(m) = 0, we have 3
3 + 2m – 7m2 + 2m3 = 0
1
(1 – m) (1 + 2m) (3 – m) = 0 = sec x dx
2 3
1
m ,1,3 1 x
2 = log tan C
2 2 6 4
We know that
c n (m) + n–1(m) = 0, which in the given 1 x
= log tan C
case becomes 2 2 12
c(2 – 14m + 6m2) + (–14m + 7m2) = 0 95. (d) Given sphere is
x2 + y2 + z2 – 6x – 12y – 2z + 20 = 0
14m 7m 2 Centre (3, 6, 1)
c Here, one end of diameter is (2, 7, 3).
2 14m 6m 2
Let the other end of the diameter be (x, y, z)
www.jeeneetbooks.in

WWW.IIT-NEET.XYZ

Solved Paper 2015 2015-25

Centre of the sphere will be the mid-point n1. n 2


of the ends of diameter. cos
n1 n 2
2 x 7 y 3 z
So 3, 6,1 , , 3 3 3 3 3 3
2 2 2
=
2+x=6 x=4 9 9 9 9 9 9
7 + y = 12 y=5
and 3 + z = 2 z = –1 9 9 9 1
=
Therefore, (x, y, z) (4, 5, –1) 27 27 3
96. (d) Given lines are
1 1
x = my + n, z = py + q cos
and x = m y + n , z = p y + q 3
Above equations can be rewritten as 98. Let , and be the angles made by the line
segment OP with X-axis, Y-axis and Z-axis,
x n y 0 z q
respectively.
m 1 p
Given: and
x n y 0 z q 4 3
and
m 1 p We know that, cos2 + cos2 + cos2 = 1
Lines will be perpendicular, if
cos 2 cos 2 cos 2 1
mm + 1 + pp = 0 4 3
mm + pp = –1
97. (c) 2 2
1 1
cos2 1
Y 2 2
B
1 1
cos 2 1
2 4
O
A X c os2
1
4
C 1
Z c os
2
Vector perpendicular to face OAB = n1
= OA OB
4
= ˆi 2ˆj kˆ 2iˆ ˆj kˆ Hence, direction cosines are cos , cos , cos
1 1 1
= 2 1 iˆ 2 1 ˆj 1 4 kˆ i.e. , , .
2 2 2
= 3iˆ 3jˆ 3kˆ 99. (a) ~ p q means F F = F, ~ r means F
[(~ p q) ~ r] p means T
Vector perpendicular to face ABC = n 2 .
100. (b) Let f(x) = x25(1 – x)75, x [0, 1]
= AB AC f (x) = 25x24 (1 – x)75 – 75x25 (1 – x)74
= (–3iˆ 3 ˆj ) ( ˆj kˆ) = 25x24 (1 – x)74 {(1 – x) – 3x}
= 25 x24 (1 – x)74 (1 – 4x)
= 3iˆ 3jˆ 3kˆ + + – –
Since, angle between faces is equal to 0 1/4 1
angle between their normals.
EBD_7443
www.jeeneetbooks.in

WWW.IIT-NEET.XYZ

2015-26 Target VITEEE


104. (a)
1
We can see that f (x) is positive for x <
4
1
and f (x) is negative for x > .
4
O /2 3 /2
1
Hence, f (x) attains maximum at x = .
4
1 Required area
101. (b) z
4 /4 5 /4
= cos x sin x dx sin x cos x dx
0 /4
1 1
= z– |z| 3 /2
4 4 cos x sin x dx
5 /4

1 1 11 = sin x cos x
/4
cos x sin x
5 /4
= (–z) 3 0 /4
4 4 4
3 /4
+ sin x cos x 5x / 4
1 11
z
4 4 = 4 2 2 sq units
102. (b) Equation of the normal at point 105. (b) [a + b – c] . [(a – b) × (b – c)
at12 , 2at1 = (a + b – c). [a × b – a × c – b × b + b × c]
on parabola is
= a. (a × b) – a. (a × c) + a. (b × c) + b.
(a × b) – b (a × c) + b. (b × c) – c. (a × b)
y –t1x 2at1 at13
+ c. (a × c) – c.(b × c)
2 = a. (b × c) – b. (a × c) – c. (a × b)
It also passes through at 2 , 2at 2 = [a b c] – [b a c] – [c a b]
=[a b c] + [a b c] – [a b c]
So, 2at 2 t1 at 22 2at1 at13 = [a b c] = a. (b × c)
106. (a) Surface area A of a cube of side x is given
2t 2 2t1 t1 t 22 t12 by A = 6x2.
On differentiating w.r.t. x, we get
2
t1 t 2 dA
t1 12x
dx
2 Let the change in x be x = m% of x
t2 –t1
t1 mx
=
a1b1 a2 b2 a3b3 100
103. (c) cos Change in surface area,
a12 a22 a32 b12 b22 b32
dA
A x 12x x
1 2 1 1 2 1 dx
=
1 1 4 4 1 1
mx 12x 2 m
2 2 2 6 12x
= 1 100 100
6 6 The approximate change in surface area
So, = 0° or = 2
sec 2 = 1 2m
= 6x 2
2 = sec–1(1) 100
= sec–1 (1) = 2m% of original surface area
www.jeeneetbooks.in

WWW.IIT-NEET.XYZ

Solved Paper 2015 2015-27

107. (d) Given equation of rectangular hyperbola


1
is x2 – y2 = 82 = hlim0 = –1
2
Length of latusrectum = 2 × (8) = 16 cos h 1 tan h
and eccentricity = 2 110. (c)
The asymptotes are perpendicular lines. B(0,3)
So, x y 0
Now, directrices are
A(3,0)
8
x = 4 2 O
2 x+y=3
108. (a) Equation of hyperbola is
3x2 – 2y2 = 6 x2+y2=9
x2 y2 Area of required region
1
2 3 1
So, a2 = 2 and b2 = 3 = × Area of circle – Area of OAB
4
Given, equation of line is x – 3y = 3.
1 1
1 = × × (3)2 – ×3×3
Slope of given line = 4 2
3
Slope of line perpendicular to given line, 1
m = –3 = 9
4 2
The equation of tangents are 111. (c) [a + b, b + c, c + a]
= (a + b) . [(b + c) × (c + a)]
y mx a 2 m2 b2
= (a + b) . [b × c + b × a + c × c + c × a]
= 3x 2 9 3 = (a + b). (b × c + b × a + c × a)
[ c × c = 0]
= 3x 18 3 = a. (b × c) + a. (b × a) + a.(c × a) + b.
= 3x 15 (b × c) + b. (b × a) + b . (c × a)
= a.(b × c) + b.(c × a)
= [a b c] + [a b c]
tan h 1
4 = [a b c] + [a b c] = 2 [a b c]
tan x 1 lim
109. (d) lim = h 0 /2
x / 4 cos2 x cos 2 h 112. (a) Let I log tan x . sin 2 xdx ...(i)
4 0

/2
I log tan x sin 2 x dx
x h 0 2 2
4
a a
1 tan h f x dx f a x dx
0 0
1
1 tan h
= lim /2
h 0 I log cot x. sin 2x dx ....(ii)
cos 2h 0
2 [ sin( – 2x) = sin 2x]
On adding eqs (i) and (ii), we get
1 tan h 1 tan h
= hlim0 sin 2h 1 tanh 2I
/2
log tan x.sin 2x dx
/2
log cot x sin 2x dx
0 0

2 tan h /2
= hlim0 2 sin h cos h 1 tan h = sin 2 x log (tan x.cot x)dx
0
[ log m + log n = log (m . n)]
EBD_7443
www.jeeneetbooks.in

WWW.IIT-NEET.XYZ

2015-28 Target VITEEE

=
/2
sin 2x log 1dx b1 3iˆ –16jˆ 7kˆ
0
I = 0[ log 1 = 0] and b2 3iˆ 8jˆ 5kˆ
/2
sin 2x log tan x dx 0 a 2 a1 . b1 b2
0
Shortest distance = b1 b2
113. (c) Here, mean = 4 and variance = 2
np = 4 and npq = 2
npq 2 1 3iˆ 34ˆj 2kˆ . 24iˆ 36ˆj 72kˆ
So, q
np 4 2 = 84
1 1
Then, p = 1 – q = 1 – 72 1224 144 1152 288
2 2 units
=
Mean = np = 4 84 84 21
1 115. (a) Equation of plane passing through
n× =4 n=8 (2, 2, 1) is
2
a(x – 2) + b(y – 2) + c(z – 1) = 0 ....(i)
P(X = r) = nCrpr qn–r Since, above plane is perpendicular to
8 3x + 2y + 4z + 1 = 0
8 1 1
= Cr p q and 2x + y + 3z + 2 = 0
2 2
3a + 2b + 4c = 0 ....(ii)
The required probability of atleast 7 and 2a + b + 3c = 0 ....(iii)
successes is [ for perpendicular, a1a2
P(X 7) = P(X = 7) + P(X = 8) + b1b2 + c1c2 = 0]
8 On multiplying eq. (iii) by 2, we get
8 8 1
= C7 C8 4a + 2b + 6c = 0 ....(iv)
2 On subtracting eq. (iv) from eq. (ii), we
8 get
8! 8! 1
= a
7!1! 8!0! 2 c
2
8
1 9 a
= 8 1 On putting c in eq. (iii), we get b
2 256 2
x 7 y 4 z 6 a
114. (b) Given, lines are =
3 16 7 2
x 10 y 30 4 z a a
and On putting b = and c = in eq. (i),
3 8 5 2 2
The vector form of given lines are a a
we get a(x – 2) – (y – 2) – (z – 1) = 0
r 7iˆ 4ˆj 6kˆ 3iˆ – 16ˆj 7kˆ 2 2
a
and r 10iˆ 30ˆj 4kˆ 3iˆ 8jˆ 5kˆ
2
[2(x – 2) – (y – 2) – (z – 1)] = 0
On comparing these equations with 2x – 4 – y + 2 – z + 1 = 0
r = a1 + b1 and r = a2 + µb2, we get 2x – y – z – 1 = 0
116. (c) Suppose, A : a male is selected
a 7iˆ 4jˆ 6kˆ
1 B: a smoker is selected
a 2 10iˆ 30jˆ 4kˆ Given:
www.jeeneetbooks.in

WWW.IIT-NEET.XYZ

Solved Paper 2015 2015-29

Now, we check the function is maximum


7 2 A 2
P A B ,P A B and P or minimum.
10 5 B 3
The probability of selecting a smoker.. 1 1
f t cos t sin t
t t2
P A B
P B 1 1 1 2
A and f t sin t 2 cos t 2 cos t 3 sin t
P t
B t t t

2 3 3
sin t 2 cos t 2 sin t
= =
5 2 5 t t2 t3
The probability of selecting a non-smoker For maximum or minimum value of f(x),
So, P(B) = 1 – P(B) put
f (x) = 0
3 2
=1– = cos t sin t tan t
5 5 0 1
t 2 t
The probability of selecting a male t
P A P A B P A B P B Now tlim0 f t

7 2 3 sin t t cos t sin t


= lim – 2 lim
10 5 5 =
t 0 t t 0 t3
7 4 6 1
= 0
10 2 form
0
Probability of selecting a smoker, if a male
is first selected, is given by cos t t sin t cos t
= 1 2 lim
B P A B t 0 3t 2
P
A P A [using L’ Hospital rule]
2 sin t
2 2 4 = 1 lim
= 3t 0 t
5 1 5
2 1
sin t = 1 1 0
117. (d) Given: f(t) = 3 3
t
So, function f(t) is maximum at t = 0
At t = 0, we will check continuity of the
118. (d) Consider the function f defined by
function.
LHL = f(0 – h) xn 1 xn x2
f x a0 an ... a n 1 anx
sin 0 h sin h n 1 n 2
= hlim0 0 h = lim 1 Since, f(x) is a polynomial, so it is
h 0 h
continuous and differentiable for all x.
RHL = f(0 + h) f(x) is continuous in the closed interval
sin 0 h [0, 1] and differentiable in the open
lim interval (0, 1).
h 0 0 h Also, f(0) = 0
sin h and
= lim 1
a0 a1 a
h h 0
f 1 .... n 1 a n 0 [say]
and f(0) = 1 n 1 n 2
LHL = RHL = f(0) i.e. f(0) = f(1)
So, the function is continuous at t = 0
EBD_7443
www.jeeneetbooks.in

WWW.IIT-NEET.XYZ

2015-30 Target VITEEE


Thus, all the three conditions of Rolle’s 120. (c) We can write given differential equation
theorem are satisfied. Hence, there is as,
atleast one value of x in the open interval (D2 – 1) x = k ....(i)
(0, 1) where f (x) = 0 d
i.e. a0 xn + a1xn–1 + ....+an = 0 where, D
dy
x
119. (d) Let f(x) = log 1 x Its auxiliary equation is m2 – 1 = 0, so that
1 x m = 1, –1
1 1 x .1 x.1 Hence, CF = C1ey + C2e–y.
f x where C1, C2 are arbitrary constants
1 x 2
1 x
1
1 1 x Now, also PI 2
k
D 1
= 1 x 2 2
1 x 1 x 1
which is positive. [ x > 0] = k. 2 e0.y
D 1
f(x) is monotonic increasing, when x > 0.
f(x) > f(0) 1
K. e0.y K
Now, f(0) = log 1 – 0 = 0 2
0 –1
f(x) > 0 So, solution of eq. (i) is
x x = C1ey + C2e–y – k ....(ii)
log 1 x 0 Given that x = 0, when y = 0
1 x
So, 0 = C1 + C2 – k (From (ii))
x C1 + C2 = k ....(iii)
log 1 x ....(i)
1 x Multiplying both sides of eq. (ii) by e –y,
Also, for x > 0, we get
x2 > 0 x2 + x > x x. e–y = C1 + C2e–2y –ke–y ....(iv)
x(x + 1) > x Given that x m when y , m being
a finite quantity.
x
x> ....(ii) So, eq (iv) becomes
x 1 x × 0 = C1 + C2 × 0 – (k × 0)
From eqs. (i) and (ii), we get C1 = 0 ....(v)
x From eqs. (iv) and (v), we get
< log (1 + x) < x C1 = 0 and C2 = k
x 1
[ log (1 + x) < x for x > 0] Hence, eq. (ii) becomes
x = ke–y – k = k(e–y –1)
www.jeeneetbooks.in

WWW.IIT-NEET.XYZ

VITEEE
SOLVED PAPER 2014

PART - I (PHYSICS) 6. Beyond which frequency, the ionosphere bands


any incident electromagnetic radiation but do
1. The amplification factor of a triode is 50. If the not reflect it back towards the earth?
grid potential is decreased by 0.20 V. What (a) 50 MHz (b) 40 MHz
increase,in plate potential will keep the plate (c) 30 MHz (d) 20 MHz
current unchanged?
7. A metallic surface ejects electrons. When
(a) 5 V (b) 10 V
exposed to green light of intensity I but no
(c) 0.2 V (d) 50 V
photoelectrons are emitted,when exposed to
2. If the nuclear fission,piece of uranium of mass yellow light of intensity I.It is possible to eject
5.0 g is lost, the energy obtained in kWh is electron from the same surface by
(a) 1.25 × 107 (b) 2.25 × 107 (a) yellow light of same intensity which is more
(c) 3.25 × 10 7 (d) 0.25 × 107 than I
(b) green light of any intensity
3. Current in the circuit will be
(c) red light of any intensity
20
(d) None of the above
8. An electron moves at right angle to a magnetic
30
field of 5 × 10–2 T with a speed of 6 × 107 m/s. If
the specific charge of the electron is 1.7 × 1011C/kg.
The radius of the circular path will be
20 5 (a) 2.9 cm (b) 3.9 cm
(c) 2.35 cm (d) 2 cm
5 5
(a) A (b) A 9. A solenoid 30 cm long is made by winding 2000
40 50 loops of wire on an iron rod whose cross-section
5 5 is 1.5 cm2.If the relative permeability of the iron
(c) A (d) A is 6000. What is the self-inductance of the
10 20
solenoid?
4. An installation consisting of an electric motor (a) 1.5 H (b) 2.5 H
driving a water pump left 75 L of water per second (c) 3.5 H (d) 0.5 H
to a height of 4.7 m.If the motor consumes a
power of 5 kW, then the efficiency of the 10. A coil of resistance 10 and an inductance 5 H
installation is is connected to a 100 V battery. The energy
(a) 39% (b) 69% stored in the coil is
(c) 93% (d) 96% (a) 325 erg (b) 125 J
5. A potential difference across the terminals of a (c) 250 erg (d) 250 J
battery is 50 V when 11 A current is drawn and 11. A galvanometer has current range of 15 mA and
60 V,when 1 A current is drawn. The emf and the voltage range 750 mV. To convert this
internal resistance of the battery are galvanometer into an ammeter of range 25 A, the
required shunt is
(a) 62 V, 2 (b) 63 V, 1
(a) 0.8 (b) 0.93
(c) 61 V, 1 (d) 64 V, 2 (c) 0.03 (d) 2.0
EBD_7443
www.jeeneetbooks.in

WWW.IIT-NEET.XYZ

2014-2 Target VITEEE


12. The denial cell is balanced on 125 cm length of a 18. A proton and an -particle, accelerated through
potentiometer. Now, the cell is short circuited by the same potential difference, enter a region of
a resistance of 2 and the balance is obtained uniform magnetic field normally. If the radius of
at 100 cm. The internal resistance of the denial the proton orbit is 10 cm, then radius of
cell is -particle is
(a) 10 cm (b) 10 2 cm
4
(a) (b) 1.5 (c) 20 cm (d) 5 2 cm
3
(c) 1.25 (d) 0.5 19. An ammeter and a voltmeter of resistance R are
13. Four resistance of 10 , 60 , 100 and 200 connected in series to an electric cell of negligible
respectively taken in order are used to form a internal resistance. Their reading are A and V
Wheatstone’s bridge. A 15V battery is connected respectively. If another resistance R is connected
in parallel with the voltmeter, then
to the ends of a 200 resistance, the current
(a) both A and V will increase
through it will be (b) both A and V will decrease
(a) 7.5 × 10–5 A (b) 7.5 × 10–4 A (c) A will decrease and V will increase
(c) 7.5 × 10 A–3 (d) 7.5 × 10–2 A (d) A will increase and V will decrease
14. A circuit has a self-inductance of 1 H and carries 20. A neutron is moving with velocity u. It collides
a current of 2A. To prevent sparking, when the head on and elastically with an atom of mass
circuit is switched off, a capacitor which can number A. If the initial kinetic energy of the
withstand 400 V is used. The least capacitance neutron is E, then how much kinetic energy will
of capacitor connected across the switch must be retained by the neutron after reflection?
æ A ö÷
2
be equal to A
çç ÷ E E
èç A + 1ø÷
(a) (b)
(a) 50 µF (b) 25 µF ( A + 1)2
(c) 100 µF (d) 12.5 µF
15. The output Y of the logic circuit shown in figure æ A -1 ö÷
2
( A -1)
çç ÷ E E
èç A + 1ø÷
(c) (d)
is best represented as ( A + 1)2
A 21. If a magnet is suspended at angle 30° to the
B magnet meridian, the dip of needle makes angle
A of 45° with the horizontal, the real dip is
æ 3 ö÷
çç ÷
(a) A + B.C (b) A + B.C (a) tan èçç 2 ø÷÷
–1 (b) tan –1 ( 3)

(c) A + B.C (d) A + B.C æ 3 ö÷ æ 2 ö


ç
16. A resistor of 6 k with tolerance 10% and another (c) tan–1 ççç 2 ÷÷÷ (d) tan–1 çç ÷÷÷
è ø èç 3 ø÷
resistance of 4 k with tolerance 10% are
connected in series. The tolerance of the 22. Which has more luminous efficiency ?
(a) A 40 W bulb
combination is about (b) A 40W fluorescent tube
(a) 5 % (b) 10 % (c) Both have same
(c) 12 % (d) 15 % (d) Cannot say
17. If we add impurity to a metal those atoms also 23. The resistance of a germanium junction diode
deflect electrons. Therefore, whose V – I is shown in figure is (Vk = 0.3 V)
(a) the electrical and thermal conductivities
both increase I
(b) the electrical and thermal conductivities
both decrease 10 mA
(c) the electrical conductivity increases but
thermal conductivity decreases
v
(d) the electrical conductivity decrease but Vk 2.3V
thermal conductivity increases
www.jeeneetbooks.in

WWW.IIT-NEET.XYZ

Solved Paper 2014 2014-3

(a) 5 k (b) 0.2 k 6 36


æ 10 ö (a) (b)
(d) ççç ÷÷÷ k
5R 5R
(c) 2.3 k
è 2.3 ø
64
24. In hydrogen discharge tube, it is observed that (c) (d) None of these
7R
through a given cross-section 3.31 × 10 15
electrons are moving from right to left and 30. Silver has a work function of 4.7 eV. When
3.12 ×105 protons are moving from left to right. ultraviolet light of wavelength 100 nm is incident
The current in the discharge tube and its on it a potential of 7.7 V is required to stop the
direction will be photoelectrons from reaching the collector plate.
(a) 2 mA towards left How much potential will be required to stop
(b) 2 mA, towards right photoelectrons, when light of wavelength 200
(c) 1 mA, towards right nm is incident on it?
(d) 2 mA, towards left (a) 15.4 V (b) 2.35 V
25. In a semiconductor, separation between (c) 3.85 V (d) 1.5 V
conduction and valence band is of the order of 31. If the distance of 100 W lamp is increased from a
(a) 0 eV (b) 1 eV photocell, the saturation current i in the photocell
(c) 10 eV (d) 50 eV varies with the distance d as
26. If 1000 droplets each of potential 1V and radius (a) i µ d2 (b) i µ d
r are mixed to form a big drop. Then, the potential
1 1
of the drop as compared to small droplets, will (c) i µ (d) i µ
be d d2
(a) 1000 V (b) 800 V 32. Following process is known as
(c) 100 V (d) 20 V hv ¬¾
®e +e + –

27. A Zener diode, having breakdown voltage equal


to 15 V is used in a voltage regulator circuit (a) Pair production (b) photoelectric effect
shown in figure. The current through the diode (c) Compton effect (d) Zeeman effect
is 33. During charging a capacitor, variations of
potential V of the capacitor with time t is shown
as

20V 15V V
V
(a) (b)
t t

(a) 10 mA (b) 15 mA
(c) 20 mA (d) 5 mA
28. The activity of a radioactive sample is measured V V
(c) (d)
as N0 counts per minute at t = 0 and N0/C counts
per minute at t = 5 min. The time, (in minute) at t t
which the activity reduces to half its value, is 34. When a resistor of 11 is connected in series
with a electric cell. The current following in it is
2 5
(a) loge (b) 0.5 A. Instead when a resistor of 5 is connected
5 log e 2 to the same electric cell in series, the current
(c) 5 log102 (d) 5 loge 2 increases by 0.4A. The internal resistance of the
29. If the electron in the hydrogen atom jumps from cell is
third orbit to second orbit, the wavelength of (a) 1.5 (b) 2
the emitted radiation in term of Rydberg constant (c) 2.5 (d) 3.5
is
EBD_7443
www.jeeneetbooks.in

WWW.IIT-NEET.XYZ

2014-4 Target VITEEE


35. A battery is charged at a potential of 15 V in 8 h
PART - II (CHEMISTRY)
when the current flowing is 10A. The battery on
discharge supplies a current of 5A for 15 h. The 41. The sodium extract of an organic compound on
mean terminal voltage during discharge is 14V. acidification with acetic acid and addition of lead
The watt-hour efficiency of battery is acetate solution gives a black precipitate.
(a) 80% (b) 90% The organic compound contains
(c) 87.5% (d) 82.5% (a) nitrogen (b) halogen
36. A circular current carrying coil has a radius R. (c) sulphur (d) phosphorus
The distance from the centre of the coil on the 42. The volume strength of 1.5 N H2O2 solution is
1 (a) 16.8 L (b) 8.4 L
axis, where the magnetic induction will be th (c) 4.2 L (d) 5.2 L
8
to its value at the centre of the coil is 43. MnO4– + 8H + + 5e – ¾¾
® Mn 2+ + 4H 2 O;
R E° = 1.51 V
(a) (b) R 3
3 MnO 2 + 4H + + 2e – ¾¾
® Mn 2+ + 2H 2 O;
2 E° = 1.23 V E°MnO – | MnO is
(c) 2 3R (d) R 4 2

3 (a) 1.70 V (b) 0.91 V


37. The incorrect statement regarding the lines of (c) 1.37 V (d) 0.548 V
force of the magnetic field B is 44. A metal has bcc structure and the edge length of
(a) magnetic intensity is a measure of lines of its unit cell is 3.04Å. The volume of the unit cell
force passing through unit area held normal in cm3 will be
to it (a) 1.6 × 1021 cm3 (b) 2.81 × 10-23 cm3
(b) magnetic lines of force forms a close curve (c) 6.02 × 10-23 cm3 (d) 6.6 × 10-24 cm3
(c) inside a magnet, its magnetic lines of force
45. Among [Fe(H2O)6]3+ , [Fe(CN)6]3– , [Fe(Cl)6]3–
move from north pole of a magnetic towards
species, the hybridisation state of the Fe atom
its south pole
are, respectively.
(d) due to a magnetic lines of force never cut
each other (a) d2sp3, d2sp3, sp3d2 (b) sp3 d2 ,d2 sp3 ,d2 sp3
38. Two coils have a mutual inductance 0.55 H. The (c) sp3d2,d2sp3,sp3d2 (d) None of the above
current changes in the first coil according to 46. Which of the following hydrogen bonds are
equation I = I0 sin t. strongest in vapour phase?
where, I0 = 10A and = 100 rad/s. (a) HF ……….. HF
The maximum value of emf in the second coil is (b) HF ………... HCl
(a) 2 (b) 5 (c) HCl ………… HCl
(c) (d) 4 (d) HF …………… HI
39. An L-C-R circuit contains R = 50 , L = 1 mH 47. The rate constant for forward reaction and
and C = 0.1µF. The impedence of the circuit will backward reaction of hydrolysis of ester are
be minimum for a frequency of 1.1 × 10–2 and 1.5 × 10–3 per minute respectively.
Equilibrium constant for the reaction is
105 106
(a) Hz (b) Hz CH3COOC2H5 + H2O CH3COOH
2 2
(c) 2 x 105 Hz (d) 2 x 106 Hz + C2H5OH
40. An eye can detect 5 × 104 photons per square (a) 33.7 (b) 7.33
meter per sec of green light ( = 5000Å ) while (c) 5.33 (d) 33.3
the ear can detect 10–13W/m2. The factor by 48. 19.85 mL of 0.1 N NaOH reacts with 20 mL of HCl
which the eye is more sensitive as a power solution for complete neutralisation. The molarity
detector then ear is close to of HCl solution is
(a) 5 (b) 10 (a) 9.9 (b) 0.99
(c) 106 (d) 15 (c) 0.099 (d) 0.0099
www.jeeneetbooks.in

WWW.IIT-NEET.XYZ

Solved Paper 2014 2014-5

49. An f-shell containing 6 unpaired electrons can 57. In the reaction,


exchange 8Al + 3Fe3O4 ¾¾ ® 4Al2O3 + 9Fe
(a) 6 electrons (b) 9 electrons the number of electrons transferred from the
(c) 12 electrons (d) 15 electrons reductant to the oxidant is
50. The standard molar heat of formation of ethane, (a) 8 (b) 4
CO2 and water (l) are respectively –21.1, –94.1 (c) 16 (d) 24
and –68.3 kcal. The standard molar heat of 58. The bond angles of NH3, NH+4 and NH 2– are in
combustion of ethane will be the order
(a) –372 kcal (b) 162 kcal
(a) NH 2– > NH 3 > NH +4
(c) –240 kcal (d) 183.5 kcal
51. The solubility product of Ag2CrO4 is 32 × 10– (b) NH+4 > NH3 > NH 2–
12. What is the concentration of CrO– ions in
4 (c) NH3 > NH 2– > NH +4
that solution? (d) NH > NH +4 > NH –2
(a) 2 × 10-4 M (b) 16 × 10-4 M 59. A gaseous mixture containing He,CH4 and SO2
(c) 8 × 10 M-4 (d) 8 × 10-8 M was allowed to effuse through a fine hole then
52. The equivalent conductivity of a solution find what molar ratio of gases coming out initially?
containing 2.54g of CuSO4 per L is 91.0 W–1 (Given mixture contains He,CH4 and SO2 in
cm2 eq–1. Its conductivity would be 1 : 2 : 3 mole ratio).
(a) 2.9 × 10-3 -1 cm-1 (a) (b) 2 : 2: 3
2 : 2 :3
(b) 1.8 × 10-2 -1 cm-1 (c) 4 : 4 : 3 (d) 1 : 1 : 3
(c) 2.4 × 10-4 -1 cm-1 60. According to Bohr ’s theory, the angular
(d) 3.6 × 10-3 -1 cm-1 momentum for an electron of 3rd orbit is
(a) 3 (b) 1.5
53. The half-life of two samples are 0.1 and 0.8 s.
Their respective concentration are 400 and 50 (c) 9 (d) 2
respectively. The order of the reaction is
61. 2.76 g of silver carbonate on being strongly
(a) 0 (b) 2
heated yields a residue weighing
(c) 1 (d) 4 (a) 3.54 g (b) 3.0 g
54. Which sequence of reactions shows correct (c) 1.36 g (d) 2.16 g
chemical relation between sodium and its 62. The final product (IV) in the sequence of
compounds? reactions
(a) Na + O2 ¾¾ ® Na2O ¾¾ ¾®
HCL(aq) PBr
CH 3 CHOH ¾¾¾
3 ® I ¾¾¾
Mg
Ether
®
® Na2CO3 ¾¾
NaCl ¾¾¾CO2
® Na D
|
CH3
(b) Na ¾¾
O2
® Na2O ¾¾¾
H2O
® NaOH ¾¾ ¾
CO2
® CH2—CH2
Na2CO3 ¾¾D
® Na
O III H2 O IV is
(c) Na + H2O ¾¾ ® NaOH ¾¾®
HCl NaCl II
(a) CH3—CH OCH2CH2OH
¾¾CO2
¾® Na2CO3 ¾¾
D
® Na
(d) Na + H2O ¾¾® NaOH ¾¾¾ CO2
® Na2CO3 CH3
(b) CH3 — CHCH2CH2Br
¾¾®
HCl
NaCl ¾¾ ¾¾
Electrolysis
® Na
(molten)
CH3.
55. Purest form of iron is
(c) CH3—CH —CH2CH2OH
(a) pig iron (b) wrought iron
(c) cast iron (d) steel
CH3
56. Which has the smallest size?
(d) CH3—CH OCH2CH3
(a) Na+ (b) Mg2+
(c) Al 3+ (d) P5+ CH3
EBD_7443
www.jeeneetbooks.in

WWW.IIT-NEET.XYZ

2014-6 Target VITEEE

63. Hg 2+ /H+
Ph — C º C — CH3 ¾¾¾¾ 71. Which of the following compounds cannot be
¾® A. prepared singly by the Wurtz reaction?
O (a) C2H6
Ph—C Ph—H2C (b) (CH3)2CHCH3
C O (c) CH3CH2CH2CH3
(a) CH2 (b)
H3C (d) All of the above can be prepared
H3C
72. Which of the following oxides is strongly basic?
OH (a) Tl2O (b) B2O3
Ph—CH (c) Al2O3 (d) Ga2O3
Ph—C
C—OH 73. In Langmuir’s model of adsorption of a gas on a
(c) CH (d) solid surface,
H3C
H3C (a) th e rate of dissociation of adsorbed
molecules from the surface does not
64. Which of the following has an ester linkage?
(a) Nylon-66 (b) Dacron depend on the surface covered
(c) PVC (d) Bakelite (b) the adsorption at a single site on the surface
65. Which of the following pairs give positive may involve multiple molecules at the same
Tollen’s test? time
(a) Glucose,sucrose (c) the mass of gas striking a given area of
(b) Glucose,fructose surface is proportional to the pressure of
(c) Hexanal,acetophenone the gas
(d) Fructose,sucrose (d) the mass of gas striking a given area of
66. Peptisation involves surface is independent of the pressure of
(a) precipitation of colloidial particles the gas
(b) disintegration of colloidal aggregates
74. How many sigma and pi-bonds are there in the
(c) evaporation of dispersion medium
molecule of dicyanoethene (CN—CH = CH — CN)?
(d) impact of molecules of the dispersion
medium on the colloidal particles (a) 3 sigma and 3 pi (b) 5 sigma and 2 pi
67. Which of the following has the maximum number (c) 7 sigma and 5 pi (d) 2 sigma and 3 pi
of unpaired d-electrons? 75. What will be the order of reactivity of the
(a) Fe2+ (b) Cu+ following carbonyl compounds with Grignard’s
(c) Zn (d) Ni3+ reagent?
68. Iodine is formed when potassium iodide reacts
with a solution of H H
(a) ZnSO4 (b) CuSO4 C O C O
(c) (NH4)2SO4 (d) Na2SO4
H H3C
69. Which of the following does not represent the
correct order of the property indicated? I II
(a) Sc3+ > Cr 3+ > Fe3+ > Mn3+ — ionic radii
(b) Sc < Ti <Cr < Mn —density (CH3)3C
(c) Mn2+ > Ni2+ > Co2+ < Fe2+ —ionic radii H3C
O C O
(d) FeO < CaO < MnO < CuO —basic nature C
70. If the elevation in boiling point of a solution of H3C (CH3)
10 g of solute (mol. wt. = 100) in 100 g of water is
D Tb, the ebullioscopic constant of water is III IV
(a) 10 (b) 100 Tb
(a) I > II > III > IV (b) IV > III > II > I
DTb (c) II > I > IV >III (d) III > II > I >IV
(b) D Tb (d)
10
www.jeeneetbooks.in

WWW.IIT-NEET.XYZ

Solved Paper 2014 2014-7

NH2
PART - III (MATHEMATICS)
dy x 2 + y 2 + 1
H2 O 81. The solution of = , satisfying
¾¾¾
® A ¾¾¾¾
CH3COOH¾
® B ¾¾¾ ®C
Ac2 O Br2
76. H+ dx 2 xy
y(1) = 0 is given by
CH3 (a) hyperbola (b) circle
The final product ‘C’ in the above reacrtion is (c) ellipse (d) parabola
NHCOCH3 dy f ( xy )
NH2 82. If x. + y = x. , then f (xy) is equal to
Br dx f '( xy )
COCH3
x2
(a) (b) (a) (b) k .e y
2/2
k .e 2
xy
2

CH3 (c) k .e x (d) k .e 2


CH3
83. The differential equation of the rectangular
COCH3 NH2 hyperbola hyperbola, where axes are the
asymptotes of the hyperbola, is
Br Br
dy dy
(a) y x (b) x y
(c) (d) dx dx
dy
(b) x y (d) xdy + ydx = c
CH3 CH3 dx
77. Which of the following isomerism is shown by 84. The length of longer diagonal of the
ethyl acetoacetate? parallelogram constructed on 5a + 2b and a – 3b,
(a) Geometrical isomerism if it is given that |a| = 2 2 , |b| = 3 and the angle
(b) Keto-enol tautomerism
(c) Enantiomerism between a and b is , is
(d) Diastereoisomerism 4
78. The final product obtained in the reaction, (a) 15 (b) 113
Br
(c) 593 (d) 369
¾¾¾¾
Mg/ether
® A ¾¾¾¾¾
Heavy water
® is
85. If r = b × c + c × a + a × b and [a b c] = 2,
D then + + is equal to
(a) r. [b × c + c × a + a × b]
(a) (b)
1
(b) r. (a + b + c)
OH OD 2
(c) 2r. (a + b + c)
(c) (d) (d) 4
86. If a, b, c are three non-coplanar vectors and p, q, r
79. Among the following the strongest nucleophile are reciprocal vectors, then (la + mb + nc).
is (lp + mq + nr) is equal to
(a) C2H5SH (b) CH3COO– (a) l + m + n (b) l 3 + m3 + n3
(c) CH3NH2 (d) NCCH2 2
(c) l + m + n 2 2 (d) None of these
80. Which set has different class of compounds? 87. If the integers m and n are chosen at random
(a) Tranquillizers-Equanil, heroin,valium from 1 to 100, then the probability that a number
(b) Antiseptics-Bithional,dettol,boric acid of the form 7n + 7m is divisible by 5, equals to
(c) Analgesics-Naproxen,morphine,asprin
1 1 1 1
(d) Bactericidal-penicillin,aminoglycosides, (a) (b) (c) (d)
ofloxacin 4 2 8 3
EBD_7443
www.jeeneetbooks.in

WWW.IIT-NEET.XYZ

2014-8 Target VITEEE


88. Let X denote the sum of the numbers obtained p/ 4
when two fair dice are rolled. The variance and then ò0 D ( x ) dx is equal to
standard deviation of X are
1 1 1
31 31 35 35 (a) (b) (c) 0 (d) -
(a) and (b) and 4 2 4
6 6 6 6
98. Let f (x), be differentiable " x. If f (1) = –2 and
17 17 31 35 f (x) ³ 2 " x [1, 6], then
(c) and (d) and
6 6 6 6 (a) f (6) < 8 (b) f (6) ³ 8
89. A four digit number is formed by the digits 1, 2, (c) f (6) ³ 5 (d) f (6) £ 5
3, 4 with no repetition. The probability that the
number is odd, is 2r 1 m
Cr 1
1 2 m
(a) zero (b) 99. If r = m 1 2 m 1 , then
3
1 sin 2 (m 2 ) sin 2 (m) sin 2 (m 1)
(c) (d) None of these
4
å D , is
m
90. If the vertices of a triangle are A(0, 4, 1), B(2, 3, –1) the value of r
and C(4, 5, 0), then the orthocentre of ABC, is r =0
(a) (4, 5, 0) (b) (2, 3, –1) (a) 1 (b) 0
(c) (–2, 3, –1) (d) (2, 0, 2) (c) 2 (d) None of these
91. The equation of normal to the curve
x -1 y + 1 z -1
y = (1 + x)y + sin–1(sin2 x) at x = 0 is 100. Two lines = = and
(a) x + y = 1 (b) x – y = 1 2 3 4
(c) x + y = –1 (d) x–y = –1 x -3 y - k
92. The value of c from the Lagrange’s mean value = = z intersect at a point, if k is
1 2
theorem for which f (x) = 25 - x 2 in [1,5], is equal to
(a) 5 (b) 1 2 1 9 1
(c) 15 (d) None of these (a) (b) (c) (d)
9 2 2 6
é 3 4ù
93. If A = êê5 7úú , then A . (adj A) is equal to x
ë û 101. The minimum value of log x is
(a) A (b) | A |
(c) | A | (d) None of these 1
94. If there is an error of k% in measuring the edge (a) e (b)
e
of a cube, then the percent error in estimating its
(c) e 2 (d) e 3
volume is
102. The triangle formed by the tangent to the curve
(a) k (b) 3 k
f (x) = x2 + bx –b at the point (1,1) and the
k coordinate axes lies in the first quadrant. If its
(c) (d) None of these area is 2, then the value of b is
3
95. If the system of equations x + ky – z = 0, (a) –1 (b) 3
3x – ky – z = 0 and x – 3y + z = 0, has non-zero (c) –3 (d) 1
solution, then k is equal to 103. The statement (p Þ q) Û (~ p Ù q) is a
(a) –1 (b) 0 (a) tautology
(c) 1 (d) 2 (b) contradiction
96. If the points (1, 2, 3) and (2, –1, 0) lie on the (c) Neither (a) nor (b)
opposite sides of the plane 2x + 3y – 2z = k, then (d) None of these
(a) k < 1 (b) k > 2
3
(c) k < 1 or k > 2 (d) 1 < k < 2 104. If x + i y = , then x2 + y2 is
2 + cos q + i sin q
1 cos x 1 - cos x equal to
97. If D (x) = 1 + sin x cos x 1 + sin x - cos x , (a) 3x – 4 (b) 4x –3
sin x sin x 1
(c) 4x + 3 (d) None of these
www.jeeneetbooks.in

WWW.IIT-NEET.XYZ

Solved Paper 2014 2014-9

105. The negation of (~ p Ù q) Ú (p Ù ~q) is (a) 20 (b) 40


(a) (p Ú ~q) Ú (~p Ú q) (c) 60 (d) 80
(b) (p Ú ~q) Ù (~p Ú q) 113. Equation of the chord of the hyperbola
(c) (p Ù ~q) Ù (~p Ú q) 25x2 –16y2 = 400 which is bisected at the point
(6, 2) is
(d) (p Ù ~q) Ù (p Ú ~q)
(a) 6x – 7y = 418 (b) 75x – 16y = 418
106. The normals at three points P, Q and R of the
parabola y2 = 4ax meet at (h, k). The centroid of (c) 25x – 4y = 400 (d) None of these
114. If a plane meets the coordinate axes at A,B and C
the D PQR lies on such that the centroid of the triangle is (1, 2, 4),
(a) x = 0 (b) y = 0 then the equation of the plane is
(c) x = –a (d) y = a (a) x + 2y + 4z = 12 (b) 4x + 2y + z = 12
107. The minimum area of the triangle formed by any (c) x + 2y + 4z = 3 (d) 4x + 2y + z = 3
x2 y2 115. The volume of the tetrahedron included between
tangent to the ellipse + = 1 with the the plane 3x + 4y – 5z – 60 = 0 and the coordinate
a2 b2 planes is
coordinate axes is
(a) 60 (b) 600
( a + b) 2 (c) 720 (d) 400
(a) a2 + b2 (b)
2
ò
2x
116. (sin x+ | sin x |) dx is equal to
( a - b) 2 0
(c) ab (d) (a) 0 (b) 4
2
108. If the line lx + my – n = 0 will be a normal to the (c) 8 (d) 1

ò
2
a2 b2 (a2 + b2 )2 117. The value of [ x 2 ] dx, where [ . ] is the
hyperbola, then - = , where 0
l 2 m2 k greatest integer function, is
k is equal to
(a) n (b) n 2 (a) 2 – 2 (b) 2 + 2
(c) n 3 (d) None of these (c) 2 –1 (d) 2 –2
109. If cos + i sin , b = cos + i sin ,
ò
1
118. If l(m, n) = t m (1 + t)n dt, then the expression
b c a
and + + = 1 , then
0
c = cos + i sin for l(m, n) in terms of l(m + 1, n + 1) is
c a b
cos ( ) + cos ( ) + cos ( ) is equal 2n n
(a) - . l (m +1, n -1)
to m +1 m +1
3 3 n
(a) (b) - (b) . l (m + 1, n -1)
2 2 m +1
(c) 0 (d) 1
2n n
110. If | z + 4 | £ 3, then the greatest and the least (c) + l. ( m +1, n -1)
value of | z + 1| are m + 1 m +1
(a) –1, 6 (b) 6, 0 m
(c) 6, 3 (d) None of these (d) . l ( m + 1, n -1)
n +1
111. The angle between lines joining the origin to the
119. The area in the first quadrant between
point of intersection of the line 3 x + y = 2 and the x2 + y2 = 2 and y = sin x is
curve y2 – x2 = 4 is 3
-8 3
2 p (a) (b)
(a) tan –1 3 (b) 4 4
6
-16 3 3
-8
æ 3 ö÷ p (c) (b)
(c) tan–1 ççç ÷÷ (d) 4 2
çè 2 ø÷ 2 120. The area bounded by y = xe |x| and lines | x | = 1,
y = 0 is
112. If the area of the triangle on the complex plane
formed by the points z, z + i z and iz is 200, then (a) 4 sq units (b) 6 sq units
the value of 3 | z | must be equal to (c) 1 sq unit (d) 2 sq units
EBD_7443
www.jeeneetbooks.in

WWW.IIT-NEET.XYZ

2014-10 Target VITEEE

SOLUTIONS
PART - I (PHYSICS) 7. (a) red violet
Vp As wavelength less than that of yellow
1. (b) Amplification factor of a triode, colour and hence can initiate photoelectric
Vg effect irrespective of intensity.
Vp Vs = –50 (–20 ) = 10V 8. (c) Radius of circular path

2. (a) As we know, energy E = mc2 mV V


r=
= 0.5 × 10 –3 × (3×108)2 B e
B
= 4.5 × 1013J m
4.5 1013 6 107
= 1.25 107 kWh
3.6 10 6 r=
1.7 1011 15 10 2
3. (b) Here, diode in lower branch is forward and = 2.35 × 10–2 m = 2.35 m
in upper branch is reversed biased 9. (a) As we know, Self-inductance of the
5 5 solenoid
i= A
20 30 50 2
L= r. 0N A
4. (b) Power consumed by motor = 5 kW
= 5 × 10 3 W = 5000W I
2
mgh 600 4 10 7 2000 1.5 10 4
Power used in lifting water = =
t 0.3
= 7.5 × 9.8 × 4.7 = 3454.5 kW = 1.5 H
Power used V 100
Efficiency = Power consumed 100% 10. (d) Current I = = = 10A
R 10
3454.5 1
= 100 69% Energy, E= LI2
5000 2
5. (c) For a closed circuit cell supplies a constant 1
current in the circuit. = × 5 × (10)2 = 250J
2
11. (c) Given : V=750×10 –3 V;
Ig=15×10–3A and I = 25A
Using, a y
Ig

750 10 3
For cell E = V + Ir = =50
15 10 3
For V = 50 V
E = 50 + 11r S
Similarly, for V = 60 V Ig = ×I
S a
E = 60 + r
From eqs. (i) and (ii), we get S
15 × 10–3 = × 25
E = 61V S 50
6. (b) The ionosphere can reflect electromagnetic S = 0.03
waves of frequency less than 40 MHz but
not of frequency more than 40 MHz.
www.jeeneetbooks.in

WWW.IIT-NEET.XYZ

Solved Paper 2014 2014-11

I1 125 2mv
12. (d) Here, r = R I 1 =2 1 18. (b) Radius of path r time =
q
2 100

5 1 r m qp
=2 1 = 2 × = 0.05
4 4 rp mp q
13. (d) Resistances 10 , 60 and 100 are in
series and they together are in parallel to r 4
or, r 10 2cm .
200 resistance. When a potential 10 2
difference of 15 V is applied across 200 19. (d) The effective resistance will decrease when
then current through it resistance R is connected in parallel with
15 the voltmeter.
I= = 7.5×10–2A
200 A V
14. (b) Energy stored in capacitor = energy stored
in inductance
(R)
1 2 1 2
i.e., CV LI
2 2
2 V
LI2 1 2 According to Ohm’s law, V = IR or, R =
C= = 2 = 25 F I
V2 400 Here, as R decreases, so V decrease and I
15. (d) Boolean expression for Logic gate-1 should increase.
B. C = Y’ 20. (c) Fraction retained by nucleus
Boolean expression for Logic gate-II m1 = 1 m2 = A

A+ B.C Y"
Neutron Nucleus at rest
Boolean expression for Logic gate-III
2
2
A + B. C = Y k m 2 m1 A 1
16. (b) In series combination equivalent k retained m1 m 2 A 1
resistance,
R = R1 + R2 After collision kinetic energy retained by
=6 + 4 = 10 k 2
A 1
Error in combination, neutron E
A 1
R= R1 + R2
tan tan 45
10 10 21. (d) Here, tan ' = cos =
6 4 cos 30
=
100 100
= 0.6 + 0.4 = 1 1 2
tan = =
3/2 3
R 1
= 10 %
R 10 2
or = tan–1
17. (b) If the number of electrons increase, their 3
number of collision, increasing the thermal 22. (b) Luminous efficiency for the same power
and electrical resistance. supply, 40 W fluorescent tube gives more
So, electrical and thermal conductivities light. Hence, 40 W fluorescent tube has
both decrease. greater
EBD_7443
www.jeeneetbooks.in

WWW.IIT-NEET.XYZ

2014-12 Target VITEEE

V 2.3 0.3 5
23. (b) Resistance, R = –1 = log 2
I 3 7
10 10
T = 5 loge 2
2
R= 103 = 0.2 × 103 = 0.2k Now, let t’ be the time after which activity
10 reduces to half
24. (c) Here, number of electrons n e = 3.13 × 1015 l'/ 5log e 2
and number of protons n p = 3.12 × 1015 1 1
Current I = n eqe + np qp 2 2
= 3.13 × 10 15 × 1.6 × 10–17 + 3.12 × 10 15 t' = 5 ologe2
× 1.6 × 10 –19
1 1 1 1 1
= 1 × 10 –3 = 1 mA 29. (b) As we know R
Now, due to excess charge on electrons, 2 2 =R 4 9
2 3
the direction of the current will be towards
right. 1 9 4 5R
=R
25. (b) In conductor separation between 36 36
conduction and valence bands is zero and
36
in insulator, it is greater than 1eV. =
Hence, in semiconductor the separation 5R
between conduction and valence band is 30. (d) From Einstein’s Photoelectric equation,
1 eV. hc
26. (c) Potential of big drop = q × n 2/3 = 100 V ev0 + =
27. (d) 250V i i1
hc
and ev0 0
'
i2
20V ev'0 100 1
15V =
eV0 ' 200 2
2ev’0 + 2 = ev0 +
For R = 1k
ev 0 7.7 4.7
15 ev’0 = = 1.5V
i1 = mA = 15 mA 2 2
1 31. (d) As we know, Photoelectric current depends
R = 250 on the intensity of incident radiation i.e.,
i I
20 15 5
i250 = = 20 mA 1 1
250 250
But, intensity of radiation I so, i
iZener = 20 – 15 = 5mA d2 d2
28. (d) After n half-lives 32. (a) The creation of an elementary particle and
n t/7 its antiparticle usually from a photon (or
N 1 1 another neutral boson ) is called Pair
N0 2 2 production. This is allowed, provided there
is enough energy available to create the
5/7
N0 cN0 1 pair.
N
e cN0 2 –
e
5/7 0
1 1 Neutron
e 2
Taking log on both sides, we get Photon )
po
sitr
5 1 on 0
log 1 – log e = log
7 2 +
e
www.jeeneetbooks.in

WWW.IIT-NEET.XYZ

Solved Paper 2014 2014-13

33. (a) For charging the capacitor, q = q 0 37. (c) Inside a magnet, magnetic lines of force
t
move from south pole to north pole.
1 e CR di d
38. (b) E.M.F. e = M = 0.005 × (i sin t)
dt dt 0
= 0.005 × i0 cos t
And, Potential difference V = V0 (1 – e–t/CR) emax = 0.005 × 10 × 100 = 5
39. (a) Impedance of L-C-R circuit will be minimum
V V0 for a resonant frequency so,
1
v0 =
0.632 V0 Growth of potential 2 LC
1 105
= = Hz
2 1 10 3 0.1 10 6 2
CR t
12375
E 40. (a) Energy = = 2.475 eV = 4 × 10–19 J
34. (c) Here, i = 5000
R r
Minimum intensity to which the eye can
E respond.
0.5 = E = 5.5 + 0.5 r leye = (photon flux) × energy of a photon
11 r
leye = (5 × 104) × 4 × 10–19
E = 2 × 10–14 W/m2
0.9 = or, E = 4.5 + 0.9r
5 r Now, lesser the intensity required by a
On solving we get r = 2.5 detector for detection more sensitive it will
35. (c) Power of battery, when charged is given by
P1 = V1I1 lear 10 13
be = l 5
Electrical energy dissipated id given by eye 2 10 14
E1 = P1t1
E1 = V1I1 t1 = 15 × 10 × 8 = 1200 Wh PART - II (CHEMISTRY)
Similarly, the electrical energy dissipated
during the discharge of battery is given by 41. (c) The organic compounds containing
E2 = V2 I2t2 sulphur when react with sodium metal give
= 14 × 5 × 15 = 1050 Wh Na 2S. The Na 2 S when react with lead
Hence, watt-hour efficiency of the battery acetate forms black ppt. of PbS.
E2 Na2S + (CH3COO)2Pb
100 0.875 × 100 = 87.5 %
E1
PbS + 2CH3COONa
BCentre 2 3/ 2 Black
x ppt
36. (b) Here the ratio, B = 1
axis R2 42. (b) Volume strength = 5.6 × normality
= 5.6 × 1.5
1 = 8.4 L
Also, Baxis = Bcentre 43. (a) On subtracting eqn. (ii) from (i) we get
8
3/ 2 MnO 4 + 4H+ + 3e– MnO2+2H2O
8 x2 x2
1 4=1+ 1.51 5 2 1.23
1 R2 R2 –E3 =
3
x2 E3 = 1.70 V
3= X2 = 3R2 44. (b) Volume of unit cell (V) = a3
R2 = ( 3.04 × 10–8 cm )3
or, x = = 2.81 × 10–23 cm3
3R
EBD_7443
www.jeeneetbooks.in

WWW.IIT-NEET.XYZ

2014-14 Target VITEEE


45. (c) Now, eqs. 2 × (b) + 3 × (c) – (a)

3d 4s 4p 4d 3
C2H6+ O 2CO2 + 3H2O
Fe= 2 2
3+
Heat of combustion of Ethane x = 2(–94.1)
[Fe(H2O)6 ] = '' '' '' '' '' '' + 3 (–68.3) – (–21.1)
= (–188.2) + (–204.9) – 21.1
3 2
sp d hybridisation = – 372 kcal

[Fe(CN)6] =
3–
'' '' '' '' '' '' 51. (a) Ag2CrO4 2Ag+ + Cr O 24
S 2S S
2 3
d sp hybridisation 2
KSP = (2s)2 s = 4s .s = 4s3
3–
[Fe(Cl)6] = '' '' '' '' '' '' 1/3
1/3
K sp 32 10 12
3 2
sp d hybridisation S=
4 4
46. (a) A compound havin g maximum
electronegative element will form strong 2 10 4 M
hydrogen bond. F is the most negative 52. (a) We know that,
element among halogens hence form
strongest hydrogen bond. = eq .C
–1 cm2 eq–1
47. (b) Given kf = 1.1 × 10–2 , kb = 1.5 × 10–3 Given, eq = 91.0
kf 1.1 10 2 = 91
1
cm 2 eq 1
kc= = 7.33
kb 1.5 10 3
2.54
normality 0.1 eq.cm 3
48. (c) Molarity of base = 0.1 159 / 2 1000
Acidity 1
= 2.9 × 10–3 1cm 1
M1V1 = M2V2
0.1 × 19.85 = M2 × 20 53. (b) It is known,
M2 = 0.09925 0.099 (n 1)
t1/2 1 a2
49. (d)
t1/2 2 a1
Here, n = order of the reaction
Given, (t1/2)1 = 0.1 s, a1= 400
(t1/2)2 = 0.8 s, a2= 50
On putting the values,

5 + 4 + 3 + 2 + 1 = 15 (n 1)
0.1 50
Equation of normal of x = 0 and y = 1 is 0.8 400
y – 1 = –1 (x – 0)
y – 1 = –x x + y = 1 Taking log on both sides

50. (a) Given, 0.1 50


log n 1 log
0.8 400
(a) 2C + 3H2 C2H6 ; H = –21.1 kcal
(b) C + O2 CO2 ; H = – 94.1 kcal 1 1
log n 1 log
8 8
(c) H2 + 1 O2 H2O ; H = –68.3 kcal n–1=1 n=2
2
www.jeeneetbooks.in

WWW.IIT-NEET.XYZ

Solved Paper 2014 2014-15

54. (d) 2Na + 2H2O 2NaOH + H2 62. (c)


2NaOH + CO2 Na2CO3 + H2O
PBr3 Mg
Na2CO3 + 2HCl 2NaCl + H2O + CO2 CH3–CH–OH CH3–CH–Br ether
NaCl Electrolysis + – CH3 CH3
Na + Cl
(molten) CH3–CH–MgBr
– –
+e –e CH3
CH2–CH2
Na Cl
O
55. (b) The purest form of iron is wrought or CH3–CH–CH2CH2OMgBr
malleable.
56. (d) P5+ having maximum nuclear charge per CH3
electron. Therefore, its size is smallest.
57. (d) 8Al 8 Al3+ + 24e– H2O
9 Fe + 24e–
8/3+ 9Fe
Total 24 electrons are transferred. CH3–CH–CH2CH2OH
58. (b) On increases the number of lone pairs of CH3
electrons, bond angle decreases. 3-methyl butanol
Therefore, order of bond angle is
63. (a)
NH 4 > NH3 > NH 2 OH
2+ +
(no/p) (one/p) (two/p) Hg /H
Ph–C C–CH3+H2O –
Ph–C–CH–CH3
'
n He 1 16 1
2+ +
59. (c) n 'CH 2 4 1 Hg /H
4
O OH

n' He 1 64 4 Ph–C–CH2CH3 –H2O


Ph–C–CH2CH3
OH
n 'SO 3 4 3
2
64. (b) Condensation of diacid with dialcohol
So, molar ratio will be, leads to ester linkage,
n 'He : n 'CH : n 'SO 4:4:3.
4 2
60. (a) Angular momentum, mvr nHOCH2CH2OH+nHOOC– –COOH
Ethylene glycol
nh 3 h 1.5h (dialcohol) Terephthalic acid
= (diacid)
2 2
h
= 3h h –O–CH2CH2–OOC– –CO–
2 –H2O
61. (d) Silver carbonate on being strongly heated Ester linkage n
decomposes as dacron
65. (b) Aldehydes and -hydroxy ketones give
Ag2CO3 1 positive Tollen’s test. Glucose being a
2Ag CO2 O2
2 polyhydroxy aldehyde and fructose an
276g 216g -hydroxy ketone give positive Tollen's
As 276g of Ag2CO3 gives = 216g of Ag test.
Hence, 2.76g of Ag2CO3 will give 66. (b) Peptisation is the process in which freshly
prepared precipitate disintegrates into
2.76 216 colloidal solution.
= 2.16g
276
EBD_7443
www.jeeneetbooks.in

WWW.IIT-NEET.XYZ

2014-16 Target VITEEE


67. (a) Fe2+(24) = [Ar] 3d6 4s0 It has 4 unpaired 76. (d)
electrons NH2 NHCOCH3
Cu+(28) = [Ar] 3d104s0 It has 0 unpaired
electron
Br2/CH3COOH
Zn(30) = [Ar] 3d10 4s2 It has 0 unpaired Ac2O
electron
Ni3+(25) = [Ar] 3d7 4s0 It has 3 unpaired
electrons
68. (b) CuSO4 + 2Kl Cul2 + K2SO4 CH3 CH3
(A)
2Cul2 2Cul + l2
Cuprous iodide NHCOCH3 NH2
69. (a) The correct order of ionic radii is Cr 3+ > Br Br
Mn3+ > Fe3+ > Sc3+. HOH
1000 K b w
70. (c) We know that, Tb
W M
1000 K b w
M= CH3 CH3
W Tb (B) (C)
1000 K b 10
Tb O O
100 100
Tb K b 77. (b) CH3 –C–CH2–C–OC2H5
71. (b) If two different alkyl halides (R1–X and R2–X) (Keto)
are used, a mixture of three alkanes is
OH O
obtained which is difficult to separate.
72. (b) On moving down the group, the nature of
the oxides of group 13 elements changes
from weakly acidic to amphoteric and (enol)
amphoteric to basic. Tl 2 O in aqueous MgBr
solution gives TlOH which soluble and a
Br
strong base.
73. (c) According to Langmuir’s adsorption Mg
isotherm, the mass of gas striking a given 78. (b) ether
area of surface is proportional to the
(A)
x k 'p
pressure of the gas as D
m 1 kp
74. (c) 2
N C–C=C–C N Heavy water
2
H H
7 sigma and 5 pi (B)
75. (a) As the number and the size of the alkyl
79. (a) Nucleophiles are the species which have
groups increases, reactivity decreases.
Hence, the correct order of reactivity is tendency to donate a pair of electrons.
They can be neutral or negatively charged.
H H H3C The nucleophilic power depends on the

C O> –
C O> C– O > tendency of species to donate the
H H3C H3C electrons. Due to the presence of +I effect,
it increases. Hence, higher the +I effect,
(CH3)3C higher the nucleophilic power.
80. (a) Heroin is a narcotic analgesic and not used
C–O
as tranquilizer.
(CH3)3C
www.jeeneetbooks.in

WWW.IIT-NEET.XYZ

Solved Paper 2014 2014-17

PART - III (MATHEMATICS) x


dy
y
dx

dy x2 y2 1 84. (c) Given : a 2 2, b 3


81. (a) One diagonal is 5a + 2b + a – 3b = 6a – b
dx 2xy
Length of one diagonal
2xy dy x 2 1 dx y 2dx = 6a b

xd y2 y2dx 36a2 b 2 2 6 a . b .cos 45


x2 1
.dx
x2 x2 1
36 8 9 12 2 2 3
2
y2 1
d 1 dx 288 9 12 6 225 15
x x2 other diagonal is (5a + 2b) – (a – 3b)
= 4a + 5b
y2 1 Its length is
x C
x x
(4a)2 (5 b 2 ) 2 | 4a || 5b | cos 45
2 2
y x 1 Cx
16 8 25 9 40 6 593
When x = 1, y = 0 85. (b) r.a = a.b c a.c a a.a b
0 = 1–1 + C
C 0 = abc 0 0
The solution is x2–y2 = 1 i.e., hyperbola.
Similarly, r.b = abc and r.c = [abc]
dy f (xy)
82. (a) x. y x. 1 1
dx f ' xy r. a b c r.a r.b r.c
2 2
d f (x, y) 1
i.e., xy x = abc
dx f ' x, y 2
f ' xy 1
d xy x dx = 2
f xy 2
86. (c) p, q and r are reciprocal vectors of a, b and
f ' xy c respectively.
d xy xdx So, p.r = 1, p.b = 0 = p.c
f xy
q.a = 0, q.b = 1, q.c = 0
r.a = 0, r.b = 0, r.c = 1
x2 (la + mb + nc) . (lp + mq + nr)
log f xy C
2 = l2 + m2 + n2
87. (a) Let I = 7n + 7m , then we observe that 7i,
x2 / 2 C
f xy e 72, 73 and 74 ends in 7, 9, 3 and 1,
respectively. Thus, 7i ends in 7, 9, 3 or 1
x2 x2 according as i is of the form 4k + 1, 4k+2,
4k–1, respectively.
e 2 .eC k.e 2 If S is the sample space, then n(S) =
83. (b) The differential equation of the (100)2
rectangular hyperbola xy = c2 is 7m + 7n is divisible by 5, if
dy (i) m is of the form 4k + 1 and n is of
y+ x 0 the form 4k – 1 or
dx (ii) m is of the form 4k + 2 and n is of
the form 4k or
EBD_7443
www.jeeneetbooks.in

WWW.IIT-NEET.XYZ

2014-18 Target VITEEE


(iii) m is of the form 4k–1 and n is of the
form 4k+1 or 2 2 1 32 2 42 3 52 4
(iv) m is of the form 4k and n is of the
form 4k + 1 or 6 2 5 7 2 6 82 5 9 2
So, number of favourable ordered pairs 4 102 3 112 2 122 1
(m, n) = 4 × 25 × 25 = 72
4 25 25 1 36
Required probability = 2 1974
100 4 = 49
36
88. (b) Let x denote the sum of the numbers
1974 1764
obtained when two fair dice are rolled. =
So, X may have values 2, 3, 4, 5, 6, 7, 8, 9, 36
10, 11 and 12. 210 35
=
1 36 6
P (X = 2) = P (1,1) =
36 35
Variance =
2 6
P (X = 3) = P{(1, 2), (2, 1)} =
36 35
3 4 And, SD =
6
P (X = 4) = , P(X=5) = :
36 36 89. (d) Given digits are 1, 2, 3, 4.
Possibilities for units place digit
5 6 (either 1 or 3) = 2
P (X = 6) = ; P(X=7) = ;
36 36 Possibilities for ten’s digit = 3
5 Possibilities for hundred’s place digit = 2
P(X=8) = ; Possibilities for thousand place’s digit = 1
36 Number of favourable outcomes
4 3 = 2 × 3 × 2 × 1 = 12
P (X = 9) = ; P (X=10) = ; P(X=11) Number of numbers formed by 1, 2, 3, 4
36 36 (without repetitions) = 4!
2 12 1
= ; Required probability =
36 4 3 2 2
1
P (X = 12) = 90. (b) Vertices of ABC are A(0, 4, 1) , B (2, 3, –1)
36
Probability distribution table is given and C (4, 5, 0).
below 2 2
AB = (2 0) 2 3 4 1 1
X 2 3 4 5 6 7 8 9 10 11 12
= 4 1 4 3
1 2 3 4 5 6 5 4 3 2 1
P(X)
36 36 36 36 36 36 36 36 36 36 36 BC = 2 2 2
4 2 5 3 0 1
Mean X XP X = 4 4 1 3
2 1 3 2 4 3 5 4 6 5 7 6 and CA = 4 0
2
5 4
2
0 1
2
8 5 9 4 10 3 11 2 12 1
= = 16 1 1 3 2
36
AB2 + BC2 = AC 2
252 ABC is a right angled triangle.
7 We know that, the orthocentre of a right
36
angled triangle is the vertex containing the
2 right angle.
Variance = X 2P X X
Orthocentre is point B (2, 3, –1).
www.jeeneetbooks.in

WWW.IIT-NEET.XYZ

Solved Paper 2014 2014-19

91. (a) Given curve is 3 4 7 4


y = (1 + x)y + sin –1 (sin2 x) A adjA
On differentiating w.r.t x, we get 5 7 5 3
dy y y dy 1 0 1 0
1 x log 1 x = = 1. = A .I
dx 1 x dx 0 1 0 1
2sin x cos x 94. (b) Volume V of a cube of side x is given by
+ V = x3
1 sin 4 x dv
3x 2
dy dx
1 at x 0, y 1
dx at 0,1 Let the change in x be x = K% of
kx
Slope of normal at (x = 0) = –1 x
Equation of normal at x = 0 and y = 1 is 100
y – 1 = –1 (x – 0) Now, the change in volume,
y– 1= – x x+y=1 dV
92. (c) It is clear that f (x) has a definite and unique V x 3x3 x
dx
value for each x [1, 5].
Thus, for every point in the interval [1, 5], kx 3x3.k
the value of f (x) exists. = 3x2
So, f(x) is continuous in the interval [1, 5]. 100 100
Approximate change in volume
x
Also, f'(x) = , which clearly exists 3kx 3 3k 3
25 x 2 = .x
100 100
for all x in an open interval (1, 5).
So, f' (x) is differentiable in (1, 5). = 3K% of original volume
So, there must be a value c [1, 5] such 95. (c) The system has non- zero solution, if
that 1 k 1
f 5 f 1 0 24 3 k 1 =0
f' (c) =
5 1 4 1 3 1
0 2 6 6 1(–k –3) –k (3 + 1) –1(–9 + k) = 0
= – 6k + 6 = 0
4 2
k=1
c 96. (d) The points (1, 2, 3) and (2, –1, 0) lie on the
But f' (c) =
2 opposite sides of the plane 2x + 3y – 2z – k = 0
25 c
So, (2 + 6 –6 – k) (4 – 3 –k) < 0
c 6 (k – 1) (k – 2) < 0 .... (i)
=– 1< k<2
25 c2 2
4 c2 = 6 (25 – c2) 1 cos x 1 cos x
4c2 = 150 – 6c2 10 c2 = 150 97. (d) (x) = 1 sin x cos x 1 sin x cos x
c2 = 15 c= 15 sin x sin x 1
c = 15 1,5 Applying C3 C3 + C2 – C1

3 4 1 cos x 0
93. (c) A= (x) 1 sin x cos x 0
5 7
sin x sin x 1
A 21 20 1
EBD_7443
www.jeeneetbooks.in

WWW.IIT-NEET.XYZ

2014-20 Target VITEEE


= cos x – cos x ( 1 + sin x)
[ expanding along C3] = – cos x.sin x x 1 y 1 z 1
100. (c) r say
1 2 3 4
= sin 2 x x = 2r + 1, y = 3r – 1, z = 4r + 1
2
/4 /4 Since, the two lines intersect.
1 So, putting above values in second line,
(x)dx sin 2 x dx
2 we get
0 0
2r 1 3 3r 1 k 4r 1
1 cos 2x / 4
= – 1 2 1
2 2 0
2r – 2 = 4r + 1
1 r = –3/2
= + cos cos 0
2 2 2 Also 3r – 1 – k = 8r + 2
1 1 15 9
= 0 1 k = –5r – 3 = –3=
4 4 2 2
98. (b) f' (x) is differentiable x [1, 6]
By Lagrange’s mean value theorem, x
101. (a) Let f (x) =
f 6 f 1 log x
f' (x) =
6 1
log x 1
f' (x) 2 x 1, 6 (given) f' (x) =
(log x)2
f 6 2
2 [ f (1) = –2] For maxima and minima, put f’ (x) = 0
5
log x – 1 = 0
f (6) 10 – 2 f (6) 8 x=e
m Now, f" (x)
2r 1 Cr 1

99. (b) r =
m2 1 2m m 1 log x
2 1
. log x 1 .
2 log x
x x
sin 2 m 2 sin 2 m sin 2 m 1 = 4
log x

m m m
2r 1 m
Cr 1 1
0 1
r 0 r 0 r 0 e
m f" (e) = = >0
2 m 1 e
r m 1 2 m 1
r 0 f (x) is minimum at x = e.
sin 2 m 2 sin 2 m sin 2 m 1
Hence, minimum value of f (x) at x = e is
e
f (e) = e
2 m log e
m 1 2 m 1
102. (c) Given curve is y = f (x) = x2 + bx – b
m2 1 2m m 1 f' (x) = 2x + b
sin 2 m 2 sin 2 m sin 2 m 1 The equation of tangent at point (1, 1) is

=0 ( two rows are identical) dy


y– 1= x 1
dx 1,1
www.jeeneetbooks.in

WWW.IIT-NEET.XYZ

Solved Paper 2014 2014-21

y – 1 = (b + 2) (x – 1)
6 3cos 3i sin
(2 + b)x – y = 1 + b = 2
4 cos 4 cos sin 2
x y
1 6 3cos 3i sin
1 b (1 b) =
5 4 cos
2 b
6 3cos 3sin
=
Y 5 4cos 5 4cos

B On equating real and imaginary parts, we


(1, 1) get
2 3 2 cos
y = x + bx – b x=
5 4cos
O A X 3sin
And y =
5 4 cos
1 b
So, OA =
2 b 9[4 cos 2 4 cos sin 2 ]
x2 y2
and OB = – (1 + b) 2
5 4 cos
1
Now, area of AOB = × 6 3cos
2 9
= =4 –3
5 4cos 5 4cos 2
1 b 1 b
2 = 4x – 3
2 b
105. (b) Let S : (~ p ^ q) (p ~ q)
4 (2 + b) + (1 + b)2 = 0 ~S :~ [(~ p ^ q) (p ~ q)]
b2 + 6b + 9 = 0 ~S :~ (~ p ^ q) ~ (p ~ q)
(b + 3)2 b = –3 ~S : (p ~ q) (~ p q)
103. (c) 106. (b) The sum of ordinates of feet of normals
drawn from a point to the parabola,
p q p p q ~ p ^ q (p q) (~ p ^ q) y2 = 4ax is always zero.
Now, as normals at three points P, Q and R
T T F T F F
of parabola y2 = 4ax meet at (h, k).
T F F F F T
The normals from (h, k) to y2 =4ax meet the
F T T T T T
parabola at P, Q and R.
F F T T F F y-coordinate y1, y2, y3 of these points and
R will be zero.
Hence, given statement is neither
y-coordinate of the centroid of PQR
tautology nor contradiction.
104. (b) x + iy y1 y 2 y3 0
i. e., 0
3 3
3 3(2 cos i sin )
= 2 cos 2 centroid lies on y = 0
i sin 2 cos sin 2
107. (c) Equation of tangent at (a cos , b sin )
to the ellipse is
EBD_7443
www.jeeneetbooks.in

WWW.IIT-NEET.XYZ

2014-22 Target VITEEE

x y 2
cos sin 1 a2 b2 a2 b2
a b
l 2 m2 n2
Y
But given equation of normal is
Q
2
a2 b2 a2 b2
(a cos , b sin )
l2 m2 k
X' O P
X k n2
109. (d) Given: a = cos i sin
b = cos i sin
Y' and c = cos i sin
Coordinates of P and Q are b cos i sin cos i sin
Now,
a b c cos i sin cos i sin
, 0 and 0, , respectively .
cos sin cos .cos sin .sin i
1 a b ab [sin .cos sin .cos ]
Area of OPQ
2 cos sin sin 2 b
cos isin .....(i)
Minimum area = ab c
108. (b) The equation of any normal
c
x2 y2 Similarly, cos i sin
to 1 is ax cos + by cot a
a 2 b2 ..........(ii)
= a2 + b2
a
ax cos + by cot – (a2 + b2) = 0 .....(i) and cos i sin ........(iii)
b
The straight line lx + my – n = 0 will be On adding Eqs. (i) , (ii) and (iii), we get
normal to the hyperbola
cos cos cos i
x2 y2
1, then eq. (i) and lx + my – n = sin( ) sin sin 1
a 2 b2
0 represent the same line, b c a
2 2
1
a cos b cot a b c a b
l m n On equating real parts, we get
na cos ( ) + cos ( ) + cos ( )=1
sec
l a 2 b2 110. (b) z 4 3 3 z 4 3
6 z 1 0
nb
and tan = 0 z 1 6
m a 2 b2
0 z 1 6
2 2 2 2
n a n b Hence, the greatest and least values are 6
1
2 2 2
2
2 2 2
2 and 0.
l a b m a b 111. (c) On homogenising y2 – x2 = 4 with the
[ sec2 tan 2 1] help of the line 3x y 2, we get
www.jeeneetbooks.in

WWW.IIT-NEET.XYZ

Solved Paper 2014 2014-23

2 Then, A ( , 0, 0 ), B( 0, ,0 ) and C(0,0, )


3x y
2 2 are the points on the coordinate axes,
y x 4
4 The centroid of the triangle is (1, 2, 4).
2 2 2
y x 3x y2 2 3xy
1 3
2 3
4x 2 3xy 0
On comparing with ax2 + 2hxy + by2 = 0,
2 6
we get 3
a = 4, h = 3 and b = 0
We know that, and 4 12
3
h 2 ab 2. 3 0 The equation of the plane is
tan 2
a b 4 0
x y z
The angle between the lines is 1
3 6 12
3
= tan 1 4x 2y z 12
2
115. (b) The given equation of the plane is
112. (c) Let z = x + iy, then 3x + 4y – 5z – 60 = 0. It can be written in the
z + iz = x + iy + i (x + iy) = (x – y) + i (x + y)
and iz = i (x + iy) = –y + ix, x y z
form 1
Then, the area of the triangle formed by 20 15 12
these lines is which meets the coordinate axes at the
x y 1 points A(20, 0, 0), B(0, 15, 0) and C(0, 0, –12).
1
x y x y 1 The coordinates of the origin are O (0, 0, 0).
2
y x 1 Therefore, volume of the tetrahedron
OABC is =
Applying R2 R2– (R1+R3),
x y 1 20 0 0
1 1 2 2 1 1
0 0 1 x y 0 15 0 20 15 12 600
2 2 6 6
y x 1 0 0 12

1 2 2
z 200 (given) 116. (b) sin x sin x dx
2 0
2
z 400 z 20 = sin x sin x dx
0
3 z = 3 × 20 = 60
2
113. (b) Given hyperbola is 25x2 – 16y2 = 400 sin x sin x dx
If (6, 2) is the midpoint of the chord, then
equation of chord is T = S1
= 2 sin x dx 0 2 cos x dx
25 (6x) – 16 (2y) = 25 (36) – 16 (4) 0 0
75x – 16y = 450 – 32 = –2 (cos – cos 0) = –2 (–1 –1) = 4
75x – 16y = 418 2 1 2
114. (b) Let the equation of the plane is 117. (c) x2 dx x 2 dx x2 dx
0 0 1
x y z 1 2
1 = 0 dx 1 dx
0 1
EBD_7443
www.jeeneetbooks.in

WWW.IIT-NEET.XYZ

2014-24 Target VITEEE

2 Required area
= x1 2 1
= Area of circle (1st quadrant ) – sin xdx
1 m n 0
118. (a) l(m, n) = l = t 1 t dt
0
. 2 3
= cos x 0 (cos cos 0)
m 1 1 4 4
n t
l m, n 1 t . 3 3 3
m 1 8
0 = 1 1 2
4 4 4
n 1
(1 t)n 1. t m 1 .dt 120. (d) x 1
m 1 0
x 1
2n n
.l m 1, n 1 x x.e x , 1 x 0
m 1 m 1 y xe
x.e x ,0 x 1
2
119. (a) x2 + y2 = is a circle of radius and
0 1
centre at origin. Re quired area xe x dx xe x dx
1 0
2 2
X +Y = 0 1
x.e x e x x.e x ex
y = sin x 1 0

0 1 1.e e e e 0 1
(0, 0) = 1 + 1 = 2 sq units
www.jeeneetbooks.in

WWW.IIT-NEET.XYZ

VITEEE
SOLVED PAPER 2013

(a) 2k/x 3 (b) –2k/x 3


PART - I (PHYSICS) (c) k/x (d) –k/x
1. The amplitude of an electromagnetic wave in 7. A copper wire of length 2.2 m and a steel wire of
vaccum is doubled with no other changes made length 1.6 m, both of diameter 3.0 mm are
to the wave. As a result of this doubling of the connected end to end. When stretched by a
amplitude, which of the following statement is force, the elonation in length 0.50 mm is produced
correct? in the copper wire. The stretching force is
(a) The frequency of the wave changes only (Ycu = 1.1 × 1011 N/m2 , Ysteel = 2.0 × 1011 N/m2)
(b) The wave length of the wave changes only (a) 5.4 × 102 N (b) 3.6 × 102 N
(c) The speed of th e wave propagation (c) 2.4 × 10 N 2 (d) 1.8 × 102 N
changes only 8. If v , rms and p represent the mean speed,
(d) Alone of the above is correct
root mean square and most probable speed of
2. An element with atomic number Z = 11 emits
the molecules in an ideal monoatomic gas at
K a – X-ray of wavelength . The atomic
temperature T and if m is mass of the molecule,
number which emits K a – X-ray of wavelength
then
4 is
(a) vp < v < vrms
(a) 4 (b) 6
(b) no molecule can have a speed greater than
(c) 11 (d) 44
3. Mobilities of electrons and holes in a sample of 2vrms
intrinsic germanium at room temperature are
(c) no molecule can have a speed less than
0.36m2 V-1s-1 and 0.17m2V–1s–1. The electron
and hole densities are each equal to 2.5 × 1019 vp / 2
m3. The electrical conductivity of germanium is (d) None of the above
(a) 4.24 Sm-1 (b) 2.12 Sm-1 9. Two balls of equal masses are thrown upwards
(c) 1.09 Sm -1 (d) 0.47 Sm-1 along the same vertical direction at an interval
4. If a radio-receiever amplifiers all the signal of 2 s, with the same initial velocity of 39.2 m/s.
frequencies equally well, it is said to have high The two balls will collide at a height of
(a) sensitivity (b) selectivity
(a) 39.2 m (b) 73.5 m
(c) distortion (d) fidelity
(c) 78.4 m (d) 117.6 m
5. If a progressive wave is represented as
10. The dimensional formula of magnetic flux is
æ t xö
y = 2sin p çç - ÷÷÷ where x is in metre and t is (a) [M1L2T–1 A–2] (b) [M1L2T–2 A–1]
èç 2 4 ø (c) [M1L2T–1 A–1] (d) [M1L0T–2 A–1]
in second, then the distance travelled by the 11. The time dependence of a physical quantity P is
wave in 5 s is given by P = P0 e a (– a t2 ), where is a
(a) 5 m (b) 10 m constant and t is time. The constant a
(c) 25 m (d) 32 m (a) is a dimensionless
6. The gravitational potential at a place varies (b) has dimensions of P
inversely with x2(i.e.,V = k/x2), the gravitational (c) has dimensions of T-2
field at that place is (d) has dimensions of T2
EBD_7443
www.jeeneetbooks.in

WWW.IIT-NEET.XYZ

2013-2 Target VITEEE


12. If the potential energy of a gas molecule is 17. A body of mass 5 m initially at rest explodes into
M N 3 fragments with mass ratio 3:1:1. Two of
U= - , M and N being positive fragments each of mass ‘m’ are found to move
r 6 r12 with a speed of 60 m/s is mutually perpendicular
constants, then the potential energy at directions. The velocity of third fragment is
equilibrium must be
(a) zero (b) NM2 /4 (a) 10 2 (b) 20 2
(c) MN /42 (d) M2 /4N (c) 20 3 (d) 60 2
13. A table fan rotating at a speed of 2400 rpm is
18. A body of mass 2 kg moving with velocity of
switched off and the resulting variation of
6 m/s strikes in elastically with another body of
revolution/minute with time is shown in figure.
same mass mass at rest. The amount of heat
The total number of revolutions of the fan before
evolved during collision is
it, comes to rest is
(a) 18 J (b) 36 J
(c) 9 J (d) 3 J
2400 19. Two particles of equal mass m go round a circle
rpm of radius R under the action of their mutual
gravitational attraction. The speed of each
particle is
600
1 Gm 4Gm
0 8 24 (a) (b)
t (s) 2 R R
(a) 160 (b) 280
(c) 380 (d) 420 Gm 1 1
(c) (d)
14. In the adjoining figure, the position time graph 2R 2 R Gm
of a particle of mass 0.1 kg is shown. The impulse 20. Four equal charges Q each are placed at four
at t = 2 s is corners of a square of side a each. Work done in
carrying a charge –q from its centre to infinity is
6
x(m) 4 2q
(a) zero (b)
pe 0 a
2
0 1 2 3 4 5 6 q2 2q 2
Time (c) (d)
2pe 0 a pe 0 a
(a) 0.02 kg m/s (b) 0.1 kg m/s
(c) 0.2 kg m/s (d) 0.4 kg m/s 21. A network of resistances, cell and capacitor
15. The pressure on a square plate is measured by C( = 2 m F) is shown in adjoining figure.In steady
measuring the force on the plate. If the maximum state condition, the charge on 2 m F capacitor is
error in the measurement of force and length are Q, while R is unknown resistance. Values of Q
respectively 4% and 2%, then the maximum error and R are respectively
in the measurement of pressure is
(a) 1% (b) 2% 2V R 4 C
(c) 4% (d) 8% 10 V 4V
A I=1A 2µF B
16. The centre of a wheel rolling on a plane surface
moves with a speed n 0. A particle on the rim of 2
the wheel at the same level as the centre will be
moving at speed
(a) zero (b) v 0
(a) 4 m C and 10 W (b) 4 m C and 4 W
(c) 2v0 (d) 2v0 (c) 2 m C and 2 W (d) 8 m C and 4 W
www.jeeneetbooks.in

WWW.IIT-NEET.XYZ

Solved Paper 2013 2013-3

22. As the electron in Bohr’s orbit of hydrogen atom 27. A lead shot of 1 mm diameter falls through a
passes from state n = 2 to, n =1, the KE (K) and long column of glycerine. The variation of the
the potential energy (U) changes as velocity with distance covered (s) is correctly
(a) K four fold, U also four fold represented by
(b) K two fold, V also two fold
(c) K four fold, U two fold
(d) K two fold, U four fold v v
23. To get an OR gate from a NAND gate, we need (a) (b)
(a) Only two NAND gates
(b) Two NOT gates obtained from NAND gates s s
and one NAND gate
(c) Four NAND gates and two AND gates v v
obtained from NAND gates
(d) None of the above (c) (d)
24. If a current I is flowing in a loop of radius r as
s s
shown in adjoining figure, then the magnetic field
induction at the centre O will be 28. If e 0 and m 0 represent the permittivity and
permeability of vaccum and and µ represent
the permittivity and permeability of medium, then
refractive index of the medium is given by

I e 0m 0 em
I (a) (b)
em e 0m 0
r
m 0 e0 m 0 e0
(c) (d)
e m
m0 I q 29. A students plots a graph between inverse of
(a) Zero (b)
4pr magnification 1/m produced by a convex thin
m 0 I sin q 2m 0 I sin q lens and the object distance u as shown in figure.
(c) (d) What was the focal length of the lens used?
4pr 4pr 2
25. Two indentical magnetic dipoles of magnetic
moment 1.0 Am2 each,placed at a separation of 1/m
2 m with their axes perpendicular to each other.
The resultant magnetic field at a point midway
b
between the dipoles is
(b) 5 × 10-7 T a c
(a) 5 ´10-7 T u
(c) 10 T -7 (d) 2 × 10-7 T
26. The natural frequency of the circuit shown in b bc
(a) (b)
adjoining figure is ca a
C C c b
(c) (d)
b c
30. Two waves y 1 = A 1 sin (wt -b1 ) and
L L
y2 = A2 sin (wt -b2 ) superimpose to form a
resultant wave whose amplitude is
1 1 (a) A1 + A2
(a) (b) (b) |A1 + A2|
2p LC 2p 2 LC
(c) A12 + A22 - 2 A1 A2 sin (b1 - b2 )
2
(c) (d) zero
2p LC (d) A12 + A22 + 2 A1 A2 cos (b1 - b2 )
EBD_7443
www.jeeneetbooks.in

WWW.IIT-NEET.XYZ

2013-4 Target VITEEE


31. When a certain metallic surface is illuminated 36. A long glass capillary tube is dipped in water. It
with monochromatic light of wavelength , the is known that water wets glass. The water level
stopping potential for photoelectric current rises by h in the tube. The tube is now pushed
3V0.When the same surface is illuminated with a down so that only a length h/2 is outside the
light of wave length 2 , the stopping potential is water surface. The angle of contact at the water
V0. The threshold wavelength for this surface surface at the upper end of the tube will be
to photoelectric effect is (a) tan -12 (b) 60º
(a) 4 (b) 6 (c) 30º (d) 15º
4 37. In the adjoining circuit, if the reading of voltmeter
(c) 8 (d) V1 and V2 are 300 volts each, then the reading
3
32. In the V-T diagram shown in adjoining figure, voltmeter V3 and ammeter A are respectively
what is the relation between p1 and p2? C
L R = 100 W

p2
V
V1 V2 V3
p1
A

~
T 220 V, 50 Hz
(a) p2 = p1 (b) p2 < p1
(a) 220 V , 2.2 A (b) 100 V, 2.0 A
(c) p2 > p1 (d) insufficient data
(c) 220 V , 2.0 A (d) 100 V, 2.2 A
33. If a gas mixture contains 2 moles of O2 and 4
38. If the work done in turning a magnet of magnetic
moles of Ar at temperature T, then what will be
moment M by an angle of 90º from the magnetic
the total energy of the system (neglecting all
meridian is n times the corresponding work done
vibrational modes)
(a) 11 RT (b) 15 RT to turn it through an angle of 60º, then the value
(c) 8 RT (d) RT of n is
34. In the adjoining figure, two pulses in a stretched (a) 1 (b) 2
string are shown. If initially their centres are 1 1
8 cm apart and they are moving towards each (b) (d)
2 4
other, with speed of 2cm/s, then total energy of 39. The capacitance of a parallel plate capacitor with
the pulses after 2 s will be air as dielectric is C. If a slab of dielectric constant
K and of the same thickness as the separation
between the plates is introduced so as to fill
8 cm
1/4th of the capacitor (shown in figure), then the
(a) Zero new capacitance is
(b) Purely kinetic
(c) Purely potential
(d) Partly kinetic and partly potential K Air t
35. When two waves of almost equal frequency n 1
and n2 are produced simultaneously, then the
time interval between succesive maxima is t
1 1 1
(a) (b) + C C
n1 + n2 n1 n2 (a) (K + 2)
4
(b) (K + 3)
4
1 1 1 C
(c) - (d) (c) (K + 1) (d) None of these
n1 n2 n1 - n2 4
www.jeeneetbooks.in

WWW.IIT-NEET.XYZ

Solved Paper 2013 2013-5

40. Seven resistance are connected between points 43. Benzene diazonium chloride on treatment with
A and B as shown in adjoining figure. The hypophosphorous acid and water yield benzene.
equivalent resistance between A and B is Which of the following is used as a catalyst in
10 this reaction?
(a) LiAlH4 (b) Red p
(c) Zn (d) Cu+
44. Consider the following reaction sequence,
10 3
A B Alcoholic
P/Br 2 KOH O3, H2O
CH2COOH A B C
5 8 6
CF3CO 3H
CH2COOH D Alkaline
E
KMno4

(a) 5 W (b) 4.5 W Isomers are


(c) 4 W (d) 3 W (a) C and E (b) C and D
(c) D and E (d) C,D and E
PART - II (CHEMISTRY) 45. When a monosaccharide forms a cyclic
41. Which of the following does not undergo hemiacetal, the carbon atom that contained the
benzoin condensation? carbonyl group is identified as the …. Carbon
atom,because
CH2CHO CHO (a) D,the carbonyl group is drawn to the right
(b) L, the carbonyl group is drawn to the left
(c) acetal,it forms bond to an –OR and an –
(a) (b) OR’
(d) anomeric, its substituents can assume an
b or position
CHO CHO
46. Which of the following is/ are - amino acid?

NH3
(c) (d) (a) (b) Q
Q
CO2 CO2
N
OCH3 CH3 H H
(c) Both (a) and (b) (d) None of these
47. Calculate pH of a buffer prepared by adding 10
42. COOH + NaHCO3 ® mL of 0.10 M acetic acid to 20 mL of 0.1 M sodium
acetate [pKa (CH3COOH) = 4.74]
(a) 3.00 (b) 4.44
CO2 + COONa + (c) 4.74 (d) 5.04
48. The equivalent conductance of silver nitrate
* solution at 250°C for an infinite dilution was
C is with the product found to be 133.3 –1 cm2 equiv-1. The transport
(a) CO2 number of Ag+ ions in very dilute solution of
AgNO3 is 0.464. Equivalent conductances of Ag+
(b) COONa and NO-3 (in –1 cm2 equiv–1) at infinite dilution
are respectively
(c) Both (a) and (b) (a) 195.2, 133.3 (b) 61.9, 71.4
(d) None of the above (c) 71.4, 61.9 (d) 133.3, 195.2
EBD_7443
www.jeeneetbooks.in

WWW.IIT-NEET.XYZ

2013-6 Target VITEEE


49. Treating anisole with the following reagents, the
major product obtained is
I. (CH3)3 CCl, AlCl3 II. Cl2, FeCl3 (a) (b)
III. HBr, Heat
(c) Both (a) and (b) (d) None is correct
OH Br KHSO HCIO
53. Glycerol 4
A B,
Br Br A –A and B respectively are
(a) (b)
O O
(a) CH2 CH CH
C(CH3)3 C(CH3)3
Cl OH
OCH3 OH
O O
Br Cl
(c) (d) (b) CH2 CHCH
OH Cl
C(CH3)3 C(CH3)3
OH
(c) CH3CH2CHO CH3CH2CH
50. Ketones [R—C—R’] where, R = R’ = alkyl Cl

O O
group can be obtained in one step by (d) CH2 CHCH
(a) Hydrolysis of esters
(b) Oxidation of primary alcohols ClO
(c) Oxidation of secondary alcohols 54. Phenol is heated with phthalic anhydride in the
(d) Reaction of acid halide with alcohols presence of conc. H2SO4. The product gives
51. An optically active compound ‘X’ has molecular pink colour with alkali. The product is
formula C4H8O3. It evolves CO2 with aqueous (a) phenolphthalein (b) bakelite
NaHCO3. ‘X’ reacts with LiAlH4 to give an
(c) salicylic acid (d) flurorescein
achiral compound.’X’ is
CuCN
(a) CH 3CH 2CHCOOH 55. C6H5NH2 0ºC X
+
H O/H
OH Y 2 Z, Z is identified as
(b) CH 3CHCOOH (a) C6H5 — NH—CH3
(b) C6H5 — CH2—NH2
OMe (c) C6H5 — CH2—COOH
(c) CH 3CHCOOH (d) C6H5—COOH
56. B can be obtained from halide by van-Arkel
method. This involves reaction
(d) CH 3CHCH2C OOH
(a) 2Bl3 Red hot W or Ta
OH
Red hot W or Ta
(b)
OH
52. conc.H2SO4
products. (c) Both (a) and (b)
(d) None of the above
Product is/are
www.jeeneetbooks.in

WWW.IIT-NEET.XYZ

Solved Paper 2013 2013-7

57. NH4Cl(s) is heated in a test tube. Vapours are 64. Select the correct statements(s).
brought in contact with red litmus paper, which (a) LiAlH4 reduces methyl cyanide to methyl
changes it to blue and then to red. It is because amine
of
(b) Alkane nitrile has electrophilic as well as
(a) formation of NH4OH and HCl
nucleophilic centres
(b) formation of NH3 and HCl
(c) saponification is a reversible reaction
(c) greater diffusion of NH3 than HCl
(d) greater diffusion of HCl than NH3 (d) Alkaline hydrolysis of methane nitrile forms
methanoic acids
58. Out of H2S2O3, H2S2O4, H2SO5 and H2S2O8
peroxy acids are
conc.HNO3 + conc.H2SO4 Cl2/FeCl3
(a) H2S2O3, H2S2O8 (b) H2SO5, H2S2O8 65. X Y
(c) H2S2O4, H2SO5 (d) H2S2O3, H2S2O4
59. The density of solid argon is 1.65 g per cc at - The product Y is
233°C. If the argon atom is assumed to be a (a) p-chloro nitrobenzene
sphere of radius 1.54 × 10-8 cm, what per cent of (b) o-chloro nitrobenzene
solid argon is apparently empty space?
(c) m-chloro nitrobenzene
(Ar = 40)
(a) 16.5% (b) 38% (d) o, p-dichloro nitrobenzene
(b) 50% (d) 62% 66. End product of the following reaction is
60. When 1 mole of CO2(g) occupying volume 10L
at 27°C is expanded under adiabatic condition, O O + HBr
temperature falls to 150 K. Hence,final volume is
(a) 5 L (b) 20 L
(c) 40 L (d) 80 L
61. Acid hydrolysis of ester is first order reaction (a) O O
and rate constant is given by
Br
2.303 V -V0
k= log ¥ where, V0, Vt and V¥
t V¥ - Vt
OH
are the volumle of standard NaOH required to
neutralise acid present at a given time, if ester is
50% neutralised then (b) HO
(a) V¥ = Vt (b) V¥ = (Vt –V0)
(c) V¥ = 2Vt –V0 (d) V¥ = 2Vt + V0 Br
62. A near UV photon of 300 nm is absorbed by a
gas and then re-emitted as two photons. One
photon is red with wavelength of the second (c) Br OH
photon is
(a) 1060 nm (b) 496 nm
HO
(c) 300 nm (d) 215 nm
63. Which of these ions is expected to be coloured
in aqueous solution?
(d) HO OH
I. Fe3 + II. Ni2+ III. Al3+
(a) I and II (b) II and III
(c) I and III (d) I, II and III Br
EBD_7443
www.jeeneetbooks.in

WWW.IIT-NEET.XYZ

2013-8 Target VITEEE


67. Following compounds are respectively …
OH O
geometrical isomers
Cl
(a)
Cl Cl

Cl O O
P Cl Cl
Q R (b)
P Q R
(a) cis cis trans O
H3C O
(b) cis trans trans
(c) H3C
(c) trans cis cis
(d) cis trans cis
68. Which is more basic oxygen in an ester?
CH3 CH3
O
(d)
R— C—O—R
(a) Carbonyl oxygen, OH OH
(b) Carboxyl oxygen, b 72. For the cell reaction 2Ce4+ + Co ® 2Ce3+ +
(c) Equally basic
(d) Both are acidic oxygen Co3+ ; Eºcell cell is 1.89 V. If ECo2 + /Co is – 0.28 V,,
69. In a Claisen condensation reaction (when an º
ester is treated with a strong base) what is the value of E Ce4 + /Ceo3+ ?
(a) a proton is removed from the -carbon to (a) 0.28 V (b) 1.61 V
form a resonance stabilised carbanion of (c) 2.17 V (d) 5.29 V
the ester 73. A constant current of 30 A is passed through an
(b) carbanion acts as a nucleophile in a aqueous solution of NaCl for a time of 1.00 h.
nucleophilic acyl substitution reaction with
What is the volume of Cl2 gas at STP produced?
another ester molecule
(c) a new C—C bond is formed (a) 30.00 L (b) 25.08 L
(d) All of the above statements are correct (c) 12.54 L (d) 1.12 L
70. An organic compound B is formed by the reaction 74. Consider the following reaction,
of ethyl magnesium iodide with a substance A,
kA
followed by treatment with dilute aqueous acid, kB
Compound B does not react with PCC or PDC in Chair
Boat
dichloromethane. Which of the following is a
possible compound for A? The reaction is of first order in each diagram,
with an equilibrium constant of 104. For the
O O
conversion of chair form to boat form e–Ea/RT =
(a) CH—CH
2 2 (b) CH3CH2CCH3 4.35 × 10-8 m at 298 K with pre-exponential factor
of 1012 s-1. Apparent rate constant (= kA / kB)
O at 298 K is
(c) CH 3CH (d) H2C O (a) 4.35 × 104 s-1 (b) 4.35 × 108 s-1
(c) 4.35 × 10 s-8 -1 (d) 4.35 × 1012 s-1
O O
71. || || 75. If for the cell reaction, Zn + Cu2+ Cu+ Zn2+
CH3 C CH 2CH 2 COCH 2 CH3
Entropy change Sº is 96.5 J mol-1K-1, then
(i) CH3MgBr(one mole) temperature coefficient of the emf of a cell is
(ii) H 2O
A formed in this (a) 5 × 10-4 VK-1 (b) 1 ×10-3 VK-1
reaction is (c) 2 × 10-3 VK-1 (d) 9.65 × 10-4 VK-1
www.jeeneetbooks.in

WWW.IIT-NEET.XYZ

Solved Paper 2013 2013-9

76. What transition in the hydrogen spectrum would


have the same wavelength as the Balmer
transition, n = 4 to n = 2 of He+ spectrum?
(a) n = 4 to n = 2 (b) n = 3 to n = 2
(c) n = 2 to n = 1 (d) n = 4 to n = 3
77. What is the degeneracy of the level of H-atom (d)
æ R ö
that has energy çç- H ÷÷÷ ?
èç 9 ø
(a) 16 (b) 9
(c) 4 (d) 1
78. Match the following and choose the correct 80. What is the EAN of [Al(C4O4)3]3–?
option given below. (a) 28 (b) 22
Compound/Type Use (c) 16 (d) 10
A. Dry ice I. Anti-knocking PART - III (MATHEMATICS)
compound
B. Semiconductor II. Electronic diode 81. The relation R defined on set A = {x : | x | < 3, x e I }
or triode by R = {(x, y) : y = | x |} is
C. Solder III. Joining circuits (a) {–2, 2), (–1, 1), (0, 0), (1, 1), (2, 2) }
D. TEL IV. Referigerant for (b) {(–2, –2), (–2, 2), (–1, 1), (0, 0), (1, –2), (1, 2),
preserving food (2, –1), (2, –2)}
A B C D (c) {0, 0), (1, 1), (2, 2)}
(a) I II IV III
(d) None of the above
(b) II III I IV
82. The solution of the differential equation
(c) IV III II I
(d) IV II III I dy yf '( x) - y 2
79. Which of the following ligands is tetradentate? = is
dx f ( x)
N (a) f(x) = y+C (b) f(x) = y(x+C)
H
(a) (c) f(x) = x+C (d) None of the above
NH2 NH2 83. If a,b and c are in AP, then determinant

x+ 2 x+3 x + 2a
x+3 x+4 x + 2b is
N x+4 x+5 x + 2c
(b) (a) 0 (b) 1
N N (c) x (d) 2x
84. If two events A and B. If odds against A are as
N
H2
2:1 and those in favour of A È B are as 3:1, then
1 3 5 3
NH (a) £ P( B) £ (b) £ P( B) £
2 4 12 4
(c) 1 3
(c) £ P( B) £ (d) None of these
4 5
NH
85. The value of 2 tan–1 (cosec tan–1 x – tan cot–1 x)
NH2 is
(a) tan–1 x (b) tan x
(c) cot x (d) cosec–1 x
EBD_7443
www.jeeneetbooks.in

WWW.IIT-NEET.XYZ

2013-10 Target VITEEE


86. The proposition ~ ( p Û q) is equivalent to (a) 4 £ x2 + y2 £ 64
(a) (p ~ q) Ù (q Ù ~ p) (b) x2+ y2 £ 25
(b) (p Ù ~ q) (q Ù ~ p)
(c) x2+ y2 ³ 25
(c) (p Ù ~q) Ù (q Ù ~ p)
(d) None of the above (d) 3 £ x2 + y2 £ 9
87. If truth values of P be F and q be T. Then, truth 94. A tower AB leans towards west making an angle
value of ~(~ p q) is with the vertical.The angular elevation of B,
(a) T (b) F the top most point of the tower is b as observed
(c) Either T or F (d) Neither T not F from a point C due east of A at a distance ‘d’
88. The rate of change of the surface area of a sphere from A. If the angular elevation of B from a point
of radius r, when the radius is increasing at the D due east of C at a distance 2d from C is r, then
rate of 2 cm/s is proportional to 2 tan can be given as
1 1 (a) 3 cot b – 2 cot (b) 3 cot – 2 cot b
(a) (b) (c) 3 cot b – cot (d) cot b – 3 cot
r r2
(c) r (d) r2 95. If and b are the roots of x2 – ax + b = 0 and if
89. If N denote the set of all natural numbers and R n + b n = V , then
n
be the relation on N × N defined by (a, b) R (c, d),
(a) Vn+1 = aVn + bVn-1
if ad (b + c) = bc (a + d ), then R is
(b) Vn+1 = aVn + aVn–1
(a) symmetric only
(c) Vn+1 = aVn – bVn–1
(b) reflexive only
(c) transitive only (d) Vn+1 = aVn-1 – bVn
(d) an equivalence relation 96. The sum of the series
90. A complex number z is such that arg æ 1 3r 7r 15r ö
çç + + + +...m terms÷÷is
å
n
r n
(–1) Crç ÷
æ z - 2 ö÷ z è2 2 ø÷
r 2r 3r 4r
çç 2 2
÷ = . The points representing this r =0
èç z + 2 ø÷ 3
complex number will lie on 2 mn -1 2 mn -1
(a) (b)
(a) an ellipse (b) a parabola 2 mn (2 n -1) 2 n -1
(c) a circle (d) a straight line
91. If a1, a2 and a3 be any positive real numbers, 2mn + 1
(c) (d) None of these
then which of the following statement is true? 2n + 1
(a) 3a1a2 a3 £ a13 + a23 + a33 97. The angle of intersection of the circles x2+ y2
– x + y – 8 = 0 and x2 + y2 + 2x + 2y – 11 = 0 is
a1 a2 a3
+ + ³3 æ19 ö
(a) tan–1 ççç ÷÷÷
(b)
a2 a3 a1 (b) tan –1(19)
è9ø
æ1 1ö
(a1 + a2 + a3 )ççç + + ÷÷÷ ³ 9
1 æ9ö
(c)
çè a1 a2 a3 ÷ø (c) tan–1 ççç ÷÷÷ (d) tan–1 (9)
è19 ø
æ1 98. The vector b = 3j + 4k is to be written as the sum
1 a ö
3

(d) (a1. a2 . a3 )ççç + + 3 ÷÷÷ ³ 27 of a vector b1 parallel to a = i + j and a vector b2


è a1 a2 a3 ÷ø perpendicular to a. Then b1 is equal to
92. If | x2 – x – 6 | = x + 2, then the values of x are 3 2
(a) –2, 2, –4 (b) –2, 2, 4 (a) (i + j) (b) (i + j)
2 3
(c) 3, 2, –2 (d) 4, 4, 3
93. The centres of a set of circles, each of radius 3, 1 1
lie on the circle x2 + y2 = 25. The locus of any (c) (i + j) (d) (i + j)
2 3
point in the set is
www.jeeneetbooks.in

WWW.IIT-NEET.XYZ

Solved Paper 2013 2013-11

99. If the points (x1, y1), (x2, y2) and (x3, y3) are
x 1
collinear, then the rank of the matrix (c) xe x C
é x1 y1 1ù
1
x x
ê x y 1ú will always be less than (d) xe x C
ê 2 2 ú 106. If f (x) = x – [x], for every real number x, where [x]
êë x3 y3 1úû
1
(a) 3 (b) 2 is the integral part of x. Then, ò f ( x ) dx is equal
(c) 1 (d) None of these -1
100. The value of the determinant to
1 cos ( ) cos (a) 1 (b) 2
cos ( ) 1 cos is
cos cos 1 1
(c) 0 (d)
2
2 2 2 2
(a) (b) 107. The value of the integral
(c) 1 (d) 0
é ù
1/ 2

êçæ x + 1÷÷ö + çæ x -1 ÷÷ö - 2ú dx is


2 2

ò–1/2 êêçè x -1÷ø çè x +1÷ø úú


1/2
101. The number of integral values of K, for which
ç ç
the equation 7 cos x + 5 sin x = 2K + 1 has a ë û
solution, is
(a) 4 (b) 8 æ4ö æ3ö
(c) 10 (d) 12 (a) log ççèç ø÷÷÷ (b) 4 log çççè ø÷÷÷
3 4
102. The line joining two points A(2,0), B(3,1) is
æ4ö æ3ö
(c) 4 log ççç ÷÷÷ (d) log ççç ÷÷÷
rotated about A in anti-clockwise direction
through an angle of 15°. The equation of the line è3ø è4ø
in the now position,is
4
(a) 3x–y–2 3 =0 108. If a tangent having slope of - to the ellipse
3
(b) x – 3 y – 2 = 0
x2 y 2
+ = 1 intersects the major and minor axes
(c) 3 x+y–2 3 =0 18 32
(d) x + 3y – 2 = 0 in points A and B respectively, then the area of
OAB is equal to (O is the centre of the ellipse)
103. The line 2x + 6y = 2 is a tangent to the curve (a) 12 sq units (b) 48 sq units
x2–2y2 = 4. The point of contact is (c) 64 sq units (d) 24 sq units
109. The locus of mid points of tangents intercepted
(a) (4, – 6) (b) (7, – 2 6)
x2 y2
(c) (2, 3) (d) ( 6,1) between the axes of ellipse + = 1 will be
a2 b2
104. The number of integral points (integral point
means both the coordinates should be integer) a2 b2 a 2 b2
(a) + =1 (b) + =2
exactly in the interior of the triangle with vertices x2 y2 x2 y2
(0, 0), (0, 21) and (21, 0) is
(a) 133 (b) 190 a 2 b2 a 2 b2
(c) + =3 (d) + =4
(c) 233 (d) 105 x2 y2 x2 y2

ò (1 + x - x ) e x+ x 110. If PQ is a double ordinate of hyperbola


—1
—1
105. dx is equal to
x2 y2
x x 1 - = 1 . Such that OPQ is an equilateral
(a) ( x 1) e C a2 b2
triangle, O being the centre of the hyperbola,
x 1
(b) ( x 1)e x C then the eccentricity ‘e’ of the hyperbola satisfies
EBD_7443
www.jeeneetbooks.in

WWW.IIT-NEET.XYZ

2013-12 Target VITEEE

2
e=
2 ìï p
(a) 1 < e < (b) ï
ï(1+ | sin x |)a /|sin x| , - < x < 0
3 3 ï 6
117. If f (x) = ïí b , x=0 , then
2 ï
ï p
e=
3
(d) e > ïe tan 2 x /tan 3 x
, 0< x <-
(c)
2 3 ïîï 6
111. The sides AB, BC and CA of a ABC have the value of a and b, if f is continuous at x = 0,
respectively 3, 4 and 5 points lying on them.The are respectively.
number of triangles that can be constructed
using these points as vertices is 2 3 2 2/3
(a) , (b) ,e
(a) 205 (b) 220 3 2 3
(c) 210 (d) None of these 3 3/ 2
(c) ,e (d) None of these
a + bx 2
112. In the expansion of , the coefficient of
ex 118. The domain of the function
x r is 1
f ( x) = + x + 2 is
a-b a - br log10 (1- x )
(a) (b)
r! r!
(a) ] 3, 2.5 2.5, 2[
a - br
(c) (-1) r (d) None of these (b) [ 2, 0 0, 1[
r!
(c) ]0, 1[
113. If n = (1999) !, then å log n x is equal to
1999
(d) None of the above
x=1 119. The solution of the differential equation
(a) 1 (b) 0 1 dy
(1+ y2) + ( x etan y ) 0, is
(c) 1999
1999 (d) –1 dx
114. P is a fixed point (a, a, a) on a line through the (a) (x – 2) = Ke tan 1
y
origin equally inclined to the axes, then any plane
through P perpendicular to OP, makes intercepts (b) 1 2 –1
2 xe tan y = e tan y + K
on the axes, the sum of whose reciprocals is
(c) xetan–1y = tan–1 y + K
equal to
(d) xe2tan–1y = etan–1y + K
3 120. If the gradient of the tangent at any point (x, y)
(a) a (b)
2a æ pö
of a curve which passes through the point ççç1, ÷÷÷
3a è 4ø
(c) (d) None of these
2
ïìï y æ öüï
2 y ֕
is íï - sin èççç ø÷÷ýï , then equation of the curve is
115. For which of the following values of m, the area
of the region bounded by the curve y = x – x2 îï x x þï
9 (a) y = cot –1 (log e x)
and the line y = mx equals
2
æ x ö÷
(a) – 4 (b) – 2 y = cot –1 ççloge ÷
èç e ø÷
(b)
(c) 2 (d) 4
116. If f : R R be such that f (1) =3 and f (1) = 6.
(c) y = x cot –1 (log e ex )
ì
ï ü
ï f (1 + x ) ï
1/ x
ï æ e ö÷
Then, lim íï ý equals to
ï y = cot –1 ççloge ÷
x ø÷
x®0 ï
î ï
þ (d)
f (1) èç
(a) 1 (b) e 1/2
(c) e 2 (d) e 3
www.jeeneetbooks.in

WWW.IIT-NEET.XYZ

SOLUTIONS
PART - I (PHYSICS) 2 8
and or = 8 m v=
= 2 m/s
1. (d) As we know, velocity of electromagnetic 4 4
So, the distance travelled by wave in t
1
wave, c = = 3 × 108 m/s second = vt = 2 × 5 = 10 m
0 0 6. (a) Gravitational intensity,
It is in dependent of amplitude of dv d k 2k
electromagnetic wave, frequency and I=
wavelength of electromagnetic wave. dx dx x 2 x3
2. (b) According to Moseley’s law v = a (z – b) 7. (d) For Cu wire, l1 = 2.2 m, r1 = 1.5 mm
= 1.5 × 10–3n
or v = a2 (z – b)2 Y1 = 1.1 × 1011 N/m2
c For steel wire, l2 = 1.6m, r2 = 1.5 mm
or = a2 (z – b)2 = 1.5 × 10–3 m
Y2 = 2.0 × 1011 N/m2
1 (z 2 1) 2 Let F be the stretching force in both the
(z1 1)2 wires the
2
Here 1 = ,, 2 = 4 , z1 = 11 and z2 = ? F l1
For Cu wire, Y1 =
r12 l1
1 (z 2 1) 2
4 (11 1) 2 Y1 r12 l1
F=
or (z2 – 1)2 = 25 or z2 = 6 l1
3. (b) As we know, conductivity,
1 1.1 10 11 22
( = × (1.5 × 10–3)2 × 0.5 × 103
= ene n nn ) 2.2 7
= 1.6 × 10–19 [0.36 + 0.17] (2.5 × 1019)] = 1.8 × 102 N
= 2.12 Sm–1 8kT
4. (d) If a radio receiver amplifies all the signal 8. (a) Mean speed, v = 0.92v rms
m
frequencies equally well, it is said to have
high fidelity. 3kT
rms speed, v rms
t x m
5. (b) Given, y = 3 sin Most probable speed vp
2 4
t x 2kT
= 3 sin = 0.816vrms
2 4 m
Comparing it with standard equation i.e., v p v vrms
2 9. (b) Let two balls collide at a height s from the
y = r sin (vt x)
ground after t second when second ball is
thrown upwards.
2 vt 2 x
= rsin Time taken by first ball to reach the point
of collision = (t + 2) s
2 v 1
We have, or v = s = 39.2 (t + 2) + (–9.8) (t + 2)2
2 4 2
EBD_7443
www.jeeneetbooks.in

WWW.IIT-NEET.XYZ

2013-14 Target VITEEE


= 39.2 (t + 2) – 4.9 (t + 2)2 ...(i) to time axis i.e., body comes to rest.
For second ball Change in velocity = dx = 2 m/s
1 Impulse = Change in momentum
s = 39.2t + (–9.8) t2 = mdv = 0.1 × 2 = 0.2 kg m/s
2
= 39.2t – 4.9 t2 ...(ii) force F
From eqs. (i) and (ii) 15. (d) Pressure =
area L2
39.2 (t + 2) – 4.9 (t + 2)2 = (39.2) t – 4.9t2
On solving we get, t = 3s p F 2 L
From Eq. (ii), = 4% + 2 (2%)
p F L
s = 39.2 × 3 – 4.9 × (3)2 = 117.6 – 44.1 = 73.5
m or percentage error, = 8%
16. (d) The situation can be shown as
F
10. (b) Magnetic flux, = B.A = .A
I
[M1L 1T 2 ]L2
= = [M1 L2 T–2 A–1] v0 R
[A.L] O O P
11. (c) Given, P = P0 exp (– t2) R R r
As P and P0 have the same units, therefore v
t 2 must be dimensionless for which
Here v0 = R
1 2
= T At. P, v = r
T2
= (R 2 R 2 ) 2R 2v0
M N
12. (d) Given, U = 6 17. (b) Using principle of conservation of linear
r r12
momentum,
du d M N
F= 3m × v = (m 60) 2 (m 60) 2
dr dr r 6 r12
6M 12N 6M 12N = m 60 2
= =
r7 r13 r7 r13 v = 20 2 m/s
For equilibrium position, F = 0 m1v1 m 2 v 2
6M 12N 18. (a) Common velocity, v =
6
2N m1 m 2
13 or r =
r7 r M
2 6 2 0
= = 3 m/s
M N M2 2 2
Hence, U =
(2N/ M) (2N / M) 2 4N Initial kinetic energy (E1)
13. (b) Total number of revolutions = area under 1 1 1
n–t graph = m1v12 m 2 v 22 2 (6) 2 = 36J
2 2 2
1 1800 600 1 600
= 8 8 16 1 1
2 60 60 2 60 = (m1 m 2 )V 2 (2 2)(3) 2 = 18J
2 2
= 120 + 80 + 80 = 280
14. (c) From the graph we can say, upto t = 2.0 s, Heat evolved = (36 – 18) J = 18 J
the body moves with a constant velocity 19. (a) From given condition
4 Gmm mv2 Gm 1 Gm
Slope of position-time graph = = 2m/s 2
,v
2 (2R) R 4R 2 R
After t = 2.0 s, position-time graph is parallel
www.jeeneetbooks.in

WWW.IIT-NEET.XYZ

Solved Paper 2013 2013-15

20. (d) At the centre O of the square due to four or K1 = 4K2


equal charges q at the corners, potential
a U 2 l2 1
q q U1 22 4
or U1 = 4U2
O 23. (b) To obtain OR gate from NAND gates we
a a need two NOT gates obtained from NAND
gates and one NAND gate as figure.

q q A
a
4q 2q Y
V= 4 (a 2) / 2 =
0 0a
W0 =– W B
0

2q Boolean expression = A.B. = A B


= – (–q)V =
0a = A + B OR gate
21. (a) In the steady state, current through 02 I
capacitor 24. (b) Magnetic field B =
4 r
10V 2V R C 4 4V Here, 2 =
A I B 0 I
2µF B=
4 r
2 25. (a) Since axes are perpendicular so mid point
lies on axial line of one magnet and on
equitorial line of other magnet
I
2M 7
0 10 2 1
D B1 = 3 = 3 = 2 × 10–7 T
4 d 1
Using Kirchhof’s voltage law to the circuit
ACD 7
0 M 10 1
We have, 10 – 2 + 1 × R + 1 × 2 = 0 and B2 = = = 10–7 T
4 d 3 3
or R = 10 1
Potential difference across C and D
As B1 B2
VC – VD = 2 × 1 = 2V
As VD = 0V Resultant magnetic field
So, VC = 2V
Potential difference across capacitor
= B12 B22 5 10 7 T
= 4 – 2 = 2V 26. (a) In the given circuit, two condensors and
Charge on capacitor Q = CV = 2 F × 2 = 4 C the inductor are in series.
1 Ls = L + L = 2L
22. (a) KE of an electron in nth orbit : Kn 1 1 1 2 C
n2 Cs
and C C C C 2
and PE of an electron in nth orbit : s
1 Natural frequency of the circuit
Un
n2 1 1
When an electron passes from state v= 2 L s Cs 2 2L C/2
n = 2 to n = 1
1
K2 l2 1 =
2 LC
K1 22 4
EBD_7443
www.jeeneetbooks.in

WWW.IIT-NEET.XYZ

2013-16 Target VITEEE


27. (a) In the beginning due to gravity pull, the 32. (b) In an isobaric process, p = constant
lead shot will be accelerated and hence will Hence, V T
move, with increasing velocity for some time
When the viscous force balance the gravity nR
i.e., V = T
pull, then the shot will move with constant P
velocity. As in the beginning, the velocity
of shot is not fully linear with the effective 1
distance covered by the shot. V–T graph is a straight line with slope
p
(slope)2 > (slope)1
28. (b) Refractive index of a medium = p2 < p1
0 0
33. (a) Total energy
1 1 1
29. (c) Lens formula, n1 n2
v u f U= 2 RT 4 RT
2 2
u u
or 1 For O2, n = 5 and for Ar, n 2 = 3
v f
5 3
1
1
u U= 2 RT 4 RT = 11RT
T
2 2
m f
34. (b) Given, speed of each pulse = 2 cm/s
1 1 Therefore distance travelled by both pulses
or u 1
m f in 2s = 4 cm toward each other. On their
this is the equation of a straight line whose superposition, the resultant displacement
slope at every point will be zero.
1 b c Hence, total energy will be purely kinetic.
f= 35. (d) Time interval between two successive
f c b
30. (d) Amplitudes A1 and A2 are added as vectors. maxima = time interval between two
Angle between the two vectors is the phase 1
difference ( 1 – 2) between them. successive beats = n n
Resultant wave, 1 2

R= A12 A 22 2A1A 2 cos( 2s cos 0 2s


1 2) 36. (b) Here, h = ...(i)
r g r g
hc
31. (a) Here, case (i) e(3V0) = 0 ...(i) According to question,
h 2scos
...(ii)
hc 2 rpg
Case (ii) eV0 = 0 ...(ii)
2 Dividing eq. (ii) by (i) we get,
From eqs. (i) and (ii),
1
3hc hc cos
3 2
0 0
2 or = 60°
3hc hc hc 37. (a) Given, V1 = V2 = 300V; V3 = ?, i = ?
or 3 0 0 = 2 0
2 2
As, V = V32 (V1 V2 ) 2
hc
or, 0= 4 220 = V32 V3 V3 = 220V
Threshold wavelength
V3 220
hc
hc I= = = 2.2A
0= = 4 4 R 100
0 hc
www.jeeneetbooks.in

WWW.IIT-NEET.XYZ

Solved Paper 2013 2013-17

38. (b) We have, W = – MB(cos 2 – cos 1)


So, W1 = –MB (cos 90° – cos0°) = MB *
42. (a) COOH NaHCO3
1
and W2 = – MB (cos 60° – cos 0°) = MB
2
As W1 = nW2 *
COONa CO2 H 2O
W1 MB
n= =2
W2 1 N=NCl
MB
2
Cu
43. (d) H 3 PO 2 H 2O
0A
39. (b) Capacitance, C =
d
As one–fourth of capacitor is filled with
dielectric of constant K, then, + N2 + HCl
K 0A / 4
C1 = 44. (c)
d
CH2COOCH P/Br2 BrCHCOOCH
3A / 4
and C2 = HVZ-reaction
d CH2COOCH CH2COOCH
A
Both C1 and C2 are in parallel.
K 0A 3 0A
Alc. KOH CHCOOCH
CP = C1 + C2 =
4d 4d CHCOOH
B (maleic acid)
0A C
= (K + 3) (K 3)
4d 4
40. (c) The equivalent circuit of the given circuit COOH
is CHCOOH COOH
HO—C—H CF3CO3H O3
H2O
2
5 C 3 H—C—OH CHCOOH COOH
A B B C
COOH
Alkaline
KMnO4

(oxalic acid)
D
d(+) and l(–)
5 8 3 tartaric acid
(racemic mixture) COOH

D H—C—OH
H—C—OH
This is a balanced Wheatstone bridge.
Therefore, the arm CD becomes in effective. COOH
E
Hence 5 and 3 are in series and they meso-tartaric acid
together are in parallel with (5 + 3)
(5 3) (5 3) 45. (d) When two cyclic forms of a carbohydrate
Net resistance = 4
(5 3) (5 3) differ in configuration only at hemiacetal
carbons, they are said to be anomers. Thus,
PART - II (CHEMISTRY) anomers are cyclic forms of carbohydrates
that are epimeric at hemiacetal carbon and
41. (a) Benzoin condensation is performed by
aromatic aldehydes (i.e., compounds in this carbon (C–1 of aldose) is called
which –CHO group is directly attached with anomeric carbon, e.g.,
benzene ring).
EBD_7443
www.jeeneetbooks.in

WWW.IIT-NEET.XYZ

2013-18 Target VITEEE


50. (c) Oxidation of Ketones, yield secondary
Anomeric carbon alcohol
R [O] R
HO—C—H CHOH C=O
H—C—OH R K2Cr2O 7/H2SO4 R
H—C—OH H—C—OH
R R
HO—C—H PCC C= O
HO—C—H CHOH
R CH2Cl2 R
H–C O H—C—OH
CH2OH R CH3
H—C O CHOH + C=O
-D-glucose
CH2OH R CH3
-D-glucose [(CH 3)3CO]3Al R
C=O
46. (c) – amino acids are bifunctional organic R
compounds. They contain a basic amino
group (–NH2) on the –carbon and one 51. (c) Since, X NaHCO3 CO 2
acidic carboxyl group (–COOH). Hence, it must contain —COOH group.
47. (d) [CH3COOH] = millimoles of CH3COOH
Achiral carbon
= 0.1 × 10 = 1.0
CH2OH CH3
[CH3COONa] = millimoles of CH3COONa
LiAlH 4
= 0.1 × 20 = 2.0 H—C—COOH H—C—CH2OH
From, Henderson Hasselbalch equation,
CH3 CH2OH
[conjugate base]
pH = pKa + log 52. (a)
[acid]
2 OH2
= 4.74 + log = 4.74 + 0.30 = 5.04 OH conc. H2SO4
1

–H2O
48. (b) (Ag + ) = tr an sport number of

Ag+ × A(AgNO3 )
= 0.464 × 133.3 = 61.9 –1 cm2 equiv–1
By Kohlrausch’s law H can be lost only from this
carbon (3° carbocation (X)
(AgNO3 ) 1, 2-methyl shift
(Ag ) (NO3 )

(NO3 ) (AgNO3 ) (Ag )


(3° carbocation (Y))
= 133.3 – 61.9 = 71.4 –1 cm2 equiv–1
49. (d) Y is less soluble than (X) due to lack of
symmetry Chiral carbon. This is reduced
OCH3 OCH 3 to —CH2OH
Activating
group 53. (b)
(CH3)3CCl/AlCl3
Friedel-Crafts CH2OH CH2 CH2
KHSO4 –
C(CH 3) 3 CHOH CH CH
OCH3 CH2OH CHO CHO
Cl Cl CH2OH
Cl2/FeCl3 HBr, –
OH–Cl CHCl
CHO
C(CH3) 3 C(CH3)3
www.jeeneetbooks.in

WWW.IIT-NEET.XYZ

Solved Paper 2013 2013-19

54. (a)
O O
H2S2O4
O HO—S—S—OH
C O O
O C H2SO5 HO—S—O—O—H
O O
O C O O
H + H
conc.
H2S2O8 HO—S—O—O—S—OH
H2 SO4
O O
59. (d) Volume of one molecule
OH OH OH OH 4 3 4
(phenolphthalein) = r = (1.54 × 10–8)3 cm3
3 3
NaOH (Phenolphthalein = 1.53 × 10–23 cm3
test) Volume of molecules in 1.65 g Ar
Pink colour
1.65
= N0 1.53 10 23 = 0.380 cm3
55. (d) 40
Volume of solid containing 1.65 g Ar = 1 cm3
NaNO2 /HCl
Empty space = 1 – 0.380 = 0.620
NH2 0°C
N NCl Per cent of empty space = 62%
60. (d) In adiabatic expansion
1
CuCN
CN T2 V1
T1 V2
Y
+ for CO2 (triatomic gas) is,
H2O/H
= 1.33
0.33
COOH 150 10
300 V2
56. (a) According to Van–Arkel method, pyrolysis 0.33
of BI3 is carried out in the presence of red 1 10
hot W or Ta filament. 2 V2
2BI3 Red hot w or Ta filament 2B + 3I2 3
1 10
57. (c) NH 4 Cl(s) NH 3 HCl 2 V2
Graham’s law of diffusion says, lighter gas
will diffuse most rapidly. Therefore, NH3 1 10
will be (mol. wt. = 17) diffused rapidly than 8 V2
HCl. (mol. wt. = 36.5). V2 = 80 L
58. (b) Peroxy acids contain —O—O— linkage.
61. (c) H
O RCOOR H2O RCOOH R OH
At t = 0, a 0 0
H2S2O3 H—O—S—O—H At time t, a – x x x
O At time , a – a a a
At t = 0, V0 = volume of NaOH due to H+ (catalyst)
Vt = x + V0
EBD_7443
www.jeeneetbooks.in

WWW.IIT-NEET.XYZ

2013-20 Target VITEEE


V = a + V0 66. (d)

a
If ester is 50% hydrolysed then, x =
2 H Br –

O O O O 1,4-addition
a
or Vt = V0
2 Br Br
or a = 2Vt – 2V0
V = 2Vt – 2V0 + V0 Enolisation
O OH HO OH
= 2Vt – V0
62. (b) Energy values are additives.
E = E1 + E2 67. (a)
hc hc hc
Cl Cl Cl
1 2

1 1 1 Cl
Cl' Cl'
300 760 2 cis cis trans
p. Q. R
1 1 1
2 300 760 O
= 495.6 nm = 496 nm 68. (a)
R—C—O—R
63. (a) I. Fe2+ = Ar –O oxygen atom can donate lone pair of
6 electron more easily, therefore, it is more
3d basic than –oxygen.
4 unpaired electrons,
Coloured ion, 69. (d) When two molecules of an ethylacetate
undergo condensation reaction, in presence
II. Ni2+ = Ar of sodium ethoxide involving the reaction
8 is called as Claisen condensation and
3d
2 unpaired electrons, product is a –keto ester.
Coloured ion O
III. Al3+ = [Ne] (i) CH3O–
No unpaired electron in 3d, colourless ion. 2CH3CH2COCH3 +
(ii) H

64. (b) CH3C N: CH3C = N :


O O
electrophilic nucleophilic
65. (c)
NO2 CH3
conc. HNO3 + conc. H 2SO 4
Mechanism
Step I
X
NO2 O O
CH3CHCOCH3 CH3CHCOCH3 + CH3OH
Cl2/FeCl 3
H –
CH3O
Cl
Y
www.jeeneetbooks.in

WWW.IIT-NEET.XYZ

Solved Paper 2013 2013-21

Step II 71. (c) Keto group is more reactive for addition of


Grignard reagent.
O O O O
|| ||
CH3CHCOCH
– 3 + CH3CH2COCH3 CH3 C CH3 CH 2 C OCH 2 CH3

O
(i)CH3MgBr(one mole)
CH3CH2COCH3 (ii) H3O
CH3CHCOCH3
CH3 CH3
O
CH3—C—CH2CH2 CH3—C—CH2
Step III
OH C=O O CH2

O
O O C2H5O C
CH3CH2C—OCH3
CH3CH2CCHCOCH3
– O
CH3CHCOCH3 + CH3O
CH3 72. (b) E cell E ox E red
O
70. (b) B is a tertiary alcohol based on given = E Co / Co 2 E
Ce4 / Ce3
properties.
1.89 = – (–0.28) + E Ce 4 / Ce3
H3O
(a) CH2 CH2 CH3CH2MgI
E = – 1.879 – 0.28 = 1.61 V
Ce 4 / Ce3
CH 3CH 2CH 2 CH 2OH 73. (c) Due to electrolysis
1 alocohol
2H 2 O(l ) 2e H 2 ( g ) 2OH ( aq)
O
2Cl (aq) Cl2 ( g ) 2e
(b) CH3CH2CCH3 CH3CH2 Mgl
2H2 O(l ) 2Cl ( aq) H2 ( g ) Cl2 ( g )
OH + 2OH– (aq)
CH3CH2CHCH2CH3 OH– formed = NaOH formed = Z i t
H3O E
CH3 = i t
3°alcohol 96500
40
H 3O = 30 1 60 60 = 44.77 g
(c) CH 3CHO CH 3 CH 2 Mgl 96500
44.77
CH3 CH CH2 CH3 = = 1.12 mol
40
-Carbon atom OH
1
2° alcohol Cl 2 formed = mol of NaOH
2
(d) HCHO CH 3 CH 2 Mgl H3 O 1.12
= = 0.56 mol
2
CH 3 CH 2 CH 2 OH = 0.56 × 22.4 L at STP = 12.54 L
1 alcohol
EBD_7443
www.jeeneetbooks.in

WWW.IIT-NEET.XYZ

2013-22 Target VITEEE

74. (b) KB = Ae E a / RT R H Z2
En =
= 1012 × 4.35 × 10–8 n2
= 4.35 × 104 s–1 where, Z = atomic number, n2 = degeneracy
kA R H (1)2
Also equilibrium constant, k = k = 104 For H–atom, En =
B n2
kA = k B × 104 = 4.35 × 108 s–1 RH RH
dE 9 n2
75. (a) G = H – nFT dT n2 = 9
P
and G = T–T S 78. (d)
Compound Symbol/ Uses
S dE
formla
nF dT P
A. Dry ice CO2 Referigerant
96.5 dE for preserving
or food
2 96500 dT P B. Semicon Ge Electronic diode
3
ductor and triode in
dE cell 1 10 computer
= 5 × 10–4 VK–1
dT P 2 C. Solder Sn/Pb Joining circuits
D. TEL (C2H5)4 Pb Antiknocking
76. (c) We have to compare wavelength of
compound for
transition in the H–spectrum with the Balmer
petroleum
transition n = 4 to n = 2 of He+ spectrum.
products
H= He 79. (c)

2 1 1 1 1
R H ZH R H Z2
n12 n 22 He 2 2
42 N
H N
1 1 1 1 NH2 NH2; N
1 4 N
n12 n 22 4 16 (a) tridentate (b) tridentate

1 1 4 1 NH2 CH3
4
n12 n 22 16 NH C=O

1 1 3 NH CH2

n12 n 22 4 C=O
NH2
If n1 = 1, then n2 = 2, 3, .... CH3
(c) tetradentate (d) bidentate
For first line n2 = 2, n1 = 1
1 1 1 1 3 80. (b) Effective atomic number EAN = Atomic
number – oxidation number + 2 ×
12 22 1 4 4
coordination number
Hence, trnasition n2 = 2 to n1 = 1 will give For [Al (C2O4)3]3–
spectrum of the same wavelength as that Z = 13
of Balmer transition, n2 = 4 to n1 = 2 in He+. ON = 3
77. (b) Energy of the electron in the nth orbit in CN = 6
terms of RH is EAN = 13 – 3 + 2 × 6 = 22
www.jeeneetbooks.in

WWW.IIT-NEET.XYZ

Solved Paper 2013 2013-23

PART - III (MATHEMATICS) P(B) P (A B) =


3
81. (a) A = { x : |x| < 3, x I} 4
A = {x : – 3 < x < 3, x I} = {–2, –1, 0, 1} 5 3
Also, R = {(x, y) : y = |x|} P(B)
12 4
R = {(–2, 2), (–1, 1), (1, 1), (0, 0), (2, 2)}
85. (a) 2 tan –1 (cosec tan–1 x – tan cot–1 x)
2
dy yf (x) y
82. (b)
dx f (x) –1 1 x2
= 2 tan–1 cosec cosec
yf (x) dx – f(x) dy = y2 dx x

yf (x)dx f (x)dy
= dx 1 1
y2 tan tan
x
f (x)
d = dx 1 x2 1
y = 2 tan–1
x x
On integration, we get
f (x)
=x+C 1 x2 1
y = 2 tan–1
f (x) = y (x + C) x

x 2 x 3 x 2a sec 1
= 2 tan–1 (put x = tan )
83. (a) Let = x 3 x 4 x 2b tan
x 4 x 5 x 2c
1 cos
= 2 tan–1
x 2 x 3 x 2a sin
1
= 0 0 2(2b a c)
2 2sin 2
x 4 x 5 x 2c
=2 tan–1 2
(using R2 2R2 – R1 – R3)
2sin .cos
But a, b and c are in AP using 2b = a + c, we 2 2
get
x 2 x 3 x 2a = 2 tan–1 tan
1 2
= 0 0 0 =0
2
x 4 x 5 x 2c 1
= 2. tan x
Since, all elements of R2 are zero. 2
86. (b) ~ (p q) ~ [( p q) (q p)]
1 3
84. (b) P (A) = , P (A B) = ~ (p q) (~ (q p))
3 4
P (A B) = P(A) + P (B) – P (A B) ( De–Morgan’s law)
P(A) + P (B) (p ~ q) (q ~ p)
3 1 87. (b)
P(B) p q ~ p ~ p q ~ (~ p q )
4 3
F T T T F
5
P(B)
12
Also, B A B Truth value of ~ (~ p q) is F..
EBD_7443
www.jeeneetbooks.in

WWW.IIT-NEET.XYZ

2013-24 Target VITEEE


88. (c) Surface area of sphere, 91. (d) We know that, GM HM
S = 4 r2
3
dr (a1.a2.a3)1/3
and =2 1 1 1
dt
a1 a2 a3
ds dr
4 2r 8 r 2 16 r
dt dt 27
(a1.a2.a3) 3
ds 1 1 1
r
dt a1 a2 a3
89. (d) For (a, b), (c, d) N × N
3
(a, b) R (c, d) 1 1 1
ad (b + c) = bc (a + d) (a1.a2.a3) 27
a1 a2 a3
Reflexive : ab (b + a) = ba (a + b), ab N
(a, b) R (a, b) 92. (b) |x2 – x – 6| = x + 2, then
So, R is reflexive, Case I : x2 – x – 6 < 0
Symmetric : ad (b + c) = bc (a + d) (x – 3) (x + 2) < 0
bc (a + d) = ad (b + c) –2<x<3
cd (d + a) = da (c + b) In this case, the equation becomes
(c, d) R (a, b) x2 – x– 6 = –x – 2
So, R is symmetric. or x2 – 4 = 0
Transitive : For (a, b), (c, d), (e, f) N × N x=±2
Let (a, b) R (c, d), (c, d) R (e, f) Clearly, x = 2 satisfies the domain of the
ad (b + c) = bc(a + d), cf (d + e) = de (c + f) equation in this case. So, x = 2 is a solution.
adb + adc = bca + bcd ...(i) Case II : x2 – x – 6 0
and cfd + cfe = dec + def ...(ii) So, x – 2 or x 3
On multiplying eq. (i) by ef and eq. (ii) by In this case, the equation becomes
ab and then adding, we have x2 – x – 6 = 0 = x + 2
adbef + adcef + cfdab + cfeab
i.e., x2 – 2x – 8 = 0 or x = –2, 4
= bcaef + bcdef + decab + defab
Both these values lie in the domain of the
adcf (b + e) = bcde (a + f)
equation in this case, so x = –2, 4 are the
af (b + e) = be (a + f)
roots.
(a, b) R (e, f)
So, R is transitive. Hence, roots are x = –2, 2, 4.
Hence R is an equivalence relation. 93. (a) Let (h, k) be any point in the set, then
equation of circle is
z 2 (x – h)2 + (y – k)2 = 9
90. (c) arg
z 2 3
8
x 2 iy
arg
x 2 iy 3
2
arg (x – 2 + iy) – arg (x + 2 + iy) =
3
y 1 y
tan–1 tan
x 2 x 2 3
(h, k) lies on x2 + y2 = 25, then h 2 + k2 = 25
4y 2 Distance between the two circles 8
3
x2 y2 4 2 h2 k2 8
3(x 2 2
y ) 4y 4 3 = 0 4 h 2 + k2 64
Locus of (h, k) is 4 (x2 + y2) 64
which is an equation of a circle.
www.jeeneetbooks.in

WWW.IIT-NEET.XYZ

Solved Paper 2013 2013-25

94. (c) By m – n theorem at C


1 1 1
(d + 2d) cot = d cot – 2d cot (90° + ) + ... upto m terms
2n 4n 8n
Y(N)
1 1
1
2 n
2 mn 2mn 1
B =
1
1 2mn (2n 1)
n
2

(W)X' X(E) 97. (c) Angle of intersection between two circles


A d C 2d D is given by

17 10
r12 r22 d 2 13
cos = 2 4
Y'(S) 2r1r2 17
2 . 13
2
3d cot = d cot + 2d tan
3 cot = cot + 2 tan
2 tan = 3 cot – cot
2 2
95. (c) Multiplying x2 – ax + b = 0 by xn–1, we get 1 1
here, r1 8
xn+1 – axn + bxn–1 = 0 ...(i) 2 2
are roots of x2 – ax + b = 0, therefore
they will satisfy (i). 17
,
Also, n+1 – a n + b n–1 = 0 ...(ii) 2
and n+1 – a n + b n–1 = 0 ...(iii) r2 13
On adding eqs. (ii) and (iii), we get
10
( n+1 + n+1) – a ( n + n) and d c1c 2
+ b( n–1 + (n–1)) = 0 2
Vn+1 – aVn + bVn–1 = 0 ( n + n = Vn)
Vn+1 = aVn – bVn–1
19
n cos =
( 1)r .n Cr 442
96. (a)
r 0
9
r r or tan =
1 3 7 19
...upto m terms
2r 22r 23r
9
= tan–1
19
n
1 n
3r
( 1)r .n Cr ( 1)r .n C r .
2r 22r 98. (a) b1|| a b1 = a (i + j)
r 0 r 0
b2 = b – b1 = (3 – a) i – aj + 4k
n
7r Also, b2 . a = 0
( 1)r .n C r . ...
r 0 23r 3
(3 – a) – a a=
2
n n n
1 3 7
1 1 1 3
2 4 8 b1 = (i + j)
2
... upto m terms
EBD_7443
www.jeeneetbooks.in

WWW.IIT-NEET.XYZ

2013-26 Target VITEEE


Thus, slope of new line is tan (45° + 15°)
x1 y1 1
= tan 60° = 3
x2 y2 1
99. (b) The given matrix is , ( it is rotated anti-clockwise, so the angle
x3 y3 1
will be 45° + 15° = 60°)
using R2 R2 – R1, R3 R3 – R1
Y C
x1 y1 1
B (3, 1)
= x2 x1 y2 y1 0 =0
x3 x1 y3 y1 0

°
15
( points are collinear i.e., area of triangle = 0)
45°
X
x2 x1 y2 y1 A (2, 0)
x3 x1 y3 y1 = 0
Hence, the equation is y = 3 x + c
So, the rank of matrix is always less than 2.
But it passes through (2, 0),
100. (d) On solving the determinant, we have
1 (1 – cos2 ) – cos ( – ) [cos ( – ) So, c = – 2 3
– cos . cos ] + cos [ cos . cos ( – ) Thus, required equation is y = 3x 2 3
–cos ] 103. (a) Solving the equation of line and curve, we
= 1 – cos2 – cos2 – cos2 ( ) get
+ 2 cos . cos .cos ( – ) 2
= 1 – cos2 – cos2 + cos ( – ) 2 2x
x2 2 =4
[2cos .cos – cos( – )] 6
= 1 – cos2 – cos2 + cos ( – )cos( + )
1
[cos ( + ) + cos ( – ) – cos ( – )] x2 – × 4 (1 + x2 – 2x) = 4
3
= 1 – cos2 – cos2 + cos2 . cos2
3x2 – 4 – 4x2 + 8x = 12
– sin2 . sin2 x2 – 8x + 16 = 0
= 1 – cos – cos (1 – cos2 )
2 2
(x – 4)2 = 0 x = 4
– sin2 .sin2
2 2
and 6 .y = 2 – 2 (4) = – 6
= 1 – cos – cos sin – sin2 .sin2
2

= (1 – cos2 ) – sin2 sin2 + cos2 ) y=– 6


= sin2 – sin2 1) = 0 Point of contact is (4, – 6 ).
101. (b) – 2 2 2 2 104. (b) x + y = 21
7 5 (7cos x 5sin x) 7 5
The number of integral solutions to the
74 (2K 1) 74 equations are x + y < 21, i.e., x < 21 – y

8.6 (2K 1) 8.6 B(0, 21)


– 9.6 2K 7.6
– 4.8 K 3.8
So, integral values of K are
–4, –3, –2, –1, 0, 1, 2, 3 (eight values)
1 C A
102. (a) Slope of AB = (0, 0)
1 (21, 0)
tan = m1 = 1 or = 45°
www.jeeneetbooks.in

WWW.IIT-NEET.XYZ

Solved Paper 2013 2013-27

Number of integral coordinates 0 x 1/2 x


= 19 + 18 + ... + 1 = 4 2
dx 4 dx
1/2 1 x 0 1 x2
19(19 1) 19 20
= = 190
2 2 = 2{log(1 x 2 )}0 1/2 2{log(1 x 2 )}1/2
0

x 1 1 1
105. (c) (1 x x 1 )ex dx
= 2log 1 2log 1
4 4
x x 1 1 x 1
= [x.e 1 2
ex ]dx 3 4
x = 4log 4log
4 3
[ xf (x) f (x)dx xf (x) C] 108. (d) Let P (x1, y1) be a point on the ellipse.

x 1 x 1 x2 y2
(1 x x 1)e x dx xe x C =1
18 32
106. (a) f(x) = x – [x], – 1 x < 0
f (x) = x + 1 x12 y12
When 0 x < 1 = 1... (i)
18 32
f(x) = x
The equation of the tangent at (x1, y1) is
1 0 1
f (x)dx f (x)dx f (x) dx xx1 yy1
= 1. This meets the axes at
1 1 0 18 32

0 1 18 32
= (x 1)dx x dx A x ,0 and B 0, y . It is given that
1 1
1 0
4
0 1 slope of the tangent at (x1, y1) is –
x2 x2 3
= 2 x
2
1 0 x1 32 4
So, – . =–
18 y1 3
( 1) 2 1
=0 1 1
2 2 x1 3
y1 = 4
2 2 1/ 2
1/ 2 x 1 x 1
107. (c) 2 dx x1 y1
1/ 2 x 1 x 1 =K (say)
3 4
x1 = 3K and y1 = 4K
2 1/ 2
1/ 2 x 1 x 1 Putting x1, y1 in (i), we get
= dx
1/ 2 x 1 x 1 K2 = 1
1
1/ 2 4x Area of OAB = OA.OB
dx 2
= 1/ 2 x2 1
1 18 32 1 (18)(32) 24
= . . = = 2
0 4x 1/ 2 4x 2 x1 y1 2 (3K)(4K) K
= dx dx
1/ 2 1 x2 0 1 x2 = 24 sq units ( K2 = 1)
EBD_7443
www.jeeneetbooks.in

WWW.IIT-NEET.XYZ

2013-28 Target VITEEE


109. (d) Let mid–point of part PQ which is in 4b2 tan2
= b2tan2a2sec2
between the axis is R (x 1 , y1 ), then 3b2 tan2 = a2 sec2
coordinates of P and Q will be (2x1, 0) and 3b2 sin2 = a2
(0, 2y1), respectively. 3a2 (e2 – 1) sin2 = a2
3 (e2 – 1) sin2 = 1
x y
Equation of line PQ is =1
2x1 2y1 1
2
= sin2 < 1,( sin 2 < 1)
3(e 1)
y1
y= x 2y1 1 4
x1 1
3 e2 1 e2
e 2
1 3 3
If this line touches the ellipse
2
x2 y2 e>
=1 3
a 2 b2
111. (a) There are 3 + 4 + 5 = 12 points in a plane.
then it will satisfy the condition,
The number of required triangles
c2 = a2m2 + b2
= (The number of triangles formed by these
2 12 points) – (The number of triangles formed
y1
So, (2y1 )2 = a2 b2 by the collinear points)
x1 = 12C3 – (3C3 + 4C3 + 5C3)
= 220 – (1 + 4 + 10) = 205
a 2 y12 112. (c) (a + bx) e–x
4y12 = b2
x12 = (a + bx)

x x2 x3 xn
a 2
b 2
a 2
b 2 1 ... ( 1)n ...
4= =4 1! 2! 3! n!
x12 y12 x12 y12
The coefficient of xr = a.
Required locus of (x1, y1) is
( 1)r ( 1)r 1 ( 1)r
a 2
b 2 b = (a br)
r! (r 1)! r!
2 =4
x y2
1999
110. (d) Let P (a sec , b tan ), Q (a sec , – b tan ) 113. (a) log n x
be end points of double ordinates and x 1
(0, 0) is the centre of the hyperbola. = log(1999)! 1+ log(1999)! 2 + ... + log(1999)! 1999
So, PQ = 2b tan = log(1999)! (1.2.3.... 1999)
= log(1999)! (1999)! = 1
114. (d) Since, the line is equally inclined to the axes
P
(0, 0) (a sec , b tan ) and passes through the origin, its direction
O ratios are 1, 1,1.
x y z
Q So, its equation is .
1 1 1
(a sec , – b tan )
A point P on it is given by (a, a, a). So,
equation of the plane through P (a, a, a)
OQ = OP = a 2 sec 2 b2 tan 2 and perpendicular to OP is
Since, OQ = OP = PQ 1 (x – a) + 1 (y – a) + 1 (z – a) = 0
( OP is normal to the plane)
www.jeeneetbooks.in

WWW.IIT-NEET.XYZ

Solved Paper 2013 2013-29

i.e., x + y + z = 3a 1
lim [log f (1 x) log f (1)]
x y z 116. (c) ex 0 x
=1
3a 3a 3a
f (1 x) / f (1 x)
Intercepts on axes are 3a, 3a and 3a, lim
ex 0 1
therefore sum of reciprocals of these =
intercepts. f (1) / f (1)
=e e613 e2
1 1 1 1
=
3a 3a 3a a
117. (b) (1 sin x )
a/ sin x
, x 0
115. (b) The equation of curve is y = x – x2
6
x2 – x = y
f (x) b , x 0
2
1 1 e tan 2x/tan 3x ,0 x
x y
2 4 6

1 1 For f (x) to be continuous at x = 0


which is a parabola whose vertex is ,
2 4
lim f (x) = f (0) = lim f (x)
x 0 x 0

Y 1 1
, lim (1 + |sin x|)a / | sinx |
2 4 y = mx x 0

a
lim |sin x|
x 0 |sin x|
X =e = ea
O (1 – m)
tan 2x / tan 3x
Now, lim e
x 0
Hence, finding the point of intersection of
the curve and the line, tan 2x tan 3x
2x 3x
x – x2 = mx x (1 – x – m) = 0 2x 3x
= lim e
i.e., x = 0 or x = 1 – m x 0

9 1 m
(x x 2 mx)dx = lim e
2/ 3
e
2 /3
2 0
x 0

1 m Since, f(x) is continuous at x = 0.


x2 x3 x2
= 2 m
3 2 2
0 ea = e2/3 a=
3
(1 m)2 (1 m)3 (1 m)3 and b = e2/3
= (1 – m)
2 3 6 118. (b) x + 2 0, i.e., x –2 or –2 x
log10 (1 – x) 0
6 9
(1 – m)3 = = 27 1–x 1 x 0
2
Again, 1 – x > 0
1 – m = (27)1/3 = 3
1>x x<1
m=–2
Combining all the results for values of x, we
Also, (1 – m)3 – (3)3 = 0
get
(1 – m)3 = 33 1 – m = 3
or m = – 2 – 2 x < 0 and 0 < x < 1
EBD_7443
www.jeeneetbooks.in

WWW.IIT-NEET.XYZ

2013-30 Target VITEEE

tan 1
y dy dv
119. (b) (1 + y2) + (x e ) =0 v+x = v – sin2 v
dx dx
dx 1 dx
(1 + y2) x e tan y
– cosec2v dv =
dy x
1
Integrating both sides, we get
dx x e tan y
dx
dy 1 y2 (1 y 2 ) – cosec 2 vdv
x
1 cot v = log x + C
dy 1
1 y2
IF = e e tan y y
cot = log x + C
x
1
e tan y 1
.e tan y .dy
1
tan y
x. e = 2 Curve passes through the point 1,
1 y 4
1 C=1
tan 1 y e 2tan y
x(e ) c y
2 cot = log x + logee
x
1 1
2xe tan y
e 2 tan y
K y
cot log xe
dy y y x
120. (c) sin 2 y = x cot–1 (log xe)
dx x x

dy dv
Put y = vx v x
dx dx
www.jeeneetbooks.in

WWW.IIT-NEET.XYZ

VITEEE
SOLVED PAPER 2012

(a) In forward biasing the voltage across R is


PART - I (PHYSICS) V
1. A square loop, carrying a steady current I, is (b) In forward biasing the voltage across R is
placed in horizontal plane near a long straight 2V
conductor carrying a steady current I1 at a (c) In reverse biasing the voltage across R is V
distance of d from the conductor as shown in (d) In reverse biasing the voltage across R is
figure. The loop will experience 2V
4. If the binding energy per nuclear in Li7 and He4
l1 nuclei are respectively 5.60 MeV and 7.06 MeV,
then energy of reactor
d l Li7 + P 2 2He4 is
(a) 19.6 MeV (b) 2.4 MeV
(c) 8.4 MeV (d) 17.3 MeV
5. The graph between the square root of the
frequency of a specific line of characterstic
spectrum of X-ray and the atomic number of the
l ttarget will be
(a) a net repulsive force away from the
conductor
(b) a net torque acting upward perpencicular
to the horizontal plane V V
(a) (b)
(c) a net torque acting downward normal to
the horizontal plane Z Z
(d) a net attractive force towards the conductor
2. The threshold frequency for a photo-sensitine
metal is 3.3 × 1014 Hz. If light of frequency
8.2 × 1014 Hz is incident on this metal, the cut-off
V
voltage for the photo-electric emission is nearly (c) V (d)
(a) 2 V (b) 3 V
Z
(c) 5 V (d) 1 V Z
3. For the given circuit of p-n junction diode which 6. A resistor R, an inductor L and capacitor C are
of the following is correct connected in series to an oscillator of frequency
R n. If the resonant frequency is nr, then the current
lags behind voltage, when
(a) n = 0 (b) n < nr
(c) n = nr (d) n > nr
7. A parallel plate capacitor has capacitance C. If it
is equally filled the parallel layers of materials of
dielectric constant K 1 and K2 its capacity
becomes C1. The ratio of C1 and C is
V
EBD_7443
www.jeeneetbooks.in

WWW.IIT-NEET.XYZ

2012-2 Target VITEEE


(a) anywhere (b) emitter-collector
K1K 2
(a) K1 + K2 (b) (c) collector-base (d) base-emitter
K1 K 2 14 The kinetic energy of an electron get tripled then
the de-Broglie wavelength associated with it
K1 K 2 2K1K 2 changes by a factor
(c) K1K 2 (d) K1 K 2
1
8 The potential of the electric field produced by (a) (b) 3
3
point charge at any point (x, y, z) is given by
V = 3x2 + 5, where x, y are in metres and V is in 1
(c) (d) 3
volts. The intensity of the electric field at (–2, 1, 0) 3
is 15 A radioactive substance contains 10000 nuclei
(a) +17 Vm–1 (b) –17 Vm–1 and its half-life period is 20 days. The number of
(c) +12 Vm –1 (d) –12 Vm–1 nuclei present at the end of 10 days is
9. The potential of a large liquid drop when eight (a) 7070 (b) 9000
liquid drops are combined is 20 V. Then the (c) 8000 (d) 7500
potential of each single drop was 16. A direct X-ray photograph of the intenstines is
(a) 10 V (b) 7.5 V not generally taken by radiologists because
(c) 5 V (d) 2.5 V (a) intenstines would burst an exposure to
10 A an d B are two metals with threshold X-rays
frequencies 1.8 × 1014 Hz and 2.2 × 104 Hz. Two (b) the X-rays would be not pass through the
identical photons of energy 0.825 eV each are intenstines
incident on them. Then photoelectrons are (c) the X-rays will pass through the intenstines
emitted by (Take h = 6.6 × 10–34 J-s) without causing a good shadow for any
(a) B alone (b) A alone useful diagnosis
(c) Neither A nor B (d) Both A and B (d) a very small exposure of X-rays causes
cancer in the intenstines
11 In the Wheatstone's network given, P = 10 ,
17. Charge passing through a conductor of
Q = 20 , R = 15 , S = 30 , the current passing
crosssection area A = 0.3 m 2 is given by
through the battery (of negligible internal q = 3t2 +5t + 2 in coulomb, where t is in second.
resistance) is What is the value of drift velocity at t = 2s?
(Given, m = 2 × 1025/m3)
P R (a) 0.77 × 10–5 m/s (b) 1.77 × 10–5 m/s
G (c) 2.08 × 10–5 m/s (d) 0.57 × 10–5 m/s
18. Two capacitors of capacities 1 µF and C µF are
Q S connected in series and the combination is
charged to a potential difference of 120 V. If the
charge on the combination is 80 µC, the energy
stored in the capacitor of capacity C in µJ is
+ – (a) 1800 (b) 1600
(c) 14400 (d) 7200
6V 19. A hollow conducting sphere is placed in an
(a) 0.36 A (b) Zero electric field produced by a point charg placed
(c) 0.18 A (d) 0.72 A at P as shown in figure. Let VA, VB, VC be the
12 Three resistors 1 , 2 and 3 are connected to potentials at points A, B and C respectively. Then
form a triangle. Across 3 resistor a 3V battery
is connected. The current through 3 resistor
is
(a) 0.75 A (b) 1 A A
(c) 2 A (d) 1.5 A C P
13. In a common emitter amplifier the input signal is
applied across
B
www.jeeneetbooks.in

WWW.IIT-NEET.XYZ

Solved Paper 2012 2012-3

(a) VC > VB (b) VB > VC (a) 25 (b) 26


(c) VA > VB (d) VA = VC (c) 56 (d) 30
20. In a hydrogen discharged tube it is observed 28. Each resistance shown in figure is 2 . The
that through a given cross-section 3.13 × 1015 eqivalent resistance between A and B is
electrons are moving from right to left and
3.12 × 1015 protons are moving from left to right.
2
What is the electric current in the discharge tube
2
and what is its direction?
(a) 1 mA towards right A
2 B
(b) 1 mA towards left
(c) 2 mA towards left 2
(d) 2 mA towards right 2
21. In CuSO4 solution when electric current equal
to 2.5 faraday is passed, the gm equivalent (a) 2 (b) 4
deposited on the cathode is (c) 8 (d) 1
(a) 1 (b) 1.5
29. If in a triode value amplification factor is 20 and
(c) 2 (d) 2.5
plate resistance is 10 k , then its mutual
22. In hydrogen a atom, an electron is revolving in
conductance is
the orbit of radius 0.53 Å with 6.6 × 10 15
(a) 2 milli mho (b) 20 milli mho
radiations/s. Magnetic field produced at the
(c) (1/2) milli mho (d) 200 milli mho
centre of the orbit is
30. The output wave form of full-wave rectifier is
(a) 0.125 Wb/m2 (b) 1.25 Wb/m2
(c) 12.5 Wb/m 2 (d) 125 Wb/m2
23. The dipole moment of the short bar magnet is
(a) (b)
12.5 A-m2. The magnetic field on its axis at a
distance of 0.5 m from the centre of the magnet
is
(a) 1.0 × 10–4 N/A-m (b) 4 × 10–2 N/A-m
(c) 2 × 10–6 N/A-m (d) 6.64 × 10–8 N/A-m (c) (d)
24. The turn ratio of transformers is given as 2:3. If
the current through the primary coil is 3 A, thus 31. Calculate the energy released when three
calculate the current through load resistance -particles combined to form a 12C nucleus, the
(a) 1A (b) 4.5 A mass defect is
(c) 2 A (d) 1.5 A (Atomic mass of 2He4 is 4.002603 u)
25. In an AC circuit, the potential across an (a) 0.007809 u (b) 0.002603 u
inductance and resistance joined in series are (c) 4.002603 u (d) 0.5 u
respectively 16 V and 20 V. The total potential 32. In the figure shown, the magnetic field induction
difference across the circuit is as the point O will be
(a) 20.0 V (b) 25.6 V
(c) 31.9 V (d) 33.6 V
26. If hydrogen atom is its ground state absorbs
O 2r
10.2 eV of energy. The orbital angular momentum
is increase by
(a) 1.05 × 10–34 J/s (b) 3.16 × 10–34 J/s
(c) 2.11 × 10–34 J/s (d) 4.22 × 10–34 J/s 0i 0 i
(a) (b) ( 2)
27. Highly energetic electrons are bombarded on a 2 r 4 r
target of an element containing 30 neutrons. The
ratio of radii of nucleus to that of Helium nucleus 0 i 0i
(c) ( 1) (d) ( 2)
is (14)1/3. The atomic number of nucleus will be 4 r 4 r
EBD_7443
www.jeeneetbooks.in

WWW.IIT-NEET.XYZ

2012-4 Target VITEEE


33. In photoelectric emission process from a metal a change in the collector current from 10 mA to
of work function 1.8 eV, the kinetic energy of 20 mA. The current gain is
most energetic electrons is 0.5 eV. Th e (a) 75 (b) 100
corresponding stopping potential is (c) 25 (d) 50
(a) 1.3 V (b) 0.5 V 37. A uniform electric field and a uniform magnetic
(c) 2.3 V (d) 1.8 V field are acting along the same direction in a
34. A current of 2 A flows through a 2 resistor certain region. If an electron is projected in the
when connected across a battery. The same region such that its velocity is pointed along
battery supplies a current of 0.5 A when the direction of fields, then the electron
connected across a 9 resistor. The internal (a) speed will decrease
resistance of the battery is (b) speed will increase
(a) 1/3 (b) 1/4 (c) will turn towards left of direction of motion
(c) 1 (d) 0.5 (d) will turn towards right of direction a motion
35. The current i in a coil varies with time as shown 38. Charge q is uniformly spread on a thin ring of
in the figure. The variation of induced emf with radius R. The ring rotates about its axis with a
time would be uniform frequency f Hz. The magnitude of
i magnetic induction at the centre of the ring is
0 qf 0q
(a) (b)
t 2R 2fR
0 T/4 T/2 3T/4 T
0q 0 qf
(c) (d)
2 fR 2 R
emf 39. A galvanometer of resistance, G is shunted by a
resistance S ohm. To keep the main current in
the circuit unchanged, the resistance to be put
(a) 0 T/4 T/2 3T/2 T t in series with the galvanometer is
S2 SG
(a) (b)
emf (S G) (S G)

G2 G
T/4 T/2 3T/2 T (c) (d)
t (S G) (S G)
(b) 0
40. Three charges, each + q, are placed at the corners
of an isosceles triagle ABC of sides BC and AC,
2a. D and E are the mid-points of BC and CA.
emf
The work done in taking a charge Q from D to E
is
T/2 3T/4 T A
(c) 0 t
T/4
E
emf
B C
T/4 D
(d) 0 t
T/2 3T/4 T eqQ qQ
(a) 8 0a (b) 4 0a
36. A transistor is operated in common emitter
3qQ
configuration at VC = 2 V such that a change in (c) Zero (d)
the base current from 100 µA to 300 µA produces 4 0a
www.jeeneetbooks.in

WWW.IIT-NEET.XYZ

Solved Paper 2012 2012-5

PART - II (CHEMISTRY) (CH3 )2 CH CH 2OC2 H5 Br


41. A bubble of air is underwater at temperature 15°C The mechanisms of reactions (i) and (ii) are
and the pressure 1.5 bar. If the bubble rises to respectively
the surface where the temperature is 25°C and (a) SN1 and SN2 (b) SN1 and SN1
the pressure is 1.0 bar, what will happen to the (c) SN2 and SN2 (d) SN2 and SN1
volume of the bubble ? 45. Which of the following complex compounds will
(a) Volume will become greater by a factor of exhibit highest paramagnetic behaviour?
1.6 (At. no. Ti = 22, Cr = 24, Co = 27, Zn = 30)
(b) Volume will become greater by a factor of (a) [Ti(NH3)6]3+ (b) [Cr(NH3)6]3+
1.1 (c) [Co(NH3)6]3+ (d) [Zn(NH3)6]2+
(c) Volume will become smaller by a factor of 46. Which of the following oxide is amphoteric?
0.70 (a) SnO2 (b) CaO
(d) Volume will become greater by a factor of (c) SiO2 (d) CO2
2.9 47. The following reactions take place in the blast
42. Match List-I with List-II for the compositions of furnace in the preparation of impure iron. Identify
substances and select the correct answer using the reaction pertaining to the formation of the
the codes given below the lists. slag.
List-I List-II (a) Fe 2 O3 (s) 3CO(g)
(Substances) (Composition)
A. Plaster of Paris 1. CaSO4.2H2O 2Fe(l) 3CO 2 (g)
1 (b) CaCO 3 (s) CaO(s) CO 2 (g)
B. Epsomite 2. CaSO4 . H 2 O
2 (c) CaO(s) SiO 2 (s) CaSiO 3 (s)
C. Kieserite 3. MgSO4.7H2O (d) 2C(s) O 2 (g) 2CO(g)
D. Gypsum 4. MgSO4.H2O
48. Among the elements Ca, Mg, P and Cl, the order
5. CaSO4
of increasing atomic radii is
Codes :
A B C D (a) Mg < Ca < Cl < P (b) Cl < P < Mg < Ca
(a) 3 4 1 2 (c) P < Cl < Ca < Mg (d) Ca < Mg < P < Cl
(b) 2 3 4 1 49. The reaction,
(c) 1 2 3 5 2A(g) B(g) 3C(g) D(g)
(d) 4 3 2 1 is begun with the concentrations of A and B
43. The pairs of species of oxygen and their both at an initial value of 1.00 M. When
magnetic behaviours are noted below. Which of equilibrium is reached, the concentration of D is
the following presents the correct description? measured and found to be 0.25 M. The value for
the equilibrium constant for this reaction is given
(a) O 2 , O 22 – Both diamagnetic
by the expression
(b) O , O 22 – Both paramagnetic (a) [(0.75)3 (0.25)] ÷ [(1.00)2 (1.00)]
(b) [(0.75)3 (0.25)] ÷ [(0.50)2 (0.75)]
(c) O2 , O2 – Both paramagnetic (c) [(0.75)3 (0.25)] ÷ [(0.50)2 (0.25)]
(d) [(0.75)3 (0.25)] ÷ [(0.75)2 (0.25)]
(d) O, O 22 – Both paramagnetic
50. Which of the following expressions correctly
44. Consider the reactions represents the equivalent conductance at infinite
C 2 H5OH dilution of Al2(SO4)3 ? Given that and
(i) (CH3 ) 2 CH CH 2 Br Al3
(CH3 ) 2 CH CH 2OC2 H5 HBr are the equivalent conductances at
SO 24

(ii) C 2 H5 O infinite dilution of the respective ions?


(CH3 ) 2 CH CH 2 Br
EBD_7443
www.jeeneetbooks.in

WWW.IIT-NEET.XYZ

2012-6 Target VITEEE

List-I List-II
(a) 2 3
Al3 SO 24 (Substances) (Processes)
A. Sulphuric acid 1. Haber's process
(b) B. Steel 2. Bessemer's process
Al3 SO 24
C. Sodium hydroride 3. Leblanc process
D. Ammonia 4. Contact process
(c) 3 6 Codes :
Al3 SO 24
A B C D
1 1 (a) 1 4 2 3
(d) Al3 2 (b) 1 2 3 4
3 2 SO4
51. The pressure exerted by 6.0g of methane gas in (c) 4 3 2 1
a0.03 m3 vessel at 129°C is (d) 4 2 3 1
56. When glycerol is treated with excess of HI, it
(Atomic masses : C =12.01, H = 1.01 and
produces
R = 8.314 JK–1 mol–1)
(a) 2-iodopropane (b) allyl iodide
(a) 215216 Pa (b) 13409 Pa
(c) propene (d) glycerol triiodide
(c) 41648 Pa (d) 31684 Pa
57. Some statements about heavy water are given
52. Match List I (Equations) with List II (Types of
below.
process) and select the correct option. (i) Heavy water is used as moderator in nuclear
List-I List-II reactors
(Equations) (Types of process) (ii) Heavy water is more associated than
ordinary water
A. KP > Q 1. Non-spontaneous
(iii) Heavy water is more effective solvent than
B. G RT ln Q 2. Equilibrium ordinary water
C. KP = Q 3. Spontaneous and Which of the above statements are correct?
endothermic (a) (i) and (ii) (b) (i), (ii) and (iii)
(c) (ii) and (iii) (d) (i) and (iii)
H
D. T 4. Spontaneous 58. Which one of the following compounds will be
S most readily dehydrated ?
Codes : O
A B C D
(a) 1 2 3 4
(b) 3 4 2 1 (a) H3C
(c) 4 1 2 3
OH
(d) 2 1 4 3
O
53. Among the following which one has the highest
cation of anion size ratio?
(a) CsI (b) CsF (b) H3C
(c) LiF (d) NaF
54. Which of the following species is not OH
electrophilic in nature? O OH

(a) Cl (b) BH3


(c) H3C
(c) H 3O (d) NO2 OH

55. Match List I (Substances) with List II (Processes


employed in the manufacture of the substances)
(d) H3C
and select the correct option.
O
www.jeeneetbooks.in

WWW.IIT-NEET.XYZ

Solved Paper 2012 2012-7

59. Which one of the following complexes is not (b) Isostructural with same hybridisation for
expected to exhibit isomerism? the central atom
(a) [Ni(NH3)4(H2O)2]2+ (c) Isostructural with different hybridisation for
(b) [Pt(NH3)2Cl2] the central atom
(c) [Ni(NH3)2Cl2] (d) Similar in hybridisation for the central atom
(d) [Ni(en)3]2+ with different structures
60. Which of the following conformers for ethylene 64. Following compounds are given
glycol is most stable ? (i) CH3CH2OH (ii) CH3COCH3
OH (iii) CH 3 CHOH (iv) CH3OH
|
H H CH3

(a) Which of the above compound(s) on being


H H warmed with iodine solution and NaOH, will give
OH iodoform?
(a) (i), (iii) and (iv) (b) Only (ii)
OH (c) (i), (ii) and (iii) (d) (i) and (ii)
OH 65. Fructose reduces Tollen's reagent due to
(a) asymmetric carbons
(b) (b) primary alcoholic group
H (c) secondary alcoholic group
H HH
(d) en olisation of fructose followed by
conversion to aldehyde by base
OH 66. In the following reaction,
H
(i) Mg, Ether
C6 H5CH 2Br X,
(ii) H3O
(c)
HO the product 'X' is
H HH (a) C6H5CH2OCH2C6H5
(b) C6H5CH2OH
OH (c) C6H5CH3
(d) C6H5CH2CH2C6H5
OH
H 67. Which of the following is not a fat soluble
vitamin?
(d)
H (a) Vitamin-B complex
H
H (b) Vitamin-D
(c) Vitamin-E
61. The IUPAC name of the compound (d) Vitamin-A
CH3CH = CHC CH is 68. Which of the statements about 'Denaturation'
(a) pent-4-yn-2-ene (b) pent-3-en-1-yne given below are correct?
(c) pent-2-en-4-yne (d) pent-1-yn-3-ene Statements :
62. Which of the following oxidation states is the (i) denaturation of proteins causes loss of
most common among the lanthanoids? secondary and tertiary structures of the
(a) 4 (b) 2 protein.
(c) 5 (d) 3 (ii) Denaturation leads to the conversion of
63. Some of the properties of the two species, NO3– double strand of DNA into single strand'.
and H3O+ are described below. Which one of (iii) Denaturation affects primary structure
them is correct? which gets destroyed.
(a) Dissimilar in hybridisation for the central (a) (ii) and (iii) (b) (i) and (iii)
atom with different structures (c) (i) and (ii) (d) (i), (ii) and (iii)
EBD_7443
www.jeeneetbooks.in

WWW.IIT-NEET.XYZ

2012-8 Target VITEEE


69. Which has the maximum number of molecules 75. Consider the following processes
among the following ? H (kJ/mol)
(a) 44 g CO2 (b) 48 g O3
(c) 8 g H2 (d) 64 g SO2 1
A B 150
70. Which of the following compounds undergoes 2
nucleophilic substitution reaction most easily ?
Cl 3B 2C D 125

(a) E A 2D 350
NO2
For B D E 2C, H will be
Cl (a) 525 kJ/mol (b) –175 kJ/mol
(c) –325 kJ/mol (d) 325 kJ/mol
76. Match the compounds given in List-I with
(b) List-II and select the suitable option using the
codes given below
CH3
List-I List-II
Cl A. Benzaldehyde 1. Phenolphthalein
B. Phthalic 2. Benzoin
Cl anhydride condensation
(c) (d) C. Phenyl benzoate 3. Oil of wintergreen
D. Methyl 4. Fries rearrangement
OCH3 salicylate
71. A 0.1 molal aqueous solution of a weak acid is Codes :
30% ionised. If Kf for water is 1.86° C/m, the A B C D
freezing point of the solution will be (a) 4 1 3 2
(a) –0.18°C (b) –0.54°C
(c) –0.36°C (d) –0.24°C (b) 4 2 3 1
72. Which of the following carbonyls will have the (c) 2 3 4 1
strongest C – O bond? (d) 2 1 4 3
(a) Mn(CO)+6 (b) Cr(CO)6 77. Which of the following compound is the most
(c) V(CO)–6 (d) Fe(CO)5 basic ?
73. The order of reactivity of phenyl magnesium
bromide (PhMgBr) with the following
compounds (a) O2N NH2

H 3C H 3C Ph
C O C O and C O (b) CH2NH2
H H 3C Ph
(I) (II) (III)
(a) III > II > I (b) II > I > III N COCH3
(c) I > III > II (d) I > II > III (c)
74. A solid compound XY has NaCl structure. If the H
radius of the cation is 100 pm, the radius of the
anion (Y–) will be NH2
(d)
(a) 275.1 pm (b) 322.5 pm
(c) 241.5 pm (d) 165.7 pm
www.jeeneetbooks.in

WWW.IIT-NEET.XYZ

Solved Paper 2012 2012-9

78. Which of the following structures is the most 84. If 3p and 4p are resultant of a force 5p, then the
preferred and hence of lowest energy for SO3? angle between 3p and 5p is

O 1 3 1 4
(a) sin (b) sin
S 5 5
(a) S (b) O O (c) 90° (d) None of these
85. If 2 tan –1(cos x) = tan–1(2 cosec x), then the value
O O
of x is
O 3
O (a) (b)
4 4
S S
(c) (d)
O
O

(c) (d) None of these


O
O

O
3
79. What is the value of electron gain enthalpy of 86. Let a be any element in a boolean algebra B.
If a + x = 1 and ax = 0, then
Na+ if IE1 of Na = 5.1 eV ?
(a) x = 1 (b) x = 0
(a) –5.1 eV (b) –10.2 eV
(c) x = a (d) x = a'
(c) +2.55 eV (d) +10.2 eV
87. Dual of (x + y) . (x + 1) = x + x . y + y is
80. The unit of rate constant for a zero order reaction
(a) (x . y) + (x . 0) = x . (x + y) . y
is (b) (x + y) + (x . 1) = x . (x + y) . y
(a) mol L–1s–1 (b) L mol–1s–1 (c) (x . y) (x . 0) = x . (x + y) . y
2
(c) L mol s –2 –1 (d) s –1 (d) None of the above
PART - III (MATHEMATICS) 88. The function f : R R defined by
f (x) = (x – 1)(x – 2)(x – 3) is
81. The solution of the differential equation (a) one-one but not onto
(b) onto but not one-one
dy 2yx 1
is (c) both one-one and onto
dx 1 x 2 (1 x 2 ) 2 (d) neither one-one nor onto
(a) y (1 + x2) = C + tan–1x 89. If the complex numbers z 1, z2 and z3 are in AP,
then they lie on a
y (a) a circle (b) a parabola
(b) = C + tan–1x
1 x2 (c) line (d) ellipse
(c) y log (1 + x2) = C + tan–1x 90. Let a, b and c be in AP and |a| < 1, |b| < 1, |c| < 1.
(d) y (1 + x2) = C + sin–1x If x = 1 + a + a2 + ... to ,
82. If x, y and z are all distinct and y = 1 + b + b2 + ... to ,
z = 1 + c + c2 + ... to , then x, y and z are in
(a) AP (b) GP
x x 2 1 x3
(c) HP (d) None of these
y y 2 1 y3 0, then the value of xyz is 91. The number of real solutions of the equation
2 3 9
z z 1 z
3 x x 2 is
10
(a) –2 (b) –1 (a) 0 (b) 1
(c) –3 (d) None of these (c) 2 (d) None of these
83. The probability that atleast one of the events A 92. The lines 2x – 3y – 5 = 0 and 3x – 4y = 7 are
and B occurs is 0.6. If A and B occur diameters of a circle of area 154 sq units, then
simultaneously with probability 0.2, then the equation of the circle is
P(A) P(B) is (a) x2 + y2 + 2x – 2y – 62 = 0
(a) 0.4 (b) 0.8 (b) x2 + y2 + 2x – 2y – 47 = 0
(c) x2 + y2 – 2x + 2y – 47 = 0
(c) 1.2 (d) 1.4
(d) x2 + y2 – 2x + 2y – 62 = 0
EBD_7443
www.jeeneetbooks.in

WWW.IIT-NEET.XYZ

2012-10 Target VITEEE


93. The angle of depressions of the top and the foot (a) independent of
of a chimney as seen from the top of a second (b) independent of
chimney, which is 150 m high and standing on (c) independent of and
the same level as the first are and respectively, (d) None of the above
then the distance between their tops when 100. Themaximum valueof 4 sin2 x – 12 sin x + 7 is
(a) 25 (b) 4
4 5
tan and tan is (c) does not exist (d) None of these
3 2 101. A straight line through the point A(3, 4) is such
150 that its intercept between the axes is bisected at
(a) m (b) 100 3 m A, its equation is
3
(a) 3x – 4y + 7 = 0 (b) 4x + 3y = 24
(c) 150 m (d) 100 m (c) 3x + 4y = 25 (d) x + y = 7
94. If one root is square of the other root of the 102. The tangent at (1, 7) to the curve x2 = y – 6
equation x2 + px + q = 0, then the relations touches the circle x2 + y2 + 16x + 12y + c = 0 at
between p and q is (a) (6, 7) (b) (–6, 7)
(a) p3 – (3p – 1) q + q2 = 0 (c) (6, –7) (d) (–6, –7)
(b) p3 – q (3p + 1) + q2 = 0 103. The equation of straight line through the
(c) p3 + q (3p – 1) + q2 = 0 intersection of the lines x – 2y = 1 and x + 3y = 2
(d) p3 + q (3p + 1) + q2 = 0 and parallel 3x + 4y = 0 is
95. The coefficient of x53 in the following expansions (a) 3x + 4y + 5 = 0 (b) 3x + 4y – 10 = 0
100 (c) 3x + 4y – 5 = 0 (d) 3x + 4y + 6 = 0
100
Cm (x 3)100 m .2 m is
dx
m 0 104. equals to
(a) 100C (b) 100C sin x cos x 2
47 53
(c) –100C 53 (d) –100C 100 1 x
96. If (–3, 2) lies on the circle x2 + y2 + 2gx + 2fy + c = 0, (a) tan C
which is concentric with the circle 2 2 8
x2 + y2 + 6x + 8y – 5 = 0, then c is equal to 1 x
(a) 11 (b) –11 (b) tan C
(c) 24 (d) 100 2 2 8
97. If a = i + j + k, b = i + 3j + 5k and c = 7i + 9j + 11k, 1 x
then the area of Parallelogram having diagonals (c) cot C
2 2 8
a + b and b + c is
1 1 x
(a) 4 6 sq. units (b) 21 sq. units (d) cot C
2 2 2 8
1
6 1 x
(c) sq. units (d) 6 sq. units 105. The value of integral dx is
2 1 x
0
1 5 7
98. If A 0 7 9 , then trace of matrix A is (a) 1 (b) 1
2 2
11 8 9 (c) –1 (d) 1
(a) 17 (b) 25 1
1
(c) 3 (d) 12 106. The value of I xx dx is
99. The value of the determinant 2
0

cos sin 1 1 1
(a) (b)
sin cos 1 is 3 4
cos( ) sin( ) 1 1
(c) (d) None of these
8
www.jeeneetbooks.in

WWW.IIT-NEET.XYZ

Solved Paper 2012 2012-11

107. The eccentricity of the ellipse, which meets the 114. Area lying in the first quadrant and bounded by
x y the circle x2 + y2 = 4, the line x 3y and x-axis
straight line 1 on the axis of x and the
7 2 is
x y (a) sq units (b) sq units
straight line 1 on the axis of y and whose
3 5 2
axes lie along the axes of coordinates, is
(c) sq units (d) None of these
3 2 2 6 3
(a) (b) 1x
7 7 1
115. The value of lim tan x is
3 x 2
(c) (d) None of there
7 (a) 0 (b) 1
(c) –1 (d) e
x2 y2
108. If 1 (a b) and x2 – y2 = c2 cut at
a 2 b2 mx 1, x
2
right angles, then 116. If f (x) is continuous at
(a) a2 + b2 = 2c 2 (b) b2 – a2 = 2c 2 sin x n, x
2 2
(c) a – b = 2c 2 (d) a2b2 = 2c 2 2
109. The equation of the conic with focus at (1, –1)
directrix along x – y + 1 = 0 and with eccentricity x , then
2
2 is n
(a) x2 – y2 = 1 (a) m = 1, n = 0 (b) m 1
2
(b) xy = 1
(c) 2xy – 4x + 4y + 1 = 0 n m m n
(c) (d)
(d) 2xy + 4x – 4y – 1 = 0 2 2
110. There are 5 letters and 5 different envelopes.
The number of ways in which all the letters can 4 x2
be put in wrong envelope, is 117. The domain of the function f (x)
sin 1 (2 x)
(a) 119 (b) 44
(c) 59 (d) 40 is
111. The sum of the series (a) [0, 2] (b) [0, 2)
(c) [1, 2) (d) [1, 2]
12 22 12 22 32 12 22 32 42 118. The general solution of the differential equation
1 ...
2! 3! 4! (1 + y2) dx + (1 + x2) dy = 0 is
is (a) x – y = C (1 – xy) (b) x – y = C (1 + xy)
17 13 19 (c) x + y = C (1 – xy) (d) x + y = C (1 + xy)
(a) 3e e (c)
(b) e (d) e 119. The order and degree of the differential equation
6 6 6
n
112. The coefficient of x in the expansion of 2 32
loga(1 + x) is dy
1
( 1)n 1 ( 1)n 1 dx
are, respectively
(a) (b) log a e 2
n n d y
n 1 n
( 1) ( 1) dx 2
(c) log e a (d) log a e
n n (a) 2, 2 (b) 2, 3
113. If a plane meets the coordinate axes at A, B and (c) 2, 1 (d) None of these
C in such a way that the centroid of ABC is at 120. The relation R defined on the set of natural
the point (1, 2, 3), then equation of the plane is numbers as {(a, b) : a differs from b by 3} is
x y z x y z given
(a) 1 (b) 1 (a) {(1, 4), (2, 5), (3, 6),... }
1 2 3 3 6 9 (b) {(4, 1), (5, 2), (6, 3),... }
x y z 1 (c) {(1, 3), (2, 6), (3, 9),... }
(c) (d) None of these (d) None of the above
1 2 3 3
EBD_7443
www.jeeneetbooks.in

WWW.IIT-NEET.XYZ

2012-12 Target VITEEE

SOLUTIONS
PART - I (PHYSICS) 6. (d) When reactance of inductance is mroe than
the reactance of cndenser, the current will
I1 lag behind the voltage.
1. (d)
F1 1 1
d Thus L or
c LC
I
F4 1
or n or n > nr
2 LC
nr = resonant frequency
F2
K1 0 A K2 0A
I 7. (c) Capacitance, CA = , CB
d d
F3 2 2

F2 = – F4 C1 2K1K 2
Ceq = C K1 K 2
0 I1Il 2
F1 =
2 dl CACB 2K1K2 0A 0A
= e
0 I1I CA CB K1 K2 d d
F2 =
2 (d l)
dV
F1 > F3 8. (d) Intensity of the electric field, E = = 6x
Fnet = F1 – F3 dx
So, wire attract loop. Potential (v) = 3x2 + 5
E at x = –2
E V h(v v0 ) = 6 (–2) = – 12V/m
2. (a) Here, V0 =
e e 9. (c) Volume of 8 small drops = Volume of big
34 drop
6.62 10 (8.2 1014 3.3 1014 )
= 19 4 3 4 3
1.6 10 r 8 R
3 3
34
6.62 10 2r = R
= 4.9 1033
1.6 ...(i)
According to charge conservation
6.62 4.9 10 1 8q = Q
=
1.6 ...(ii)
V0 = 2 volt q
3. (a) In forward biasing, resistance of p–n junctin Potential of one small drop (V ) = 4
diode is zero, so whole voltage appears 0r
across the resistance. Q
4. (d) BE of Li7 = 39.20 MeV Similarly, potential of big drop (V) = 4
and He4 = 28.24 MeV 0R
Hence binding energy of 2He4 = 56.84 MeV V q R V 9 2r
Energy of reaction = 56.84 – 39.20 Now,
V Q r 20 8q r
= 17.28 MeV
V = 5V
5. (b) v (Z b)
www.jeeneetbooks.in

WWW.IIT-NEET.XYZ

Solved Paper 2012 2012-13

10. (b) Threshold energy of A EA = hvA


= 6.6 × 10–34 × 1.8 × 1014
= 11.88 × 10–20 J 2
1
11.88 10 20 I2 I2 = 1A
eV = 0.74 eV 3
= 19
1.6 10
Similarly, EB = 0.91eV I1 = 1A I1 + I2 = 2A
As the incident photons have energy
greater than EA but less than EB
So, photoelectrons will be emitted from
metal A only. Current in the circuit,
11. (a) Balanced wheatstone bridge condition 3
I= = 2A
P R (3/ 2)
Q S I
No, current flows through galvanometer Current in 3 resistor =
= IA
2
Now, P and R are in series, so 13. (b) In CE amplifier, the input signal is applied
Resistance R1 = P + R across base–emitter junction.
= 10 + 15 = 25 C
Similarly, Q and S are in series, so C1
B C2
Resistance R2 = R + S O
= 20 + 30 = 50 E Output
Input
Net resistance of the network as R1 and R2 RB RC
are in parallel
V1 O V0
1 1 1 RBB VCC
R R1 R2
25 50 50
R = 25 50 3 14. (c) de–Broglie wavelength of an electron
V 6 h h 1
Hence, current, I = = 0.36A = or
R 50 / 3 mv 2mK K
12. (b) The arrangement is shown in figure.
1 K 1
B
3K 1 3
1
2

or =
3
A C
3 i.e. de–Broglie wavelength will change by
1
factor .
3V 3
15. (a) We know,
Here, two reisistance of 1 and 2 are in
series, which form 3 which is in parallel 10
t/T
N 1 N 1 20
with 3 resistance.
Therefore, the effective resistance N0 2 10000 2
(1 2) 3 3 10000 10000
(1 2) 3 2 N= = 7070
2 1.414
EBD_7443
www.jeeneetbooks.in

WWW.IIT-NEET.XYZ

2012-14 Target VITEEE


16. (c) As X-rays pass through the intestine
0 2N
without casting a clear shadow. 23. (c) The magnetic field, B = .
17. (b) Given : A = 0.3 m2 n = 2 × 1025/m3 4 d3
q = 3t2 + 5t + 2 2 1.25
= 10–7 × = 2 × 10–6 N/A–m
dq (0.5)3
i= = 6t + 5 = 17
dt
IP nS
i 24. (c) Transformation ratio, I
Drift velocity, vd = S nS
neA
17 3 3
= i.e. I 2 or, IS = 2A
2 10 25 1.6 10 19 0.3 S
25. (b) Voltage
17
= = 1.77 × 10–5 m/s V= VR2 VC2 = (20)2 (16)2
0.96 106
18. (b) Capacitance 1 F and C F are connected = 25.6V
in series, 26. (a) Electron goes to its first excited state
(n = 2) from ground state (n = 1) after
C
Ceq = absorbing 10.2 eV energy
1 C
Given, V = 120 V and q = 80 C h
Increase in momentum =
q = CeqV 2
C 6.6 10 34
80 = 20 =
C 1 6.28
or C = 2 F = 1.05 × 10–34 J–s
Energy stored in the capacitor of capicity C 27. (b) Using R = R0 A1/3
1q 1/3
U= R1 A1
2 C
R2 A2
1 (80 10 6 )2
= 6
1
2 2 10 R A 3
R He 4
1 80 10 6 80 10 6
=
2 2 10 6 1
U = 1600 J A 3
(14)1/3 = A = 56
19. (d) Conducting surface behaves as 4
equipotential surface.
So, = 56 – 30 = 26
20. (a) I = neqe + npqE = 1 mA (towards right)
28. (a) Given circuit is a balanced Wheatstone
21. (a) 1 faraday deposited 1 g equivalent
bridge.
22. (c) The magnetic field
0 2 (qv) 2
B= . 2
4 r
19
2 3.14 (1.6 10 1.6 1015 ) A B
= 10–7 × 10 2
0.53 10
2
= 12.5 Wb/m3
2

Equivalent resistance of upper arms


www.jeeneetbooks.in

WWW.IIT-NEET.XYZ

Solved Paper 2012 2012-15

=2+2=4 From Eqs. (i) and (ii), we have


Equivalent resistance of lowre arms 2 9 r 9 r
=2+2=4 4=
0.5 2 r 2 r
4 4
RAB = =2 1
4 4 3r = 1 r=
3
35. (d) We know, induced emf
29. (a) Mutual conductance gm = R
P di
e=–L
20 dt
= = 2 × 10–3 = 2 milli mho T di
10 103 During 0 to , = constant
30. (c) Full-wave rectifier output wave form 4 dt
So, e = –ve
T T di
For to , =0
4 2 dt
31. (c) Mass defect
i.e., e = 0
m = Total mass of –particles
– mass of 12C nucleus T 3T di
For to , = constant
= 3 × 4.002603 – 12 = 12.007809 – 12 4 4 dt
= 0.007809 unit i.e., e = +ve
32. (a) Field due to a straight wire of infinite length
IC (20 100)mA
0i 36. (d) Current gain, = IB (300 100)mA
is if the point is on a line perpendicular
4 r
to its length while at the centre of a 3
10 10
= = 50
i 0 200 10 6
semicurcular coil is 37. (a) Field B not applied only force. Field E will
4 r
apply a force opposite to velocity of the
a electron hence, speed will decreases.
38. (a) We know magnetic field
r
b 0i
O =
2R
c q
q = it i= = qf
B = Ba + Bb + Bc t
i 0 i 0 i 0 qf
=
0
. . =
4 r 4 r 4 r 2R
39. (c) If resistance remains same so current will
0 i be unchanged.
= ( 2) out of the phase
4 r R
33. (b) Stopping potential = Maximum KE G G
eV = KEmax
E
34. (d) Current i = S
R r
GS GS
E G= R R=G–
2= ... (i) G S G S
2 r
G2
E or, R =
0.5 = ...(ii) G S
9 r
EBD_7443
www.jeeneetbooks.in

WWW.IIT-NEET.XYZ

2012-16 Target VITEEE


40. (c) Here, AC = BC
* *
O2 1s 2 , 1s 2 , 2s 2 , 2s 2 , 2 pz2 , 2 px2
C
q
* *
2 p 2y , 2 p1x 2 p1y
and the electronic configuration of O and
E D O+ are
O = 1s 2 ,2s 2 , 2 p2x , 2 p1y ,2 p1z
q q
A B O+ = 1s 2 ,2s 2 , 2 p1x , 2 p1y , 2 p1z
D
VD = VE = V As O2 , O2 ,O 2 ,O and O+ have unpaired
W = Q[VE – VD] electrons, hence are paramagnetic.
W = Q [V – V] 44. (a) C2H5OH being a weaker nucleopbile, when
W= 0 used as a solvent in case of hindered 1°
halide, favours SN1 mechanism while
PART - II (CHEMISTRY) C2H5O–being a strong nucleophile in this
p1V1 p2V2 (By ideal gas equation) reaction favours SN2 mechanism.
41. (a)
T1 T2 45. (b) (a) Electronic configuration of Ti 3+ in
[Ti(NH3)6]3+
1.5 V1 1 V2 Ti3+ = 3d1;
or
288 298 3d 4s 4p
V2 = 1.55 V1
i.e, volume of bubble will be almost 1.6 times
to initial volume of bubble.
2
1 d sp3 hybridisation
42. (b) (A) Plaster or Paris = CaSO4 HO
2 2 (b) Electronic configuration of Cr 3+ in
(B) Epsomite = MgSO4 · 7H2O [Cr(NH3)6]3+
(C) Kieserite = MgSO4 · H2O
Cr3+ = 3d3;
(D) Gypsum= CaSO4 · 2H2O
43. (c) The molecular orbital configurations of 3d 4s 4p
O2 , O2 , O 22 and O2 are

* *
O2 1s 2 , 1s 2 , 2s 2 , 2s 2 , 2 p 2z , 2 px2 2
d sp3 hybridisation
* * (c) Electronic configuration of Co3+ in
2 p 2y , 2 p1x 2 p 0y [Co(NH3 )6]3+;
Co3+ = 3d6.
* * 3d 4s 4p
O2 1s 2 , 1s 2 , 2s 2 , 2s 2 , 2 pz2 , 2 px2

* *
2 p 2y , 2 p x2 2 p1y In the presence of strong field ligand
NH3, pairing of electrons takes place
* * and hence, octahedral complex,
O22 1s 2 , 1s 2 , 2s 2 , 2s 2 , 2 pz2 , 2 px2
[Co(NH3)6]3+ is diamagnetic.
* * inner orbital or
2 p 2y , 2 p 2x 2 p 2y
[Co(NH3)6]3+ low spin complex

(6NH3 molecles)
www.jeeneetbooks.in

WWW.IIT-NEET.XYZ

Solved Paper 2012 2012-17

On the other hand, on increasing the


number of electron in the same shell, the
3d 4s 4p
atomic radii decreases because effective
×× ×× ×× ×× ×× ×× nuclear charge is increases.
In Mg, P and C1, the number of electrons
are increasing in the same shell, thus the
order of their atomic radii is
d 2sp3 hybridisation C1 < P < Mg
In case of Ca, the electron is entering in
(d) Electronic configuration of Zn2+ in [Zn higher shell. So, its atomic radii is highest.
(NH3)6]2+ Thus, the order of radii is
3d Cl < P < Mg < Ca
Zn2+ = 3d10; 49. (b) The reaction-
2 A(g) B (g) 3C (g) D(g)
4s 4p 4d Initial 1 1 0 0
At equil 1 0.50 1 0.25 0.75 0.25

3
(0.75)3 (0.25)
sp d 2 hybridisation K=
(0.50)2 (0.75)

[Zn(NH3 ) 6 ] 2+ is an outer orbital 50. (b) Al2 (SO4 )3 2Al3 3SO24


complex and is diamagnetic. We can calculate th e equivalent
46. (a) SnO2 reacts with acids as well as bases to conductance only for ions, so the
form corresponding salts. So it is an equivalent conductance at infinite dilution,
amphoteric oxide.
SnO2 + 4HCl SnCl4 + 2H2O eq Al3 SO24
SnO2 + 2NaOH Na 2SnO3 + H2O 51. (c)
sod.stannate
w(given mass of methane) = 6g
47. (c) A slag is an easily fusible material which is
formed when gangue still present in the temperature, T = 129 + 273 = 402 K
roasted or the calcined ore combines with mol mass of methane, M = 12. 01 + 4 1.01
the flux. For example, in the metallurgy of =16.05
iron, CaO (flux) combines with silica gangue
to form easily fusible calcium silicate From, ideal gas equation,
(CaSiO3) slag. nRT
CaO + SiO2 CaSiO3 (slag) pV = nRT P
v
48. (b) Atomic radii increases, as the number of
shells increases. Thus, on moving down a 6 8.314 402
p= = 41648 Pa
group atomic radii increases. 16.05 0.03
The electronic configuration of the given 52. (c) (A) If kp > Q and goes in forward direction
element is than reaction is spontaneous
Mg12 = [Ne] 3s2 (B) Given, G°< RT ln Q,
Ca20 =[Ar] 4s2 thus, G° = + ve
p15 = [Ne] 3s23p3 and hence, the reaction is non–
Cl17 = [Ne]3s23p5 spontaneous.
EBD_7443
www.jeeneetbooks.in

WWW.IIT-NEET.XYZ

2012-18 Target VITEEE


(C) At equilibrium, Kp = Q 57. (a) In nuclear reactors heavy water is used as
a moderator. It has higher boiling point as
H
(D) T > compared to the ordinary water. Thus, it is
S more associated as compared to ordinary
or T S = H water. The dielectric constant is however
This is valid condition for higher for H2O, thus, H2O is a more effective
spontaneous endothermic reactions solvent as compare to heavy water (D2O).
(as G H – T S) 58. (c) Dehydration of alcohols involve formation
53. (b) The size of cation is in order of- of carbocation intermediate. Higher the
Li+ < Na+ < Cs+ stability of carbocation, higher is the ease
and the size of anions in the order of- of dehydration. The order of stability of
I– > F – carbocation, is
Thus, when the cation is largest and anion
O O
is smallest, the ratio of their sizes is
maximum. < H3C
H3C
Hence, cation to anion size ratio is maximum
for CsF. (b) and (d) (a)
54. (c) Electron deficient species are known as
electrophiles. O
+
Among the given, H3 O has lone pair of < H3C
electrons for donation, so it is not electron (c)
deficient and hence, not an electrophile.
55. (d) Contact process is used for sulphuric acid, Hence, compound given in option (c)
steel is manufactured by Bessemer's readily undergoes dehydration.
process, Leblanc process is used for the 59. (c) Compounds having tetrahedral geometry
production of NaOH while Haber's process does not exhibit isomerism due to presence
is used for NH3 production. of symmetry elements. Here, [Ni(NH3)2Cl2]
56. (a) has tetrated ral geometry.
60. (d)

OH H-bonding
CH2OH CH2I CH2

3HI H OH
CHOH CHI CH
–3H 2O –I2

CH2OH CH2I CH2I


(highly unstable) allyl iodide
H H
CH3 CH3
H
(HI (excess)
CHI CH This conformation is most stable due to
–I2
intramolecular H-bonding.
CH2I CH2 5 4 3 2 1
(unstable) 61. (b) C H3 — C H CH — C CH
pent-3-en-1-yne
CH3
62. (d) The most common oxidation state exhibited
HI by lanthanoids is +3.
CHI
63. (a) In NO3 ,
CH3
2-iodopropane 1
H = [5 + 0 – 0 + 1] = 3. So, sp3
2
www.jeeneetbooks.in

WWW.IIT-NEET.XYZ

Solved Paper 2012 2012-19

hybridization. 67. (a) Fat soluble vitamins are A, D and E.


Thus, it has trigonal planar geometry. Whereas vitamin-B complex is soluble in
O water.
68. (c) In the process denaturation secondary and
tertiary structures of protein destroyed but
N primary structure remains undisturbed.
O O
– Heat, acid and alkali denature DNA molecule
and double strand of DNA converts into
In H3O+, single strand.
1 69. (c) 44 g CO2 = 1 mol CO2 = NA molecules of
H = [6 + 3 – 1 + 0] = 4; So, sp3 CO2
2
hybridization and it has pyramidal geometry 48g O3 = 1 mol O3 = NA molecules of O3
due to the presence of one lone pair of 8 g H2 = 4 mol H2 = 4 × NA molecules of H2
electrons. 64 g SO2 = 1 mol SO2 = NA molecules of SO2
NA = 6.023 × 1023
70. (a)
Cl
O+ H
NO2
H H
It has electron withdrawing group — NO2
O which reduces the double bond character
|| between carbon of benzene ring and
64. (c) Compoun ds having eith er CH3 C—
chlorine. Hence, the correct order of
group or CH3CHOH— group, give iodoform nucleophilic substitution reactions are,
when warmed with I2 and NaOH.
Thus, compounds Cl Cl Cl

O Cl
||
CH3 C HOH,CH3 — C— CH 3 , CH 3 — CHOH > > >
| |
H CH3
NO2 CH3 OCH3
give iodoform when heated with I2 and
NaOH. (Note : NaOI oxidises CH3CH2OH 71. (d) Freezing point depression ( Tf) = iKf m
to CH3CHO, and gives positive iodoform HA H+ + A–
test.) 1–
65. (d) In aqueous medium, fructose is enolised 1 – 0.3 0.3 0.3
and converted into aldehyde in basic i = 1 – 0.3 + 0.3 + 0.3
medium. Generally all aldehydes reduce i = 1.3
Tollen's reagent, thus fructose can also Tf = 1.3 × 1.86 × 0.1 = 0.2418°C
reduces Tollen's reagent. Tf = 0 – 0.2418°C
Mg,Ether = – 0.2418°C
66. (c) C6H5CH2Br C6H5CH2MgBr 72. (a) As positive charge on the central metal atom
Grignard reagent increases, the less readily the metal can
Br donate electron density into the anti–
bonding –orbitals of C–O ligand to
H3O /H
C6 H5CH3 Mg weaken the C—O bond. Thus, the C–O
toluene
bond would be strongest in Mn(CO)6+
OH
EBD_7443
www.jeeneetbooks.in

WWW.IIT-NEET.XYZ

2012-20 Target VITEEE


73. (d) Since alkyl group has +I–effect and aryl OH OH
group has + R–effect, Hence greater the
number of alkyl and aryl groups attached
to the carbonyl group, its reactivity towards
nucleophilic addition reaction. Secondly, as
the steric crowding on carbonyl group
conc. H2SO4/
increases, the r eactivity decreases H H
accordingly. O –H2O
Correct reactivity order for reaction with
PhMgBr is C
H3C H3C Ph O OH OH
C O > C O > C O
C
H H3C Ph
(I) (II) (III) O
phthalic anhydride
C
r
74. (c) Radius ratio of NaCl like crystal =
r O

100 C
= 0.414 or r – = = 241.5 pm
0.414
O
1 phenolphthalein
75. (b) A B ; H = 150 kJ/mol ...(i) (c) Fries rearrangement Phenyl benzoate
2
heated with anhydrous AlCI 3 in the
3B 2C + D; H = – 125 kJ/mol ...(ii)
presence of inert solvent gives ortho–and
E + A 2D; H = + 350 kJ/mol ...(iii)
Para–hydroxybenzophenone. In this
By [2 × (i) + (ii)] – (iii), we have rearrangement, there is only a benzoyl
B + D E + 2C group migration from the phenolic oxygen
H = 150 × 2 + (–125) – 350 to an ortho–and para–position.
= –175 kJ/mol
76. (d) (a) Benzoin condensation : Heating
O
ethanolic solution with strong alkali
like KCN or NaCN, benzoin is obtained. O—C—C6H5 OH
C—C6H5
O AlCl3/
– O
CN
2C6H5 — C — H

O phenyl p-hydroxy-
benzoate benzophenone
C6H5 — C — CH — C6H5 OH

OH
benzoin +
(b) Formation of phenolphthalein phenol is
treated with phthalic anhydride in the
presence of conc. H 2 SO 4 , it gives O C — C6H5
phenolphthalein, an indicator. p-hydroxy-
benzophenone
www.jeeneetbooks.in

WWW.IIT-NEET.XYZ

Solved Paper 2012 2012-21

(d) Methylsalicylate
PART - III (MATHEMATICS)
OH
dy 2yx 1
COOCH3 81. (a) 2
dx 1 x (1 x 2 ) 2
which is a linear differential equation.
2x 1
Here, P = ,Q
2
(A chief constituent of oil of wintergreen) 1 x (1 x 2 ) 2
77. (b) Now, IF = e P dx
CH2NH2
2x 2
= 2
dx 2 = (1 + x )
Compound is most basic due to localised e1 x elog(1 x )
lone pair of electrons on nitrogen atom Solution of differential equation is
While in other compounds, because of
resonance, the lone pair of electrons on 1
y. (1 + x2) = .(1 x 2 )dx C
nitrogen atom gets delocalised over (1 x 2 ) 2
benzene ring and thus is less easily
available for donation. 1
78. (d) Formal charges help in selection of the y (1 + x2) = dx C
1 x2
lowest energy structure from a number of
possible Lewis structures for a given y (1 + x2) = tan–1 x + C
species. Generally the lowest energy
structure is the one with the smallest formal x x 2 1 x3
charges on the atoms. 82. (b) y y2 1 y3 = 0
Formal charge on an atom
= total no. of valence electrons – non -bonding z z2 1 z3
1
electrons – × bonding electrons.
2 x x2 1 x x2 x3
For Lewis structure of SO3
y y2 1 y y2 y3 = 0
O
..
..

z z2 1 z z2 z3
S
.O... ..O.. x x2 1 1 x x2
Formal charge on S atom y y2 1 xyz 1 y y 2
=0
1 z z2 1 1 z z2
=6–0– × 12 = 0
2
Formal charge on three O atoms
1 x x2 1
6 4 4 0
2 y y2 1
79. (a) IE1 of Na = – Electron gain enthalpy of Na+ ion (1 + xyz) =0
= –5.1eV. z z2 1
80. (a) For zero order reaction,
Rate = k [Reactants]° (1 + xyz) [x(y2 – z2) –y (x2 – z2)
Rate = k + z (x2 – y2)] = 0
and unit of k = mol L–1 s –1 (1 + xyz) (x – y) (y – z) (z – x) = 0
1 + xyz = 0 xyz = –1
EBD_7443
www.jeeneetbooks.in

WWW.IIT-NEET.XYZ

2012-22 Target VITEEE


83. (c) P (A B) = 0.6 and P (A B) = 0.2 87. (a) (x + y) . (x + 1) = x + x.y + y
we know that Replace ‘.’ by ‘+’, ‘+’ by ‘.’, ‘1’ by ‘0’, we
P (A B) = P (A) + P(B) – P(A B) get (x . y) + (x . 0) = x.(x + y) . y
0.6 = P(A) + P(B) – 0.2 88. (b) f (x) = (x – 1) (x – 2) (x – 3)
P(A) + P(B) = 0.8 f(1) = f(2) = f (3) = 0
1 – P( A ) + 1 – P ( B ) = 0.8 f(x) is not one–one.
For each y R, there exists x R such that
– [P( A ) + P( B )] = 0.8 – 2 f (x) = y.
P( A ) + P( B ) = 1.2 f is onto.
Note that if a continuous function has more
R sin than one roots, then the function is always
84. (b) Q =
sin( ) many–one.
Also, (5P) = (4P)2 + (3P)2
2 89. (c) Let z1, z2 and z3 be affixes of points A, B
+ 2 (4P) (3P) cos ( + ) and C, respectively. Since, z1, z2 and z3 are
25P2 = 16p2 + 9P2 + 24P2 cos ( + ) in AP, therefore
24P2 cos ( + ) = 0 2z2 = z1 + z3
cos ( + ) = 0 = cos90° z1 z 3
+ = 90° z2 =
2
4P So, B is the mid–point of the line AC.
A, B and C are collinear.
z1, z2 and z3 lie on a line.
R 1
5P 90. (c) x = 1 + a+ a2 + ... =
1 a
1
3P y = 1 + b + b2 + .... =
1 b
5P sin 1
Now, 4P = and z = 1 + c + c2 + .... =
sin 90 1 c
Since, a, b and c are in AP.
4
sin = 1 – a, 1 – b and 1 – c are also in AP.
5
1 1 1
4 , and are in HP..
= sin–1 1 a 1 b 1 c
5
x, y and z are in HP.
85. (b) 2 tan–1 (cos x) = tan –1 (2 cosec x) Note that if the common ratio of a GP is not
less than 1, then we do not determined the
2 cos x
tan–1 = tan–1 (2 cosec x) sum of an infinite GP that series.
1 cos 2 x 91. (a) Let f(x) = – 3 + x – x2
Then, f(x) < 0 for all x because coefficient of
2cos x x2 < 0 and disc < 0. Thus, LHS of the given
= 2 cosec x
1 cos 2 x equation is always positive whereas the
RHS is always less than zero.
2 cos x
= 2 cosec x Hence, the given equation has no solution.
sin 2 x Alternate Solution :
Given, equation is
sinx = cos x x= 9
4
= –3 + x – x2
86. (d) Given conditions are a + x = 1 and ax = 0. 10
These two conditions will be true, if x = a .
www.jeeneetbooks.in

WWW.IIT-NEET.XYZ

Solved Paper 2012 2012-23

In ABE,
Y
150
tan =
d
d = 150 cot
y= 9
10 2
= 150 × = 60 m
X X 5
In DCE,
h
y = – 3 + x – x2 tan =
d
4 h 4
Y h 60 h = 80 m
3 d 3
Now in DCE,
9 DE2 = DC2 + CE2
Let y = , therefore
10 x2 = 602 + 802 = 10000
y = –3 + x – x2 x = 100 m
94. (a) Given equation x2 + px + q = 0 has roots
1 1
y= x2 x 3 and 2.
4 4 Sum = + 2 = –p and Product = 3 = q
2 ( + 1) = –p
11 1 3 [ 3 + 1 + 3 ( + 1)] = – p3
y+ x
4 2 q (q + 1 – 3p) = –p3
It is clear from the graph that two curves do p3 – (3p – 1) q + q2 = 0
not intersect. Hence, no solution exists. 100
100
92. (c) The centre of the required circle lies at the 95. (c) C m (x 3)100 m
.2m
intersection of 2x – 3y – 5 = 0 and 3x – 4y – m 0
7 = 0. Thus, the coordinates of the centre Above expansion can be rewritten as
are (1, –1). [(x – 3) + 2]100 = (x – 1)100 = (1 – x)100
Let r be the radius of the circle. x53 will occur in T54.
r2 = 154 T54 = 100C53 (–x)53
22 2 Required coefficient is – 100C53.
r = 154 r = 7 96. (b) Equation of family of concentric circles to
7
the circle x2 + y2 + 6x + 8y – 5 = 0 is
Hence, the equation of required circle is
x2 + y2 + 6x + 8y + = 0
(x –1)2 + (y + 1)2 = 72
which is similar to
x2 + y2 – 2x + 2y – 47 = 0
x2 + y2 + 2gx + 2fy + c = 0
4 5 Thus, the point (–3, 2) lies on the cirlce
93. (d) Given : tan = and tan = x2 + y2 + 6x + 8y + c = 0
3 2
(–3)2 + (2)2 + 6(–3) + 8 (2) + c = 0
E 9 + 4 – 18 + 16 + c = 0 c = –11
97. (a) a = i + j + k, b = i + 3j + 5k
and c = 7i + 9j + 11k
x h Let A = a + b
= (i + j + k) + (i + 3j + 5k)
150 m = 2i + 4j + 6k
D
d C and B = b + c
= (i + 3j +5k) + (7i + 9j + 11k)
= 8i + 12j + 16k
A d B Area of parallelogram
EBD_7443
www.jeeneetbooks.in

WWW.IIT-NEET.XYZ

2012-24 Target VITEEE

1 We know that, –1 sin x 1


= |A B|
2 5 3 1
– sin x
( A and B are diagonals) 2 2 2
2
i j k 1 3 25
1 sin x
= 2 4 6 4 2 4
2
8 12 16 2
3
1 4 sin x 25
1 2
= |i (64 – 72) – j (32 – 48) + k (24 – 32)|
2 2
3
1 1 4 sin x 2 23
= |–8i + 16j – 8k| 2
2 101. (b) A is mid point of line PQ.
= ( 4)2 (8)2 ( 4)2 a 0
3= a=6
2
= 96 = 4 6 sq units
0 b
n and 4 = b=8
a ii 2
98. (a) We know that, tr (A) =
i 1 Y
1 5 7
If A = 0 7 9 , then P(0, b)
11 8 9
tr (A) = 1 + 7 + 9 = 17 A(3, 4)
cos sin 1
sin cos 1 Q(a, 0)
99. (a) Given, X
O
cos( ) sin( ) 1
[Applying R3 R3 – R1(cos ) + R2 (sin )] Thus, equation of line is
x y
cos sin 1 =1
6 8
= sin cos 1 4x + 3y = 24
0 0 1 sin cos 102. (d) The tangent at (1, 7) to the curve x2 = y – 6
is
= (1 + sin – cos ) (cos2
+ sin2 )
= 1 + sin – cos , which is independent of . 1
x= (y + 7) – 6
100. (d) 4 sin 2 x – 12sin x + 7 2
= 4 (sin2x – 3 sin x) + 7 2x = y + 7 – 12
2 y = 2x + 5
3 9 which is also tangent to the circle
=4 sin x 7
2 4 x2 + y2 + 16x + 12y + c = 0
i.e., x2 + (2x +5)2 + 16x + 12 (2x + 5) + c = 0
2 5x2 + 60x + 85 + c = 0, which must have
3
= 4 sin x 9 7 equal roots.
2 Let and are the roots of the equation.
2 Then + = – 12 = –6
3 ( = )
= 4 sin x 2
2 x = – 6, y = 2x + 5 = – 7
Point of contact is (–6, –7).
www.jeeneetbooks.in

WWW.IIT-NEET.XYZ

Solved Paper 2012 2012-25

103. (c) The intersection point of lines x – 2y = 1


1 1 x
7 1 105. (b) Let I = dx
0 1 x
and x + 3y = 2 is ,
5 5
Since, required line is parallel to 3x + 4y = 0. 1 1 x
= 0 dx
3 1 x2
Therefore, the slope of required line is .
4
1 1 1 x
Equation of required line which passes = dx dx
0 0
1 x2 1 x2
7 1
through , is given by
5 5 1 x
= [sin 1 x]10 dx
0
1 3 7 1 x2
y x
5 4 5 Put t2 = 1 – x2 2t dt = – 2x dx
3x 21 1 t dt = –x dx
y
4 20 5 0t
3x + 4y – 5 = 0 I = (sin–1 1 – sin–1 0) + 1
dt
t
dx
104. (c) Let I = = [t]10 1
sin x cos x 2 2 2
dx 1 1
106. (c) Let I = 0 x x dx
2 sin x sin cos x cos 1 2
4 4
1/ 2 1 1 1
1 dx =– x x dx x x dx
= 0 2 1/ 2 2
2
1 cos x
4 1/ 2 x 1 x
= 0 x 2 dx x x2 dx
2 1/ 2 2
1 dx
=
2 x 1/2 1
2 sin 2 x2 x3 x3 x2
2 8
= 4 3 3 4
0 1/2
1 2 x
= cosec dx
2 2 8 1 1 1 1 1 1
=
16 24 3 4 24 16
x
cot
1 2 8 6 4 32 24 4 6
= . C =
2 2 1 96 96
2
12 1
1 x =
= cot C 96 8
2 2 8
EBD_7443
www.jeeneetbooks.in

WWW.IIT-NEET.XYZ

2012-26 Target VITEEE


[using eq. (i)]
x2 y2 On substituting these values in x2 –y2 =
107. (b) Let the equation of the ellipse be
a2 b2 c2, we get
= 1.
a2 b2
It is given that it passes through (7, 0) and = c2
(0, –5). 2 2
Therefore, a2 = 49 and b2 = 25 a2 – b2 = 2c2
The eccentricity of the ellipse is given by 109. (c) Let P (x, y) be any point on the conic. Then,
x y 1
b2 (x 1) 2 (y 1) 2 2
e= 1 2
a2
(x – 1)2 + (y + 1)2 = (x – y + 1)2
25 24 2 6 2xy – 4x + 4y + 1 = 0
= 1 = 110. (b) Required numbers
49 49 7
1 1 1 1 1
x2 y2 = 5! 1 = 44
108. (c) =1 1! 2! 3! 4! 5!
a 2 b2 Note that if r (0 r n) objects occupy the
...(i) original places and none of the remaining
On differentiating w.r.t. x, we get (n – r) objects occupies its original places
then the number of such arrangements
2x 2y dy
2
. =0 = nCr. (n – r)!
a b 2 dx
1 1 1 1
1 .... ( 1) n 2
dy xb 2 1! 2! 3! (n r)!
and
dx a2y
12 22 32 ... n 2
x2 – y2 = c2 111. (b) Tn =
On differentiating w.r.t. x, we get n!

dy n2 n(n 1)(2n 1)
2x – 2y =0 = =
dx n! 6n!
dy x
= 1 2n 3 3n 2 n
dx y =
6 n!
The two curves will cut at right angles, if
dy dy 1 n3 3n 2 n
= –1 = 2.
dx c1 dx c2 6 n! n! n!
Sum of the series
b2 x x
. = –1
a2y y 1 n3 n2 n
= 2 3
6 n 1 n! n 1 n! n 1
n!
x2 y2
a2 b2 1
= (2 5e 3 2e e)
6
x2 y2 1
1 17
a 2
b 2 2 = (10e 6e e) e
6 6
www.jeeneetbooks.in

WWW.IIT-NEET.XYZ

Solved Paper 2012 2012-27

112. (b) loga (1 + x) = loge (1 + x) logae


3 1 1 3 1
n = 2 sin 2 sin 0
n 1 x 2 2 2
= logae ( 1)
n 1
n
3 3
So, the coefficient of xn in loga (1 + x) is = 2 = sq units
2 6 2 3
( 1) n 1
log a e. 1
n 115. (b) Let y = lim tan x
x 2
113. (b) Let the equation of the required plane be
Taking log on both sides, we get
x y z
= 1. 1
a b c 1
log y = lim log tan x form
This meets the coordinate axes at A, B and x x 2
C, the coordinates of the centroid of ABC
a b c form
are , ,
3 3 3

a b c 1
1, 2, 3
3 3 3 1 x2
= lim
a = 3, b = 6, c = 9 x 1
tan x
Hence, the equation of the plane is 2

x y z
=1 (using L ‘Hospitals’ rule)
3 6 9
114. (c) Required area 2x
(1 x 2 )2
Y = lim
x 1
x = 3y 1 x2
(using L Hospital’s rule)
X X 2x
O
= lim =0 y = e° = 1
2 2
x 1 x2
x +y =4
116. (c) f(x) is continuous at x = .
2
Y
So, lim f (x) lim f (x)
1 x x
= (x 2 x1 )dy 2 2
0

1
= ( 4 y2 3y) dy m 1 sin n
0 2 2
1 m
1 2 1 1 y 3y 2 m 1 1 n =n
= 2y 4 y 2
(4)sin
2 2
2 2
0
EBD_7443
www.jeeneetbooks.in

WWW.IIT-NEET.XYZ

2012-28 Target VITEEE

4 x2 3
117. (c) f(x) = 2 2
dy
sin 1 (2 x) 1
dx
2
4 x 2 is defined for 4 – x 0. 119. (a) =
d2 y
x2 4
–2 x 2 dx 2
and sin–1 (2 – x) is defined for –1 2 – x 1
–3 – x – 1 2 3/ 2
d2 y dy
1 x 3 1
Also, sin–1 (2 – x) = 0 for x = 2 dx 2 dx
Domain of f (x) = [–2 , 2] [1, 3] – {2} On squaring both sides, we get
= [1, 2)
118. (c) (1 + y2) dx + (1 + x2) dy = 0
2 2 3
2 d2y dy
1
dx dy dx 2 dx
=0
1 x 2 1 y2 Clearly, it is a second order differential
On integrating, we get equation of degree 2.
tan–1 x + tan–1 y = tan –1 C Note that the higher order derivative is in
x y the transcendental, then we do not
=C determined the degree of that equation.
1 xy 120. (b) Let R = {(a, b) : a, b N, a – b = 3}
x + y = C (1 – xy) = [{(n + 3), n} : n N]
= {(4, 1), (5, 2), (6, 3), ...}
www.jeeneetbooks.in

WWW.IIT-NEET.XYZ

VITEEE
SOLVED PAPER 2011

PART - I (PHYSICS) (a) 7/3 A from a to b through e


(b) 7/3 A from b to a through e
1. A glass rod rubbed with silk is used to change a (c) 1A from b to a through e
gold leaf electroscope and the leaves are (d) 1A from a to b through e
observed to diverge. The electroscope thus
7. An electric bulb of 100 W is connected to a
charged is exposed to X-rays for a short period.
supply of electricity of 220 V. Resistance of the
Then
filament is
(a) the divergence of leave will not affected
(b) the leaves will diverge further (a) 484 (b) 100
(c) the leaves will collapse (c) 22000 (d) 242
(d) the leaves will melt 8. Pick out the wrong statement.
2. An infinite number of charge, each of charge 1 C (a) In a simple battery circuit, the point of
are placed on the x-axis with coordinates x=1, 2, lowest potential is the negative terminal of
4, 8,...... If a charge of 1C is kept at the origin, the battery.
then what is the net force acting on 1C charge? (b) The resistance of an incandescent lamp is
(a) 9000 N (b) 12000 N greater when the lamp is switched off.
(c) 24000 N (d) 36000 N (c) An ordinary 100W lamp has less resistance
3. A cube of side is placed in a uniform field E, than a 60 W lamp.
where E = Ei . The net electric flux through the (d) At constant voltage, the heat developed in
cube is a uniform wire varies inversely as the length
(a) zero (b) l2E of the wire used.
(c) 4l E 2 (d) 6l2E 9. The electrochemical equivalent of magnesium is
4. The capacity of a capacitor is 4×10-6 F and its 0.126 mg/C. A current of 5A is passed in a suitable
potential is 100 V. The energy released on solution for 1h. The mass of magnesium
discharging it fully will be deposited will be
(a) 0.02 J (b) 0.04 J (a) 0.0378 g (b) 0.227 g
(c) 0.025 J (d) 0.05 J
(c) 0.378 g (d) 2.27 g
5. Dimensions of a block are 1cm×1cm×100cm. If
10. In producing chlorine through electrolysis
specific resistance of its material is 3×10-7 m,
100 W power at 125 V is being consumed. How
then the resistance between the opposite
much chlorine per minute is leberated? (ECE of
rectangular faces is
chlorine is 0.367 × 10–6 kg/C)
(a) 3×10-7 (b) 3×10-9
(a) 24.3 mg (b) 16.6 mg
(c) 3×10-5 (d) 3×10-3 (c) 17.6 mg (d) 21.3 mg
6. The magnitude and direction of the current in 11. A particle carrying a charge to 100 times the
the circuit shown will be charge on an electron is rotating per second in a
1 e 2 circular path of radius 0.8 m. The value of the
a b
magnetic field produced at the centre will be
10V 4V
( 0= permeability for vacuum)
7
10
(a) (b) 10-17 0
0

d c (c) 10-6 0 (d) 10-7 0


3
EBD_7443
www.jeeneetbooks.in

WWW.IIT-NEET.XYZ

2011-2 Target VITEEE


12. A rectangular loop carrying a current i is placed 19. Light of two different frequencies whose
in a uniform magnetic field B. The area enclosed photons have energies 1 eV and 2.5 eV,
by the loop is A. If there are n turns in the loop, respectively, successively illuminate a metal
the torque acting on the loop is given by whose work function is 0.5 eV. The ratio of the
(a) ni A × B (b) ni A · B maximum speed of the emitted electrons will be
l l (a) 1:5 (b) 1:4
(c) (iA×B) (d) (iA · B) (c) 1:2 (d) 1:1
n n
13. In a magnetic field of 0.05 T, area of a coil changes 20. An electron accelerated under a potential
from 101 cm2 to 100 cm2 without changing the difference V volt has a certain wavelength .
resistance which is 2 . The amount of charge Mass of proton is some 2000 times of the mass
that flow during this period is of the electron. If the proton has to have the
(a) 2.5 × 10–6 C (b) 2 × 10–6 C same wavelength , then it will have to be
(c) 10 C –6 (d) 8 × 10–6 C accelerated under a potential difference of
14. A solenoid has 2000 turns wound over a length
of 0.30 m . The area of its cross-section is (a) V volt (b) 2000 V volt
1.2 × 10-3 m2. Around its central section, a coil of V
300 turn is wound. If an initial current of 2A in (c) volt (d) 2000 V volt
2000
the solenoid is reversed in 0.25 s, then the emf
induced in the coil is 21. The ratio of momentum of an electron and
(a) 6× 10–4 V (b) 4.8× 10–3V -particle which are accelerated from rest by a
(c) 6× 10 V –2 (d) 48mV potential difference of 100 V is
15. An inductive circuit contains a resistance of
(a) 1 (b) 2m e / m a
10 and an inductance of 2.0 H. If an AC voltage
of 120 V and frequency of 60 Hz is applied to this
circuit, the current in the circuit would be nearly (c) me / m (d) me / 2m
(a) 0.32 A (b) 0.16 A 22. Sky wave propagation is used in
(c) 0.43 A (d) 0. 80 A
(a) radio communication
16. In a Millikan's oil drop experiment the charge on
an oil drop is calculated to be 6.35 × 10–19 C. The (b) satellite communication
number of excess electrons on the drop is (c) T V communication
(a) 3.2 (b) 4 (d) Both T V and satellite communication
(c) 4.2 (d) 6 23. The frequency of an FM transmitter without
1 1 signal input is called
17. The values and of spin quantum (a) the centre frequency
2 2
number show (b) modulation
(a) rotation of electron clockwise and anti- (c) the frequency deviation
clockwise directions respectively (d) the carrier sweing
(b) rotation of electron anti-clockwise and 24. What is the age of an ancient wooden piece if it
clockwise directions respectively is known that the specific activity of C14 nuclide
(c) rotation in any direction according to in its amounts is 3/5 of that in freshly grown
convention
trees? Given the half of C nuclide is 5570 yr.
(d) None of the above
18. The frequency of incident light falling on a (a) 1000 yr (b) 2000 yr
photosensitive metal plate is doubled, the kinetic (c) 3000 yr (d) 4000 yr
energy of the emitted photoelectrons is 25. A thin metallic spherical shell contains a charge
(a) double the earlier value Q on it. A point charge q is placed with the centre
(b) unchanged of the shell and another charge q1s placed
(c) more than doubled outside it as shown in the figure. All the three
(d) less than doubled charges are positive.
www.jeeneetbooks.in

WWW.IIT-NEET.XYZ

Solved Paper 2011 2011-3

Q 30. Five resistances are connected as shown in the


figure. The effective resistance between points
A and B is
2.5
B
q q1
7.5 10
1

The force on the charge at the centre is


(a) towards left (b) towards right A
(c) upward (d) zero 9
26. As shown in the figure, charges + q and – q are 10 10
placed at the vertices B and C of an isosceles (a) (b)
3 17
triangle. The potential at the vertex A is
(c) 40 (d) 45
A 31. A potentiometer is connected across A and B
and a balance is obtained at 64.0 cm. When
potentiometer lead to B is moved to C, a balance
a is found at 8.0 cm. If the potentometer is now
connected across B and C, a balance will be
b b found at
B -q C
+q
D B C
1 2a (a) 8.0 cm (b) 56.0 cm
(a) . (b) zero
4 a2 b2 (c) 64.0 cm (d) 72.0 cm
32. In an electromagnetic wave, the average energy
1 q 1 ( q) density associated with magnetic field is
(c) . (d) 4 .
4 a b 2 2
a2 b2 (a) L i 20 /2 (b) B2 / 2 0
27. On moving a charge of 20 C by 2 cm, 2 J of work
2
is done, then the potential difference between (c) 0B /2 (d) 0 / 2B2
the points is 33. An electromagnetic wave going through vacuum
(a) 0.1 V (b) 8 V is described by
(c) 2 V (d) 0.5 V E=E0 sin (kx – t)
28. The insulation property of air breaks down at Which of the following is/are independent of the
3 × 106V/m. The maximum charge that can be wavelength?
given to a sphere of diameter 5 m is nearly (a) k (b) 2
(a) 2×10–2C (b) 2×10–3C (c) k/ (d) k
(c) 2×10 C–4 (d) 2×10–5C 34. An ammeter reads upto 1A. Its internal resistance
29. Two capacitors of capacities C and 2C are is 0.81 . To increase the range to 10 A, the value
connected in parallel and then connected in
of the required shunt is
series with a third capacitor of capacity 3C. The
combination is charged with V volt. The charge (a) 0.09 (b) 0.03
on capacitor of capacity C is (c) 0.3 (d) 0.9
1 35. A coil of resistance 10 and inductance 5 H is
(a) CV (b) CV connected to a 100 V battery. Then the energy
2
stored in the coil is
3 (a) 250 J (b) 250 erg
(c) 2CV (d) CV (c) 125 J (d) 125 erg
2
EBD_7443
www.jeeneetbooks.in

WWW.IIT-NEET.XYZ

2011-4 Target VITEEE

36. A nucleus A X emits an - particle. The 2


Z 42. Amongst Ni(CO)4, [Ni(CN)4]2– and NiCl4
resultant nucleus emits a + – particle. The
respective atomic and mass number of final (a) Ni(CO)4 and NiCl 24 are diamagnetic but
nucleus will be [Ni(CN)4]2– is paramagnetic
(a) Z–3, A–4 (b) Z–1, A–4 (b) Ni(CO)4 and [Ni(CN)4]2– are diamagnetic
(c) Z–2, A–4 (d) Z, A–2 but NiCl 24 is paramagnetic
37. In Young's double slit experiment, the intensity
of light at a point on the screen where the path (c) NiCl24 and [Ni(CN)4]2– are diamagnetic
difference is = I. The intensity of light at a but Ni(CO)4 is paramagnetic
point where the path difference becomes /3 is
(a) I / 4 (b) I / 3 (d) Ni(CO)4 is diamagnetic but NiCl 24 and
[Ni(CN)4]2– is paramagnetic
I
(c) (d) I 43. The equivalent conductances of two ions at
2 infinite dilution in water at 25ºC are given below
38. Polarising angle for water is 53°4'. If light is o
incident at this angle on the surface of water and Ba 2
= 127.00 Scm2/ equiv..
reflected the angle of refraction is
o
(a) 53°4' (b) 126°56' = 76.00 Scm2/equiv..
Cl
(c) 36°56' (d) 30°4'
The equivalent conductance (in S cm2/equiv) of
39. A 2V battery, a 15 resistor and a potentiometer
BaCl2 at infinite dilution will be
of 100 cm length, all are connected in series. If
(a) 203 (b) 279
the resistance of potentiometer wire is 5 , then
(c) 205.5 (d) 139.5
the potential gradient of the potentiometer wire
44. The product formed when phthalimide is treated
is
with a mixture of Br2 and strong NaOH solution
(a) 0.005 V/cm (b) 0.05 V/cm is
(c) 0.02 V/cm (d) 0.2 V/cm (a) aniline (b) phthalamide
40. The output voltage of a transformer connected (c) phthalic acid (d) anthranilic acid
to 220 V line is 1100 V at 2 A current. Its efficiency 45. In a set of reactions acetic acid yielded a product
is 100%. The current coming from the line is D.
(a) 20A (b) 10A
SOCl2 Benzene HCN
(c) 11A (d) 22A CH3 COOH A B
anhy. AlCl3
PART - II (CHEMISTRY) H 2O
C D
41. An alkene having molecular formula C8H12 on The structure of D would be
ozonolysis yields glyoxal and 2, 2-dimethyl
OH
butane-1, 4-dial. The structure of alkene is
CH3 CH2 — C — CH3
CH3
(a) (b) CH3 (a) CN
H 3C
OH
CH3
H3C C — COOH
(c) (d) H3C (b)
CH3
www.jeeneetbooks.in

WWW.IIT-NEET.XYZ

Solved Paper 2011 2011-5

CN This reaction is called


(a) Reimer-Tiemann reaction
C — CH3
(c) (b) Lederer-Manasse reaction
OH (c) Sandmeyer reaction
COOH (d) Kolbe's reaction

CH2 — C — CH3 O
||
(d) SeO 2
OH 52. CH 3 C CH 3 X Se H 2 O : X
46. The alcohol having molecular formula C4H9OH,
when shaken with a mixture of anhydrous ZnCl2 O O O
and conc. HCl gives an oily layer product after || || ||
(a) CH 3 C C H (b) CH3 C OCH3
five minutes. The alcohol is
(a) H3C — (CH2)3 — OH
O
(b) (CH3)2CH — CH2OH ||
(c) (CH3)3C — OH (c) CH3 C CH2OH (d) None of the above
(d) H3C — CH(OH) CH2 — CH3 53. Which of the following will give Cannizzaro
47. p-toluidine and benzyl amine can be distinguished reaction?
by
(a) CH3CHO (b) CH3COCH3
(a) Sandmeyer's reaction
(b) Dye test (c) (CH3)3C – CHO (d) CH3CH2CHO
(c) Molisch test 54. The secondary structure of a protein refers to
(d) Gattermann reaction (a) -helical backbone
48. CH3CH2Br undergoes Wurtz reaction. We may (b) hydrophobic interactions
expect some of the following product (c) sequence of -amino acids
A : CH3CH2CH2CH3
(d) fixed configuration of the polypeptide
B : CH2 = CH2
backbone
C : CH3 — CH3
55. Self condensation of two moles of ethyl acetate
Select correct product.
in the presence of sodiumethoxide after
(a) Only A (b) A and B
acidification yields
(c) A, B and C (d) A and C
49. Sometimes explosion occurs while distilling (a) acetic acid (b) acetoacetic ester
ethers. It is due to the presence of (c) ethyl propionate (d) ethyl butyrate
(a) peroxides (b) oxides 56. Which one of the following will be most basic?
(c) ketones (d) aldehydes (a) Aniline (b) p-methoxyaniline
50. Glycerine is used as a preservative for fruits and (c) p-methyl aniline (d) Benzylamine
eatables because
57. Mn2O7 dissolves in water to give an acid. The
(a) it makes them sweet
colour of the acid is
(b) it acts as an insecticide
(a) green (b) blue
(c) it keeps the food moist
(d) all of the above (c) purple (d) red
58. "925 fine silver" means an alloy of
OH OH OH
(a) 7.5% Ag and 92.5% Cu
CH2OH
51. HCHO
OH
+ (b) 92.5% Ag and 7.5% Cu
or H
(c) 80% Ag and 20% Cu
(d) 90% Ag and 10% Cu
EBD_7443
www.jeeneetbooks.in

WWW.IIT-NEET.XYZ

2011-6 Target VITEEE


59. In which of the following octahedral complexes (a) 75 cm3 (b) 125 cm3
(c) 150 cm 3 (d) 250 cm3
of Co (At. no. 27), will the magnitude of 0 be the
highest? 67. 20 mL of 0.2 M NaOH is added to 50 mL of 0.2 M
(a) [Co(CN)6]3– (b) [Co(C2O4)3]3– acetic acid. The pH of this solution after mixing is
(c) [Co(H2O)6] 3+ (d) [Co(NH3)6]3+ (Ka = 1.8 × 10–5)
(a) 4.5 (b) 2.3
60. Assertion (A) Cu and Cd2+ are separated by
2+
(c) 3.8 (d) 4
first adding KCN solution and then passing H2S 68. Consider the followin g equation, which
gas. represents a reaction in the extraction of chromium
Reason (R) KCN reduces Cu2+ to Cu+ and forms from its ore
a complex with it. 2Fe2O3 . Cr2O3 + 4Na2CO3 + 3O2
The correct answer is
(a) Both (A) and (R) are true and (R) is the 2Fe 2 O 3 4Na 2 CrO 4 4CO 2
correct explanation of (A) Which one of the following statements about the
(c) Both (A) and (R) are true but (R) is not the oxidation states of the substances is correct?
correct explanation of (A) (a) The iron has been reduced from +3 to +2
(c) (A) is true but (R) is not true state.
(d) (A) is not true but (R) is true (b) The chromium has been oxidised from +3 to
61. The effective atomic number of cobalt in the +6 state.
(c) The carbon has been oxidised from +2 to +4
complex [Co(NH3)6]3+ is
state.
(a) 36 (b) 24 (d) There is no change in the oxidation state of
(c) 33 (d) 30 the substances in the reaction.
62. The IUPAC name for th e complex 69. The freezing point of a solution composed of 10.0
[Co(NO2)(NH3)5]Cl2 is g of KCl in 100 g of water is 4.5ºC. Calculate the
(a) nitrito-N-pentammine cobalt (III) chloride van't Hoff factor, i for this solution.
(b) nitrito-N-pentammine cobalt (II) chloride (a) 2.50 (b) 1.8
(c) pentaminenitrito-N-cobalt (II) chloride (c) 1.2 (d) 1.3
(d) pentaminenitrito-N-cobalt (III) chloride 70. In the reversible reaction,
63. The radio-isotope used for treatment of thyroid k1
2NO 2 N 2O 4
disorders is k2
(a) Na-24 (b) P-32 the rate of disappearance of NO2 is equal to
(c) Co-60 (d) I-131
2k1 2
64. Tetragonal crystal system has the following unit (a) NO 2
cell dimensions k2
(a) a = b = c, = = = 90º (b) 2k1[NO2]2 – 2k2[N2O4]
(b) a = b c, = = = 90º (c) 2k1[NO2]2 – k2[N2O4]
(c) a b c, = = = 120º (d) (2k1 – k2) [NO2]
71. A chemical reaction was carried out at 300 K and
(d) a = b c, = = 90º, = 120º
280 K. The rate constants were found to be k l and
65. A crystalline solid k2 respectively. Then
(a) changes rapidly from solid to liquid (a) k2 = 4kl (b) k2 = 2kl
(b) has no definite melting point (c) k2 = 0.25 kl (d) k2 = 0.5 kl
(c) undergoes deformation of its geometry 72. The rate constant of a reaction at temperature
easily 200 K is 10 times less than the rate constant at
(d) soften easily 400 K. What is the activation energy of the
66. Two glass bulbs A and B are connected by a very reaction?
small tube having a stop-cock. Bulb A has a volume (a) 1842.4 R (b) 460.6 R
of 100 cm3 and contained the gas while bulb B (c) 230.3 R (d) 921.2 R
was empty. On opening stop-clock, the pressure 73. A vessel at 1000 K contains CO2 with a pressure
fell down to 40%. The volume of the bulb B must of 0.5 atm. Some of the CO2 is converted into CO
be on the addition of graphite. The value of K if the
total pressure at equilibrium is 0.8 atm, is
www.jeeneetbooks.in

WWW.IIT-NEET.XYZ

Solved Paper 2011 2011-7

(a) 1.8 atm (b) 3 atm


(a) CN < O2 (b) CN = N2
(c) 0.3 atm (d) 0.18 atm
74. For the reaction 2A + B C, H = x cal, (c) N2 < O2 (d) H 2 He 2
which one of the following conditions-would
favour the yield of C on the basis of Le-Chatelier PART - III (MATHEMATICS)
principle?
(a) High pressure, high temperature 81. To the lines ax 2 2hxy by2 0 , the line
(b) Only low temperature
a 2 x2 2h(a b)xy b2 y2 0 are
(c) High pressure, low temperature
(d) Only low pressure (a) equally inclined
75. The EMF of the cell, (b) perpendicular
Mg | Mg2+ (0.0IM) || Sn2+ (0.1 M) | Sn at 298K is (c) bisector of the angle
(d) None of the above
Eo 2.34V, E o 0.14V 82. If R be a relation from A={1, 2, 3, 4} to B={1, 3, 5}
Mg 2 / Mg Sn 2 / Sn such that (a, b) R a < b, then ROR–1 is
(a) 2.17 V (b) 2.23 V (a) {(1,3) , (1, 5), (2, 3), (2, 5), (3, 5), (4, 5)}
(c) 2.51 V (d) 2.45 V (b) {(3, 1), (5, 1), (3, 2), (5, 2), (5, 3), (5, 4)}
76. Heat of formation, H of of an explosive (c) {(3, 3), (3, 5), (5, 3), (5, 5)
compound like NCl3 is (d) {(3, 3), (3, 4), (4, 5)}
(a) positive (b) negative 83. If x + iy =(1 – i 3 ) 100 , then find (x, y).
(c) zero
(d) positive or negative (a) 299 , 299 3 (b) 299 , 299 3
77. For the reaction,
(c) 299 , 299 3 (d) None of these
C3 H8 (g) 5O 2 (g) 3CO 2 (g) 4H 2 O(l)
84. For a GP, an 3 2n , n N. Find the
at constant temperature, H – E is
common ratio.
(a) RT (b) – 3RT
(a) 2 (b) 1/2
(c) 3RT (d) – RT
(c) 3 (d) 1/3
78. The favourable conditions for a spontaneous
reaction are a b c
85. If a, b, c are in HP, then , , will be
(a) T S > H, H = + ve, S = +ve b c c a a b
(b) T S > H, H = + ve, S = –ve in
(c) T S = H, H = –ve, S = –ve (a) AP (b) G P
(d) T H = H, H = + ve, S = +ve (c) HP (d) None of these
79. Compound A and B are treated with dil. HCl
separately. The gases liberated are Y and Z x2 2x 7
86. If <6, x R, then
respectively. Y turns acidified dichromate paper 2x 3
green while Z turns lead acetate paper black. The 3
compound A and B are respectively. (a) x 11 or x
2
(a) Na2CO3 and NaCl
(b) x 11 or x 1
(b) Na2SO3 and Na2S
(c) Na2S and Na2SO3 3
(c) x 1
(d) Na2SO3 and Na2SO4 2
80. Which of the following is correct comparison of
3
the stability of the molecules? (d) 1 x 11 or x
2
EBD_7443
www.jeeneetbooks.in

WWW.IIT-NEET.XYZ

2011-8 Target VITEEE


87. The number of ways of painting the faces of a
95. If the roots of the equation x 2 ax b 0 are c
cube of six different colours is
and d, then one of the roots of the equation
(a) 1 (b) 6
(c) 6! (d) 36 x2 2c a x c2 ac b 0 is
88. A line passes through (2, 2) and is perpendicular (a) c (b) d – c
to the line 3x+y=3. What is its y-intercept? (c) 2d (d) 2c
(a) 1/3 (b) 2/3 96. The sum of the coefficients of (6a–5b)n, where
(c) 1 (d) 4/3 n is a positive integer, is
89. The number of common tangents to the circles (a) 1 (b) –1
(c) 2n (d) 2n–1
x2 y2 4 and x 2 y2 6x 8 y 24 is 97. Find the value of (7.995)1/3 correct to four decimal
(a) 0 (b) 1 places.
(c) 3 (d) 4 (a) 1.9995 (b) 1.9996
(c) 1.9990 (d) 1.9991
90. If D is the set of all real x such that 1 e 1/ x 1 is 98. The values of constants a and b, so that
positive, then D is equal to
(a) ( ,1] (b) ( , 0) x2 1
lim ax b 0 are
x x 1
(c) (1, ) (d) ( , 0) (1, )
(a) a = 0, b = 0 (b) a =1, b = –1
(c) a = –1, b = 1 (d) a = 2, b = –1
1 cos x
91. Find the value of the limit lim 99. The projection of the vector i – 2j+k on the
x 0 x vector 4i – 4j+7k is
(a) 0 (b) 1
5 6 19
(c) 2 (d) does not exist (a) (b)
10 9
x2 4
92. Evaluate dx. 9 6
4 (c) (d)
x 16 19 19
1 x2 4 100. If a, b, c are three non-zero vectors such that
1
(a) tan c a + b + c=0 and m = a.b+ b.c + c.a, then
2 2 2x 2 (a) m < 0 (b) m > 0
(c) m = 0 (d) m = 3
1 1 x2 4 101. A line making angles 45 and 60 with the
(b) tan c
2 2 2 2 positive directions of the axes of x and y makes
with the positive direction of z-axis, an angle of
1 1 x2 4 (a) 60 (b) 120
(c) tan c
2 2 x 2 (c) 60 or 120 (d) None of these
(d) None of the above 1 0 0 1
102. If I = 0 1 , J= 1 0 and
1
3 /4
93. Evaluate dx cos sin
/ 4 1 cos x B= , then B is equal to
sin cos
(a) 2 (b) –2
(a) I cos J sin (b) I sin J cos
(c) 1/2 (d) –1/2
94. If one AM 'A' and two GM p and q are inserted (c) I cos J sin (d) I cos J sin
between two given numbers, then find the value 103. Which of the following is correct?
(a) Determinant is a square matrix
p2 q2 (b) Determinant is a number associated to a
of matrix
q p
(c) Determinant is a number associated to a
(a) A (b) 2A square matrix
(c) 3A (d) 4A (d) All of the above
www.jeeneetbooks.in

WWW.IIT-NEET.XYZ

Solved Paper 2011 2011-9

104. If , and are the roots of x3 + ax2 +b = 0, then (c) both p and q are true
(d) None of the above
113. In how many ways 6 letters be posted in 5
the value of is different letter boxes?
(a) 56 (b) 65
(c) 5! (d) 6!
(a) –a3 (b) a3–3b 114. If A and B be two sets such that A×B consists of
(c) a3 (d) a2–3b 6 elements. If three elements A × B are (1, 4), (2, 6)
105. If the axes are shifted to the point (1, –2) without and (3, 6), find B×A.
solution, then the equation 2x2 + y2 – 4x + 4y = 0 (a) {(1, 4), (1, 6), (2, 4), (2, 6),(3, 4), (3, 6)}
becomes (b) {(4, 1), (4, 2), (4, 3), (6, 1),(6, 2), (6, 3)}
(a) 2X2+3Y2 = 6 (b) 2X2 + Y2=6
(c) {(4, 4), (6, 6)}
(c) X2 + 2Y2 = 6 (d) None of these
(d) {(4, 1), (6, 2), (6, 3)}
x2 , x 0 115. Let f : R R be defined as f (x) = x2+1, find f –1(–5).
106. If f(x)= , then x = 0 is
2sin x, x 0
(a) (b)
(a) point of minima
(b) point of maxima (c) {5} (d) {–5, 5}
(c) point of discontinuity 116. If X is a poisson variate such that P(X = 1)
(d) None of the above = P(X = 2), then P(X=4) is equal to
107. In a group (G,*), then equation x * a = b has a 1 1
(a) 2 (b)
(a) unique solution b * a 1
2e 3e 2
(b) unique solution a 1 * b 2 1
(c) 2 (d)
(c) unique solution a 1 * b 1 3e e2
(d) many solutions 117. The area enclosed by y = 3x – 5, y = 0, x = 3 and
108. A die is rolled twice and the sum of the numbers x = 5 is
appearing on them is observed to be 7. What is (a) 12 sq units (b) 13 sq units
the conditional probability that the number 2 has 1
appeared at least once? (c) 13 sq units (d) 14 sq units
2
1 1 2 2 118. The order and degree of the differential equation
(a) (b) (c) (d)
2 3 3 5 2/3
dy d2y
109. The locus of the mid-points of the focal chord of 1 4 4 are respectively
dx dx2
the parabola y 2 4ax is
(a) y2 = a(x – a) (b) y2 =2a(x – a) 2
(a) 1, (b) 3, 2
2
(c) y = 4a(x – a) (d) None of these 3
110. Find the value of sin 12° sin 48° sin 54° .
2
1 1 (c) 2, 3 (d) 2,
(a) (b) 3
2 4 119. The solution of the differential equation
1 1 dy 2
(c) (d) 4 x y 1 , is
6 8 dx
111. In an equilateral triangle, the inradius, (a) (4x + y + 1) = tan (2x+ C)
circumradius and one of the exradii are in the (b) (4x + y +1)2 = 2tan (2x + C)
ratio (c) (4x + y +1)3 = 3tan (2x + C)
(a) 2 : 3 : 5 (b) 1 : 2 : 3 (d) (4x + y + 1) = 2tan (2x + C)
(c) 1 : 3 : 7 (d) 3 : 7 : 9 120. The system of equations 2x + y –5=0,
112. Let p and q be two statements. Then, p q is x – 2y + 1 = 9, 2x –14y – a = 0, is consistent. Then,
false, if a is equal to
(a) p is false and q is true (a) 1 (b) 2
(b) both p and q are false (c) 5 (d) None of these
EBD_7443
www.jeeneetbooks.in

WWW.IIT-NEET.XYZ

2011-10 Target VITEEE

SOLUTIONS
PART - I (PHYSICS) 5. (a) Here length l = 1 cm = 10 –2 m
Area of cross-section, A = 1 cm × 100 cm
1. (b) As charge on glass rod is +(ve) so charge = 10 –2 m2
on gold leaves will also be +(ve). Due to 1 cm
X-rays more electrons from leaves will be
emitted, so leaves becomes more positive 1 cm
and diverge further. 100 cm
2. (b) In figure there is the schematic diagram of 2
10
distribution of charges on x-axis Resistance, R = = 3 × 10 –7 ×
A 10 2

= 3 × 10 –7
6. (d) The current in the circuit will be clockwise
1C 1 C 1 C 1 C 1 C Since E1 (10 V) > E2 (4V)
x=1 x=2 x=3 x=8 1C E1 e E2 2
a b
10 V 4V
Using principle of superposition total force
acting on 1 C charge
6 6
1 1 1 10 1 1 10 3
F
4 0 (1)2 (2) 2
Applying Kirchhoff’s junction rule
6 6
1 1 10 1 1 10 –1 × i – 10 – 4 × –2 × i –3i = 0
...
(4)2 (8)2 i = 1A (a to b via e)
V 10 4
10 6
1 1 1 1 Current I = = =1A
... R 6
4 0 1 4 16 64
V 2 220 220
7. (a) Power P = = = 484 W
R 100
1 8. (b) When switched off, resistance of an
6
9 10 109 incandescent lamp increases.
1
1 9. (d) Using Faraday’s law of electrolysis,
4
Mass m = Zit
4 = 0.216 × 10 – 3 × 5 × 3600
9 109 10 6
12000 N
3 = 2.27 g
3. (a) Here, net flux is zero, as there is no charge 10. (c) P = 100 W and V = 125 V
residing inside the cube.
P 100
4. (a) Energy released on discharge of capacitor P = VI I= = A
V 125
1 Mass of chlorine liberated = zit
U CV 2
2
100
1 = 0.367 × 10– 6 × × 60 = 17.6 mg
= × 4 × 10 – 6 × (100)2 = 0.02J 125
2
www.jeeneetbooks.in

WWW.IIT-NEET.XYZ

Solved Paper 2011 2011-11

q 18. (b) Since, r y g


11. (b) Current, i = = 100 × e
t the wevelength of red light is greater than
2 i threshold wavelength and hence no
0
Magnetic field, Bcentre . electrons are emitted.
4 r
19. (c) If E is the energy of incident photon and
0 2 100e W0 the work function, then
.
4 r Available energy = E – W0
200 1.6 10 19 1 2
0
10 17 E – W0 = mv
4 0.8
0 2
12. (a) Torque, T = ni (A × B) 2( E W0 )
B A v=
13. (a) Magnetic Flux, m
Change in flux, d B dA v1 1 0.5 1
= 0.05(101 – 100) × 10 – 4 v2
=
2.5 0.5 2
= 5× 10 – 6 Wb
20. (c) According to question e P
d
Charge dQ =
R h h
6
or 2meQeV 2m pQ pVp
5 10
= 2.5 10 6 C
2 me QeV m p QpV p
di
14. (d) Induced emf e = M
dt me Qe
Vp V
N1 N 2 A di mp Qp
0
= .
l dt
1
7 3
(1)V
4 10
2000 300 1.2 10 2000
0.30
V
2 ( 2) volt
2000
0.25
–3 1 2
= 48.2 × 10 V = 48 mV 21. (d) As we know, mv eV
2
15. (b) Here, impedence z = R2 X L2
2eV
v
(10) 2
(2 60 2) = 753.7
2
m
V 120 p = mv = 2meV
=
Current i = = 0.159 A
Z 753.7
Now, pe = 2me e 100
16. (b) As we know, Q = ne
and p = 4m e 100
Q
Number of electron n =
e
pe me
6.35 10 19 = 2m
p
3.9 4
1.6 10 19 22. (a)
17. (c) 23. (c) Frequency deviation is the frequency of an
FM transmitter without signal input.
EBD_7443
www.jeeneetbooks.in

WWW.IIT-NEET.XYZ

2011-12 Target VITEEE

3 t Q
24. (d) Here, N0 = N0e Charge on capacitor, C =
5 V
t 5 CV
e Charge on capacitor of capacity C =
3 2
30. (a) It is a parallel combination of three
log e t = loge 5 or
t = loge
5 resistances, each of 10 .
3 3 The effective resistance between
points A and B.
or t = 1 loge 5
3 1 1 1 1
T 0.693 R 10 10 10
= 0.5 T
0.693 10
or R
5570 0.5 3
yr 4018.7 yrs 31. (b) Here, E1 64
0.693
E1 – E2 8
25. (d) Field inside the metallic shell is zero.
E2 l
26. (b) Potential at A
64 – l = 8 or l = 64 –8 = 56 cm
q q
0 B2
2 2
a b a b2 2
32. (b) Emag =
2 0
27. (a) Potential difference between two points in
an electric field k
33. (c) is independent of
W
VA – VB =
q0 GI g
where W is work done by moving charge 34. (a) Shunt S = I I
g
q0 from point A to B
Here, W = 2J, q0 = 20 c 0.81 1 0.81
= 0.09
10 1 9
2
VA – VB = = 0.1 volt E
20 35. (a) Final current, I = =10 A
R
1 Qmax Energy stored in the magnetic field
28. (b) Maximum electric field, Emax
4 0 R2
1
U= Li
9
9 10 Qmax 2
3 × 106 =
2.5 2.5 1
× 5 × (10)2 = 250 J
3 10 6 2.5 2.5 2
or Qmax
9 109 36. (a) A
Z X 4
2 He A 2
Z 2 Y
= 2×10 – 3 C A 4
Y e A 4
Y'
Z 2 Z 3
29. (a) In parallel combination, the potential + emission.
difference across each individual capacitor During
1 1 +
is the same as the potenital difference across 1p 0n +
the combination. The proton changes into neutron. So,
So, potential difference across capacitor of charge number decreases by 1 but mass
number remains unchanged.
V
capacity C is .
2
www.jeeneetbooks.in

WWW.IIT-NEET.XYZ

Solved Paper 2011 2011-13

4a 2 cos2 43. (b) The equivalent conductance of BaCl2 can


37. (a) Intensity, I
2 be calculated as
In the first case, 2 ( BaCl2 ) 2
( Ba 2 ) (Cl )
2
I' 4a = 127 + 2 × 76
2 = 127 + 152
In the second case, = 279 Scm2/equiv.
3
2 O
I' 4a 2 cos 2 or I ' a2
3 C
I' 1 1 44. (d) NH + Br2 + 4NaOH
or I ' C
I 4 4
38. (c) Here, p + r = 90° O
r = 90° – p
phthalimide
or r = 90° – ' = 36°56
dv e R O
39. (a) Potential gradient .
dx ( R Rh r) L C OH
+ 2NaBr + Na2CO3+H2O
2 5 NH2
0.5 V / m
(15 5 0) 1
anthranilic acid
= 0.005 V/cm
40. (b) For 100% efficiency SOCl2 Benzene
45. (b) CH3COOH CH3COC1
Anhy.AlCl3
Vsis =Vpip
1100 × 2 = 220 × ip O
i.e., Primary current, ip = 10A
C CH3
PART - II (CHEMISTRY) HCN
41. (b) The products formed as a result of (B)
ozonolysis suggests
OH
CH 3
| CHO OH
CH3 C CH3
O CH CH 2 C CHO | CH3 +
| CHO H C CH3
CH 3 CN
(C) COOH
2, 2 - dimethyl butane glyoxal (C8H12)
1, 4 - dial (D)

42. (b) The electronic configuration of Ni is 46. (d) Secondary alcohol, when shaken with a
mixture of anthydrous ZnCl2 and conc. HCl
Ni(28) = [Ar] 3d8, 4s2
(Lucas regent) gives an oily layer product
Ni2+ = [Ar] 3d 8 after five minutes.
Both Ni and Ni 2+ have two unpaired
electrons. Lucas
reagent
CO and CN– are strong field ligands and H3C CH CH 2 CH3
thus unpaired electrons get paired. Hence, |
Ni(CO)4 and [Ni(CN)4]2– are diamagnetic. OH
Cl– is a weak field ligand hence, no pairing oily layer product after 5 min.
The amines in which amino group is directly
of e– will take place. hence NiCl 24 is attached to benzene ring undergo
paramagnetic. diazotisation reaction.
EBD_7443
www.jeeneetbooks.in

WWW.IIT-NEET.XYZ

2011-14 Target VITEEE


47. (b) p-toluidine contains amino group attached
directly to benzene ring, thus undergo O O O
|| || ||
diazotisation reaction and gives red dye. In SeO 2
CH3 C CH3 CH3 C C H
benzyl amine, the amino group is not
directly attached to benzene ring. Hence, it
will not undergo diazotisation reaction. Se H 2O
53. (c) Aldehydes which have no -H atom give
NH2
Cannizzaro’s reaction, (CH3)3C – CHO does
CH2NH2
not contain -H atom, hence it will give
Cannizzaro reaction.
CH3
CH3
benzyl amine 2CH3 C CHO + KOH
p-toluidine
|
Thus, these two can be distinguished by CH3
dye test.
CH3 CH3
48. (c) C2H5Br + Na• CH3CH 2 + NaBr |
CH3 C CH2OH + CH3— C — COOK
Intermediate free radical CH 3CH 2 | |
combines to form CH3CH2 – CH2CH3 (as a CH3 OH
main product) and also CH2 == CH2 and
54. (a) Secondary structure involves -helical and
CH3CH3 by disproportion
-pleated sheet like structure. Where as
CH3CH 2 + CH3CH 2 CH3CH2 – CH2CH3 Primary strucure involves sequence of
n-butane -amino acids polypeptide chain.
CH3CH 2 + CH3CH 2 CH2 CH2 + CH3 – CH3 55. (b)
(disproportion reaction)
NaOC2 H5
2CH3 COOC2 H5 CH3COCHNaCOOC2H 5
49. (a) Ethers, In the presence of air and light form
peroxides +HCl
50. (c) Glycerine is used as a preservative because
it keeps the fruit moist. CH3COCH2COOC2H5
51. (b) acetoacetic ester

OH OH
– + CH2OH 56. (d) In aniline, p-methoxyaniline and p-methyl
OH or H aniline, the lone pair of electrons on the N-
+ HCHO
(Lederer - Menasse atom is delocalised on the benzene ring
reaction) while in benzylamine it is delocalised, and
more available for donation. Hence
OH
benzylamine is most basic among the given.
57. (c) Mn 2 O 7 dissolves in water to give
+
permanganic acid which is purple in colour.
Mn2O7 + H2O 2HMnO4
CH2OH Purple
52. (a) In presence of SeO2 compounds containing 58. (b) “925 fine silver” means 925 parts of pure
active methylene (i.e., CH2 next to the Ag in 1000 parts of an alloy. Therefore, in
carbonyl group) oxidises to another CO terms of percentage, it will be 92.5% Ag
group. and 7.5% Cu.
www.jeeneetbooks.in

WWW.IIT-NEET.XYZ

Solved Paper 2011 2011-15

59. (a) Spectrochemical series. According to the 3 3


strength of splitting is as: 68. (b) 2 Fe 2 O3 Cr 2 O3 4Na 2 CO3 3O2
3 6
CO CN NO2 en NH3 py NCS 2 Fe2 O3 4Na 2 Cr O 4 4CO 2
2 2–
H2O O ox OH F C1 Hence, the oxidation state of chromium increase
SCN S 2
Br I from + 3 to + 6.
Crystal field stabilisation energy (CFSE) for 69. (b) T f = iK f m
octahedral complex, depends on the strength 10 1000
of negative ligand. [Higher the strength more m= 1.34m
74.55 100
will be the CFSE]
60. (b) KCN forms complex with Cu+ and Cd2+ as [Molecular Mass of KCl = 74.55]
K3[Cu(CN)4] and K2[Cd(CN)4] respectively. Tf 4.5 C
On passing H2S, only Cd2+ complex gets i 1.8
Kfm (1.86 C / m)(1.34m)
decomposed to yellow CDS.
61. (a) EAN =at. no. of central atom – oxidation k1
state +2 (number of ligands) 70. (c) 2NO2 k2 N2O4
= 27 –3 + 2(6)
1 d[NO 2 ]
= 27 –3 +12 = 36 Rate = = k1[NO2]2– k2[N2O4]
62. (d) [Co(NO2)(NH3)5]Cl2 2 dt
Pentamminenitrito-N-cobalt (III) chloride. Rate of disappearance of NO2 i.e.,
63. (d) I-131 radio-isotope is used for thyroid d[NO 2 ]
disorders. – =2k1[NO2]2 – k2[N2O4]
dt
64. (b) Tetragonal crystal system has the unit cell
71. (c) For every 10° rise in temperture, Rate gets
dimensions
double. Hence, in this case rate constant
a = b c, = = = 90° will become four time. (d) i.e.,
65. (c) A crystalline solid undergoes deformation k1 = 4k2
of its geometry by application of pressure k2 = 0.25k1
or heat. as the Rise in temperature given is 20°C
40 p 72. At T1 = 200 K, if k1 = k
66. (c) 100 × p = V2 × then at T2 = 400 K, k2 = 10k
100
V2 = 250 cc (Total volume) 10k Ea 400 200
Volume of bulb B = 250 – 100 = 150 cc log =
k 2.303R 400 200
67. (a) NaOH + CH3COOH CH3COONa + H2O Ea = 921.2R
m mol 20 × 0.2 50 ×0.2 0 0
added = 4 = 10 73. (a) CO2(g) + C(s) 2CO(g)
m mol after
Initial 0.5 atm
reaction 0 6 4 4
pressure
6 4 At equi (0.5 – x)
[CH3COOH = : [CH3COONa] =
70 70 Total no. of moles at equilibrium
Now since for a basic buffer = (0.5 – x) + 2x = 0.8
[salt] x = 0.3 atm.
pH = –logka+ log pCO = 0.5 – 0.3 = 0.2
[base] 2
PCO = 2x = 2 0.3 = 0.6 atm
4 / 70
pH = – log (1.8 ×10–5) + log 2 2
6 / 70 pCO 0.6
= 4.56 K= p 0.2 = 1.8 atm
CO 2
EBD_7443
www.jeeneetbooks.in

WWW.IIT-NEET.XYZ

2011-16 Target VITEEE


74. (a) 2A + B C, H = x cals. 1 0
BO = =0.5
Since the reaction is endothermic and np < nr. 2
Hence, high pressure and high temperature Molecular orbital configuration of He2+ is
will favour forward reaction.
75. (b) The cell reaction is He+2(3) = 1s2, * 1s1
Mg + Sn2+ Mg2+ + Sn
2 1
2 BO = = 0.5
0.0591 [Mg ] 2
Ecell = E°cell – log
2 [Sn 2 ] Hence, stability of H+2 > stability of He+2.

0.0591 10 2 PART - III (MATHEMATICS)


= (2.34 – 0.14) – log = 2.23 V
2 1 81. (a) The equation of the bisectors of the angle
10
76. (a) Explosive compound has high heat content between the lines given by
i.e., it is formed by absorption of heat. ax2 + 2hxy + by2 = 0 is

Hence, H f is positive. x 2 y2 xy
...(i)
a b h
77. (b) C3H8(g) + 5O2(g) 3CO2(g) + 4H2O(l)
And the equation of the bisectors of the
ng = np – nr angle between the lines given by
= 3 – 6 = –3 a2x2 + 2h (a + b) xy + b2y2 = 0 is
H = E + ngRT
H = E – 3RT x2 y2 xy
2 2 h(a b)
H – E = –3RT a b
78. (a) G= H–T S
x2 y2 x
For a spontaneous reaction, G should be ...(ii)
negative. Hence, H = +ve, S = +ve and a b h
T S> H From eqs. (i) and (ii), it is clear that both
79. (b) Gas (Y) SO2 which can be obtained from the pair of straight lines have the same
Na2SO3(A) and gas (Z) is H2S which can be bisector, hence, the given two pairs of
obtained from Na2S(B). straight lines are equally inclined.
This can be easily understood by the 82. (c) A = {1, 2, 3, 4} and B = {1, 3, 5}
following reactions. R = {(1, 3), (1, 5), (2, 3), (2, 5), (3, 5), (4, 5)}
and R–1 = {(3, 1), (5, 1), (3, 2), (5, 2), (5, 3),
Na2SO3 + 2HCl 2NaCl + SO2 + H2O
(5, 4)}
(A) (Y) ROR–1 = {(3, 3), (3, 5), (5, 3), (5, 5)}
Na2S + 2HCl 2NaCl + H2S
100
(B) (Z) 1 i 3
83. (c) (1 i 3)100 2100
K2Cr2O7 + H2SO4 + 3SO2 K2SO4 2 2
+ Cr2(SO4)3 + H2O
= 2100 100
= 2100
Pb(CH3COO)2 + H2S PbS + 2CH3COOH
black 100 1 3i
80. (d) Higher the bond order higher will be the =2 = – 299 299 3i
2 2
stability.
When the bond order of two molecules are Now, x + iy = (1 i 3)100
same, the molecule with least number of
antibonding electrons is more stable. = 299 299 3i
Molecular orbital configuration of H2+ is x = –299, y = 299 3
H+2 (1) = 1s1 (x, y) = (–299, –299 3)
www.jeeneetbooks.in

WWW.IIT-NEET.XYZ

Solved Paper 2011 2011-17

84. (a) Given, an = 3 (2 ) n 89. (b) The centres of the given circles x + y2 = 4 2

an+1 = 3 (2n+1) and x2 + y2 – 6x – 8y = 24 are C1 (0, 0) and


C2 (3, 4) respectively. Their radii are r 1 = 2
an 1 3(2n 1 )
r= =2 and r2 = 7 respectively.
3(2 n )
an C1C2 = 5 < sum of radii
85. (c) a, b, c are in HP. But C1C2 = difference of radii
1 1 1 Thus, the given circles touch each other
, , are in AP.. internally.
a b c
Hence, number of common tangent is only
a b c a b c a b c one.
, , are in AP..
a b c 1
1
b c a c a b 90. (d) 1 ex 0
1 ,1 ,1 are in AP..
a b c 1
1 1
b c a c a b ex 1 1 log1
, , are in AP.. x
a b c
1 1
a b c 1 0 1
, , are in HP.. x x
b c c a a b
x ( , 0) (1, )
2
x 2x 7
86. (d) 6 x
2x 3 2 sin
1 cos x 2
x2 2x 7 91. (d) lim lim
6 0 x 0 x x 0 x
2x 3
x
x 2 10x 11 sin
0 2
2x 3 = 2 lim
x 0 x
(x 11)(x 1)
0 It can be easily seen that LHL at x = 0 is
2x 3
(x 11)(x 1)(2x 3) 1 1
0 x= – and RHL at x = 0 is .
(2x 3) 2 2 2
(x – 11) (x + 1) (2x + 3) < 0 1 cos x
Hence, lim does not exist.
3 x 0 x
x ,
( 1,11)
2
87. (a) The number of faces is equal to the number x2 4
92. (a) Let I = dx
of colours. x 4 16
Therefore the number of ways of painting
them is 1. 4 4
88. (d) Equation of line perpendicular to 1 1
2
x dx = x2 dx
3x + y – 3 = 0 is = 2
x – 3y + c = 0 2 16 4
x x 8
Since, it passes through (2, 2). x2 x
c=4
Equation of line perpendicular to 4
3x + y – 3 = 0 is x – 3y + 4 = 0 Putting x = t,
x
For y - intercept, put x = 0
4 4
y= So that 1 dx = dt
3 x2
EBD_7443
www.jeeneetbooks.in

WWW.IIT-NEET.XYZ

2011-18 Target VITEEE

dt 97. (b) (7.995) = (8 – 0.005)


1/3 1/3

I= 1/ 3
t2 (2 2 )2 0.005
=2 1
8
1 1 t
= tan C
2 2 2 2 1 0.005
=2 1 = 2 [1 – 0.000208]
3 8
4
x = 1.9996
1 1 x
I tan C
2 2 2 2 x2 1
98. (b) lim ax b = 0
x x 1
2
1 x 4
= tan 1 C x 2 (1 a) (a b)x b 1
2 2 2x 2 lim
x x 1 =0
3 /4 1 3 /4 1 cos x
93. (a) dx dx 1 – a = 0 and a + b = 0 a = 1, b = –1
/4 1 cos x /4 sin 2 x 99. (b) Let a = i – 2j + k and b = 4i – 4j + 7k, then
3 /4 projection of a on b is equal to
= (cosec2 x – cot x cosec x ) dx
/4
a.b 4 8 7 19
3 /4 =
= cot x cosec x |b| 16 16 49 9
/4
100. (a) Here, a + b + c = 0
= (1 2) ( 1 2) = 2 |a + b + c|2 = 0
94. (b) Let a and b are two numbers. |a|2 + |b|2 + |c|2 + 2 {a.b + b.c + c.a} = 0
a b a.b + b.c + c.a
Then, A = ...(i)
2 1
=– {|a|2 + |b|2 + |c|2} < 0
2
Also, a, p, q, b are in GP. m<0
p q b 101. (a) Let is the required angle, then
a p q cos245° + cos2 60° + cos2 = 1
1 1
p2 q2 cos2 =1
= a and =b 2 4
q p
3 1
2 2 cos2 = 1 –
p q 4 4
= a + b = 2A [Using eq. (i)]
q p 1
95. (b) f(x) = x2 + ax + b, then cos = = 60°
2
f(x + c) = (x + c)2 + a (x + c) + b
= x2 + (2c + a) x + c2 + ac + b cos sin
which shows that the roots of f (x) 102. (a) B= sin cos
are transformed to (x – c) i.e., roots of
f (x + c) = 0 are c – c and d – c. cos 0 0 sin
Hence, one of the roots of the equation = 0 cos sin 0
f(x + c) is (d – c).
96. (a) For the sum of the coefficients in the 1 0 0 1
expansion of (6a – 5b)n, put a = b = 1 = cos sin
Sum of the coefficients = (6 – 5)n 0 1 1 0
= 1n = 1 = I cos + J sin
www.jeeneetbooks.in

WWW.IIT-NEET.XYZ

Solved Paper 2011 2011-19

103. (c) According to the definition of determinant, 109. (b) Any chord PQ which bisected point
determinant is a number associated to a R (h, k) is T = S
square matrix. i.e., ky – 2a (x + h) = k2 – 4ah
104. (c) are the roots of given equation, Since, it is a focal chord, so it must pass
Therefore + + = – a through focus (a, 0).
+ + =0 k (0) – 2a (a + h) = k2 – 4ah
and = –b k2 = 2ah – 4a2
Hence, locus is
y2 = 2a (x – a)
Now, =–( + + ) 110. (d) sin 12° sin 48° sin 54°
= sin 12° sin (60° – 12°) sin (90° – 36°)
( 2+ 2+ 2– – ) sin12 sin(60 12 ) sin 72 cos 36
= – ( + + ) [( + + )2 =
sin 72
– 3( + + )]
= – (–a) (a2 – 0) = a3 [sin12 sin(60 12 )sin(60 12 )cos36 ]
=
105. (b) Substituting x = X + 1 and y = Y – 2 in sin 72
given equation, we get
2 (X + 1)2 + (Y – 2)2 – 4(X + 1) + 4 (Y – 2) = 0 sin12 (sin 2 60 sin 2 12 ) cos 36
=
2X2 + Y2 = 6 sin 72

x2 , x 0 3
106. (a) Given f (x) = sin12 sin 2 12 cos36
2sin x, x 0, 4
=
sin 72
2x, x 0
non differentiable, x 0 3sin12 4sin3 12 ) cos 36
f (x) = = .
4 sin 72
2 cos x, x 0
So, x = 0 is a critical point f(0 ) > 0 as well as
– sin 36 cos 36 1 sin 72 1
= =
f (0+) > 0 and f (0) = 0 4 sin 72 2 4sin 72 8
Hence, it is a point of minima.
107. (a) x * a = b (x * a) * a–1 = b * a–1 3 2 3a
111. (b) We have, = a ,s
x *(a * a–1) = b * a–1 4 2
x *e = b * a–1
a
x = b * a–1 Inradius r =
108. (b) Let A and B be two events such that s 2 3
A= getting number 2 at least once
B = getting 7 as the sum of the numbers on abc a3 a
Circumradius R = 2
two dice 4 3a 3
Here,
A = {(2, 1), (2, 2), (2, 3), (2, 4), (2, 5), (2, 6), 3 / 4a 2
(1, 2), (3, 2), (4, 2), (5, 2), (6, 2)} and exradii r1 =
s a a/2
and
B = {(2, 5), (5, 2), (6, 1), (1, 6), (3, 4), (4, 3)} 3
= a
11 6 2
P(A) = , P(B) = Required ratio = r : R : r 1
36 36
2 a a 3
P(A B) = : : a = 1 : 2 : 3.
36 2 3 3 2
Required probability 112. (b) p q is false only when both p and q are
P(A B) 2 / 36 2 1 false.
P (A/B) =
P(B) 6 / 36 6 3
EBD_7443
www.jeeneetbooks.in

WWW.IIT-NEET.XYZ

2011-20 Target VITEEE


113. (a) Since, each letter can be posted in any one 3
2
of the five different letter boxes. So, a letter dy 3 d2 y
can be posted in 5 ways. Since, there are 1 4 4
dx dx 2
six letters and each can be posted in 5 ways.
So, total number of ways Here, order is 2 and degree is 3.
= 5 × 5 × 5 × 5 × 5 × 5 = 56 dy
114. (b) Since, (1, 4), (2, 6) and (3, 6) are the elements 119. (d) = (4x + y + 1)2 …(i)
dx
of A × B, therefore 1, 2, 3 are the elements
Put 4x + y + 1 = v
of A and 4, 6 are the elements of B.
Also, A × B has 6 elements dy dv
4
A = {1, 2, 3} dx dx
and B = {4, 6}
B × A = {(4, 1), (4, 2), (4, 3), (6, 1), (6, 2), dv
4 = v2 ( From eq. (i))
(6, 3)} dx
115. (b) Let f –1 (–5) = x. Then, f (x) = – 5 dv
x2 + 1 = – 5 x2 = –6 x = ± dx
6 v 2
4
which does not belong to R.
f –1 (–5) = 1 1 v
tan x C
116. (c) Given : P (X = 1) = P (X = 2) 2 2
2
e e 4x y 1
1
tan 2x C
1! 2! 2
=2 4x + y + 1 = 2 tan (2x + C)
2 4 120. (d) Given system of equations are
e 2 2
P (X = 4) = 2x + y – 5 = 0 ...(i)
4! 3e2 x – 2y + 1 = 0 ...(ii)
5 and 2x – 14y – a = 0 ...(iii)
117. (d) Required area = (3x 5)dx This system is consistent
3

5 2 1 5
3x 2 75 27 1 2 1 =0
= 2 5x = 25 15
2 2 2 14 a
3

48 2 (2a + 14) – 1 (–a –2) – 5 (–14 + 4) = 0


= 10 = 14 sq units 4a + 28 + a + 2 + 50 = 0
2
118. (c) Given differential equation is 5a = –80 a = –16
2
dy 3 d2 y
1 4 4
dx dx 2
www.jeeneetbooks.in

WWW.IIT-NEET.XYZ

VITEEE
SOLVED PAPER 2010

PART - I (PHYSICS) 6. An electric dipole consists of two opposite


charges of magnitude q =1×10–6 C separated by
1. A straight wire carrying current i is turned into a
2.0 cm. The dipole is placed in an external field of
circular loop. If the magnitude of magnetic
moment associated with it in MKS unit is M, the 1×105 NC–1. What maximum torque does the field
length of wire will be exert on the dipole? How much work must an
external agent do to turn the dipole end to end,
4 4 M starting from position of alignment ( = 0°) ?
(a) (b)
M i
(a) 4.4 × 106N-m, 3.2 × 10–4J
4 i M (b) –2 × 10–3N-m, – 4×103J
(c) (d)
M i (c) 4 × 103N-m, 2 × 10– 3J
2. The ratio of the amounts of heat developed in (d) 2 × 10–3N-m, 4 × 10–3J
the four arms of a balance Wheatstone bridge,
7. The electron of hydrogen atom is considered to
when the arms have resistances P=100 ,
be revolving round a proton in circular orbit of
Q=10 , R=300 and S=30 respectively is
radius h2/me2 with velocity e2/h, where h= /2 .
(a) 3 : 30 : 1 : 10 (b) 30 : 3 : 10 : 1
(c) 30 : 10 : 1 : 3 (d) 30 : 1 : 3 : 10 The current i is
3. An electric kettle takes 4 A current at 220V. How 2
much time will it take to boil 1 kg of water from 4 me5 4 2
me5
(a) 2 (b) 3
temperature 20 °C ? The temperature of boiling
water is 100°C.
(a) 12.6 min (b) 4.2 min 4 2
m 2e 2 4 2
m 2 e5
(c) 6.3 min (d) 8.4 min (c) 3 (d) 3
4. Magnetic field at the centre of a circular loop of
area A is B. The magnetic moment of the loop will 8. In a double slit experiment, 5th dark fringe is
be formed opposite to one of the slits, the
BA 2 BA 3/ 2 wavelength of light is
(a) (b)
0 0
d2 d2
(a) (b)
BA3 / 2 2BA3 / 2 6D 5D
(c) 1/ 2 (d) 1/ 2
0 0
d2 d2
5. In Young's double slit experiment, the spacing (c) (d)
15D 9D
between the slits is d and wavelength of light
used is 6000 Å. If the angular width of a fringe 9. Which of the following rays is emitted by a
formed on a distance screen is 1°, then value of d human body?
is
(a) 1 mm (b) 0.05 mm (a) X-rays (b) UV rays
(c) 0.03 mm (d) 0.01 mm (c) Visible rays (d) IR rays
EBD_7443
www.jeeneetbooks.in

WWW.IIT-NEET.XYZ

2010-2 Target VITEEE


10. A proton of mass 1.67 × 10–27kg enters a uniform 15. A small coil is introduced between the poles of
magnetic field 1T of at point A shown in figure an electromagnet so that its axis coincides with
with a speed of 107 ms–1. the magnetic field direction. The number of turns
is n and the cross-sectional area of the coil is A.
When the coil turns through 180° about its
45 B
A diameter, the charge flowing through the coil is
Q. The total resistance of the circuit is R. What is
C the magnitude of the magnetic induction?
QR 2QR
(a) (b)
nA nA
The magnetic field is directed normal to the plane
of paper downwards. The proton emerges out of Qn QR
(c) (d)
the magnetic field at point C, then the distance 2RA 2nA
AC and the value of angle will respectively be 16. The attenuation in optical fibre is mainly due to
(a) absorption (b) scattering
(a) 0.7 m , 45 (b) 0.7 m, 90
(c) neither absorption nor scattering
(c) 0.14 m, 90 (d) 0.14 m, 45 (d) Both (a) and (b)
11. A neutral water molecule (H2O) in its vapour state 17. An arc of radius r carries charge. The linear
has an electric dipole moment of magnitude density of charge is and the arc subtends an
6.4×10–30C-m. How far apart are the molecules
centres of positive and negative charges? angle at the centre. What is electric potential
(a) 4 m (b) 4mm 3
(c) 4 m (d) 4 pm at the centre?
12. Figure shows a straight wire length l carrying
current i. The magnitude of magnetic field (a) (b)
4 0 8 0
produced by the current at point P is

P (c) (d)
12 0 16 0
i 18. Sinusoidal carrier voltage of frequency 1.5 MHz
and amplitude 50 V is amplitude modulated by
sinusoidal voltage of frequency 10 kHz producing
50% modulation. The lower and upper side-band
2 0i 0i frequencies in kHz are
(a) (b)
l 4 l (a) 1490, 1510 (b) 1510, 1490
1 1 1 1
2 0i 0i (c) , (d) ,
(c) (d) 1490 1510 1510 1490
8 l 2 2 l
13. Zener diode is used for 19. 50 and 100 resistors are connected in series.
(a) producing oscillations in an oscillator This connection is connected with a battery of
(b) amplification 2.4 V. When a voltmeter of 100 resistance is
(c) stabilisation connected across 100 resistor, then the
(d) rectification reading of the voltmeter will be
14. Two light sources are said to be coherent if they (a) 1.6V (b) 1.0V
are obtained from (c) 1.2V (d) 2.0V
(a) two independent point sources emitting 20. In space charge limited region, the plate current
in a diode is 10 mA for plate voltage 150V. If the
light of the same wavelength
plate voltagte is increased to 600V, then the plate
(b) a single point source
current will be
(c) a wide source
(a) 10 mA (b) 40 mA
(d) two ordinary bulbs emitting light of (c) 80 mA (d) 160 mA
different wavelengths
www.jeeneetbooks.in

WWW.IIT-NEET.XYZ

Solved Paper 2010 2010-3

21. Light of wavelength strikes a photo-sensitive 27. Using an AC voltmeter the potential difference
surface and electrons are ejected with kinetic in the electrical line in a house is read to be 234V.
energy E. If the kinetic energy is to be increased If line frequency is known to be 50 cycles/s, the
to 2E, the wavelength must be changed to equation for the line voltage is
wher
(a) V=165sin (100 t)
(a) (b) (b) V=331sin (100 t)
2
2 (c) V=220sin (100 t)
(d) V=440sin (100 t)
(c) (d)
2 28. There are a 25W – 220 V bulb and a 100W–220V
22. The maximum velocity of electrons emitted from line.Which eletric bulb will glow more brightly?
a metal surface is v, when frequency of light (a) 25W bulb
falling on it is f. The maximum velocity when
(b) 100W bulb
frequency becomes 4f is
(a) 2v (b) > 2v (c) Both will have equal incadescene
(c) < 2v (d) between 2v and 4v (d) Neither 25 W nor 100 W bulb will give light
23. The collector plate in an experiment on 29. Silver has a work function of 4.7 eV. When
photoelectric effect is kept vertically above the ultraviolet light of wavelength 100 nm is incident
emitter plate. Light source is put on and a upon it , potential of 7.7 V is required to stop
saturation photo-current is recorded. An electric photoelectrons from reaching the collector plate.
field is switched on which has a vertically The potential required to stop electrons when
downward direction, then light of wavelength 200 nm is incident upon silver
(a) the photo-current will increase
is
(b) the kinetic energy of the electrons will
increase (a) 1.5V (b) 1.85V
(c) the stopping potential will decrease (c) 1.95V (d) 2.37V
(d) the threshold wavelength will increase 30. Two particles X and Y having equal charges, after
24. A cylindrical conductor of radius R carries a being accelerated through the same potential
current i. The value of magnetic field at a point difference, enter a region of uniform magnetic
R field and describe circular paths of radii R1 and
which is distance inside from the surface is R2, Respectively. The ratio of masses of X and Y
4
10 T. The value of magnetic field at point which is
is 4R distance outside from the surface (a) (R1/R2)–2 (b) (R2/R1)
(c) (R1/R2) 2 (d) (R1/R2)
4 8
(a) T (b) T 31. According to the Bohr's theory of hydrogen
3 3
atom, the speed of the electron, energy and the
40 80 radius of its orbit vary with the principal quantum
(c) T (d) T
3 3 number n, respectively, as
25. The power of a thin convex lens (ang= 1.5) is
5.0 D. When it is placed in a liquid of refractive 1 1 2 1 2 1
(a) , ,n (b) ,n , 2
index an , then it behaves as a concave lens of n n2 n n
focal length 100cm. The refractive index of the
liquid anl will be 1 1 1
(c) n2 , , n2 (d) n, 2 , 2
(a) 5/3 (b) 4/3 n 2
n n
(c) 3 (d) 5/4 32. In the hydrogen atom, the electron is making
26. Find the value of magnetic field between plates 6.6 × 1015 rps. If the radius of orbit is 0.53 × 10–10 m,
of capacitor at a distance 1m from centre, where then magnetic field produced at the centre of
electric field varies by 1010 V/m per second. the orbit is
(a) 5.56×10-8T (b) 5.56×10-3T (a) 140T (b) 12.5T
(c) 5.56 T (d) 5.55T (c) 1.4T (d) 0.14T
EBD_7443
www.jeeneetbooks.in

WWW.IIT-NEET.XYZ

2010-4 Target VITEEE


33. Two identical light sources S1 and S2 emit light 39. An AM wave has 1800 W of total power content.
of same wavelength . These light rays will For 100% modulation the carrier should have
exhibit interference if power content equal to
(a) their phase differences remain constant (a) 1000 W (b) 1200 W
(b) their phases are distributed randomly (c) 1500 W (d) 1600 W
(c) their light intensities remain constant 40. Two light rays having the same wavelength in
(d) their light intensities change randomly vacuum are in phase initially. Then the first ray
34. In Meter bridge or Wheatstone bridge for travels a path l1 through a medium of refractive
measurement of resistance, the known and the index n1 while the second ray travels a path of
unknown resistances are interchanged. The error length l2 through a medium of refractive index
so removed is n2. The two waves are then combined to observe
(a) end correction interference. The phase difference between the
(b) index error two waves is
(c) due to temperature effect 2 2
(d) random error (a) l2 l1 (b) n1l2 n2 l1
35. A fish, looking up through the water, sees the
outside world contained in a circular horizon. If 2 2 l1 l2
the refractive index of water is 4/3 and the fish is (c) n2 l2 n1l1 (d) n1 n2
12cm below the surface of water, the radius of
the circle in centimetre is
PART - II (CHEMISTRY)
12 3
(a) (b) 12×3× 5 41. The correct formula of the complex
5
tetraammineaquachlorocobalt (III) chloride is
12 3 (a) [Cl(H2O) (NH3)4 Co] Cl
(c) (d) 12 3 7
7 (b) [CoCl(H2O) (NH3)4] Cl
36. Radio waves diffract around building althrough (c) [Co (NH3)4(H2O)Cl] Cl
light waves do not. The reason is that radio waves (d) [CoCl (H2O) (NH3)4] Cl2
(a) travel with speed larger than c
42. The equivalent conductance at infinite dilution
(b) have much larger wavelength then light
(c) carry news of a weak acid such as HF
(d) are not electromagnetic waves (a) can be determined by extrapolation of
37. In the Bohr model of a hydrogen atom, the measurements on dilute solutions of HCl,
centripetal force is furnished by the coulomb HBr and HI
attraction between the proton and the electron. (b) can be determined by measurement on very
If a0 is the radius of the ground state orbit, m is dilute HF solutions
the mass and e is charge on the electron and 0 is (c) can best be determined from measurements
the vacuum permittivity, the speed of the electron on dilute solutions of NaF, NaCl and HCl
is
(d) is an undefined quantity
e
Alcoholic Br2 KCN
(a) 0 (b) 43. C2 H5 I X Y Z
0a 0 m KOH CCl4
+
H3O
e 4 0a0m A
(c) (d) The product 'A' is
4 0a0m e
(a) succinic acid (b) melonic acid
38. A potential difference of 2V is applied between
(c) oxalic acid (d) maleic acid
the opposite faces of a Ge crystal plate of area
1 cm2 and thickness 0.5 mm. If the concentration 44. For a reaction of type A + B products, it is
of electrons in Ge is 2×1019/m2 and mobilities of observed that doubling concentration of A
electrons and holes are 0.36 m2V-1s-1 and 0.14 causes the reaction rate to be four times as great,
m2V-1s-1 respectively, then the current flowing but doubling amount of B does not affect the
through the plate will be rate. The unit of rate constant is
(a) 0.25 A (b) 0.45 A (a) s – 1 (b) s–1 mol L–1
–1
(c) s mol L –1 (d) s s–1 mol–2 L2
(c) 0.56 A (d) 0.64 A
www.jeeneetbooks.in

WWW.IIT-NEET.XYZ

Solved Paper 2010 2010-5

45. A chemical reaction was carried out at 320 K and (c) spherically symmetrical
300 K. The rate constants were found to be k1 (d) arranged in planes
and k2 respectively. Then 56. [Co (NH3)4Cl2]NO2 and [Co (NH3)4ClNO2]Cl
(a) k2 = 4k1 (b) k2 = 2k1 exhibit which type of isomerism?
(c) k2 = 0.25 k1 (d) k2 = 0.5 k1 (a) Geometrical (b) Optical
46. The formula of ethyl carbinol is (c) Linkage (d) Ionisation
(a) CH3OH (b) CH3CH2OH 57. Which of the following compounds is not
(c) CH3CH2CH2OH (d) (CH3)3COH
coloured?
47. Which of the following gives red colour in Victor
Meyer's test? (a) Na2[Cu(Cl4] (b) Na[Cd(Cl)4]
(a) n-propyl alcohol (b) Isopropyl alcohol (c) K4[Fe(CN)6] (d) K3[Fe(CN)6]
(c) tert-butyl alcohol (d) sec-butyl alcohol 58. Which of the following is a Gattermann aldehyde
48. Enthalpy of a compound is equal to its synthesis?
(a) heat of combustion(b) heat of formation H 2 / Pd
(c) heat of reaction (d) heat of solution (a) COCl BaSO4
CHO
49. For which one of the following reactions will there
be a positive S?
(b) H + CO + HCl
(a) H 2 O (g) H 2 O (l )
(b) H2 I2 2HI AlCl3
CHO
(c) CaCO 3 (s) CaO(s) CO 2 (g)
(d) N 2 (g) 3H 2 (g) 2NH 3 (g)
(c) + HCl + HCN
50. Across the lanthanide series, the basicity of the
lanthanide hydroxides
(a) increases (i) Anhy. AlCl3
CHO
(b) decreases (ii) H 3O
(c) first increases and then decreases
(d) first decreases and then increases CH3 CrO 2Cl 2
CHO
51. When p-nitrobromobenzene reacts with sodium (d)
ethoxide, the product obtained is 59. Aldol is
(a) p-nitroanisole (b) ethyl phenyl ether (a) -hydroxybutyraldehyde
(c) p-nitrophenetole (d) no reaction occurs (b) -hydroxybutanal
52. A radioactive element X emits 3 , 1 and 1 - (c) -hydroxypropanal
particles and forms 76Y235. Element X is (d) None of the above
(a) 81 X 247 (b) 80 X 247 60. Nitrobenzene can be converted into azobenzene
(c) 81 X 246 (d) 80 X 246 by reduction with
53. For the reaction, (a) Zn, NH4Cl,
2A(g) B2 (g) 2AB 2 (g) (b) Zn/NaOH, CH3OH
the equilibrium constant, Kp at 300 K is 16.0. The (c) Zn/NaOH
(d) LiAlH4, ether
value of Kp for AB2 (g) A(g) + 1/2 B2 (g) 61. The one which is least basic is
is (a) NH3 (b) C6H5NH2
(a) 8 (b) 0.25 (c) (C6H5)3N (d) (C6H5)2NH
(c) 0.125 (d) 32 62. Coordination number of Ni in [Ni(C2O4)3]4– is
54. Frenkel defect is generally observed in (a) 3 (b) 6
(a) AgBr (b) AgI (c) 4 (d) 5
(c) ZnS (d) All of the above 63. Mg is an important component of which
55. Most crystals show good cleavage because their biomolecule occurring extensively in living
atoms, ions or molecules are world?
(a) weakly bonded together (a) Haemoglobin (b) Chlorophyll
(b) strongly bonded together (c) Florigen (d) ATP
EBD_7443
www.jeeneetbooks.in

WWW.IIT-NEET.XYZ

2010-6 Target VITEEE


64. Sterling silver is 73. The yield of the product in the reaction
(a) AgNO3 A 2 (g) 2B (g) C (g) Q kJ
(b) Ag2 S would be higher at
(c) Alloy of 80% Ag + 20% Cu (a) high temperature and high pressure
(d) AgCl (b) high temperature and low pressure
65. Identify the statement which is not correct (c) low temperature and high pressure
regarding CuSO4 . (d) low temperature and low pressure
(a) It reacts with KI to give iodine 74. In which of the following case, does the reaction
(b) It reacts with KCl to give Cu2Cl2 go farthest to completion?
(c) It reacts with NaOH and glucose to give (a) K = 102 (b) K = 10
Cu2O (c) K = 10–2 (d) K = 1
(d) It gives CuO on strong heating in air 75. Formation of cyanohydrin from a ketone is an
66. Transition metals usually exhibit highest oxidation example of
states in their (a) electrophilic addition
(a) chlorides (b) fluorides (b) nucleophilic addition
(c) bromides (d) iodides (c) nucleophilic substitution
67. The number of Faradays needed to reduce 4 g (d) electrophilic substitution
equivalents of Cu2+ to Cu metal will be 76. Glycerol on treatment with oxalic acid at 110ºC
(a) 1 (b) 2 forms
1 (a) formic acid (b) allyl alcohol
(c) (d) 4 (c) CO2 and CO (d) acrolein
2
77. The activity of an old piece of wood is just 25%
68. Which one of the following cells can convert
of the fresh piece of wood. If t 1/2 of C-14 is 6000
chemical energy of H2 and O2 directly into
yr, the age of piece of wood is
electrical energy?
(a) 6000 yr (b) 3000 yr
(a) Mercury cell (b) Daniel cell
(c) 9000 yr (d) 12000 yr
(c) Fuel cell (d) Lead storage cell
69. On treatment of propanone with dilute Ba(OH)2, 78. The radius of Na+ is 95 pm and that of Cl– ion is
the product formed is 181 pm. Hence, the coordination number of Na +
(a) aldol will be
(b) phorone (a) 4 (b) 6
(c) propionaldehyde (c) 8 (d) unpredictable
(d) 4-hydroxy-4-methyl-2-pentanone 79. The reaction, ROH + H2CN2 in the presence of
70. Which of the following converts CH3CONH2 to HBF4, gives the following product
CH3NH2? (a) ROCH3 (b) RCH2OH
(a) NaBr (b) NaOBr (c) ROHCN2N2 (b) RCH2CH3
(c) Br2 (d) None of the above 80. The fatty acid which shows reducing property is
71. Which metal aprons are worn by radiographer to (a) acetic acid (b) ethanoic acid
protect him from radiation? (c) oxalic acid (d) formic acid
(a) Mercury coated apron
(b) Lead apron
PART - III (MATHEMATICS)
(c) Copper apron 81. If F is function such that F (0) = 2, F (1) = 3,
(d) Aluminimised apron F (x+2)= 2F (x) - F (x+1) for x 0, then F (5) is
72. The standard Gibb's free energy change, Gº is equal to
related to equilibrium constant, Kp as (a) –7 (b) –3
Gº (c) 17 (d) 13
e 82. Let S be a set containing n elements. Then,
(a) Kp RT ln Gº (b) K p
RT number of binary operations on S is
2
G (a) nn (b) 2n
Kp Gº/ RT
(c) (d) K p e 2
RT (c) nn (d) n2
www.jeeneetbooks.in

WWW.IIT-NEET.XYZ

Solved Paper 2010 2010-7

83. The numerically greatest term in the expansion 92. The chances of defective screws in three boxes
1 1 1 1
of (3–5x)11 when x = , is A, B, C are , , respectively. A box is selected
5 5 6 7
(a) 55 × 39 (b) 55 × 36 at random and a screw drawn from it at random is
(c) 45 × 3 9 (d) 45 × 36 found to be defective. Then, the probability that
84. The number of solutions of the equation it came from box A, is
sin (ex) = 5x + 5–x, is
16 1
(a) 0 (b) 1 (a) (b)
(c) 2 (d) infinitely many 29 15
85. If ax = by = cz = du and a, b, c, d are in GP, then x, 27 42
y, z, u are in (c) (d)
(a) AP (b) G P 59 107
(c) HP (d) None of these cos
86. If z satisfies the equation |z|–z = 1+2i, then z is 93. The value of is equal to
1 sin
equal to
3 3 (a) tan (b) tan
(a) + 2i (b) –2i 2 4 4 2
2 2
3 3 (c) tan (d) tan
(c) 2– i (d) 2 + i 4 2 4 2
2 2
94. If 3sin 5cos 5 , then the value of
1 i 3
87. If z = , then arg (z) is 5 sin – 3 cos is equal to
1 i 3
(a) 5 (b) 3
(a) 60 (b) 120 (c) 4 (d) None of these
(c) 240 (d) 300
5
95. The principal value of sin 1 sin is
88. If f (x) = log10 x 2 . The set of all values of x for 6
which f (x) is real, is
5
(a) [–1, 1] (b) [1, ] (a) (b)
(c) (– , –1] (d) (– , –1] [1, ) 6 6
89. For what values of m can the expression 7
2x2 + mxy + 3y2 – 5y – 2 (c) (d) None of these
6
be expressed as the product of two linear factors?
96. A rod of length l slides with its ends on two
(a) 0 (b) 1 perpendicular lines. Then, the locus of its mid
(c) 7 (d) 49 point is
90. If B is a non-singular matrix and A is a square
matrix, then det (B–1AB) is equal to l2 l2
(a) det (A–1) (b) det (B–1) (a) x2 y2 (b) x 2 y2
4 2
(c) det (A) (d) det (B)
91. If f (x), g(x) and h (x) are three polynomials of l2
degree 2 and (c) x2 y2 (d) None of these
4
f (x) g(x) h(x) 97. The equation of straight line through the
intersection of line 2x + y=1 and 3x + 2y=5 and
(x) f' (x) g' (x) h' (x)
passing through the origin is
f'' (x) g'' (x) h'' (x) (a) 7x + 3y = 0 (b) 7x – y = 0
then (x) is a polynomial of degree (c) 3x + 2y = 0 (d) x + y = 0
(a) 2 (b) 3 98. The line joining (5,0) to (10 cos , 10 sin ) is
(c) 0 (d) atmost 3 divided internally in the ratio 2:3 at P. If varies,
then the locus of P is
EBD_7443
www.jeeneetbooks.in

WWW.IIT-NEET.XYZ

2010-8 Target VITEEE


(a) a straight line 105. The equation of the common tangents to the two
(b) a pair of straight lines
x2 y2 y2 x2
(c) a circle hyperbolas 1 and 1 , are
(d) None of the above a2 b2 a2 b2
99. If 2x + y + k = 0 is a normal to the parabola
(a) y= x b2 a2
y2 = – 8x, then the value of k, is
(a) 8 (b) 16 (b) y= x a2 b2
(c) 24 (d) 32
1 1 1 1 (c) y= x a2 b2
100. nlim ..... is equal
1.2 2.3 3.4 n( n 1) (d) y= x a 2 b2
to 106. Domain of the function f (x) log x cos x, is
(a) 1 (b) –1
(c) 0 (d) None of these , {1} , {1}
(a) (b)
101. The condition that the line lx + my =1 may be 2 2 2 2
normal to the curve y2 = 4ax, is
(c) , (d) None of these
(a) al 3 2alm2 m 2 (b) al 2 2alm3 m2 2 2
(c) al 3 2alm 2 m3 (d) al 3 2alm2 m2 x2
2
107. Range of the function y = sin 1 , is
102. If f ( x)dx f ( x), then f ( x) dx is equal to 1 x2

1 2 3 0, 0,
(a) f (x) (b) f (x) (a) (b)
2 2 2
3
f (x) 2 (c) 0, (d) 0,
(c) (d) f (x) 2 2
3
108. If x = sec cos , y sec n cos n , then
1 (2 x 2)
103. sin dx is equal to dy
2
4 x 2 8 x 13 (x 2 4) is equal to
dx
2x 2 3 4 x 2 8 x 13
(a) (x+1) tan 1
log c (a) n2 ( y 2 4) (b) n2 (4 y 2 )
3 4 9
(c) n2 ( y 2 4) (d) None of these
3 1 2x 2 3 4 x 2 8 x 13
(b) tan log c dy
2 3 4 9
109. If y= x y x y , then is
dx
1 2x 2 3
(c) ( x 1) tan log 4 x 2 8 x 13 c equal to
3 2
y x y3 x
3 1 2x 2 3 (a) (b)
(d) x 1 tan log 4 x 2 8 x 13 c y 2
2x 2y 2
2 xy 1
2 3 4
104. If the equation of an elipse is y3 x
2 2 (c) 2 (d) None of these
3x 2 y 6 x 8 y 5 0 , then which of the 2y x
following are true ? x
dt
1 110. If = , then x can be equal to
(a) e 1 t t2 1 6
3
(b) centre is (–1, 2) 2
(c) foci are (–1, 1) and (–1, 3) (a) (b) 3
3
(d) All of the above (c) 2 (d) None of these
www.jeeneetbooks.in

WWW.IIT-NEET.XYZ

Solved Paper 2010 2010-9

111. The area bounded by the curve y= sin x , x-axis 3 9


(a) (b)
and the lines x = , is 2 2
(a) 2 sq unit (b) 1 sq unit 2 3
(c) 4 sq unit (d) None of these (c) (d)
9 2
112. The degree of the differential equation of all
curves having normal of constant length c is 117. The two curves y = 3 and y = 5x intersect at an
x

(a) 1 (b) 3 angle


(c) 4 (d) None of these 1 log 3 log 5
(a) tan
113. If a 2i 2j 3k, b i 2jˆ kˆ and c 3iˆ ˆj ,
ˆ ˆ ˆ ˆ 1 log 3log 5

then a tb is perpendicular to c , if t is equal to 1 log 3 log 5


(b) tan
(a) 2 (b) 4 1 log 3log 5
(c) 6 (d) 8
114. The distance between the line 1 log 3 log 5
(c) tan
1 log 3log 5
r 2iˆ 2ˆj 3kˆ iˆ ˆj 4kˆ and the plane
1 log 3 log 5
r ˆi 5jˆ kˆ 5 , is (d) tan
1 log 3log 5
10 10
(a)
3
(b) 118. The equation x2 4xy y 2 x 3y 2 0
3
represents a parabola, if is
10 10 (a) 0 (b) 1
(c) (d)
3 3 9 (c) 2 (d) 4
115. The equation of sphere concentric with the 119. If two circles 2 x 2 2 y2 3x 6 y k 0 and
sphere x 2 y2 z 2 4x 6y 8z 5 0 and
x 2 y 2 4x 10y 16 0 cut orthogonally,,
which passes through the origin, is
then the value of k is
(a) x2 y2 z 2 4x 6y 8z 0 (a) 41 (b) 14
(c) 4 (d) 1
(b) x2 y2 z 2 6y 8z 0 120. If A (–2, 1), B (2, 3) and C (–2, –4) are three
points. Then, the angle between BA and BC is
(c) x2 y2 z2 0
1 2 1 3
(d) x2 y2 z2 4x 6y 8z 6 0 (a) tan
3
(b) tan
2
x 1 y 1 z 1 1 1
116. If the lines and 1 1
2 3 4 (c) tan (d) tan
3 2
x 3 y k z
intersect, then the value of k,
1 2 1
is
EBD_7443
www.jeeneetbooks.in

WWW.IIT-NEET.XYZ

2010-10 Target VITEEE

SOLUTIONS
PART - I (PHYSICS) 4. (d) Magnetic field,

1 02 i 0i 2Br
1. (b) Here, length l = 2 r or r B or i
2 4 r 2r 0
2
Area of circular loop A r 1/ 2
A
Magnetic moment M = iA = i r2 Also, A = r2 or r
l2 Magnetic moment,
M i 2
4
2 Br
M iA A
4 M
l 0
i
1/ 2
2. (b) Current through arms of resistances P and 2 BA A 2 BA3/ 2
Q in series 1/ 2
0 0
i 330 3
i1 i Y
330 110 4 Y
5. (c) Here, sin
Here i = total current D D
Similarly, current through arms of Angular fringe width 0=
resistances R and S in series (width Y = )
i 110 1 D 1
i2 i
330 110 4 0
D d D d
Heat developed per second = i2R
Ratio of heat developed per sec 1
0 rad and = 6 × 10–7 m
H P : HQ : H R : H S 180

2 2 180 7
3 3 d 6 10 0.03 mm
i 100 : i 10 : 0
4 4
6. (d) Here, charge q = ± 1 × 10–6 C
1
2
1
2 2a = 2.0 cm = 2.0 × 10–2 m
i 300 : i 30 E = 1 × 105 NC–1, max = ?
4 4
W = ?, 1 = 0°, 2 = 180°
= 30 : 3 : 10 : 1
3. (c) Heat taken by water when its temperature max = pE = q(2a)E
= 1 × 10–6 × 2.0 × 10–2 × 1 × 105
changes from 20°C to 100°C.
= 2 × 10–3 Nm
H1 mc( 2 1 ) 1000 1 (100 20) cal
W pE (cos 1 cos 2 )
= 1000 × 80 × 4.2 J
Heat produced in time t due to current in = (10–6 × 2 × 10–2)(105) (cos 0° – cos 180°)
resistor = 4 × 10–3 J
H2 = Vit = 220 × 4 × t J e e ev
According to question, 7. (b) Current, i
t 2 r /v 2 r
220 × 4 × t = 1000 × 80 × 4.2
2
e2
t
1000 80 4.2
381.8 s = 6.3 min
Here, v and r
220 4 me 2
www.jeeneetbooks.in

WWW.IIT-NEET.XYZ

Solved Paper 2010 2010-11

11. (d) In a neutral water molecule, there are 10


e (e 2 / ) e3 me 2 me5
i electrons and 10 protons.
2
2 ( / me2 ) 2 3
2 3
So, its dipole moment p = q (2l) = 10 e (2l)
Hence length of the dipole = distance
h
(given) between centres of positive and negative
2 charges
me5 4 2 me5 p 6.4 10 30
i 3 3 2l 4 10 12 m
h h 10e 10 1.6 10 19
2
2 12. (c) Magnetic field due to finite length of a wire,
8. (d) For dark fringe, 0 i
B (sin 1 sin 2)
xd 4 r
(2m 1)
D 2
l
d P
Here, m = 5, x 45°
2 i
S1 P l
d
x=
2
d
2 Here, 1 = 0°, = 45°

S2 D 0 i 0 i
B (sin 0 sin 45 )
4 r 4 2l
d d
(2 5 1)
2 D 2 2 0i
B
8 l
d2
or 9 13. (c) Zener diode is suitable for voltage
D regulating purpose. It is used as voltage
d2 stabilizer in many applications in
Wavelength, electronics.
9D
9. (d) Generally, temperature of human body is 14. (a) If two independent sources emitting light
37°C (= 98.4°F) corresponding to which IR of the same wavelength are said to be
and microwave radiations are emitted from coherent.
the human body. 15. (d) Induced charge
10. (d) The path of moving proton in a normal nBA
magnetic field is circular. If r is the radius of Q (cos 2 cos 1 )
R
the circular path, then from the figure,
From the symmetry of figure, the angle nBA
= 45°. (cos180 cos 0 )
R
1
AC 2r cos 45 2r 2r ...(1) QR
2 B
2nA
mv 2 mv 16. (d) From an optical fibre due to absorption or
As Bqv or r light leaving the fibre area resulting
r Bq
scattering of light sideways by impurities
2mv 2 1.67 10 27
107 in the glass fibre. And due to this reason a
AC 19 very small part of light energy is lost.
Bq 1 1.6 10
= 0.14 m
EBD_7443
www.jeeneetbooks.in

WWW.IIT-NEET.XYZ

2010-12 Target VITEEE

r hc hc
17. (c) Length of the arc = r 21. (c) Here, E W0 and 2E W0
3 2 '

r E W 1 W0 / E
Charge on the arc = 2 E W0 2 W0 / E
3
Potential at centre v (1 W0 / E ) 1
Since (2 So
kq 1 r W0 / E ) 2 2
r 4 0 3 r 12 0 22. (b) According to Einstein's photoelectric
equation,
l, 1 2 2(hf W0 )
E W0 mvmax vmax
2 m
If frequency becomes 4f then

r r W0
2 hf
3 2(h 4 f W0 ) 4
v' 2
m m
v ' 2v
18. (a) Herefc = 1.5 MHz = 1500 kHz, fm = 10 kHz 23. (b) In electric field photoelectron will
Lower side-band frequency experience force and accelerate opposite
= fc – fm = 1500 kHz – 10 kHz = 1490 kHz to the field so its KE increases (i.e., stopping
Upper side-band frequency potential will increase), no change in
= fc + fm = 1500 kHz + 10 kHz = 1510 kHz photoelectric current, and threshold
19. (c) Equivalent resistance of the circuit wavelength.
Req = 100 24. (b) Magnetic field inside the cyclindrical

V 2.4 0 2ir
Current through the circuit, i A conductor Bin
R 100 4 R2
(R = radius of cylinder and r = distance of
Potential difference across combination of observation point from axis of cylinder)
voltmeter and 100 resistance Magnetic field outside the cylinder at a
2.4 0 2i
50 1.2 V distance r' from its axis, Bout
100 4 r'
Since the voltmeter and 100 resistance
R
are in parallel, the voltmeter reads the same R ( R 4 R)
Bin rr ' 10 4
value i.e., 1.2 V.
20. (c) In space charge limited region, the plate Bout R2 Bout R2

current is given by Child's law i p KV p3/ 2 Bout


8
T
3
Thus,
25. (a) By using lens maker's formula,
3/ 2 3/ 2
ip2 Vp 2 600 1 1 1
(4)3/ 2 8 ( 1)
i p1 V p1 150 f R1 R2

or, i p 2 i p1 8 10 8 mA 80 mA 2
5 (1.5 1) ...(i)
R
www.jeeneetbooks.in

WWW.IIT-NEET.XYZ

Solved Paper 2010 2010-13

If a lens of refractive index g is immersed Since P '1 P '2 , so, 25 W bulb will glow
in a liquid of refractive index 1, then its
focal length in liquid more brightly.
29. (a) Given : = 100 nm = 1000 Å
1 1 1 Energy corresponding to 1000 Å
( g 1)
fl R1 R2 12375
12.375 eV
1.5 2 1000
1 1 ...(ii) Now, 7.7 = 12.375 – 0
n R
or 0 = 12.375 – 7.7 = 4.675 eV
0.5n In the second case,
Dividing, (i) by (ii) 5 Energy corresponding to 2000 Å
1.5 n
7.5 5n 0.5n 7.5 4.5n 12375
eV 6.1875 eV
75 5 2000
n
45 3 Now, 4.7 = 6.1875 – '0
26. (a) Magnetic field or '0 = 6.1875 – 4.7 = 1.4875 1.5 V
dE0 0r 1 1 2 2qV
B 1010 30. (c) As we know, mv qV or v
2 dt 9 1016 2 2 m
= 5.56 × 10–8 T
27. (b) E E0 sin t mv 2
Centripetal force q B v
Voltmeter read rms value R

E0 2 234 V 331 V qBR


v
and t 2 nt 2 50 t 100 t m
Thus, the equation of the line voltage 1/ 2
2 qV qBR 2mV 1
E = 331 sin (100 t) Hence, or R
m m q B
V2 V2 Here, V, q and B are constants.
28. (a) Power, P , R
R P R m
For the first bulb,
2
V2 (220)2 m1 R1
R1 1936 And,
P1 25 m2 R2
For the second bulb, 31. (a) According to Bohr's theory of hydrogen
atom,
V2 (220)2 (i) The speed of the electron in the nth orbit
R2 484
P2 100
1 e2 1
Current in series combination is the same Vn or vn
n4 0 (h / 2 ) n
in the two bulbs,
(ii) The energy of the electron in the nth orbit
V 220 220 1
i A
R1 R2 1936 484 2420 11 me4 13.6 1
En eV or En
If the actual powers in the two bulbs be P1 8n2 20 h2 n 2
n2
and P2 then (iii) The radius of the electron in the nth orbit
2
1 n2 h2
P '1 i 2 R1 1936 16 W rn 0
n 2 a0 or rn n2
11 me 2

2
1 h2 0
and P '2 i 2 R2 484 4W where a0 5.29 10 11
m
11 me
EBD_7443
www.jeeneetbooks.in

WWW.IIT-NEET.XYZ

2010-14 Target VITEEE


32. (b) Current, i = qv 3
l l 0.5 10 25
R
0i 0 qv A A 4
8
B 1.6 10
2r 2r
V 2
4 10 7
1.6 10 19
6.6 1015 i 0.64A
R 25 / 8
10
2 0.53 10
ma 2
21.6 6.6 39. (b) Total power Pt Pc 1
2 ma 2 1
12.513 T
5.3
33. (a) For interference phase difference must be 1
constant. 1800 Pc 1 Pc 1200 W
2
34. (a) To remove the error, resistance box and the
40. (b) Optical path for ray 1 = n1l1
unknown resistance must be interchanged
Optical path for ray 2 = n2l2
and then the mean reading must be taken.
Phase difference,
r
35. (c) Here, tan ic or r h tan ic 2
x
2
( n1l1 n2 l2 )
h
sin ic sin ic PART - II (CHEMISTRY)
or r h or r h
cos ic 1 sin ic 2
41. (d) The correct formula of the given complex is
tetraammine aqua chlorocobalt (III) chloride
1 is [CoCl(H2O)(NH3)4]Cl2, because in it the
But sin ic
oxidation number of Co is +3. While in rest
other options O. No. of Co is +2
1 [CoCl(H2O)(NH3)4]Cl2
h 12 x + (–1) + 0 + (0 × 4) + (–1) 2 = 0
r h x – 3 = 0 x = +3
1 2
1 16
1 2
1 42. (c) According to Kohlrausch's law, equivalent
9
conductance at infinite dilution of HF,
HF = NaF + HCl – NaCl
12 3
cm 43. (a) C2 H5 I
Alc. KOH
C2 H 4
Br2
7 (dehydrohalogenation) ethylene 'X' CCl 4
36. (b) Diffraction takes places when the
wavelength of waves is comparable with Br CN
the size of the obstacle in path. | |
KCN
Pradio > Plight CH 2 CH 2 CH 2 — CH 2
| |
Hence, radio waves are diffracted around Br CN
building. 1, 2-dibromoethane 'Y' 'Z'
37. (c) Centripetal force = force of attraction of
nucleus on electron. COOH
|
H 3O
mv 2 1 e2 e CH 2 — CH 2
v |
a0 4 0 a02 4 0 ma0 COOH
succinic acid 'A'
38. (d) As we know, conductivity
44. (c) Let the initial rate be R
ne( e h) and order with respect to A be x and B be y.
(0.36 + 0.14) Thus, rate law can be written as,
2 1019 1.6 10 19
rate, R = [A]x [B]y ...(i)
= 1.6 ( m)–1
After doubling the concentration of A, rate
becomes 4R,
www.jeeneetbooks.in

WWW.IIT-NEET.XYZ

Solved Paper 2010 2010-15

4R = [2A]x [B]y ...(ii) 48. (b) Heat of formation is equal to enthalpy of a


After doubling the concentration of B, compound.
rate remains R, 49. (c) S (entropy change) is the measure of
R = [A]x [2B]y ...(iii) randomness and thus in solid, liquid and
From Eq. (i) and (ii), we get gas, the order of entropy is
x 2 x gas > liquid > solid
R 1 1 1 Thus, S is positive for the reaction given
4R 2 2 2 in option (c) because solid CaCO3 is
So, x = 2 forming gaseous CO2.
From Eq. (i) and (iii), we get 50. (b) We know that basicity is depend on ionic
y o y
character. So, as the ionic size of lanthanide
R 1 1 1 decreases, the covalent character of their
R 2 1 2 hydroxide increases. Hence, their basicity
So, Y = 0 decreases.
Hence, the rate law is, rate R = [A]2 [B]0 51. (c) Commonly aryl halides do not take part in
This clearly shows that the order of this Williamson's synthesis, due to their high
reaction is 2 and for second order reaction stability but due to the presence of strong
units of rate constant are mol–1 Ls–1. electron withdrawing group like –NO2
45. (c) As we know that for every 10° rise in makes the C–X bond weaker and
temperature, rate constant, k becomes substitution of –Br takes place by –OR.
doubled. Hence, on rising the temperature Br OC2H5
20°, the rate constant will be four times,
1 + C2H5ONa
i.e., k1 4k 2 k2
k1 0.25 k1
4
46. (c) The other name of methanol is carbinol. So,
the formula of ethyl carbinol is NO2 NO2
p-nitrophenetole
CH3CH2 CH2OH
52. (a) The complete nuclear reaction is
ethyl carbinol
47. (a) In Victor Meyer's test, Red colour is given z XA 76 Y
235
32 He 4 1e
0

by primary alcohols (1°) (alcohols having – particle particle


CH2 OH). The structres of the given
Alcohols are particle
(a) CH3 CH 2 CH 2 OH On observing the reaction, mass number of
n propyl alcohol (1°) 'X' is
A = 235 + 12 + 0 = 247
(b) CH3 C H — CH3
| On observing the reaction, atomic number
OH of 'X' is
iso-propyl alcohol (2°)
Z = 76 + 6 – 1 = 81
CH3 Hence, element 'X' is 81X247.
| 53. (b) For the given reaction,
(c) CH3 C CH3 2 A( g ) B2 ( g ) 2 AB2 ( g )
|
OH the equilibrium constant,
tert -butyl alcohol (3°) 2
p AB2
Kp 16 ...(i)
(d) CH 3 C HCH 2 CH3 p 2A p B2
|
OH For the other given reaction,
sec-butyl alcohol (2°)

Hence, n-propyl alcohol is a 1° alcohol and 1


AB2 ( g ) A( g ) B2 ( g )
gives red colour in Victor-Meyer's test. 2
EBD_7443
www.jeeneetbooks.in

WWW.IIT-NEET.XYZ

2010-16 Target VITEEE


The equilibrium constant, 59. (a) Aldol is -hydroxybutyraldehyde
(or 3-hydroxybutanal) i.e.
p A p1/
B2
2
K 'p ...(ii) OH
p AB2
|
On squaring Eq. (ii), we obtain, CH 3 CH CH 2 CHO

p A2 p B2 60. (b) Nitrobenzene can be converted into


2
(K ' p ) ...(iii) azobenzene, on reduction in the presence
p 2AB2
of Zn/NaOH in CH3OH.
Now, from Eq. (i) and (iii), we obtain,
NO2
K p .( K ' p ) 2 1 16.( K ' p ) 2 1 Zn/NaOH
in CH 3OH
( Kp = 16.0) –N = N–
8[H]
1/2 azobenzene
1 1
(K 'p ) K 'p 0.25
16 4 61. (c) Due to the presence of electron
54. (d) When the size of cation is much smaller withdrawing group like Ph group decreases
than the anion then frenkel defect is the electron density of nitrogen and hence,
observed. Hence, AgBr, AgI and ZnS all the lone pair of nitrogen are not available
exhibit Frenkel defect. for donation.
55. (d) Crystals show good cleavage when, the So, (C6 H5 )3 N is least basic due to the
constitutents are arranged in orderly presence of three electron withdrawing
pattern, i.e., in planes,
Ph(C6H5) groups.
56. (d) On ionisation they give different ions
62. (b) Coordination number of Ni in [Ni(C2O4)3]4–
[Co(NH 3 ) 4 Cl 2 ]NO2
is 6 because C2 O42 (oxalate) is a bidentate
[Co(NH 3 ) 4 Cl2 ] NO 2 ligand and each have two sites to
[Co(NH3 ) 4 ClNO 2 ]Cl coordinated with the central atom .
63. (b) Chlorophyll is rich source of Mg, the green
[Co(NH3 ) 4 ClNO 2 ] Cl pigment of plants.
So, they show ionisation isomerism. 64. (c) An alloy of 80% Ag and 20% other metals,
57. (c) We know that complex compound having usually copper is sterling silver.
no unpaired electron is colourless. 65. (b) As CuSO4 reacts with KI to give white
Among the given complexes, K4[Fe(CN)6] precipitate of Cu2I2 and to evolve I2.CuSO4
has no unpaired electron as CN– is a strong
does not react with KCl.
field ligand and causes pairing of electrons.
So, it is colourless. 66. (b) Due to highest reduction potential of
fluorine. Transition metals exhibit highest
CH=NH oxidation states in their fluoride.

Anhy. AlCl3
67. (d) Cu 2 2e Cu
58. (c) + HCN + HCl 1 mol 2F 1 mol
1 1
mol 1F mol
2 2
CHO 1 gequi. 1F 1g equi.

H3 O
+
Thus, to reduce 4 g equivalent of Cu2+ into
–NH3
Cu 4F are required.
(Benzaldehyde)
68. (c) Fuel cell, which convert chemical energy
of fuels like H2, O2, CH4, etc, is converted
This is Gattermann aldehyde synthesis. into electric energy, e.g., H2–O2 fuel cell.
www.jeeneetbooks.in

WWW.IIT-NEET.XYZ

Solved Paper 2010 2010-17

69. (d) In Aldol condensation propanone gives


CH2 – OOC
diacetone alcohol in presence of Ba(OH)2
110°C COOH
O CHOH
Ba(OH) 2 –H2O
CH3–C–CH3 + HCH2COCH3 CH2OH

OH
OH H
CH2 OOCH
H3C – C – CH2COCH3
CHOH
CH3 –CO 2
4-hydroxy-4-methyl pentanone-2
(diacetone alcohol)
CH2OH

CH2OH
70. (b) CH3CONH 2 NaOBr CH3 NH 2 Na 2 CO3
71. (b) Radiographer to protect themself from
CHOH + HCOOH
radiation worn lead apron. formic acid
72. (d) G° and Kp are related as
CH2OH
G RT ln K p
77. (d) As we know that,
G
ln K p K p e G / RT 0.693 0.693 4
RT k 1.155 10
73. (c) A2 ( g ) 2 B ( g ) C ( g ) Q kJ t1/ 2 6000
Since, the reaction is exothermic, So, it is 2.303 N 2.303 100
favoured by low temperature. t log 0 4
log
k N 1.155 10 25
In addition, the number of moles of
products is lesser than the number of moles = 12000 yr (age of piece of wood)
of reactants, thus high pressure favours Radius of cation, r 95
the forward reaction. 78. (b) 0.525
74. (a) Larger the value of K more the reaction Radius of anion, r 181
moves towards completion. As this value lies in between 0.414 – 0.732,
75. (b) As CN is a nucleophile. So it is an example thus, the coordination number of Na + ion
of nucleophilic addition will be 6.
HBF
CN 79. (a) ROH H 2 CN 2 4
ROCH3 N 2
– –
80. (d) The compounds having –CHO group
R2C = O + CN R2 – C – O reduces Tollen's reagent, Fehling solution
nucleophile etc. Thus, formic acid (HCO/ OH) has
CN reducing property.
+
H
R2 – C – OH PART - III (MATHEMATICS)
81. (d) F ( x 2) 2 F ( x) F ( x 1) ...(i)
CH2OH
HOOC Putting x = 0, we get
76. (a) CHOH + F (2) 2 F (0) F (1)
COOH F (2) 2(2) 3
CH2OH
{ F (0) 2, F (1) 3}
EBD_7443
www.jeeneetbooks.in

WWW.IIT-NEET.XYZ

2010-18 Target VITEEE

F (2) 4 3 F (2) 1 11 10 9 1
311 . 55 39
Putting x = 1, in eq. (i), we get 1.2.3 27
F (3) 2 F (1) F (2) Greatest term (numerically)
2(3) 1 { F (1) 3, F (2) 1} = T3 = T4 = 55 × 39

F (3) 5 84. (a) We have, sin(e x ) 5x 5 x ...(i)


Putting x = 2, in eq. (i), we get Let 5x = t, then eq. (i), reduces to
F (4) 2F (2) F (3) 1
sin(e x ) t
2(1) 5 { F (2) 1, F (3) 5} t

F (4) 3 1
sin(e x ) t 2 2
Putting x = 3, in eq. (i), we get t
F (5) 2F (3) F (4) 1
2
x
sin(e ) t 2
2(5) 3 { F (3) 5, F (4) 3} t
F (5) 13
{ 5x 0, 5x t exists}
82. (c) The number of binary operations on a set S
having n elements in n n .
2 sin(e x ) 2

11 which is not possible as also sin 1.


11 11 5x Thus, given equation has no solution.
83. (a) (3 5x) 3 1
3 85. (c) ax by cz du
11
5 1 1 Let, a x by cz du k
311 1 . x
3 5 5
a k1/ x , b k1/ y , c k1/ z , d k1/ u
11 ...(i)
1
311 1 a, b, c, d are in GP.
3
b c d
1 a b c
(11 1)
| x | (n 1) 3 4
Now, r k 1/ y k 1/ z k 1/ u
|x| 1 1 4 {using Eq. (i)}
1 k 1/ x k 1/ y k 1/ z
3 3
1 1 1 1 1 1
r 3 ky x
kz y
ku z
Therefore, 3rd (T3) and (3 + 1) = 4th (T4)
terms are numerically greatest in the 1 1 1 1 1 1
expansion of (3 – 5x)11. y x z y u z
So, greatest term = T3
1 1 1 1
1
2
11 10 , , , are in A.P..
3 11 11
C2 (1) 9
3 11 x y z u
3 1.2.9
x, y, z, u are in H.P.
= 55 × 39 86. (b) Given : | z | – z = 1 + 2i
If z = x + iy, then this equation reduces to
3
1 | x iy | ( x iy ) 1 2i
and T4 311 11
C3 (1)8
3
( x2 y2 x) ( iy) 1 2i
www.jeeneetbooks.in

WWW.IIT-NEET.XYZ

Solved Paper 2010 2010-19

On comparing real and imaginary parts of m 5


both sides of this equation, we get a 2, h ,b 3, c 2, g 0, f
2 2
x2 y2 x 1 The given expression is resolvable into
linear factors, if
x2 y2 1 x x2 y2 (1 x )2 abc 2 fgh af 2 bg 2 ch 2 0
2 2 2
x y 1 x 2x
25 m2
(2)(3)( 2) 2(0) 2 0 ( 2) 0
2 4 4
y1 2x ...(i)
and –y = 2
25 m2
y 2 12 0
2 2
Putting this value in eq. (i), we get
( 2)2 m2 49
1 2x m2 49 m 7
2 2
3
2x 3 x 90. (c) det (B–1AB) det( B 1 ) det A det B
2
3 det( B 1 ) det B det A
z x iy 2i
2
det( B 1B) det A det I .det A
1 i 3 = 1 . det A = det A
87. (c) z
1 i 3 91. (c) Since f(x), g(x) and h(x) are the polynomials
of degree 2,
1 i 3 1 i 3 therefore f '''( x ) g '''( x ) h '''( x ) 0
z
1 i 3 1– i 3
f ( x) g ( x) h( x )
1 3 2 3i 2 2 3i Now, ( x) f '( x) g '( x) h '( x)
z
1 3 4 f ''( x ) g ''( x) h ''( x)
1 3
z i f '( x) g '( x) h '( x )
2 2
z cos 240 i sin 240 '( x) f '( x) g '( x) h '( x )
Thus, arg (z) = 240° f ''( x) g ''( x) h ''( x )

88. (d) f ( x) log10 x 2 is real, if


f ( x) g ( x) h( x)
2
log10 x 0 f ''( x) g ''( x) h ''( x)
f ''( x) g ''( x) h ''( x)
x2 1
x 1 and x 1
f ( x) g ( x) h ( x)
x ( , 1] [1, ) f '( x ) g '( x ) h '( x)
89. (c) The given expression is f '''( x) g '''( x ) h '''( x)
2 x 2 mxy 3 y 2 5 y 2
'( x ) 0 0 0 0
Comparing the given expression with
ax2 2hxy by 2 2 gx 2 fy c, ( x) constant.
we get Thus, (x) is the polynomial of degree zero.
EBD_7443
www.jeeneetbooks.in

WWW.IIT-NEET.XYZ

2010-20 Target VITEEE


92. (d) Let E 1, E2 and E3 denote the events of
3
selecting boxes A, B, C respectively and A tan
be the event that a screw selected at random 2 5
is defective. Then, Now, 5 sin – 3 cos
1 1 1
P ( E1 ) , P ( E2 ) , P ( E3 ) 2 tan 1 tan 2
3 3 3 2 2
5. 3.
A 1 A 1 A 1 1 tan 2 1 tan 2
P ,P ,P 2 2
E1 5 E2 6 E3 7
By Baye's rule, the required probability 3 9
2. 1
A 5 25
P( E1 ) P 5. 3.
E E1 9 9
P 1 1 1
A A A A 25 25
P( E1 ) P P( E2 ) P P( E3 ) P
E1 E2 E3
16
6 3.
1 1 25 150 48 102
. 3.
E 3 5 9 34 34
P 1 1
A 1 1 1 1 1 1 25
. . .
3 5 3 6 3 7
1 1
1 95. (a) sin sin sin sin
6 6
5 42
1 1 1 107 [ sin( ) sin ]
5 6 7
1
sin sin
sin
6
cos 2
93. (c) [ Principal value [0, / 2]]
1 sin
1 cos
2
6
2 sin cos which is the required principal value.
4 2 4 2 96. (a) Let both of the ends of the rod are on x-axis
2 and y-axis. Let AB be rod of length l and
2 cos coordinates of A and B be (a, 0) and (0, b),
4 2
respectively.
sin Y
4 2
tan
4 2
cos
4 2 B (0, b)
94. (b) 3sin 5cos 5 3sin 5(1 cos )
P (h, k)
3.2 sin cos 5.2 sin 2
2 2 2
A (a, 0)
sin 2 sin cos X
2 2 O
and 1 cos 2 sin 2
2
www.jeeneetbooks.in

WWW.IIT-NEET.XYZ

Solved Paper 2010 2010-21

Let P (h, k) be the mid point of the rod AB. [Using the internal section formula]
0 a a h 3 k
Then, h ...(i) cos and sin
2 2 4 4
b 0 b Squar ing and adding both of these
k equations,
2 2
In OAB, (h 3)2 k2
cos 2 sin 2
OA 2
OB 2
AB 2 16 16

a 2
b 2
l 2 (h 3)2 k 2 16
Therefore, locus of point P is
(2h)2 (2k )2 l2 [using eq. (i)]
( x 3) 2 y 2 16 which is a circle.
2
l 99. (c) The equation of any normal to the parabola
h2 k2
4
y2 8 x is y = mx + 4m + 2m3 ...(i)
The equation of locus is
(using equation of normal of parabola in
l2 slope form y = mx – 2am – am3 and a = –2)
x2 y2
4 The given normal is
97. (a) Let L1 2x y 1 0 2x y k 0 y 2x k ...(ii)
Comparing eqs. (i) and (ii), we get
L2 3x 2 y 5 0
m = – 2 and –4m –2m3 = k
The equation of straight line passing k = 8 + 16 = 24
through the intersection point of the lines
L1 and L2 is given by 1 1 1 1
100. (a) lim .....
L1 L2 0 n 1 2 2 3 3 4 n( n 1)

(2 x y 1) (3 x 2 y 5) 0 1 1 1 1 1
Since, this line passes through the origin lim 1 ......
n 2 2 3 3 4
also
1 1
(0 0 1) (0 0 5) 0
n n 1
1
5 1
5 1 n
lim 1 lim
Required line is n n 1 n n 1
1 n 1
(2 x y 1) (3 x 2 y 5) 0 lim lim 1
5 n 1 n 1
n 1 1
3 2 n n
2 x 1 y 1 1 0
5 5 101. (d) Let P(x1, y1) be a point on the curve

7 3 y2 4ax ...(i)
x y 0 7x 3y 0
5 5 On differentiating y 2 4ax w.r.t. 'x', we get
98. (c) Let coordinates of P be (h, k), then
dy
2(10 cos ) 3(5) 2y 4a
h 4 cos 3 dx
2 3
dy 2a
2(10 sin ) 3(0)
and k 4 sin dx ( x1 , y1 ) y1
2 3
EBD_7443
www.jeeneetbooks.in

WWW.IIT-NEET.XYZ

2010-22 Target VITEEE


Thus, the equation of normal at (x1, y1) is
1 2x 2
y 103. (a) Let I sin dx
y y1 – 1 ( x x1 ) 4 x 2 8 x 13
2a
y1 x 2ay y1 ( x1 2a) ...(ii) 1 2x 2
I sin dx
2
But lx + my = 1 ...(iii) 4x 8x 4 9
is also a normal.
Therefore, coefficients of eqs. (ii) and (iii), 1 2x 2
I sin dx
must be proportional. (2 x 2)2 32
y1 2a y1 ( x1 2a ) Substituting 2x + 2 = 3 tan ,
i.e.,
l m 1 2dx 3sec 2 d , we get
2al 1 1 3tan 3 2
y1 and x1 2a I sin sec d
m l 3sec 2
Putting these values of x1 and y1 in eq. (i),
3
we get I sin 1 (sin ) sec 2 d
2
2
2al 1 3
4a 2a I sec 2 d
m l 2
3
4a 2l 2 4 a 8a 2 l I tan tan d
2
2
m l (integrating by parts)
3 2 2 3 2 2
al m 2am l al 2alm m 3
I [ tan log | sec |] c
f ( x ) dx f ( x) 2
102. (a)
3 1 2x 2 2x 2
d tan
f ( x) f ( x) 2 3 3
dx
2
d 2x 2
f ( x) f ( x)dx log 1 c
dx 3

Now, { f ( x )}2 dx f ( x) f ( x )dx 1 2x 2


( x 1) tan
3
d
f ( x ) f ( x)dx f ( x) f ( x )dx dx 3 4 x 2 8 x 13
dx log c
4 9
(integrating by parts)
104. (c) The equation of ellipse is
f ( x) f ( x) f ( x ) f ( x ) dx
3 x2 2 y2 6x 8 y 5 0
2 { f ( x)}2 dx { f ( x)}2
3( x 2 2 x) 2( y 2 4 y) 5 0
1 3( x 2 2 x 1) 2( y 2 4 y 4)
{ f ( x)}2 dx { f ( x )}2
2 5 3 8 0
www.jeeneetbooks.in

WWW.IIT-NEET.XYZ

Solved Paper 2010 2010-23

106. (d) We have


3( x 1)2 2( y 2) 2 6
f ( x) log x cos x
( x 1)2 ( y 2) 2 f (x) is defined for cos x > 0.
1
2 3 x > 0, x 1
Comparing with
cos x > 0 x
( x h) 2 ( y k )2 2 2
1 , we get Also, x > 0, x 1
a2 b2
h = –1, k = 2, a2 = 2, b2 = 3
Here, centre (h, k) = (–1, 2) Domain of f is 0, {1}
2
And using a2 = b2 (1 – e2)
1 x2
2 1 107. (b) y sin
2 3(1 e ) e 1 x2
3
And foci are (h, k + be) and (h, k – be) x2
=(–1, 2 + 1) and (–1, 2 – 1) For y to be defined 1
1 x2
= (–1, 3) and (–1, 1)
105. (b) We have the two hyperbolas as which is true for all x R.
x2 y2 1 x2
1 ...(i) Now, y sin
a2 b2 1 x2
y2 x2 x2 sin y
and 1 ...(ii)
a b 2 2
2
sin y x
1 x 1 sin y
Any tangent to the hyperbola eq(i)
y = mx + c For the existance of x
sin y 0 and 1 – siny > 0
where c a 2 m 2 – b2 … (iii)
But this tangent touches the parabola 0 sin y 1 0 y
eq. (ii) also 2
Thus, range of the given function is
( mx c ) 2 x2
– 1
a2 b2 0, .
2
2 2 2 2 2 2 2 2
b (m x c 2mcx ) – a x a b
108. (c) x sec cos
2 2 2 2 2 2 2 2
(b m a )x 2mcb x b (c a ) 0 dx
For the tangency, it should have equal roots sec tan sin ,
d
(2mcb2 )2 4(b2 m 2 a 2 ).b 2 (c 2 a2 ) y secn cosn
4m 2 c 2 b 4 4b2 (b2 m 2c 2 b 2 m2 a2 a 2 c2 a4 ) dy
n sec n 1
sec tan n cos n 1
sin
c 2
a 2
b m 2 2 d

a 2 m2 b2 a2 b 2 m 2 [ using Eq. (iii)] dy (sec n tan cos n 1 sin )


n
dx (sec tan sin )
(a 2 b 2 )m 2 a2 b2

m2 1 m 1 dy (sec n cos n ) tan


n
Hence, the equation of common tangent are dx (sec cos ) tan

y x a 2 b2
EBD_7443
www.jeeneetbooks.in

WWW.IIT-NEET.XYZ

2010-24 Target VITEEE


111. (c) Required area = Shaded area
dy n(sec n cos n )
dx (sec cos ) Y

2
dy n 2 {(sec n – cos n ) 2 4}
2
dx (sec – cos ) 4
X
2 2 2
O
dy n (y 4)
2
dx x 4
x=– x=
2
dy
( x2 4) n2 ( y 2 4)
dx
| sin x | dx 2 | sin x | dx
0
109. (d) y x y x y ......
2[cos x]0 2(cos cos 0)
y2 x y x y ...... = 4 sq units
112. (d) Length of normal = c
y2 x y y y2 x 2y
2
dy
( y 2 x)2 2 y y 1 c
dx
On differentiating both sides w.r.t. x, we get
2
dy dy dy
2( y 2 x) 2 y 1 2 y2 1 c2
dx dx dx

dy dy Clearly, this is the differential equation of


2( y 3 xy ) ( y2 x) degree 2.
dx dx
113. (d) a 2iˆ 2 ˆj 3kˆ
dy
(2 y 3 2 xy 1) y2 x
dx b –iˆ 2 ˆj kˆ
dy y2 x c 3iˆ ˆj
dx 3
2y 2 xy 1 Now,
x a tb (2iˆ 2 ˆj 3kˆ) ( tiˆ 2tjˆ tkˆ)
dt
110. (a)
1 | t | t2 1 6 (2 t )iˆ (2 2t ) ˆj (3 t )kˆ

Since, a tb is perpendicular to c
[sec 1 t ]1x
6 (a tb) c 0

1 {(2 t )iˆ (2 2t ) ˆj (3 t ) kˆ} (3iˆ ˆj ) 0


sec x sec 1 1
6 3(2 t ) 2 2t 0
1 6 3t 2 2t 0 t 8
sec x 0 x sec
6 6 114. (c) The given line is

2 r 2iˆ 2 ˆj 3kˆ (iˆ ˆj 4kˆ)


x
3 On comparing it with r a tb , we get
www.jeeneetbooks.in

WWW.IIT-NEET.XYZ

Solved Paper 2010 2010-25

a 2iˆ 2 ˆj 3kˆ, b iˆ ˆj 4kˆ 3


r
Also, the plane is 2
Taking second and third part of eq. (iii), we
r (iˆ 5 ˆj kˆ) 5
get
On comparing it with r n d , we get 3r – 1 – k = 8r +2
3r 1 k 8r 2 0 k 5r 3
n iˆ 5 ˆj kˆ and d = 5
3 3
Since, b n (iˆ ˆj 4kˆ) (iˆ 5 ˆj kˆ) k 5 3 r
2 2
= 1 – 5 + 4 =0
Given line is parallel to the given plane. 15 9
k 3 k
Now, distance between the line and the 2 2
plane is given by required distance
117. (a) Given curves are y 3x ...(i)
a n d
and y 5 x ...(ii)
|n|
intersect at the point (0, 1).
| (2iˆ 2 ˆj 3kˆ) (iˆ 5 ˆj kˆ) 5 | Now, differentiating eqs. (i) and (ii) w.r.t. x,
we get
1 25 1
dy dy
| 2 10 3 5 | 10 3x log 3 and 5 x log 5
dx dx
27 3 3
dy
115. (a) The equation of the sphere concentric with log 3 and dy log 5
the sphere dx (0,1) dx (0,1)

x2 y2 z 2 4 x 6 y 8z 5 0 is m1 log 3 and m2 log 5


2 2 2
x y z 4 x 6 y 8z c 0 ...(i) Angle between these curves is given by
Since, this sphere eq. (i) passes through m1 m2
tan
origin, therefore 1 m1m2
0+0+0–0–0–0+c=0
c 0 log 3 log 5
tan
Hence, the required equation of sphere is 1 log 3 log 5
x 2 y 2 z 2 4 x 6 y 8z 0 1 log 3 log 5
tan
116. (b) We have the lines 1 log 3log 5
x 1 y 1 z 1 118. (d) We have the given equation as
...(i)
2 3 4 x 2 4 xy y 2 x 3y 2 0
x 3 y k z On comparing this equation with
and ...(ii)
1 2 1 ax2 2hxy by 2 2 gx 2 fy c 0 , we
Let a point (2r 1, 3r 1, 4r 1) be on the get
line Eq. (i). If this is an intersection point of
3
both the lines, then it will lie on Eq. (ii), also a ,h 2, b 1, g ,c 2 , f
2 2
2r 1 3 3r 1 k 4r 1 Since, given equation represents a parabola
...(iii)
1 2 1
h2 ab 4 1 4
Taking first and third part of eq. (iii), we get
2r – 2 = 4r + 1
EBD_7443
www.jeeneetbooks.in

WWW.IIT-NEET.XYZ

2010-26 Target VITEEE


119. (c) The given two circles are 120. (a) A (–2, 1), B (2, 3) and C (–2, –4) are three
2 2 given points.
2x 2y 3x 6 y k 0
Slope of the line BA
3 k 1 3 1
x2 y2 x 3y 0 ...(i) m1
2 2 2 2 2
and x2 y 2 4 x 10 y 16 0 ...(ii)
y2 y1
Since, general equation of circle is Using slope formula, m
x2 x1
x2 y 2 2 gx 2 fy c 0 ...(iii) Slope of the line BC
Therefore, comparing eqs. (i) and (ii) with
4 3 7
eq. (iii), we get m2
2 2 4
3 3 k Now, angle between AB and BC is given by
g1 , f1 , c1
4 2 2
17
and g2 2, f 2 5, c2 16
m1 m2 24
Both the circles cut orthogonally, tan
1 m1m2 1 7
2( g1 g2 f1 f 2 ) c1 c2 1
2 4
3 15 k
2 16 10 2
2 2 2 tan tan
15 3
k k
18 16 2 k 4 1 2 1 2
2 2 tan tan
3 3
[ | x | x]
www.jeeneetbooks.in

WWW.IIT-NEET.XYZ

VITEEE
SOLVED PAPER 2009
PART - I (PHYSICS) 4. A motor of power P0 is used to deliver water at a
certain rate through a given horizontal pipe. To
1 When a wave traverses a medium the increase the rate of flow of water through the
displacement of a particle located at x at a time t same pipe n times, the power of the motor is
is given by y = a sin (bt – cx). Where a, b and c increased to P1. The ratio of P1 to P0 is
are constants of the wave. Which of the (a) n : 1 (b) n2 :1
3
(c) n : 1 (d) n4 :1
following is a quantity with dimensions?
5. A body of mass 5 kg makes an elastic collision
y with another body at rest and continues to move
(a) (b) bt
a in the original direction after collision with a
b 1
(c) cx (d) velocity equal to th of its original velocity..
c 10
2. A body is projected vertically upwards at time Then the mass of the second body is
t = 0 and it is seen at a height H at time t1 and t2 (a) 4.09 kg (b) 0.5 kg
second during its flight. The maximum height (c) 5 kg (d) 5.09 kg
attained is (g is acceleration due to gravity) 6. A particle of mass 4 m explodes into three pieces
of masses m, m and 2m. The equal masses move
g (t2 t1 )2 g (t1 t2 ) 2 along X-axis and Y-axis with velocities 4 ms–1and
(a) (b) 6 ms–1 respectively. The magnitude of the
8 4 velocity of the heavier mass is
g (t1 t2 ) 2 g (t2 t1 )2 (a) 17 ms–1 (b) 2 13 ms–1
(c) (d)
8 4 13 –1
3. A particle is projected up from a point at an angle (c) 13 ms–1 (d) ms
with the horizontal direction. At any time t, if p 2
7. A body is projected vertically upwards from the
is the linear momentum, y is the vertical surface of the earth with a velocity equal to half
displacement, x is horizontal displacement, the the escape velocity. If R is the radius of the earth,
graph among the following which does not maximum height attained by the body from the
represent the variation of kinetic energy KE of surface of the earth is
the particle is R R
(a) (b)
6 3
(A) KE (B) KE 2R
(c) (d) R
3
y t 8. The displacement of a particle executing SHM is
given by

(C) KE (D) KE y 5sin 4t


3
If T is the time period and the mass of the particle
x p2 is 2g, the kinetic energy of the particle when
t = T/4 is given by
(a) graph (A) (b) graph (B)
(a) 0.4 J (b) 0.5 J
(c) graph (C) (d) graph (D) (c) 3 J (d) 0.3 J
EBD_7443
www.jeeneetbooks.in

WWW.IIT-NEET.XYZ

2009-2 Target VITEEE


9. If the ratio of lengths, radii and Young's modulus Which of the following curves represents the
of steel and brass wires shown in the figure are equivalent cyclic process?
a, b and c respectively, the ratio between the A B A B
increase in lengths of brass and steel wires would be
P V
(a) (b)
b2a bc C C
(a) (b) Brass D
2c 2a2 D
2kg T T
A B
2 a A B
ba
(c) (d) Steel P
2c 2b2c 2kg V
(c) C
(d)
10. A soap bubble of radius r is blown up to form a D D C

bubble of radius 2r under isothermal conditions. T


If T is the surface tension of soap solution, the T
15. An ideal gas is subjected to cyclic process
energy spent in the blowing involving four thermodynamic states, the
(a) 3 Tr2 (b) 6 Tr2 amounts of heat (Q) and work (W) involved in
(c) 12 Tr 2 (d) 24 Tr2 each of these states are
11. Eight spherical rain drops of the same mass and Q1 = 6000 J, Q2 = –5500 J; Q3 = –3000 J;
radius are falling down with a terminal speed of Q4 = 3500 J
6 cm-s–1. If they coalesce to form one big drop, W1= 2500 J; W2 = –1000 J; W3 = –1200 J;
what will be the terminal speed of bigger drop? W4 = x J.
(Neglect the buoyancy of the air) The ratio of the net work done by the gas to the
(a) 1.5 cm-s–1 (b) 6 cm-s–1 total heat absorbed by the gas is ). The values
(c) 24 cm-s –1 (d) 32 cm-s–1 of x and respectively are
12. A clock pendulum made of invar has a period of (a) 500 ; 7.5 % (b) 700; 10.5 %
0.5 s, at 20°C. If the clock is used in a climate (c) 1000 ; 21 % (d) 1500; 15 %
where the temperature averages to 30°C, how 16. Two cylinders A and B fitted with pistons contain
much time does the clock lose in each oscillation? equal number of moles of an ideal monoatomic
(For invar, = 9 × 10–7/°C, g = constan t) gas at 400 K. The piston of A is free to move while
(a) 2.25 × 10–6 s (b) 2.5 × 10–7 s that of B is held fixed. Same amount of heat energy
(c) 5 × 10 s –7 (d) 1.125 × 10–6s is given to the gas in each cylinder. If the rise in
13. A piece of metal weighs 45 g in air and 25 g in a temperature of the gas in A is 42 K, the rise in
liquid of density 1.5 × 103 kg-m–3 kept at 30°C. temperature of the gas in B is
When the temperature of the liquid is raised to (a) 21 K (b) 35 K
40° C, the metal piece weights 27 g. the density (c) 42 K (d) 70 K
of liquid of 40°C is 1.25 × 103 kg–m–3. the 17. Three rods of same dimensional have thermal
coefficient of linear expansion of metal is conductivity 3 K, 2 K and K. They are arranged
(a) 1.3 × 10–3/°C (b) 5.2 × 10–3/°C as shown in the figure below
–3
(c) 2.6 × 10 /°C (d) 0.26 × 10–3/°C 50°C
14. An ideal gas is subjected to a cyclic process T 2K
ABCD as depicted in the p-V diagram given 100°C 3K
below: K
0°C
Then, the temperature of the junction in steady
A B state is
P
200 100
(a) C (b) C
3 3
50
(c) 75°C (d) C
3
V
www.jeeneetbooks.in

WWW.IIT-NEET.XYZ

Solved Paper 2009 2009-3

18. Two sources A and B are sending notes of Superposition of which two waves give rise to
frequency 680 Hz. A listener moves from A and B interference?
with a constant velocity u. If the speed of sound (a) (i) and (ii) (b) (ii) and (iii)
in air is 340 ms–1, what must be the value of u (c) (i) and (iii) (d) (iii) and (iv)
so that he hears 10 beats per second? 23. The two lenses of an achromatic doublet should
(a) 2.0 ms–1 (b) 2.5 ms–1 have
(c) 3.0 ms –1 (d) 3.5 ms–1 (a) equal powers
19. Two identical piano wires have a fundamental
(b) equal dispersive powers
frequency of 600 cycle per second when kept
(c) equal ratio of their power and dispersive
under the same tension. What fractional increase
power
in the tension of one wires will lead to the
occurrence of 6 beats per second when both wires (d) sum of the product of their powers and
vibrate simultaneously? dispersive power equal to zero
(a) 0.01 (b) 0.02 24. Two bar magnets A and B are placed one over
(c) 0.03 (d) 0.04 the other and are allowed to vibrate in a vibration
20. In the Young's double slit experiment, the magnetometer. They make 20 oscillations per
intensities at two points P1 and P2 on the screen minute when the similar poles of A and B are on
are respectively I1 and I2. If P1 is located at the the same side, while they make 15 oscillations
centre of a bright fringe and P2 is located at a per minute when their opposite poles lie on the
distance equal to a quarter of fringe width fromP1, same side. If MA and MB are the magnetic
moments of A and B and if MA > MB, the ratio of
I1 MA and MB is
then I is
2 (a) 4 : 3 (b) 25 : 7
(c) 7 : 5 (d) 25 : 16
1
(a) 2 (b) 25. A bar magnet is 10 cm long is kept with its north
2
(N)-pole pointing north. A neutral point is formed
(c) 4 (d) 16 at a distance of 15 cm from each pole. Given the
21. In Young's double slit experiment, the 10th horizontal component of earth's field is 0.4 Gauss,
maximum of wavelength 1 is at a distance of y1, the pole strength of the magnet is
from the central maximum. When the wavelength
(a) 9 A-m (b) 6.75 A-m
of the source is changed to 2, 5th maximum is at
a distance of y2 from its central maximum. The (c) 27 A-m (d) 1.35 A-m
26. An infinitely long thin straight wire has uniform
y1
ratio y is 1
2 linear charge density of cm –1 . Then, the
3
2 1 2 2 magnitude of the electric intensity at a point
(a) (b) 18 cm away is (given 0 = 8.8 × 10–12 C2Nm–2)
2 1
(a) 0.33 × 1011 NC–1 (b) 3 × 1011 NC–1
1 2 (c) 0.66 × 1011 NC–1 (d) 1.32 × 1011 NC–1
(c) (d) 27. Two point charges –q and +q are located at
2 2 2 1
points (0, 0, – a) and (0, 0, a) respectively. The
22. Four light sources produce the following four
electric potential at a point (0, 0, z), where z > a is
waves:
(i) y1 a sin( t 1)
qa q
(a) 2 (b)
(ii) y2 a sin 2 t 4 0z 4 0a

(iii) y3 a sin( t 2) 2qa 2qa


(c) 2 2 (d) 2
(iv) y4 a sin(3 t ) 4 0 (z a ) 4 0 (z a2 )
EBD_7443
www.jeeneetbooks.in

WWW.IIT-NEET.XYZ

2009-4 Target VITEEE


28. In the adjacent shown circuit, a voltmeter of I ampere and the radius of the circular loop is R
internal resistance R, when connected across B metre. Then, the magnitude of magnetic
induction at the centre of the circular loop is
100
and C reads V. Neglecting the internal
3 R
resistance of the battery, the value of R is I O

A
0I 0 nI
(a) (b)
50k 2 R 2R
B 0I 0I
100V (c) ( 1) (d) ( – 1)
50k 2 R 2 R
33. The work function of a certain metal is 3.31 × 10–19
C J. Then, the maximum kinetic energy of
(a) 100 k (b) 75k photoelectrons emitted by incident radiation of
(c) 50k (d) 25k wavelength 5000 Å is (Given, h = 6.62 × 10–34 J-
29. A cell in secondary circuit gives null deflection s, c = 3 × 108 ms–1, e = 1.6 × 10–19 C)
for 2.5 m length of potentiometer having 10 m (a) 2.48 eV (b) 0.41 eV
length of wire. If the length of the potentiometer (c) 2.07 eV (d) 0.82 eV
wire is increased by 1 m without changing the 34. A photon of energy E ejects a photoelectron
cell in the primary, the position of the null point from a metal surface whose work function is W0.
now is If this electron enters into a uniform magnetic
(a) 3.5 m (b) 3 m field of induction B in a direction perpendicular
(c) 2.75 m (d) 2.0 m to the field and describes a circular path of radius
30. The following series L-C-R circuit, when driven r, then the radius r is given by, (in the usual
by an emf source of an gular frequency notation)
70 kilo-radians per second, the circuit effectively 2m(E W0 )
behaves like (a) (b) 2m(E W0 )eB
eB
100 µH 1µH 10
2e(E W0 ) 2m(E W0 )
(c) (d)
mB eB
35. Two radioactive materials x1 and x2 have decay
constants 10 and respectively. if initially they
have the same number of nuclei, then the ratio of
(a) purely resistive circuit the number of nuclei of x1 to that of x2 will be 1/e
(b) series R-L circuit after a time
(c) series R-C circuit (a) (1/10 ) (b) (1/11 )
(d) series L-C circuit with R = 0 (c) 11/(10 ) (d) 1/(9 )
31. A wire of length l is bent into a circular loop of 36. Current flowing in each of the following circuit A
radius R and carries a current I. The magnetic and B respectively are
field at the centre of the loop is B. The same wire
4 4
is now bent into a double loop of equal radii. If
both loops carry the same current I and it is in
the same direction, the magnetic field at the
centre of the double loop will be 4 4
(a) Zero (b) 2 B
(c) 4 B (d) 8 B
32. An infinitely long straight conductor is bent into 8V 8V
(Circuit A) (Circuit B)
the shape as shown below. It carries a current of
www.jeeneetbooks.in

WWW.IIT-NEET.XYZ

Solved Paper 2009 2009-5

(a) 1A, 2A (b) 2 A, 1 A 42. Identify the alkyne in the following sequence of
(c) 4 A, 2 A (d) 2 A, 4 A reactions,
37. A bullet of mass 0.02 kg travelling horizontally H2 Ozonolysis
with velocity 250 ms–1 strikes a block of wood of Alkyne A B
Lindlar 's catalyst only
mass 0.23 kg which rests on a rough horizontal
surface. After the impact, the block and bullet Wacker
CH 2 CH 2
move together and come to rest after travelling a Process
distance of 40 m. The coefficient of sliding friction (a) H3C — C — C — CH3
of the rough surface is (g = 9.8 ms–2) (b) H3C — CH2 — C CH
(a) 0.75 (b) 0.61 (c) H2C = CH— C CH
(c) 0.51 (d) 0.30 (d) HC C — CH2 — C CH
38. Two persons A and B are located in X-Y plane at 43. Fluorine reacts with dilute NaOH and forms a
the points (0, 0) and (0,10) respectively. (The gaseous product A. The bond angle in the
distances are measured in MKS unit). At a time molecule of A is
t = 0, they start moving simultaneously with (a) 104°40' (b) 103°
(c) 107° (d) 109°28'
velocities va 2 ˆjms 1 and vb 2ims ˆ 1
44. One mole of alkene X on ozonolysis gave
respectively. The time after which A and B are at one mole of acetaldehyde and one mole of
their closest distance is acetone. The IUPAC name of X is
(a) 2.5s (b) 4s (a) 2-methyl-2-butene (b) 2-methyl-1-butene
10 (c) 2-butene (d) 1-butene
(c) 1s (d) s 45. The number of p – d ‘pi’ bonds present in
2
XeO3 and XeO4 molecules, respectively are
39. A rod of length l is held vertically stationary (a) 3, 4 (b) 4, 2
with its lower end located at a point P, on the (c) 2, 3 (d) 3, 2
horizontal plane. When the rod is released to 46. The wavelengths of electron waves in two
topple about P, the velocity of the upper end of orbits is 3 : 5. The ratio of kinetic energy of
the rod with which it hits the ground is electrons will be
g (a) 25 : 9 (b) 5 : 3
(a) (b) 3gl (c) 9 : 25 (d) 3 : 5
l
47. Which one of the following sets correctly
represents the increase in the paramagnetic
g 3g
(c) 3 (d) property of the ions?
l l (a) Cu2+ > V2+ > Cr2+ > Mn2 +
40. A wheel of radius 0.4 m can rotate freely about (b) Cu2+ < Cr 2+ < V2+ < Mn2+
its axis as shown in the figure. A string is wrapped (c) Cu2+ < V 2+ < Cr2+< Mn2+
over its rim and a mass of 4 kg is hung. An angular (d) V2+< Cu2+ < Cr2+ < Mn2+
acceleration of 8 rad-s–2 is produced in it due to 48 . Electrons with a kinetic energy of 6.023 × 104 J/ mol
the torque. Then, moment of inertia of the wheel are evolved from the surface of a metal, when it
is ( g = 10 ms–2) is exposed to radiation of wavelength of
(a) 2 kg-m2 (b) 1 kg-m2 600 nm. The minimum amount of energy
(c) 4 kg-m 2
(d) 8 kg-m2 required to remove an electron from the metal
atom is
PART - II (CHEMISTRY) (a) 2.3125 × 10–19 J (b) 3 × 10–19 J
(c) 6.02 × 10–19 J (d) 6.62 × 10–34 J
41. Given that Hf(H) = 218 kJ/mol, express the 49. The chemical entities present in thermosphere
H—H bond energy in kcal/mol. of the atmosphere are
(a) 52.15 (b) 911 (a) O2 , O , NO (b) O3
(c) 104 (d) 52153
(c) N2, O2, CO2, H2O (d) O3, O2 , O2
EBD_7443
www.jeeneetbooks.in

WWW.IIT-NEET.XYZ

2009-6 Target VITEEE


50. The type of bonds present in sulphuric The correct answer is
anhydride are (a) Both (A) and (R) are true and (R) is the
(a) 3 and three p – d correct explanation of (A)
(b) 3 , one p – p and two p – d (b) Both (A) and (R) are true but (R) is not
(c) 2 and three p – d the correct explanation of (A)
(d) 2 and two p – d (c) (A) is true but (R) is not true
51. In Gattermann reaction, a diazonium group is (d) (A) is not true but (R) is true
replaced by X using Y. X and Y are 59. How many mL of perhydrol is required to
X Y produce sufficient oxygen which can be used to
completely convert 2 L of SO2 gas to SO3 gas?
(a) Cl – Cu/HCl (a) 10 mL (b) 5 mL
(b) CuCl2/HCl (c) 20 mL (d) 30 mL
Cl
60. pH of a buffer solution decreases by 0.02 units
(c) Cl – CuCl2/HCl when 0.12 g of acetic acid is added to 250 mL of
(d) Cl2 Cu2O/HCl a buffer solution of acetic acid and potassium
52. Which pair of oxyacids of phosphorus contains acetate at 27°C. The buffer capacity of the
' P—H' bonds? solution is
(a) H3PO4, H3PO3 (b) H3PO5, H4 P2O7 (a) 0.1 (b) 10
(c) H3PO3, H3PO2 (d) H3PO2, HPO3 (c) 1 (d) 0.4
53. Dipole moment of HCl = 1.03 D, HI = 0.38 D. 61. Match the following
Bond length of HC1 = 1.3 Å and HI =1.6 Å. The List I List II
ratio of fraction of electric charge, , existing on (A) Flespar (I) [Ag3Sb3]
each atom in HCl and HI is (B) Asbestos (II) Al2O3. H2O
(a) 12 : 1 (b) 2.7 : 1 (C) Pyrargyrite (III) MgSO4. H2O
(c) 3.3 : 1 (d) 1 : 3.3 (D) Diaspore (IV) KAlSi3O8
54. SiC14 on hydrolysis forms ‘X’ and HC1. (V) CaMg3(SiO3)4
Compound ‘X’ loses water at 1000°C and gives The correct answer is
(A) (B) (C) (D)
‘Y’. Compounds ‘X’ and ‘Y’respectively are
(a) IV V II I
(a) H2SiCl6 , SiO2 (b) H4SiO4, Si
(b) IV V I II
(c) SiO2, Si (d) H4 SiO4, SiO2
(c) IV I III II
55. 1.5 g of CdCl 2 was found to contain 0.9 g of Cd.
(d) II V IV I
Calculate the atomic weight of Cd.
62. Which one of the following order is correct for
(a) 118 (b) 112
the first ionisation energies of the elements?
(c) 106.5 (d) 53.25
(a) B < Be < N < O (b) Be < B < N < O
56. Aluminium reacts with NaOH and forms (c) B < Be < O < N (d) B < O < Be < N
compound ‘X’. If the coordination number of 63. What are X and Y in the following reaction
aluminium in ‘X’ is 6, the correct formula of X is sequence?
(a) [Al(H2O)4(OH)2]+ (b) [Al (H2O)3(OH)3]
Cl
2 2 Cl
(c) [Al(H2O)2 (OH)4 ]– (d) [Al (H2O)6 ](OH)3 C 2 H 5 OH X Y
57. The average kinetic energy of one molecule of (a) C2H5C1, CH3CHO
an ideal gas at 27°C and 1 atm pressure is (b) CH3CHO, CH3CO2H
(a) 900 cal K –1mo1–1 (c) CH3CHO, CC13CHO
(b) 6.21 × 10– 21 JK–1 molecule–1 (d) C2H5C1, CCl3CHO
(c) 336.7 JK–1 molecules–1 64. What are A, B, C in the following reactions?
(d) 3741.3 JK–1 mol–1
(i) (CH 3CO 2 ) 2 Ca A
58. Assertion (A) : K. Rb and Cs form superoxides.
Reason (R) : The stability of the superoxides HI
(ii) CH3 CO2 H B
increases from ‘K’ to ‘Cs’ due to decrease in Red P
lattice energy. P4 O10
(iii) 2CH3CO 2 H C
www.jeeneetbooks.in

WWW.IIT-NEET.XYZ

Solved Paper 2009 2009-7

A B C 69. During the depression in freezing point


(a) C2H6 CH3COCH3 (CH3CO)2O experiment, an equilibrium is established
(b) (CH3CO)2O C2H6 CH3COCH3 between the molecules of
(c) CH3COCH3 C2H6 (CH3CO)2O (a) liquid solvent and solid solvent
(d) CH3COCH3 (CH3CO)2O C2H6 (b) liquid solute and solid solvent
65. One per cent composition of an organic (c) liquid solute and solid solute
compound A is, carbon : 85.71% and hydrogen (d) liquid solvent and solid solute
14.29%. Its vapour density is 14. Consider the 70. Consider the following reaction,
following reaction sequence EtOH/H2O
C2H5Cl + AgCN X (major)
Cl2 /H2O (i) KCN/EtOH
A B C Which one of the following statements is true
(ii)H3O
for X?
Identify C. (I) It gives propionic acid on hydrolysis
(a) CH3 — CH — CO2 H (II) It has an ester functional group
| (III) It has a nitrogen linked to ethyl carbon
OH
(IV) It has a cyanide group
(b) HO — CH 2 — CH 2 — CO 2 H (a) IV (b) III
(c) HO — CH 2 — CO 2 H (c) II (d) I
71. For the following cell reaction,
(d) CH3 — CH 2 — CO 2 H
Ag | Ag | AgCl | Cl – | Cl2,Pt
66. How many tripeptides can be prepared by
linking the amino acids glycine, alanine and G f (AgCl) –109kJ / mol
phenyl alanine?
(a) One (b) Three G f (Cl – ) 129 kJ/ mol
(c) Six (d) Twelve
67. A codon has a sequence of A and specifies Gf (Ag ) 78kJ / mol
a particular B that is to be incorporated into a C.
E° of the cell is
What are A, B, C?
(a) –0.60 V (b) 0.60 V
A B C
(c) 6.0 V (d) None of these
(a) 3 bases amino acid carbohydrate
72. The synthesis of crotonaldeh yde from
(b) 3 acids carbohydrate protein
acetaldehyde is an example of ....... reaction.
(c) 3 bases protein amino acid
(a) nucleophilic addition
(d) 3 bases amino acid protein
(b) elimination
68. Parkinson's disease is linked to abnormalities in
the levels of dopamine in the body. The structure (c) electrophilic addition
of dopamine is (d) nucleophilic addition-elimination
73. At 25°C, the molar conductances at infinite
CH2NH2 CH2CH2CH2NH2 dilution for the strong electrolytes NaOH, NaCl
and BaCl 2 are 248 × 10–4 , 126 × 10–4 and
(a) (b)
280 × 10–4 Sm2 mol–1 respectively, m Ba(OH)2
OH OH
in Sm2 mol–1 is
OH OH
(a) 52.4 × 10–4 (b) 524 × 10–4
(c) 402 × 10 –4 (d) 262 × 10–4
CH2CH2NH2 CH2–CHCO2H 74. The cubic unit cell of a metal (molar mass = 63.55
NH2 g mol–1) has and edge length of 362 pm. Its
(c) (d) density is 8.92g cm -3 . The type of unit cell is
OH OH (a) primitive (b) face centred
OH OH (c) body centred (d) end centred
EBD_7443
www.jeeneetbooks.in

WWW.IIT-NEET.XYZ

2009-8 Target VITEEE


75. The equilibrium constant for the given reaction PART - III (MATHEMATICS)
is 100.
81. If f : [2, 3] R is defined by f(x) = x3 + 3x – 2,
N2 (g) + 2O2(g) 2NO2 (g) then the range f(x) is contained in the interval
(a) [1, 12] (b) [12, 34]
What is the equilibrium constant for the reaction
(c) [35, 50] (d) [–12, 12]
given below? 82. The number of subsets of {1, 2, 3, ..., 9} containing
1 at least one odd number is
NO2 (g) N 2 (g) O 2 (g) (a) 324 (b) 396
2
(c) 496 (d) 512
(a) 10 (b) 1 83. A binary sequence is an array of 0's and l's. The
(c) 0.1 (d) 0.01 number of n-digit binary sequences which
76. For a first order reaction at 27°C, the ratio of time contain even number of 0's is
required for 75% completion to 25% completion (a) 2n –1 (b) 2n –1
n
(c) 2 – 1 –1 (d) 2n
of reaction is
(a) 3.0 (b) 2.303 84. If x is numerically so small so that x2 and higher
(c) 4.8 (d) 0.477 powers of x can be neglected, then
77. The concentration of an organic compound in 3/2
2x
chloroform is 6.15 g per 100 mL of solution. A 1 .(32 5 x ) –1/5
3
portion of this solution in a 5cm polarimeter tube
causes an observed rotation of –1.2°. What is is approximately equal to
the specific rotation of the compound? 32 31x 31 32 x
(a) +12° (b) –3.9° (a) (b)
64 64
(c) –39° (d) +61.5°
31 32 x 1 2x
78. 20 ml of 0.1 M acetic acid is mixed with 50 mL of (c) (d)
potassium acetate. Ka of acetic acid = 1.8 × 10–5 64 64
at 27°C. Calculate concentration of potassium 85. The roots of
acetate if pH of the mixture is 4.8. ( x a )( x a 1) ( x a 1)( x a 2)
(a) 0.1 M (b) 0.04 M ( x a)( x a 2) 0
(c) 0.4 M (d) 0.02 M
a R are always
79. Calculate H° for the reaction,
(a) equal (b) imaginary
Na2O(s) + SO3(g) Na2SO4 (g) (c) real and distinct (d) rational and equal
given the following :
86. Let f (x)= x2 + ax + b, where a, b R. If f(x) = 0
1 has all its roots imaginary, then the roots of
(A) Na(s) + H2O(l) NaOH(s) + H (g) f (x)+ f (x) + f (x) = 0 are
2 2
(a) real and distinct (b) imaginary
H°= –146 kJ (c) equal (d) rational and equal
(B) Na2SO4(s) + H2O(l) 87. If f(x)= 2x4 – 13x2 + ax + b is divisible by
2NaOH(s) + SO3(g) x2 – 3x + 2, then (a, b) is equal to
H° = + 418 kJ (a) (–9, – 2) (b) (6, 4)
(C) 2Na2O(s) + 2H2 (g) 4Na(s) + 2H2O(l) (c) (9, 2) (d) (2, 9)
H° = + 259 kJ 88. If x, y, z are all positive and are the pth, qth and rth
(a) +823 kJ (b) –581 kJ terms of a geometric progression respectively,
(c) –435 kJ (d) +531 kJ then the value of the determinant
80. Which one of the following is the most effective log x p 1
in causing the coagulation of an As2S3 sol? log y q 1 equals
(a) KCl (b) AlCl3 log z r 1
(c) MgSO4 (d) K3Fe(CN)6 (a) log xyz (b) (p – 1) (q – 1)(r – 1)
(c) pqr (d) 0
www.jeeneetbooks.in

WWW.IIT-NEET.XYZ

Solved Paper 2009 2009-9

89. The locus of z satisfying the inequality 96. If m 1, m 2, m 3 and m 4 are respectively the
magnitudes of the vectors
z 2i
1 , where z = x + iy, is a1 2iˆ ˆj kˆ, a2 3iˆ 4 ˆj 4kˆ,
2z i
(a) x2 y2 1 (b) x2 y2 1 a3 iˆ ˆj kˆ, and a4 –iˆ 3 ˆj kˆ ,
then the correct order of m1, m2, m3 and m4 is
(c) x 2 y 2 1 (d) 2 x 2 3 y 2 1 (a) m3 < m1 < m4 < m2
90. If n is an integer which leaves remainder one (b) m3 < m1 < m2 < m4
(c) m3 < m4 < m1 < m2
when divided by three, then
(d) m3 < m4 < m2 < m1
(1 3i ) n (1 – 3i ) n equals 97. If X is a binomial variate with the range {0, 1, 2, 3, 4,
(a) –2n + 1 (b) 2n + 1 5, 6} and P(X = 2) = 4P(X = 4), then the parameter
n p of X is
(c) –(–2) (d) –2n
1 1 2 3
91. The period of sin 4 x cos 4 x is (a) (b) (c) (d)
3 2 3 4
4 2 98. The area (in square unit) of the circle which
(a) (b) (c) (d) touches the lines 4x + 3y = 15and 4x + 3y = 5is
2 2 4 2
(a) 4 (b) 3
92. If 3cos x 2sin x, then the general solution of (c) 2 (d)
99. The area (in square unit) of the triangle formed
sin 2 x – cos 2 x 2 sin 2 x is by x + y + 1 = 0 and the pair of straight lines
x2 – 3xy + 2y2 = 0 is
(a) n ( 1) n ,n Z
2 7 5 1 1
(a) (b) (c) (d)
n 12 12 12 6
(b) ,n Z 100. The pairs of straight lines x2 – 3xy + 2y2 = 0 and
2
x2 – 3xy + 2y2 + x – 2 = 0 form a
(a) square but not rhombus
(c) (4 n 1) ,n Z (b) rhombus
2
(c) parallelogram
(d) (2n 1) , n Z (d) rectangle but not a square
101. The equations of the circle which pass through
1 1 1 1 1 1 the origin and makes intercepts of lengths 4 and
93. cos 2sin 3cos
2 2 2 8 on the x and y-axes respectively are
(a) x2 + y2 ± 4x ± 8y = 0
4 tan 1 ( 1) equals (b) x2 + y2 ± 2x ± 4y = 0
(c) x2 + y2 ± 8x ± 16y = 0
19 35 47 43 (d) x2 + y2 ± x ± y = 0
(a) (b) (c) (d)
12 12 12 12 102. The point (3, –4) lies on both the circles
94. In a ABC x2 + y2 – 2x + 8y + 13 = 0
and x2 + y2 – 4x + 6y + 11 = 0
(a b c )(b c a )(c a b)(a b c ) Then, the angle between the circles is
4b2 c 2 1
1
equals (a) 60° (b) tan
(a) cos2A (b) cos2B 2
2
(c) sin A (d) sin2 B 3 1
(c) tan (d) 135°
95. The angle between the lines whose direction 5
cosines satisfy the equations l + m + n = 0,
103. The equation of the circle which passes through
l2 m2 n2 0 is the origin and cuts orthogonally each of the
circles x 2 y2 6x 8 0 and
(a) (b) (c) (d)
6 4 3 2 x2 y2 2x – 2 y 7 is
EBD_7443
www.jeeneetbooks.in

WWW.IIT-NEET.XYZ

2009-10 Target VITEEE


(a) 3x2 + 3y2– 8x – 13y = 0 (a) 0 (b) tan t
(b) 3x2 + 3y2 – 8x + 29y = 0 (c) 1 (d) sin t cos t
(c) 3x2 + 3y2 + 8x + 29y = 0
3x2 + 3y2 – 8x – 29y = 0 d x 1 1
(d) 113. a tan 1 x b log
104. The number of normals drawn to the parabola dx x 1 4
x 1
y2 = 4x from the point (1, 0) is a – 2b is equal to
(a) 0 (b) 1 (c) 2 (d) 3 (a) 1 (b) –1 (c) 0 (d) 2
105. If the circle x2 + y2 = a2 intersects the hyperbola
a sin 1 x
xy = c2 in four points (x1, y1) for i = 1, 2, 3 and 4, 114. y e (1 x 2 ) yn 2 (2n 1) xyn 1
is
then y1 + y2 + y3 + y4 equals equal to
(a) 0 (b) c (c) a (d) c 4
106. The mid point of the chord 4x – 3y = 5 of the (a) (n2 a 2 ) yn (b) (n2 – a 2 ) yn
hyperbola 2x2 – 3y2 = 12 is
(c) (n2 a 2 ) yn (d) (n2 a2 ) yn
5
(a) 0, (b) (2, 1) 115. The function f ( x) x3 ax 2 bx c, a 2 3b
3
has
5 11 (a) one maximum value
(c) ,0 (d) ,2
4 4 (b) one minimum value
107. The perimeter of the triangle with vertices at (c) no extreme value
(1, 0, 0), (0, 1, 0) and (0, 0, 1) is (d) one maximum and one minimum value
(a) 3 (b) 2 2 sin 2 x x
(c) 2 2 (d) 3 2 116. e dx is equal to
1 cos 2 x
108. If a line in the space makes angle , and with (a) – ex cot x + c (b) ex cot x + c
the coordinate axes, then x
(c) 2e cot x + c (d) – 2ex cot x + c
cos 2 cos 2 cos 2 sin 2 sin 2 117. If I n sin n x dx , then nI n (n 1) I n 2
2 equals equals
sin
(a) –1 (b) 0 (c) 1 (d) 2 (a) sin n 1 x cos x (b) cos n 1
x sin x
109. The radius of the sphere
x2 + y2 + z2 = 12x + 4y + 3z is (c) sin n 1
x cos x (d) cosn 1
x sin x
(a) (b) (c) (d) 52
118. The line x divides the area of the region
x 3 4
x 5
110. lim equals bounded by y = sin x, y = cos x and x-axis
x x 2
(a) e (b) e2 (c) e3 (d) e5 0 x into two regions of areas A1 and
111. If f : R R is defined by 2
A2. Then A1 : A2 equals
2sin x sin 2 x (a) 4 : 1 (b) 3 : 1 (c) 2 : 1 (d) 1 : 1
, if x 0
f ( x) 2 x cos x 119. The solution of the differential equation
,
a, if x 0 dy
sin ( x y ) tan ( x y ) – 1 is
then the value of a so that f is continuous at 0 is dx
(a) 2 (b) 1 (c) –1 (d) 0 (a) cosec (x + y) + tan (x + y) = x + c
(b) x + cosec (x + y) = c
1 1
112. x cos , (c) x + tan (x + y) = c
1 t2 (d) x + sec (x + y) = c
120. If p (~ p q) is false, the truth value of p
1 t dy
y sin and q are respectively
1 t 2 dx is equal to (a) F, T (b) F, F (c) T, F (d) T, T
www.jeeneetbooks.in

WWW.IIT-NEET.XYZ

Solved Paper 2009 2009-11

SOLUTIONS
PART - I (PHYSICS) Maximum height attained
2
1. (d) Here, y a sin (bt cx) 1 1 t t
H max g (T ) 2 g 1 2
Comparing this equation with geneal wave 2 2 2
equation
2 t 2 x 1 (t1 t2 )2
y a sin g
T 2 4
1
2 2 Or, H max g (t1 t 2 ) 2 m
we get b ,c 8
T
p
y [ L] 3. (a) Momentum, p m v v
(a) Dimensions of = Dimensionless m
a [ L]
y
2 [T ] v
(b) Dimensions of bt t
T [T ] vy
= Dimensionless
P vx
[ L] 2 (x, y)
(c) Dimensions of cx x
[ L] x
O
= Dimensionless
Kinetic energy, KE
2
b 1 2 1 p2 1 2
(d) Dimensions of T [ LT 1 ] mv m 2 p
c 2 T 2 2 m 2m

1
2. (b) Let t' be the time taken by the body to fall or, KE p2 constant
from point C to B. 2m
Hence, the graph between KE and p2 will
t2 t1 be linear.
Then t1 2t ' t2 t' ...(i)
2
1 2
Now, kinetic energy KE = mv
C 2
The velocity component at point P,
t' vy (u sin
gt ) and vx u cos
Resultant velocity at point P,
B Hmax
v v y ˆj vx iˆ
t1 H
(u sin gt ) ˆj u cos iˆ
A
Total time taken to reach point C |v| (u cos )2 (u sin gt ) 2
t2 t1 u 2 cos 2 u 2 sin 2 g 2t 2 2ugt sin
T t1 t ' t1
2
2t1 t2 t1 t1 t2
u 2 (cos 2 sin 2 ) g 2 t 2 2ugt sin
2 2
EBD_7443
www.jeeneetbooks.in

WWW.IIT-NEET.XYZ

2009-12 Target VITEEE

1
If rate of flow of water is increased by n
KE m (u 2 g 2t 2 2ugt sin ) times, i.e., (nx).
2 Increased power,
i.e., KE t 2
mgy ' y'
Hence, graph will be parabolic intercept on P1 mg nmgx ...(ii)
t t
y-axis.
Hence, the graph between KE and t. The ratio of power,
Now, in case of height P1 nmgx
; P1 : P0 n :1
1 P0 mgx
KE m(v 2 ) and v 2 (u 2
2 gy )
2 5. (a) Mass of the first body m1 = 5 kg and for
1 elastic collision coefficient of restitution,
KE m(u 2 2 gy ) e = 1.
2
u =u
1 m1 1 M
KE mgy mu 2
2 u2 = 0
1 Let initially body m1 moves with velocity v
Intercept on y-axis = mu 2
2 u
after collision velocity becomes .
10
Let after collision velocity of M block
B becomes (v2).
1 2 By conservation of momentum
mu –mg = tan
2 m1u1 m2 u2 m1v1 m2 v2

O A u
or 5u M 0 5 Mv2
10
1 2 u
Now, KE mv
or 5u Mv2 ...(i)
2 2
Since, v1 v2 e(u1 u2 )
u u
or v2 1(u ) or u v2
KE 10 10
11u
or v2 ...(ii)
x 10
2 Substituting value of v2 in Eq. (i) from Eq. (ii)
1 x
KE m u 11u
2 t 5u M
2 10
i.e., KE x 2 . Thus graph between KE and
x will be parabolic. 1 11 9 10 45
or 5 M M
4. (a) Power of motor initially = P0 2 10 2 11 11
Let, rate of flow of motor = (x)
4.09 kg
Since, power,
6. (c) Let third mass particle (2m) moves making
work mgy y angle with X-axis.
P0 mg
time t t The horizontal component of velocity of
2m mass particle = u cos
y And vertical component = u sin
x = rate of flow of water = mgx ...(i)
t
www.jeeneetbooks.in

WWW.IIT-NEET.XYZ

Solved Paper 2009 2009-13

Velocity of particle
Y
dy 5d
u=? sin 4t
–1 dt dt 3
6ms 2m1
5cos 4t 4 20cos 4t
m 3 3
T
X Velocity at t
4m –1 4
m 4ms
dy T
From conservation of linear momentum in 20 cos 4
dt t
T 4 3
X-direction 4
m1u1 m2 u2 m1v1 m2 v2
or u 20cos T ...(ii)
or 0 m 4 2m(u cos ) 3
or 4 2u cos or 2 u cos ...(i) Comparin g the given equation with
Again, applying law of conservation of standard equation of SHM.
linear momentum in Y-direction y a sin ( t )
0 m 6 2m(u sin ) We get, = 4

6 As
2 2
u sin or 3 u sin T
...(ii) T
2
Squaring Eqs. (i) and (ii) and adding, 2
2 2 2 2 or T =
(4) (9) u cos u sin 4 2
Now, putting value of T in Eq. (ii), we get
u 2 (cos2 sin 2 )
or 13 u 2 u 20 cos 20sin
2 3 3
u 13 ms 1
10 3
7. (b) Here, maximum height attained by a
The kinetic energy of particle,
projectile
1 2 1 3
v2 R KE mu 2 10 ( 10 3) 2
h ...(i) 2 2
2 gR v 2 3
10 100 3 0.3J
Velocity of body = half the escape velocity
l1 r Y
ve 9. (d) Given, a, 1 b, 1 c
i.e., v l2 r2 Y2
2
Free body diagram of the two blocks brass
2 gR 2 gR gR and steel are
or v v2 v2
2 4 2
Now, putting value of v2 in Eq. (i), we get T' T

gR gR 2 Brass Steel
R
2 2 R
Height, h
gR 3 gR 3
2 gR 2g
2 2
8. (d) Particle executing SHM.
T 2g
Displacement y 5sin 4t ...(i)
3
EBD_7443
www.jeeneetbooks.in

WWW.IIT-NEET.XYZ

2009-14 Target VITEEE


Let Young's modulus of steel is Y1 and of
brass is Y2. 2 r2
vT ( )g
9
F1 l1
Y1 ...(i) where is coefficient of viscosity and is
A1 l1
density of drop is density of liquid.
F2 l2 Terminal speed drop is 6 cm s–1
and Y2 A2 l2
...(ii)
2 r2
6 ( )g ...(i)
Dividing Eq. (i) by (ii), 9
F1 l1 Let terminal velocity becomes v' after
coalesce, then
Y1 A1 l1
Y2 F2 l2 2 R2
v' ( )g ...(ii)
A2 l2 9
Dividing Eq. (i) by (ii), we get
Y1 F1 A2 l1 l2
or Y2 F2 A1 l2 l1
...(iii) 2 r2
( )g
Force on steel wire from free body diagram 6 9 6 r2
T = F1 = (2g) newton v' 2 R2 v' (2r ) 2
( )g
Force on brass wire from free body diagram 9
F2 T ' T 2 g (4 g ) newton or v' = 24 cm s–1
Now, putting the value of F1, F2, in Eq. (iii), 12. (a) Time period of oscillation,
we get
l dT 1 dl
Y1 2g r22 l1 l2 T 2
g T 2 l
Y2 4g r12 l2 l1
dl
As, dt
1 1 l2 l
or c a
2 b2 l1 dT1 1
dt 9 10 7 (30 20)
T 2 2
l1 a
or = 4.5 × 10–6
l2 2b 2 c Loss in time = 4.5 × 10–6 × 0.5
10. (d) Initially area of soap bubble, A1 = 4 r2 = 2.25 × 10–6 s
Under isothermal condition radius becomes 13. (c) Volume of the metal at 30°C
2r.
loss of weight
Area A2 4 (2 r )2 16 r 2 V30
specific gravity × g
Increase in surface area
(45 25) g
A 2( A2 A1 ) 2(16 r 2 4 r 2 ) 24 r 2 13.33 cm3
1.5 g
Energy spent,
Similarly, volume of metal at 40°C
W T A T 24 r 2 24 Tr 2 J
(45 27) g
11. (c) Let radius of big drop = R. V40 14.40 cm 3
1.25 g
4 3 4
R r3 8 Now, V40 V30 [1 (t2 t1 )]
3 3
R 2r V40 V30 14.40 13.33
Here r = radius of small drops. or
V30 (t 2 t1 ) 13.33(40 30)
Now, terminal velocity of drop in liquid
= 8.03 × 10–3/°C
www.jeeneetbooks.in

WWW.IIT-NEET.XYZ

Solved Paper 2009 2009-15

Coefficient of linear expansion of the W1 W2 W3 W4


metal 100
Q1 Q4
3
8.03 10 3
2.6 10 / C (2500 1000 1200 700)
3 3 100
6000 3500
14. (a) Process A B C D A is
clockwise. 1000
100 10.5%
During A B , pressure is constant and 9500
1 16. (c) From first law of thermodynamics
B C , process follows p
ie., T is Q U W
V
constant. During process C D , both p For cylinder A pressure remains constant.
Work done by a system
an d V changes and process D A
R
1 W (T1 T2 )
follows p i.e., T is constant. 1
V
5
A B For monoatomic gases, 1;
3

p 1 R 3
C W (442 400) R 42
5 2
1
D 3
or W = 63R
V
But U 0 , for cylinder A
Hence, equivalent cyclic process is has
follows. Q 0 63R 63R
For cylinder B volume is constant,
W = 0 and Q CV T
A B For monoatomic gas
3 3
p CV R Q 1 R T
2 2
C As heat given on both cylinder is same
D 3
63R R T T 42 K
T 2
17. (a) From figure, H = H1 + H2
15. (b) From first law of thermodynamics
3 KA(100 T ) 2 KA(T 50) KA(T 0)
Q U W or U Q W
l l l
U1 Q1 W1 6000 2500 3500 J 300 3T 2T 100 T 6T 400
U2 Q2 W2 5500 1000 4500 J 200
T C
U3 Q3 W3 3000 1200 1800 J 3
U 4 Q4 W4 3500 x 18. (b) Let listener go from A B with velocity
For cyclic process U 0 ( ).
As Bs
3500 4500 1800 3500 x 0
or x 700 J 680 Hz 680 Hz
And the apparent frequency of sound from
output source A by listener using Doppler's effect,
Efficiency, 100
input
EBD_7443
www.jeeneetbooks.in

WWW.IIT-NEET.XYZ

2009-16 Target VITEEE


constructive inference takes place.
v vo 340 u The position of fringe at p2.
n' n n ' 680
v vs 340 0
n D
The apparent frequency of sound from x
d
source B by listener
v vo 340 u Given, '
n '' n 680 4
v vs 340 0
P2
Listener hear 10 beats per second.
Hence, n'' – n' = 10
340 u 340 u S1
680 680 10
340 340 P1
d
2(340 u 340 u) 10 S2
D
u 2.5 ms –1
19. (b) When both the wires vibrate
simultaneously, beats per second, D n D 1
n1 n2 6 or n
4d d 4
1 T 1 T'
6
I1 a2
or 16 :1
2l m 2l m I2 a
2

1 T' 1 T 4
6
2l m 2l m 21. (a) Position fringe from central maxima
n 1D
1 T' 1 T' y1
600 6 606 ...(i) d
2l m 2l m Given, n = 10
Given that fundamental frequency
10 1 D
y1 ...(i)
1 T d
600 ...(ii)
2l m For second source
Dividing Eq. (i) by (ii), 5 2D
y2 ...(ii)
1 T' d
2l m 606 10 1 D
1 T 600 y1 d 2 1
2l m y2 5 2D 2
d
T' T' 22. (c) Inteference takes place between two waves
(1.01) (1.02)%
T T having equal frequency and propagate in
same direction.
T ' T (1.02)
Hence, y1 a sin( t 1 )
Increase in tension
T ' T 1.02 T (0.02T ) y3 a 'sin( t 2 )
T = 0.02 will give interference as the two waves have
same frequency .
D 23. (d) The two lenses of an achromatic doublet
20. (d) Fringe width
d should have, sum of the product of their
Let be the amplitude of the place where powers and dispersive power = zero.
www.jeeneetbooks.in

WWW.IIT-NEET.XYZ

Solved Paper 2009 2009-17

24. (b) Ratio of magnetic moments According to Gauss theorem


MA Td2 Ts2 vs2 vd2 E dS
q
MB Td2 Ts2 vs2 vd2 0

2 2 q q
1 1 E dS or E 2 rl
20 15 400 225 0 0
2 2
1 1 400 225 q q /l
E
15 20 2 0 rl 2 0r
M A : M B 25 : 7 2 2
25. (d) Here, length of magnet = 10 cm = 10 × 10–2 m, 2 0r 2 4 0r
r = 15 × 10–2 m
P 1 1
15 9 109 2
3 18 10 2
cm cm
15 = 0.33 × 1011 NC–1
27. (c) Potential at P due to (+q) charge of dipole
1 q
SA BN V1
4 0 (z a)
O
5 cm 5 cm Potential at P due to (–q) charge of dipole
1 q
OP 225 25 200 cm V2
4 0 ( z a)
Since, at the neutral point, magnetic field
due to the magnet is equal to BH, y

0 M –q
BH 2a q P
4 (OP 2 AO 2 )3/ 2 Z
A B (0,0,z)
4 7 M (0,0,–a) (0,0,a)
0.4 10 10
(200 10 4
25 10 4 )3 / 2 x
4
Total potential at P due to electric dipole
0.4 10
7
(225 10 4 )3/ 2 M V V1 V2
10
1 q 1 q
0.4 103 10 6 (225)3/ 2 M
4 0 ( z a) 4 0 ( z a)
M 1.35 A-m
26. (a) Charge density or charge per unit length of q ( z a z a)
long wire 4 ( z a)( z a)
0
1 1
Cm and r = 18 × 10–2 m 2 qa
3 or V 2
4 0 (z a2 )
28. (c) Internal resistance of voltmeter = R.
Effective resistance across B and C
1 1 1 50 R
l
l R' R 50 50 R

or R ' 50 R
50 R
EBD_7443
www.jeeneetbooks.in

WWW.IIT-NEET.XYZ

2009-18 Target VITEEE

I 2.5 l
A I
A 10 11 A

50 k 2.5 11
or l 2.75 m
10
100V B 100 H 1 F 10
VL
R V 30. (c) I
50 k VR
~ VC
C
According to Ohm's law, V ' IR ' Impedance, Z ( X L ~ X C )2 R2
100 50R 2
or I 1
3 50 R or Z L~ R2
C
100 50 R Inductive reactance
or I ...(i)
3 50R XL L 70 103 100 10 6
7
Now, total resistance of circuit Capacitance reactance
50 R 1 1
R '' 50 XC
50 R C 70 103 1 10 6

(2500 100 R ) 1 102 100


or R ''
(50 R ) 7 10 2
7 7
Now, V '' IR '' As XC > XL
So, circuit behaves like R-C circuit.
100 50 R 2500 100 R 31. (c) At the centre of the loop, magnetic field
100
3 50 R (50 R ) I 2 R
0
B ...(i)
150 R 2500 100R or R 50k 4 R2
29. (c) Here length of potentiometer wire, l = 10 m l
Resistance of potentiometer wire
l 10
R or R B
A A
The value of 2.5 m length wire R
10 2.5 I
R' 2.5 R'
A 10 A 10 For the wire which is looped double let
radius becomes r
2.5
Potential, V ' I R' I l l
A 10 Then, 2 r or (r )
2 4
Again the length of potentiometer wire is
increased by 1 m. 0 I 2 r 2
B'
Resistance of null position 4 r2
l l
R '' I 2
11 A 0 2
or B ' 2
V '' IR '' and V V' 4 1
4
www.jeeneetbooks.in

WWW.IIT-NEET.XYZ

Solved Paper 2009 2009-19

2 = (3.972 – 3.31) × 10–19


or B ' 0 Il 16 ...(ii) = 0.662 × 10–19 J
2
4 l 19
or E 0.662 10
I l 1 0.41 eV 19
Now, B 0
R ...(iii) 1.6 10
4 2 2
l 34. (d) From Einstein's photoelectric equation
2
1 2 2( E W0 )
Dividing Eq. (ii) by (iii), E W0 mv or v
2 m
I l 16 2 A charged particle placed in uniform
0
B' 4 magnetic field experience a force
l2 4
2
B 0 Il 4 mv 2
F evB
4 l2 r
or B' = 4B mv
or r
32. (c) Magnetic field due to long wire at O point eB

2( E W0 )
R m
m 2m( E W0 )
or r r
I O eB eB
10 t t
35. (d) Here, N1 N0 e and N 2 N0 e
l0
B1 (upward) N1 1 1
2 R e e( 10 )t
e 9 t
N2 e
Magnetic field due to loop at O point
I 2 R 1
B2 0 t
9
4 R2
36. (c) Here current flows in circuit A as both (p-n)
I 0 junction diode act as forward biasing.
B2 (in upward direction)
2 R Total resistnace R.
Resultant magnetic field at centre O 1 1 1 2
B = B1 + B2 or R = 2
R 4 4 4
0I According to Ohm's law
B ( 1)T
2 R V I AR
33. (b) Work function W0 3.31 10 19 J or 8 = IA × 2 or IA = 4A
Wavelength of incident radiation In circuit B, lower p-n-junction diode is
reverse biased. Hence, no current will flow
5000 10 10 m but upper diode is forward biased so
According to Einstein's photoelectric current cna flow through it
equation E W0 KE V IB R

hc or 8 I B 4 or I B 2 A
19
3.31 10 KE 37. (c) After collision the bullet and block move
together and comes to rest after covering a
34
19 6.62 10 3 108 distance of 40 m.
KE 3.31 10 10
5000 10 250 ms
–1

6.62 3 19 19
m 0.23 kg
3.31 10 10
5 m = 0.2 kg
= (–3.31 × 1.324 × 3) × 10–19 u2 = 0
EBD_7443
www.jeeneetbooks.in

WWW.IIT-NEET.XYZ

2009-20 Target VITEEE


By conservation of momentum
ml 2
m1u1 m2 u2 m1v1 m2 v2 I about point A .
3
0.02 250 0.23 0 0.02v 0.23v By law of conservation of energy
5 0 v (0.25) or v 20 ms 1 l 1 1 ml 2 vB
2
2
Now, by conservation of energy mg I
2 2 2 3 l
1
or Mv 2 R d By solving, we get vB 3gl
2
40. (a) Given, r = 0.4 m,
1 = 8 rad s–1,
or 0.25 400 0.25 9.8 40
2 m = 4 kg, I = ?
200 Torque, I
0.51 mgr I
9.8 40
38. (a) Let after the time (t) the position of A is 16
(0, vAt) and position of B = (vBt, 10). Distance 4 10 0.4 I 8 or I 2 kg.m 2
8
between them
PART - II (CHEMISTRY)
y (0 vB t ) 2 (v At 10) 2
41. (c) Given : H f (H) = 218 kJ/mol
or y 2 (2t )2 (2t 10)2
1
or y 2 l 4t 2 4t 2 100 40t i.e., H2 H; H = 218 kJ/mol
2
l 8t 2 100 40t or H 2 2H; H = 436 kJ/mol
dl 436
Now, (16t 40) 0 104.3 kcal/mol
dt 4.18
or Hence, H–H bond energy is 104.3 kcal/mol.
40 d 2l 42. (a) In Wacker process,alkene is oxidised into
t 2.5 s 16 (+ve) aldehyde.
16 dt 2
So, l will be minimum. 1 PdCl2 CuCl2
CH 2 CH 2 O2 CH3CHO
39. (b) Here, potential energy of the metre stick 2 H 2O (B)
will be converted into rotational kinetic
energy. Since only alkenes produce aldehydes, on
ozonolysis hence 'A' must be an alkene.
B Now to find the structure of alkene we
should add two molecules of aldehyde and
replace O by double bond
CG H3C CH3
C=O+O=C
l/2 H ‘B’ ‘B’ H
H3C CH3
A Replacement of
C =C
v O by double bond H H
'A'
Because centre of gravity of stick lies at Therefore, alkyne must be
the middle of the rod, H2
CH3–C C– CH3
mgl Lindlar's catalyst
(alkyne)
PE of metre stick =
2 H3C CH3
C =C
1 2 H H
Rotational kinetic energy E I
2 'A'
www.jeeneetbooks.in

WWW.IIT-NEET.XYZ

Solved Paper 2009 2009-21

2 2
43. (b) 2F2 2NaOH 2NaF OF2 H2O K1 2 5
dilute ( A)
K2 1 3
The structure of 'A' (OF2) is as
O K1 : K 2 25 : 9
47. (c) Paramagnetic nature depends upon the
F 103.2 F
number of unpaired electrons. Higher the
bonds made by O = 2 number of unpaired electrons, higher the
Due to repulsion between two lone pairs of paramagnetic property will be.
electrons, its shape gets distorted. Cu2+ = [Ar] 3d9, no. of unpaired electrons = 1
Therefore, the bond angle in the molecule V2+ = [Ar] 3d3, no. of unpaired electrons = 3
is 103°. Cr2+ = [Ar] 3d4, no. of unpaired electrons = 4
44. (a) To decide the structure of alkene that Mn2+ = [Ar] 3d5, no. of unpaired electrons = 5
undergoes ozonolysis, add the products Hence, correct order is
and replace O by double (=) bond. Thus,
Cu 2 V2 Cr 2 Mn 2
H3C CH3 48. (a) 23
1 mol = 6.023 × 10 atoms
C=O+O=C KE of 1 mol = 6.023 × 104 J
H CH3 or KE of 6.023 × 1023 atoms = 6.023 × 104 J
acetaldehyde acetone
4 1 6.023 104
H3C 3 2 CH3 KE of 1 atom =
Replacement 6.023 1023
C= C
of O by double bond
H CH3 = 1.0 × 10–19 J
34
2- methyl-2-butene hc 6.626 10 3 108
hvenergy 9
45. (a) Structure of XeO3 600 10
= 3.313 × 10–19 J
Xe Now since Threshold energy
O O hv KE
O
3.313 10 19 1.0 10 19
3p -d pi bonds.
= 2.313 × 10–19 J
Structure of XeO4
Hence minimum amount of energy required
O to remove an electron from the metal ion
will be 2.313 × 10–19J.
Xe 49. (a) The earth's thermosphere also includes the
region of the atmosphere, called the
O O ionosphere. The ionosphere is the region
O of the atmosphere that is filled with charged
4p -d bonds. particles such as O 2 , O+, NO+. The high
46. (a) According to de-Broglie's equation.
temperature in the thermosphere can cause
h 2 h2 molecules to ionize.
mv m 2v 2 50. (b) The formula of sulphuric anhydride is SO3
and its structure is as follows :
2 h2 O
or mv 2
m
1 2 S
KE( K ) mv
2 O O
3 , 1p -p , 2 p -d bonds are present.
1 h2
KE ( K )
2m 2
EBD_7443
www.jeeneetbooks.in

WWW.IIT-NEET.XYZ

2009-22 Target VITEEE


CH3 CH3 57. (b) Average kinetic energy per molecule
+ – –
N2Cl Cl 3
kT
51. (a) Cu/HCl 2
+ N2
Gattermann 3 R 3 8.314
reaction or T 300
2 N0 2 6.023 10 23
H
P–H bonds = 6.21 × 10–21 JK–1 molecule–1
52. (c) H3PO3 H – O – P – O – H} 58. (c) The species having an O–O bond and O in
1
O an oxidation state of are super oxides
2
O and is represented as O2–. Usually these
are formed by active metals such as KO2,
H3PO2 P – OH RbO2 and CsO2. For the salts of larger
H
H anions (like O2 ), lattice energy increases
two P–H bonds in a group. Since, lattice energy is the
53. (c) Dipole moment, driving force for the formation of an ionic
compound and its stability, the stability of
( ) d the superoxides from 'K' to 'Cs' also
where, = magnitude of electric charge increases.
d = distance between particles (or bond 59. (a) 30% solution of H 2 O 2 is known as
length) perhydrol .
H2O2 decomposes as
d 2H 2 O2 2H 2 O O 2
d HI Volume strength of 30% H2O2 solution is
HCl HCl
or 100 i.e., 1 mL of th is solution on
HI d HCl HI decomposition gives 100 mL oxygen.
1.03 1.6 1
3.3 :1 SO2 O2 SO3
1.3 0.38 2
1L 1
L 1L
54. (d) SiCl4 4H 2 O H 4SiO4 4HCl 2

2L 1L 2L
H 4SiO 4 SiO2 2H 2 O
1000 C Since, 100 mL of oxygen is obtained by
55. (c) Mass percent of Cd in CdCl 2 = 1 mL of H2O2
0.9 1000 mL of oxygen will be obtained by
= 100 60% 1
1.5 1000 mL of H O = 10 mL of H O
100 2 2 2 2
Mass percent of Cl2 in CdCl2
= 100 – 60 = 40% dCHA
40% part (Cl2) has atomic weight 60. (d) Buffer capacity, ,
= 2 × 35.5 = 71.0 d pH
60% part (Cd) has atomic weight where, dCHA = no. of moles of acid added
per litre
71.0 60
106.5 dpH = change in pH
40
moles of acetic acid
56. (c) 2Al 2NaOH 2H 2 O 2NaAlO 2 3H 2 dCHA =
volume
sodium meta
aluminate
0.12/60 1
Sodiummetaaluminate, thus formed, is =
250/1000 125
soluble in water and changes into the
complex [Al(H2 O) 2 (OH) 4 ] – , in which 1/125 1
0.4
coordination number of Al is 6. 0.02 2.5
www.jeeneetbooks.in

WWW.IIT-NEET.XYZ

Solved Paper 2009 2009-23

61. (b) (A) Felspar (orthoclase) (KAlSi3O8) EtOH


(B) Asbestos {CaMg3(SiO3)4} KCN CH 2 CH 2 CN
|
(C) Pyrargyrite (Ruby silver) (Ag3SbS3) OH
(D) Diaspore (Al2O3 . H2O)
62. (c) Along a period first ionisation energy H 3O
CH 2 CH 2 COOH
increases. Thus, the first IE of the elements |
of the second period should follow the order OH
(C )
Be < B < N < O
But in practice. The first IE of these 66. (c) Tripeptides are amino acids polymers in
elements follows the order B < Be < O < N which three individual amino acid units,
The lower IE of B than that of Be is because called residues, are linked together by
in B (1s2, 2s2 2p1), electron is to be removed amide bonds.
from 2p which is easy while in Be (1s2, 2s2), glycine (NH2 –CH2 –COOH), alanine
electron is to be removed from 2s which is (CH3 CH COOH) and phenyl alanine
difficult. The low IE of O than that of N is |
because of the half-filled 2p orbitals in N NH2
(1s2, 2s2 2p3).
C6 H5 CH 2 CHCOOH can be linked
63. (c) CH 3 CH 2 OH Cl2 CH 3CHO |
2HCl NH2
X
Acetaldehyde in six different ways.
3Cl 2 67. (d) A codon is a specific sequence of three
3HCl
C Cl3 CHO adjacent bases on a strand of DNA or RNA
Y
chloral that provides genetic code information for
a particular amino acid.
CH3 COO 68. (c) The IUPAC name of dopamine is
64. (c) I. Ca CH3COCH3 2-(3,4-dihydroxyphenyl) ethylamine and its
–CaCO3
CH3CO O structure is as follows :
II. CH3 COOH 6HI CH2CH2NH2
Red P
CH3 CH3 3I 2 2H 2O
CH3CO OH P4O 10
CH3CO
III. O + H2O
CH3COO H CH3CO OH
OH
85.71 7.14
65. (b) C = 85.71% 7.14; 1 69. (a) Freezing point of a substance is the
12 7.14
temperature at which the solid and the liquid
14.29 14.29 forms of the substance are in equilibrium.
H = 14.29% = 14.29; 2
1 7.14 EtOH /
70. (b) C 2 H 5 Cl AgCN
Empirical formula = CH2 H2 O
and, empirical formula weight = 12 + 2 = 14 C2H 5 N C AgCl
Now since molecular formula weight
= 2 × vapour density (N-linked to ethyl carbon)
= 2 × 14 = 28 71. (a) For the given cell,
28 Ag | Ag+ | AgCl | Cl– | Cl2, Pt
n 2
14 The cell reactions are as follows
Molecular formula = (CH2)2 = C2H4 At anode : Ag Ag e
CH 2 CH 2 HOCl CH 2 CH 2 Cl
( A) | At cathode : AgCl e Ag( s ) Cl
OH
(B) Net cell reaction : AgCl Ag Cl
EBD_7443
www.jeeneetbooks.in

WWW.IIT-NEET.XYZ

2009-24 Target VITEEE

G oreaction G op G Ro 1
Again, [NO2 ] N2 O2
= (78 – 129) – (–109) 2
= + 58 kJ/mol
[N 2 ]1/2 [O2 ]
G o
nFE o K2
[NO2 ]
3 o
58 10 J 1 96500 Ecell
[N 2 ][O 2 ]2
or K 22 ...(ii)
58 1000
o [NO 2 ]2
Ecell 0.6 V
96500
72. (d) Crotonaldehyde is produced by the aldol Eqs. (i) × (ii), we get 100 K 22 1
condensation of acetaldehyde. 1 1
K 22 or K 2 0.1
H 100 10
H
Dil. NaOH 76. (c) For a first order reaction,
CH3 – C = O + H – C – CHO
acetaldehyde
(nucloeophilic 2.303 a
H addition) t log10
a x
acetaldehyde
H H Let initial amount of reactant is 100.

CH3 – C – C – CHO CH3CH = CHCHO 100


log
–H2O t1 100 75 [
(elimination) crotonaldehyde remains constant]
OH H t2 100
log
aldol 100 25

73. (b) BaCl2 2NaOH Ba(OH) 2 2NaCl 100


log
25 log 4
2 2 100
m Ba(OH) 2 m BaCl2 m NaOH m NaCl log 4 / 3
= 280 × 10–4 + 2 × 248 × 10–4 log
75
– 2 × 126 × 10–4
= (280 + 496 – 252) × 10–4 log 4
= 524 × 10–4 Sm2 mol–1 log 4 log 3
MZ 2 0.3010
74. (b) Density, d
N0 a3 2 0.3010 0.4771
where, Z = number of atoms in unit cell
0.6020
4.81
dN 0 a3 0.1249
Z
M
[ ] observed 1.2
23 10 3 77. (c) [ ] 39
8.92 6.023 10 (362 10 ) l C 6.15
5
63.55 1000
= 4.0 78. (a) Let the concentration of potassium acetate
Thus, metal has face centred unit cell. is x. According to Henderson's equation,
75. (c) N2 2O 2 2NO 2 [salt]
pH = pK a log
[acid]
[NO2 ]2
K1 x 50
[N 2 ][O2 ]2 4.8 log (1.8 10 5 ) log
20 0.1M
[NO2 ]2 4.8 4.74 log 25x
or 100 ..(i)
[N 2 ][O2 ]2
or log 25 x 0.06
www.jeeneetbooks.in

WWW.IIT-NEET.XYZ

Solved Paper 2009 2009-25

25x 1.148 2x
3/ 2
1/5
x 0.045M 84. (a) 1 (32 5x)
3
C 1/5
79. (b) By ' 2 A B ' , we get 3 2x 5
2 1 (32) 1/ 5 1 x
2 3 32
Na 2 O SO3 Na 2SO4 ; (neglecting higher powers of x)
259 1 1 5
H 2 146 418 [1 x ]2 1 x
2 5 32
H 580.5 581 kJ (neglecting higher powers of x)
80. (a) According to Hardy Schulze rule, greater 1 x
(1 x) 1
the valency of the coagulating ion, greater 2 32
is its coagulating power. Thus, out of the
(1 x)(32 x) 32 31x
given, AlCl3 (Al3 ) is most effective for
64 64
causing coagulation of As2S3 sol. (neglecting x2 term)
85. (c) ( x a)( x a 1) ( x a 1)( x a 2)
PART - III (MATHEMATICS)
( x a )( x a 2) 0
81. (b) Given, f ( x) x3 3 x 2 Let x – a = t, then
f '( x) 3 x2 3 t (t 1) (t 1)(t 2) t (t 2) 0
t 2 t t 2 3t 2 t 3 2t 0
Put f '( x) 0 3x 2 3 0
2
3t 6t 2 0
x2 1
f (x) is either increasing or decreasing. 6 36 24 6 2 3
t
At x = 2, f (2) = 23 + 3(2) – 2 = 12 2(3) 2(3)
At x = 3, f (3) = 33 + 3(3) – 2 = 34
f ( x ) [12, 34] . 3 3
x a
82. (c) The total number of subsets of given set is 3
29 = 512 3 3
Even numbers are {2, 4, 6, 8}. x a
3
Case I : When selecting only one even
Hence, x is real and distinct.
number = 4C1 = 4
Case II : When selecting only two even 86. (b) f ( x) x2 ax b has imaginary roots.
numbers = 4C2 = 6 Discriminant, D
Case III : When selecting only three even 0 a 2 4b 0
numbers = 4C3 = 4 f '( x ) 2 x a
Case IV : When selecting only four even f ''( x ) 2
numbers = 4C4 = 1
Required number of ways Also, f ( x ) f '( x) f ''( x) 0 ...(i)
= 512 – (4 + 6 + 4 + 1) – 1 = 496 x2 ax b 2 x a 2 0
[Here, we subtract 1 for due to the null set] 2
83. (a) The required number of ways = The even x (a 2) x b a 2 0
number of 0's i.e., {0, 2, 4, 6, …}
( a 2) ( a 2) 2 4( a b 2)
n! n! n! x
2
n ! 2!(n 2)! 4!( n 4)!
( a 2) a2 4b 4
n n n n 1
C0 C2 C4 ... 2 2
EBD_7443
www.jeeneetbooks.in

WWW.IIT-NEET.XYZ

2009-26 Target VITEEE


Since, a 2 4b 0 3x 2 3 y 2 3 x2 y2 1
a 2 4b 4 0 90. (c) (1 3i) n (1 3i) n
Hence, Eq. (i) has imaginary roots.
n n
87. (c) f ( x) 2 x4 13x 2 ax b is divisible by 1 3i 1 3i
2 2
2 2
( x 2)( x 1) .
2 n
f (2) 2(2)4 13(2) 2 a (2) b 0 ( 2 ) ( 2 )n

2a b 20 ...(i) ( 2)n [( 2 )3r 1 ( )3r 1 ]


( n = 3r + 1, where r is an integer)
and f (1) 2(1)4 13(1)2 a b 0
...(ii)
( 2)n ( 2
) ( 2)n
a b 11
On solving Eqs. (i) and (ii), we get 91. (d) Let f ( x) sin 4 x cos4 x
a = 9, b = 2
88. (d) Let a and R be the first term and common (sin 2 x cos2 x)2 2sin 2 x cos2 x
ratio of a GP respectively. 1
p 1
1 2(sin 2 x )2
So, Tp aR x 4
1 3 cos 4 x
Tq aR q 1
y and Tr aRr 1
z 1 (1 cos 4 x )
4 4 4
log x log a ( p 1) log R 2
Period of f ( x)
log y log a (q 1)log R 4 2
and log z log a (r 1) log R 92. (c) 2
sin x cos 2 x 2 sin 2 x
1 cos x (2cos 2 1)
2
2 2sin x cos x
log x p 1 log a ( p 1) log R p 1
2
log y q 1 log a (q 1) log R q 1 3cos x 2sin x cos x 0
log z r 1 log a (r 1) log R r 1 cos x (2sin x 3cos x) 0
cos x 0, ( 2sin x 3cos x 0)
log a p 1 ( p 1) log R p 1
log a q 1 (q 1) log R q 1 x 2n
2
log a r 1 (r 1)log R r 1
x (4n 1) , n Z
1 p 1 p 1 p 1 1 2
log a 1 q 1 log R q 1 q 1 1 1 1 1 1 1 1
93. (d) cos 2sin 3cos
1 r 1 r 1 r 1 1 2 2 2

(C2 C2 C3 ) 4 tan 1 ( 1)
=0+0= 0 ( two columns are identical) 1 1 1 1
cos 2 3 cos
89. (c) Let z = x + iy 2 6 2
z 2i 4 tan 1(1)
Given : 1
2z i
3 4
( x)2 ( y 2) 2 3 3 4 4
1
2 2 3 43
(2 x ) (2 y 1)
3
3 4 12
x2 y2 4 4 y 4x2 4 y2 1 4 y
www.jeeneetbooks.in

WWW.IIT-NEET.XYZ

Solved Paper 2009 2009-27

94. (c) We know that, 2s = a + b + c


3 p2 2 p 1 0
(a b c)(b c a)(c a b)(a b c)
( p 1)(3 p 1) 0
4b2c2
2 s (2 s 2a )(2 s 2b)(2 s 2c ) 1
p ( p cannot be negative)
2 2 3
4b c
98. (d) Given lines are parallel.
s( s a) (s b)( s c)
4 15 5 10
bc bc d
42 32 5
A 2 A
4cos sin 2 2
sin A d 2 diameter of the circle
2 2
Radius of circle = 1
95. (c) l m n 0, l m n Area of circle = r2 = sq unit
and l 2 m2 n2 0 99. (c) x2 2 xy xy 2 y 2 0
( m n) 2 m2 n 2 0 ( x 2 y )( x y ) 0
2m2 2mn 0 x 2 y, x y ...(i)
Also, x + y + 1 = 0 ...(ii)
2m(m n) 0
On solving Eqs. (i) and (ii), we get
m 0 or m n 0
2 1 1 1
If m = 0, then l = –n A , ,B , , C (0, 0)
3 3 2 2
l1 m1 n
1 0 1 2 1
and if m + n = 0 m n , then l = 0
1
3 3
l2 m2 n2 1 1 1
0 1 1 Area of ABC 1
2 2 2
ie., (l1 , m1 , n1 ) ( 1, 0, 1)
0 0 1
and (l2 , m2 , n2 ) (0, 1, 1)
0 0 1 1 1 1 1 1 1 1
cos 2 3 6 2 6 12
1 0 1 0 1 1 2
100. (c) Given pair of lines are
x2 3xy 2 y 2 0
3
and x2 3 xy 2 y 2 x 2 0
96. (a) m1 | a1 | 22 ( 1)2 (1)2 6
( x 2 y )( x y) 0
m2 | a2 | 32 ( 4)2 ( 4) 2 41 and ( x 2 y 2)( x y 1) 0
m3 | a3 | 12 12 ( 1) 2 3 x 2y 0, x y 0
and x 2 y 2 0, x y 1 0
and m4 | a4 | ( 1)2 (3)2 (1)2 11
The lines x 2 y 0, x 2 y 2 0 and
m3 m1 m4 m2
x y 0, x y 1 0 are parallel.
97. (a) Here, n = 6
Also, angle between x – 2y = 0 and x – y = 0
According to the question
is not 90°.
6
C2 p 2 q 4 4 6C4 p 4 q 2 It is a parallelogram.
q2 4 p2 (1 p)2 4 p2
EBD_7443
www.jeeneetbooks.in

WWW.IIT-NEET.XYZ

2009-28 Target VITEEE


101. (a) In OAC, OC2 = 22 + 42 = 20 y
2
y = 4x

x' x
4
C(±2, ±4) (1, 0)
4
A
O 2 32
y'
Also, point (1, 0) is the focus of the
Required equation of circle is parabola.
( x 2)2 ( y 4)2 20 It is clear from the graph that only one
normal is possible.
x2 y 2 4 x 8 y 0 105. (a) x2 y 2 c4
102. (d) Given circles are
2 2 y 2 (a2 y2 ) c4
x y 2 x 8 y 13 0
2 2
y 4 a 2 y 2 c4 0
and x y 4 x 6 y 11 0 Let y1, y2, y3 and y4 are the roots.
Here C1 (1, 4), C2 (2, 3) y1 y2 y3 y4 0
r1 1 16 13 2 106. (b) Solving 4 x 3 y 5 and 2 x2 3 y 2 12
2
and r2 4 9 11 2 5 3y
2 3y2 12
2 2
4
So, d C1C2 (2 1) ( 3 4) 2
(25 9 y 2 30 y )
d2 r12 r22 2 4 2 1 3y2 12
cos 8
2r1r2 2 2 2 2
15 y 2 30 y 71 0
135
103. (b) Let the required equation of circle be 30 900 4260 3360
y 1
2
x y 2 gx 2 fy 02 30 30
The above circle cuts the given circles 2
2 4x 5
orthogonally. Also, 2 x 3 12
3
8
2( 3 g ) 2 f (0) 8 2g 10 x 2 40 x 61 0
3
and 2 g 2 f 7 40 1600 4 10 61
x
2 10
8 29
2f 7 40 840 840
3 3 2
Required equation of circle is 20 20
8 29 840 3360
x2 y2 x y 0 Points are A 2 ,1
3 3 20 30

or 3 x2 3 y 2 8 x 29 y 0 840 3360
and B 2 ,1 .
104. (b) Given curve is y2 = 4x. 20 30
Mid point of AB is (2, 1).
www.jeeneetbooks.in

WWW.IIT-NEET.XYZ

Solved Paper 2009 2009-29

107. (d) Let A = (1, 0, 0), B = (0, 1, 0) and C = (0, 0, 1) 2 2


2 2 2
lim 0
So, AB (0 1) (1 0) 0 2 x 0 2(1 0)

Since, f (x) is contineous at x = 0


BC 02 (0 1)2 (1 0)2 2 f (0) = lim f ( x) a 0
x 0
and CA (1 0)2 02 (0 1) 2 2
Perimeter of triangle 1 1
112. (c) We have, x cos
= AB + BC + CA 2 2 2 3 2 1 t2
108. (c) cos 2 cos 2 cos 2 1 t
2 2 2 and y sin
+ sin sin sin 1 t2
(cos2 sin 2 ) (cos2 sin 2 ) x tan 1 t and y tan 1 t
(cos2 sin 2 ) sin 2 sin 2 sin 2 dy
y x 1
2 2 2 dx
cos cos cos 1
109. (a) Given equation of sphere is d x 1 1
113. (b) a tan 1 x b log 4
x 2
y 2
z 2
12 x 4 y 3z 0 dx x 1 x 1
On integrating both sides, we get
3
Centre of sphere is 6, 2, . 1 x 1
2 a tan x b log
x 1
2
3 1 1 1
Radius of sphere (6)2 (2)2 dx
2 2 x 2
1 x 2
1
9 169 13 1 x 1
36 4 a tan x b log
4 4 2 x 1

x 5
x 3
3
x 3 1 x 1 1 1
lim lim 1 log tan x
110. (c) 4 x 1 2
x x 2 x x 2
1 1
3( x 3) a ,b
x 2 x 2 2 4
3 3
lim 1 1 1
x x 2 a 2b 2 1
2 4
1
3
1 114. (c) y e a sin x
lim 3 x
x 1
2 On differentiating w.r.t. x, we get
e x
e3 1 1
y1 e a sin x
a
2 sin x sin 2 x 1 x2
, if x 0
111. (d) f ( x) 2 x cos x
y1 1 x 2 ay
a, if x 0
2
(1 x ) y12
a2 y2
2sin x sin 2 x 0
lim f ( x) lim form Again differentiating w.r.t. x, we get
x 0 x 0 2 x cos x 0
(1 x 2 )2 y1 y2 2 xy12 a 2 2 yy1
2cos x 2cos 2 x
lim (1 x 2 ) y2 xy1 a 2 y 0
x 0 2(cos x x sin x )
EBD_7443
www.jeeneetbooks.in

WWW.IIT-NEET.XYZ

2009-30 Target VITEEE


Using Leibnitz's rule,
Y
(1 x 2 ) yn n
C1 yn 1 ( 2 x) n
C2 yn ( 2) 118. (d) y = cos x
2
n
xyn 1 C1 yn a 2 yn 0 y = sin x
(1 x 2 ) yn 2 xyn 1 ( 2n 1) X' X
O /4 /2
2
yn [ n(n 1) n a ] 0
(1 x 2 ) yn 2 (2n 1) xyn 1 (n2 a 2 ) yn
115. (c) Given, f ( x ) x 3 ax 2 bx c, a 2 3b Y'
f '( x) 3x 2 2ax b /4
Area, A1 sin x dx
Put f '( x) 0
0
3x 2 2ax b 0 1
[cos x]0 / 4 1
2a 4a 2
12b 2
x
2 3 2 1
2a 2 a 2 3b 2
/2
3
and area, A2 cos x dx
Since, a 2 3b , /4
x has an imaginary value.
Hence, no extreme value of x exists. /2 1 2 1
[sin x] /4 1
2 sin 2 x x 2 2
116. (a) Let I e dx
1 cos 2 x 2 1 2 1
A1 : A2 : 1:1
2 2sin x cos x x 2 2
e dx
2sin 2 x dy
119. (b) sin ( x y) tan ( x y ) 1
dx
cosec 2 xe x dx cot x e x dx Put x + y = z
cot xe x ( cot x ) e x dx dy dz
1
dx dx
cot x e x dx c
dz
1 sin z tan z 1
cot xe x c dx
117. (c) We know that, if cos z
n dz dx
In sin x dx, then sin 2 z
Putting sin z t cos z dz dt , we have
sin n 1 x cos x n 1
In In 2 1 1
n n dt x c x c
where n is a positive integer. t2 t
cosec z x c x cosec( x y) c
nI n (n 1) I n 2 sin n 1 x cos x
120. (c) p (~ p q) is false means p is true and
~ p q is false.
p is true and both ~p and q are false.
p is true and q is false.
www.jeeneetbooks.in

WWW.IIT-NEET.XYZ

VITEEE
SOLVED PAPER 2008
PART - I (PHYSICS) 6. An electron microscope is used to probe the
atomic arrangements to a resolution of 5Å. What
should be the electric potential to which the
1. Two beams of light will not give rise to an
electrons need to be accelerated ?
interference patern, if
(a) 2.5 V (b) 5 V
(a) they are coherent
(c) 2.5 kV (d) 5 kV
(b) they have the same wavelength
7. Which phenomenon best supports the theory
(c) they are linearly polarized perpendicular to
that matter has a wave nature ?
each other
(a) Electron momentum
(d) they are not monochromatic (b) Electron diffraction
2. A slit of width 'a' is illuminated with a (c) Photon momentum
monochromatic light of wavelength from a (d) Photon diffraction
distant source and the diffraction pattern is 8. The radioactivity of a certain material drops to
observed on a screen placed at a distance 'D'
from the slit. To increase the width of the central 1
of the initial value in 2 hours. The half life of
maximum one should 16
(a) decrease D this radionuclide is
(b) decrease a (a) 10 min (b) 20 min
(c) decrease (c) 30 min (d) 40 min
(d) the width cannot be changed 9. An observer 'A' sees an asteroid with a
3. A thin film of soap solution (n = 1.4) lies on the radioactive element moving by at a speed = 0.3 c
top of a glass plate (n = 1.5). When visible light and measures the radioactivity decay time to be
TA. Another observer 'B' is moving with the
is incident almost normal to the plate, two
asteroid and measures its decay time as TB. Then
adjacent reflection maxima are observed at two
TA and TB are related as below
wavelengths 420 and 630 nm. The minimum
(a) TB < TA
thickness of the soap solution is
(b) TA = TB
(a) 420 nm (b) 450 nm (c) TB > TA
(c) 630 nm (d) 1260 nm (d) Either (A) or (C) depending on whether the
4. If the speed of a wave doubles as it passes from asteroid is approaching or moving away
shallow water into deeper water, its wavelength from A
will be 10. 234U has 92 protons and 234 nucleons total in its
(a) unchanged (b) halved nucleus. It decays by emitting an alpha particle.
(c) doubled (d) quadrupled After the decay it becomes
5. A light whose frequency is equal to 6 × 1014 Hz (a) 232U (b) 232Pa
is incident on a metal whose work function is (c) 230 Th (d) 230Ra
34 19 11. K and K x-rays are emitted when there is a
2eV. h 6.63 10 Js 1eV 1.6 10 J transition of electron between the levels
(a) n = 2 to n = 1 and n = 3 to n = 1 respectively
The maximum energy of the electrons emitted
(b) n = 2 to n = 1 and n = 3 to n = 2 respectively
will be
(c) n = 3 to n = 2 and n = 4 to n = 2 respectively
(a) 2.49 eV (b) 4.49 eV
(d) n = 3 to n = 2 and n = 4 to n = 3 respectively
(c) 0.49 eV (d) 5.49 eV
EBD_7443
www.jeeneetbooks.in

WWW.IIT-NEET.XYZ

2008-2 Target VITEEE


12. A certain radioactive material ZXA starts emitting 18. The reverse saturation of p-n diode
and particles successively such that the end (a) depends on doping concentrations
product is Z 3 Y A 8 . The number of and (b) depends on diffusion lengths of carriers
particles emitted are (c) depends on the doping concentrations and
(a) 4 and 3 respectively diffusion lengths
(b) 2 and 1 respectively (d) depends on the doping concentrations,
(c) 3 and 4 respectively diffusion length and device temperature
(d) 3 and 8 respectively 19. A radio station has two channels. One is AM at
13. 10k 1020 kHz and the other FM at 89.5 MHz. For
good results you will use
1k
– (a) longer antenna for the AM channel and
+1 V A Vout shorter for the FM
+
(b) shorter antenna for the AM channel and
longer for the FM
In the circuit shown above, an input of 1V is fed
(c) same length antenna will work for both
into the inverting input of an ideal Op-amp A.
The output signal Vout will be (d) information given is not enough to say
(a) +10 V (b) –10 V which one to use for which
(c) 0 V (d) infinity 20. The communication using optical fibers is based
14. When a solid with a band gap has a donor level on the principle of
just below its empty energy band, the solid is (a) total internal reflection
(a) an insulator (b) Brewster angle
(b) a conductor
(c) polarization
(c) a p-type semiconductor
(d) an n-type semiconductor (d) resonance
15. A p-n junction has acceptor impurity 21. In nature, the electric charge of any system is
concentration of 1017 cm–3 in the p-side and always equal to
donor impurity concentration of 1016 cm–3 in the (a) half integral multiple of the least amount of
n-side. What is the contact potential at the charge
junction (kT = thermal energy, intrinsic carrier
(b) zero
concentration ni = 1.4×1010 cm–3) ?
(a) (kT/e) ln (4×1012) (c) square of the least amount of charge
(b) (kT/e) ln (2.5×1023) (d) integral multiple of the least amount of
(c) (kT/e) ln (1023) charge
(d) (kT/e) ln (109) 22. The energy stored in the capacitor as shown in
16. A Zener diode has a contact potential of 1V in Fig. (a) is 4.5×10–6 J. If the battery is replaced by
the absence of biasing. It undergoes Zener another capacitor of 900 pF as shown in Fig. (b),
breakdown for an electric field of 106 V/m at the
then the total energy of system is
depletion region of p-n junction. If the width of
the depletion region is 2.5 m, what should be
the reverse biased potential for the Zener + – + –
+ – + –
breakdown to occur ? + – + –
+ – + –
(a) 3.5 V (b) 2.5 V + – 900pF 900pF
(c) 1.5 V (d) 0.5 V + ––
+ –
17. In Colpitt oscillator the feedback network +– + –
100V + 900pF
consists of
(a) two inductors and a capacitor Fig. (a) Fig. (b)
(b) two capacitors and an inductor
(c) three pairs of RC circuit (a) 4.5 × 10–6 J (b) 2.25 × 10–6 J
(d) three pairs of RL circuit (c) zero (d) 9×10–6 J
www.jeeneetbooks.in

WWW.IIT-NEET.XYZ

Solved Paper 2008 2008-3

23. Equal amounts of a metal are converted into 28. Three resistances of 4 each are connected
cylindrical wires of different lengths (L) and
as shown in figure. If the point D divides the
cross-sectional area (A). The wire with the
resistance into two equal halves, the resistance
maximum resistance is the one, which has
(a) length = L and area = A between point A and D will be

L A
(b) length = and area = 2A
2
A 4 4
(c) length = 2L and area =
2
(d) all have the same resistance, as the amount
D
of the metal is the same
24. If the force exerted by an electric dipole on a B 4 C
charge q at a distance of 1m is F, the force at a
point 2m away in the same direction will be (a) 12 (b) 6
F F 1
(a) (b) (c) 3 (d)
2 4 3
F F 29. The resistance of a metal increases with
(c) (d) increasing temperature because
6 8
(a) the collisions of the conducting electrons
25. A solid sphere of radius R1 and volume charge
with the electrons increase
0 (b) the collisions of the conducting electrons
density is enclosed by a hollow sphere
r with the lattice consisting of the ions of the
of radius R2 with negative surface charge density metal increase
, such that the total charge in the system is (c) the number of conduction electrons
zero. 0 is a positive constant and r is the decreases
distance from the centre of the sphere. The ratio (d) the number of conduction electrons
R2 increases
30. In the absence of applied potential, the electric
R1 is
current flowing through a metallic wire is zero
because
(a) (b) 2 / 0 (a) the electrons remain stationary
0 (b) the electrons are drifted in random direction
0 with a speed of the order of 10–2 cm/s
(c) 0 / 2 (d)
26. A solid spherical conductor of radius R has a (c) the electrons move in random direction with
spherical cavity of radius a (a < R) at its centre. A a speed of the order close to that of velocity
charge + Q is kept at the centre. The charge at of light
the inner surface, outer surface and at a position (d) electrons and ions move in opposite
r (a < r < R) are respectively direction
(a) + Q, – Q, 0 (b) – Q, + Q, 0 31. A meter bridge is used to determine the resistance
(c) 0, – Q, 0 (d) + Q, 0, 0 of an unknown wire by measuring the balance
27. A cylindrical capacitor has charge Q and length point length l. If the wire is replaced by another
L. If both the charge and length of the capacitor wire of same material but with double the length
are doubled, by keeping other parameters fixed, and half the thickness, the balancing point is
the energy stored in the capacitor expected to be
(a) remains same
1 1
(b) increases two times (a) (b)
(c) decreases two times 8 4
(d) increases four times (c) 8 (d) 16
EBD_7443
www.jeeneetbooks.in

WWW.IIT-NEET.XYZ

2008-4 Target VITEEE


32. Identify the INCORRECT statement regarding a (a) both bulbs will glow alternatively
superconducting wire (b) both bulbs will glow with same brightness
(a) transport current flows through its surface
1
(b) transport current flows through the entire provided frequency f
area of cross-section of the wire 2 1/ LC
(c) it exhibits zero electrical resistivity and
(c) bulb b1 will light up initially and goes off,
expels applied magnetic field
bulb b2 will be ON constantly
(d) it is used to produce large magnetic field
(d) bulb b1 will blink and bulb b2 will be ON
33. A sample of HCl gas is placed in an electric field
constantly
3×104 NC–1. The dipole moment of each HCl
molecule is 6×10–30cm. The maximum torque 37. A transformer rated at 10 kW is used to connect
that can act on a molecule is a 5kV transmission line to a 240V circuit. The
(a) 2 × 10–34 C2Nm–1 (b) 2 × 10–34 Nm ratio of turns in the windings of the transformer
(c) 18 × 10–26 Nm (d) 0.5×1034 C–2 Nm–1 is
34. When a metallic plate swings between the poles (a) 5 (b) 20.8
of a magnet (c) 104 (d) 40
(a) no effect on the plate 38. Three solenoid coils of same dimension, same
(b) eddy currents are set up inside the plate number of turns and same number of layers of
and the direction of the current is along the winding are taken. Coil 1 with inductance L1 was
motion of the plate wound using a Mn wire of resistance 11 /m;
(c) eddy currents are set up inside the plate Coil 2 with inductance L2 was wound using the
and the direction of the current oppose the similar wire but the direction of winding was
motion of the plate reversed in each layer; Coil 3 with inductance L3
(d) eddy currents are set up inside the plate was wound using a superconducting wire. The
35. When an electrical appliance is switched on, it self inductance of the coils L1, L2, L3 are
responds almost immediately, because
(a) L1 = L2 = L3 (b) L1 = L2; L3 = 0
(a) the electrons in the connecting wires move
(c) L1 = L3; L2 = 0 (d) L1 > L2 > L3
with the speed of light
39. Light travels with a speed of 2 × 108 m/s in crown
(b) the electrical signal is carried by
glass of refractive index 1.5. What is the speed
electromagnetic waves moving with the
speed of light of light in dense flint glass of refractive index
1.8 ?
(c) the electrons move with the speed which is
close to but less than speed of light (a) 1.33 × 108 m/s (b) 1.67 × 108 m/s
8
(c) 2.0 × 10 m/s (d) 3.0 ×108 m/s
(d) the electrons are stagnant.
36. Two identical incandescent light bulbs are 40. A parallel beam of fast moving electrons is
connected as shown in the Figure. When the incident normally on a narrow slit. A screen is
circuit is an AC voltage source of frequency f, placed at a large distance from the slit. If the
which of the following observations will be speed of the electrons is increased, which of the
correct ? following statement is correct ?
(a) diffraction pattern is not observed on the
R R screen in the case of electrons
(b) the angular width of the central maximum
C of the diffraction pattern will increase
L
Bulb b1 (c) the angular width of the central maximum
Bulb b2 will decrease
(d) the angular width of the central maximum
will remain the same
www.jeeneetbooks.in

WWW.IIT-NEET.XYZ

Solved Paper 2008 2008-5

50. Which of the following is ketohexose ?


PART - II (CHEMISTRY) (a) Glucose (b) Sucrose
(c) Fructose (d) Ribose
41. 675 K 51. The oxidation number of oxygen in KO3, Na2O2 is
CH 3CH 3 HNO 3
(a) 3, 2 (b) 1, 0
(a) CH3CH2NO2 (c) 0, 1 (d) –0.33, –1
(b) CH3CH2NO2 + CH3NO2 52. Reaction of PCl3 and PhMgBr would give
(c) 2CH3NO2 (a) bromobenzene
(d) CH2 = CH2 (b) chlorobenzene
42. When acetamide is hydrolysed by boiling with (c) triphenylphosphine
acid, the product obtained is : (d) dichlorobenzene
(a) acetic acid (b) ethyl amine 53. Which of the following is not a characteristic of
(c) ethanol (d) acetamide transition elements ?
43. Which will not go for diazotization ? (a) Variable oxidation states
(a) C6H5NH2 (b) C6H5CH2NH2 (b) Formation of colored compounds
(c) H 2N (d) H2N (c) Formation of interstitial compounds
C 6H 4 C6H4 (d) Natural radioactivity
H 3C O2N 54. Cl - P - Cl bond angles in PCl5 molecule are
44. Secondary nitroalkanes can be converted into (a) 120° and 90° (b) 60° and 90°
ketones by using Y. Identify the Y from the (c) 60°and 120° (d) 120° and 30°
following 55. The magnetic moment of a salt containing Zn 2+
R R ion is
CHNO2 + Y C=O (a) 0 (b) 1.87
R R (c) 5.92 (d) 2
(a) Aqueous HCl (b) Aqueous NaOH 56. The number of formula units of calcium flouride
(c) KMnO4 (d) CO CaF2 present in 146.4 g of CaF2 are (molar mass
45. Alkyl cyanides undergo Stephen reduction to of CaF2 is 78.08 g/mol)
produce
(a) aldehyde (b) secondary amine (a) 1.129 1024 CaF2 (b) 1.146 1024 CaF2
(c) primary amine (d) amide
46. The continuous phase contains the dispersed
(c) 7.808 1024 CaF2 (d) 1.877 1024 CaF2
phase throughout, Example is 57. The IUPAC name of the given compound
(a) Water in milk
Co NH3 5
Cl Cl 2 is
(b) Fat in milk
(c) Water droplets in mist (a) pentaamino cobalt chloride chlorate
(d) Oil in water (b) cobalt pentaamine chloro chloride
47. The number of hydrogen atoms present in 25.6 (c) pentaamine chloro cobalt (III) chloride
g of sucrose (C12H22O11) which has a molar mass (d) pentaamino cobalt (III) chlorate
of 342.3 g is 58. When SCN– is added to an aqueous solution
(a) 22 × 1023 (b) 9.91 × 1023 containing Fe(NO3)3,the complex ion produced is
(c) 11 × 10 23 (d) 44 × 1023 2
48. Milk changes after digestion into : (a) Fe OH 2 2
SCN –
(a) cellulose (b) fructose
(c) glucose (d) lactose 2
(b) Fe OH 2 SCN –
49. Which of the following sets consists only of 5
essential amino acids ?
2
(a) Alanine, tyrosine, cystine (c) Fe OH 2 8
SCN –
(b) Leucine, lysine, tryptophane
(c) Alanine, glutamine, lycine 6
(d) Leucine, proline, glycine (d) Fe OH 2 SCN
EBD_7443
www.jeeneetbooks.in

WWW.IIT-NEET.XYZ

2008-6 Target VITEEE


59. Hair dyes contain 68. The electrochemical cell stops working after
(a) copper nitrate (b) gold chloride sometime because :
(c) silver nitrate (d) copper sulphate (a) electrode potential of both the electrodes
60. Schottky defects occurs mainly in electrovalent becomes zero
compounds where (b) electrode potential of both the electrodes
(a) positive ions and negative ions are of becomes equal
different size (c) one of the electrodes is eaten away
(b) positive ions and negative ions are of same (d) the cell reaction gets reversed
size 69. The amount of electricity required to produce
(c) positive ions are small and negative ions one mole of copper from copper sulphate
are big solution will be
(d) positive ions are big and negative ions are (a) 1 Faraday (b) 2.33 Faraday
small (c) 2 Faraday (d) 1.33 Faraday
61. The number of unpaired electrons calculated in 70. Dipping iron article into a strongly alkaline
3 3 solution of sodium phosphate
Co NH3 6
and Co F6 are (a) does not affect the article
(a) 4 and 4 (b) 0 and 2 (b) forms Fe2O3. xH2O on the surface
(c) 2 and 4 (d) 0 and 4 (c) forms iron phosphate film
62. The standard free energy change of a reaction (d) forms ferric hydroxide
71. Hydroboration oxidation of 4-methyl-octene
is G 115kJ at 298 K. Calculate the would give
equilibrium constant k p in log k p (a) 4-methyl octanol
(b) 2-methyl decane
R 8.314 Jk 1 mol 1 .
(c) 4-methyl heptanol
(a) 20.16 (b) 2.303 (d) 4-methyl-2-octanone
(c) 2.016 (d) 13.83 72. When ethyl alcohol is heated with conc. H2SO4,
63. If an endothermic reaction occurs spontaneously the product obtained is :
at constant temperature T and P, then which of (a) CH3COOC2H5 (b) C2H2
the following is true ? (c) C2H6 (d) C2H4
(a) G>0 (b) H<0 73. Anisole is the product obtained from phenol by
(c) S>0 (d) S<0 the reaction known as
64. If a plot of log10C versus t gives a straight line (a) coupling (b) etherification
for a given reaction, then the reaction is (c) oxidation (d) esterification
(a) zero order (b) first order 74. Ethylene glycol gives oxalic acid on oxidation
(c) second order (d) third order with
65. A spontaneous process is one in which the
(a) acidified K2Cr2O7
system suffers :
(b) acidified KMnO4
(a) no energy change
(c) alkaline KMnO4
(b) a lowering of free energy
(d) periodic acid
(c) a lowering of entropy
75. Diamond is hard because
(d) an increase in internal energy
(a) all the four valence electrons are bonded to
66. The half life period of a first order reaction is 1
each carbon atoms by covalent bonds
min 40 secs. Calculate its rate constant.
(b) it is a giant molecule
(a) 6.93 × 10–3 min–1 (b) 6.93 × 10–3 sec–1
(c) it is made up of carbon atoms
(c) 6.93 × 10–3 sec (d) 6.93 × 103 sec
67. The molar conductivities of KCl, NaCl and KNO3 (d) it cannot be burnt
are 152, 128 and 111 S cm2 mol–1 respectively. 76. A wittig reaction with an aldehyde gives
What is the molar conductivity of NaNO3? (a) ketone compound
(a) 101 S cm2 mol–1 (b) 87 S cm2 mol–1 (b) a long chain fatty acid
(c) –101 S cm2 mol–1 (d) –391 S cm2 mol–1 (c) olefin compound
(d) epoxide
www.jeeneetbooks.in

WWW.IIT-NEET.XYZ

Solved Paper 2008 2008-7

77. Cannizaro reaction is given by


1 3i
(a) HCHO (b) CH3COCH3 85. Argument of the complex number is
(c) CH3CHO (d) CH3CH2OH 2 i
(a) 45° (b) 135°
CHO
(c) 225° (d) 240°
AlCl3 86. In a triangle ABC, the sides b and c are the roots
78. + ? of the equation x2 – 61x + 820 = 0 and
HCl
4
Identify the reactant A = tan–1 , then a2 is equal to
(a) H2O (b) HCHO 3
(c) CO (d) CH3CHO (a) 1098 (b) 1096
79. Maleic acid and Fumaric acids are (c) 1097 (d) 1095
(a) Position Isomers (b) Geometric Isomers 87. The shortest distance between the straight lines
(c) Enentiomers (d) Functional Isomers through the points A1 = (6, 2, 2) and
80. The gas evolved on heating alkali formate with A2 = (–4, 0, –1), in the directions of (1, –2, 2) and
soda-lime is (3, –2, –2) is
(a) CO (b) CO2 (a) 6 (b) 8
(c) Hydrogen (d) water vapor (c) 12 (d) 9
88. The center and radius of the sphere
PART - III (MATHEMATICS) x2 + y2 + z2 + 3x – 4z + 1 = 0 are
3 21
81. If a, b, c be three unit vectors such that (a) , 0, 2 ;
2 2
1
a (b c) b, b and c being non-parallel. If 3
2 (b) , 0, 2 ; 21
2
1 is the angle between a and b and 2 is the
angle between a and c , then 3 21
(c) , 0, 2 ;
2 2
(a) 1 , 2 (b) 1 , 2
6 3 3 6 3 21
(d) , 2, 0 ;
, , 2 2
(c) 1 2 (d) 1 2
2 3 3 2 89. Let A and B are two fixed points in a plane then
82. The equation r 2
2r.c h 0, c h, locus of another point C on the same plane such
that CA +CB = constant, (> AB) is
represents
(a) circle (b) ellipse (a) circle (b) ellipse
(c) cone (d) sphere (c) parabola (d) hyperbola
83. The simplified expression of sin (tan–1 x), for any 90. The directrix of the parabola y2 + 4x + 3 = 0 is
real number x is given by 4 1
(a) x 0 (b) x 0
1 x 3 4
(a) (b)
1 x2 1 x2 3 1
(c) x 0 (d) x 0
1 x 4 4
(c) (d) 91. If g (x) is a polynomial satisfying
1 x2 1 x2 g (x) g(y) = g(x) + g(y) + g(xy) – 2
z 25 for all real x and y and g (2) = 5 then Lt g(x) is
84. If 5 , the value of |z| x 3
z 1
(a) 3 (b) 4 (a) 9 (b) 10
(c) 5 (d) 6 (c) 25 (d) 20
EBD_7443
www.jeeneetbooks.in

WWW.IIT-NEET.XYZ

2008-8 Target VITEEE


97. Area enclosed by the curve
( ex 2x )
92. The value of f(0) so that may be 2
x 4 x 2 y2 8 is
continuous at x = 0 is
(a) (b) 2
1 (c) 3 (d) 4
(a) log (b) 0
2
a
(c) 4 (d) – 1 + log 2 a x
98. The value of dx is
93. Let [ ] denote the greatest integer function and x
0
f (x) = [tan2 x]. Then
a a
(a) lim f (x) does not exist (a) (b)
x 0 2 4
(b) f (x) is continuous at x = 0 a a
(c) f (x) is not differentiable at x = 0 (c) (d)
2 4
(d) f (x) = 1
99. Let y be the number of people in a village at time
94. A spherical balloon is expanding. If the radius is
t. Assume that the rate of change of the
increasing at the rate of 2 centimeters per minute,
population is proportional to the number of
the rate at which the volume increases (in cubic people in the village at any time and further
centimeters per minute) when the radius is 5 assume that the population never increases in
centimetres is time. Then the population of the village at any
(a) 10 (b) 100 fixed time t is given by
(c) 200 (d) 50 (a) y = ekt + c, for some constants c < 0 and k>0
95. The length of the parabola y2 = 12x cut off by (b) y = cekt, for some constants c > 0 and k < 0
the latus-rectum is (c) y = ect + k, for some constants c < 0 and k>0
(d) y = kect, for some constants c > 0 and k < 0
(a) 6 2 log(1 2)
100. The differential equation of all straight lines
touching the circle x2 + y2 = a2 is
(b) 3 2 log(1 2)
2 2
dy dy
6 2 log(1 2) (a) y a2 1
(c) dx dx

(d) 3 2 log(1 2) 2 2
dy dy
(b) y x a2 1
dx dx
x5
96. If I dx , then I is equal to
1 x3 dy dy
(c) y x a2 1
5 3
dx dx
2 2
(a) (1 x 3 ) 2 (1 x3 ) 2 C dy dy
9 3 (d) y a2 1
dx dx
(b) log x 1 x3 C
dy
101. The differential equation y 3 0
dx
(c) log x 1 x3 C admits
(a) infinite number of solutions
3 1 (b) no solution
2 2 (c) a unique solution
(d) (1 x 3 ) 2 (1 x3 ) 2 C
9 3 (d) many solutions
www.jeeneetbooks.in

WWW.IIT-NEET.XYZ

Solved Paper 2008 2008-9

102. Solution of the differential equation 109. Mean and standard deviation of marks obtained
in some particular subject by four classes are
xdy ydx x2 y2 dx 0 is given below. Report the class with best
performance
(a) y x2 y2 C x2
(a) 80, 18 (b) 75, 5
(c) 80, 21 (d) 76, 7
(b) y x2 y2 C x2
110. A random variable X follows binomial
(c) x x2 y2 C y2 distribution with mean and variance . Then
(a) 0 < < (b) 0 < <
(d) x x 2 y2 C y 2 (c) <0< (d) <0<
103. Let P, Q, R and S be statements and suppose 111. The system of equations
x + y+ z = 0
that P Q R P. if ~ S R , then
2x + 3y + z = 0
(a) S ~ Q (b) ~ Q S x + 2y = 0
(c) ~ S ~ Q (d) Q ~ S has
104. In how many number of ways can 10 students (a) a unique solution; x = 0, y = 0, z = 0
be divided into three teams, one containing four (b) infinite solutions
students and the other three? (c) no solution
(a) 400 (b) 700 (d) finite number of non-zero solutions
(c) 1050 (d) 2100
105. If R be a relation defined as a R b iff |a – b| > 0, 4
0 a
then the relation is 112. 1 , then
b 0
(a) reflexive (b) symmetric
(c) transitive (a) a = 1 = 2b (b) a = b
(d) symmetric and transitive (c) a = b2 (d) ab = 1
106. Let S be a finite set containing n elements. Then 113. If D = diag (d1, d2, ..., dn) where d1 0, for
the total number of commutative binary i = 1, 2, ..., n, then D–1 is equal to
operation on S is
(a) DT
n(n 1 n(n 1 (b) D
(a) 2 (b) 2
n n (c) Adj (D)
2 (d) diag (d1–1, d2–1, ...dn–1)
(c) n (n ) (d) 2(n 2 )
114. If x, y, z are different from zero and
107. A manufacturer of cotter pins knows that 5% of
his product is defective. He sells pins in boxes a b y c z
of 100 and guarantees that not more than one
a x b c z
pin will be defective in a box. In order to find the = 0 then the value of
probability that a box will fail to meet the a x b y c
guaranteed quality, the probability distribution
one has to employ is a b
c
(a) Binomial (b) Poisson the expression is
x y
z
(c) Normal (d) Exponential
108. The probability that a certain kind of component (a) 0 (b) –1
(c) 1 (d) 2
3 115. Probability of getting positive integral roots of
will survive a given shock test is . The
4 the equation x2 – n = 0 for the integer n, 1 < n < 40
probability that exactly 2 of the next 4 is
components tested survive is
1 1
9 25 (a) (b)
(a) (b) 5 10
41 128
3 1
1 27 (c) (d)
(c) (d) 20 20
5 128
EBD_7443
www.jeeneetbooks.in

WWW.IIT-NEET.XYZ

2008-10 Target VITEEE


116. The number of real roots of the equation 119. The plane through the point (–1, –1, –1) and
containing the line of intersection of the planes
x4 x 4 20 22 is
(a) 4 (b) 2 r.(iˆ 3jˆ k)
ˆ 0 and r.(ˆj 2k) ˆ 0 is
(c) 0 (d) 1 (a) r.(iˆ 2ˆj 3k)
ˆ 0
117. Let , be the roots of the equation
x2 – ax + b = 0 and An = n + n. (b) r.(iˆ 4jˆ k)
ˆ 0
Then An+1 – aAn + bAn–1 is equal to
(a) –a (b) b (c) r.(iˆ 5jˆ 5k)
ˆ 0
(c) 0 (d) a – b
118. If the sides of a right-angle triangle form an A.P., (d) r.(iˆ ˆj 3k)
ˆ 0
the 'Sin' of the acute angles are 120. a ˆi ˆj kˆ and b 2iˆ 4jˆ 3kˆ are one of the
3 4 sides and medians respectively, of a triangle
(a) ,
5 5 through the same vertex, then area of the triangle
is
1
(b) 3, 1
3 (a) 83 (b) 83
2

5 1 5 1 1
, (c) 85 (d) 86
(c) 2 2 2

3 1 3 1
(d) ,
2 2
www.jeeneetbooks.in

WWW.IIT-NEET.XYZ

Solved Paper 2008 2008-11

SOLUTIONS
PART - I (PHYSICS)
2
1. (d) Two beams of light give rise to an
interference pattern if they are coherent, they t 2
1
have same wavelength/frequency in same 1
phase or having a constant phase difference
and same state of polarisation. Putting, 1 = 420 nm, 2 = 630 nm, = 1.4
Interference pattern can not be obtained if We get, t = 450 nm
the two beams are not mono chromatic. 4. (c) We have,
2. (b) By the theory of diffraction at a single slit, speed = frequency × wavelength
the width of the central maximum is given by Also, when a wave passes from one medium
2D to the other, its frequency remains constant.
W W D, speed wavelength
a
Therefore, when the speed of a wave doubles,
1 its wavelength also doubles.
W and W 5. (c) Here,
a
Therefore, to increase the width of the central Frequency, = 6 ×1014 Hz
maximum a should be decreased. Work-function, = 2eV = 2 × 1.6 × 10–19 J
3. (b) R1 and R2 are the two rays considered for = 3.2 × 10–19J
interference. R1 is the result of reflection at Maximum energy, Tmax = ?
denser medium; hence it suffers an additional By Einstein's photo electric equation, we
have
path difference of . Ray R2 originates after h = + Tmax
2 Tmax = h –
reflection at narer medium. = (6.63 × 10–34 × 6 × 1014)– (3.2 × 10–19)
= (3.97 × 10–19) – (3.2 × 10–19)
Net path difference = 2 t+ , = 0. 77 × 10–19 J
2
where t is the thickness of the soap solution. 19
0.77 10
For constructive interference, = eV = 0.49 eV
19
1.6 10
2 t+ = m , where m = 0, 1, 2..... 6. (b) We have, d sin
2
Let the two adjacent reflection maxima be For = 90° and n = 1, we get d =
observed at m and m–1. Then
h h h2
1 But = =
2 t m 1 p 2meV 2meV
2
1 34 2
(6.63 10 )
or 2 t m 1 ... (1) =
2 31 19
2 9.1 10 1.6 10 V
3
Similarly, 2 t m 2 ...(2) 1.5
10 9
m
1.5
10 9
2 = d=
V V
Solving (1) & (2), we get
R1 R2 1.5 1.5
or 5 10 10
10 9 or 0.5
n=1 V V
Soap solution n = 1.4 1.5 1.5
Glass Plate or 0.5 0.5 V = 6V
T n = 1.5 V 0.5 0.5
EBD_7443
www.jeeneetbooks.in

WWW.IIT-NEET.XYZ

2008-12 Target VITEEE


No option is matching with the exact answer. 12. (b) Let thenumber of and particles emitted
But 5V is approximately equal to the exact be m and n respectively. Then
potential. Therefore, option (b) should be the A – 4m = A – 8 m = 2
correct option. [ the mass number of a radioactive nuclide
7. (b) Davison and Germer performed an experiment decreases by 4 due to emission of one -
to prove that matter has a wave nature. The particle]
experiment was based on electron diffraction.
Again,
t (Z – 2m) + n = Z – 3
1 T1/ 2 [ the atomic number decreases by 2 due to
8 (c) We have, Nt = N0 ,
2 emission of 1 -particle but increases by 1
When due to emission of 1 - particle]
Nt = number of atoms present after time t or –2m + n = –3
N0 = initial number of atoms or 2m – n = 3
T1/2 = half life of the nuclide or (2 × 2) –n = 3 ( m=2)
t 2 n=1
Nt 1 T1/ 2 1 1 T1/ 2
or 13. (b) Rf
N0 2 16 2
2
4 2
1 1 T1/ 2 4 RI
or T1/ 2
2 2 VI – Op – amp Vo
2 1
T1/2 = Hr = Hr T1/2 = 30 min. +
4 2
9. (c) Due to time dilation the interval between two
events at the same point in a moving frame
appears to be longer by a factor For the Op-amp shown above, we have
1 Vo Rf
2 VI RI
v to an observer in a stationary
1
c 2 Comparing this circuit with the given one,
frame. We get VI = 1V, Rf = 10k = 10 × 103
Time dilation is independent of the direction RI = 1k = 1 × 103
of velocity and depends only on its
magnitude. Vo 10 103
Vo = – 10 V
10. (c) The - decay in the case of 234U takes place 1 1 103
as follows : 14. (d) The solid is an n-type semiconductor. In an
234 230 Th 4 n-type semiconductor, the impurity is
92 U 90 2 He
pentavalent which is also called the Donar
11. (a) When electron jumps from the level n = 2 to impurity because one impurity atom generate
the level n = 1, K x - rays are emitted.
Similarly, K x-rays are emitted when there is one electron. The Donor energy level lies
atransition of electron from the level n = 3 to just below the conduction band as shown in
the level n = 1. X-ray spectra has been shown the figure.
below.
O n=5 Conduction band
N n=4
M M Donor levels
M n=3
Valence band
L L L
L n=2 15. (a) We have, ID = IS (e VD / VT 1)
K
K kT
K
K where, VT
e
K n=1
www.jeeneetbooks.in

WWW.IIT-NEET.XYZ

Solved Paper 2008 2008-13

V0
20. (a) The principle of communication using optical
ID VT ID VD fibers is based on the principle of total
1 e or ln 1 internal reflection.
IS IS nVT
ID ID
VD nVT ln 1 = VT ln 1 21. (d) According to quantization of charge, the
IS IS charge of any system is an integral multiple
I majority of the charge of electron which is the least
= VT ln amount of charge on any system.
IS 22. (b) The charge on the capacitor when connected
to the battery is given by
Here, n e (1017 1016 )cm 3
+ – + –
= 1016 (10 1)cm 3
9 1016 cm 3 +
+


+
+


+ – + –
We know that, ne nh ni2 900pF 900pF
n i2 (1.4 1010 )2
nh cm3 + –
ne 9 1016 + –
+
+


Imajority 100 V + –
ne 900pF
Also,
IS nh Q= CV = (900 × 10–12 F) × 100 V = 9 × 10–8 C
When the battery is replaced by another
9 1016 capacitor of 900 pF capacitance, the charge
kT of 9 × 10–8 C is distributed on both. Let Q1
VD ln (1.4 1010 ) 2
e and Q2 be the charge on each of them.
9 1016 Q = Q1 + Q2
= C1V + C2V, where V is the common potential.
kT Q
= ln(4 1012 )
e or V = C C
1 2
16. (c) Here, electric field, E = 106 V/m
width of depletion region, As the two capacitors are in parallel, the
d = 2.5 m = 2.5 × 10–6m equivalent capacitance is given by C = C1 + C2
Potential required for breakdown 1
(V) = Ed = (106 × 2.5 × 10–6) V = 2.5 V Total energy of the capacitors = CV 2
2
Contact Potential = 1.0 V
Reverse biased potential for zener 1 Q2 Q2
breakdown = (2.5 – 1.0) V = 1.5 V = C1 C2
2 =
17. (b) In a colpitt oscillator, the feed-back network 2 C1 C2 2(C1 C2 )
consists of two capacitors and one inductor. 2
18. (d) The reverse saturation of p-n diode depends 9 10 8
on the doping concentrations, diffusion =
length and device temperature. 2(900 10 12 900 10 12 )

19. (a) Antenna length = (9) 2 10 16


4 = = 2.25 × 10–6 J
10
1 2 2 9 10
for cosntant velocity,,
l
23. (c) We know that R =
1 A
Antenna length
L
For l = L and A = A; R1
Here, AM FM A
Antenna length for AM should be longer L
than that of FM. For l = and A = 2A;
2
EBD_7443
www.jeeneetbooks.in

WWW.IIT-NEET.XYZ

2008-14 Target VITEEE


Total charge on the whole solid sphere,
L
R1 R1
2 1 L 1 Qs dQ 4 0 0 rdr
R2 = = R1 0
2A 4 A 4
R12 2
A = 4 0 2 0 R1
For l = 2L and A = ; 2
2 2
QS = 2 0R1 ...(1)
2L L
R3 = 4 R1
A A Now, the total charge on the hollow sphere,
Qh = – (4 R22) ... (2)
2
By question, Qs + Qh = 0
Thus R3 > R1 > R2 2 0R12 = 4 R22
Therefore, the wire having length 2L and area
2
A R2 0 R2 0
has the maximum resistance. R1 2 R1 2
2
24. (d) We know that the electric field at any point 26. (b) A charge Q will be induced on the inner
due to an electric dipole varies inversely with surface of the solid spherical conductor. An
the cube of the distance of the point from equal but opposite charge will be induced
the centre of the dipole, that is, on the outer surface of the conductor. There
will be no charge at a position between the
qE inner and outer surface.
P q
1 +Q
E R
r3 r –Q
– + +Q a
Also, force on a charge (q) is given by

F = qE 1
F
r3 27. (b) The energy stored in a capacitor is given by
When the distance of the charge becomes Q2
2m, i.e. double of its initial value, then new E
force (F´) will become 2C
1 2 0 L
F´ = .F = F Here, C , where
(2) 3
8 R2
ln
25. (c) Let us first calculate the total charge on the R1
soild sphere. L = length of the cylindrical conductor
Let us consider a concentric sphere of radius R1 = inner radius
r and thickness dr. R2 = outer radius
Then volume of thesphere, dV = 4 r2dr
Given, the volume charge density of the Q2
C L E
0
L
sphere = When both Q and L are doubled, by keeping
r
other parameters fixed, the energy stored (E´)
R1 becomes
r dr
(2Q) 2

2L
Charge on this sphere, 2Q 2
L
dQ = .dV
0
.4 r 2 dr = 4 .rdr..
r E´ 2E
www.jeeneetbooks.in

WWW.IIT-NEET.XYZ

Solved Paper 2008 2008-15

28. (c) Redrawing the figure, we get R l


A S 100 l
A where R = unknown resistor
4 S = known resistor
4
l = balancing length
B C 6 6 100R
l
2 S R
When the resistance of the new wire is 8R
2

D D then the new balancing length will be


100 8R 100 8R
(a) (b) l
The 4 and 2 resistors are in series and S 8R (S R) 7R
the same is the case with another 4 and 2 100 8R 800l
resistors. So, the four resistors are equivalent = l' .
100R 100 7l
to following two resistors. 7R
Now, in fig (b) these two 6 resistors are in l
parallel. No option is correct.
6 6 36 32. (b) The transport current flows through the
Equivalent resistance, R = surface of the superconducting wire and not
6 6 12 through the entire area of cross-section of
=3 . the wire.
29. (b) In a conductor, the charge carriers are 33. (c) Here, E = 3 × 104 NC–1
electrons. As the temperature is increased, p = 6 × 10–30 Cm
the collisions of these conduction electrons
with the fixed ions of the lattice of the metal max = ?
increases and hence the resistance of the We have,
conductor also increases. torque acting on a dipole = pE sin
30. (b) If there is no potential difference through a The maximum value of , i.e. max = pE
metallic wire, the current is zero because the 4 –30
electrons drift in a random direction with a max = (3 × 10 × 6 × 10 ) Nm
= 18 × 10–26 Nm
speed of the order of 10–2 cm/s so that the
34. (c) When a metallic plate swings between the
net charge crossing a particular cross-section
poles of a magnet, eddy currents are set up
in a given time is zero.
inside the plate. These currents set up their
31. If the wire is replaced by another wire of same
own magnetic field which opposes the
material but with double the length and half
magnetic field of the poles. Thus, the
the thickness, the resistance of the wire as a
direction of the current opposes the motion
whole will change. Let us calculate this
of the plate.
change.
35. (b) When an electrical appliance is switched on,
L L the electrons in the conducting wires move
Initial resistance, R = with their drift speed which is very less than
A r2
the speed of light. But as soon as the switch
2L 2L is on, an electromagnetic wave is set up inside
Fubak resistance, R ' = 2 the conductor and the electrical signal is
A' r
carried by them. The speed of
2 electromagnetic wave is, of course, equal to
8L L the speed of light and hence the appliance
= =8 = 8R responds almost immediately after the switch
r 2
r2 is made on.
Therefore, the resistance will increase by 36. (b) The circuit shown is a parallel resonant
eight times. 1
In a meter-bridge, we have circuit. The frequency is f = at
2 LC
EBD_7443
www.jeeneetbooks.in

WWW.IIT-NEET.XYZ

2008-16 Target VITEEE


resonance. Also, at resonance, the
capacitative reactance is equal to the PART - II (CHEMISTRY)
inductive reactance. Therefore, equal current
will flow through both the bulbs b1 and b2. 41. (b) Aliphatic nitro compounds are prepared by
So, both will glow with same brightness. vapour phase nitration of alkanes at 693-
37. (b) In a transformer, the ratio of turns in the 793K, under pressure. Alkanes though less
windings is given by reactive do undergo nitration to give a
N P VP mixture of nitro alkanes resulting through
NS VS cleavage of carbon-carbon bond alongwith
oxidation product like CO2, NO2, H2O etc.
NP 5kV 5000V 675K
NS 240V = 240V = 20.8 CH3CH3 HNO3 CH3CH 2 NO2
(fuming) (80%)
N2A
38. (a) Self - inductance (L) of a solenoid = o CH 3 NO 2
l (20%)
where o = absolute permittivily of space/
air 42. (a) Amides are hydrolysed rapidly by acids to
N = number of turns in the coil produce carboxylic acid and ammonium salt;
A = area of cross-section of the H O
3
solenoid RCONH 2 RCOOH NH 4
l = length of solenoid. Hence acetamide will give acetic acid on
This expression shows that for all three hydrolysis.
solenoids, the self-inductances will be equal. 43. (b) Diazotisation reactions are shown by primary
39. (b) We know that the refractive index of a aromatic amine only as the arene diazonium
medium is given by salt formed is stable at 273-278 K. Compound
co C6H5CH2NH2 is not an aromatic amine, hence
c will not give the test/reaction.
Where c = velocity of light in the medium 44. (a) Secondary nitro alkanes upon hydrolysis
co = velocity of light in vacuum with boiling HCl gives a ketone & nitrous
For crown glass, oxide.
co HCl
1.5 ... (1) 2R 2CHNO 2 2R 2 CO N 2 O H 2O
8 boiling
2 10
co 45. (a) Stephen reduction is partial reduction of alkyl
For flint glass, 1.8 ...(2) or aryl cyanides to give corresponding
c aldehydes with a suspension of anhydrous
2 108 1.8 stannous chloride in ether.
Dividing (2) by (1), we get
c 1.5 SnCl2 2HCl SnCl4 2[H]
8
1.5 2 10 CH3C N 2[H] HCl
or c = m / s = 1.67 × 108 m/s
1.8 Ether
40. (c) Fast moving electrons create electromagnetic CH3CH NH.HCl
A
waves. So, the diffraction pattern will be diamine hydrochloride
observed. Also, the angular width of the
central maximum is given by hydrolysis
CH 3CHO NH 4 Cl
boiling H 2O
2
W= , where a = width of the slit 46. (a) In a colloidal system, the substance present
a
= wavelength of the light. in large amount in the mixture is called the
When the speed of electrons will increase, dispersed medium & the solute is called
the frequency will increase which will result dispersed phase. In case of milk and water
in decreasing the wavelength as the speed solution the dispersed phase is milk protein
of light is a constant. Therefore, the angular & fat and water is dispersed medium.
width will decrease.
www.jeeneetbooks.in

WWW.IIT-NEET.XYZ

Solved Paper 2008 2008-17

47. (b) No. of moles of a compound Hence the Ph– will attack the phosphorous
given mass (gm) in PCl 3 to form organic phosphine with
= Molar mass (gm) formula Ph3P.
3PhMgBr PCl3 Ph 3 P 3MgBrCl
25.6 Triphenylphosphine
i.e. = 0.0748 moles.
342.3 53. (d) Transition elements or d-block elements have
1 mole of sucrose (C12 H22O11) contains 6.022 variable oxidation states, they form coloured
× 1023 molecules of it. compounds because of partially filled d-
Hence 0.0748 moles contains orbitals and also because of small size they
= 6.022 × 1023 × 0.0748 form interstitial compounds. They are stable
= 0.4504 × 1023 molecules. elements and does not show radio activity.
1 molecule of sucrose by formula is having 54. (a) In PCl 5 , phosphorous undergoes sp 3 d
22 atoms of hydrogen. hybridization and has trigonal bipyramidal
0.4504 × 1023 × 1023 × 22 = 9.91 × 1023 geometry. It has two axial chlorine atoms &
atoms of hydrogen. three equatorial chlorine atoms bonded to
48. (c) Milk contains lactose as milk sugar. After the central P.
digestion of milk lactose is broken down by Hence bond angles for axial are 90°, Cl–P–Cl
enzymes lactase to form glucose and & for equatorial Cl–P–Cl it is 120°.
galactose before it enter the blood stream.
49. (b) Out of 20 amino acids, the 10 amino acids Cl
which human body cannot synthesize are 90°
called essential amino acids. The ten Cl
essential amino acids are :
1) Valine 2) Leucine 3) Isolucine 4) Histidine Cl P 120°
5) Phenylanaline 6) Methionine 7)
Tryptophan 8) Lysine 9) Arginine 10) Cl
Threonine.
50. (c) Among the given examples, glucose is an Cl
alcohexose, sucrose is a disaccharide, 55. (a) Magnetic moment of a salt depends upon
fructose is a ketohexose while ribose is a the number of unpaired d-electrons. In Zn 2+
aldopentose.
salt configuration of cation is 4s03d10. Hence
51. (d) To find the oxidation number of a given
compound we have to equate the charge on total no. of unpaired electron, n is zero. So
the overall compound with the charge on magnetic moment i.e.
individual atom of which the compound is B.M. n(n 2) 0.
made of. 56. (a) Formula unit = no. of molecules of CaF2.
In KO3 . K is an alkali metal.
hence its oxidation number is +1. massin gm 146.4 gm
Moles = molar mass 78.08gm 1.875
1
(+1) + 3 × (x) = 0 or x = 0.33
3 Molecules = Mole × 6.022 × 1023
hence oxidation number of oxygen = 1.875 × 6.022 × 1023
i.e. x = –0.33. = 1.129 × 1024 units of CaF2
In Na2 O2, again Na is an alkali metal. 57. (c) For writing IUPAC name of a co-ordination
Hence 2 × (+1) + 2 × x = 0 compound we first write the name of (+) ive
2 complex here. [Co(NH3)5Cl]2+
x= 1 The names of ligands will come first in
2 alphabatical order, followed by metal ion with
52. (c) In a reaction the alkyl part of grignard reagent its oxidation state written in bracket or
acts as a nucleophile as carbon is more
parentheis in Roman number i.e. Co (III) here.
electronegative than magnesium. Hence the
IUPAC name for cationic complex
alkyl part will get attached to the electron
deficient species. Pentaamine chloro cobalt (III).
In PCl3, chlorine is more electronegative than This will follow the name of anion with a gap.
phosphorous. i.e. Pentaamine chloro cobalt (III) chloride.
EBD_7443
www.jeeneetbooks.in

WWW.IIT-NEET.XYZ

2008-18 Target VITEEE


58. (b) Fe (III) ion from ferric nitrate will react with 65. (b) The driving forces which are responsible for
thiocyanate, SCN– ion to form a blood red a process to be spontaneous are :
complex i.e. FeSCN2+. But in presence of i) Tendency for minimizing energy
water it forms a complex containing five water ii)Tendency for maximum randomness. i.e.
molecules. i.e. [Fe (H2O)5 (SCN–)]2+.
maximum entropy
59. (c) Silver nitrate has been used since the
begining of nineteenth century to dye hair. 66. (b) For 1st order reaction.
Silver salts darken when exposed to light and 0.693
silver combines with protein yeilding a dark t½ (half life time) =
K
coloured proteinate.
60. (b) Schottky defect is generally shown by 0.693 0.693
compounds which have ionic nature and Hence K = = = 0.693 ×
t½ (60 40)sec
small difference in the size of cations &
anions. 10–2 sec–1 = 6.93 × 10–3 sec–1
In this defect equal no. of cations & anions 67. (b) Since NaNO3 is formed by the reaction
is found missing from their lattice sites.
61. (d) In [Co(NH3) 6]3+ and {CoF6}3– both the Nacl KNO3 NaNO3 KCl
oxidation state of cobalt ion is +3. In first hence, using Kohlrausch's law
case NH3 is the neutral ligand which is a
o o o o
strong field ligand. m NaNO 3 NaCl KNO 3 KCl
hence the electrons in Co (+III) i.e. 4s03d6
get paired to form inner orbital complex. = 128 + 111 – 152 = 87 S cm2 mol–1
Hence no unpaired electron. 68. (c) As we know that
On the other hand F– is a weak field ligand Ecell = Ecathode – Eanode
hence it forms an outer orbital complex with when Ecathode = Eanode
4 unpaired electrons. Ecell = 0
62. (a) G° = –115 kJ at 298K. If Ecell = 0 no net reaction occurs. The
Now, G° = –2.303 RT log kP reactants and products are at equilibrium and no
R = 8.314 JK–1 mol –1 & T = 298K. current will flow.
G° = –2.303 × 8.314 × 298 × log kP Note that it is only possible to obtain electrical
115 103 work from a system that is not at equilibrium. In
log kP = = 20.155 order for current to flow, there must be a net
2.303 8.314 298 reaction occurring. As the oxidation- reduction
= 20.16 reaction proceeds toward equilibrium, and the
63. (c) For a reaction to take place spontaneously concentrations of the reacting species approach
the value of G must be negative i.e. G< 0. their equilibrium values, the EMF of the cell
Now, G = H– T S. decreases to zero. When the system is at
As the reaction is endothermic, so value of equilibrium, the cell potential is zero and we have
H must be positive, i.e. H > 0. a dead battery.
Hence to have a negative G.
H < T S. As T & P are constant. 69. (c) In CuSO4 solution, oxidation state of Cu is
T S must be positive to give the total value +2. Hence one mole of copper sulphate will
a negative sign. require charge equal to two moles of
Hence S> 0. electrons to form metallic Cu. Mole charge =
64. (b) Any first order reaction follows the equation IF. Hence 2 Faraday is required.
k 70. (c) Rusting of iron is generally promoted in an
log [A] = t + log [A]o acidic aqueous medium. Alkaline medium
2.303
it resembles equation of straight line prevents availability of H+ ions. Sodium
y = mx + C phosphate will cause formation of a
y = log [A] i.e. log10 C protective film of iron phosphate on the iron
k preventing rusting. These solutions are used
m= if x = t & C = log [A]0 in car radiators to prevent rusting of iron
2.303
hence the plot is for a 1st order reaction. parts.
www.jeeneetbooks.in

WWW.IIT-NEET.XYZ

Solved Paper 2008 2008-19

71. (a) Hydroboration-oxidation of alkenes give 76. (c) The witting reaction is a chemical reaction of
alcohols containing same number of carbon an aldehyde or ketone with triphenyl
atoms. The addition follow anti-Markowi phosphonium ylide to give an alkene and
Koff's rule. Boron atom act as an electrophile. triphenyl phosphine oxide.
Main two steps re involved. Reagent used R
BH3 & NaOH/H2O2.
C = O + (C6 H5)3 P = C
H H H Me H R´
| | | | |
3H3C — C — C— C — C — C— C CH 2 BH 3 OR
| | | | | |
H H H H H H R
–+
4 - methyl octene (C6H5)3P – C
Phosphorus ylide R´
R 3B
where R=4-methyl octyl R
C=C + O = P(C6 H5)3
H 2O 2 R´
3CH 3 (CH 2 ) 3 CH (CH 2 ) 3 OH
NaOH | Triphenyl phosphine
Me oxide
4 Methyloctanol
77. (a) Cannizaro's reaction is for those aldehydes
72. (d) Ethyl alcohol on treating with conc. H2SO4
which does not contain - hydrogen atom.
undergoes dehydration to form alkene i.e.
This is also called self oxidation - reduction
ethylene
reaction. Among the given carbonyl
Conc
C2 H5 OH C2 H 4 H 2O compounds only HCHO does not have -
H 2SO4
hydrogen.
OCH3
CHO
73. (b) Anisole is O , Phenyl methyl ether. It 78. (c) O + CO
AlCl3
HCl O
can be prepared by treating phenol first with
a base like NaOH to form phenoxide ion. The When a mixture of CO and HCl gas is passed
phenoxide ion will then substitute the halide through benzene in presence of catalyst
of an R-X molecule, to form methyl phenyl consisting anhydrous AlCl3, benzaldehyde
ether. is formed.
C6 H5ONa CH3 Cl CH3 O C6 H5 CO HCl HOCl
Anisole
formyl chloride
74. (c) Alkaline Potassium permanganate is a strong
oxidising agent. It oxidises ethylene glycol AlCl3
C6 H 6 HOCl C6 H 5 CHO HCl
to oxalic acid.
79. (b) Maleic acid & fumaric acid are both the
CH2OH COOH
alk. isomers of butene dioic acid. Maleic acid is
KMnO 4 the cis isomer & fumaric is the trans-isomer.
CH2OH COOH
Ethylene Oxalic O O
glycol acid O OH
75. (a) In the structure of diamond each carbon atom OH OH
in sp3 hybridised & is covalently bonded OH O
with four other carbon atom held at the Maleic acid Fumaric acid
corners of a regular tetrahedron by covalent
bonds. This results in a very big three 80. (c) Alkali formate i.e. HCOONa with soda-lime
dimensional polymeric structure in which C i.e. NaOH + CaO will react to give Na2CO3
– C distance is 154 pm and bond angle is and hydrogen gas is liberated.
109.5°. Owing to very strong covalent bonds CaO
HCOONa NaOH Na 2 CO3 H 2
by which atoms are held to gether diamond
is the hardest substance known.
EBD_7443
www.jeeneetbooks.in

WWW.IIT-NEET.XYZ

2008-20 Target VITEEE

PART - III (MATHS) 1 x


sin
1 1 x2
81. (c) a (b c) b
2 1 1 x
tan x sin
1
(a.c)b (a.b)c b [Vector triple product] 1 x2
2
1 1 x
( a . c cos 2 )b ( a . b cos 1 )c b Now, sin (tan–1x) = sin sin
2 1 x2
[ a b c 1] x
=
Equating the coefficients of b and c on 1 x2
both sides, we get 84. (c) Given that
1 z 25
cos 2 and cos 1 0 5 Z 25 5Z 1
2 z 1
Let Z = x + iy, then
cos 2 cos and cos 1
3 2 x iy 25 5 x iy 1
and 1
2 (x 25) iy 5 (x 1 iy
3 2
82. (d) Vector equation of a sphere in central form Squaring both sides, we get
(x – 25)2 + y2 = 25 {(x – 1)2 + y2}
with centre having position vector c and
radius R is x 2 50x 625 y 2
2
r c R r c R2 25x 2 50x 25 25y 2
24x2 + 24 y2 – 600 = 0
(r c).(r c) R2
x2 + y2 – 25 = 0
r.r r.c c.r c.c R2 |x + iy|2= 25 |Z|2 = 52
|Z| = 5
2 2
r 2(r.c) c R2 85. (c) We have,
2 2 1 3i 1 3i 2 i 2 5i 3i 2
r 2(r.c) c R2 0
2 i 2 i 2 i 4 i2
2
If h c R 2 i.e. c h , then = –1 – i
Now, let us put –1 = r cos , –1 = r sin
2
r 2(r.c) h 0 Squaring and adding, r2 = 2 i.e., r = 2
Hence, the given equation represents a 1 1
sphere. So that cos = , sin =
83. (b) Let tan–1 x = 2 2
Therefore, = 225°
sin sin Thus, argument is 225°.
x tan
cos 1 sin 2 86. (c) Since b and C are the roots of
x2 – 61x + 820 = 0, so
x 1 sin 2 sin b + c = 61, bc = 820
4 4 3
x 2 (1 sin 2 ) sin 2 A tan 1
tan A cos A
3 3 5
x2 sin 2 (1 x 2 ) Now, using the formula,
x2 sin
x b2 c2 a 2
sin 2 cos A =
1 x2 1 x2 2bc
www.jeeneetbooks.in

WWW.IIT-NEET.XYZ

Solved Paper 2008 2008-21

a2 b2
= + c2 – 2bc cos A x2 + y2 + z2 + 3x – 4z + 1 = 0,
= (b + c)2 – 2bc – 2bc cos A
3
= (b + c)2 – 2bc (1 + cos A) Centre , 0, 2 , and
2
3
= (61)2 – 2 × 820 1 2
5 3
Radius = 02 (2)2 1
= 3721 – 2624 = 1097 2
87. (d) Equation of first line,
x 6 y 2 z 2 9 21
k(say) 4 1
1 2 2 4 2
x = k + 6, y = –2k + 2, z = 2k + 2 89. (b) Let two fixed points be A (ae, 0) and B (–ae,
Hence, general point on the first line, 0). Let C (x, y) be a moving point such that
P (k + 6, – 2k + 2, 2k + 2) AC + CB = constant = 2a (say)
Equation of second line,
i.e. , (x ae) 2 (y 0)2
x 4 y Z 1
l (say)
3 2 2 (x ae) 2 (y 0)2 2a
x = 3l – 4, y = –2l, z = –2l –1
Hence, general point on the second line, Or x2 y2 a 2e2 2aex
Q (3l – 4, – 2l, –2l – 1)
Direction ratios of PQ are x 2 y 2 a 2e 2 2aex 2a ...(1)
3l – 4 – k –6, –2l + 2k –2, – 2l –1 –2k – 2 Or l + m = 2a ...(2)
i.e. 3l – k – 10, –2l + 2k– 2, – 2l –2k – 3 Where, l2 = x2 + y2 + a2e2 – 2aex ...(3)
Now |PQ| will be the shortest distance and m2 = x2 + y2 + a2e2 + 2aex ...(4)
between the two lines if PQ is perpendicular From, (3) and (4)
to both the lines. Hence, m2 – l2 = 4aex
1(3l k 10) ( 2) or (m – l) (l + m) = 4 aex
2a (m – l) = 4aex [From (2)]
( 2l 2k 2) 2( 2l 2k 3) 0
m – l = 2ex ...(5)
and 3 (3l – k – 10) + (–2) (–2l Adding (2) and (5), we get
+ 2k – 2) + (–2) (–2l – 2k – 3) = 0 m = a + ex ...(6)
i.e. 3l – 9k = 12 or l – 3k = 4 ...(i) From (4) and (6),
and 17l – 3k = 20 ... (ii) a2 +e2 x2 + 2aex = x2 +y2 + a2e2 + 2aex
Subtracting equation (i) from (ii), we get x2 (1 – e2) + y2 = a2 (1 – e2)
16l = 16 l=1 Dividing both sides by a2 (1 – e2), we get
Putting this value of l in equation (i), we get
– 3k = 3, k = – 1 x2 y2
P (–1 + 6, – 2 (– 1) + 2, 2 (– 1) + 2) 1
a2 a 2 (1 e 2 )
(5, 4, 0)
Similarly, Q = (–1, –2, –3) x2 y2
Hence, shortest distance, PQ, Or 1 , where b2 = a2 (1 – e2)
a2 b2
= ( 1 5)2 ( 2 4)2 ( 3 0)2 This is the equation of ellipse.
90. (d) The equation of the parabola is
= ( 6)2 ( 6)2 ( 3)2 = 36 36 9
y2 4x 3 0
= 9 units
88. (c) Since the centre and radius of the sphere 3
x2 + y2 + z2 + 2 ux + 2vy + 2wz + d = 0 are or y2 4 x ...(1)
4
(–u, –v, – w) and u 2 v 2 w2 d The directrix of the parabola
respectively. So, for the sphere Y2 = – 4aX ... (2)
is X = a.
EBD_7443
www.jeeneetbooks.in

WWW.IIT-NEET.XYZ

2008-22 Target VITEEE


On comparing the equation (1) and (2), we 93. (b) Check the continuity of the function
3 f (x) = [tan2 x] at x = 0.
get 4a = 4 and X x L.H.L. (at x = 0)
4
2
3 = lim [tan x] lim [tan 2 (0 h)]
x 0 h 0
or a= 1 and X x
4 2 2
Hence the directrix of the parabola (1) is = lim [tan h] [tan 0] [0] 0
h 0
3 1 R.H.L. (at x = 0)
x 1or x 0.
4 4 2
= lim [tan x] lim [tan 2 (0 h)]
91. (b) g (x). g(y) = g(x) + g (y) + g (x y) – 2 ...(1) x 0 h 0
Put x = 1, y = 2, then
2 2
g (1). g(2) = g (1) + g (2) + g (2) – 2 = lim [tan h] [tan 0] [0] 0
5g (1) = g (1) + 5 + 5 – 2 h 0
4g (1) = 8 g(1) = 2 Now, determine the value of f(x) at x = 0.
f (0) = [tan 2 0] = [0] = 0
1 Hence, f (x) is continuous at x = 0.
Put y in equation (1) , we get
x 94. (c) Let r and V be the respectively radius and
1 1 volume of the balloon. Let t represents the
g(x).g g(x) g g(1) 2 time. The rate of increament in radius is
x x
dr
1 1 2 cm/minute. The volume of the
g(x).g g(x) g 2 2 dt
x x balloon is given by
[ g(1) 2] 4 3
V r
This is valid only for the polynomial 3
g (x) = 1 + xn ... (2) Differentiating w.r. to t, we get
Now g (2) = 5 (Given)
1 + 2n = 5 [Using equation (2)] dV 4 dr
(3r 2 )
+ 2n = 4, 2n = 4, –4 dt 3 dt
Since the value of 2n cannot be –Ve. dr
So, 2n = 4, n = 2 Substituting the values of and , we get
Now, put n = 2 in equation (2), we get dt
g (x) = 1 + x2 dV 4
(3 52 2) 200 cm3 / minute
2
Lt g(x) = Lt (1 x ) = 1 + (3)2 dt 3
x 3 x 3 95. (a) On comparing the equation of the parabola
= 1 + 9 = 10, – 8 y2 = 12 x with the standard equation,
y2 = 4 ax, we get 4 a = 12 or a = 3.
ex 2x
92. (d) f (x) y
x
A(3, 6)
1 x x2 x3 log 2
=x (1 ..) 1 x
1! 2! 3! 1!

(bg 2) 2 2 (log 2)3 3


x x ..
2! 3! x
C (3, 0)
(0, 0) O
x
f(x) = log 2 1 {(log 2) 2 1}
2!
x2
{(log 2 )3 1} ....
3!
Putting x = 0, we get
f(0) = log 2 – 1 + 0 + 0 + .... = – 1 + log 2. B (3, –6)
www.jeeneetbooks.in

WWW.IIT-NEET.XYZ

Solved Paper 2008 2008-23

Hence, the focus, point C will be at (3, 0) and


1
the extremities of the latus-rectum AB will be = 3.6 2 18log 6 6 2 18 log 6
at (a, 2a) and (a, –2a). So the coordinates of 3
A and B are (3, 6) and (3, –6) respectively.
6(1 2)
Now we need to find the length ( curve AOB) = 6 2 6log
of the parabola. As it is not a straight line so 6
we cannot directly find the length of this
curve as we cannot directly apply = 6 ( 2 log(1 2)
Pythagorous theorem. Let us consider a small
length ds on the parabola. Using x5 x 3 .x 2
pythagorous theorem for this length, 96. (d) I dxdx
1 x3 1 x3
2 Let 1 + x3 = t2, so that 3x2 dx = 2t dt
dx
ds = (dx) 2
(dy) 2 = 1dy
dy 2
x 2 dx td t
3
2
6 dx 2
s 1 .dy ...(1) (t 2 1) t dt
6 dy 3 2 2
I t 1)dt
t 3
y2 1
From y2 = 12 x x= 2 t3 2 (1 x 3 )3 / 2
12 = t C (1 x 3 ) 2 C
3 3 3
3
dx 2y y
Putting in (1), 2 2
dy 12 6 = (1 x 3 )3 / 2 (1 x 3 )1/ 2 C
9 3
6 2 6
y y2 36 97. (d) The given curve is
s 1 dy 2 dy
6
6
0
36 [4(x 2)2 y2 ] 8
8
2
6
2 2 4(x 2) 2 y2
= 6 y 6 dy
0 2
y 2
x 2 (x 2)2
2 2 2 2
Using x a a x
2
a2 (x 2)2 y2
log x a 2
x 2
C 1
2 2
2 2 2 ...(1)
2
2
1 y 2 6
We get s 6 y2
3 2 2 This is the equation of the ellipse having

6
centre ( 2, 0 ).
2 2 Observe the figure of ellipse (1). The centre
log y 6 y C
0
2
1 6 2 P is ( 2, 0 ). A and B are 2, , 0 and
= 6 62 18log 6 6 2
6 2
3 2
2
C 0 18log 0 62 0 C 2 , 0 respectively..
EBD_7443
www.jeeneetbooks.in

WWW.IIT-NEET.XYZ

2008-24 Target VITEEE


y Q
1 1
=8 (0) + sin –1 (1)–0–0

1
8 4 square units.
2
98. (c) Let x = cos2 , so that dx = – 2a sin cos d
2 ,0 2 ,0
2 2 a a x 0 a a cos2
( 2, 0)
Now dx
O A P B 0 x 2 a cos 2
( a sin cos )d

[ at x 0; 0 at x a]
2

/2 1 cos 2
=a 0 2sin cos d
cos 2
The required area = 4 × area of figure PQB 0 t
f (x) dx f (x) dx
2 t 0
2
=4 ydx
2 /2 sin
2
=a 2 .sin cos
2 8
0 cos
=4 4(x 2) 2 dx /2 /2
2
=a 2sin 2 =a (1 cos 2 ) d
2 2
0 0
2 2
4 2 (x 2) 2 dx [ cos 2 1 2sin 2 ]
2
/2
sin 2
x 2 2 =a
2)2 2 0
=8 2
(x
sin
2
2 =a (0 0)
(2 / ) x 2 2 2
1
sin
2 2/ a
2
=a 0
a 2
x
2 2
1 1
a2 x 2 dx x a2 x2 sin C 99. (b) According to the question,
2 2 a
dy dy
2 y ky
2 2 dt dt
2 2 2
8 ( 2 2)
= 2 x dy
Separating the variables, we get kdt
dt

2 dy
2 2 Integrating both sides, we get k dt
1 1 2 2 y
sin
2 2 log y = k t + M (as y cannot be –ve)
y = ekt+M y = eM . ekt
y = C ekt, where C = eM
2 1 2 2 Constant k cannot be positive because the
( 2 2) 2 sin 1
2/ population never increases in time. And
www.jeeneetbooks.in

WWW.IIT-NEET.XYZ

Solved Paper 2008 2008-25

another constant C cannot be negative This is the linear differential equation.


because of eM > 0 always. dy dv
Hence y = Cekt, for some constants C > 0 and Put y = vx, so that v x . Then
dx dx
k < 0.
100. (b) The given circle is, x2 + y2 = a2 dv vx x 2 v2 x 2
Differentiating with respect to x, we get v x
dx x
dy dy dv
2x 2y 0 x y 0 v x v 1 v2
dx dx dx
2
dy dv dx
x y 0 (Squaring both sides)
dx 1 v2 x
2 Integrating both sides, we get
dy dy
x2 2xy y2 0 dv dx
dx dx
1 v 2 x
2
dy 2 2 dy
2xy x y log(v 1 v2 ) log x log C
dx dx
2 v 1 v2 Cx
2 2 dy dy
y x 2xy
dx dx y y2
1 Cx [ y vx]
dy
2
dy
2 x x2
2 2 2 2
x y x y
dx dx y x2 y2 Cx 2
2 103. (b) ~ Q S ~S Q
2 dy
(Adding y x2 both sides) But ~ S R
dx
Q R, True [As P Q R P]
2 2
dy dy Hence ~ Q S is true
y x a2 1
dx dx 104. (d) We know that the number of ways of
dividing (m+n+p) things into three groups
containing m, n and p things respectively
x2 y2 a2
(m n p)!
dy dy m!n!p!
101. (b) y 3 0 Since 0, y 0
dx dx Further if any two groups out of the three
have same number of things then number of
dy ways
y 3 3
dx (m n p)!
dy m!n!p! 2
Hence y 3 0 is not possible.
dx Hence number of ways to divide 10 students
Therefore, the given differential equation has into three teams one containing four students
no solution. and each remaining two teams contain three
102. (b) The given differential equation is 10! 10 9 8 7 6 5
= 4! 3! 3! 2 = = 2100
xdy ydx x2 y2 dx 0 3 2 3 2 2
105. (d) We observe the following properties :
xdy y x2 y2 dx Reflexivity - Let a be an arbitrary element.
Then,

(y x2 y2 a a 0| 0 a R/ a
dy
dx x This, R is not reflexive on R.
EBD_7443
www.jeeneetbooks.in

WWW.IIT-NEET.XYZ

2008-26 Target VITEEE


Symmetry – Let a and b be two distinct 2 2
n 3 1
elements, then (a, b) R Cr pr q n r 4
C2
4 4
|a – b| > 0 |b – a| > 0
a b b a 3 3 1 27
=6
4 4 4 4 128
(b, a) R 109. (b) Performance of the class will be best if mean
Thus, (a, b) R (b, a) R. So, R is of the marks obtained is maximum but
symmetric. standard deviation of the marks obtained is
Transitivity – Let (a, b) R and (b, c) R. minimum.
Then |a – b| > 0 and |b – c| > 0 Hence the class which has mean and
|a – c| > 0 (a, c) R standard deviation of the marks obtained as
So, R is transitive. 75 and 5 respectively performs best.
106. (a) Let S = {ai } where i = 1.2.....n 110. (b) Mean, np = ; and variance, npq = where n
From commutative operations, = number of trials and p + q = 1.
a i *a j a j *ai … (i) i, j 1, 2,3....n npq
So, q (1 p)
where * represents a binary operation np
Number of distinct elements in S × S
0 p 1
i.e., {a i } {a j} subject to the condition (i)
–1< – p < 0 0<1–p<1
i 1,2...n j 1,2...n

= n{(a1, a1 ), (a1, a 2 )......(a1, a n ), 0 1 0< <


(a 2 , a 2 ), (a 2 , a 3 ),....(a 2 , a n ), 111. (b) x + y + z = 0 ...(1)
...(a n 1, a n 1 ), (a n 1 , a n ), (a n ,a n ) 2x + 3y + z = 0 ...(2)
x + 2y = 0 ... (3)
n(n 1) From equation (3), we have
n (n 1) (n 2) .... 2 1 x = – 2y
2
No. of commutative binary operations Putting this value of x in equations (1), we
= No. of functions f : S × S S subject to (i) get – 2y + y + z = 0 y = z
Hence x = – 2z
n(n 1)
n(n 1) Thus, the solution of the given system of
= n.n.n.... times n 2 equations is (–2z, z, z), where z is a parameter
2
(Z R). Hence the system has infinite
107. (b) Poisson distribution is a probability
number of solutions including zero solution.
distribution which is obtained when the
probability (p) of the happening of an event 2 2
0 a 0 a 0 a
is same in all the trials and there are only two 112. (d) Here,
b 0 b 0 b 0
events in each trials generally says
successes and failures probability (p) of the 0 ab 0 0 ab 0
happening of the event in trial is very less = 0 0 ab 0 = 0 ab
but number of trials (n) is very large.
5 1 4
Here, p = 5% = is very less and 0 a ab 0 ab 0
100 20 Similarly,
b 0 0 ab 0 ab
n = 100, is very large. Hence, one has to
employ the Poisson distribution in the given a 2 b2 0 0 0 a2b2 0
question. = =
108. (d) The probability that a component survives 0 0 0 a2b2 0 a 2 b2
1 Now,
is p = . Then q = 1 – p = 1
4 4 4 4 a 2 b2 0 1 0
0 a
[ p q 1] I
b 0 0 a b 2 2 0 1
n takes the value 4 and r = 2. Hence the
required probability is a2b2 =1 ab = 1
www.jeeneetbooks.in

WWW.IIT-NEET.XYZ

Solved Paper 2008 2008-27

113. (d) D = diag (d1, d2, ..., dn)


d1 1 0 0 0
d1 0 0 0
0 d2 0 0 0 d 21 0 0
1
0 0 d3 0 0 0 d3 0
D

0 0 0 0 0 0 0 0 0 d n1

d1 0 0 0 = diag ( d1 1 , d 2 1,...., d n 1 )
0 d2 0 0
a b y c z
0 0 d3 0
D 114. (d) a x b c z 0
a x b y c

0 0 0 0 0 a(bc bc bz cy yz) (b y)
= d1 d2 d3 ....... dn (ac cx ac az cx zx) (c z)
(ab ay bx xy ab bx) 0
d 2 d 3d 4 ...dn 0 0 0 abz + acy – ayz – abz + bzx + ayz –
0 d1d 3d 4 ...dn 0 0
0 0 d1d 2 d 4 ...dn 0 xyz – acy + cxy + ayz – xyz = 0
adj(D)
ayz bzx cxy 2xyz 0
0 0 0 0 d1d 2 d 3 ...d n 1 a b c
2 0 (Dividing by xyz)
1 1 x y z
d adj (D)
D a b c
2
1 x y z
adj(D)
d1d 2 d3....d n 115. (c) x2 – n = 0 x n
d 2 d3d 4 ...dn 0 0 0 For each integral value of x [1, 40], There is
0 d1d3 d 4 ...dn 0 0 a positive root. Hence for 40 integral values
0 0 d1d 2 d 4 ...dn 0 of x from 1 to 40, there are 40 positive roots,
out of which only six roots 1, 2, 3, 4, 5 and 6
are positive integral roots. Hence, probability
0 0 0 0 d1d 2 d3 ...d n 1
6 3
of getting positive integral roots =
40 20
1
0 0 0
d1 116. (a) x4 x4 20 22
1
0 0 0 or x4 – 22 =x 4 20
d2 4
Put x = y and square both the sides.
1 (y – 22)2 = y + 20
0 0 0
d3 y2 + 484 – 44y = y + 20
y2 – 45y + 464 = 0
y2 – 29y – 16y + 464 = 0
1 (y – 29) (y – 16) = 0
0 0 0 0 y = 16, 29
dn
x4 = 16, 29 or x = + 2, + 2.31
EBD_7443
www.jeeneetbooks.in

WWW.IIT-NEET.XYZ

2008-28 Target VITEEE


117. (c) Since , be the roots of the equation
i j k .[i (3 ) j (2 1)k] 0
x2 – ax + b = 0, so each of them must satisfy
the equation. Therefore 1 (3 ) (2 1) 0
2 2
a b 0 ...(1) and a b 0 3 3 0 1
...(2) Substituting this value for in (i), we get the
Now, An + 1 – a An + b An–1 = n+1 + n+1 required plane r.(i 2j 3k) 0
– a ( n n) + b ( n–1 + n–1)
= n–1 ( – a + b) + n–1( 2 – a + b)
2 120. (d) In the figure, OAC is a triangle and OB is a
n–1 median such that
= (0) + n–1(0) = 0 [From (1) & (2)]
118. (a) Let the sides of the triangle be a – d, a, a+d, OA a i ˆj kˆ
where d is greater than zero. From the figure,
it is clear that the angles A and C are acute OB b 2i 4ˆj 3kˆ
angles. Now, by the theorem of Pythagorus,
OC c (say)
AC2 = AB2 + BC2
(a + d)2 = a2 + (a – d)2 O
a2 + d2 + 2ad = a2 + a2 + d2 – 2ad
4ad = a2 d = a/4

A a
b
a+d

a
A B C

OA OC 2OB
B a–d C a c 2b
c 2b a 2(2iˆ 4jˆ 3k)
ˆ (iˆ ˆj k)
ˆ
a
a = (3iˆ 9jˆ 5k)
ˆ
a d 4 3
sin A Now, the area of the triangle,
a d a 5
a
4 1 1
OA OC a c
2 2
a 4 a Here,
sin C
a d a 5
a ˆi ˆj k
4
119. (a) The plane containing the line of intersection a c 1 1 1
3 9 5
of the planes r.(i 3j k) 0 and

r.( j 2k) 0 is = ˆi( 5 9) ˆj(5 3) k(9


ˆ 3)

r.(i 3j k) [r.( j 2k)] 0


ˆ = 2( 7iˆ ˆj 6k)
= 14iˆ 2jˆ 12k ˆ

r.[i (3 ) j (2 1)k] 0 ...(1) a c = 2 ( 7)2 ( 1)2 (6)2


Since the plane (i) passes through the point = 2 49 1 36 2 86
(–1, –1, –1) or ( i j k ), so this point must 1
2 86 86 .
satisfy (i). Hence, 2
www.jeeneetbooks.in

WWW.IIT-NEET.XYZ

VITEEE
SOLVED PAPER 2007

PART - I (PHYSICS) 4. Assuming f to be the frequency of first line in


Balmer series, the frequency of the immediate
1. The magnetic moment of the ground state of an next (i.e. second) line is
atom whose open sub shell is half filled with (a) 0.50 f (b) 1.35 f
five electrons is (c) 2.05 f (d) 2.70 f
(a) µB (b) 35 µB 5. The velocity of a particle at which the kinetic
35
energy is equal to its rest energy is
(c) 35 µB (d) µB 35 3c c
(a) (b) 3
2. Indicate which one of the following statements 2 2
is NOT CORRECT ?
1
(a) Intensities of reflections from different 3c 2 c 3
crystallographic planes are equal (c) (d)
2 2
(b) According to Bragg's law higher order of
reflections have high values for a given 6. One electron and one proton is accelerated by
equal potential. Ratio in their deBroglie
wavelength of radiation
wavelength is
(c) For a given wavelength of radiation there
is a smallest distance between the me
crystallographic planes which can be (a) 1 (b) mp
determined
(d) Bragg's law may predict a reflection from a mp me
crystallographic plane to be present but it (c) (d)
me mp
may be absent due to the crystal symmetry
3. Identify the graph which correctly represents 7. Two electrons one moving in opposite direction
the Moseley's law with speeds 0.8c and 0.4c where c is the speed
of light in vacuum. Then the relative speed is
about
f f (a) 0.4c (b) 0.8c
(a) (b) (c) 0.9c (d) 1.2 c
O z O z 8. A photo-sensitive material would emit electrons
if excited by photons beyond a threshold. To
overcome the threshold, one would increase
f f (a) the voltage applied to the light source
(c) (d) (b) the intensity of light
(c) the wavelength of light
O z O z
(d) the frequency of light
EBD_7443
www.jeeneetbooks.in

WWW.IIT-NEET.XYZ

2007-2 Target VITEEE


9. The radius of nucleus is 15. Identify the logic gate from the following TRUTH
(a) proportional to its mass number table
(b) inversely proportional to its mass number Inputs Output
(c) proportional to the cube root of its mass A B Y
number
0 0 1
(d) not related to its mass number
0 1 0
10. Radio carbon dating is done by estimating in
specimen 1 0 0
(a) the amount of ordinary carbon still present 1 1 0
(b) the amount of radio carbon still present (a) NOR gate (b) NOT gate
(c) the ratio of amount of 14C6 to 12C6 still (c) AND gate (d) NAND gate
present
(d) the ratio of amount of 12C6 to 14C6 still 16. In Boolean algebra, A B is equal to
present (a) A B (b) A B
11. Ionization power and penetration range of
(c) A.B (d) A + B
radioactive radiation increases in the order
(a) and respectively 17. Radar waves are sent towards a moving airplane
(b) and respectively and the reflected waves are received. When the
(c) and respectively airplane is moving towards the radar, the
(d) and respectively wavelength of the wave
12. The half life of a radioactive element is 3.8 days. (a) decrease
The fraction left after 19 days will be (b) increase
(a) 0.124 (b) 0.062 (c) remains the same
(c) 0.093 (d) 0.031 (d) sometimes increase or decrease
13. Two identical P-N junctions are connected in 18. The transmission of high frequencies in a coaxial
series in three different ways as shown below to cable is determined by
a battery. The potential drop across the P-N
junctions are equal in 1
(a) where L and C are inductance and
(LC)1/ 2
P N N P capacitance
(b) (LC )2
(c) the impedance L alone
1 (d) the dielectric and skin effiect
P N P P N P N 19. The output stage of a television transmitter is
most likely to be a
(a) plate-modulated class C amplifier
2 3 (b) grid-modulated class C amplifier
(c) screen-modulated class C amplifier
(a) in circuits 2 and 3
(d) grid-modulated class A amplifier
(b) in circuits 1 and 2
(c) in circuits 1 and 3 20. The antenna current of an AM transmitter is 8A
(d) in none of the circuit when only the carrier is sent, but it increases to
8.93A when the carrier is modulated by a single
14. The temperature coefficient of a zener
mechanism is sine wave. Find the percentage modulation.
(a) negative (b) positive (a) 60.1 % (b) 70.1%
(c) infinity (d) zero (c) 80.1% (d) 50.1%
www.jeeneetbooks.in

WWW.IIT-NEET.XYZ

Solved Paper 2007 2007-3

21. Two point like charges Q1 and Q2 of whose 24. Theelectric field intensity E , due to an electric
strength are equal in absolute value are placed dipole of moment p , at a point on the equatorial
at a certain distance from each other. Assuming line is
the field strength to be posiive in the positive (a) parallel to the axis of the dipole and
direction of x-axis the signs of the charges Q1 opposite to the direction of the dipole
and Q2 for the graphs (field strength versus moment p
distance) shown in Figures 1,2,3 and 4 are (b) perpendicular to the axis of the dipole and
is directed away from it
(c) parallel to the dipole moment
E E
(d) perpendicular to the axis of the dipole and
x Q1 x is directed toward it
Q1
Q2 Q2 25. Twelve wires of each of resistance 6 ohms are
connected to form a cube as shown in the figure.
1 2 The current enters at a corner A and leaves at
the diagonally opposite corner G. The joint
resistance across the corners A and G are
E E
E F
Q1 x Q1 x A B
Q2 Q2
3 4

(a) Q1 positive, Q2 negative; both positive; H G


Q1 negative, Q2 positive; both negative
D C
(b) Q1 negative, Q2 positive;Q1 positive, Q2
(a) 12 ohms (b) 6 ohms
negative; both positive; both negative (c) 3 ohms (d) 5 ohms
(c) Q1 positive, Q2 negative; both negative; 26. A conductor and a semi-conductor are
connected in parallel as shown in the figure. At
Q1 negative, Q2 positive; both positive
a certain voltage both ammeters registers the
(d) Both positive; Q1 positive, Q2 negative; same current. If the voltage of the DC source is
Q1 negative, Q2 positive; both negative increased then
Conductor
22. ABCD is a rectangle. At corners B, C and D of
A1
the rectangle are placed charges + 10 × 10–12C,
–20×10–12C and 10 × 10–12C respectively. Semiconductor
Calculate the potential at the fourth corner. The A2
side AB = 4cm and BC = 3cm
(a) 1.65 V (b) 0.165 V
(a) the ammeter connected to the
(c) 16.5 V (d) 2.65 V semiconductor will register higher current
than the ammeter connected to the
23. A parallel plate capacitor of capacitance 100 pF
conductor
is to be constructed by using paper sheets of (b) the ammeter connected to the conductor
1 mm thickness as dielectric. If the dielectric will register higher current than the ammeter
constant of paper is 4, the number of circular connected to the semiconductor
(c) the ammeters connected to both
metal foils of diameter 2 cm each required for the
semiconductor and conductor will register
purpose is the same current
(a) 40 (b) 20 (d) the ammeter connected to both
semiconductor and conductor will register
(c) 30 (d) 10 no change in the current
EBD_7443
www.jeeneetbooks.in

WWW.IIT-NEET.XYZ

2007-4 Target VITEEE


27. A uniform copper wire of length 1m and cross- 32. A wire AB, in the shape of two semicircular
sectional area 5×10–7m2 carries a current of 1A . segments of radius R each and carrying a current
Assuming that there are 8×1028 free electrons/ I, is placed in a uniform magnetic field B directed
m3 in copper, how long will an electron take to into the page (see figure). The magnitude of the
drift from one end of the wire to the other force due to the field B on the wire AB is
(a) 0.8×103s (b) 1.6×103s
(c) 3.2×103s (d) 6.4×103s
28. The temperature coefficient of resistance of a
wire is 0.00125/K. At 300K its resistance is
I R R
1 .The resistance of the wire will be 2 at
A B
(a) 1154 K (b) 1100 K
(c) 1400 K (d) 1127 K (a) zero (b) 4 RIB
29. A rectangular coil ABCD which is rotated at a
(c) R2 I B (d) 2 RIB
constant angular velocity about an horizontal
as shown in the figure. The axis of rotation of 33. There are two solenoids of same length and
the coil as well as the magnetic field B are inductance L but their diameters differ to the
horizontal. Maximum current will flow in the extent that one can just fit into the other. They
circuit when the plane of the coil is are connected in three different ways in series.
1) They are connected in series but separated
B
by large distance 2) they connected in series
A with one inside the other and senses of the
turns coinciding 3) both are connected in series
C
with one inside the other with senses of the turns
D opposite as depicted in figures 1,2 and 3
respectively. The total inductance of the
(a) inclined at 30 degrees to the magnetic field
solenoids in each of the case 1, 2 and 3 are
(b) perpendicular to the magnetic field respectively
(c) inclined at 45 degrees to the magnetic field
(d) parallel to the magnetic field
30. If the total emf in a thermocouple is a parabolic
1 2 1
function expressed as E = at + bt , which of
2
the following relations does not hold good
(a) neutral temperature t n = –a/b
(b) temperature of inversion ti = –2a/b
2
(c) thermo-electric power p = a + bt
(d) tn = a/b
31. The proton of energy 1 MeV describes a circular
path in plane at right angles to a uniform
magnetic field of 6.28 ×10–4T. The mass of the 3
proton is 1.7 × 10–27 Kg.The cyclotron frequency
of the proton is very nearly equal to (a) 0, 4L0, 2L0 (b) 4L0, 2L0, 0
(a) 107 Hz (b) 105 Hz (c) 2L0, 0, 4L0 (d) 2 L0, 4L0, 0
6
(c) 10 Hz (d) 104 Hz
www.jeeneetbooks.in

WWW.IIT-NEET.XYZ

Solved Paper 2007 2007-5

34. From figure shown below a series LCR circuit 38. Following diffraction pattern was obtained using
connected to a variable frequency 200V source. a diffraction grating using two different
L = 5H, C = 80 F and R = 40 . Then the source wavelengths and . With the help of the
figure identify which is the longer wavelength
frequency which drive the circuit at resonance
and their ratios.
is I

1 1
2 2
1 1
C = 80 F L = 5H 1 2 2 1

R = 40
(a) is longer than and the ratio of the
longer to the shorter wavelength is 1.5
(b) is longer than and the ratio of the
V = 200 V longer to the shorter wavelength is 1.5
(c) and are equal and their ratio is 1.0
(d) is longer than and the ratio of the
25 longer to the shorter wavelength is 2.5
(a) 25 Hz (b) Hz
39. In Young's double slit experiment, the interference
pattern is found to have an intensity ratio
50 between bright and dark fringes is 9. This implies
(c) 50 Hz (d) Hz
the
(a) the intensities at the screen due to two slits
35. If the coefficient of mutual induction of the
are 5 units and 4 units respectively
primary and secondary coils of an induction coil (b) the intensities at the screen due to the two
is 5H and a current of 10A is cut off in 5×10–4 slits are 4 units and 1 units respectively
second, the emf induced(in volt) in the secondary (c) the amplitude ratio is 7
coil is (d) the amplitude ratio is 6
(a) 5×104 (b) 1×105 40. Rising and setting sun appears to be reddish
(c) 25×10 5 (d) 5×106 because
(a) Diffraction sends red rays to earth at these
36. A voltage of peak value 283 V and varying
times
frequency is applied to a series L, C, R
(b) Scattering due to dust particles and air
combination in which R = 3 ohm, L=25 mH and molecules are responsible
C= 400 F. The frequency (in Hz) of the source at (c) Refraction is responsible
which maximum power is dissipated in the above (d) Polarization is responsible
is
(a) 51.5 (b) 50.7 PART - II (CHEMISTRY)
(c) 51.1 (d) 50.3
37. Four independent waves are represented by 41. The catalyst used in Rosenmund reaction is
equations (a) Zn / Hg (b) Pd / BaSO4
(1) X1 = a1sin t (3) X2 = a2sin2 t (c) Raney Ni (d) Na in Ethanol
(2) X3 = a1sin t (4) X4 = a1sin( t+ ) H 2O
42. CH 2 CO 2
O RMgX ?
Interferen ce is possible between waves
represented by equations (a) ROOC(CH2)COOR
(b) RCOCH2CH2COOH
(a) 3 and 4 (b) 1 and 2 (c) RCOOR
(c) 2 and 3 (d) 1 and 4 (d) RCOOH
EBD_7443
www.jeeneetbooks.in

WWW.IIT-NEET.XYZ

2007-6 Target VITEEE


43. Identify, which of the below does not possess 51. In TeCl4 the central atom tellurium involves
any element of symmetry? (a) sp3 hybridization
(a) (+)– Tartaric acid (b) sp3d hybridization
(b) Carbon tetrachloride (c) sp3d2 hybridization
(c) Methane (d) dsp2 hybridization
(d) Mesotartatric acid 52. The purple colour of KMnO4 is due to the
transition
44. The weakest acid amongst the following is
(a) C.T. (L M) (b) C.T. (M L)
(a) ClCH2COOH (b) HCOOH
(c) d – d (d) p –d
(c) FCH2CH2COOH (d) CH2(I)COOH
53. A nuclear reaction of 235 with a neutron
92 U
45. HOOC CH 2 4 COOH 2C 2 H5OH
H2SO 4 produces 90
36 Kr and two neutrons. Other element
C2 H5OOC CH 2 COOC2 H 5
Toluene produced in this reaction is
The purpose of using toluene here is (a) 137 (b) 144
52Te 55 Cs
(a) to make both substances (acid & alcohol)
miscible (c) 137 (d) 144
56 Ba 56 Ba
(b) that the product is insoluble in toluene 54. AgCl dissolves in a solution of NH3 but not in
(c) that the reactants are insoluble in water water because
(d) because of the formation of low boiling (a) NH3 is a better solvent than H2O
azotrope (b) Ag+ forms a complex ion with NH3
46. Trans esterification is the process of (c) NH3 is a stronger base than H2O
(a) conversion of an aliphatic acid to ester (d) the dipole moment of water is higher than
(b) conversion of an aromatic acid to ester NH3
(c) conversion of one ester to another ester 55. Which of the following is hexadenate ligand?
(d) conversion of an ester into its components (a) Ethylene diamine
namely acid and alcohol (b) Ethylene diamine tetra acetic acid
(c) 1, 10- phenanthroline
47. The correct sequence of base strengths in
(d) Acetyl acetonato
aqueous solution is
56. A coordinate bond is a dative covalent bond.
(a) (CH3)2NH > CH3NH2>(CH3) 3N
Which of the below is true?
(b) (CH3)3N > (CH3)2 NH>CH3NH2 (a) Three atoms form bond by sharing their
(c) (CH3)3N > CH3 NH2= (CH3 )2 NH electrons
(d) (CH3)2N H > (CH3 )3 N> CH3 NH2 (b) Two atoms form bond by sharing their
48. When aqueous solution of benzene electrons
diazoniumchloride is boiled, the product formed (c) Two atoms form bond and one of them
is provides both electrons
(a) C6H5CH2OH (b) C6H6+N2 (d) Two atoms form bond by sharing electrons
(c) C6H5COOH (d) C6H5OH obtained from third atom
49. Carbylamine reaction is given by aliphatic 57. Which of the following complex has zero
(a) primary amine magnetic moment (spin only)?
(b) secondary amine (a) [Ni(NH3 )6 ]Cl2 (b) Na 3[FeF6 ]
(c) tertiary amine (c) [Cr(H 2O)6 ]SO4 (d) K 4 [Fe(CN)6 ]
(d) quaternary ammonium salt
58. The IUPAC name of [Ni(PPh3)2Cl2]2+ is
NH3 (a) bis dichloro (triphenylphosphine) nickel (II)
50. C6 H5 CHO ?
H 2 ,Ni (b) dichloro bis (triphenylphosphine) nickel (II)
(a) C6H5NH2 (b) C6H5NHCH3 (c) dichloro triphenylphosphine nickel (II)
(c) C6H5 CH2NH2 (d) C6H5NHC6H5 (d) triphenyl phosphine nickel (II) dichloride
www.jeeneetbooks.in

WWW.IIT-NEET.XYZ

Solved Paper 2007 2007-7

59. Among the following the compound that is both 67. Given the equilibrium system:
paramagnetic and coloured is NH4Cl (s) NH4+(aq) + Cl–(aq)
(a) K2Cr2O7 (b) (NH4)2 [TiCl6] ( H = +3.5kcal/mol).
(c) VOSO4 (d) K3Cu (CN)4 What change will shift the equilibrium to the
60. On an X-ray diffraction photograph the intensity right?
of the spots depends on (a) Decreasing the temperature
(a) neutron density of the atoms/ions (b) Increasing the temperature
(b) electron density of the atoms/ions (c) Dissolving NaCl crystals in the equilibrium
(c) proton density of the atoms/ions mixture
(d) photon density of the atoms/ions (d) Dissolving NH 4NO 3 crystals in the
equilibrium mixture
61. An ion leaves its regular site occupy a position
68. According to Arrhenius equation, the rate
in the space between the lattice sites is called
constant (k) is related to temperature (T) as
(a) Frenkel defect (b) Schottky defect
(c) Impurity defect (d) Vacancy defect k2 Ea 1 1
(a) In –
62. The 8:8 type of packing is present in k1 R T1 T2
(a) MgF2 (b) CsCl
(c) KCl (d) NaCl k2 Ea 1 1
(b) In – –
63. When a solid melts reversibly k1 R T1 T2
(a) H decreases (b) G increases
(c) E decreases (d) S increases k2 Ea 1 1
(c) In
64. Enthalpy is equal to k1 R T1 T2

G G/T
(a) –T 2 (b) –T 2 In
k2

Ea 1 1
T T (d) k1 R T1 T2
V P

G/T
69. Equivalent amounts of H2 and I2 are heated in a
G
(c) T2 (d) T 2 closed vessel till equilibrium is obtained. If 80%
T V T of the hydrogen can be converted to HI, the Kc
P
65. Condition for spontaneity in an isothermal at this temperature is
process is (a) 64 (b) 16
(c) 0.25 (d) 4
(a) A W 0 (b) G U 0
70. For the reaction H2(g)+I2(g) 2HI(g), the
(c) A U 0 (d) G–U 0 equilibrium constant Kp changes with
66. Given: 2C s 2O 2 g 2CO2 g ; (a) total pressure
(b) catalyst
H –787kJ (c) the amount H2 and I2
(d) temperature
1 71. How long (in hours) must a current of 5.0 amperes
H2 g O2 g H 2O( ); H –286kJ
2 be maintained to electroplate 60g of calcium from
molten CaCl2?
1
C2 H 2 g 2 O2 g 2CO 2 g H 2O( ) (a) 27 hours (b) 8.3 hours
2 (c) 11 hours (d) 16 hours
; H –1310kJ 72. For strong electrolytes the plot of molar
The heat of formation of acetylene is conductance vs C is
(a) –1802 kJ (b) +1802 kJ (a) parabolic (b) linear
(c) +237 kJ (d) –800 kJ (c) sinusoidal (d) circular
EBD_7443
www.jeeneetbooks.in

WWW.IIT-NEET.XYZ

2007-8 Target VITEEE


73. If the molar conductance values of Ca2+ and Cl– 80. In which of the below reaction do we find , -
at infinite dilution are respectively 118.88×10–4 unsaturated carbonyl compounds undergoing
m2 mho mol–1 and 77.33×10–4 m2 mho mol–1 then a ring closure reaction with conjugated dienes?
that of CaCl2 is (in m2 mho mol–1) (a) Perkin reaction
(a) 118.88 × 10–4 (b) 154.66 × 10–4 (b) Diels-Alder reaction
(c) 273.54 × 10–4 (d) 196.21× 10–4
(c) Claisen rearrangement
74. The standard reduction potentials at 298K for
(d) Hoffman reaction
the following half reactions are given against
each
Zn2+ (aq) + 2e– Zn(s) E0 = – 0.762 V PART - III (MATHEMATICS)
Cr3+ (aq) + 3e– Cr(s) E0 = – 0.740 V
2H+ (aq) + 2e– H2(g) E0 = 0.00 V
Fe3+ (aq) + 3e– Fe2+(aq) E0 = + 0.762 V 81. Let the pairs a , b and c , d each determine a
The strongest reducing agent is plane. Then the planes are parallel if
(a) Zn(s) (b) Cr(s)
(c) H2(g) (d) Fe2+(aq) (a) a c b d 0
75. The epoxide ring consists of which of the
following ? (b) a c b d 0
(a) Three membered ring with two carbon and
one oxygen (c) a b c d 0
(b) Four membered ring with three carbon and
one oxygen a b c d 0
(d)
(c) Five membered ring with four carbon and
one oxygen
82. The area of a parallelogram with 3iˆ ˆj – 2kˆ and
(d) Six membered ring with five carbon and
one oxygen ˆi – 3jˆ 4kˆ as diagonals is
76. In the Grignard reaction, which metal forms an
organometallic bond? (a) 72 (b) 73
(a) Sodium (b) Titanium
(c) Magnesium (d) Palladium (c) 74 (d) 75
77. Phenol is less acidic than 83. If cosx + cos2 x=1then the value of
(a) p-chlorophenol sin12 x+ 3sin10 x + 3sin8x + sin 6 x–1 is equal to
(b) p-nitrophenol (a) 2 (b) 1
(c) p-methoxyphenol (c) –1 (d) 0
(d) ethanol
84. The product of all values of (cos +i sin )3/5 is
78. Aldol condensation is given by
equal to
(a) trimethylacetaldehyde
(b) acetaldehyde (a) 1 (b) cos +i sin
(c) benzaldehyde (c) cos3 +i sin3 (d) cos5 +i sin5
(d) formaldehyde 2
79. Give the IUPAC name for 1 i
85. The imaginary part of is
O O i 2i –1
|| ||
H3C CH 2 C H 2 C CH 2 C OCH3 4
(a) (b) 0
(a) Ethyl-4-oxoheptonate 5
(b) Methyl-4-oxoheptonate
(c) Ethyl-4-oxohexonate 2 4
(c) (d) –
(d) Methyl-4-oxohexonate 5 5
www.jeeneetbooks.in

WWW.IIT-NEET.XYZ

Solved Paper 2007 2007-9

u u
86. If sin –1 x sin –1 y , then cos–1x + cos–1y is 93. If (x +y )sin u = x2y2, then x y
2 x y
equal to
(a) sin u (b) cosec u
(a) (b) (c) 2 tan u (d) tan u
2 4
94. The angle between the tangents at those points
3 on the curve x = t2 + 1 and y = t2 – t – 6 where it
(c) (d)
4 meets x-axis is
87. The equation of a directrix of the
4 5
x 2 y2 (a) tan –1 (b) tan –1
ellipse 1 is 29 49
16 25
(a) 3y = 5 (b) y = 5 10 8
(c) 3y = 25 (d) y = 3 (c) tan –1 (d) tan –1
49 29
88. If the normal at (ap2, 2ap) on the parabola
y2 = 4ax, meets the parabola again at (aq2, 2aq),
then 4
(a) p2 + pq + 2 = 0 (b) p2 – pq + 2 = 0 95. The value of x – 3 dx is equal to
(c) q2 + pq + 2 = 0 (d) p2 + pq + 1 = 0 1
89. The length of the straight line x – 3y = 1
intercepted by the hyperbola x2 – 4y2 = 1is 5
(a) 2 (b)
6 2
(a) 10 (b)
5
1 3
(c) (d)
1 6 2 2
(c) (d) 10
10 5 96. The area of the region bounded by the straight
90. The curve described parametrically by lines x = 0 and x = 2 and the curves y = 2x and
x = t2 + 2t–1, y = 3t + 5 represents y = 2x –x2 is equal to
(a) an ellipse (b) a hyperbola
(c) a parabola (d) a circle 2 4 3 4
91. If the normal to the curve y = f(x) at (3,4) makes (a) – (b) –
log 2 3 log 2 3
3
an angle with the positive x-axis, then f '(3)
4 1 4 4 3
(c) – (d) –
is equal to log 2 3 log 2 2
3
(a) – 1 (b)
4 dx
97. The value of 7
is equal to
3 0 a2 x2
(c) 1 (d) –
4
92. The function f(x) = x2 e–2x, x > 0. Then the
maximum value of f(x) is 231 1 235 1
(a) (b)
1 1 2047 a13 2048 a13
(a) (b)
e 2e
232 1 231 1
1 4 (c) (d)
(c) (d) 2047 a13 2048 a13
e2 e4
EBD_7443
www.jeeneetbooks.in

WWW.IIT-NEET.XYZ

2007-10 Target VITEEE


101. The general solution of the differential equation
2
x 1– x d2 y dy
98. The value of the integral e 2
dx is 2 y 2e3x is given by
1 x dx 2 dx
e3x
ex
1– x
C (a) y c1 c 2 x e x
(a) 8
1 x2
e –3x
(b) y c1 c 2 x e – x
1 x 8
(b) ex C
1 x2 e3x
(c) y c1 c2 x e – x
8
ex e –3x
(c) C y c1 c2 x e x
1 x 2 (d)
8
(d) ex(1–x) + C 102. The solution of the differential equation
ydx+(x–y3)dy = 0 is
y y 1 3
99. If x sin dy y sin – x dx (a) xy y C (b) xy = y4 + C
x x 3
(c) y4 = 4xy + C (d) 4y = y3 + C
103. The number of positive integral solutions of the
y equation x1 x2x3x4x5=1050 is
and y 1 , then the value of cos is
2 x (a) 1870 (b) 1875
(c) 1865 (d) 1880
equal to 104. Let A= {1,2,3,....., n}and B ={a,b,c}, then the
number of functions from A to B that are onto is
1 (a) 3n – 2n (b) 3n – 2n –1
(a) x (b) n
x (c) 3 (2 – 1) (d) 3n – 3(2n – 1)
105. Everybody in a room shakes hands with
(c) log x (d) ex everybody else. The total number of hand
100. The differential equation of the system of all shakes is 66. The total number of persons in the
room is
circles of radius r in the XY plane is (a) 9 (b) 12
(c) 10 (d) 14
2 2
dy
3
d2 y 106. If(G,*) is a group and the order of an element
(a) 1 r2 a G is 10, then the order of the inverse of a* a
dx dx 2 is
1
(a) 10 (b)
3 2 3 10
dy 2 d2 y
(b) 1 r 1
dx dx 2 (c) 5 (d)
5
107. A box contains 9 tickets numbered 1 to 9
inclusive. If 3 tickets are drawn from the box one
2 3 2
dy 2 d2 y at a time, the probability that they are
(c) 1 r alternatively either {odd, even, odd} or {even,
dx dx 2 odd, even} is
5 4
(a) (b)
2 3 3 17 17
dy d2 y
(d) 1 r2 5 5
dx dx 2 (c) (d)
16 18
www.jeeneetbooks.in

WWW.IIT-NEET.XYZ

Solved Paper 2007 2007-11

1 5 1 –1 2 5
108. If P(A) = , P(B) = and P(B/A)= then
12 12 15 114. If the rank of the matrix 2 –4 a – 4 is 1,
p A B is equal to 1 –2 a 1

89 90 then the value of a is


(a) (b) (a) –1 (b) 2
180 180
(c) –6 (d) 4
91 92 115. If b2 4ac for the equation ax4 + bx2 + c = 0, then
(c) (d) all the roots of the equation will be real if
180 180
109. If the probability density function of a random (a) b>0, a<0, c>0 (b) b<0, a>0, c>0

x (c) b>0, a>0, c>0 (d) b>0, a>0, c<0


variable X is f(x) = in 0 x 2, then
2
116. If x > 0 and log3x + log3( x ) + log3 ( 4 x ) +
P(X > 1.5 X > 1) is equal to
8 16
7 3 log3 x + log3 x +....= 4, then x equals
(a) (b)
16 4 (a) 9 (b) 81
7 21 (c) 1 (d) 27
(c) (d) 117. The number of real roots of the equation
12 64
3
110. If X is a poisson variate such that 1 1
2P(X = 0)+ P(X = 2 ) = 2P(X = 1) then E(X) is x x 0 is
x x
equal to
(a) 0 (b) 2
(a) 1 (b) 2
(c) 4 (d) 6
(c) 1.5 (d) 1.75
1 tan 118. If H is the harmonic mean between P and Q, then
111. If A and AB = I, then
– tan 1 H H
the value of is
(cos2 )B is equal to P Q

(a) A( ) (b) A PQ
2 (a) 2 (b)
P Q
– 1 P Q
(c) A(– ) (d) A (c) (d)
2 2 PQ

2x 1 4 8 119. If a and b are two unit vectors, then the vector


112. If x = –5 is a root of 2 2x 2 = 0, then ( a + b ) × ( a × b ) is parallel to the vector
7 6 2x
(a) a +b (b) 2 a + b
the other roots are
(c) a – b (d) 2 a – b
(a) 3, 3.5 (b) 1, 3.5
120. If is the angle between the lines AB and AC
(c) 1, 7 (d) 2, 7
where A, B and C are the three points with
113. The simultaneous equations Kx + 2y–z =1,
coordinates (1,2,–1), (2,0,3), (3,–1,2) respectively,
(K –1)y–2z = 2 and (K + 2)z = 3 have only one
solution when then 462 cos is equal to
(a) K = –2 (b) K = –1 (a) 20 (b) 10
(c) K = 0 (d) K = 1 (c) 30 (d) 40
EBD_7443
www.jeeneetbooks.in

WWW.IIT-NEET.XYZ

2007-12 Target VITEEE

2007 SOLUTIONS

PART - I (PHYSICS) 3 3 36
2 Rc f 1.35f .
16 16 5
1. (d) The magnetic moment of the ground state 5. (d) Here, Kinetic energy = Rest energy
of an atom whose open sub-shell is half we know that
filled with n electrons is given by Kinetic energy (Relativistic)
= (m – m0)c2,
n(n 2). B and Rest energy = m0c2,
where B is the gyromagnetic moment of where m0 = rest mass, c = velocity of light
the atom. Also, mass (m) of a particle moving with
Here, n = 5. velocity v is given by
5(5 2). B 35 B m0
m .
2. (a) Bragg's law gives 2dsin = n , n= order of v2
reflection, d= distance between planes. For 1
c2
same and d, n for a given , smallest
d for least n, can be found. If crystal is (m – m0)c2 = m0c2 provides
symmetric reflections from different planes
may cancel out.
3. (b) According to Moseley's law, square root
of frequency of X-ray is plotted against m0
m0 c 2 m 0c2
atomic number it gives straight line, the v 2
relation is 1
c2
f c Z – 1 where c = constant
1
(for f 0, Z c , for Z = 0, f –c ) or, 1 1
Option (b) is correct. as Z can not be v2
1
negative. c2
4. (b) Balmer series is given for n 1 = 2 and
n2 = 3, 4, ... v2 1
1 1
or, 2 or, 2
1 1 c 4
Rc – v2
1
22 n 22 c2
For Ist line in spectrum n2 = 3
v2 1 3
or, 2
1
1 1 1 1 c 4 4
1 Rc – Rc – f
2 2 4 9
2 n2 3
v c.
5 36 2
f Rc Rc f
36 5 v2 1
1– 2
=
For second line n 2 = 4 c 4
1 1 1 1 v2 1 3 3 2
Rc – Rc –
2
2 2
4 2 4 16 2 =1 = v2 = c
c 4 4 4
4 –1 3
Rc v= c
16 2
www.jeeneetbooks.in

WWW.IIT-NEET.XYZ

Solved Paper 2007 2007-13

6. (d) de-Broglie wavelength ( ) of a particle of 10. (c) Radio carbon dating is done by measuring
mass m and moving with a velocity v is ratio of 14C present in the sample, since
proportion of 14C and 12 C in a body is same;
h
given by, , where h is Planck's but after death 14C decays. Hence knowing
mv the present ratio of 14C to 12 C, sample can
constant. be dated.
When a particle having charge q is 11. (b) particles are positively charged He
accelerated through a potential V then nucleus, it can accept 2e– , rays are
1 negatively charged which are similar to e–,
qV mv 2 can donate 1e–, are radiations. Hence
2
ionisation power of is maximum. are
m 2 v2 most energetic and is least energetic
or, qV mv 2mqV Penetration power of is maximum
2m
12. (d) Given half life T= 3.8 days; t = 19 days
h N t 19
2mqV ? now 5
N0 T 3.8
Hence, de-Broglie wavelength of electron,
t
5
h N 1 T 1 1
e
2me eV N0 2 2 32

and de-Broglie wavelength of proton, N0


N 0.03 N 0
h 32
p
, where e represents the
2m p eV 13. (a) If the p-n junctions are identical, then their
resistances would be same. In circuit 1, the
charge of electron (or proton) first p-n function is forward biased and
second is reverse biased. Hence the
mp
e voltages across them would be different.
p me In circuit 2 both are reverse biased. So
potential drop would be same. In circuit 3
7. (c) Relative speed is given by
both are forward biased; again voltages
u' v would be same.
u 14. (a) Zener diode is a semiconductor device and
u 'v
1 for semiconductors, temperature coefficient
c2
is negative
Here, u' = 0.8 c and v = 0.4 c 15. (a) The truth table corresponds to the logic
0.8c 0.4c 1.2 c A B hence it is NOR gate.
u 0.9c.
(0.8c)(0.4c) 1.32
1 16. (d) A.B in Boolean algebra, can be written as
c2
8. (d) By Einstein's equation, photoemission A.B A B A B
occurs when h > 0 or h > h 0 i.e., 17. (a) By Dopplers effect, if source moves
frequency of incident photon is greater than towards the observer, frequency received
threshold frequency. will increase.

9. (c) Radius of nucleus is given by R A1/ 3 1


As
R R 0 A1/ 3 , where A = mass number
wavelength will decrease.
EBD_7443
www.jeeneetbooks.in

WWW.IIT-NEET.XYZ

2007-14 Target VITEEE


18. (d) A coaxial cable consists of a conducting
wire surrounded by a dielectric space, over AC 42 32 25 5cm.
which there is a sleeve of coppermesh Potential at A = VB + VC + VD.
covered with a shield of PVC insulation.
The power transmission is regulated by 1 q1 q2 q3
VA
dielectric. At high frequencies energy loss 4 0 AB AC AD
due to mesh is significant (called skin
effect). 10 10 –12 –20 10 –12 10 10 –12
VA 9 109
–2 –2
19. (b) Due to their inherent distortion, plate- 4 10 5 10 3 10 –2
modulated class C amplifiers are not used
as audio amplifiers. Also, class A amplifiers 10 –20 10
are not used owing to low efficiency. A grid- 9 109 10 –10
4 5 3
modulated amplifier has very high
frequency. Therefore, in output stage of a 150 – 240 200
TV transmitter, grid-modulated class C 0.9
60
amplifier is used.
20. (b) The rms value of carrier current is Ic = 8A. 0.9 110
The rms value of modulated current 1.65V
60
= Ic + Im = 8.93 A = It
Percentage modulation = ma × 100 23. (d) The capacitance (C) of a parallel plate
The current relation in AM wave is capacitor with dielectric is given by
A
It 2 ma 2 It2 C 0
1 ma –1 2 1
Ic2 2 Ic2 d – t 1–
K

8.93
2 C = 100 pF = 100 × 10–12 F, = 8.85 × 10–12,
Modulation index, m a –1 2 A = r2 = 3.14×(10–2)2 (r = lcm)
82
t = 10–3m, d = t, K = 4
79.7
–1 2 1.24 –1 2
64 8.85 10 –12 3.14 10 –4
C1
= 0.701 1
10 –3 –10 –3 (1 – )
Percentage modulation = ma × 100 =70.1% 4
1
21. (d) For a point charge, E . For positive 27.79 10 –16
x2
10 –3 – 0.75 10 –3
charges, electric field will decrease in
positive direction as distance increases,
27.79 10 –13
(case 1), for negative charge, as distance
increases field will increase (case 4). As we 0.25
move from positive charge to negative
charge, field will keep on decreasing (case 27.79 10 –13
111.16 10 –13 F
2), As we move from negative to positive 0.25
charge, field will keep on increasing (case 3) (for 1 set)
A 4 cm B(q 1) Required C = 100 10 –12
F
22. (a) 3 cm 3 cm
C 100 10 –12
n 10
D(q 3 ) 4 cm C(q 2) C1 111.16 10 –13
www.jeeneetbooks.in

WWW.IIT-NEET.XYZ

Solved Paper 2007 2007-15

24. (a) The resultant intensity at a point on 26. (c) Since conductor and semiconductor are
connected in parallel hence voltage across
equatorial line is E . E is parallel and them is same. If ammeters show same
opposite to direction of p . V
reading hence their resistances R are
I
E2 same. If voltage is increased by small value
E 3 cm
then following the same relation V I for
E1 constant R, both conductor and
semiconductor show same current.
27. (d) Time taken by free electrons to cross the
–q +p
O conductor
I
25. (d) Let a total current of 6I enter at A. It divides t where drift velocity v d
vd neA
into three equal parts, each of 2I, along AE,
AB and AD. At E, B and D each the current
1
2I divides into two equal parts, each of I, vd 28
along EF, EH, BF, BC, DH and DC. 8 10 1.6 10 –19 5 10 –7
At F, H and C, the two currents each of I, 10 –2
combine together to give a current of 2I at m/s .
64
each corner. Thus, at G we get the same
1 64
current 6I as shown in the figure. t 64 102 s 6.4 103 sec
vd 10–2
E I F
2I
R 2 – R1
I I 28. (b) Temperature coefficient,
A B 2I R1 t 2 – t1
2I I 2I G 6I
2I H
R 2 – R1
6I
I 2I t 2 – t1
D I C R1

2 –1
E t 2 – t1
1 0.00125
Let r be the value of resistance of each arm 1
of the cube and R be the joint resistance t2 t1 800 300 1100K
0.00125
across the conners A and G.
29. (d) Torque on the coil is = nIB A cos . If coil
Applying Kirchhoff's law along the loop is set with its plane parallel to direction of
AEFGA, we get magnetic field B, then =0°, cos = 1
2Ir + Ir + 2Ir = E = nIBA.1 = nIBA = maximum.
or, 5Ir = E ........... (1) Hence, I = maximum (as n,B, A are constant)
Also, by Ohm's law, 1 2
6I × R = E 30. (d) E at
bt is the parabolic equation for
2
6IR = 5Ir, using (1) thermo emf. The thermoelectric power is
5 dE
or, R r. S
6 dt
Here, r = 6 dE
S = a + bt. The graph between S
5 dt
R 6 5
6 and straight line.
EBD_7443
www.jeeneetbooks.in

WWW.IIT-NEET.XYZ

2007-16 Target VITEEE


When t = 0, S = a (intercept).
1 1
dE –6
At neutral temperature 0 and 2 5 80 10 2 400 10 –6
dt
1
–a
t = tn 0 = a + btn tn and at cold 2 20 10 –3
b

–2a 103 1000 25


junction t i 2t n
b 40 40
31. (d) Energy of proton = K.E. = 1MeV = 106 eV. 35. (b) M = 5 H, I = 10 A, t = 5×10–4 s.
Now, emf induced in secondary is
Bq
Frequency
2 m dI
e –M , (–ve sign shows direction)
dt
6.28 10 –4 1.6 10 –19
2 3.14 1.7 10 –27 10
e 5 –4
1 10 5 V
5 10
0.9 10 4 Hz 10 4 Hz
36. (d) Average power of an LCR circuit is
32. (d) Force on a conductor of length due to a P = EvIvcos . Maximum power is dissipated
magnetic field of strength B is given by at resonance when XL =XC. The resonance
frequency is given by
F I( B)
1
F I B sin
2 LC
Here, = 90°
F I B I R B RIB 1
Therefore, a force of magnitude ( RIB) will 2 3.14 25 10 –3 400 10 –6
act on each wire. The direction of the forces
on each wire will be same. Thus, 1
total force on the wire AB = RIB + RIB 2 3.14 5 20 10 –5 10
= 2 RIB
33. (d) In first case, the two inductances are in 105
series hence total inductance = L0 + L0 3.14 200 10
= 2L0
In second case, the current is same, but no. 103 159.2
50.31 Hz
of turns has doubled since the sense of 6.28 10 3.16
turning is same.
37. (d) Coherent sources sh ould have same
2 2 frequency and wavelength and constant
Thus L n L 2n
or zero phase difference. Frequency is same
hence inductance L = 4L0
only for wave X1 and X4 and
In third case, the sense of turns is opposite.
phase difference =
So net inductance cancel each other
38. (c) Since position of central maximum is same,
L= 0
hence asin = is same for both
34. (b) At resonance, impedance of circuit is
wavelengths. i.e, a = constant, = same,
minimum.When impedance of capacitor and
is same for both waves
inductor is same, XL = XC Resonance
1 1
frequency and 1
2 LC 2
www.jeeneetbooks.in

WWW.IIT-NEET.XYZ

Solved Paper 2007 2007-17

39. (b) For bright fringes, I max = (a + b)2 43. (a) (+) – Tartaric acid does not have element of
For dark fringes, Imin = (a – b)2 symmetry.

COOH COOH COOH


2 | | |
Im ax a b a b
Now 9 3 H C OH HO C H H C OH
Imin 2 a–b
a–b | | |
HO C H H C OH H C OH
| | |
a + b = 3(a – b) a + b = 3a–3b 2a = 4b COOH COOH COOH

a a2 (+) – Tartaric acid (A) Meso tartaric acid (D)


2 2
4.
b b (Plane of symmetry)

Ratio of intensities of the two slits, Cl H


| |
I1 a2 4 Cl C Cl H C H
2 | |
I2 b 1 Cl H
Carbon tetracholride (C)
40. (b) Rising and setting sun appears red because (B) Methane
light from the sun travels slightly more
distance from the horizon, than when it is 44. (b) Proton doner s are acids. Electr on
overhead. Hence blue light is scattered by withdrawing groups (like halogens)
dust in the atmosphere because scattering increase the acidity of carboxylic acids.
Therefore, HCOOH is weakest acid among
1 the given choices.
and b < r. Red colour is less
4 45. (a) Toluene is used in the reaction to make both
substances (acid and alcohol) miscible.
scattered and reaches us. 46. (c) Trans esterification is the process of
conversion of one ester to another ester.
PART - II (CHEMISTRY) RCOOR' + R''OH RCOOR'' + R'OH
47. (a) It is expected that the basic nature of
41. (b) Roseumund's reaction – amines should be in order tertiary >
secondary > primary but the observed
Pd / BaSO4
RCOCl H 2 RCHO HCl order in the case of lower members is found
to be as secondary > Primary > tetriary. This
CH2CO anomalous behaviour of tetriary amines is
42. (b) O + RMg X
CH2CO due to steric factors i.e crowding of alkyl
groups cover nitrogen atom from all sides
thus makes the approach and bonding by a
O Mg X R proton relatively difficult which results the
maximum steric strain in tetiary amines. The
CH2 C H2O electrons are there but the path is blocked,
O
CH2 C – Mg(OH) X resulting the reduction in basicity.
Thus the correct order is
O R2 NH > R NH2 > R3N.
Boiling
R C CH2 CH2 COOH 48. (d) C6H5N2+Cl– + H2O
C6H5OH + N2 + HCl
O
Phenol
EBD_7443
www.jeeneetbooks.in

WWW.IIT-NEET.XYZ

2007-18 Target VITEEE


49. (a) Carbylamine test is given by aliphatic 235 144 92 1
1
primary amine. 53. (d) 92U n 56 BaKr 2 0n
0 36
Aliphatic (or aromatic) 1° amine + CHCl3 + Sum of atomic number of reactants = sum
KOH Bad smelling isocyanide. of atomic masses of products.
54. (b) AgCl dissolves in a solution of NH3 but
50. (c) Benzaldehyde reacts with ammonia to form
not in water because Ag+ forms soluble
hydrobenzamide.
complex ion with NH3.
C6H5 CH = O H2 NH 55. (b) EDTA is hexadentate ligand. 4 oxygen and
+ + O = CH C6H5 2 nitrogen atoms act as donars.
C6H5 CH = O H2 NH – OOCH C
2
CH2COO–
C 6H 5 CH = N
CH C 6H5 – OOCH C CH2COO–
2
C 6H 5 CH = N
56. (c) A co-ordinate bond is a dative covalent
Hydrobenzamide.
bond in which two atoms form a bond and
Under one of them provides both electrons.
H2, Ni
Pressure 57. (d) [Ni(NH3)6]Cl2 sp3d2 hybridisation
2 C6H5 CH2 NH2 2 unpaired electrons
Na3[FeF6] sp3d2 hybridisation
1 3 unpaired electrons
51. (b) Hybridisation [Number of valence
2 [Cr(H2O)6]SO4 d2sp3 hybridisation
electrons of central atom + no. of 3 unpaired electrons
monovalent atoms attached to it + negative K4[Fe(CN)6] d2sp3 hybridisation
charge if any – positive charge if any] No unpaired electrons
Zero magnetic momoment means all the
1
[6 + 4 + 0 – 0] = 5 = sp3 d electrons paired.
2
58. (c) The IUPAC name is dichloro triphenyl
phosphine nickel (II).
Te 52 Kr 4d10 5s 2 5p4
59. (c) (d) In K4[Fe(CN)6] Cu is in + 1 oxidation
52. (a) The permanganate ion has an intense state hence has no unpaired electron
purple colour. Mn (+ VII) has a d 0 hence colourless and diamagnetic.
configuration. So the colour arises from (b) In (NH4)2 [TiCl6] Ti is in + 4 oxidation
charge transfer and not from d—d spectra. state. hence has no unpaired electron
In MnO4 – an electron is momentarily hence colourless and diamagnetic.
changing O– – to O– and reducing the (c) In VOSO4, V is in +4 oxidation state
oxidation state of the metal from Mn(VII) to hence has one unpaired electron, thus
Mn (VI). Charge transfer requires that the it is coloured and paramagnetic.
energy levels on the two different atoms (a) In K2Cr2O7, Cr is in +6 oxidation. hence
are fairly close. has no unpaired electron and thus it
is diamagnetic. Though K2Cr2O7 has
O = (8) = 2, 6 no unpaired electron but it is coloured.
K L
This is due to charge transfer
Mn (25) = 2 , 8, 15 spectrum.
K L M
60. (b) On an X-ray diffraction photograph the
hence the charge transfer occurs from
intensity of the spots depends on electron
L M. density of atoms/ions.
www.jeeneetbooks.in

WWW.IIT-NEET.XYZ

Solved Paper 2007 2007-19

61. (a) In Frankels defect an ion leaves its regular 65. (a) Since G A P. V
site and occupy a position in the space For a spontaneous process G should be
between the lattice sites.
negative which is possible only if
+ – + –
A P. V or A W 0 .
A B A B 66. (c) We have to find
+
A
– – +
B B A 2C(s) H 2(g) C2 H2(g) H ?
This is the equation for formation of
+ – + –
A B A B acelytene Given
– + – + 2C (s) 2O 2(g) 2CO 2(g) ; H 787 kJ
B A B A
....(1)
62. (b) 8 : 8 type of packing is present in CsCl. 1
H 2(g) O2(g) H 2 O( ) ; H 286 kJ
6 : 6 type of packing is present in NaCl and 2
KCl. ...(2)
63. (d) When solid melts S increases. because 1
when solid changes into liquid randomness C2 H 2(g) 2 O2(g)
2
increases.
64. (d) Gibbs Helmholtz Equation– 2CO 2(g) H 2 O( ) ;
G H – T S .......(1)
H 1310 kJ ....(3)
differentiate this equation w.r.t. temperature
at constant pressure Add eq.(1) and (2)
1
G Gy Gx 2C(s) H 2(g) 2 O 2(g) 2CO 2(g)
– ....(2) 2
T P T T P
P H 2 O( ) ; H 1073kJ ......(4)
= – Sy – ( – Sx)
Subtract eq. (3) form eq. (4) we get
= – ( Sy – Sx) = S ......(3)
where S change in entropy 2C(s) H 2(g) C 2 H 2(g) ;
on combining equation (1) & (3) we get H 237 kJ
G 67. (b) Endothermic reactions are favoured at high
G H T ....(4) temperature. Therefore, increasing the
T temperature will shift equilibrium to the
P

Equation (4) is on alternative form of Gibbs right.


Helmholtz equation k2 Ea 1 1
Dividing equ. (4) by T2, we get 68. (a) ln
k1 2.303R T1 T2
G H 1 G
2 2 69. (a) H 2 I2 2HI
T T T T
P Initial moles 1 1 0
At equilibrium (1 x) (1 x) 2x
on rearrangement, we get
G H [HI]2 (2x) 2
– Kc =
T T2 (1 x)2
[H 2 ][I2 ]
P
Given x = 80% = 0.80
G
H –T 2 (2 0.80) 2
T Kc = 64
P (1 0.80) 2
EBD_7443
www.jeeneetbooks.in

WWW.IIT-NEET.XYZ

2007-20 Target VITEEE


70. (d) The equilibrium constant Kp will change 74. (a) Lower the value of reduction potential,
with temperature for the reaction greater will be the reducing power of
H2(g) + I2(g) 2HI(g). element.
Since Zn has lowest reduction potention
Catalyst does not alter the state of
hence Zn is the strongest reducing agent.
equilibrium.
75. (a) Th e epoxide ring consists of thr ee
Equilibrium constant depends only upon
membered ring with two carbon atoms and
temperature. The relation of k with temp
one oxygen.
can be shown as
O
k H 1 1
log 2 – CH2
k1 2.303R T1 T2
76. (c) Grignard reagent is a sigma bonded
Mol.mass 40 organometallic compound in which Mg is
71. (d) Eq. mass of Ca++ = 20
2 2 bonded with one alkyl and one halogen
group. This can be prepared as
Eq.mass of water 20
Z= R – X Mg
ether
R – Mg – X
96500 96500
Given w = 60 g, i = 5 amp. 77. (a,c) Presence of electron attracting group like –
w = zit NO2 , – Cl increases the acidity of phenol
as it enables the ring to draw more electrons
20
or 60 = 5 t from the phenoxy oxygen and thus
96500 releasing easily the proton. Presence of
96500 60 electron releasing group e.g. – OCH3 on
57900 sec = 16 h.
t= benzene sing decreases the acidity of
20 5
72. (b) According to Debye – Huckel – Onsagar phenol as it strengthens the negative charge
on phenoxy oxygen and thus proton release
o o
equation m m – A B m C becomes difficult.
Further phenols are much more acidic than
where A and B are the Debye – Huckel
alcohols. The acidic nature of phenol is due
constants. If we plot a graph between molar
to the formation of stable phenoxide ion in
conductance m against the square solution.

roots of the concentration C a straight C6 H5 OH H 2 O C6 H5O – H3O


No resonance is possible in alkoxide ions
line is obtained
(RO–) derived from alcohols. The negative
charge is localized on oxygen atom. Thus
alcohols are not acidic.
Molar Conductance

78. (b) Aldol condensation is given by aldehydes


Strong electrolyte which have -H atoms. So acetaldehyde
gives this reaction.
O O
|| ||
(CH 3 )3 C C H CH3 C H
C Trimethyl acetaldehyde
acetaldehyde
73. (c) Molar conductance of CaCl2
= Molar conductance of Ca2++ + 2 × (molar O O
conductance of Cl–) || ||
= 118.88 × 10 + 2 (77.33 × 10–4)
–4 C6 H 5 C H H C H
= 273.54 × 10–4 m2 mho mol–1 benzaldehyde formaldehyde
www.jeeneetbooks.in

WWW.IIT-NEET.XYZ

Solved Paper 2007 2007-21

O O 83. (d) Given : cos x + cos2x = 1


|| || cos x =1– cos2x cos x = sin 2 x
79. (d) H3C CH 2 C H 2 C CH 2 C OCH3 2
cos x = sin x4 1– sin2 x = sin 4x
6 5 4 3 2 1 4 2
sin x + sin x =1
Methyl 4 oxohexanoate
cubic both sides, we have
80. (b) It is cycloaddition reaction between a sin12x + sin6x + 3 sin6x (sin4 x + sin2x) =1
conjugated diene and subsituted alkene or sin12 x + sin6x + 3 sin10x + 3sin8x =1
, unsaturated carboxyl compound. sin12x + 3sin10x + 3 sin8x + sin6 x–1= 0
CH2 84. (c) (cos + i sin )3/5 = (cos 3 + i sin 3 )1/5
= [cos (2k + 3 ) + i sin (2k + 3 )]1/5
+ CHO
(acrolein – CHO 2k 3 2k 3
dienophile) = cos i sin ,
diene adduct 5 5
where k = 0, 1,2,3,4
PART - III (MATHEMATICS) Product of all values.
3 3 2 3
81. (c) Since a and b are coplanar, therefore = cos i sin . cos
5 5 5
a × b is a vector perpendicular to the plane 2 3 4 3 4 3
i sin . cos i sin
containing a and b . Similarly c × d is a 5 5 5
vector perpendicular to the plane
4 3 4 3 6 3 6 3
cos
containing c and d . Thus, the two planes i sin . cos i sin .
5 5 5 5
will be parallel if their normals i.e. a × b
8 3 8 3
and c × d are parallel. cos i sin
5 5
(a ×b )× (c ×d )=0
3 2 3 4 3 6 3 8 3
82. (d) Given : diagonals d1 = 3iˆ ˆj 2kˆ and = cos
5 5 5 5 5
d2 ˆi 3jˆ 4kˆ
3 2 3 4 3 6 3 8 3
i sin
5 15 5 5 5
Area of a parallelogram = d1 d 2
2
ˆi ˆj kˆ
Now, d1 × d 2 3 1 2 55 33 22
cos
= cos 2.2. 55 11. .
1 3 4 22 55 55

= ˆi(4 6) ˆj (12 2) kˆ ( 9 1) 55 33 22
i isin
sin 2.2. 55 11 ..
22 55 55
d1 × d 2 = 2iˆ 14ˆj 10kˆ
5 6 8 5 6 8
d1 d 2 = ( 2) 2 ( 14) 2 ( 10) 2 = cos . i sin
2 5 5 2 5 5
= 4 196 100 300 2 75
5 6 2 5 6 8
1 cos . i sin
Area of parallelogram = d1 d 2 2 5 5 2 5 5
2
= cos (3 + 4 ) + i sin(3 + 4 )
1 = cos (4 + 3 ) + i sin (4 + 3 )
= × 2 75 = 75 square units
2 = cos 3 + i sin 3
EBD_7443
www.jeeneetbooks.in

WWW.IIT-NEET.XYZ

2007-22 Target VITEEE

2
1 i 1 i2 2i
85. (d) 2 Y
i 2i 1 2i i y= a/e

directrix
1 1 2i 2i 2 i 4i 2i 2 4i 2 2 4i (0,5)
2 i 2 i 2 i 4 i2 4 1 5

[ i2 = –1]
F (0,c)
(–4,0) (4,0)
2i 2i 4i 2i 2 4i 2 2 4i 2 4i X
2 1 2 i 4 i 2 4 1 5 5 5 X'
F' (0, – c)
4
The imaginary part = –
5 (0,–5) y= – a/e

86. (a) Given sin –1x + sin –1y = (i) Y'


2

we know that sin–1x + cos–1x =


2 88. (a) Since the normal at (ap2, 2ap) on y2 = 4ax
sin–1 x = /2 – cos–1x meets the curve again at (aq2, 2aq), therefore
Equation (1) becomes. px + y = 2ap + ap3 passes through (aq2,2aq)
paq2 + 2aq = 2ap + ap3
– cos–1x + – cos–1y = p(q2–p2) = 2(p – q)
2 2 2 p (q + p) = –2
p2 + pq + 2 = 0
cos–1x + cos–1y = 89. (d) Given : equation of line, x–3y=1 (1)
2 and hyperbola x2 – 4y2 = 1 (2)
putting x = 1 + 3y in equation (2), we get
x2 y2 (1 + 3y)2 – 4y2 =1 1+ 9y2 + 6y – 4y2 = 1
87. (c) Equation of ellipse 1 , where
b2 a2 2
5y + 6y = 0 y(5y + 6) = 0
a > b. 6
y = 0 or y =
x2 y2 5
Given, 1 b = 4, a = 5
16 75 18 13
x = 1 for y = 0 & x =1 =
5 5
b2 16
But e = 1 2
1
a 25 for y = –6/5
the line (1) cuts the hyperbola (2) in at
3 most two point.
e= co-ordinates of points are P(1,0) &
5
13 6
a Q ,
equation of directrix y = 5 5
e

5 2 2
13 6
y= 3y = ± 25 PQ = 1 0
3/ 5 5 5
www.jeeneetbooks.in

WWW.IIT-NEET.XYZ

Solved Paper 2007 2007-23

2 U U sin U
18 36 x y tan U
324 36 360 x y cos U
5 25
25 25 U U
x y tan U
6 10 x y
length of straight line PQ = units.
5 94. (c) Equation of the given curve in parametric
90. (c) Given x = t2 + 2t–1 & y = 3t + 5 form,
x = t2 + 1 and y = t2 – t – 6
x = t2 + 2t + 1–2 & y = 3t + 3+2
Y-coordinate of the point, where the given
x = (t+1)2 –2 & y = 3(t + 1) + 2 (2)
curve meets X-axis is 0.
(t+1) = x 2 .......... (1) When y = 0, then t2 – t – 6 = 0
Equation (2) becomes [using equation (1)] t 2 3t 2t 6 0
y= 3 x 2 2 t(t 3) 2(t 3) 0
y–2 = 3 x 2 (t 3)(t 2) 0
squaring both sides, we get t 3 or 2
(y–2)2 = 9 (x + 2)
when t = 3, then x = 10
Y2 =9X where Y = y – 2 & X = x + 2
when t = –2, then x = 5
This equation represents a parabola Hence, the points where the curve meets
91. (c) Slope of the normal at (3,4) is the value of the X-axis are (10, 0) and (5, 0).
1 1 3 dy
at x = 3 or = tan =–1
f '(x) f '(3) 4 dy dt 2t 1
f`'(3) = 1 Now, dx dx 2t
92. (c) Given : f (x) = x2 e–2x, x > 0 dt
f '(x) = x2.e–2x(–2) + e–2x.2x Slope of the tangent at point (10, 0)
put f '(x) = 0 2e–2x. x (–x + 1) = 0 dy dy 5
x = 1 or x = 0 m1
dx x 10 dx t 3 6
f"(x) = (–4x2 – 6x + 1)e–2x
f"(1) = –9e–2x < 0 Slope of the tangent at point (5, 0),
f"(0) = e–2x > 0 dy dy 5 5
value of f(x) is maximum at x = 1 m2
dx x 5 dx t 2 4 4
1 If be the angle beween two tangents, then
f(x) = x2.e–2x f(1) = e–2 = 2
e 55
93. (d) Given : (x + y) sin U = x2y2 m 2 m1 64
tan
1 m1m 2 5 5
x 2 y2 1
sinU = = v (let) 6 4
x y
Here n = 2 – 1 = 1 15 10 5
12 12 10
v v
Euler's theorem x. y. nv 24 25 49 49
x y
24 24
sin U sin U 10
x y sin U tan
x y 49
U U 1 10 1 10
x.cos U y.cos U. sin U tan tan
x y 49 49
EBD_7443
www.jeeneetbooks.in

WWW.IIT-NEET.XYZ

2007-24 Target VITEEE

4 3 4
dx
95. (b) | x 3 | dx (x 3)dx (x 3)dx 97. (d) Let I 2
1 1 3 0 (a x 2 )7

x2
3
x2
4
Put x a tan dx a sec2 d
3x 3x
2 2
1 3
limit at x 0 0&x
2
1 2 2 1 2 2
[3 1 ] 3[3 1] .[4 3 ] 3[4 3]
2 2

1 1
2
a sec 2
(8) 3(2) .(7) 3(1) I d
2 2 0 a14 (1 tan 2 )7
7 5
4 6 3
2 2
2 2
1 1 1
2 d . cos12 .d
96. (b) Required area (y 2 y1 )dx a13 0 sec12 a13 0
0
But

y 2 1
x
y=2 (2,4) sin 2m 1 .cos 2n 1 . d B(m, n)
2
(x2y2) 0

(0,1) Q
(1,1) m n 1
& B(m, n) &
m n 2
P

2
1
x' I sin 0 . cos12 d
o
x a13 0
(2,0)
2
–x
2x

1 13
m , n
y=

2 2

1 13
.
1 2 2
2
I
x 2 2a13 1 13
(2 2x x )dx
2 2
0

2 11 9 7 5 3 1
2x x3 . . . . . . .
1
x2 . 2 2 2 2 2 2
log 2 3 2a13 6.5.4.3.2.1
0

4 8 1 3 4 231 1
4 .
log 2 3 log 2 log 2 3 2048 a13
www.jeeneetbooks.in

WWW.IIT-NEET.XYZ

Solved Paper 2007 2007-25

(1 x)2
98. (c) Let I ex dx cos log(1) c c 0
(1 x 2 )2 2
y
x 1 x 2 2x cos log x
I e dx x
(1 x 2 )2 100. (c) The equation of the family of circles of
radius r is
1 x2 2x (x – a)2 + (y – b)2 = r2 ...(1)
ex dx
2 2
(1 x ) (1 x 2 )2 Where a & b are arbitrary constants.
Since equation (1) contains two arbitrary
1 x constants, we differentiate it two times w.r.t
I ex dx 2 e x . dx
(1 x 2 ) (1 x 2 )2 x & the differential equation will be of
second order.
1 Differentiating (1) w.r.t. x, we get
I .e x ex . (1 x 2 ) 2 2xdx
(1 x 2 )
dy
2(x a) 2(y b) 0
ex x dx
2 dx
(1 x 2 )2 dy
(x a) (y b) 0 ...(2)
ex x dx
ex dx
I 2 Differentiating (2) w.r.t. x, we get
1 x2 (1 x 2 )2
2
d2 y dy
ex x 1 (y b) 0 ...(3)
2
2 2
dx dx 2 dx
(1 x )
2
x dy
e 1
I +C dx
1 x2 (y b)
d2 y ...(4)
y y dx 2
99. (c) Given : x sin dy y sin x dx
x x On putting the value of (y – b) in equation
(2), we get
y
y sin x
dy x dy
2
dy
dx y 1
x sin dx dx
x x a
d2 y ...(5)
y dy dz dx 2
Put z z.1 x.
x dx dx Substituting the values of (x – a) & (x – b)
dz zx sin z x in (1), we get
x. z z cosec z
dx x sin z
2 2 2 2 2
dy dy dy
dz dx 1 1
x cosec z sin z dz dx dx dx
dx x r2
2 2
cos z log x c d2 y d2 y
2
y dx dx 2
cos log x c
x
2 3 2
dy 2 d2 y
1 r
But y(1) x 1, y dx dx 2
2 2
EBD_7443
www.jeeneetbooks.in

WWW.IIT-NEET.XYZ

2007-26 Target VITEEE


104. (d) Number of onto functions: If A & B are two
d2 y dy 3x sets having m & n elements respectively
101. (c) Given : 2 y 2e
dx 2 dx such that 1 n m then number of onto
The auxiliary equation is functions from A to B is
D2 + 2D + 1 = 0 or m2 + 2m + 1 = 0 n
n r n
(m 1)(m 1) 0 m 1, 1 –1 . Cr r n
r 1
i.e., repeated roots
Given A = {1, 2, 3, ---- n} & B = {a, b, c}
Complementary function = (c1 + c2x)e–x
Number of onto fun ction s
Now Particular Integral (P.I.)
3
3–r 3 n
1 –1 . Cr r
. 2e3x [D = 3] r 1
D2 2D 1
3 13 n 3 2 3 n
1 C1 1 –1 C2 2
1 3x 2e3x e3x
P.I. . 2.e
32 2.3 1 16 8 3
C3 3
n
–1
3 3

Solution y = C. F. + P. I.
3 3
C1 C2 2n 3
C3 3n
3x
e
y c1 c2 x e – x 3! 3! n 3! n
8 2 3
2!1! 2!1! 3! 0!
102. (c) Given ydx + (x – y3) dy = 0
dx dx 1 3 3. 2n 3n
y x y3 0 x y2
dy dy y
3n 3 2n 1
compare this equation to general equation
105. (b) Let there are n persons in the room. The
dx 1 2 total number of hand shakes is same as the
i.e. dy Px Q P ,Q y
y number of ways of selecting 2 out of n.

1 n n n 1
Pdy
dy C2 66 66
I.f . e e y
e log y
y 2!

Q I.f .dy C1 n2 n 132 0


x × I.f.
n 12 n 11 0 n 12
y4 106. (a) We know that, let (G, 0) be a group & e be
x.y y 2 .y dy C1 xy C1
4 the identity then
(a * a)–1 = a–1 o a–1
4xy y4 4C1 = (a–1)–1 = a.
107. (d) No. of tickets = 9
y4 4xy 4C1 y4 4xy C , No. of odd numbered tickets = 5
where C = –4C1 No. of even numbered tickets = 4
103. (b) Given, x1x 2 x3 x 4 x 5 Required probability = P {odd, even, odd}
1050
+ P (even, odd, even)
x1 x 2 x 3 x 4 x 5 2 3 52 7 5
C1 4
C1 4
C1 4
C1 5
C1 3
C1
Each of 2, 3 or 7 can take 5 places and 52 9
C1 8
C1 7
C1 9
C1 8
C1 7
C1
can be disposed in 15 ways.
Hence, number of positive integral solution 5 4 4 4 5 3 5
= 53 × 15 = 1875 9 8 7 9 8 7 18
www.jeeneetbooks.in

WWW.IIT-NEET.XYZ

Solved Paper 2007 2007-27

108. (a) Given


1 1 1 tan
1 5 1 A 2
.
P A ,P B ,P B/ A sec tan 1
12 12 15
P A B 1 10 1 tan
We know that P B / A B 2
.
P A sec 01 tan 1

1 P A B 1 1 tan
B 2
.
15 1/12 sec tan 1
1 1
P A B 1
15 12 180 B .A
sec2
But,
P A B P A P B P A B sec 2 .B A

1 5 1 89 1
P A B cos 2 .B A
12 12 180 180
2 x
109. (a) f x dx , where f x 112. (b) Given : 2x 1 4 8
1.5 2
2 2x 2 0
2
2 x 1 2 1 x 2 7 6 2x
dx . xdx
1.5 2 2 1.5 2 2
1.5
2x 1 4x 2 12 4 4x 14
1 1 175 7
4 2.25 1.75
4 4 400 16 8 12 14x 0
110. (b) If 2 P(X = 0 ) + P (X = 2) = 2 P (X = 1)
Let probability distribution of X be given 8x 3 24x 4x 2 12 16x
by 56 96 112 x 0
r m
m .e 8x 3 4x 2 152x 140 0
P(X = r) where r = 0, 1, 2, .....
r! (x + 5) is a factor of above equation
m0 e m m2 .e m
m.e m
8x 3 40x 2 36x 2 180x
2 2
0! 2! 1!
28x 140 0
2
m2 8x x 5 36x x 5 28 x 5 0
2 2m m2 4m 4 0
2
x 5 8x 2 36x 28 0
2
m 2 0 m 2
x 5 4 2x 2 9x 7 0
1 tan
111. (c) A & AB I
tan 1
4 x 5 2x 2 7x 2x 7 0
1
B I A , A11 1, A12 tan ,
4 x 5 x 2x 7 1 2x 7 0
A 21 tan , A 22 1
4 x 5 2x 7 x 1 0
1 tan 2 2
|A| 1 tan sec x 5, 3.5,1
tan 1
EBD_7443
www.jeeneetbooks.in

WWW.IIT-NEET.XYZ

2007-28 Target VITEEE

113. (b) For only one solution | A | 0 118. (a) Given : H is the harmonic mean between
P&Q
k 2 1
2PQ 1 P Q
0 k 1 2 0 H
P Q H 2PQ
0 0 k 2
2 1 1 H H
k k 1 k 2 0 2
H Q P P Q
k 0, k 1, k 2. k 1
119. (c) We have a b a b
114. (c) Let
1 2 5 1 2 5 a a b b a b
A 2 4 a 4 0 0 a 6
1 2 a 1 0 0 a 6 a .b a a .a b b.b a b.a b

R2 R2 2R1 , R 3 R 3 R1 a .b a b
clearly rank of A is 1 if a = –6
2 2
115. (c) Given : ax 4 bx 2 c 0 a b b.b b 1, a .a a 1

Equation will be real if D 0


a .b 1 a b
b2 4ac 0 b2 4ac
4 x a b where x a.b 1 is a scalar
116. (a) Given : log 3 x log 3 x log 3 x
The given vector is parallel to a b .
log 3 8 x log 3 16
x 4
120. (a) Given : A, B & C are three points with co-
1
1 1 1 1 ordinates (1, 2, –1), (2, 0, 3) & (3, –1, 2)
log3 x 2 4 8 16 4 respectively.
Now, direction ratio's of
1
1
AB = 2 – 1, 0 – 2, 3 + 1 = 1, – 2, 4 &
1 a direction ratio's of
log3 x 2 4 S
1 r AC = 3 –1, –1 –2, 2 + 1 = 2, – 3, 3
log3 x 2 4 x2 34 x 9 we know that

3
a1a 2 b1b 2 c1c 2
1 1 cos
117. (a) x x 0 a12 b12 c1 . a 2 2 b 2 2
2
c22
x x

2
1 2 2 3 4 3
1 1 cos
x x 1 0 1 4 16 . 4 9 9
x x
2 6 12 20
1
x 0 x2 1 0 x i 21 . 22 462
x
Thus, the given equation has no real roots. 462 cos 20
www.jeeneetbooks.in

WWW.IIT-NEET.XYZ

VITEEE
SOLVED PAPER 2006

4. A current of 5A is passing through a metallic


PART - I (PHYSICS) wire of cross-sectional area 4 × 10–6m2. If the
density of the charge carriers in the wire is 5 ×
1 A potential difference of 300 V is applied to a 1026m–3, the drift speed of the electrons will be
combination of 2.0µF and 8.0 µF capacitors [e = 1.602 × 10–19C]
connected in series. The charge on the 2.0µF (a) 1.56×10–2ms–1 (b) 1.98×10–2ms–1
(c) 2.42×10–2ms–1 (d) 2.84×10–2ms–1
capacitor is
5. The series combination of two capacitors shown
(a) 2.4 × 10–4C (b) 4.8 × 10–4C in figure is connected across 1000V. The
(c) 7.2 × 10 C–4 (d) 9.6 × 10–4 C magnitude of the charges on the capacitors will
2. Two point charges 4 C and – 2 C are separated be
by a distance of 1 m in air. Then the distance of + – + –
+ –
the point on the line joining the charges, where
3pF 6pF
the resultant electric field is zero, is (in metre)
(a) 0.58 (b) 0.75
(c) 0.67 (d) 0.81 1000V
3. Figure shows a triangular array of three point
(a) 3 × 10–9 C (b) 2 × 10–9 C
charges. The electric potential V of these source
(c) 2.5 × 10–9 C (d) 3.5 × 10–9 C
charges at the midpoint P of the base of the 6. Three resistances of values 2 , 3 and 6 are
triangle is to be connected to produce an effective
resistance of 4 . This can be done by
1
9 109 Nm 2 C 2 connecting
4 0 (a) 6 resistance in series with the parallel
combination of 2 and 3
(b) 3 resistance in series with the parallel
–6 combination of 2 and 6
q3 = 3 × 10 C
(c) 2 resistance in series with the parallel
combination of 3 and 6
(d) 2 resistance in parallel with the parallel
combination of 3 and 6
7. The resistance of a field coil measures 50 at
20ºC and 65 at 70ºC. The temperature
0.300 m coefficient of resistance is
(a) 0.0086/ºC (b) 0.0068/ºC
–6 –6 (c) 0.0096/ºC (d) 0.0999/ºC
q1 = 1 × 10 C q2 = –2 × 10 C
0.200 m P 0.200 m 8. The electrolyte used in Lechlanche cell is
(a) copper sulphate solution
(b) ammonium chloride solution
(a) 55kV (b) 45kV (c) dilute sulphuric acid
(c) 63kV (d) 49kV (d) zinc sulphate
EBD_7443
www.jeeneetbooks.in

WWW.IIT-NEET.XYZ

2006-2 Target VITEEE


9. A galvanometer has a resistance of 50 . If a 17. Our eyes respond to wavelengths ranging from
resistance of 1 is connected across its (a) 400 nm to 700 nm
terminals, the total current flow through the (b) 700 nm to 800 nm
galvanometer is [Ig represents the maximum (c) 0 to
current that can be passed through the (d) – to +
galvanometer] 18. A new system of units is evolved in which the
(a) 42 Ig (b) 53 Ig values of 0 and 0are 2 and 8 respectively. Then
(c) 46 Ig (d) 51 Ig
the speed of light in this system will be
10. In a tangent galvanometer, a current of 1A
(a) 0.25 (b) 0.5
produces a deflection of 30º. The current required
(c) 0.75 (d) 1
to produce a deflection of 60º is
(a) 3A (b) 2A 19. A ray of light strikes a piece of glass at an angle
(c) 4A (d) 1A of incidence of 60º and the reflected beam is
11. In the presence of magnetic field ‘B’ and electric completely plane polarised. The refractive index
field ‘E’, the total force on a moving charged of glass is
particle is (a) 2 3 (b) 3
(a) F v[(q B) E]
3 1
(b) (c) (d)
F q[(v E) B] 2 2
(c) F q[(v B) E] 20. In an experiment on Newton’s rings, the diameter
of the 20th dark ring was found to be 5.82mm and
(d) F B[(q E) v] that of the 10th ring 3.36 mm. If the radius of the
12. A circular coil of radius 40 mm consists of 250 plano-convex lens is 1 m, the wavelength of light
turns of wire in which the current is 20mA. The used is
magnetic field in the center of the coil is (a) 5646 Aº (b) 5896 Aº
[ = 4 × 10–7 Hm–1] (c) 5406 Aº (d) 5900 Aº
(a) 0.785 G (b) 0.525 G 21. What is the angular momentum of an electron in
(c) 0.629 G (d) 0.900 G the fourth orbit of Bohr’s model of hydrogen
13. RMS value of AC is _______ of the peak value. atom?
(a) 7% (b) 7.7%
(c) 70% (d) 70.7% h 2h
(a) (b)
14. Q-factor can be increased by having a coil of 2
(a) large inductance, small ohmic resistance
(b) large inductance, large ohmic resistance h
(c) h (d)
(c) small inductance, large ohmic resistance 4
(d) small inductance, small ohmic resistance 22. The transition of an electrom from n2 = 5,6, ..........
15. A small piece of metal wire is dragged across the to n1 = 4 gives rise to
gap between the pole pieces of a magnet in 0.5 (a) Pfund series (b) Lyman series
second. The magnetic flux between the pole (c) Paschen series (d) Brackett series
pieces is known to be 8 × 10–4 Wb. The emf 23. The ground state energy of hydrogen atom is –
induced in the wire is
13.6 eV. What is the potential energy of the
(a) 16 m V (b) 1.6 V
electron in this state?
(c) 1.6 m V (d) 16V
(a) –27.2 eV (b) –13.6 eV
16. Current in the LCR circuit becomes extremely
large when (c) +13.6 eV (d) 0 eV
(a) frequency of AC supply is increased 24. The longest wavelength that can be analysed
(b) frequency of AC supply is decreased by a sodium chloride crystal of spacing d = 2.82
(c) inductive reactance becomes equal to Aº in the second order is
capacitive reactance (a) 2.82 Aº (b) 5.64 Aº
(d) inductance becomes equal to capacitance (c) 8.46 Aº (d) 11.28 Aº
www.jeeneetbooks.in

WWW.IIT-NEET.XYZ

Solved Paper 2006 2006-3

25. Which is the incorrect statement of the 33. When the conductivity of a semiconductor is
following? only due to the breaking up of the covalent
(a) Photon is a particle with zero rest mass bonds, the semiconductor is known as
(b) Photon is a particle with zero momentum (a) donor (b) extrinsic
(c) Photons travel with velocity of light in (c) intrinsic (d) acceptor
vacuum 34. In a P-type semiconductor, the acceptor impurity
(d) Photons even feel the pull of gravity produces an energy level
26. The deBroglie wavelength associated with a (a) just below the valence band
steel ball of mass 1000 gm moving at a speed of (b) just above the conduction band
1 ms–1 is [h = 6.626 × 10–34 Js] (c) just below the conduction band
(a) 6.626 × 10–31m (b) 6.626 × 10–37m (d) just above the valence band
(c) 6.626 × 10–34m (d) 6.626 × 1034m 35. An oscillator is essentially
27. The velocity v, at which the mass of a particle is (a) an amplifier with proper negative feedback
double its rest mass is network circuits
(b) converts alternating current into direct
3 current
(a) v = c (b) v c
4 (c) an amplifier with no feedback network
(d) an amplifier with proper positive feedback
3 network circuits
(c) v c (d) v = 2c 36. Which of the following gates can perform perfect
2
binary addition?
28. How much energy is produced, if 2 kg of a
(a) AND gate (b) OR gate
substance is fully converted into energy?
(c) EXOR gate (d) NAND gate
[c = 3 × 108 ms–1]
37. The frequency of an FM transmitter without
(a) 9 × 1016 J (b) 11 × 1016 J
signal input is called
(c) 15 × 1016 J (d) 18 × 1016 J (a) the centre frequency
29. The difference between the rest mass of the (b) modulation factor
nucleus and the sum of the masses of the (c) the frequency deviation
nucleons composing a nucleus is known as (d) the carrier swing
(a) packing fraction (b) mass defect 38. The fundamental radio antenna is a metal rod
(c) binding energy (d) isotopic mass which has a length equal to
30. The half life period of Radium is 3 minute. Its (a) in free space at the frequency of operation
mean life time is
3 (b) in free space at the frequency of
(a) 1.5 minute (b) minute 2
0.6931 operation

(c) 6 minute (d) (3 × 0.6931) minute (c) in free space at the frequency of
31. ‘Pair production’ involves conversion of a 4
photon into operation
(a) a neutron-electron pair 3
(b) a positron-neutron pair (d) in free space at the frequency of
4
(c) an electron-proton pair operation
(d) an electron-positron pair 39. Vidicon works on the principle of
32. The sub atomic particles proton and neutron fall (a) electrical conductivity
under the group of (b) photoconductivity
(a) mesons (b) photons (c) thermal conductivity
(c) leptons (d) baryons (d) SONAR
EBD_7443
www.jeeneetbooks.in

WWW.IIT-NEET.XYZ

2006-4 Target VITEEE


40. The maximum range, dmax, of radar is 49. How many lattice points belong to a face
(a) proportional to the cube root of the peak centered cubic unit cell?
transmitted power (a) 1 (b) 2
(b) proportional to the fourth root of the peak (c) 4 (d) 3
transmitted power 50. Schottky defect in solids is due to
(a) a pair of cation and anion vacancies
(c) proportional to the square root of the peak
(b) occupation of interstitial site by a pair of
transmitted power
cation and anion
(d) not related to the peak transmitted power (c) occupation of interstitial site by a cation
at all (d) occupation of interstitial site by an anion
51. Which one of the following is amorphous?
PART - II (CHEMISTRY) (a) Polystyrene (b) Table salt
(c) Silica (d) Diamond
41. The equivalent weight of potassium 52. The metal that crystallises in simple cubic system
permanganate when it acts as oxidising agent in is
ferrous ion estimation is (a) Po (b) Na
(c) Cu (d) Ag
(a) 158 (b) 31.6
53. When ideal gas expands in vacuum, the work
(c) 79 (d) 39.5 done by the gas is equal to
42. The magnetic moment of lanthanide ions is (a) PV (b) RT
determined from which one of the following (c) 0 (d) nRT
relation? 54. For a closed system consisting of a reaction
(a) n(n 2) (b) g J(J 1) N2O4(g) 2NO2(g), the pressure
(a) remains constant (b) decreases
(c) g n(n 1) (d) 2 n(n 1) (c) increases (d) becomes zero
55. 6 moles of an ideal gas expand isothermally and
43. Which one of the following has maximum number
reversibly from a volume of 1 litre to a volume of
of unpaired electrons? 10 litres at 27ºC. What is the maximum work
(a) Mg2+ (b) Ti3+ done?
(c) V 3+ (d) Fe2+ (a) 47 kJ (b) 100 kJ
44. Excess of NaOH reacts with Zn to form (c) 0 (d) 34.465 kJ
(a) ZnH2 (b) Na2ZnO2 56. The reaction,
(c) ZnO (d) Zn(OH)2 Zn(s) + CuSO4(aq) ZnSO4(aq) + Cu(s) is an
45. How many isomers does Co(en)2Cl2+ have? example of a
(a) 1 (b) 3 (a) spontaneous process
(c) 2 (d) 4 (b) isobaric process
46. NH3 group in a coordination compound is named (c) non-spontaneous process
as (d) reversible process
(a) ammonium (b) ammine 57. For the reaction, H2 (g) + I2 (g) 2HI (g)
(c) amine (d) ammonia (a) Kp = –Kc (b) Kc = 0
47. Name the complex Ni(PF3)4 (c) Kp = Kc (d) Kp = 0
(a) tetrakis (phosphorus (III) fluoride) nickel 58. The increase of pressure on ice water at a
(0) constant temperature will cause
(b) tetra (phosphorus (III) fluoride) nickel (a) water to vaporize (b) water to freeze
(c) Nickel tetrakis phosphorus (III) fluoride (c) no change (d) ice to melt
(d) (phosphorus (III) tetrakis fluoride) nickel 59. The order of the reaction
(0) 1
N2O5 N2O4(g) + O (g) is
48. The purple colour of KMnO4 is due to 2 2
(a) charge transfer (b) d-d transition (a) 3 (b) 2
(c) f-f transition (d) d-f transition (c) 1 (d) 0
www.jeeneetbooks.in

WWW.IIT-NEET.XYZ

Solved Paper 2006 2006-5

60. The reactions with low activation energy are 68. Which one of the following products obtained
always when diethyl ether is boiled with water in
(a) adiabatic presence of dilute acid?
(b) slow (a) Glycol (b) Ethy1 alcohol
(c) non-spontaneous (c) Ethylene oxide (d) Peroxide
(d) fast 69. Identify the product for the following reaction
61. For a cell reaction to be spontaneous, the O
standard free energy change of the reaction must CH2OH HCl
be CH3–C–CH3 + ?
CH2OH
(a) zero (b) positive
(c) infinite (d) negative CH 3
62. Equivalent conductance of an electrolyte COOH
(a) CH3–CHOH +
containing NaF at infinite dilution is 90.1 COOH
Ohm–1cm2. If NaF is replaced by KF what is the
value of equivalent conductance? H3C O–CH2
(b) C
(a) 90.1 Ohm–1cm2 (b) 111.2 Ohm–1cm2 H3C O–CH2
(c) 0 (d) 222.4 Ohm–1cm2
63. The tendencies of the electrodes made up of Cu, H3C
C–OH
Zn and Ag to release electrons when dipped in H3C
their respective salt solutions decrease in the (c) CH3–CHOH
order (d) No reaction
(a) Zn > Ag > Cu (b) Cu > Zn > Ag 70. What is the reaction of acetaldehyde with
(c) Zn > Cu > Ag (d) Ag > Cu > Zn concentrated sulphuric acid?
64. The electrode reaction that takes place at the (a) No reaction
anode of CH4 – O2 fuel cell is (b) Decomposition
(a) 2O2 + 8H+ + 8e– 4H2O (c) Charred to black residue
(b) CH4 + 2H2O CO2 + 8H+ + 8e– (d) Polymerisation
(c) CH4 + 2O2 CO2 + 2H2O 71. Calcium Acetate on heating under distillation
(d) 2H+ + 2e– H2 gives
65. What is the hybridization of oxygen atom in an (a) Acetaldehyde and Calcium Oxide
alcohol molecule? (b) Calcium Carbonate and Acetic acid
(a) sp 3 (b) sp (c) Acetone and Calcium Carbonate
(c) sp 2 (d) p 2 (d) Calcium Oxide and CO2
72. Identify the correct statement
O LiAlH 4
66. ? (a) Aldehydes on reduction give secondary
R–C–OH
(a) RCH2CH2OH (b) RCHO alcohols
(c) RCOR (d) RCH2OH (b) Ketones on reduction give primary alcohols
67. Which one of the following is correct? (c) Ketones reduce Fehling’s solution and give
red cuprous oxide
(a) RCH2OH KMnO4 No reaction (d) Ketones do not react with alcohols
Na 2Cr2O7 ,H 2SO4 73. The O – H stretching vibration of alcohols
(b) CH3CH2OH No reaction
absorbs in the region 3700 – 3500 cm–1. The O –
H stretching of carboxylic acids absorb in the
(c) CH3CHO Na 2Cr2O7 ,H 2SO4 No reaction region
(a) 3900 – 3700 cm–1 (b) 3000 – 2500 cm–1
CH3 (c) 3700 – 3500 cm–1 (d) 1700 – 2000 cm–1
(d) CH3–C–OH 74. Which among the following reduces Fehling’s
CH3 solution?
(a) Acetic acid (b) Formic acid
alkaline KMnO 4 No reaction (c) Benzoic acid (d) Salicylic acid
EBD_7443
www.jeeneetbooks.in

WWW.IIT-NEET.XYZ

2006-6 Target VITEEE


75. Determine the experimental condition for the
x 3 7
following reaction
82. If x = –9 is a root of 2 x 2 0 , then other
COOH
7 6 x
+ CO2
OH OH two roots are
(a) 3, 7 (b) 2, 7
(a) in presence of KOH
(c) 3, 6 (d) 2, 6
(b) on heating 83. The values of for which the system of equation
(c) in presence of NaOH x + y + z = 1, x + 2y + 4z = , x + 4y + 10z = 2 is
(d) in presence of HCl consistent are given by
76. Which one of the following is an ingredient of (a) 1, –2 (b) –1, 2
Pthalic acid manufacture by catalytic oxidation (c) 1, 2 (d) 1, 1
(a) Benzene (b) Salicylic acid
1 3 2
(c) Anthranilic acid (d) naphthalene
84. Let A 2 5 t , then the values of t
77. On comparison with H C H bond angle of
4 7 t 6
methane, the C N C bond angle of
trimethylamine is for which inverse of A does not exist
(a) higher (b) no change (a) –2, 1 (b) 3, 2
(c) not comparable (d) lower (c) 2, –3 (d) 3, –1
78. The treatment of acylazide (RCON3) with acidic 85. The non integer roots of
or alkaline medium gives x4 3x 3 2x 2 3x 1 0
(a) RCONH2 (b) R – NH2
1 1
(c) RCH2 NH2 (d) RCOCHNH (a) (3 13), (3 13)
2 2
79. The sequence of basic strength of alky1 amines
follows the order 1 1
(b) (3 13), (3 13)
(a) RNH2 < R2NH > R3N 2 2
(b) R2NH2 < R2NH < R3N
1 1
(c) R2NH < RNH2 < R3N (c) (3 17 ), (3 17 )
2 2
(d) RNH2 < R2NH < R3N
80. Activation of benzene ring in aniline can be 1 1
(d) (3 17), (3 17)
decreased by treating with 2 2
(a) dil. HCl (b) ethyl alcohol
(c) acetic acid (d) acetyl chloride 86. If e x y 1 y2 , then the value of y is
1 x x 1 x x
PART - III (MATHEMATICS) (a) (e e ) (b) (e e )
2 2
x x
81. The value of x, for which the matrix x x 2
(c) e e 2(d) e e
87. Consider an infinite geometric series with the
2 first term a and common ratio r. If its sum is 4 and
1 2
x
3
A 1 x 2x 2 is singular, is the second term is , then
4
1
1 2 4 3 3
x (a) a = ,r= (b) a = 2, r =
7 7 8
(a) ±1 (b) ±2 3 1 1
(c) a = ,r= (d) a = 3, r =
(c) ±3 (d) ±4 2 2 4
www.jeeneetbooks.in

WWW.IIT-NEET.XYZ

Solved Paper 2006 2006-7

88. If and are the roots of the equation 95. If sin , cos , tan are in G.P. then cos9 +
ax2 + bx + c = 0, then the value of 3 + 3 is
cos6 + 3cos5 – 1 is equal to
3abc b3 a 3 b3 (a) –1 (b) 0
(a) (b) (c) 1 (d) 2
a3 3abc
96. If in a triangle ABC,
3 3 5cosC + 6cosB = 4 and 6cosA + 4cosC = 5,
3abc b (3abc b )
(c) (d)
a3
a3 A B
then tan tan is equal to
89. The volume of the tetrahedron with vertices 2 2
P (–1, 2, 0), Q ( 2, 1, –3), R (1, 0, 1) and S (3, –2, 3)
is 2 3
(a) (b)
3 2
1 2
(a) (b) 1
3 3
(c) (d) 5
5
1 3
(c) (d) 97. In a model, it is shown that an arc of a bridge is
4 4 semielliptical with major axis horizontal. If the
90. If a iˆ 2jˆ 3k,
ˆ b ˆi 2jˆ kˆ and length of the base is 9m and the highest part of
the bridge is 3m from horizontal; the best
c 3iˆ ˆj then t such that a tb is at right angle approximation of the height of the arch, 2m from
to c will be equal to the centre of the base is
(a) 5 (b) 4 11 8
(c) 6 (d) 2 (a) m (b) m
4 3
91. An equation of the plane passing through the
line of intersection of the planes x + y + z = 6 and 7
(c) m (d) 2 m
2x + 3y + 4z + 5 = 0 and passing through (1, 1, 1) 2
is 98. The number of real tangents through (3,5) that
(a) 2x + 3y + 4z = 9 (b) x + y + z = 3 can be drawn to the ellipses 3x2 + 5y2 = 32 and
(c) x + 2y + 3z = 6 (d) 20x + 23y + 26z = 69 25x2 + 9y2 = 450 is
92. The length of the shortest distance between the (a) 0 (b) 2
lines r 3iˆ 5jˆ 7kˆ (iˆ 2ˆj k)
ˆ and (c) 3 (d) 4
99. If the normal to the rectangular hyperbola xy =
r ˆi ˆj kˆ (7iˆ 6jˆ k)
ˆ is
c
(a) 83 units (b) 6 units c2 at the point ct, meets the curve again at
t
(c) 3 units (d) 2 29 units
c
93. The region of the argand plane defined by ct ', , then
t'
z i z i 4 is (a) t3t' = 1 (b) t3t' = –1
(a) interior of an ellipse (c) tt' = 1 (d) tt' = –1
(b) exterior of a circle 100. An equilateral triangle is inscribed in the
(c) interior and boundary of an ellipse parabola y2 = 4x one of whose vertex is at the
(d) interior of a parabola vertex of the parabola, the length of each side of
13
the triangle is
94. The value of the sum (i n i n 1 ) where 3 3
n 1 (a) (b) 4
2 2
i 1 equals
(a) i (b) i –1 3
(c) 8 (d) 8 3
(c) – i (d) 0 2
EBD_7443
www.jeeneetbooks.in

WWW.IIT-NEET.XYZ

2006-8 Target VITEEE

101. If f(2) = 4 and f '(2) = 1,


2
xf (2) 2f (x) 106. The value of the integral log(tan x)dx
then lim is equal to 0
x 2 x 2
(a) 0 (b) 1
1
(a) 0 (b) (c) (d)
2 2 4
(c) 1 (d) 2 107. What is the area of a loop of the curve r = asin3 ?
102. What is the least value of k such that the function a2 a2
x2 + kx + 1 is strictly increasing on (1,2) (a) (b)
6 8
(a) 1 (b) –1
(c) 2 (d) –2 a2 a2
(c) (d)
12 24
x
1 9
103. The maximum value of is
x 108. The value of the integral e t dt
1
(a) e (b) ee
(a) e 3 (b) 4e3
1 1 (c) 4(e3 – e) (d) 4e3 – 2e
1 e 109. The differential equation that represents all
(c) ee (d)
e parabolas each of which has a latus rectum 4a
and whose axes are parallel to the x – axis is

x 3 y3 d2 y dy
u u (a) 2 0
104. If u = tan –1 x y , then x x y dx 2 dx
y
3
(a) sin 2u (b) cos 2u d2 y dy
(b) 0
(c) sec2 2u (d) tan 2u dx 2 dx
3
x d2 y dy
105. If f ' x and f(0) = 0, then f(x) = (c) a 0
1 x dx 2 dx
3
3 1 d2 y dy
2 (d) 2a 0
(a) (1 x) 2 6(1 x) 2 1 dx 2 dx
3
y
110. The solution of xcosec y dx + xdy = 0
x
3 1
2 is
(b) (1 x) 2 3(1 x) 2 2
3 x
(a) log x cos c
y
3 1 y
2 (b) log x cos c
(c) (1 x) 2 4(1 x) 2 2 x
3
x
(c) log x sin c
y
3 1
2
(d) (1 x) 2 3(1 x) 2 1 y
3 (d) log x sin c
x
www.jeeneetbooks.in

WWW.IIT-NEET.XYZ

Solved Paper 2006 2006-9

117. In rolling two fair dice, what is the probability of


d2 y 2 obtaining a sum greater than 3 but not exceeding
111. The particular integral of 2y x is
dx 2 6?
(a) x2 –1 (b) x2 + 1
1 1
(a) (b)
1 2 1 2 2 3
(c) (x 1) (d) (x 1)
2 2
1 1
112. The solution of (D2 + 16) y = cos4x is (c) (d)
4 6
x
(a) Acos4x + Bsin 4x + sin 4x 2
8 118. Team A has probability of winning whenever
3
x it plays. Suppose A plays four games. What is
(b) Acos4x + Bsin 4x sin 4x
8 the probability that A wins more than half of its
games?
x
(c) Acos4x + Bsin 4x sin 4x
4 16 19
(a) (b)
27 27
x
(d) Acos4x + Bsin 4x sin 4x
4 19 32
(c) (d)
113. Determine which one of the following relations 81 81
on X = {1,2,3,4} is not transitive. 119. An unprepared student takes five-questions of
(a) R1= , the empty relation true-false type quiz and guesses every answer.
(b) R2 = X x X, the universal relation What is the probability that the student will pass
(c) R3 = {(1,3), (2,1)} the quiz if at least four correct answers is the
(d) R4 = {(1,1), (1,2), (2,3), (1,3), (4,4)} passing grade?
114. Find the number of ways in which five large
books, four medium-size books, and three small 1 3
(a) (b)
books can be placed on a shelf so that all books 16 16
of the same size are together.
1 3
(a) 5 × 4 × 3 (b) 5! × 4! × 3! (c) (d)
32 32
(c) 3 × 5! × 4! × 3! (d) 3! × 5! × 4! × 3!
115. Consider the set Q of rational numbers. Let be 120. The probability density f(x) of a continuous
the operation on Q defined by a b = a + b – ab. random variable is given by f(x) =
The identity element under is Ke
x
, x . Then the value of K is
(a) 0 (b) 1
(c) 2 (d) not exist 1
(a) (b) 2
116. The statement ~ p q is equivalent to 2
(a) p q (b) ~ p q
1
(c) ~p ~q (d) p ~q (c) (d) 4
4
EBD_7443
www.jeeneetbooks.in

WWW.IIT-NEET.XYZ

2006-10 Target VITEEE

2006 SOLUTIONS

PART - I (PHYSICS) 3. (b) The net electric potential is algebraic sum


of potential due to individual point charges.
1. (b) V= 300V, C1= 2.0 F, C2= 8.0 F, 1 q1 q2 q3
V
1 1 1 4 0 r1 r2 r3
Net capacitance,
Cs C1 C2 6 6 6
1 1 10 2 10 3 10
V
C1C2 4 0 0.2 0.2 0.3
Cs
C1 C2 9 109 5 –10 10 10 –6
2 8 16 = 1.6 F.. 9 103 5 45 103 V = 45 kV
Cs
2 8 10
4. (a) In a metal, conduction current is due to
Now total charge,
electrons given by
Q = Vs ×Cs= 300 × 1.6 × 10–6 = 4.8 ×10– 4 C.
In series charge is same on capacitors I nAev
Charge on 2 F capacitor is 4.8 × 10– 4C I
drift velocity, v
nAe
2. (a) C P C
x 5
v
Let the point P where resultant field is zero 5 10 26
4 10 6
1.60 2 10 –19
be x m from 4 C charge and (1– x) m distance
apart from –2 C charge. Since field is zero 1
at this point then, 4 1.602 101
E E1 E 2 0
10 –1
=1.56 ×10 –2 m/s
1 q1 1 q2 6.408
E= 2 5. (b) In series combination of capacitors, charges
4 0 r1 4 0 r22
on both capacitors will be same.
Q Q
1 4 C –2 C Vs
0 C1 C2
4 0 x2 1– x
2

1 1
4 C 2 C 2 1 1000 Q
C1 C2
x2 1– x
2
x2 1– x
2

C1 C2
2 2
1000 Q
2 1– x x C1C2

Taking root 2 1– x x 1000 C1C 2


Q
C1 C2
1.414 1– x x 1.414 –1.414x x
3 10 –12 6 10–12 18
1.414 Q 1000 –12
10 –9
1.414 1 1.414 x x 3 6 10 9
2.414
x 0.58m 2 10 –9 C
www.jeeneetbooks.in

WWW.IIT-NEET.XYZ

Solved Paper 2006 2006-11

6. (c) Parallel combination of 3 and 6 gives ( n = no. of turns, I = current through


effective resistance, coil, r = radius of coil)
3 6 18 4 250 20 10 –7 –3 3
Rp 2 . This in series with B
3 6 9 2 40
2 gives net ressistance as 4 .
250 3.14 10 –7
7. (b) The value of temperature coefficient of
resistance is given by 785 10 –7 0.785 10 –4 tesla
R 2 – R1 65 – 50 = 0.785 gauss
13. (d) RMS value of A.C is
R1 t 2 – t1 50 70 – 20
I0
(t1 and t2 are in °C) Iv 0.707I 0
2
15
0.006 / C I0 = peak value
50 50
it is 70.7% of peak value.
8. (b) In Leclanche cell a strong solution of
ammonium chloride acts as an electrolyte. 1 L
14. (a) Q-factor is given by Q
9. (d) In the galvanometer, Ig = max. current R C
th rough galvan ometer, S = shunt If resistance R is decreased, Q increases
resistance, G = galvanometer resistance and inductance L is increased, Q increases.
then
–d
S 15. (c) Induced emf e . Assuming, small
Ig(galvanometer) = I dt
G S
change in flux d =8×10 – 4 Wb
G S 50 1 change in time dt = 0.5s
I Ig Ig 51Ig
S 1
8 10– 4 80 10 – 4
10. (a) Current in tangent galvanometer |e| =
0.5 5
H =16 × 10 =1.6×10–3 V = 1.6 mV
– 4
I tan
G 16. (c) Current through an LCR circuit is maximum
Where G = galvanometer constant when impedance is minimum. Now
H= earth's horizontal field = constant impedance
I1 tan 1 1 tan 30 1
2

I2 tan I2 tan 60 Z R2 L– is minimum at


2 C
tan 60 1.7321
I2 2.999Å 1
tan 30 0.5774 resonance frequency when and L
C
I 2 3A Z= R = minium i.e., inductive reactance ( L)
11. (c) Lorentz force on a charged particle in is equal to capacitive reactance (1/ C)
presence of magnetic and electic field is 17. (a) Our eyes respond to visible range from 400
nm to 700 nm
F Fe Fm 18. (a) Velocity of electromagnatic wave in space
F qE q v B 1 1
is c c
12. (a) In a circular coil of n turns, magnetic field is 0 0 2 8

0 nI 4 10 –7 250 20 10 –3 1 1
B 0.25
2r 2 40 10 –3 16 4
EBD_7443
www.jeeneetbooks.in

WWW.IIT-NEET.XYZ

2006-12 Target VITEEE


19. (b) According to Brewster's law, reflected light 25. (b) Photon moves with speed of light ie,
is plane polarised if unpolarised light falls v = c and rest mass of a particle is
at the interface of air and medium at an angle
m0 m 1 – v2 / c2
ip called polarising angle then
hence m0 (photon)= 0
= tan ip (glass) = tan 60 3 photon has zero rest mass.
20. (a) Newton's ring arrangement is used for
h
determin ing the wavelength of Momentum of photon =
monochromatic light. For this the diameter
of nth dark ring (Dn) and (n + p)th dark ring 26. (c) de Broglie wavelength is given by
(Dn + p) are measured then h 6.626 10 –34
–3
6.626 10 –34 m
D2(n p) 4(n p) R and D2n 4n R mv 1000 10 1
27. (b) Let the velocity of a particle be v where
D2n D2n mass m is double the rest mass i.e., m = 2m0
p
then
4pR
Here, n = 10, n + p = 20; v2 v2
m0 m 1– m0 2m 0 1 –
p = 10; R = 1 m, D10 = 3.96 × 10–3 m, c2 c2
D20 = 5.82 × 10–3 m
1 v2 1 v2
1– =1 – 2
D 220 2
D10 2 c2 4 c
4pR
v2 1 3
3 2 3 2 1–
(5.82 10 ) (3.36 10 ) c2 4 4
4 10 1
3
= 5646 Å v c
4
21. (b) Angular momentum in any stationary orbit
28. (d) By Einstein's equation E= mc2 where
nh m = 2kg
is mvr for 4th orbit, n = 4
2 E = 2×(3 × 10 8 )2
= 2 × 3 × 3× 10 16 = 18 × 1016 J
4h 29. (b) By definition, the difference between the
mvr
2 sum of the masses of neutrons and protons
forming a nucleus and mass of nucleus is
called mass defect
22. (d) According to Bohr's, Brackett series is
30. (b) Mean life time = 1.44 T where T is half
obtained when an electron jumps to 4th orbit
life period of an atom
from any other outer orbit
23. (a) Total energy of electron T 3
= 1.44 T = minute
0.6931 0.6931
KZe 2 31. (d) (by conservation of charge)
E K.E. P.E.
2r 32. (d) Baryons are proton, neutron, lamda,
0 –
–KZe2 sigma ( –), X
i ,
Potential energy in the orbit P.E.
r 33. (c) As donor and acceptor impurities are added
P.E. = 2 × E P.E.=2 × ( –13.6)= –27.2 eV to semiconductor to make an extrinsic semi-
24. (a) Bragg's condition is 2dsin = n for second conductor, intrinsic semiconductor is
order n = 2 , sin =1. For longest d=2 formed by internal generation of e– by
=d breaking up of covalent bonds.
www.jeeneetbooks.in

WWW.IIT-NEET.XYZ

Solved Paper 2006 2006-13

34. (d) In p-type semiconductor, valency = 3, thus


Y
there is one unformed bond or hole created. PART - II (CHEMISTRY)
This hole is in valence band and is able to
cause hole current. The energy levels of 41. (b) The oxidation of ferrous ion by KMnO4
acceptor are in forbidden gap just above takes place in acidic medium as per
valence band following reaction
35. (d) In an oscillator, L-C circuit is coupled with
transistor amplifier in such a way that there 2KMnO4 + 8H2SO4 + 10FeSO4
is a positive feed back to the LC circuit i.e., K2SO4 + 2MnSO4 + 8H2O + 5Fe2(SO4)3
proper energy supply to LC at proper
Eq. mass of KMnO4
timings. So that total energy of LC circuit
remains same.
Molecular mass
36. (c) The gates AND, OR, NAND do not give
binary addition, however in EXOR gate change in oxidation number

truth table is A B Y
Molecular mass 158
0 0 0 = = 31.6
0 1 1 5 5
1 0 1
42. (b) In case of lanthanoids, 4f orbitals lie too
1 1 0
deep and hence the magnetic effect of the
This shows it gives perfect binary addition motion of the electron in its orbital is not
37. (d) In FM, carrier frequency is the constant quenched out. Here spin contribution S and
frequency which is modulated by signal orbital contribution L couple together to
amplitude. It is also called carrier swing. give a new quantum number J.
(Centre frequency is fc in AM wave,
Thus magnetic moment of lanthanoids is
frequency deviation f max – fc ,
given by, g J(J 1)
max )
mod ulation factor
fc where J = L – S when the shell is less than
38. (c) The common antenna is a straight half fill
J = L + S when the shell is more than half fill
conductor of length l held vertically
4 1 S(S 1) L(L 1)
with its lower end touching the ground. and g 1
2 2J(J 1)
39. (b) The vidicon is a storage-type camera tube
in which a charge-density pattern is formed 43. (d) Mg2+ = 1s2, 2s2, 2p6 (No unpaired electrons)
by the imaged scene radiation on a
photoconductive surface which is then Ti3+ = 1s2, 2s2, 2p6, 3s2, 3p6, 4s0, 3d1
scanned by a beam of low-velocity (One unpaired electrons)
electrons. The fluctuating voltage coupled
V3+ = 1s2, 2s2, 2p6, 3s2, 3p6, 4s0, 3d2
out to a video amplifier can be used to
reproduce the scene being imaged. The (Two unpaired electrons)
electrical charge produced by an image will Fe2+ = 1s2, 2s2, 2p6, 3s2, 3p6, 4s0, 3d6
remain in the face plate until it is scanned
or until the charge dissipates. (Four unpaired electrons)

40. (b) Maximum range of radar d max Fe2+has highest number of unpaired
l and
electrons.
power transmitted by antenna of length l is
44. (d) Zn + 2NaOH Na2ZnO2 + H2
2 1/ 4
p l/ l p and d p Sod. zincate
EBD_7443
www.jeeneetbooks.in

WWW.IIT-NEET.XYZ

2006-14 Target VITEEE


54. (b) As the system is closed, hence the reaction
45. (c) + + will be reversible, hence according to
en en
Le-chatelier principle pressure decreases
Cl Cl since the volume is increasing.
Co Co V2
55. (d) W = – 2.303 nRT log
V1
Cl Cl
en en Given n = 6, T = 27°C = 273 + 27 = 300 K
V1 = 1 L, V2 = 10 L
d - cis form l - cis form
10
W = – 2.303 × 6 × 8.314 × 300 log
1
Cl +
= 34.465 kJ
56. (a) It is spontaneous process because zinc is
more reactive than copper, hence can easily
en Co en
repace Cu from CuSO4.
57. (c) Kp = Kc (RT) n
n = np(g) – nr(g) = 2 – 2 = 0
Cl
Kp = Kc
trans form (meso)
46. (b) Neutral ligands are given the same names 58. (d) Ice Water
as the neutral molecule. However, two very The volume of ice is more than water.
important exceptions to this rule are: Therefore when pressure is increased the
H2O Aquo (Aqua) NH3 Ammine. equilibrium shifts in forward direction. It
47. (a) Ni(PF3)4 – tetrakis phosphours (III) fluoride favoures melting of ice.
nickel (0). 59. (c) It is a first order reaction because
48. (a) The colour of KMnO4 is due to charge
rate of reaction [N2O5]
transfer. The configuration of manganese
in permagnate ion is d0 but it is coloured 60. (d) The reactions with low activation energies
because its electrons are photo-exited. are always fast whereas the reactions with
49. (c) In face centred cubic lattice, the atoms are high activation energy are always slow.
present at eight corners of faces and one 61. (d) For spontaneous reaction free energy
each at 6 faces. change is negative.
Lattice points belonging to face centred G = – nFE
62. (a) Because at infinite dilution the equivalent
1 1
cubic unit cell = 8 6 4 conductance of strong electrolytes
8 2 furnishing same number of ions is same.
50. (a) Schottky defect is caused when equal 63. (c) Reducing character i.e tendency to loose
number of cations and anions are missing electron decreases down the series, hence
from their lattice sites. the correct order is Zn > Cu > Ag.
51. (a) Polystyrene is thermoplastic substance.
64. (b) At anode the following reaction takes place
52. (a) Po – Simple cubic lattice
CH4 + 2H2O CO2 + 8H + + 8e –
Na – bcc
Cu – fcc 65. (a) Oxygen atom in alcohol molecule is sp3
hybridised.
R P1T2 P2 T1 66. (d) In this reaction LiAlH4 acts as reducing
53. (c) Wirr Pext
P1P2 O
During expansion in vaccum Pext = 0 || LiAlH4
agent. R C OH RCH2OH
work done = 0.
www.jeeneetbooks.in

WWW.IIT-NEET.XYZ

Solved Paper 2006 2006-15

67. (d) 3° alcohols are resistant to oxidation and 73. (d) The O – H strecting of carboxylic acid
are oxidised only by strong oxidising agents absorb in region of 1700-2000 cm–1
like conc. HNO3. They are resistant to
oxidation in neutral or alkaline KMnO4. O
||
68. (b) C2H5 – O – C2H5
H 2O
2C2H5OH 74. (b) Formic acid H C OH has – CHO
boil, dil.acid
ethyl alcohol group and therefore it reduces Fehling
solution.
O
|| CH 2OH HCl COOH
69. (b) CH3 C CH3 + | NaOH
CH 2OH 75. (c)
OH OH
H3C O – CH2 + CO2
This is decarboxylation reaction.
H3C O – CH2
cyclic ketal Catalytic
76. (d)
70. (b) oxidation

CH3 Napthalene
Conc. H 2SO4 drops
Room temp. CH
O O COOH
3 CH3CHO
Dil. H2SO4 CH CH
H3C O CH3 COOH
Para aldehyde Phthalic acid
(pleasant smelling liquid,
used as hypnotic and soporofic) 77. (b) In both the cases carbon is sp3 hybridised
(sleep producing) and bond angle is 109°28'.
78. (b) This reaction is known as curtius
Conc. H 2SO4 H3C – CH – O – CH – CH3 rearrangement.
or HCl gas
4 CH3CHO O O N2 2NaOH
dil H 2SO 4
RCON3 RNCO
H3C – CH – O – CH – CH3
Na2CO3 + RNH2
Metaldehyde
(used as solid fuel in spirit lamps) 1° amine is formed.
79. (a) It is expected that the basic nature of
distillation
71. (c) 2(CH3COO)2Ca amines should be in order tertiary >
CH3COCH3 + 2CaCO3 secondary > primary but the observed
Acetone Calcium order in the case of lower members is found
carbonate to be as secondary > Primary > tetriary. This
72. (d) Ketones do not react with alcohol. anomalous behaviour of tetriary amines is
reduction due to steric factors i.e crowding of alkyl
(i) Aldehydes 1° alcohol
groups cover nitrogen atom from all sides
e.g., CH3CHO CH 3CH 2OH thus makes the approach and bonding by a
reduction proton relatively difficult which results the
(ii) Ketones 2° alcohol maximum steric strain in tetiary amines. The
CH3 CH3 electrons are there but the path is blocked,
C=O CHOH
CH3 CH3 resulting the reduction in basicity.
(iii) Ketones do not reduce Fehling solution but Thus the correct order is
aldehydes do so. R2 NH > R NH2 > R3N.
EBD_7443
www.jeeneetbooks.in

WWW.IIT-NEET.XYZ

2006-16 Target VITEEE


80. (d) On acetylation aniline is converted into
(x 9)(x 2 9x 14) 0
acetamide which is resonance stablised and
therefore less reactive. (x 9) (x 2 7x 2x 14) 0
NH2 NHCOCH3
(x 9) (x 7) (x 2) 0

+ CH3COCl x 9, 7, 2
83. (c) We have
Acetanilide
1 1 1 : 1
PART - III (MATHEMATICS) A:B 1 2 4 :
2
1 4 10 :
81. (a) We know that, A is singular if |A| = 0

2 1 1 1 : 1
1 2
x ~ 0 1 3 : 1
2 2
|A| 1 x 2x 0 0 3 9 : 1
1
1 2
x applying R 2 R2 R1
& R3 R 3 R1
2 2 1
|A| 2x 2x 1 2 2x 2 x 0
x x
1 1 1 : 1
2 2 2 ~ 0 1 3 : 1
[0] 2 2x 2x 0
x x 2
0 0 0 : 3 2
3 2
2x 2x 2 2x 0
applying R 3 R 3 3R 2
x3 x2 x 1 0
But the system is consistent
2
x (x 1) 1(x 1) 0 2
3 2 0
(x 1) (x 2 1) 0 ( 2) ( 1) 0 2 or 1
x 1 84. (c) We know that inverse of A does not exist
only when |A| = 0
x 3 7
1 3 2
82. (b) Given 2 x 2 0
2 5 t 0
7 6 x
4 7 t 6
x[x 2 12] 3[2x 14] 7[12 7x] 0
( 30 7t t 2 ) 3( 12 4t)
x 3 67x 126 0 2(14 2t 20) 0
But given (x = 9) is a root of given
determinant. 30 7t t 2 36 12t 12 4t 0
(x + 9) is a factor
t2 t 6 0 t 2 3t 2t 6 0
x 3 9x 2 9x 2 81x 14x 126 0
t(t 3) 2(t 3) 0
2
x (x 9) 9x(x 9) 14(x 9) 0 (t 3)(t 2) 0 t 2, 3
www.jeeneetbooks.in

WWW.IIT-NEET.XYZ

Solved Paper 2006 2006-17

85. (a) Given x4 – 3x3 – 2x2 + 3x + 1 = 0


3
By using Hit & trial method, we have Equation (1) becomes 4
4r(1 r)
(x – 1) is a factor of given equation
(x – 1) (x3 – 2x2 – 4x – 1) = 0
16r 2 16r 3 0
3 2 2
(x 1) [x x 3x 3x x 1] 0
(4r 3) (4r 1) 0
2
(x 1) x (x 1) 3x(x 1) 1(x 1) 0 3 1
r or r
4 4
(x 1) (x 1) (x 2 3x 1) 0
1 3
x = 1, – 1 or x2 – 3x – 1 = 0 when r then a 3
Now x2 – 3x – 1 = 0 4 1
4
4
3 9 4
x 1
2 a 3 & r
4
b b2 4ac 88. (c) Given : & are roots of equation
x ax2 + bx + c = 0
2a
b c
&
3 13 a a
x
2 Now, 3 3
( )3 3 ( )
non-integer roots of given equation are
3
1 1 3 3 b c b
(3 13), (3 13) 3 .
2 2 a a a

86. (b) Given e x y 1 y2 3 3 b3 3bc


3
a a2
x 2
e y 1 y
3 3 b 3 3abc
Squaring both side, we have
e2x + y2 – 2exy = 1 + y2 a3
2ex y = e2x – 1 89. (b) Given : The vertices of tetrahedron are
P(–1, 2, 0), Q(2, 1, –3), R(1, 0, 1) & S(3, –2, 3)
e 2x 1 1
y y ex e x
1
x 2 Volume of tetrahedron PQ PR PS
2e
6
87. (d) First term = a & common ratio = r
Now,
3
Given S 4 & a2 PQ (2 1)iˆ (1 2)ˆj ( 3)kˆ 3iˆ ˆj 3kˆ
4
Similarly, PR 2iˆ 2ˆj kˆ
a
4 ...(1)
1 r & PS 4iˆ 4jˆ 3kˆ
3 a Volume of tetrahedron
& ar S & an ar n 1
4 1 r 3 1 3
1 2
3 2 2 1
a 6 3
4r 4 4 3
EBD_7443
www.jeeneetbooks.in

WWW.IIT-NEET.XYZ

2006-18 Target VITEEE

90. (a) We have, a tb (iˆ 2ˆj 3k)


ˆ t( ˆi 2ˆj k)
ˆ b1 b2 ˆi( 2 6) ˆj(1 7) k(
ˆ 6 14)

(1 t)iˆ (2 2t)ˆj (3 t)kˆ b1 b2 4iˆ 6jˆ 8kˆ


It is to c 3iˆ ˆj Shortest distance
If 3(1 t) (2 2t) (3 t) (0) 0 (4iˆ 6ˆj 8k)
ˆ . ( 4iˆ 6ˆj 8k)
ˆ
PQ
3 3t 2 2t 0 t 5 16 36 64
91. (d) The equation of the plane through the line
of intersection of the given planes is 16 36 64
(x + y + z – 6) + (2x + 3y + 4z + 5) = 0 ... (1) PQ
116
If equation (1) passes through (1, 1, 1), we
have
116
3 116 2 29
3 14 0 116
14
3 PQ 2 29 units
Putting in (1), we obtain the
14 93. (c) Given, | z i | | z i | 4
equation of the required plane as
| z (0 i) | | z (0 i) | 0
3
(x y z 6) (2x 3y 4z 5) 0 This equation represent the interior and
14
boundary of ellipse with foci at (0, 1) &
20x 23y 26z 69 0 (0, –1), whose major axis is along the y-axis.
13 13 13
b1 b 2 . a 2 a1
92. (d) Shortest distance PQ 94. (b) in in 1
in in 1

b1 b 2 n 1 n 1 n 1

Now, a 2 a1 ˆi ˆj kˆ 3iˆ 5jˆ 7kˆ 1 i13 i13


i i2
1 i 1 i
a 2 a1 4iˆ 6jˆ 8kˆ
(1 i) (1 i)
i i 1
B (a 2) (1 i) (1 i)
>

6j + k)
>
>

( 7i –
>

j–k+ µ
>
>

r = –i – Q
95. (b) Given : sin , cos , tan are in G.P..
L2
cos 2 sin tan cos3 sin 2

cos3 1 cos 2

(cos3 cos 2 ) 1 ...(1)


P A (a1) Cubic both sides, we have
L1
>

>

>

>

>
>

r = (3i + 5j + 7k) + (i – 2j + k)
cos cos6 3cos5 . (cos3 cos2 ) 1

ˆi ˆj kˆ cos cos 6 3cos5 1


And b1 b2 1 2 1 [Using equation (1)]
7 6 1 cos cos6 3cos5 1 0
www.jeeneetbooks.in

WWW.IIT-NEET.XYZ

Solved Paper 2006 2006-19

96. (c) Given : 5cosC + 6cosB = 4 ...(1)


6cosA + 4cosC = 5 ...(2) 4x 2 y2
1 ....... (2)
Adding eq. (1) & (2), we have 81 9
9cosC + 6(cosA + cosB) = 9
A B A B
9cos C 6 2cos . cos 9 Y
2 2
P(2, y)1
C A B y1
9 cos C 9 12cos . cos 0
2 2 2 X' O 2m Q X
C A B
9(cos C 1) 12sin . cos 0 Y'
2 2
C C A B
9 1 2sin 2 1 12sin .cos 0
2 2 2 Here, OQ = 2 m, let PQ = y1
P(2, y1)
C C A B
18sin 2 12sin .cos 0 Since point P lies on the ellipse (2)
2 2 2
C A B 4 4 y12
3sin 2 cos 1
2 2 81 9
B A B
3cos 2cos
2 2 y12 16 81 16 65
1
A B A B 9 81 81 81
3 cos .cos sin .sin
2 2 2 2
65 65 8
A B A B y12 y1 m
2 cos .cos sin .sin 9 3 3
2 2 2 2
Hence, best approximation of the height of
A B A B
5sin . sin cos . cos 8
2 2 2 2 the arch m.
3
A B
5 tan . tan 1 98. (c) Given : Equations of ellipses
2 2
3x2 + 5y2 = 32 ...(1)
A B 1
tan .tan & 25x2 + 9y2 = 450 ...(2)
2 2 5
97. (b) Equation of the semielliptical bridge Tangents to the ellipse (1) & (2) are passing
through the point (3, 5)
x2 y2
1 ....... (1) 3(3)2 + 5(5)2 – 32 = 27 + 75 – 32 > 0
a2 b2
So the given point lies outsides the ellipse.
Here, 2a = 9
Hence, two real tangents can be drawn from
9 the point to the ellipse,
a , b= 3
2 & 25(3)2 + 9(5)2 – 450 = 225 + 225 – 450 = 0
x2 y2 The point lie on the ellipse. Hence one
1
81 9 real tangent can be drawn.
4 No. of real tangents = 3
EBD_7443
www.jeeneetbooks.in

WWW.IIT-NEET.XYZ

2006-20 Target VITEEE


& f '(x) = a f '(2) = a = 1 a = 1
c
99. (b) The equation of tangent at ct, is 2 × 1 + b = 4 b = 2 [using equation (1)]
t
f(x) = x + 2
ty = t3x – ct4 + c
xf (2) 2f (x)
c Now, lim
If it passes through ct ', then x 2 x 2
t'
4x 2(x 2) 2x 4
tc 3 4
lim lim
t ct ' ct c x 2 x 2 x 2 x 2
t'
2(x 2)
t t 3 t '2 t 4 t ' t ' lim 2
x 2 (x 2)
t . t ' t 3 t '(t '.t) t 3 t ' 1 102. (d) Let f(x) = x2 + kx + 1
Note : If we take the co-ordinate axes along f '(x) = 2x + k
the asymptotes of a rectangular hyperbola, f(x) is strictly increasing on (1, 2)
then the general equation x2 – y2 = a 2 if f '(x) > 0 for x (1, 2)
becomes xy = c2, where c is a constant. 2x + k > 0 for x (1, 2)
k > –2x for x (1, 2)
3
100. (d) Let AB = , then AM cos 30º Now, 1 < x < 2 2 < 2x < 4
2 –2 > –2x > –4
– 4 < –2x < –2
& BM sin 30º
2 k 2
Hence least value of k = –2.
Y
x
1 x
103. (c) Let y y x
B x
Then log y = –x log x
A 30º M 1 dy
X´ X (1 log x)
30º
y dx

C dy
or y(1 log x)
dx

d y 1 dy
& y. (1 log x).
2 x dx
3 dx
So, the coordinates of B are ,
2 2 x 1
d y 1 dy
(1 log x).
Since, B lies on y2 = 4x dx 2 x dx
2
3 dy
4 0 1 log x 0
4 2 Now,
dx

216 1
. 3 8 3 l og x 1 log e log
2 e
101. (d) Let f(x) = ax + b
Given f(2) = 4 & f '(2) = 1 1
x
f(2) = a . 2 + b = 4 2a + b = 4 ...(1) e
www.jeeneetbooks.in

WWW.IIT-NEET.XYZ

Solved Paper 2006 2006-21

1 Let 1 + x = t2 x = t2 – 1
d2 y 1 1
dx = 2t . dt
Also, at x is e e 0
dx 2 e
t2 1
dy f (x) . 2t dt 2 (t 2 1)dt
0 t
dx
t3
1 f (x) 2 t c
So, x is a point of local maxima. 3
e
Maximum value
(1 x)3/ 2
1 f (x) 2 (1 x)1/ 2 c
1 3
= value of y when x ee
e
.......... (1)
z z 1
104. (a) Euler's theorem x y nz But f (0) 0 2 1 c 0
x y 3

x3 y3 4 4
Given : U = tan–1 c 0 c
x y 3 3
Equation (1) becomes
x3 y3
tan U z (let)
x y (1 x)3/ 2 4
f (x) 2 (1 x)1/ 2
n= 3–1=2 3 3

z z 2
x y 2z f (x) (1 x)3 / 2 3(1 x)1/ 2 2
x y 3

x tan U y .tan U 2 tan U


x y 2
106. (a) Let I log(tan x)dx ...(1)
2 U U
2 0
x . sec U . y.sec U . 2 tan U
x y

U U
sec 2 U . x y 2 tan U 2
x y Then, I log tan x dx
2
0
U U sin U
x. y 2. . cos2 U
x y cos U a a
f (x)dx f (a x)dx
U U 0 0
x y sin 2U
x y

x 2
105. (b) Given : f '(x) , f (0) 0 I log(cot x)dx
1 x
0
x
f '(x)dx dx
1 x
2
1
x I log dx
f (x) dx 0
tan x
1 x
EBD_7443
www.jeeneetbooks.in

WWW.IIT-NEET.XYZ

2006-22 Target VITEEE

2 2
1 a2 2
I log tan x dx log(tan x)dx sin 2 d
0 0 6
0
I I 2I 0 I 0
[Using eq. (1)]
a 2 2 1 cos 2
107. (d) If curve r = a sin 3 . d
To trace the curve, we consider the 6 2
0
following table :
cos 2 1 2sin
3 5
3 0 2 3
2 2 2
2 5 a2 sin 2 2
0 .
6 3 2 3 6 12 2 0
r 0 a 0 a 0 a 0
a2 a2
. sin
Thus there is a loop between 12 2 24

0& as r varies from r = 0 to r = 0. 9


3 108. (b) Let I e t dt
1
Put t = x2 dt = 2x . dx
For limit : x = 1 & x = 3
3 3
I e x . 2x dx 2 e x . x dx
1 1
X 3
3
I 2 x . ex e x dx
1
1

3 3
2 xe x ex
1 1

I 2 3e3 e1 e3 e1 2e3 .2 4e3


Hence, the area of the loop lying in the
109. (d) Equation of the family of such parabolas is
(y – k)2 = 4a(x – h) ...(1)
13 2 where h & k are arbitrary constants
positive quadrant r d
2 Differentiating w.r.t. x, we get
0
dy
(y k) 2a ...(2)
dx
3
1 1
sin 2 . d Differentiating again
2 3
0
2
1 d2 y dy
[On putting, 3 d d ] (y k) 0 ...(3)
3 dx 2 dx
www.jeeneetbooks.in

WWW.IIT-NEET.XYZ

Solved Paper 2006 2006-23

Putting value of (y – k) from (2) in (3), we


1
get Particular integral (P.I.) 2
.x 2
D 2
3
d2 y dy
2a 0 , which is required 1
dx 2 dx 1 2 1 D2
.x . 1 . (x 2 )
2 2 2
equation. D
2 1
2
y
110. (b) Given : x cos ec y dx x dy 0
x
(1 D) 1 1 D D2 D3 ......
2
1 D2 D2
x P.I. . 1 ..... (x 2 )
y dx x dy 0 2 2 2
y
sin
x
1 D2 2
P.I. . x2 (x )
2 2
y y
x y . sin dx x sin dy 0
x x 1
P.I. . x2 1
2
y 112. (a) If (D2 + 16)y = cos 4x
ysin x
dy x Here the auxiliary equation is m2 + 16 = 0
...(1)
dx y m =±4
x sin
x Complementary function
= (A cos 4x + B sin 4x)
y & Particular Integral (P.I.)
Put z y zx
x 1
2
.cos 4x
D 16
dy dz dz
z.1 x. z x
dx dx dx 1 x
But 2 2
cos ax sin ax
Equation (1) becomes D a 2a

dz zx.sin z x x x
z x z cosec z P.I. .sin 4x sin 4x
dx x sin z 2 4 8
Solution y = Complementary function
dz dx + Particular Integral
x cos ecz sin z dz
dx x
x
y = A cos 4x + B sin 4x + sin 4x
log | x | cos z c 8
113. (c)
y 114. (d) Let us make one packet for each of the
log | x | cos c
x books on the same size. Now, 3 packets can
be arranged in P(3, 3) = 3! ways
d2 y 5 large books can be arranged in 5! ways
111. (c) If 2y x2 4 medium size books can be arranged in
dx 2
4! ways
d 3 small books can be arranged in 3! ways
(D2 2)y x2 D Required number of ways
dx
= 3! × 5! × 4! × 3! ways
EBD_7443
www.jeeneetbooks.in

WWW.IIT-NEET.XYZ

2006-24 Target VITEEE


115. (a) An identity relation is one in which every
3 4 3 4 4 4
element of a set is related to itself only. 4 2 1 4 2 1
a * b = a + b – ab C3 . C4 .
3 3 3 3
As in identity relation 'a' is related to 'a', so
the correct option will be the one which 4! 8 1 4! 16
. . .
gives the value of the relation = 'a'. So, 3!1! 27 3 4! 0! 81
equating a + b – ab = a, we get b(1 – a) = 0.
Now putting the values of a, we find b and 32 16 48 16
the option in which a = b, will be the answer. 81 81 81 27
For a = 0, b = 0, so the correct option. 119. (b) n = total number of ways = 25 = 32
For a = 1, b(1 – 1) = 0 b can have multiple Since each answer can be true or false
values. & m = favourable number of ways
For a = 2, b(1 – 2) = 0 b = 0 but a = 2. = 5C4 + 5C5
116. (a) p q ~p ~ pvq p q
T T F T T 5! 5!
5 1 6 m 6
T F F F F 4!1! 5! 0!
F T T T T Since to pass the quiz, student must give 4
F F T T T or 5 true answers.
117. (b) Let S be the sample space
m 6 3
n(S) 36 Hence, p p p
n 32 16
Events
[sum greater than 3 but not exceeding 6] 120. (a) Since f(x) is the probability density function
= {(2, 2), (3, 1), (1, 3), (4, 1), (1, 4), (5, 1) (1, 5), of random variable X.
(3, 2), (2, 3), (4, 2), (2,4), (3, 3)}
n(E) 12 f (x) 1

n(E) 12 1 Now we have


Required probability =
n(S) 36 3
118. (a) Let 'p' denote the probability of winning of Ke | x | dx 1 2 K . e | x | dx 1
team A whenever it plays 0
2 2 1
p &q 1
3 3 3 2 K . e x dx 1
Let X denotes the number of winning games 0
out of 4 games i.e. n = 4
The probability of r success 2K . e x
1 2K 1
P(X = r) = ncr pr qn – r, r = 0, 1, 2, 3, 4 0
Probability of winning more than half 1
games = P(X > 2) K
= P (X = 3) + P(X = 4) 2
www.jeeneetbooks.in

WWW.IIT-NEET.XYZ

MOCK
VITEEE Mock Test Paper
1
Max. Marks : 125 Time : 2½ hrs
PART - I : PHYSICS 6. Kirchhoff’s first law of electricity follows
(a) law of conservation of energy only
1. If s, k and r are coefficients of static friction, (b) law of conservation of charge only
sliding friction and rolling friction, then (c) law of conservation of both energy and
(a) s< k< f (b) k < r < s charge
(c) r < k < s (d) r= k= s (d) sometimes law of conservation of energy
2. In a region of space having a uniform electric and some other times law of conservation
field E, a hemispherical bowl of radius r is placed. of charge
The electric flux through the bowl is 7. In a Wheatstone’s bridge all the four arms have
(a) 2 rE (c) 4 r 2 E equal resistance R. If the resistance of the
galvanometer arm is also R, the equivalent
(c) 2 r 2 E (d) r2 E resistance of the combination as seen by the
3. Which of the following relation is true ? battery is
9 Y R
(a) 3Y K (1 ) (b) K (a) 2R (b)
Y 4
05.Y R
(c) (6K )Y (d) (c) (d) R
2
4. A capacitor C 1 is charged to a potential 8. A particle is acted by a force F = kx, where k is
difference V. The charging battery is then a +ve constant. Its potential energy at x = 0 is
removed and the capacitor is connected to an zero. Which curve correctly represents the
unchar ged capacitor C 2 . The potential variation of potential energy of the block with
difference across the combination is respect to x
VC1 C2 U U
(a) (b) V 1
(C1 C 2 ) C1

C1 VC 2
(a) x (b) x
(c) V 1 (d)
C2 (C1 C 2 )
5. In the network shown in the Fig, each resistance
is 1 . The effective resistance between A and B
is U U
1

1 1 (c) x (d) x
1 1

A 1 1 B

4 3
(a) (b) 9. A straight wire of diameter 0.5 mm carrying a
3 2 current of 1 A is replaced by another wire of
8 1 mm diameter carrying same current. The
(c) 7 (d) strength of magnetic field far away is
7
EBD_7443
www.jeeneetbooks.in

WWW.IIT-NEET.XYZ

MT-2 Target VITEEE


(a) twice the earlier value in the coil is 220V, the self-inductance of the
(b) same as the earlier value coil is
(c) one-half of the earlier value (a) 5 H (b) 6 H
(d) one-quarter of the earlier value (c) 11 H (d) 12 H
10. A positively charged particle moving due east 16. In a region of uniform magnetic induction
enters a region of uniform magnetic field B = 10–2 tesla, a circular coil of radius 30 cm
directed vertically upwards. The particle will and resistance 2 ohm is rotated about an axis
(a) continue to move due east which is perpendicular to the direction of B
(b) move in a circular orbit with its speed and which forms a diameter of the coil. If the
unchanged coil rotates at 200 rpm the amplitude of the
(c) move in a circular orbit with its speed
alternating current induced in the coil is
increased
(a) 4 2 mA (b) 30 mA
(d) gets deflected vertically upwards.
(c) 6 mA (d) 200 mA
11. Power dissipated across the 8 resistor in the
17. If 0 and 0 are the electric permittivity and
circuit shown here is 2 watt. The power
magnetic permeability in vacuum, and are
dissipated in watt units across the 3 resistor
corresponding quantities in medium, then
is refractive index of the medium is
1 3 i1
0
(a) (b)
i 0 0

(c) 0 0 (d)
8 i2 0 0

(a) 1.0 (b) 0.5 18. Th e Young’s double slit experiment is


(c) 3.0 (d) 2.0 performed with blue and with green light of
12. A battery is charged at a potential of 15V for 8 wavelengths 4360Å and 5460Å respectively. If
hours when the current flowing is 10A. The x is the distance of 4th maxima from the central
battery on discharge supplies a current of 5A one, then
for 15 hour. The mean terminal voltage during (a) x (blue) = x (green)
discharge is 14V. The “Watt-hour” efficiency (b) x (blue) > x (green)
of the battery is (c) x (blue) < x (green)
(a) 87.5% (b) 82.5% x (blue) 5460
(c) 80% (d) 90% (d)
x (green) 4360
13. A coil of 40 henry inductance is connected in
19. A cylindrical metal rod is shaped into a ring
series with a resistance of 8 ohm and the
with a small gap as shown. On heating the
combination is joined to the terminals of a 2 system :
volt battery. The time constant of the circuit is
(a) 20 seconds (b) 5 seconds
(c) 1/5 seconds (d) 40 seconds
14. In an inductor of self-inductance L = 2 mH,
current changes with time according to relation
i = t2e–t. At what time emf is zero? (a) x decreases, r and d increase
(a) 4s (b) 3s
(b) x and r increase, d decreases
(c) 2s (d) 1s
(c) x, r and d all increase
15. A varying current in a coil changes from 10A
(d) x and r decreased, d remains constant
to zero in 0.5 sec. If the average e.m.f induced
www.jeeneetbooks.in

WWW.IIT-NEET.XYZ

Mock Test 1 MT-3

20. The angular separation d between two 26. Einstein’s photoelectric equation is
wavelength and + d in a diffraction grating Ek h . In this equation Ek refers to
is directly proportional to
(a) kinetic energy of all the emitted electrons.
(a) frequency of light
(b) mean kinetic energy of emitted electrons
(b) grating element (c) maximum kinetic energy of emitted
(c) spatial frequency of grating electrons.
(d) wavelength of light (d) minimum kinetic energy of emitted
21. In the Bragg scattering of a beam of electrons electrons.
each of mass m and velocity v by a nickel 27. In a photoelectric experiment anode potential
crystal, the first maximum is observed at = is plotted against plate current
30° ( being the angle the beam makes with the
crystal plane). What is the inter-planar distance I
d for the crystal?
h 2h C B
(a) (b) A
mv mv
h mv V
(c) (d)
2mv h (a) A and B will have different intensities while
22. What is the ground state energy of B and C will have different frequencies
positronium? (The ground state energy of (b) B and C will have different intensities while
hydrogen is A and C will have different frequencies
–13.6 eV) (c) A and B will have different intensities while
(a) – 3.4 eV (b) – 6.8 eV A and C will have equal frequencies
(c) – 13.6 eV (d) – 27.2 eV (d) A and B will have equal intensities while B
23. The minimum wavelength in Lymann series of and C will have different frequencies
hydrogen spectra is 91.2 nm, the longest 28. Which figure shows the correct force acting
wavelength in this series must be on the body sliding down an inclined plane?
(m mass, fs force of friction)
eh ch
(a) (b) N B
Vc eV fs

eV cV
(c) (d) (a)
ch eh sin
mg mg
24. The glancing angle in a X-rays diffraction C mg cos A
experiment is 30° and the wavelength of the X- N B
rays used is 20 mm. The interplanar spacing of fs
the crystal diffracting these X-rays will be
(a) 40 nm (b) 20 nm (b)
sin
(c) 15 nm (d) 10 nm mg mg mg cos
C A
25. The velocity of a body of rest mass m o is
N sin B
3 mg
c (Where c is the velocity of light in
2
vacuum). The mass of this body is (c) fs
mg mg cos
C A
3 1
(a) mo (b) mo N B
2 2

2 (d) fs
(c) 2mo (d) mo sin
3 mg mg mg cos
C A
EBD_7443
www.jeeneetbooks.in

WWW.IIT-NEET.XYZ

MT-4 Target VITEEE


29. The energy released per fission of a 92 U235 37. In frequency modulation
nucleus is nearly (a) the amplitude of modulated wave varies as
(a) 200 eV (b) 20 eV frequency of carrier wave
(c) 200 MeV (d) 2000 eV (b) the frequency of modulated wave varies as
30. Half-lives of two radioactive substances A and amplitude of modulating wave
B are respectively 20 minutes and 40 minutes. (c) the amplitude of modulated wave varies as
Initially, the sample of A and B have equal number frequency of carrier wave
of nuclei. After 80 minutes, the ratio of remaining (d) the frequency of modulated wave varies as
number of A and B nuclei is frequency of modulating wave
(a) 1 : 16 (b) 4 : 1 38. Two liquids of densities d1 and d2 are flowing
(c) 1 : 4 (d) 1 : 1 in identical capillary tubes uder the same
31. The binding energy per nucleon is largest for pressure difference. If t1 and t2 are time taken
for the flow of equal quantities (mass) of
(a) 56Fe (b) 16O
liquids, then the ratio of coefficient of viscosity
(c) 4He (d) 208Pb
of liquids must be
32. The radius of Ge nucleus is measured to be twice
the radius of 9Be4 nucleus. How many nucleons d1 t 1 t1
(a) (b)
are there in the Ge nucleus ? d2t2 t2
(a) 72 (b) 96
(c) 120 (d) 144 d2 t2 d1t 1
33. A Zener diode has a breakdown voltage of 9.1 V (c) (d)
d1 t 1 d 2t 2
with a maximum power dissipation of 364 mW.
39. Two masses ma and mb moving with velocities
What is the maximum current that the diode can
va and v b in opposite dir ection collide
withstand ?
elastically and after the collision ma and mb
(a) 0.04 A (b) 0.4 A
move with velocities Vb and Va respectively.
(c) 4.0 A (d) 40 A
Then the ratio ma/mb is
34. The voltage across a diode in a full wave rectifier
having input voltage of peak value Vm during Va Vb ma mb
(a) (b)
its non conducting period is Va Vb ma
(a) 0 (b) – Vm
(c) – 2Vm (d) – 4Vm 1
(c) 1 (d)
35. A n-p-n transistor conducts when 2
(a) both collector and emitter are negative with 40. The most suitable device at present for solid
respect to the base state picture tube is
(b) both collector and emitter are positive with (a) LED (b) LCD
respect to the base (c) Silicon (d) Quartz crystal
(c) collector is positive and emitter is negative PART - II : CHEMISTRY
with respect to the base 41. Some Gem stones used to show colour. Ruby
(d) collector is positive and emitter is at same
shows colour due to __________.
potential as the base
(a) d-d transition of Al3+ and Cr3+
36. The diagram of a logic circuit is given below.
The output F of the circuit is represented by (b) d -d transition of Al3+
(c) d -d transition of Cr 3+
W (d) d-d transition of Cr 3+
X 42. The lanthanide contraction is responsible for
F the fact that
(a) Zr and Y have about the same radius
W (b) Zr and Nb have similar oxidation state
(c) Zr and Hf have about the same radius
Y
(d) Zr and Zn have the same oxidation states
(a) W . (X + Y) (b) W . (X . Y) (Atomic numbers : Zr = 40, Y = 39, Nb = 41,
(c) W + (X . Y) (d) W + (X + Y) Hf = 72, Zn = 30)
www.jeeneetbooks.in

WWW.IIT-NEET.XYZ

Mock Test 1 MT-5

43. At equilibrium, if Kp = 1, then 50. Which one of the following ionic species will
(a) G 0 (b) G 1 impart colour to an aqueous solution?
(c) G 1 (d) None of these (a) Ti4+ (b) Cu+
44. Carbon - 14 dating method is based on the fact (c) Zn2+ (d) Cr3+
that: 51. When electrons are trapped into the crystal in
(a) C-14 fraction is same in all objects anion vacancy, the defect is known as :
(b) C-14 is highly insoluble (a) Schottky defect
(c) Ratio of carbon-14 and carbon-12 is constant (b) Frenkel defect
(d) all the above (c) Stoichiometric defect
45. 235 1 236 fission products +
92 U 0n 92 U (d) F-centres
neutrons + 3.20 × 10–11 J 52. The number of atoms contained in a fcc unit
235 cell of a monoatomic substance is
The energy released when 1 g of 92 U finally
(a) 1 (b) 2
undergoes fission is
(c) 4 (d) 6
(a) 12.75 × 108 kJ (b) 16.40 × 107 kJ
(c) 8.20 × 107 kJ (d) 6.50 × 106 kJ 53. The second order Bragg diffraction of X-rays
with = 1.00 Å from a set of parallel planes in a
46. K 3[Al(C 2O 4 )3 ] is called metal occurs at an angle 60º. The distance
(a) Potassium alumino oxalate between the scattering planes in the crystal is
(b) Potassium trioxalateoaluminate (III) (a) 0.575 Å (b) 1.00 Å
(c) Potassium aluminium (III) oxalate (c) 2.00 Å (d) 1.15 Å
(d) Potassium trioxalato aluminate (VI)
54. Identify the correct statement regarding
47. A co-ordination complex compound of cobalt entropy:
has the molecular formula containing five
ammonia molecules, one nitro group and two (a) At absolute zero of temperature, entropy
chlorine atoms for one cobalt atom. One mole of a perfectly crystalline substance is taken
of this compound produces three mole ions in to be zero
an aqueous solution. On reacting this solution (b) At absolute zero of temperature, the
with excess of AgNO3 solution, we get two entropy of a perfectly crystalline substance
moles of AgCl precipitate. The ionic formula is +ve
for this complex would be (c) At absolute zero of temperature, the
(a) [Co(NH3)4 (NO2) Cl] [(NH3) Cl] entropy of all crystalline substances is
(b) [Co (NH3)5 Cl] [Cl (NO2)] taken to be zero
(c) [Co (NH3)5 (NO2)] Cl2 (d) At 0ºC, the entropy of a perfectly crystalline
(d) [Co (NH3)5] [(NO2)2Cl2] substance is taken to be zero
48. Which of thefollowing does not have a metal- 55. Standard Gibb’s free energy change for
carbon bond? isomerization reaction
(a) Al(OC 2 H 5 )3 (b) C 2 H 5MgBr cis-2 pentene trans-2-pentene
(c) K[Pt (C 2 H 4 )Cl 3 ] (d) Ni(CO) 4 is – 3.67 kJ/mol at 400 K. If more trans-2
pentene is added to the reaction vessel, then
49. Which one of the following octahedral
complexes will not show geometric isomerism? (a) more cis-2 pentene is formed
( A and B are monodentate ligands) (b) equilibrium remains unaffected
(a) [MA5B] (b) [MA2B4] (c) additional trans-2 pentene is formed
(c) [MA3B3] (d) [MA4B2] (d) equilibrium is shifted in forward direction
EBD_7443
www.jeeneetbooks.in

WWW.IIT-NEET.XYZ

MT-6 Target VITEEE


56. In the following reaction, how is the rate of 3 5
(a) K(510.1 – 457.6) ln (3.07 × 10 / 2.2 10 )
appearance of the underlined product related to
the rate of disappearance of the underlined 3
(b) 3 . 048 10
reactant ? 5
(c) ln 2. 2 10 457.6 K cal.
BrO 3( aq ) 5Br ( aq ) 6 H (aq )
(d) (510 .1 457 .6 / 52 .5) R ln (307)/2.2) cal.
61. The ionic conductance of Ba2+ and Cl– are
3Br2( l) 3H 2 O ( l) respectively 127 and 76 ohm–1 cm2 at infinite
dilution. The equivalent conductance (in ohm–
d[Br2 ] 5 d[Br ] 1 cm2) of BaCl at infinite dilution will be :
(a) 2
dt 3 dt (a) 139.5 (b) 203
(c) 279 (d) 101.5
d[Br2 ] d[ Br ] 62. On passing a current of 1.0 ampere for 16 min
(b)
dt dt and 5 sec through one litre solution of CuCl2, all
copper of the solution was deposited at
d[Br2 ] 3 d[ Br ] cathode. The strength of CuCl2 solution was
(c)
dt 5 dt (Molar mass of Cu= 63.5; Faraday constant =
96,500 Cmol–1)
d[Br2 ] 3 d[Br ] (a) 0.01 N (b) 0.01 M
(d)
dt 5 dt (c) 0.02 M (d) 0.2 N
57. The rate of reaction between two reactants A 63. Standard electrode potentials are :
and B decreases by a factor of 4 if th e Fe+2/Fe Eº = –0.44; Fe+3/Fe+2 Eº = + 0.77 ; If
concentration of reactant B is doubled. The Fe+2, Fe+3 and Fe blocks are kept together, then
(a) Fe+2 increases
order of this reaction with respect to reactant
(b) Fe+3 decreases
B is:
2
(a) 2 (b) 2 Fe
(c) remains unchanged
(c) 1 (d) 1 Fe 3
58. According to Le-chatelier’s principle, adding (d) Fe+2 decreases
heat to a solid liquid equilibrium will cause 64. The most durable metal plating on iron to
protect against corrosion is
the
(a) nickel plating (b) copper plating
(a) temperature to increase (c) tin plating (d) zinc plating.
(b) temperature to decrease 65. Reaction of CH — CH with RMgX leads to
2 2
(c) amount of liquid to decrease
O
(d) amount of solid to decrease. formation of
59. Which of the following is true at chemical (a) RCHOHR (b) RCHOHCH3
equilibrium? R
(a) ( G)T,p is minimum and ( S)U,V is also (c) RCH2CH2OH (d) CHCH 2OH
minimum R
(b) ( G)T,V is minimum and ( S)U,V is maximum 66. Increasing order of acid strength among
(c) ( G)T,V is maximum and ( S)U,V is zero p-methoxyphenol, p-methylphenol and
p-nitrophenol is
(d) ( G)T,p is zero and ( S)U,V is also zero
(a) p-Nitrophenol, p-Methoxyphenol,
60. The racemisation of - pinene is first order p-Methylphenol
reaction. In the gas the specific reaction rate was (b) p-Methylphenol, p-Methoxyphenol,
found to be 2.2 10 5
mm 1
at 457.6 K and p-Nitrophenol
(c) p-Nitrophenol, p-Meth ylphenol,
3
3.07 10 at 510.1 K. The energy of activation p-Methoxyphenol.
is (d) p-Methoxyphenol, p-Methylphenol,
p-Nitrophenol
www.jeeneetbooks.in

WWW.IIT-NEET.XYZ

Mock Test 1 MT-7

67. When phenol is treated with excess bromine 73. Aspirin is an acetylation product of
water. It gives (a) o-hydroxybenzoic acid
(a) m-Bromophenol (b) o-dihydroxybenzene
(b) o-and p-Bromophenols (c) m-hydroxybenzoic acid
(c) 2,4-Dibromophenol (d) p-dihydroxybenzene
(d) 2,4, 6-Tribomophenol. 74. Which of the following is incorrect?
68. In the reaction: (a) NaHSO3 is used in detection of carbonyl
CH3 compound
| (b) FeCl3 is used in detection of phenolic group
Heated
CH3 CH CH 2 O CH 2 CH3 HI (c) Tollen reagent is used in detection of
Which of the following compounds will be unsaturation
formed? (d) Fehling solution is used in detection of
glucose
(a) CH3 CH CH3 CH3CH 2 OH
| 75. Formic acid is obtained when
CH3 (a) Calcium acetate is heated with conc. H2SO4
(b) Calcium formate is heated with calcium
(b) CH3 CH CH 2 OH CH3CH3 acetate
| (c) Glycerol is heated with oxalic acid at 373 K
CH3
(d) Acetaldehyde is oxidised with K2Cr2O7 and
CH3 H2SO4.
| 76. When aniline reacts with oil of bitter almonds
(c) CH3 CH CH 2 OH CH3 CH 2 I
(C 6 H 5CHO ) condensation takes place and
CH3 benzal derivative is formed. This is known as
| (a) Million's base
(d) CH3 CH CH 2 I CH3CH 2OH
(b) Schiff's reagent
69. n-Propyl alcohol and isopropyl alcohol can be (c) Schiff's base
chemically distinguished by which reagent? (d) Benedict's reagent
(a) PCl5 77. The consituent of the powerful explosive RDX
(b) Reduction is formed during the nitration of
(c) Oxidation with potassium dichromate (a) toluene (b) phenol
(d) Ozonolysis (c) glycerol (d) urotropine
70. The thermodynamic efficiency of cell is given 78. Aniline in a set of reactions yielded a product
by D.
(a) H/ G (b) nFE/ G
(c) nFE/ H (d) nFE
71. Pinacolone is
(a) 2, 3-Dimethyl-2 3-butanediol
(b) 3, 3-Dimethyl-2 butanone
(c) I-Phenyl-2Propanone
(d) 1,1-Diphenyl-2-ethandiol. The structure of the product D would be:
72. An ester is boiled with KOH. The product is (a) C6H5NHOH
cooled and acidified with concentrated HCl. A
(b) C6H5NHCH2CH3
white crystalline acid separates. The ester is
(a) Methyl acetate (b) Ethyl acetate (c) C6H5CH2NH2
(c) Ethyl formate (d) Ethyl benzoate (d) C6H5CH2OH
EBD_7443
www.jeeneetbooks.in

WWW.IIT-NEET.XYZ

MT-8 Target VITEEE


C N (a) 4 (b) 3
(c) 2 (d) 1
H 3O 83. If tan x + tan (x + /3) + tan (x + /3) = 3, then
79. + CH3MgBr P
(a) tan x = 1 (b) tan 2x = 1
OCH3 (c) tan 3x =1 (d) none of these.
Product 'P' in the above reaction is 84. If b and c are any two non-collinear mutually
OH O
CH – CH3
perpendicular unit vectors and a is any vector,,
C – CH3

(a) a . ( b c)
(b) then ( a b) b ( a . c) c (b c)
| b c |2
OCH3 OCH3
is equal to :
COOH
CHO (a) a (b) 2 a

(c) 3a (d) None


(c) (d)
OCH3 85. Which one of the following graphs represents
OCH3 the function
80. What is the decreasing order of basicity of y = 1+ |x| for all x R?
primary, secondary and tertiary ethylamines and Y
NH3 ?
(–1,0) (1,0)
(a) NH 3 C 2 H 5 NH 2 (C 2 H 5 ) 2 NH (a) O X

(C 2 H 5 )3 N
(b) (C2 H 5 )3 N (C2 H 5 )2 NH Y

C2 H 5NH 2 NH 3
(–1,0) (1,0)
(b) O X
(c) (C2 H 5 )2 NH C2 H 5 NH 2
(C 2 H 5 )3 N NH 3
(d) (C 2 H 5 ) 2 NH (C 2 H 5 ) 3 N Y
(0,1)
C 2 H 5 NH 2 NH 3 .
(c) X
PART - III : MATHEMATICS O

81. If R be a relation “less than” from A = {1, 2, 3,


4} B = {1, 3, 5}, i.e. (a, b) R iff a < b, then Y

ROR–1 is (0,1)
(1,0)
(a) {(1, 3), (1, 5), (2, 3), (2, 5), (3, 5), (4, 5)} (d) X
(–1,0) O
(b) {(3,1), (5, 1), (3, 2), (5, 2), (5, 3), (5, 4)}
(c) {(3, 3), (3, 5), (5, 3), (5, 5)}
(d) {(3, 3), (3, 4), (4, 5)}
82. The smallest positive integer n for which 86. The point (2a, a) lies inside the region bounded
by the parabola x2 = 4y and its latus rectum.
1 i
n Then,
1 is (a) 0 a 1 (b) 0 < a < 1
1– i
(c) a > 1 (d) a < 0
www.jeeneetbooks.in

WWW.IIT-NEET.XYZ

Mock Test 1 MT-9

87. The linear inequations for which the shaded 91. Three concurrent lines with direction cosines
area in the following figure is the solution set, l1, m1, n1; l2, m2, n2 and l3, m3, n3 are coplanar
are l1 m1 n1
then l 2 m 2 n2
l3 m 3 n3
(a) 0 (b) 1
(c) –1 (d) 2
x–y=1 92. The resultant moment of three forces
î 2ˆj 3k̂ , 2î 3ˆj 4k̂ and î ˆj k̂ acting
on a particle at a point P (0, 1, 2) about the
point A (1, –2, 0) is
X X (a) 6 2 (b) 140
x + 2y = 8 (c) 21 (d) None
2x + y = 2 th th th
93. If the p , q , r , terms of a G.P. are respectively x,

(a) x + y 1, 2x + y 2, x – 2y 8, x 0, y 0 y, z, then the value of x q r y r p z p q is


(b) x – y 1, 2x + y 2, x + 2y 8, x 0, y 0 (a) xyz (b) pqr
(c) x – y 1, 2x + y 2, x + 2y 8, x 0, y 0 (c) 1 (d) none of these
(d) x + y 1, 2x + y 2, x + 2y 8, x 0, y 0 94. A signal which can be green or red with
88. The roots of the equation probability 4/5 and 1/5 respectively, is received
(3 – x)4 + (2 – x)4 = (5 – 2x)4 are by station A and then trasmitted to station B.
(a) two real and two imaginary The probability of each station receiving the
(b) all imaginary signal correctly is 3/4. If the signal received at
(c) all real station B is given, then the probability that
the original signal is green, is
(d) none of these
89. Coloured balls are distributed in four boxes as 3 6
(a) (b)
shown in the following table 5 7
Colo ur 20 9
Box (c) (d)
Black W hite Red Blue 23 20
I 3 4 5 6 95. If A and B are two events, the probability that
II 2 2 2 2 at most one of these events occurs is :
III 1 2 3 1 (a) P(A' ) P(B' ) P(A ' B' )
IV 4 3 1 5 (b) P(A' ) P(B' ) P(A B) 1
A box is selected at random and then a ball is (c) P(A B' ) P(A' B) P(A' B' )
randomly drawn from the selected box. The
(d) All above are correct.
colour of the ball is black. Probability that the
96. If x, y, z are distinct number in A.P. with common
ball drawn from Box III, is differences d and the rank of the matrix
(a) 0.161 (b) 0.162
(c) 0.165 (d) 0.104 4 5 x
p q y r z 5 6 y is 2 then the values of d and k are :
90. If p x q r z 0 , then the value of 6 k z
p x q y r x
(a) ; arbitrary number
p q r 4
is (b) arbitrary number, 7
x y z
(a) 0 (b) 1 x
(c) x, 5 (d) ,6
(c) 2 (d) 4pqr 2
EBD_7443
www.jeeneetbooks.in

WWW.IIT-NEET.XYZ

MT-10 Target VITEEE


104. The number of all five digit numbers which are
97. The solution of sin–1 x – sin –12x = is
3 divisible by 4 that can be formed from the
digits 0, 1, 2, 3, 4 (without repetition) is
1 1
(a) (b) (a) 36 (b) 30
3 4
(c) 34 (d) None of these
3 1 2
105. If the parabola x = ay makes an intercept of
(c) (d) ±
2 2 length 40 on the line y – 2x = 1, then a is
2
1 1 u 1 2u equal to
98. If y tan cos 1 sin 1
2 1 u2 2 1 u2 (a) 1 (b) 3
(c) –1 (d) 2
2u dy
and x , then 1 1
1 u 2 dx 106. If sin x dx f (x ) sin x g (x ) c . Then
(a) –1 (b) 0
(c) 1 (d) None of these (a) f(x) = x, g(x) 1 x2
99. If the sides of a triangle ABC are in A.P., and a is
the smallest side, then cos A equals to : (b) f(x) = 1 x 2 , g(x) = 1 + x2
3c 4b 3c 4b
(a) (b) x
2c 2b (c) f(x) = x, g(x) =
1 x2
4c 3b
(c) (d) none of these.
2c (d) none of these.
100. The octal numeral 23450 is equal to the binary 107. The line x cos y sin p is a tangent to
number
(a) 10011100101000 (b) 10011100011000 x2 y2
(c) 10111000101000 (d) 10011100111000 the ellipse 1 if
a2 b2
101. nCr + 2 nCr–1 + nCr–2 is equal to:
(a) n+2Cr (b) nCr+1 (a) p a cos b sin
(c) n–1
Cr+1 (d) None of these
102. The domain of the function defined by (b) p2 a 2 cos 2 b 2 sin 2

f x sin 1 x 1 is (c) p2 a 2 sec 2 b 2 tan 2


(a) [1, 2] (b) [–1, 1]
(c) [0, 1] (d) None of these (d) p a sec b tan
103. Let f(x) be a continuous function such that the 3x 4
area bounded by the curve y = f(x), x-axis and 108. If f : B A is defined by f x and g
5x 7
a2 a 7x 4
the lines x = 0 and x = a is sin a + cos a, : A B is defined by g x , where A
2 2 2 5x 3
3 7
then f = =R– and B = R – and IA is an
5 5
2
identity function on A and IB is identity
function on B, then
1
(a) 1 (b) (a) fog = IA and gof = IA
2
(b) fog = IA and gof = IB
1 (c) fog = IB and gof = IB
(c) (d) None of these
3 (d) fog = IB and gof = IA
www.jeeneetbooks.in

WWW.IIT-NEET.XYZ

Mock Test 1 MT-11

109. In [0,1] Lagranges Mean Value theorem is NOT 114. The locus of a point, such that the sum of the
applicable to squares of its distances from the planes
x + y + z = 0, x – z =0 and x – 2y + z = 0 is 9, is
1 1
x x (a) x2 y2 z2 3
2 2
(a) f (x) 2
1 1 (b) x 2 y 2 z 2
x x 6
2 2
(c) x2 y2 z2 9
sin x
(b) , x 0 (d) x 2 y2 z2 12
f ( x) x
1, x 0 115. The distance of the point (–5, –5, –10) from
the point of intersection of the line
(c) f ( x) xx
r. 2iˆ ˆj 2kˆ 3iˆ 4ˆj 2kˆ and the
(d) f ( x) x
plane r · ˆi ˆj kˆ 5 is
110. With respect to multiplication modulo 5, the
set G = {1, 2, 3, 4} is (a) 13 (b) 12
(a) a finite abelian group of order 4 (c) 4 15 (d) 10 2
(b) not a finite abelian group of order 4 116. The asymptotes of the hyperbola
(c) both (a) and (b) xy – 3x + 4y + 2 = 0 are
(d) none (a) x = – 4 and y = 3 (b) x = 4 and y = –3
111. Let f be function such that f (x + y) = f(x) + f(y) (c) x = –4 and y = –4 (d) x = –3 and y = 3
for all x and y and f(x) = (2x2 + 3x) g (x) for all x,
117. The integer n for which
where g (x) is continuous and g(0) = 3. Then
f’(x) is equal to (cos x 1)(cos x e x )
lim is a finite non-zero
(a) 9 (b) 3 x 0 xn
(c) 6 (d) none of these number is
112. The differential equation (a) 1 (b) 2
(1 y 2 ) x dx (1 x 2 ) y dy 0 (c) 3 (d) 4
represents a family of : 118. The order and degree of the differential
(a) ellipses of constant eccentricity equation whose solution is
(b) ellipses of variable eccentricity y cx c 2 3c 3 2 2 , where c is a parameter,,
(c) hyperbolas of constant eccentricity
are respectively
(d) hyperbolas of variable eccentricity
(a) 1 and 4 (b) 1 and 3
1 x (c) 1 and 2 (d) none of these
113. dx is equal to 119. A football is inflated by pumping air in it.
x
When it acquires spherical shape its radius
1 x 1 increases at the rate of 0.02 cm/s. The rate of
(a) 2 1 x 2log c increase of its volume when the radius is 10 cm
1 x 1 is ___________ cm/s
(a) 0 (b) 2
1 x 1 (c) 8 (d) 9
(b) 4 1 x 2log c
1 x 1 120. Let B is a set containing the elements of
boolean algebra and a, b B , then a. (a + b) =
1 x 1
(c) 4 1 x 2log c (a) b (b) a . b
1 x 1 (c) a + b (d) a
(d) None of these.
EBD_7443
www.jeeneetbooks.in

WWW.IIT-NEET.XYZ

MT-12 Target VITEEE


PART - IV : (ENGLISH) 122. The character of a nation is the result of its-
Direction (Qs. 121 - 123) : Read the passage carefully (a) gross ignorance
and answer the questions given below. (b) cultural heritage
(c) socio-political conditions
At this stage of civilization, when many nations are
brought in to close and vital contact for good and (d) mentality
evil, it is essential, as never before, that their gross 123. The need for a greater understanding between
ignorance of one another should be diminished, that nations-
they should begin to understand a little of one (a) is more today than ever before
another's historical experience and resulting mentality. (b) was always there
It is the fault of the English to expect the people of
(c) is no longer there
other countries to react as they do, to political and
international situations. Our genuine goodwill and (d) will always be there
good intentions are often brought to nothing, because 124. Direction: Choose the correct meaning of the
we expect other people to be like us. This would be idiom from the options given below.
corrected if we knew the history, not necessarily in At one's wit's end
detail but in broad outlines, of the social and political (a) Perplexed
conditions which have given to each nation its present
(b) Clear Up
character.
121. According to the author the 'Mentality' of a (c) Explain
nation is mainly product of its- (d) Enlighten
(a) present character 125. Choose the best pronunciation of the word,
(b) international position Poem, from the following options.
(c) politics (a) poy-em (b) poe-um
(d) history
(d) poh-om (d) poi-um
www.jeeneetbooks.in

WWW.IIT-NEET.XYZ

MOCK
VITEEE Mock Test Paper
2
Max. Marks : 125 Time : 2½ hrs
PART - I : PHYSICS
10 10 10
A B
1. In an inelastic collision, which of the following
does not remain conserved? 10 10
(a) Momentum
(b) kinetic energy
(c) Total energy C D
(d) Neither momentum nor kinetic energy 10 10 10
2. Three capacitors C1, C2 and C3 are connected to (a) 10 (b) 20
a battery as shown. With symbols having their (c) 30 (d) 40
usual meanings, the correct conditions are 6. According to stokes law, the relation between
terminal velocity (vt) and viscosity of the
V
Q2 2 C 2 medium (n) is
V1
Q1 C1
(a) vt = n (b) vt n
Q3 C3 1
(c) vt
V3 n
V (d) vt is independent of n.
7. Kirchhoff’s first law, i.e. i = 0 at a junction, deals
with the conservation of
(a) Q1 = Q2 = Q3 and V1 = V2 = V
(a) charge (b) energy
(b) V1 = V2 = V3 = V
(c) Q1 = Q2 + Q3 and V = V1 + V2 (c) momentum (d) angular momentum
(d) Q2 = Q3 and V2 = V3 8. The potential difference between the terminals
3. A parallel plate condenser with oil between the of a cell in an open circuit is 2.2 V. When a resistor
plates (dielectric constant of oil K = 2) has a of 5 is connected across the terminals of the
capacitance C. If the oil is removed, then cell, the potential difference between the
capacitance of the capacitor becomes terminals of the cell is found to be 1.8 V. The
internal resistance of the cell is
(a) 2C (b) 2C
7 10
(a) (b)
C C 12 9
(c) (d)
2 2 9 12
4. At the centre of a cubical box + Q charge is (c) (d)
10 7
placed. The value of total flux that is coming out 9. Direct current is passed through a copper
a wall is sulphate solution using platinum electrodes. The
(a) Q / o (b) Q / 3 o elements liberated at the electrodes are
(a) copper at anode and sulphur at cathode
(c) Q / 4 o (d) Q / 6 o
(b) sulphur at anode and copper at cathode
5. What will be the equivalent resistance of circuit (c) oxygen at anode and copper at cathode
shown in figure between two points A and D (d) copper at anode and oxygen at cathode
EBD_7443
www.jeeneetbooks.in

WWW.IIT-NEET.XYZ

MT-14 Target VITEEE


10. Two electric bulbs, one of 200 V, 40W and other (a) 10 s (b) 0.1 s
of 200 V, 100W are connected in a domestic (c) 0.01 s (d) 1 s
circuit. Then 16. Two coils of self inductances 2 mH and 8 mH are
(a) they have equal resistance placed so close together that the effective flux
(b) the resistance of 40W bulb is more than in one coil is completely linked with the other.
100W bulb The mutual inductance between these coils is
(c) the resistance of 100W bulb is more than (a) 6 mH (b) 4 mH
40 W bulb (c) 16 mH (d) 10 mH
(d) they have equal current through them 17. Which one of the following electromagnetic
11. A beam of metal supported at the two edges is radiations has the smallest wavelength?
loaded at the centre. The depression at the (a) Ultraviolet waves (b) X-rays
centre is proportional to (c) -rays (d) Microwaves
18. Which one of the following phenomena is not
explained by Huygen’s constr uction of
wavefront?
(a) Refraction (b) Reflection
(a) Y 2 (b) Y
(c) Diffraction (d) Orgin of spectra
(c) 1/Y (d) 1/Y 2
19. Two light waves having their intensities in the
12. A charged particle of charge q and mass m enters
ratio 16 : 9 interfere to produce interference
perpendicularly in a magnetic field B . Kinetic pattern. What is the ratio of maximum intensity
energy of the particle is E; then frequency of to minimum intensity in this pattern ?
rotation is (a) 4 : 3 (c) 25 : 7
(b) 625 : 49 (d) 49 : 1
qB qB
(a) (b) 20. A slab of thickness t and refractive index 1.5 is
m 2 m
placed in between point A and B as shown in the
qBE qB figure given below. The optical path length
(c) (d) between A and B is
2 m 2 E
13. If N is the number of turns in a coil, the value of t
self inductance varies as
(a) N0 (b) N x y
A B
(c) N2 (d) N–2
14. In an A.C. circuit with voltage V and current I
the power dissipated is n = 1.5
(a) dependent on the phase between V and I
t
1 (a) x y (b) (x + t + y)
(b) VI 2
2
3 5
(c) x t y (d) x t y
(c) 1 VI 2 2
2
21. If according to the Bohr model of the hydrogen
(d) VI atom, the ionization energy of the atom in its
15. A magnetic field of 2 × 10–2 T acts at right angles ground state is 13.6 eV, then the energy required
to a coil of area 100 cm2, with 50 turns. The to ionize the atom from its first excited state will
average e.m.f. induced in the coil is 0.1 V, when it be
is removed from the field in t sec. The value of t (a) 6.8 eV (b) 3.4 eV
is (c) 1.7 eV (d) 0.85 eV
www.jeeneetbooks.in

WWW.IIT-NEET.XYZ

Mock Test 2 MT-15

22. Match List-I with List-II and select the correct 26. Radiations of two photon’s energy, twice and
answer using the codes given belowthe Lists ten times the work function of metal are incident
List-I List-II on the metal surface successsively. The ratio of
maximum velocities of photoelectrons emitted
1 1 in two cases is
A. Balmer series 1. v RH
32 n2 (a) 1 : 2 (b) 1 : 3
(c) 1 : 4 (d) 1 : 1
1 1
B. Brakett series 2. v RH 2
27. If an electron and positron annihilate, then the
1 n2 energy released is
1 1 13 13
(a) 3.2 10 J (b) 1.6 10 J
C. Lyman series 3. v RH 2 2
2 n
(c) 4.8 10 13 J (d) 6.4 10 13 J
1 1 28. The ratio of molar specific heat at constant
D. Paschen series 4. v RH pressure CP, to molar specific heat at constant
2 2
4 n
volume Cv for a monoatomic gas is
Codes :
A B C D CP 3 CP 5
(a) (b)
(a) 3 2 1 4 CV 5 CV 3
(b) 3 4 2 1
(c) 4 3 2 1 CP 7 CP 9
(c) (d)
(d) 2 3 1 4 CV 9 CV 7
23. A force Fx acts on a particle such that its position 29. Which of the following are true regarding forces
x changes as shown in the figure.
between nucleons inside the nucleus :
1. attractive in nature
2. electrical in nature
3 3. extremely short range
Fx (N)

4. strongest forces in nature


Which of the above are correct ?
0 5 15 20 (a) 1, 2 and 4 (b) 2 and 3
x(m)
(c) 1, 3 and 4 (d) 3 and 4
The work done by the particle as it moves from x 2 2 3 1
30. 1H 1H 2 He 0n 13 MeV
= 0 to 20 m is
(a) 37.5 J (b) 10 J In the above nuclear reaction, the binding energy
(c) 45 J (d) 22.5 J
of 21 H is 2.2 MeV. The binding energy of 23 He is
24. The population inversion necessary for laser
action used in solid state lasers is (a) 4.4 MeV (b) 8.6 MeV
(a) electrical discharge (c) 13 MeV (d) 17.4 MeV
(b) inelastic atom-atom collision 31. A radioactive nucleus has a half-life of 100 years.
(c) direct conversion If the number of nuclei at t = 0 is N0, what will be
(d) optical pumping the number of nuclei which have decayed in 300
25. The photo electric threshold of Tungsten is years ?
2300Å. The energy of the electrons ejected from 7N0 N0
the surface by ultraviolet light of wavelength (a) (b)
8 2
1800Å is
(a) 0.15 eV (b) 1.5 eV 3N 0 N0
(c) (d)
(c) 15 eV (d) 150 eV 4 8
EBD_7443
www.jeeneetbooks.in

WWW.IIT-NEET.XYZ

MT-16 Target VITEEE


32. The diagram of a logic circuit is given below. (a) f3 – f2 (b) f4 – f2

W f 4 f2
(c) (d) f3 – f1
X 2
Ic
36. In common emitter amplifier the is 0.98.
W Ie
Y The current gain will be
(a) 4.9 (b) 7.8
The output F of the circuit is represented by (c) 49 (d) 78
(a) W + (X + Y) (b) W + (X.Y) 37. A student takes 50gm wax (specific heat = 0.6
(c) W, (X + Y) (d) W, (X.Y) kcal/kg°C) and heats it till it boils. The graph
33. NAND and NOR gates are called universal gates between temperature and time is as follows. Heat
primarily because they supplied to the wax per minute and boiling point
(a) are available universally are respectively

Temperature (°C)
250
(b) can be combined to produce OR, AND and 200
NOT gates (a) 500 cal, 50°C 150
(c) are widely used in Integrated circuit 100
(b) 1000 cal, 100°C 50
packages 0
(d) are easiest to manufacture 1 2 3 4 567 8
(c) 1500 cal, 200°C Time (Minute)
34. Of the diodes shown in the following diagrams,
which one is reverse biased ? (d) 1000 cal, 200°C
+10 V 38. Consider telecommunication through optical
fibres. Which of the following statements is not
R true ?
(a)
+5 V (a) Optical fibres may have homogeneous core
with a suitable cladding
(b) (b) Optical fibres can be of graded refractive
–12 V R
index
–5 V (c) Optical fibres are subject to electromagnetic
interference from outside
(c) R (d) Optical fibres have extremely low
transmission loss
–10 V 39. What should be the maximum acceptance angle
at the aircore interface of an optical fibre if n 1
+5 V
and n2 are the refractive indices of the core and
R the cladding, respectively
(d)
(a) sin 1 (n 2 n1 ) (b) sin 1
n12 n 22

35. The frequency response curve of RC coupled 1 n2 1 n1


(c) tan (d) tan
amplifier is shown in figure. The band with of n1 n2
the amplifier will be
40. The length of a metal is 1 when the tension in it
is T1 and is 2 when the tension is T2. The original
length of the wire is
A max
0.707 A max

1 2 1T2 2T1
(a) (b)
2 T1 T2

1T2 2T1
(c) T2 T1
(d) T1T2 1 2
f1 f 2 f3 f 4
www.jeeneetbooks.in

WWW.IIT-NEET.XYZ

Mock Test 2 MT-17

PART - II : CHEMISTRY 48. For orthorhombic system axial ratios are


41. Identify the correct statement for change of Gibbs a b c and the axial angles are
energy for a system ( Gsystem) at constant
temperature and pressure : (a) 90
(a) If Gsystem = 0, the system has attained (b) 90
equilibrium
(b) If Gsystem = 0, the system is still moving in (c) 90 , 90
a particular direction (d) 90
(c) If G system < 0, the process is not 49. Which one of th e following will show
spontaneous paramagnetism corresponding to 2 unpaired
(d) If G system > 0, the process is not
electrons?(Atomic numbers : Ni = 28, Fe = 26)
spontaneous
(a) [Fe F6]3– (b) [Ni Cl4]2–
42. Which is a planar molecule ?
(c) [Fe (CN)6]3– (d) [Ni (CN)4]2–
(a) XeO4 (b) XeF4
50. In gaseous equilibrium the correct relation
(c) XeOF4 (d) XeO2F2
between Kc and Kp is
43. Considering entropy (S) as a thermodynamic
parameter, the criterion for the spontaneity of (a) Kc K p ( RT) n (b) K p K c ( RT) n
any process is
Kc Kp
(a) Ssystem Ssurroundings 0 (c) (K p ) n (d) (K c ) n
RT RT
Ssystem Ssurroundings 0 51. Which one of the following reaction occurs at
(b)
the cathode?
(c) Ssystem 0 only (a) 2OH H 2 O O 2e

(d) Ssurroundings 0 only (b) Ag Ag e


44. In the solid state, MgO has the same structure (c) Fe 2 Fe 3 e
as that of sodium chloride. The number of (d) Cu 2 2e Cu
oxygens surrounding each magnesium in MgO
is 52. Which of the following coordination compounds
(a) 6 (b) 1 would exhibit optical isomerism?
(c) 2 (d) 4 (a) pentamminenitrocobalt(III) iodide
45. A solid with high electrical and thermal (b) diamminedichloroplatinum(II)
conductivity from the following is (c) trans-dicyanobis (ethylenediamine)
(a) Si (b) Li chromium (III) chloride
(c) NaCl (d) Ice (d) tris-(ethylendiamine) cobalt (III) bromide
46. Four successive members of the first row 53. Lanthanoids are
transition elements are listed below with their (a) 14 elements in the sixth period (atomic no.
atomic numbers. Which one of them is = 90 to 103) that are filling 4f sublevel
expected to have the highest third ionization (b) 14 elements in the seventh period (atomic
enthalpy? no. = 90 to 103) that are filling 5f sublevel
(a) Vanadium (Z = 23) (c) 14 elements in the sixth period (atomic no.
(b) Chromium (Z = 24) = 58 to 71) that are filling 4f sublevel
(c) Manganese (Z = 25) (d) 14 elements in the seventh period (atomic
(d) Iron (Z = 26) no. = 58 to 71) that are filling 4f sublevel
47. Which one of the following is expected to 60
54. The radioactive isotope 27 Co which is used in
exhibit optical isomerism?
(en = ethylenediamine) the treatment of cancer can be made by (n, p)
(a) cis-[Pt(NH3)2 Cl2] reaction. For this reaction the target nucleus is
(b) trans-[Pt(NH3)2Cl 2] (a) 59 (b) 59
(c) cis-[Co(en)2Cl 2] 28 Ni 27 Co
(d) trans-[Co(en)2Cl 2] (c) 60 (d) 60
28 Ni 27 Co
EBD_7443
www.jeeneetbooks.in

WWW.IIT-NEET.XYZ

MT-18 Target VITEEE


55. The age of most ancient geological formations 62. The standard e.m.f. of a galvanic cell involving
is estimated by cell reaction with n = 2 is found to be 0.295 V at
(a) Potassium–argon method 25°C. The equilibrium constant of the reaction
(b) Carbon-14 dating method would be (Given F = 96500 C mol –1 ;
(c) Radium-silicon method R = 8.314JK–1mol–1)
(d) Uranium-lead method.
56. The activation energy for a simple chemical
(a) 2.0 1011 (b) 4.0 1012
reaction A B is Ea in forward direction. The (c) 1.0 10 2 (d) 1.0 1010
activation energy for reverse reaction 63. In the silver plating of copper, K[Ag(CN)2] is
(a) Is always double of Ea used instead of AgNO3. The reason is
(b) Is negative of Ea (a) A thin layer of Ag is formed on Cu
(c) Is always less than Ea (b) More voltage is required
(d) Can be less than or more than E a (c) Ag+ ions are completely removed from
57. Select the rate law that corresponds to data solution
shown for the following reaction
(d) Less availability of Ag+ ions, as Cu cannot
A+B products.
displace Ag from [Ag(CN)2]– ion
Exp. [A] [B] Initial rate
64. If 0.5 amp current is passed through acidified
1 0.012 0.035 0.1
silver nitrate solution for 100 minutes. The mass
2 0.024 0.070 0.8
3 0.024 0.035 0.1 of silver deposited on cathode, is (eq.wt.of silver
4 0.012 0.070 0.8 nitrate = 108)
(a) rate = k [B]3 (b) rate = k [B]4 (a) 2.3523 g (b) 3.3575 g
(c) rate = k [A] [B]3 (d) rate = k [A]2 [B]2 (c) 5.3578 g (d) 6.3575 g
58. The plot of concentration of the reactant vs. time 65. For the reaction A + B C + D. The
for a reaction is a straight line with a negative variation of the concentration of the products is
slope. The reaction follows a given by the curve
(a) zero order rate equation Y
Z
(b) first order rate equation
(c) second order rate equation
Conc
(d) third order rate equation
59. A substance 'A' decomposes by a first order W
reaction starting initially with [A] = 2.00 m and X
Time
after 200 min, [A] becomes 0.15 m. For this (a) Z (b) Y
reaction t1/2 is (c) W (d) X
(a) 53.72 min (b) 50.49 min 66. Acetamide and ethylamine can be distinguished
(c) 48.45 min (d) 46.45 min
by reacting with
60. Which one of the following information can be
(a) Aqueous HCl and heat
obtained on the basis of Le Chatelier principle?
(b) Aqueous NaOH and heat
(a) Dissociation constant of a weak acid
(c) Acidified KMnO4
(b) Entropy change in a reaction
(d) Bromine water.
(c) Equilibrium constant of a chemical reaction
(d) Shift in equilibrium position on changing 67. Aniline when diazotized in cold and when
value of a constraint. treated with dimethyl aniline gives a coloured
61. Equivalent conductances of NaCl, HCl and product. Its structure would be
CH3COONa at infinite dilution are 126.45, 426.16 (a) CH3NH N=N NHCH3
and 91 ohm–1 cm2 respectively. The equivalent
conductance of CH3COOH at infinite dilution (b) CH3 N=N NH2
would be
(c) (CH3)2N N=N
(a) 101.38 ohm–1 cm2 (b) 253.62 ohm–1 cm2
(c) 390.71 ohm–1 cm2 (d) 678.90 ohm–1 cm2 (d) (CH3)2N NH
www.jeeneetbooks.in

WWW.IIT-NEET.XYZ

Mock Test 2 MT-19

68. Electrolytic reduction of nitrobenzene in weakly 74. Ethanol and dimethyl ether form a pair of
acidic medium gives functional isomers. The boiling point of ethanol
(a) N-Phenylhydroxylamine is higher than that of dimethyl ether, due to the
(b) Nitrosobenzene presence of
(c) Aniline (a) H-bonding in ethanol
(d) p-Hydroxyaniline (b) H-bonding in dimethyl ether
69. If an organic compound shows a streching (c) CH3 group in ethanol
( C = O) frequency of 1720 cm–1. The organic (d) CH3 group in dimethyl ether
compound is __________. 75. Propan - 1- ol may be prepared by the reaction of
(a) An aliphatic aldehyde propene with
(b) An , unsaturated aldehyde (a) H3BO3
(c) A phenolic aldehyde (b) H2SO4/H2O
(d) An aromatic aldehyde (c) B2H6,NaOH–H2O2
70. In the given reaction, O
||
NO 2 X
(d) CH 3 C O O H
Sn
76. Sodium formate on heating yields
HCl (a) Oxalic acid and H2
X- stands for (b) Sodium oxalate and H2
(a) – NH 2 (b) SnCl2 (c) CO2 and NaOH
(d) Sodium oxalate.
(c) – Cl (d) – NH 3 Cl 77. An ester (A) with molecular fomula, C9H10O2
71. In a set of reactions acetic acid yielded a was treated with excess of CH3MgBr and the
product D. complex so formed was treated with H2SO4 to
SOCl2 Benzene
give an olefin (B). Ozonolysis of (B) gave a ketone
CH3COOH A B with molecular formula C8H8O which shows +ve
Anhy.AlCl3
iodoform test. The structure of (A) is
HCN H.OH
C D (a) C6H5COOC2H5
The structure of D would be: (b) C2H5COOC6H5
(c) H3COCH2COC6H5
COOH CN (d) p — H 3CO — C 6 H 4 — COCH 3
CH2 – C – CH3 C – CH3
(a) (b) 78. Self condensation of two moles of ethyl acetate
OH OH
in presence of sodium ethoxide yields
OH OH (a) acetoacetic ester
CH2 – C – CH3 C – COOH (b) methyl acetoacetate
(c) (d)
CN CH3
(c) ethyl propionate
72. Ketones (d) ethyl butyrate
[ R — C — R1 , where R = R1 = alkyl groups]\ 79. Methanol is industrially prepared by
|| (a) Oxidation of CH4 by steam at 900°C
O (b) Reduction of HCHO using LiAIH4
can be obtained in one step by (c) Reaction HCHO with a solution of NaOH
(a) oxidation of primary alcohols (d) Reduction of CO using H2 and ZnO – Cr2O3.
(b) hydrolysis of esters 80. Indicate which of the nitrogen compounds
(c) oxidation of tertiary alcohols amongst th e following would undergo
(d) reaction of acid halides with alcohols Hoffmann’s reaction, i,e., reaction with Br 2 and
73. Phenylmethyl ketone can be converted into strong KOH to furnish the primary amine (R –
ethylbenzene in one step by which of the NH2) ?
following reagents?
(a) R CO NHCH 3 (b) R – CO – ONH4
(a) LiAlH4 (b) Zn-Hg/HCl
(c) NaBH4 (d) CH3MgI (c) R – CO – NH2 (d) R – CO – NHOH.
EBD_7443
www.jeeneetbooks.in

WWW.IIT-NEET.XYZ

MT-20 Target VITEEE


PART - III : MATHEMATICS `15 on each magazine A and B respectively.
These are processed on three machines I, II &
81. If sin (y + z – x), sin (z + x – y) and sin (x + y – z)
III and total time in hours available per week on
are in A.P. (with x, y, z /2), then tan x, tan y each machine is as follows:
and tan z are in
(a) A.P. (b) G.P. Magzine A(x) B(y) Time available
(c) H.P. (d) none of these. Machine
82. The number of roots of equation I 2 3 36
1 1 II 5 2 50
tan 2x tan is 3x
4 III 2 6 60
(a) 0 (b) 1
(c) 2 (d) infinite The number of constraints is
83. In order that the equation (a) 3 (b) 4
(c) 5 (d) 6
2ax(ax nc) (n 2 2)c 2 0 may have real
88. If
roots, n belongs to interval
sin 3 x cos 5 dx A sin 4 x B sin 6 x C sin 8 x D .
(a) [ 2, 2] (b) ( , 2)
Then
(c) (2, ) (d) ( , )
84. The portion of a tangent to a parabola y2 = 4ax 1 1 1
(a) A , B , C , D R
cut off between the directrix and the curve 4 3 8
subtends an angle at the focus, where =
1 1 1
(b) A , B , C , D R
(a) (b) 8 4 3
4 3
1 1
2 (c) A 0, B , C , D R
(c) (d) 6 8
2 3
(d) none of these.
85. There are 25 trees at equal distances of 5 meters
89. In a ABC, the area of the triangle is
in a line with a well, the distance of the well from
the nearest tree being 10 metres. A gardener b 2 (c a ) 2 , then tan B is equal to
waters all the trees separately starting from the
well and he returns to the well after watering 1 8
(a) (b)
each tree to get water for the next. The total 4 15
distance the gardener will cover in order to water
all the trees is 1 15
(c) (d)
(a) 3550 m (b) 3434 m 2 8
(c) 3370 m (d) 3200 m ( x) log 5 log 3 x , then ' (e) =
90. If
86. If nCr denotes the number of combination of n (a) 1 (b) e loge5
things taken r at a time, then the expression 1
n n n
(c) log5e (d) loge5
Cr 1 Cr 1 2 Cr equals e

n 1 n 2 91. If f(x) = x log x and f(0) = 0, then the value of


(a) Cr 1 (b) Cr
for which Rolle’s theorem can be applied in
(c) n 2
Cr (d) n 1
Cr [0, 1] is
1
(a) –2 (b) –1
87. A printing company prints two types of
magazines A and B. The company earns `10 and (c) 0 (d) 1/2
www.jeeneetbooks.in

WWW.IIT-NEET.XYZ

Mock Test 2 MT-21

92. The spheres x 2 y2 z2 x y z 1 0 98. If sin 1 (1 x ) 2 sin 1


x , then :
2
2 2 2
and x y z x y z 5 0
1 1
(a) intersect in a plane (a) x 0, (b) x
2 2
(b) intersect in five points
(c) do not intersect (c) x = 0 only (d) none of these
(d) none of these. 99. If a is a non-zero real number, then the vectors
93. The function f : R R defined by aiˆ 2ajˆ 3ak,
ˆ
f (x) = (x – 1) (x – 2) (x – 3) is
(a) one-one but not onto (2a 1)iˆ (2a 3)ˆj (a 1)k,
ˆ
(b) onto but not one-one
(c) both one-one and onto (3a 5)iˆ (a 5)ˆj (a 2)kˆ are
(d) neither one-one nor onto (a) coplanar if a < 0 (b) coplanar if a > 0
94. If f :R S, defined by (c) always coplanar (d) never coplanar
f ( x) sin x 3 cos x 1, is onto, then the 100. If B is a non-singular matrix and A is a square
interval of S is matrix, then det (B–1 AB) is equal to
(a) [ –1, 3] (b) [–1, 1] (a) det (A–1) (b) det (B–1)
(c) [ 0, 1] (d) [0, 3] (c) det (A) (d) det (B)
95. The locus of the mid points of the focal chords of
the parabola y2 = 4ax is another parabola whose 3 7 3
101. x 1 x 4 dx is equal to
vertex is given by
(a) (a, 0) (b) (0, a) 21 3 4
8/7
32 3 4
8/7
(a) 1 x C (b) 1 x C
(c) (–a, 0) (d) None of these 32 21
96. The solution to the differential equation
8/7
7 3 4
dy (c) 1 x C (d) None of these.
2 2 32
(x y) a is
dx
102. For a binary operation * on the set {1, 2, 3, 4, 5},
consider the following multiplication table.
y c
(a) x y a tan
a * 1 2 3 4 5
1 1 1 1 1 1
(b) y a tan 1 ( x y) c
2 1 2 1 2 1
x y 3 1 1 3 1 1
(c) y 1 tan c
4 a
4 1 2 1 4 1
(d) none of these 5 1 1 1 1 5
m
Which of the following is correct?
97. If (k 2 1)k! 1999 2000! , then m is
k 1
(a) (2 * 3) * 4 = 1
(b) 2 * (3 * 4) = 2
(a) 1999 (b) 2000
(c) * is not commutative
(c) 2001 (d) None of these
(d) (2 * 3) * (4 * 5) = 2
EBD_7443
www.jeeneetbooks.in

WWW.IIT-NEET.XYZ

MT-22 Target VITEEE

103. If x cos y sin p , a variable chord of the (b) a tumbler is half empty if and only if it is
half full
x2 y2 (c) Both (a) and (b)
hyperbola 1 subtends a right angle (d) None of the above
2
a 2a 2
109. A car will hold 2 in the front seat and 1 in the
at the centre of the hyperbola, then the chords rear seat. If among 6 persons 2 can drive, then
touch a fixed circle whose radius is equal to the number of ways in which the car can be
(a) (b) filled is
2a 3a
(a) 10 (b) 20
(c) 2a (d) 5a (c) 30 (d) None of these
110. The value of
104. If x 2 cos x dx f (x ) sin x g(x ) cos x c .
2 2
Then log e sin x dx log e cos x dx is
(a) f(x) = x2, g(x) = x 0 0
(b) f(x) = x2 + 2, g(x) = x
(c) f(x) = x2, g(x) = 2x – 1 (a) log 2 (b) log 2
(d) f(x) = x2 – 2, g(x) = 2x 2 2
105. Let P be a variable point on the ellipse 1
(c) log (d) none of these.
2
x2 y2
1 with foci F and F . If A is the area 111. If A, B, C are the angles of a triangle then the
a 2 b2 1 2
sin 2A sin C sin B
of the triangle PF1F2, then the maximum value of A
is value of determinant sin C sin 2B sin A
(a) 2abe (b) abe sin B sin A sin 2C
1 is
(c) abe (d) None of these (a) (b) 0
2
106. In each of a set of games it is 2 to 1 in favour of (c) 2 (d) None of these
the winner of the previous game. The chance 112. The length of projection of the segment joining
P(–1, 2, 0) and Q (1, –1, 2) on the plane
that the player who wins the first game shall win
three at least of the next four is 2x y 2z 4 is
(a) 3 (b) 4
8 4
(a) (b) (c) 5 (d) 3 3
27 81
x sin1 / x , x 0
4 113. f(x) =
(c) (d) None 0 , x 0 at x = 0 is
9
(a) continuous as well as differentiable
107. The normal to the curve represented
(b) differentiable but not continuous
parametrically by x = a (cos + sin ) and y = (c) continuous but not differentiable
a (sin – cos ) at any point , is such that it (d) neither continuous nor differentiable
(a) makes a constant angle with x-axis 114. Area enclosed between the curves y = sin 2x,
(b) is at a contant distance from the origin y = cos2x and y = 0 in the interval [0, /2] is
(c) touches a fixed circle 1
(d) satisfies (b) and (c) (a) 2 1 sq. units
3
108. Consider the following statements
p : A tumbler is half empty. 1
(b) 3 sq. units
q : A tumbler is half full. 2
Then, the combination form of “p if and only if
1
q” is (c) 2 sq. units
(a) a tumbler is half empty and half full 4
(d) (2 + 3) sq. units
www.jeeneetbooks.in

WWW.IIT-NEET.XYZ

Mock Test 2 MT-23

115. If ABCDEF is a regular hexagon and PART - IV : ENGLISH

AB AC AD AE AF n AD . Then n is Direction (Qs. 121 - 125) : Read the passage carefully


and answer the questions given below.
(a) 1 (b) 2
I felt the wall of the tunnel shiver. The master alarm
5 squealed through my earphones. Almost
(c) 3 (d) simultaneously, Jack yelled down to me that there was
2
a warning light on. Fleeting but spectacular sights
116. The altitude through vertex C of a triangle ABC, snapped into ans out of view, the snow, the shower of
with position vectors of vertices a , b, c debris, the moon, looming close and big, the dazzling
respectively is : sunshine for once unfiltered by layers of air. The last
twelve hours before re-entry were particular bone-
b c c a a b a b c chilling. During this period, I had to go up in to
command module. Even after the fiery re-entry
(a) (b)
b a b a splashing down in 81o water in south pacific, we could
still see our frosty breath inside the command module.
b c c a a b 121. The word 'Command Module' used twice in the
(c) (d) None of these given passage indicates perhaps that it deals
b a with
117. If the solution of the differential (a) an alarming journey
(b) a commanding situation
dy ax 3
equation represents a circle, then (c) a journey into outer space
dx 2y f (d) a frightful battle.
the value of ‘a’ is 122. Which one of the following reasons would one
(a) 2 (b) – 2 consider as more as possible for the warning
(c) 3 (d) – 4 lights to be on?
118. Let A = {1,2,3,4,5,6,7,8} which of the following (a) There was a shower of debris.
relations from A to R is NOT a function? (b) Jack was yelling.
(c) A catastrophe was imminent.
(a) R1 {( x, f ( x ) ) : x A, f ( x ) 6 x 7}
(d) The moon was looming close and big.
(b) R2 {( x, f ( x)) : x A, f ( x ) | x | 9} 123. The statement that the dazzling sunshine was
"for once unfiltered by layers of air" means
1 (a) that the sun was very hot
(c) R3 (x , f ( x )) : x A, f ( x ) (b) that there was no strong wind
2
x 7
(c) that the air was unpolluted
(d) R4 {( x, f ( x )) : x A, f ( x ) 4x} (d) none of above
124. His musical tastes are certainly ____ ; he has
119. Let f : R R be such that f(1) = 3 and recordings ranging from classical piano
1/ x performances to rock concerts, jazz and even
f (1 x )
f ' (1) = 6. Then xlim0 equals Chinese opera.
f (1)
(a) antediluvian (b) eclectic
(a) 1 (b) e½ (c) harmonious (d) sonorous
(c) e 2 (d) e 3 125. Choose the best pronunciation of the word,
120. The value of (2 – ) (2 – 2) (2 – 10) (2 – 11) is Poignant, from the following options.
(a) poig-nant (b) pohing-nant
(a) 49 (b) 16
(c) poing-nat (d) poi-nyant
(c) 16 (d) 49 2
EBD_7443
www.jeeneetbooks.in

WWW.IIT-NEET.XYZ

MOCK
VITEEE Mock Test Paper
3
Max. Marks : 125 Time : 2½ hrs
PART - I : PHYSICS 5. In an explosion, a body breaks up into two pieces
1. At temperature T, the emissive power and of unequal masses. In this
absorption power of a body for certain (a) both parts will have numerically equal
wavelength are e and a respectively, then momentum
(a) e = a (b) lighter part will have more momentum
(b) e > a (c) heavier part will have more momentum
(c) e < a
(d) there will not be any difinite relation (d) both parts will have equal kinetic energy
between e and a 6. When a wire of uniform cross–section a,
2. The work done in placing a charge of 8 × 10–18 length l and resistance R is bent into a
coulomb on a condenser of capacity 100 micro- complete circle, resistance between any two
farad is of diametrically opposite points will be
(a) 3.1 × 10–26 joule
(b) 4 × 10–10 joule R R R
(c) 32 × 10–32 joule (a) (b) 4R (c) (d)
4 8 2
(d) 16 × 10–232 joule
3. ABC is an equilateral triangle. Charges +q are 7. Three resistances P, Q, R each of 2 and an
placed at each corner as shown as fig. The electric unknown resistance S form the four arms of a
intensity at centre O will be Wheatstone bridge circuit. When a resistance
+q A of 6 is connected in parallel to S the bridge
gets balanced. What is the value of S?
r
(a) 3 (b) 6
O
r r (c) 1 (d) 2
+q +q 8. A jar is filled with two non-mixing liquids 1 and 2
B C
having densities 1 and, 2 respectively. A solid
1 q 1 q ball, made of a material of density 3 , is dropped
(a) (b)
4 o r 4 o r2 in the jar. It comes to equilibrium in the position
shown in the figure.Which of the following is
1 3q
(c) (d) zero true for 1, 1and 3?
4 o r2
(a) 3 < 1
4. A metallic sphere is placed in a uniform electric 2

field. The line of force follow the path (s) shown (b) 1> 3 2
in the figure as (c) 1< 2 3
1 1
2 2 (d) 1 < 3 2
3 3 9. In a mass spectrometer used for measuring the
4 4
masses of ions, the ions are initially accelerated
(a) 1 (b) 2 by an electric potential V and then made to
(c) 3 (d) 4 describe semicircular path of radius R using a
www.jeeneetbooks.in

WWW.IIT-NEET.XYZ

Mock Test 3 MT-25

magnetic field B. If V and B are kept constant, 15. In a series resonant circuit, having L, C and R as
its elements, the resonant current is i. The power
charge on the ion
the ratio will be dissipated in the circuit at resonance is
mass of the ion
proportional to i 2R
(a) 1/R2 (b) R2 (a) 1 (b) zero
(c) R (d) 1/R L
C
10. A charged particle (charge q) is moving in a circle
of radius R with uniform speed v. The associated (c) i 2 L (d) i 2 R
magnetic moment µ is given by
(a) qvR2 (b) qvR2/2 where is the angular resonance frequency.
(c) qvR (d) qvR/2 16. Two coils have a mutual inductance 0.005 H.
11. In a certain region of space electric field E and The current changes in the first coil according
magnetic field B are perpendicular to each other to equation I=I0 sin t, where I0 = 10A and =
and an electron enters in region perpendicular 100 radian/sec. The maximum value of e.m.f. in
to the direction of B and E both and moves the second coil is
undeflected, then velocity of electron is (a) 2 (b) 5
(c) (d) 4
E
(a) (b) E B 17. The electric and magnetic field of an
B electromagnetic wave are
(a) in opposite phase and perpendicular to each
B
(c) (d) E B other
E (b) in opposite phase and parallel to each other
12. For a conservative force in one dimension, (c) in phase and perpendicular to each other
potential energy function V(x) is related to the (d) in phase and parallel to each other.
force F(x) as 18. In Young’s double slit experiment, the fringe
dV(x) width is found to be 0.4 mm. If the whole
(a) F(x)
dx
apparatus is immersed in water of refrative index
dV(x)
(b) F(x)
dx 4
, without disturbing the geometrical
(c) F(x) = V(x) dx 3
–dV(x) arrangement, the new fringe width will be
(d) F(x)
dx
(a) 0.30 mm (b) 0.40 mm
13. A transformer is used to light a 100 W and 110 V
lamp from a 220 V mains. If the main current is 0.5 (c) 0.53 mm (d) 450 microns
amp, the efficiency of the transformer is 19. A ball of mass m hits the floor making an angle
approximately as shown in the figure. If e is the coefficient of
(a) 50% (b) 90% restitution, then which relation is true, for the
(c) 10% (d) 30%. velocity component before and after collision?
14. What is the value of inductance L for which the
current is maximum in a series LCR circuit with
C = 10 µF and = 1000s– 1 1 1
(a) 1 mH v v
(b) cannot be calculated unless R is known
(c) 10 mH
(d) 100 mH
EBD_7443
www.jeeneetbooks.in

WWW.IIT-NEET.XYZ

MT-26 Target VITEEE


(a) V1 sin = V sin 24. The X-rays of wavelength 0.5 Å are scattered by
(b) V1 sin = – sin a target. What will be the energy of incident X-
(c) V1 cos = V cos rays, if these are scattered at an angle of 72º ?
(d) V1 cos = –V cos (a) 12.41 keV (b) 6.2 keV
20. Unpolarized light is incident on a glass plate (c) 18.6 keV (d) 24.82 keV
25. The momentum of photon whose frequency f is
having refractive index 1.5. The angle of
incidence at which the plane polarised light hf hc
(a) (b)
obtained, is c f
(a) 52° (b) 57° h c
(c ) (d)
(c) 60° (d) 66° f hf
21. If in the first Bohr orbit of a bydrogen atom, the 26. A proton and -particle are accelerated through
the same potential difference. The ratio of their
total energy of the electron is – 21.76 × 10–19 J,
de-Broglie wavelength will be
then its potential energy will be (a) 1 : 1 (b) 1 : 2
(a) – 43.52 × 10–19 J (b) – 21.76 × 10–19 J
(c) 2 : 1 (d) 2 2 : 1
(c) – 10.88 × 10–19 J (d) – 13.60 × 10–19 J
27. In a photoelectric effect measurement, the
22. Given that the formula for the various spectral
stopping potential for a given metal is found to
series in the hydrogen atoms is given by be V0 volt when radiation of wavelength 0 is
used. If radiation of wavelength 2 0 is used with
1 1 1 , where n and n are integers, the same metal then the stopping potential (in
R 1 2
n12 n 22 volt) will be

match List-I with List-II and select the correct V0


(a) (b) 2 V0
answer using the codes given below the Lists : 2
List-I List-II hc hc
A. Balmer series 1. n1 = 1 (c) V0 (d) V0
2e 0 2e 0
B. Brackett series 2. n1 = 2
28. The value of tan (90° – ) in the graph gives
C. Paschen series 3. n1 = 3
D. Lyman series 4. n1 = 4
Codes :
A B C D
Strain

(a) 2 1 3 4
(b) 2 4 3 1
(c) 2 4 1 2
Stress
(d) 4 3 2 1
(a) Young's modulus of elasticity
23. To obtain laser from a system, the stimulating
(b) compressibility
radiation must be
(c) shear strain
(a) an electromagnetic wave of any frequency (d) tensile strength
with suitable phase 29. Which one of the following groups of particles
(b) an electromagnetic wave of any frequency is a Boson group ?
with any phase (a) Photon, muon, pion, proton
(c) an electromagnetic wave of suitable (b) Photon, pion, graviton, Kaon
frequency (c) Pion, neutron, graviton, photon
(d) any wave with suitable frequency (d) Electron, muon, neutrino, proton
www.jeeneetbooks.in

WWW.IIT-NEET.XYZ

Mock Test 3 MT-27

30. A radioactive sample containing N0 nuclei emits


IC
N -particles per second on decaying. The half- 35. In the study of transistor as amplifier, if
life of the sample, in second is IE

N IC
N and where IC, IB and IE are the collector,,
(a) 0.693 (b) N0 IB
N0
base and emitter currents, then
N N0
(c) 0.693 (d) (1 ) (1 )
N0 N (a) (b)

31. Which one of the following nuclear reactions is


not possible ? (c) (d)
(1 ) (1 )
7 1 8 4 4
(a) 3 Li 1H 4 Be 2 He 2 He 36. A semi-conducting device is connected in a series
circuit with a battery and a resistance. A current
7 1 8 8 is found to pass through the circuit. If the polarity
(b) 3 Li 1H 4 Be 4 Be
of the battery is reversed, the current drops to
7 2 9 8 1 almost zero. The device may be
(c) 3 Li 1H 4 Be 4 Be 0n (a) a p-n junction
(b) an intrinsic semi-conductor
6 1 7 2 4
(d) 3 Li 0n 3 Li 1H 2 He (c) a p-type semi-conductor
(d) an n-type semi-conductor
32. If a nucleus X emits an alpha particle and a beta
particle, then the daughter nucleus will have 37. For sky wave propagation of a 10 MHz signal,
which of the following configurations ? what should be the minimum electron density in
1. A – 4 nucleons ionosphere
2. Z – 2 protons (a) – 1.2 × 1012 m–3 (b) – 106 m–3
(c) – 10 m14 –3 (d) – 1022 m–3
3. A – Z – 3 neutrons
38. A hockey player is moving northward and
Select the correct answer using the codes given
below : suddenly turns westward with the same speed
(a) 1, 2 and 3 (b) 1 and 2 to avoid an opponent. The force that acts on
(c) 1 and 3 (d) 2 and 3 the player is
33. The cause of the potential barrier in a p-n diode (a) frictional force along westward
is (b) muscles force along southward
(a) Depletion of positive charges near the (c) frictional force along south-west
junction (d) muscle force along south-west
(b) Concentration of positive charges near the 39. For an ideal gas, the molar specific heat
junction capacities at constant pressure and volume
(c) Depletion of negative charges near the satisfy the relation
junction (a) CP + CV = R (b) CP – CV = R
(d) Concentration of positive and negative
charges near the junction CP CV
(c) =R (d) =R
34. The following circuit represents CV CP
A 40. The position of a particle of mass 4 g, acted upon
by a constant force is given by x = 4t2 + t, where
Y
x is in metre and t in second. The work done
B during the first 2 seconds is
(a) 128 mJ (b) 512 mJ
(a) OR gate (b) XOR gate
(c) 576 mJ (d) 144 mJ
(c) AND gate (d) NAND gate
EBD_7443
www.jeeneetbooks.in

WWW.IIT-NEET.XYZ

MT-28 Target VITEEE


PART - II : CHEMISTRY (a) 10–18 (b) 10–37
41. 3A B C , it would be a zero order reaction (c) 1018 (d) 1037
when 48. 4.5 g of aluminium (at. mass 27 amu) is
(a) the rate of reaction is proportional to square deposited at cathode from Al 3+ solution by a
of concentration of A certain quantity of electric charge. The volume
(b) the rate of reaction remains same at any of hydrogen produced at STP from H+ ions in
concentration of A solution by the same quantity of electric
(c) the rate remains unchanged at any charge will be
concentration of B and C (a) 44.8 L (b) 22.4 L
(c) 11.2 L (d) 5.6 L
(d) the rate of reaction doubles if concentration
49. Which one of the following is correct relation of
of B is increased to double
the First Law of Thermodynamics
42. The rate of a first order reaction is 1.5 × 10–2 mol
(a) E = q – W (b) E = q + W
L–1 min–1 at 0.5 M concentration of the reactant.
(c) E = q + W (d) E = q + W
The half life of the reaction is
50. Which of the following is most basic in nature?
(a) 0.383 min (b) 23.1 min
(a) NH3 (b) CH3NH2
(c) 8.73 min (d) 7.53 min
(c) (CH3)2NH (d) C6H5NHCH3
43. Activation energy of a chemical reaction can be 51. Intermediates formed during reaction of
determined by
(a) evaluating rate constant at standard R C NH 2 with Br2 and KOH are
||
temperature O
(b) evaluating velocities of reaction at two
different temperatures (a) RNHBr and RCONHBr
(c) evaluating rate constants at two different (b) RNHCOBr and RNCO
temperatures (c) RCONHBr and RNCO
(d) RCONBr2
(d) changing concentration of reactants
52. The final product C, obtained in this reaction
44. If the rate of the reaction is equal to the rate
constant, the order of the reaction is NH2
(a) 3 (b) 0
(c) 1 (d) 2 Ac2O Br2 H2O
45. In a reversible reaction the energy of activation A B + C
CH3COOH H
of the forward reaction is 50 kcal. The energy of
activation for the reverse reaction will be CH3
(a) < 50 kcal
(b) either greater than or less than 50 kcal would be
(c) 50 kcal
(d) > 50 kcal NHCOCH3 NH2
46. If 0.01 M solution of an electrolyte has a Br COCH3
resistance of 40 ohms in a cell having a cell (a) (b)
constant of 0.4 cm–1, then its molar conductance
in ohm–1 cm2 mol–1 is CH3
CH3
(a) 102 (b) 104
(c) 10 (d) 103
COCH3 NH2
47. Eº for the cell, Zn Zn 2 (aq) Cu 2 (aq) Cu is
Br Br
1.10 V at 25ºC. The equilibrium constant for the (c) (d)
cell reaction:
2+ 2+ CH3 CH3
Zn + Cu (aq) Cu + Zn (aq) ,
is of the order of
www.jeeneetbooks.in

WWW.IIT-NEET.XYZ

Mock Test 3 MT-29

53. Which of the following is the strongest acid? 59. The pyknometric density of sodium chloride
crystal is 2.165 × 103 kg m–3 while its X-ray density
OH
OH
is 2.178 × 103 kg m–3. The fraction of unoccupied
(a) (b) sites in sodium chloride crystal is
NO2 (a) 5.96 × 10–3 (b) 5.96
Cl (c) 5.96 × 10–2 (d) 5.96 × 10–1
60. A reaction occurs spontaneously if
OH OH (a) T S < H and both H and S are + ve
(b) T S > H and H is + ve and S is ve
(c) (d) (c) T S > H and both H and S are + ve
NO2 (d) T S = H and both H and S are + ve
NO2 61. Which one of the following is an inner orbital
complex as well as diamagnetic in behaviour?
54. A colourless liquid, at room temperature, reacts
(Atomic number: Zn = 30, Cr = 24, Co = 27,
with soda-lime to form sodium salt of a carboxylic
Ni = 28)
acid and ammonia gas. The liquid is
(a) [Zn(NH3)6]2+ (b) [Cr(NH3)6]3+
(a) propanoic acid
(c) [Co(NH3)6]3+ (d) [Ni(NH3)6]2+
(b) formamide 62. [Co(NH3)4 (NO2)2] Cl exhibits
(c) propanamide (a) linkage isomerism, ionization isomerism and
(d) methyl enthanoate geometrical isomerism
55. 59 g of an amide obtained from a carboxylic acid, (b) ionization isomerism, geometrical isomerism
RCOOH, upon heating with alkali liberated 17 g and optical isomerism
NH3. The acid is (c) linkage isomerism, geometrical isomerism and
(a) Formic acid (b) Acetic acid optical isomerism
(c) Propionic acid (d) Benzoic acid (d) linkage isomerism, ionization isomerism and
56. The correct relationship between free energy and optical isomerism
equilibrium constant K of a reaction is 63. The d electron configurations of Cr 2+, Mn2+,
(a) G RT ln K (b) G RT lnK Fe 2+ and Ni 2+ are 3d 4 , 3d 5 , 3d 6 and 3d 8
respectively. Which one of the following aqua
(c) G RT lnK (d) G RT lnK complexes will exhibit the minimum paramagnetic
57. The intermetallic compound LiAg crystallizes in behaviour?
a cubic lattice in which both lithium and silver (a) [Fe(H2O)6]2+ (b) [Ni(H2O)6]2+
(c) [Cr(H2O)6] 2+ (d) [Mn(H2O)6]2+
atoms have coordination number of eight. To
what crystal class does the unit cell belong ? (At. No. Cr = 24, Mn = 25, Fe = 26, Ni = 28)
(a) Simple cubic 64. The radioactive isotope, tritium, (13 H ) has a half-
(b) Face-centred cubic
life of 12.3 years. If the initial amount of tritium is
(c) Body-centred cubic 32 mg, how many milligrams of it would remain
(d) None after 49.2 years?
58. Schottky defect in crystals is observed when (a) 8 mg (b) 1 mg
(a) an ion leaves its normal site and occupies an (c) 2 mg (d) 4 mg
interstitial site 65. A human body required 0.01M activity of
(b) unequal number of cations and anions are radioactive substance after 24 hours. Half life of
missing from the lattice radioactive substance is 6 hours. Then injection
(c) density of the crystal is increased of maximum activity of radioactive substance that
can be injected will be
(d) equal number of cations and anions are
(a) 0.08 M (b) 0.04 M
missing from the lattice
(c) 0.32 M (d) 0.16 M
EBD_7443
www.jeeneetbooks.in

WWW.IIT-NEET.XYZ

MT-30 Target VITEEE


66. The hypothetical complex chlorodiaquotriammine 73. Iodoform test is not given by
cobalt (III) chloride can be represented as (a) 2-Pentanone (b) Ethanol
(a) [CoCl(NH3)3 (H2O)2]Cl2 (c) Ethanal (d) 3-Pentanone
(b) [Co(NH3)3 (H2O)Cl3] 74. Acetaldehyde reacts with
(c) [Co(NH2)3 (H2O)2 Cl] (a) Electrophiles only
(d) [Co(NH3)3 (H2O)3]Cl3 (b) Nucleophiles only
67. The basic character of the transition metal (c) Free radicals only
monoxides follows the order (d) Both electrophiles and nucleophiles.
(Atomic Nos.,Ti = 22, V = 23, Cr = 24, Fe = 26) 75. Aldehydes and ketones will not form crystalline
(a) TiO > VO > CrO > FeO derivatives with
(b) VO > CrO > TiO > FeO (a) Sodium bisulphite
(c) CrO > VO > FeO > TiO (b) Phenylhydrazine
(d) TiO > FeO > VO > CrO (c) Semicarbazide hydrochloride
68. The complex ion [Co(NH3)6]3+ is formed by sp3d2 (d) Dihydrogen sodium phosphate.
hybridisation. Hence the ion should possess 76. Reaction of phenylacetylene with dil. H2SO4 and
(a) Octahedral geometry HgSO4 gives
(b) Tetrahedral geometry (a) acetophenone
(c) Square planar geometry (b) 2-phenylethanol
(d) Tetragonal geometry. (c) phenylacetaldehyde
69. Identify the correct order of boiling points of the (d) phenylacetic acid
following compounds : 77. A carbonyl compound reacts with hydrogen
CH3CH2CH2CH2OH CH3CH2CH2CHO cyanide to form cyanohydrin which on hydrolysis
1 2 forms a racemic mixture of -hydroxy acid. The
CH3CH2CH2COOH carbonyl compound is
3 (a) acetone (b) diethyl ketone
(a) 1 > 2 > 3 (b) 3 > 1 > 2 (c) formaldehyde (d) acetaldehyde
(c) 1 > 3 > 2 (d) 3 > 2 > 1 78. Among the following the strongest acid is
70. Glacial acetic acid is (a) CH 3 COOH
(a) pure acetic acid at 100º C (b) CH 2 ClCH 2COOH
(b) pure acetic acid at 0º C
(c) CH 2 ClCOOH
(c) acetic acid mixed with methanol
(d) pure acetic acid at 16.6º C (d) CH 3CH 2 COOH.
71. 2-Phenylethanol may be prepared by the reaction 79. [Ti(H2O)6]3+ shows purple colour due to
of phenylmagnesium bromide with (a) d yz electronic transition
d
(a) HCHO (b) CH3CHO x2 y2

O (b) d xz d 2 2 electronic transition


x y
(c) CH3COCH3 (d)
(c) d xy eg [d ,d ] electronic transition
x 2 y2 z2
72. Among acetic acid, phenol and n-hexanol, which
of the following compounds well react with (d) d xz eg [d ,d ] electronic transition
x 2 y2 z2
NaHCO3 solution to give sodium salt and carbon
80. C – H rocking frequency is found to observed in
dioxide ?
__________. In case of only long chain alkanes.
(a) Acetic acid (a) 725 - 720 cm–1
(b) n-Hexanol (b) 745 - 795 cm–1
(c) acetic acid and phenol (c) 790 - 760 cm–1
(d) Phenol. (d) 720 - 700 cm–1
www.jeeneetbooks.in

WWW.IIT-NEET.XYZ

Mock Test 3 MT-31

PART - III : MATHEMATICS (a) 1 (b) –2

81. If 2y cos = x sin and 2x sec – y cosec = 3, 1


(c) 2 (d)
then x2 + 4y2 = 2
(a) 2 (b) 4 89. P is a variable point on the hyperbola
(c) 1 (d) none
82. Range of the function x2 y2
2
1 whose vertex is A(a, 0) The locus
x x 2 a 2 b2
f ( x) ; x R is
x2 x 1 of the middle point of AP is
(a) (1, ) (b) (1,11/7] (2x a) 2 2y 2
(c) (1, 7/3] (d) (1, 7/5] (a) 1
2
a b2
83. If sin (cot–1(1 +x)) = cos (tan–1x), then x =
2
4y 2
1 1 (b) ( 2 x a ) 1
(a) (b) 1 (c) 0 (d) a 2
b2
2 2
84. If (2x a ) 2 8y 2
(c) 1
2
2 2 a b2
sin 1 sin 1 2 tan 1 x, (d) None of these
2 2
1 1 90. Two persons throw a die alternately till one of
then x = them gets a ‘three’ and wins the game. The ratio
(a) / (b) / of their respective probabilities of winning is
(a) 6 : 5 (b) 4 : 5
(c) (d) (c) 3 : 2 (d) 3 : 5
1 1 91. If m men and n women are to be seated in a row
85. Let A = {1, 2, 3, 4, 5} and the functions so that no two women sit together. If m > n, then
f : A A and g : A A be defined by the number of ways in which they can be seated
f(1) = 3, f(2) = 5, f(3) = 3, f(4) = 1, f(5) = 2; g(1) = 4, is
g (2) = 1, g (3) = 1, g (4) = 2, g (5) = 3. Then
(a) fog = {(1,1), (2, 3), (3, 2), (4, 5),} m! n! (m 1)! ( n 1)!
(a) (b)
(b) fog = {(1,1), (2,3), (3,3), (4,5), (5,3)} (m n 1)! (m n 1)!
(c) gof = {(1, 1), (2, 3), (3, 3), (4, 4), 5, 5)
(d) gof = {(2, 2), (2, 3), (3, 1), (4, 1), (5, 1)} m!(m 1)!
86. ABC is a triangle and P is any point on BC such (c) (d) None of these
(m n 1)!
that PQ is the resultant of the vectors dy
1
92. If y sin x ax a ax , then
AP, PB and PC , then dx
(a) the position of Q depends on position of P 1
(b) Q is a fixed point (a) (b) sin x . sin a
sin a ax
(c) Q lies on AB or AC
(d) None of these 1
87. If A, B, C are the angles of a triangle such that (c) (d) zero
angle A is obtuse, then tan B tan C will be less 2 x 1 x
than 3 2
93. If the roots of z iz 2 i 0 represent the
1 3 vertices of a ABC in the Argand plane, then
(a) (b)
3 2 the area of the triangle is
(c) 1 (d) none of these.
3 7 3 7
88. If a chord which is normal to the parabola at one (a) (b)
end and subtends a right angle at the vertex, then 2 4
slope of the chord is (c) 2 (d) None
EBD_7443
www.jeeneetbooks.in

WWW.IIT-NEET.XYZ

MT-32 Target VITEEE


94. If the mean and variance of a binomial variate x
1 1 26 1 1 26
35 35 (c) (d)
27 0 27 27 0 27
are respectively and , then the
6 36 100. The area (in sq. units) bounded by the curves
probability of x > 6 is :
y x, 2y – x + 3 = 0 and x-axis lying in the first
1 57
(a) (b) quadrant is
62 67 (a) 9 (b) 36
1 1 1
(c) (d) 27
7 6
6 7
67 (c) 18 (d)
4
95. Let f(x) = |sin x|. Then 101. The number of values of x in [0, 2 ] satisfying the
(a) f is everywhere differentiable
(b) f is everywhere continuous but not equation | cosx – sin x | 2 , is
differentiable at
(a) 0 (b) 1
x=n ,n Z
(c) 2 (d) 3
(c) f is everywhere continuous but not
differentiable at 102. If one root of the equation (l m) x 2 lx 1 0
is double of the other and if l is real then the
x 2n 1
, n Z.
2 greatest value of m is (l m) :
(d) None of these 1 8
96. The locus of all the points on the curve (a) (b)
3 9
x
y2 4a x a sin at which the tangent is
a 9
(c) (d) 3
parallel to x-axis is : 8
(a) y = 4a (b) y = 4ax 103. A sphere of constant radius 2k passes through
x the origin and meets the axes in A, B, C. The
(c) y2 = 4ax (d) y2 = 4a2 sin locus of the centroid of the tetrahedron OABC
a
is
97 In the set N N , the relation R is defined by
2
(a, b) R (c,d) ad = bc. Then R is (a) x y2 z2 4k 2
(a) partial order relation
(b) equivalence relation (b) 9( x 2 y2 z2 ) 4k 2
(c) reflexive and transitive but not symmetric
(d) symmetric and transitive but not reflexive (c) x 2 y 2 z 2 k 2
(d) None of these
98. If a , b , c are three unit vectors such that b
1 2
is not parallel to c and a (b c) b,
2 104. The value of the integral cos sin 3 d is
then the angle between a and c is 0

8 21
(a) (b) (a) (b)
3 2 21 8
2
(c) (d) (c) (d) none of these.
6 4 7
3 2 105. Number of 6-digit telephone numbers, which can
99. If A = , then (A–1)3 is equal to be constructed with digits 0, 1, 2, 3, 4, 5, 6, 7, 8, 9,
0 1
if each number starts with 35 and no digit appears
1 1 26 1 1 26 more than once is
(a) (b)
27 0 27 27 0 27 (a) 1680 (b) 8!
(c) 6! (d) 6.6 !
www.jeeneetbooks.in

WWW.IIT-NEET.XYZ

Mock Test 3 MT-33

106. The diffrerential equation 113. A rod AB 13 ft long moves with its ends A, B on
dy 1 two perpendicular lines OX and OY respectively.
sin 2 y x 3 cos 2 y represents a family If the end A is 12 ft from O and is slipping away at
dx x
of curves given by the equation 1
2 ft/sec. then the end B is moving at
2
(a) x6 6x 2 C tan y
(a) 5ft/sec. (b) 6ft/sec.
(b) 6x 2 tan y x6 C (c) 2.5ft/sec. (d) 3ft/sec.
114. Two common tangents to the circle x2 + y2 = 2a2
(c) sin 2 y x 3 cos 2 y C
and parabola y2 = 8ax are
(d) none of these
107. If the system of equations x + y + 2 = 0, (a) x = ±(y + 2a) (b) y = ±(x + 2a)
x + y – 2 = 0, x + y + 3 = 0 is consistent, then (c) x = ±(y + a) (d) y = ±(x + a)
(a) 1 (b) 2 115. Children have been invited to a birthday party. It
(c) 1, 2 (d) 1, 2 is necessary to give them return gifts. For the
108. A steam boat is moving at velocity V when steam purpose, it was decided that they would be given
is shut off. Given that the retardation at any pens and pencils in a bag. It was also decided
subsequent time is equal to the magnitude of the that the number of items in a bag would be atleast
velocity at that time. The velocity v in time t after 5. If the cost of a pen is `10 and cost of a pencil
steam is shut off is is `5, minimize the cost of a bag containing pens
(a) v = Vt (b) v = Vt – V and pencils. Formulation of LPP for this problem
(c) v = Vet (d) v = Ve–t is
109. If the binary operation * is defined on the set Q +
(a) Minimize C = 5x + 10y subject to x + y 10,
ab x 0, y 0
of all positive rational numbers by a * b = .
4 (b) Minimize C = 5x + 10y subject to x + y 10,
1 1 x 0, y 0
Then 3 * * is equal to (c) Minimize C = 5x + 10y subject to x + y 5,
5 2
x 0, y 0
3 5 3 3
(a) (b) (c) (d) (d) Minimize C = 5x + 10y subject to x + y 5,
160 160 10 40 x 0, y 0
110. P, Q, R, S are the points (–2, 3, 4), (–4, 4, 6), (4, 3, 5)
and (0, 1, 2). Then projection of PQ on RS is sin 2 x
116. The integral dx is
(a) 0 (b) 29 cos 4 x
16 (a) a polynomial of degree 5 in sin x
(c) (d) none (b) a polynomial of degree 4 in tan x
29
111. If X follows Binomial distribution with mean 3 (c) a polynomial of degree 3 in tan x
and variance 2, then P(X 8) is equal to : (d) a polynomial of degree 5 in cos x
17 18 19 20 117. The equation of the ellipse with focus at (±5, 0)
(a) 9 (b) 9 (c) 9 (d)
3 3 3 39 36
112. Sum of the series and x = as one directrix is
5
1 4 9 16
......
1. 2 . 3. 4 3. 4 .5 . 6 5. 6 . 7 .8 7 .8. 9 .10 x2 y2 x2 y2
(a) 1 (b) 1
36 25 36 11
5 3
(a) log 2 (b) log 2
2 2
x2 y2
1 1 (c) 1 (d) None of these
(c) log 2 (d) None of these 25 11
6 24
EBD_7443
www.jeeneetbooks.in

WWW.IIT-NEET.XYZ

MT-34 Target VITEEE


him. I had never shot an elephant and never wanted
((a n )nx tan x ) sin nx
118. If xlim0 0, where n is to. (Somehow it always seems worse to kill large
x2 animal.) Besides, there was the beast's owner to be
non-zero real number, then a is equal to considered. But I had got to act quickly. I turned to
some experienced-looking Burmans who had been
n 1 there when we arrived, and asked them how the
(a) 0 (b)
n elephants had been behaving. They all said the same
thing; he took no notice of you if you left him alone,
1
(c) n (d) n but he might charge if you went too close to him.
n 121. The phrase 'Preoccupied grandmotherly air'
119. The equation of the line passing through (1, 2, 3) signifies
(a) being totally unconcerned
and parallel to the planes r. ˆi ˆj 2kˆ 5 and
(b) pretending to be very busy
(c) a very superior attitude
r. 3iˆ ˆj kˆ 6 is
(d) calm, dignified and affectionate disposition
ˆi 2ˆj 3kˆ 2iˆ 3jˆ 4kˆ 122. From the passage it appears that the author was
(a) r
(a) an inexperienced hunter
(b) r 3iˆ 5jˆ 4kˆ iˆ 2ˆj 3kˆ (b) kind and considerate
(c) possessed with fear
(c) r ˆi 2jˆ 3kˆ 3iˆ 5jˆ 4kˆ (d) a worried man
123. The author did not want to shoot the elephant
(d) r 3iˆ 5jˆ 4kˆ because he
120 Let p: Kiran passed the examination, (a) was afraid of it
q: Kiran is sad (b) did not have the experience of shooting
The symbolic form of a statement “It is not true big animals
that Kiran passed therefore he is said” is (c) did not wish to kill animal which was not
(a) (~ p q) (b) (p q) doing anybody any harm
(c) ~ (p ~ q) (d) ~ ( p q) (d) did not find the elephant to be ferocious
124. Direction: Choose the word which is most similar
PART - III : ENGLISH to the word given in the bold. Foment
Direction (Qs. 121 - 123) Read the passage carefully (a) interrogate (b) spoil
and answer the question given below. (c) spray (d) incite
125. Choose the best pronunciation of the word,
But I did not want to shoot the elephant. I watched Wednesday, from the following options.
him beating his bunch of grass against his knees, with (a) wed-ness-day (b) wed-nesh-day
the preoccupied grandmotherly air that elephants (c) wenz-day (d) wens-day
have. It seemed to me that it would be murder to shoot
www.jeeneetbooks.in

WWW.IIT-NEET.XYZ

MOCK
VITEEE Mock Test Paper
4
Max. Marks : 125 Time : 2½ hrs
PART - I : PHYSICS 5. A steady current of 1.5 amp flows through a
copper voltameter for 10 minutes. If the
1. In a hollow spherical shell, potential (V) changes
electrochemical equivalent of copper is
with respect to distance (s) from centre as
30 × 10–5 g coulomb–1, the mass of copper
deposited on the electrode will be
(a) 0.50 g (b) 0.67 g
V (c) 0.27 g (d) 0.40 g.
(a) (b) V 6. Kirchhoff’s first and second laws for electrical
circuits are consequences of
S
(a) conservation of electric charge and energy
S
respectively
(b) conservation of electric charge
V V (c) conservation of energy and electric charge
(c) (d) respectively
(d) conservation of energy
S S
7. For the network shown in the Fig. the value
2. Four charges equal to -Q are placed at the four of the current i is
corners of a square and a charge q is at its centre.
If the system is in equilibrium the value of q is
Q Q
(a) (1 2 2 ) (b) (1 2 2 )
2 4
Q Q
(c) (1 2 2 ) (d) (1 2 2 )
4 2
3. The stored energy of Capacitor is W after giving
a charge q. If we increase charge form q to 2q
then stored energy is (The capacitance of
capacitor is C)
(a) 2 W (b) W/2 9V 18V
(c) 4W (d) W/4 (a) (b)
4. When air is replaced by a dielectric medium of 35 5
force constant K, the maximum force of 5V 5V
attraction between two charges, separated by a (c) (d)
distance 9 18
(a) decreases K-times 8. Two cells, having the same e.m.f., are connected
(b) increases K-times in series through an external resistance R. Cells
(c) remains unchanged have internal resistances r 1 and r2 (r1 > r2 )
respectively. When the circuit is closed, the
1
(d) becomes times potential difference across the first cell is zero.
K2 The value of R is
EBD_7443
www.jeeneetbooks.in

WWW.IIT-NEET.XYZ

MT-36 Target VITEEE


14. The core of a transformer is laminated because
r1 r2 r1 r2
(a) (b) (a) the weight of the transformer may be
2 2 reduced
(c) r1 r2 (d) r1 r2 (b) rusting of the core may be prevented
(c) ratio of voltage in primary and secondary
9. If 25W, 220 V and 100 W, 220 V bulbs are may be increased
connected in series across a 440 V line, then (d) energy losses due to eddy currents may be
(a) only 25W bulb will fuse minimised
(b) only 100W bulb will fuse 15. A step-up transformer operates on a 230 V line
(c) both bulbs will fuse and supplies a load of 2 ampere. The ratio of the
primary and secondary windings is 1 : 25. The
(d) none of these
current in the primary is
10. A mild steel wire of length 2L and cross-sectional (a) 25 A (b) 50 A
area A is stretched, well within elastic limit,
(c) 15 A (d) 12.5 A
horizontally between two pillars. A mass m is
suspended from the mid point of the wire. Strain 16. An inductance L having a resistance R is
in the wire is connected to an alternating source of angular
frequency . The quality factor Q of the
2L
x inductance is

m R L
2
(a) (b)
L R
x x
(a) (b)
2L L
1/ 2
2 2
R L
x x (c) (d)
(c) (d) L R
L 2L
11. An electron enters a region where magnetic field 17. Assuming no heat losses, the heat released by
(B) and electric field (E) are mutually the condensation of x g of steam at 100°C can
perpendicular, then be used to convert y g of ice at 0°C into water at
(a) it will always move in the direction of B 100°C, the ratio x : y is :
(b) it will always move in the direction of E (a) 1 : 1 (b) 1 : 2
(c) it always possesses circular motion (c) 1 : 3 (d) 3 : 1
(d) it can go undeflected also. 18. Interference is possible in
12. A wire carries a current. Maintaining the same (a) light waves only
current it is bent first to form a circular plane coil of (b) sound waves only
one turn which produces a magnetic field B at the
(c) both light and sound waves
centre of the coil. The same length is now bent
more sharply to give a double loop of smaller radius. (d) neither light nor sound waves
The magnetic field at the centre of the double loop, 19. Two sources of light are said to be coherent if
caused by the same current is they emit light
(a) 4B (b) B/4 (a) of equal amplitudes
(c) B/2 (d) 2B (b) having the same wavelength
13. A coil of inductive reactance 31 has a (c) having a constant phase relationship
resistance of 8 . It is placed in series with a (d) having the same intensity
condenser of capacitative reactance 25 . The
20. The figure gives the potential energy function
combination is connected to an a.c. source of
110 volt. The power factor of the circuit is U(x) for a system in which a particle is in one-
dimensional motion. In which region the
(a) 0.64 (b) 0.80
magnitude of the force on the particle is greatest
(c) 0.33 (d) 0.56
www.jeeneetbooks.in

WWW.IIT-NEET.XYZ

Mock Test 4 MT-37

25. The photo electric work function for a metal


surface is 4.125 eV. The cut-off wavelength for
this surface is
(a) 4125 Å (b) 3000 Å
(c) 6000 Å (d) 2062 Å
26. The variation of maximum kinetic energy
photoelectrons with applied frequency ( ) is
(a) OA (b) AB
(c) BC (d) CD (a) (b)
21. The frequency of the radiation emitted by a

K.E.max.
K.E.max.
hydrogen atom for the transition between n = 2
and n = 1 states is v0. What is the frequency of
the radiation emitted by the hydrogen atom for
transition between n = 4 and n = 1 states ?
(a) 3v0/2 (b) 2v0
(c) (d)
(c) 4v0 (d) 5v0/4
K.E.max.

K.E.max.
22. Match List-I (Series Spectra of Hydrogen) with
List-II (Region in which the series lies) and select
the correct answer using the codes given below
the Lists :
List-I List-II 27. If n bullets each of mass m are fired with a velocity
(Series spectra (Region in which the v per second from a machine gun, the force
of Hydrogen) series lies) required to hold the gun in position is
A. Lyman 1. Visible
B. Balmer 2. Infra red mv
C. Paschen 3. Ultra violet (a) (n + 1) mv (b)
n2
D. Brackett 4. X-ray
5. -ray mv
Codes : (c) (d) mnv
n
A B C D
(a) 3 1 2 2 28. The ratio of de-Broglie wavelengths of proton
(b) 1 3 2 4 and -particle having same kinetic energy is
(c) 3 1 4 5
(d) 1 2 3 5 (a) 2 :1 (b) 2 2 :1
23. In Millikan oil drop experiment a drop of charge (c) 2 : 1 (d) 4 : 1
Q and radius r is kept constant between two 29. Match List-I (Classification) with List-II
plates of potential difference of 800 volt. Then
(Elementary Particles) and select the correct
charge on other drop of radius 2 r which is kept
constant with a potential difference of 3200 V is answer using the codes given below the Lists :
(a) Q/2 (b) 2 Q List-I List-II
(c) 4 Q (d) Q/4 (Classification) (Elementary particles)
24. White X-rays are called white due to the fact A. Baryons 1. Nucleon
that B. Mesons 2. Neutrino
(a) they are electromagnetic radiations hav- C. Leptons 3. Pion
ing nature same as that of white light.
(b) they are produced most abundantly in X D. Bosons 4. Photon
ray tubes. Codes :
(c) they have a continuous wavelength A B C D
range. (a) 1 3 2 4
(d) they can be converted to visible light us- (b) 2 3 1 4
ing coated screens and photographic (c) 1 4 2 3
plates are affected by them just like light.
(d) 2 4 1 3
EBD_7443
www.jeeneetbooks.in

WWW.IIT-NEET.XYZ

MT-38 Target VITEEE


30. Mp is the mass of proton and mn is the mass of (a) NAND (b) XOR
neutron. If the mass of nucleus of an atom ZXA (c) OR (d) NOR
is measured and found to be M, then the nuclear 37. A ball is thrown up at an angle with the
binding energy would be (c is the velocity of horizontal. Then the total change of momentum
light) : by the instant it returns to ground is
(a) {Zmp + (A – Z)mn – M}c2
(a) acceleration due to gravity × total time of
(b) {Amn + Zmp + M}c2
(c) {(Z – A)mp + Amn – M}c2 flight
(d) {M – Zmp– (A – Z)mn}c2 (b) weight of the ball × half the time of flight
31. When a radioactive element decays by gamma (c) weight of the ball × total time of flight
radiation (d) weight of the ball × horizontal range
(a) its mass number will decrease by one unit 38. A particle of mass m has momentum p. Its
with no change in atomic number kinetic energy will be
(b) its mass number will not change but the (a) mp (b) p2m
atomic number will increase by one unit
(c) both mass number and atomic number of p2 p2
(c) (d)
the element change m 2m
(d) there will be no change in either mass 39. The reading of Centigrade thermometer
number or atomic number of the element coincides with that of Fahrenheit thermometer
32. In the uranium radioactive series, the initial in a liquid. The temperature of the liquid is
nucleus is 92U238 and that the final nucleus is (a) – 40ºC (b) 313°C
206
82Pb . When uranium nucleus decays to lead, (c) 0°C (d) 100°C
the number of particles and particles emitted
40. If a rubber ball is taken at the depth of 200 m in a
are
pool, its volume decreases by 0.1%. If the
(a) 8 , 6 (b) 6 , 7
density of the water is 1 × 103 kg/m3 and g =
(c) 6 , 8 (d) 4 , 3
10m/s2, then the volume elasticity in N/m2 will
33. In semiconductors at a room temperature
be
(a) the conduction band is completely empty
(a) 108 (b) 2 × 108
(b) the valence band is partially empty and the 9
(c) 10 (d) 2 × 109
conduction band is partially filled
(c) the valence band is completely filled and
the conduction band is partially filled PART - II : CHEMISTRY
(d) the valence band is completely filled 41. The alcohol which does not give a stable
34. The peak voltage in the output of a half-wave compound on dehydration is
diode rectifier fed with a sinusoidal signal (a) Ethyl alcohol (b) Methyl alcohol
without filter is 10V. The d.c. component of the (c) n-Propyl alcohol (d) n-Butyl alcohol
output voltage is
42. IR spectrum of an organic compound is found
(a) 20/ V (b) 10/ 2 V
(c) 10/ V (d) 10V about 1715 cm–1. The organic compound is
35. An oscillator is nothing but an amplifier with (a) An aliphatic ketone
(a) positive feedback (b) An aliphatic aldehyde
(b) large gain (c) , - unsaturated ketones
(c) no feedback (d) Phenolic ketone
(d) negative feedback 43. The most suitable method of separation of a
36. The following configuration of gate is equivalent mixture ortho and para-nitrophenols mixed in
to the ratio of 1 : 1 is
A OR (a) Steam distillation
B
Y (b) Crystallisation
AND (c) Vapourization
NAND (d) Colour spectrum
www.jeeneetbooks.in

WWW.IIT-NEET.XYZ

Mock Test 4 MT-39

44. When diethyl ether is treated with excess of Cl 2 51. Indicate which of the nitrogen compounds
in the presence of sunlight, then the product amongst th e following would undergo
formed is Hoffmann’s reaction, i,e., reaction with Br 2 and
(a) CH 3CHCl O CH 2 CH 3 strong KOH to furnish the primary amine (R –
NH2) ?
(b) CH 3CHCl O CHClCH 3
(a) R CO NHCH 3
(c) CCl 3CCl 2OCCl 2 CCl3 (b) R – CO – ONH4
(d) CH 3CCl 2 O CHClCH 3 (c) R – CO – NH2
45. The ether that undergoes electrophilic (d) R – CO – NHOH.
substitution reactions is 52. Benzaldehyde reacts with ethanoic KCN to give
(a) CH3OC2H5 (b) C6H5OCH3 (a) C6H5CHOHCN
(c) CH3OCH3 (d) C2H5OC2H5 (b) C6H5CHOHCOC6H5
46. What is the Eºcell for the reaction (c) C6H5CHOHCOOH
2+ 2+ 4+
(d) C6H5CHOHCHOHC6H5
Cu (aq) + Sn (aq) + Cu (S) + Sn (aq) 53. Which of the following compound will undergo
at 25ºC if the equilibrium constant for the reaction self aldol condensation in the presence of cold
is 1 × 106? dilute alkali ?
(a) 0.5328 V (b) 0.3552 V (a) CH 2 CH CHO
(c) 0.1773 V (d) 0.7104 V
(b) CH C CHO
47. The ionic conductance of Ba 2+ and Cl – are
respectively 127 and 76 ohm–1 cm2 at infinite (c) C 6 H 5CHO
dilution. The equivalent conductance (in ohm–1
(d) CH 3CH 2CHO.
cm2) of BaCl2 at infinite dilution will be :
(a) 139.5 (b) 203 HCl
(c) 279 (d) 101.5 54. 3CH 3COCH 3
3H 2O
48. A solution of [Ni(H2O)2+ 6 ] is green due to (A)
(a) d-d transition from t2g eg
(b) d-d transition from eg t2g (CH 3 )2 C CH CO CH C(CH 3 ) 2
(c) d-d electronic transition from t 2g eg state (B)
associated with an amount of energy which This polymer (B) is obtained when acetone is
comes under visible green region. saturated with hydrogen chloride gas, B can be
(d) d-d electronic transition from t2g* eg* (a) phorone
49. Prevention of corrosion of iron by Zn coating is (b) formose
called (c) diacetone alcohol
(a) Galvanization (d) mesityl oxide.
(b) Cathodic protection
55. Formic acid is obtained when
(c) Electrolysis
(a) Calcium acetate is heated with conc. H2SO4
(d) Photoelectrolysis
(b) Calcium formate is heated with calcium
50. Indicate which of the nitrogen compounds
acetate
amongst th e following would undergo
(c) Glycerol is heated with oxalic acid at 373 K
Hoffmann’s reaction, i,e., reaction with Br 2 and
strong KOH to furnish the primary amine (d) Acetaldehyde is oxidised with K2Cr2O7 and
(R – NH2) ? H2SO4.
56. In the Freidel Craft's acylation reaction, the
(a) R CO NHCH 3
effective electrophile is
(b) R – CO – ONH4
(a) RCOCl (b) AlCl3
(c) R – CO – NH2
(d) R – CO – NHOH. (c) RCOCl (d) RCO
EBD_7443
www.jeeneetbooks.in

WWW.IIT-NEET.XYZ

MT-40 Target VITEEE


57. The total number of possible isomers for the 1
complex compound [CuII (NH3)4] [PtII Cl4] (a) log k vs (b) k vs T
(a) 3 (b) 6 log T
(c) 5 (d) 4 1 1
58. Identify the incorrect statement among the (c) k vs (d) log k vs
log T T
following:
(a) Lanthanoid contraction is the accumulation 64. The activation energy for a simple chemical
of successive shrinkages. reaction A B is Ea in forward direction. The
(b) As a result of lanthanoid contraction, the activation energy for reverse reaction
properties of 4d series of the transition (a) Is always double of Ea
elements have no similarities with the 5d (b) Is negative of Ea
series of elements. (c) Is always less than Ea
(d) Can be less than or more than E a
(c) Shielding power of 4f electrons is quite
65. If is the fraction of HI dissociated at equilibrium
weak.
(d) There is a decrease in the radii of the atoms in the reaction, 2 HI (g) H2 (g) + I2 (g),
or ions as one proceeds from La to Lu. starting with 2 moles of HI, the total number of
59. Wooden artifact and a freshly cut down tree give moles of reactants and products at equilibrium
7.6 and 15.2 counts min –1 g –1 of carbon are
(t1/2 = 5760 years) respectively. The age of the (a) 2 + 2 (b) 2
artifact is (c) 1 + (d) 2 –
66. Aniline is an activated system for electrophilic
(a) 5760 years
substitution. The compound formed on heating
(b) 5760 × (7.6/15.2) years
aniline with acetic anhydride is
(c) 5760 × (15.2/7.6) years
NH2 NH2
(d) 5760 × (15.2 – 7.6) years
60. Half-life of a radioactive particle is 1 second. The
rate of dissociation of A is 1000 per second. Then (a) (b)
COCH3
after 3 sec, (A) will be
COCH3
(a) 500 (b) 250
(c) 125 (d) 333 NH2 NHCOCH3

61. Half life of a first order reaction is 4 s and the (c) (d)
initial concentration of the reactants is 0.12 M. COCH3
The concentration of the reactant left after 16 s 67. Which of the following reagents will convert p-
is methylbenzenediazonium chloride into p-
(a) 0.0075 M (b) 0.06 M cresol?
(c) 0.03 M (d) 0.015 M (a) Cu powder (b) H2O
62. When a biochemical reaction is carried out in (c) H3PO2 (d) C6H5OH
laboratory in the absence of enzyme then rate of 68. [A ]
reduction
[ B]
CHCl 3 KOH
reaction obtained is 10–6 times, then activation
reduction
energy of reaction in the presence of enzyme is [ C] N Methylanil ine , A is
6 (a) Formaldehyde (b) Trichloromethane
(a) (c) Nitrobenzene (d) Toluene
RT 69. A nitro alkane reacts with HONO to give
(b) Different from Ea obtained in laboratory insoluble product in alkali which turns blue on
(c) P is required treatment with an alkali. The nitric alkane is
(d) Can't say anything (a) CH 3 CH 2 NO 2
63. The temperature dependence of rate constant
(k) of a chemical reaction is written in terms of (b) CH 3 CH CH 2 NO 2
|
-E / RT*
CH 3
Arrhenius equation, k = Ae a . Activation
energy (E*a ) of the reaction can be calculated (c) (CH 3 ) 2 CHNO 2
by plotting (d) (CH 3 )3 CNO 2
www.jeeneetbooks.in

WWW.IIT-NEET.XYZ

Mock Test 4 MT-41

73. Indenfity Z in the sequence


70. MgBr
P2 O5 H 2O / H
CH 3 COONH 4 X Y Z
( i ) CO 2
P
( ii ) H 3O
(a) CH3CH2CONH2 (b) CH3CN
In the above reaction product 'P' is (c) CH3COOH (d) (CH3CO)2O
CHO COOH 74. Which of the following pairs of a chemical
reaction is certain to result in a spontaneous
(a) (b) reaction?
(a) Exothermic and increasing disorder
(b) Exothermic and decreasing disorder
OH
O (c) Endothermic and increasing disorder
||
(c) (d) C 6H5 C C6H 5 (d) Endothermic and decreasing disorder
75. What is the entropy change (in JK–1 mol–1) when
one mole of ice is converted into water at 0º C?
71. In a set of the given reactions, acetic acid yielded
(The enthalpy change for the conversion of ice
a product C. to liquid water is 6.0 kJ mol–1 at 0ºC)
C6 H6 (a) 21.98 (b) 20.13
CH3COOH PCl5 A B
Anh.AlCl3 (c) 2.013 (d) 2.198
C2H 5MgBr 76. The densities of graphite and diamond at 298 K
C
Ether are 2.25 and 3.31 g cm–3, respectively. If the
standard free energy difference ( Gº) is equal
Product C would be - to 1895 J mol–1, the pressure at which graphite
C2 H 5 will be transformed into diamond at 298 K is
| (a) 9.92 × 105 Pa (b) 9.92 × 108 Pa
(a) CH 3 C (OH)C 6 H 5 7
(c) 9.92 × 10 Pa (d) 9.92 × 106 Pa
(b) CH 3CH(OH)C 2 H 5 77. In the fluorite structure, the coordination number
of Ca2+ ion is :
(c) CH 3COC 6 H 5 (a) 4 (b) 6
(d) CH 3CH(OH)C6 H 5 (c) 8 (d) 3
78. On doping Ge metal with a little of In or Ga, one
CH2CH3 gets
(a) p-type semi conductor
(i ) KMO 4 / OH
72. X (b) n-tpe semi conductor
( ii ) H / H 2O (c) insulator
(d) rectifier
Predict ‘X’ in the above reaction 79. During the process of digestion, the proteins
present in food materials are hydrolysed to
CH2COOH CH2 CHO
amino acids. The two enzymes involved in the
process
(a) (b) Proteins Enzyme( A) Polypeptides
Enzyme(B) Amino acids
COOH CHO are respectively
(a) Diastase and Lipase
(c) (d) (b) Pepsin and Trypsin
(c) Invertase and Zymase
(d) Amylase and Maltase
EBD_7443
www.jeeneetbooks.in

WWW.IIT-NEET.XYZ

MT-42 Target VITEEE


80. Which one of the following chemical units is (a) 42 (b) 49
certainly to be found in an enzyme? (c) 50 (d) 51
OH H
O log e (1 x 2 tan x )
O N—C 86. If f ( x ) for x 0 is to be
(a) (b) sin x 3
HO HO O O continuous at x = 0, then f(0) must be defined as:
O (a) 0 (b) 1
O O R
N O R 1
(c) (d) (c) (d) –1
O R 2
N
87. If the eccentricity of the hyperbola x – y2 sec2
2
O
= 4 is 3 times the eccentricity of the ellipse
PART - III : MATHEMATICS x2sec2 + y2 = 16, then the value of equals
3 1 3
1 1 1 (a) (b)
81. tan cos 2 tan sin 2 cot is 6 4
4 2

(c) (d)
(a) not real (b) equal to 3 2
4 88. If A, B, C are points (1, 0, 4), (0, –1, 5) and
(2, –3, 1) respectively, then the coordinates of
(c) greater than (c) less than foot of the perpendicular drawn from A to the
4 4 line BC are
82. The general value of that satisfies the equation
1 1 9
2cot 2 2 3 cot 4cosec 8 0 is : (a) , , (b) (1, –2, 3)
2 2 2
(a) n (b) n
6 6 3 3 5
(c) , , (d) None of these
(c) 2n (d) 2n 2 2 2
6 6
83. Which of the following functions is NOT 1
2 x
one-one ? 89. The value of log dx is
2 x
(a) f : R R defined by f (x) 6x 1 1
(a) 0 (b) 2
(b) f:R R defined by f (x) x 2 7 (c) loge2 (d) log e 2
(c) f:R R defined by f (x) x 3 90. The values of p, for which both the roots of
equation 4x2 – 20px + 25p2 + 15p – 66 = 0 are less
2x 1 than 2, belong to :
(d) f : R {7} R defined by f (x)
x 7 4 4
(a) ,2 (b) 1,
3 5 5
84. sin 2 cos 1 is equal to (c) (2, ) (d) (– , –1)
5
91. Let A 2î k̂ , B î ĵ k̂ and
6 24
(a) (b)
25 25 C 4î 3ˆj 7 k̂ . The vector R which
4 24 satisfies the equations
(c) (d) –
5 25 R B C B and R . A 0 is given by
85. Suppose that the number of elements in set A
is p, the number of elements in set B is q and (a) 2î k̂ (b) î 8 ĵ 2k̂
the number of elements in A × B is 7. Then
1
p2 + q2 = (c) (î ĵ 2k̂ ) (d) None of these
6
www.jeeneetbooks.in

WWW.IIT-NEET.XYZ

Mock Test 4 MT-43

92. Let A = {1, 2, 3, 4}. The function f : A A and 98. The minimum value of |z| + | z – i| is
g:A A are defined in the table given below.. (a) 0 (b) 1
x 1 2 3 4 (c) 2 (d) none
f (x) 3 2 4 1
dy
g (x) 4 3 2 2 99. If x m y n ( x y ) m n then
The value of x, for which (fog) (x) = (gof) (x), is : dx
(a) 1 (b) 2 my ny
(c) 3 (d) 4 (a) (b)
mx nx
93. The population of a village increases
continuously at the rate proportional to the y
number of its inhabitants present at any time. If (c) (d) none of these
x
the population of the village was 20, 000 in 1999 100. If the system of equations x + 2y – 2z = 1, 4x +
and 25000 in the year 2004, what will be the 2 y – z = 2, 6x + 6y + z = 3 has a unique solution,
population of the village in 2009? then
(a) 3125 (b) 31250 (a) 1 (b) 2
(c) 21350 (d) 12350 (c) 3 (d) None of these
94. If a, b, c are sides of a triangle and 101. A plane passes through a fixed point (a, b, c).
a2 b2 c2 The locus of the foot of the perpendicular to it
from the origin is the sphere
(a 1) 2 (b 1) 2 (c 1) 2 0 , then
(a 1) 2 (b 1) 2 (c 1) 2 (a) x2 y2 z 2 ax by cz 0

(a) ABC cannot be equilateral triangle (b) x2 y2 z2 2ax 2by 2cz 0


(b) ABC is a right angled isosceles triangle
(c) ABC is an isosceles triangle (c) x 2 y 2 z 2 4ax 4by 4cz 0
(d) None of these (d) None of these
95. Let A, B, C be three events. If the probability of 102. The family of curves for which the area of the
occurring exactly one event out of A and B is triangle formed by the x-axis, the tangent drawn
1– a, out of B and C is 1– 2a and out of C and A at any point on the curve and radius vector of
is 1 – a, and that of occurring three events the point of tangency is constant equal to 2a2,
simultaneously is a2, then the probability that at is given by
least one out of A, B, C will occur is
a2 a2
1 1 (a) x cy (b) y cx
(a) (b) y x
2 2
1 (c) x 2 ay 2 cy (d) a 2 x 2 y 2 cy
(c) (d) none of these 103. Out of 7 consonants and 4 vowels, how many
2
words can be made each containing 3
96. The number of values of x ( x ) which
consonants and 2 vowels?
satisfy the equation (a) 120 (b) 25200
2 (c) 4200 (d) None of these
81 |cos x| |cos x| ..... to 43 is
(a) 2 (b) 4 104. The equation of normal to the curve x y xy,
(c) 6 (d) 8 where it cuts x-axis, is :
97. If a point (h, k) satisfies an inequation ax + by (a) y = x (b) y = x + 1
4, then the half plane represented by the (c) y = x – 1 (d) x + y = 1
inequation is
(a) The half plane containing the point (h, k) 105. If a is any vector, then
but excluding the points on ax + by = 4 ˆi a ˆi ˆj a ˆj kˆ a kˆ is equal to
(b) The half plane containing the point (h, k)
and the points on ax + by = 4 (a) (b)
a 2a
(c) Whole xy-plane
(d) None of these (c) 3a (d) 0
EBD_7443
www.jeeneetbooks.in

WWW.IIT-NEET.XYZ

MT-44 Target VITEEE


106. Sum of the series ( x 2 1)
111. dx is equal to
1 2 3 x x 4 3x 2 1
....
1 12 14 1 22 24 1 32 34
1 1
up to n terms is equal to (a) log x x2 3 C
x x2
n2 n 1 n2 n 1 1
(a) 2 (b) 2 (b) log x x2 3 C
2(n n 1) 2(n n 1) x x2

n2 n 1 n2 n (c) log x x2 3 C
(c) 2 (d) 2
n n 1 2(n n 1) (d) None of these
107. There is 30% chance that it rains on any particular 112. Let f (z) = sinz and g(z) = cosz. If * denotes a
day. Given that there is at least one rainy day, composition of functions, then the value of
then the probability that there are at least two (f + ig) * (f – ig) is :
rainy days is –iz eiz
(a) i e– e (b) ie
6
14 7 – e– i z
6
5 10 7 14 (c) – i e (d) None of these
(a) (b) 113. Suppose that the probability that an item
7
7 10 17
1 produced by a particular machine is defective
10 equals 0.2. If 10 items produced from this machine
are selected at random, the probability that not
6 more than one defective is found is
14 7
6 1
13 7 15 10 1 2
(c) (d) (a) 2 (b)
5 10 7
7 e e2
1
10 3
(c) (d) none of these
108. The logical expression X, in its simplest form for e2
the truth table 114. Four distinct points (2k, 3k), (1, 0), (0, 1) and
(0, 0) lie on a circle for
a b X
1 0 0
(a) only one value of k
1 1 1 is (b) 0 < k < 1
0 1 0
(c) k < 0
0 0 0
(d) all integral values of k
(a) X = a . b (b) X = a + b (c) |x| 1 , x 0
X = a'. b (d) X = a . b' 115. If f (x) = 0 , x 0 then lim f (x) exists for
x a
109. < N, + > where N is set of natural numbers is |x| 1 , x 0
(a) a semi group (b) a monoid group all
(c) both (a) and (b) (d) none (a) a 1 (b) a 0
110. The angle between the lines whose direction (c) a –1 (d) a 2
cosines are given by the equations 116. Area bounded by the parabola y = x2 – 2x + 3
3l m 5n 0 , 6 nm 2 nl 5lm 0 is and tangents drawn to it from the point P(1, 0) is
equal to
1 1 1 1
(a) cos (b) cos 4 2
6 6 (a) 4 2 sq. units (b) sq. units
3
1 2 1 5
(c) cos (d) cos 8 2 16
3 6 (c) sq. units (d) 2 sq. units
3 3
www.jeeneetbooks.in

WWW.IIT-NEET.XYZ

Mock Test 4 MT-45

117. The length of the line segment joining the vertex do? The administration set up remains week mainly
of the parabola y2 = 4ax and a point on the because the employees do not have the right example
parabola where the line segment makes an angle to follow and they are more concerned about being in
the good books of the bosses than doing work.
4am
to the x-axis is . Here, m and n 121. The employees in our country
n
respectively are (a) are quite punctual but not duty conscious
(a) sin , cos (b) cos , sin
2 (b) are not punctual, but somehow manage to
(c) cos , sin (d) sin2 , cos
complete their work
118. A balloon ascends with uniform acceleration of
(c) are somewhat lazy but good natured
5
122 cm/sec units, at the end of half a minute a (d) are not very highly qualified
8
body is released from it. The time that elapses 122. According to the writer, the administration in
before the body reaches the ground in sec is : India
(a) 10 (b) 15 (a) is by and large effective
(c) 20 (d) none
(b) is very strict and firm
1
119. The domain of f (x) = - 1- x 2 is: (c) is affected by red tape
2x -1
ù1 é (d) is more or less ineffective
(a) ú ,1ê (b) [ – 1, [
úû 2 êë 123. The word 'assessment' means
(c) [1, [ (d) None of these (a) enquiry
120. The number of ways of selecting 8 books from a
library which has 9 books each of Mathematics, (b) report
Physics, Chemistry and English is : (c) evaluation
(a) 165 (b) 27C8
8 (d) summary
(c) 3 (d) None of these
124. Choose the word which is most dissimilar to the
PART - IV : ENGLISH
word given in bold.
Direction (Qs. 121 - 123) Read the passage carefully
Abate
and answer the questions given below
(a) augment
What needs to be set right is our approach to work. It
is a common sight in our country of employees (b) free
reporting for duty on time and at the same time doing (c) provoke
little work. If an assessment is made of time they spent
in gossiping, drinking tea, eating "pan" and smoking (d) wane
cigarettes, it will be shocking to know that the time 125. Choose the best pronunciation of the word, Genre,
devoted to actual work is negligible. The problem is from the following options.
the standard which the leadership in administration
sets for the staff. Forgot the ministers because they (a) zhon-ruh (b) jen-ner
mix politics and administration. What do top (c) zon-er (d) zon-ra
bureaucrats do? What do the below down officials
EBD_7443
www.jeeneetbooks.in

WWW.IIT-NEET.XYZ

MOCK
VITEEE Mock Test Paper
5
Max. Marks : 125 Time : 2½ hrs
PART - I : PHYSICS 5. In a metre-bridge, the balancing length from the
left end when standard resistance of 1 is in
1. Six charges of equal magnitude, 3 positive and right gap is found to be 20 cm. The value of
3 negative are to be placed on PQRSTU corners unknown resistance is
of a regular hexagon, such that field at the centre (a) 0.25 (b) 0.4
is double that of what it would have been if (c) 0.5 (d) 4
only one +ve charge is placed at R. 6. A wire has a resistance of 3.1 at 30ºC and a
P Q resistance 4.5 at 100ºC. The temperature
coefficient of resistance of the wire
U R
O (a) 0.0064 ºC–1 (b) 0.0034 ºC–1
T S –1
(c) 0.0025 ºC (d) 0.0012 ºC–1
(a) + , +, +, –, –, – (b) –, + , +, +, –, – 7. Two batteries, one of emf 18 volt and internal

(c) –, + , +, –, +, – (d) + , –, +, –, +, –
2. Steam is passed into 22 g of water at 20°C . The
mass of water that will be present when the water
acquires a temperature of 90°C is (Latent heat of
steam is 540 cal/gm)
(a) 24.8 gm (b) 24 gm resistance 2 and the other of emf 12 volt and
(c) 36.6 gm (d) 30 gm internal resistance 1 , are connected as
3. Two capacitors of capacitances C1 and C2 are shown. The voltmeter V will record a reading
of
connected in parallel across a battery. If Q1 and
(a) 30 volt (b) 18 volt
Q2 respectively be the charges on the capacitors,
(c) 15 volt (d) 14 volt
Q1 8. A galvanometer can be converted into a
then will be equal to
Q2 voltmeter by connecting
(a) A high resistance in parallel
C2 C1 (b) A low resistance in series
(a) (b) (c) A high resistance in series
C1 C2
(d) A low resistance in parallel
9. An electron moves in a circular orbit with a
C12 C 22 uniform speed v. It produces a magnetic field
(c) (d)
C 22 C12 B at the centre of the circle. The radius of the
circle is proportional to
4. Streamline flow is more likely for liquids with
(a) high density and low viscosity B B
(a) (b)
(b) low density and high viscosity v v
(c) high density and high viscosity v v
(d) low density and low viscosity (c) (d)
B B
www.jeeneetbooks.in

WWW.IIT-NEET.XYZ

Mock Test 5 MT-47

10. The Young’s modulus of a perfectly rigid body 18. A parallel beam of monochromatic light of
is wavelength 5000Å is incident normally on a
single narrow slit of width 0.001 mm. The light is
(a) unity
focussed by a convex lens on a screen placed in
(b) zero focal plane. The first minimum will be formed for
(c) infinity the angle of diffraction equal to
(d) some finite non-zero constant (a) 0° (b) 15°
(c) 30° (d) 50°
11. A beam of electron passes undeflected through
mutually perpendicular electric and magnetic fields. 19. The angular separation d between two
If the electric field is switched off, and the same wavelength and + d in a diffraction grating
magnetic field is maintained, the electrons move is directly proportional to
(a) in a circular orbit (a) frequency of light
(b) grating element
(b) along a parabolic path
(c) spatial frequency of grating
(c) along a straight line
(d) wavelength of light
(d) in an elliptical orbit.
20. A stationary body of mass 3 kg explodes into
12. A heating coil is labelled 100 W, 220 V. The coil is
cut in half and the two pieces are joined in parallel three equal pieces. Two of the pieces fly off in
to the same source. The energy now liberated two mutually perpendicular directions, one with
per second is
a velocity of 3iˆ ms 1 and the other with a
(a) 25 J (b) 50 J
(c) 200 J (d) 400 J velocity of 4jˆ ms 1. If the explosion occurs in
13. Eddy currents are produced when 10–4 s, the average force acting on the third piece
(a) a metal is kept in varying magnetic field in newton is y
(b) a metal is kept in steady magnetic field
(c) a circular coil is placed in a magnetic field (a) (3iˆ 4j)
ˆ 10 4
(d) through a circular coil, current is passed 1 4ˆj
14. If the number of turns per unit length of a coil of (b) (3iˆ 4j)
ˆ 10 4 x
1 3iˆ
solenoid is doubled, the self-inductance of the )
4j
solenoid will (c) (3iˆ 4ˆj) 10 4 (3
i+
(–
(a) remain unchanged 1×
(b) be halved (d) (3iˆ 4j)
ˆ 104
(c) be doubled 21. Which one of the following is accompanied by
(d) become four times the characteristic X-rays emission ?
15. The time constant of C-R circuit is (a) -particle emission
(a) 1/CR (b) C/R (b) Electron emission
(c) CR (d) R/C (c) Positron emission
16. In a circuit, L, C and R are connected in series (d) k-electron capture
with an alternating voltage source of frequency 22. A spectral line results from the transition n = 2 to
f. The current leads the voltage by 45°. The value n = 1 in the atoms/species given below. Which
of C is one of these will produce th e shortest
1 1 wavelength emission ?
(a) (b) (a) Hydrogen atom
f (2 fL R ) 2 f (2 fL R )
(b) Singly ionised helium
1 1 (c) Doubly ionised lithium
(c) f (2 fL R )
(d) 2 f (2 fL R ) (d) Deuterium atom
17. The frequency of electromagnetic wave, which 23. K wavelength emitted by an atom of atomic
is best suited to observe a particle of radius number Z = 11 is . Find the atomic number for
3 × 10–4 cm is of the order of an atom that emits K radiation with wavelength
4
(a) 1015 (b) 1014
13
(a) Z = 6 (b) Z = 4
(c) 10 (d) 1012 (c) Z = 11 (d) Z = 44
EBD_7443
www.jeeneetbooks.in

WWW.IIT-NEET.XYZ

MT-48 Target VITEEE


24. The glancing angle in a X-ray diffraction is 30º M(2 He4) =4.00387 amu, then in the nuclear
and the wavelength of X-rays used is 20 nm. the reaction
6 2 2 2He4, the energy released will be
interplanar spacing of the crystal dffracting these 3Li + 1H
X-rays will be (a) 20 MeV (b) 22 MeV
(a) 40 nm (b) 20 nm (c) 50 MeV (d) 44 MeV
(c) 15 nm (d) 10 nm 32. What is the product aXb of the following nuclear
25. If the energy of a photon is 10 eV, then its reaction ?
14 4 16 b
momentum is 7N + 2He 8 O + aX
(a) 5.33 × 10–23 kg m/s (a) Electron (b) Neutron
(b) 5.33 × 10–25 kg m/s (c) Proton (d) Deutron
(c) 5.33 × 10–29 kg m/s 33. The following truth table belongs to which of
(d) 5.33 × 10–27 kg m/s the following four gates?
26. In a photoelectric experiment the stopping A B Y
potential for the incident light of wavelength 1 1 0
4000Å is 2 volt. If the wavelength be changed to 1 0 0
3000 Å, the stopping potential will be
0 1 0
(a) 2 V (b) Zero
(c) Less than 2 V (d) More than 2 V 0 0 1
27. A particle with rest mass m0 is moving with speed (a) NOR (b) XOR
of light c. The de-Broglie wavelength associated (c) NAND (d) OR
with it will be 34. The transfer ratio of a transistor is 50. The
(a) (b) zero input resistance of the transistor when used in
(c) m0 c/h (d) h /m0c the common emitter configuration is 1 k . The
peak value of the collector A.C. current for an
28. Figure shows three forces applied to a trunk that A.C. input voltage of 0.01 V peak is
moves leftward by 3 m over a smooth floor. The (a) 100 A (b) 0.01 mA
force magnitudes are F1 = 5N, F2 = 9N, and F3 = (c) 0.25 mA (d) 500 A
3N. The net work done on the trunk by the three 35. Which of the following gates will have an output
forces of 1?
1 0
1 1
(A) (B)
0 0
1 0
(C) (D)
(a) D (b) A
(c) B (d) C
36. The following figure shows a logic gate circuit
(a) 1.50 J (b) 2.40 J with two inputs A and B and the output C. The
(c) 3.00 J (d) 6.00 J voltage waveforms of A, B and C are as shown
29. The activity of a radioactive sample is measured below
as 9750 counts per minute at t = 0, as 975 counts A
Logic gate
per minute at t = 5 min. The decay constant is B circuit C
approximately,
1
(a) 0.230 per minute (b) 0.461 per minute
(c) 0.691 per minute (d) 0.922 per minute A t
30. A radioactive substance has a half life of four 1
months. Three fourth of the substance will decay B t
in 1
(a) three months (b) four months
t
(c) eight months (d) twelve months C
31. If the masses of the nuclei are The logic circuit gate is
M(1H2) = 2.0147 amu, (a) NAND gate (b) NOR gate
M(3Li6) =6.017 amu and (c) OR gate (d) AND gate
www.jeeneetbooks.in

WWW.IIT-NEET.XYZ

Mock Test 5 MT-49

37. A marble block of mass 2 kg lying on ice when NH 2


given a velocity of 6 m/s is stopped by friction Br
(1) HONO
A
Sn / HCl
B
(1) HONO
C
in 10 s. Then the coefficient of friction is (Take g ( 2 ) CuCl ( 2 ) H 3 PO2

= 10 ms–2)
NO 2
(a) 0.06 (b) 0.03
(c) 0.04 (d) 0.01 Cl
(a) Br Br (b)
38. Advantage of optical fibre is
(a) high bandwidth and EM interference
(b) low bandwidth and EM interference Br Br
(c) high and width, low transmission capacity NO 2
Cl
and no EM interference
(d) high bandwidth, high data transmission (c) Br Br (c) None of these
capacity and no EM interference
39. If 1 and 2 are the refractive indices of the
44. Which of the following gives the best yield of
materials of core and cladding of an optical fibre,
metabromo aniline ?
then the loss of light due to its leakage can be
minimised by having HNO3 (1) Sn , HCl
(a)
(a) 1> 2 (b) 1 < 2 H 2SO 4 (2 ) NaOH
(c) 1 = 2 (d) none of these
Br2 / FeCl3
40. A particle of mass m1 moving with velocity v
HNO3 Br2
strikes with a mass m2 at rest, then the condition (b)
for maximum transfer of kinetic energy is H 2SO 4 FeBr3

(a) m1 >> m2 (b) m2 >> m2 (1) Sn , HCl

(c) m1 = m2 (d) m1 = 2m2 (2 ) NaOH

Br2 HNO3
PART - II : CHEMISTRY (c)
FeBr3 H 2SO 4
41. One organic alcohol is irradiated with IR source
and it produces three streching frequency such (1) Sn , HCl
as 3391 cm–1, 2981 cm–1 and 1055 cm–1. Name of (2 ) NaOH
that organic alcohol is
(a) Methanol (b) Ethanol (d) (1) Sn , HCl HNO3
(c) Isobutanol (d) Isopropanol (2 ) NaOH H 2SO 4
42. The following reaction is Br2
NO2 FeBr3

heat 45. Which reaction sequence would be best to


+ KOH (solid)
prepare 3-chloroanilne from benzene ?
NO2 NO2 (a) Chlorination, nitration, reduction
OH
+
(b) Nitration, chlorination, reduction
(c) Nitration, reduction, chlorination
(d) Nitration, reduction, acetylation,
OH
chlorination, hydrolysis
(a) nucleophilic substituton 46. General electronic configuration of lanthanides
(b) electrophilic substitution is
(c) fee radical substitution (a) (n – 2) f1 –14 (n –1) s2p6d0 – 1 ns2
(d) electrophilic addition (b) (n – 2) f10 –14 (n –1) d0 – 1 ns2
43. The product – (C) obtained in the following (c) (n – 2) f0 –14 (n –1) d10 ns2
sequence of reactions is (d) (n – 2) d0 –1 (n –1) f1 – 14 ns2
EBD_7443
www.jeeneetbooks.in

WWW.IIT-NEET.XYZ

MT-50 Target VITEEE


47. According to IUPAC nomenclature sodium 53. Which one of the following radioisotopes is
nitroprusside is named as used in the treatment of blood cancer ?
(a) Sodium pentacyanonitrosyl ferrate (III) (a) P32 (b) Co60
(b) Sodium nitroferrocyanide (c) I 131 (d) Na24
(c) Sodium nitroferrocyanide 54. For a first order reaction A B the reaction
(d) Sodium pentacyanonitrosyl ferrate (II) rate at reactant concentration of 0.01 M is
48. The number of unpaired electrons in the complex found to be 2.0 10 5 mol L 1 S 1. The half
ion [CoF6]3– is (Atomic no.: Co = 27) life period of the reaction is
(a) Zero (b) 2 (a) 30 s (b) 220 s
(c) 3 (d) 4 (c) 300 s (d) 347 s
49. Which of the following is considered to be an 55. The rate of certain reaction increases by 2-3 times
anticancer species ? when the temperature is raised from 300 K to 310
K. If K is the rate constant at 300 K then rate
Cl Cl
Cl CH2 constant at 310 K will be
(a) (b) Pt
Pt CH2 (a) 3 K 2 (b) 2.3 K
Cl Cl Cl
Cl (c) 2 K (d) K
56. In acidic medium the rate of reaction between
H3N Cl H3N Cl
(c) (d) BrO3 and Br ion is given by the expression
Pt Pt
H3N Cl Cl NH3 d(BrO3 ) 2
= K [BrO3 ][Br ][H ]
50. The main reason for larger number of dt
oxidation states exhibited by the actinoids Which of the following is correct ?
than the corresponding lanthanoids, is
(a) more energy difference between 5f and (a) Doubling the concentration of H ions
6d orbitals than between 4f an d 5d will increase the reaction rate by 4 times.
orbitals. (b) Rate constant of overall reaction is 4 sec–1.
(b) lesser energy difference between 5f and (c) Rate of reaction is independent of the conc.
6d orbitals than between 4f an d 5d of acid
orbitals. (d) The change in pH of the solution will not
(c) larger atomic size of actinoids than the affect the rate.
lanthanoids. 57. The reaction
(d) greater reactive nature of the actinoids 2N2O5 2N2O4 + O2 is
than the lanthanoids. (a) Bimolecular and second order
51. Colour of cupric chloride is blue is due to (b) Unimolecular and first order
__________. (c) Bimolecular and first order
(a) p -d electronic transition causes emission (d) Bimolecular and zero order
of energy 58. In a two-step exothermic reaction
(b) d-d electronic transition causes emission A2(g) + B2(g) 3C(g) D(g)
of energy Step 1 Step 2
(c) d-d electronic transition causes absorption Steps 1 and 2 are favoured respectively by
of energy at red visible wave length (a) high pressure, high temperature and low
(d) p-d electronic transition of Cu2+ state. pressure, low temperature
52. Which metal Aprons are worn by radiographer (b) high pressure, low temperature and low
to protect him from radiation? pressure, high temperature
(a) Mercury Coated Apron (c) low pressure, high temperature and high
(b) Lead Apron pressure, high temperature
(c) Copper Apron (d) low pressure, low temperature and high
(d) Aluminimised Apron pressure, low temperature
www.jeeneetbooks.in

WWW.IIT-NEET.XYZ

Mock Test 5 MT-51

59. The OH group of an alcohol or the –COOH group 65. The appearance of colour in solid alkali metal
of a carboxylic acid can be replaced by halides is generally due to
–Cl using (a) Schottky defect
(a) phosphorus pentachloride (b) Frenkel defect
(b) hypochlorous acid (c) Interstitial positions
(c) chlorine (d) F-centres
(d) hydrochloric acid 66. CsBr crystallises in a body centered cubic lattice.
60. A and B in the following reactions are The unit cell length is 436.6 pm. Given that the
OH atomic mass of Cs = 133 and that of
B R– C
R–C–R' HCN/ A Br = 80 amu and Avogadro number being
KCN CH2NH2
O R' 6.02 × 1023 mol–1, the density of CsBr is
CN (a) 0.425 g/cm3 (b) 8.25 g/cm3
(a) A = RR'C , B = LiAlH4 (c) 4.25 g/cm 3 (d) 42.5 g/cm3
OH
OH 67. Cu aq is unstable in solution and undergoes
(b) A = RR'C , B = NH3
COOH simultaneous oxidation and reduction according
CN to the reaction :
(c) A = RR'C , B H 3O
OH 2Cu ( aq ) Cu 2 (aq) Cu (s) choose correct
(d) A = RR'CH2CN, B = NaOH
Eº for above reaction if Eº Cu2+/Cu = 0.34 V and
61. Consider the following transformations :
Eº Cu2+/Cu+ = 0.15 V
CaCO 3 heat I2 (a) –0.38 V (b) +0.49 V
CH3COOH A B C
NaOH (c) +0.38 V (d) –0.19 V
The molecular formula of C is 68. Specific conductance of 0.1 M sodium chloride
solution is 1.06 × 10–2 ohm–1 cm–1. Its molar
OH
| conductance in ohm–1 cm2 mol–1 is
(a) CH 3 C CH 3 (b) ICH2 — COCH3 (a) 1.06 × 102 (b) 1.06 × 103
| 4
I (c) 1.06 × 10 (d) 5.3 × 102
(c) CHI3 (d) CH3I 69. Hydrogen-Oxygen fuel cells are used in space
62. Caboxylic acid group does not give the usual craft to supply
addition and elimination reactions of aldehydes (a) Power for heat and light
and ketones because (b) Power for pressure
(c) Oxygen
(a) O–H bond is more polar than C = O group (d) Water
(b) Carboxylate ion gets ionised 70. Corrosion is basically a
(c) Carboxylate ion gets stabilised by (a) altered reaction is presence of H2O
resonance (b) electrochemical phenomenon
(d) it exists as – COOH and there is no carbonyl (c) interaction
group (d) union between two light metals and a heavy
63. The enthalpy and entropy change for the metal
reaction 71. When during electrolysis of a solution of AgNO3
Br2(l) + Cl2 (g) 2 BrCl (g) 9650 coulombs of charge pass through the
are 30kJ mol–1 and 105 JK–1 mol–1 respectively. electroplating bath, the mass of silver deposited
The temperature at which the reaction will be in
on the cathode will be
equilibrium is
(a) 1.08 g (b) 10.8 g
(a) 273 K (b) 450 K
(c) 21.6 g (d) 108 g
(c) 300 K (d) 285.7 K
64. In graphite electrons are : 72. How many isomers of C5H11 OH will be primary
(a) Localised on each carbon atom alcohols ?
(b) spread out between the sheets (a) 5 (b) 4
(c) localised on every third carbon atom (c) 2 (d) 3.
(d) present in antibonding orbital. 73. HBr reacts fastest with
(a) 2-Mehtylpropan-1-ol
EBD_7443
www.jeeneetbooks.in

WWW.IIT-NEET.XYZ

MT-52 Target VITEEE


(b) 2-Methylpropene-2-ol
1
(c) propan-2-ol (a) ( y12 2ax1 )( y12 4a 2 )
(d) propan-1-ol. |a|
74. Which one of the following on oxidation gives a
1
ketone ? (b) ( y12 4ax1 )( y12 4a 2 )
(a) Primary alcohol |a|
(b) secondary alcohol 1
(c) tertiary alcohol (c) ( y12 4ax1 )( y12 2a 2 )
|a|
(d) All of these
75. An aromatic ether is not cleaved by HI even at (d) None of these
525 K. The compound is 31
(a) C6H5OCH3 (b) C6H5OC6H5 83. In a ABC, a = 5, b = 4 and cos( A B) .
32
(c) C6H5OC3H7 (d) Tetrahydrofuran Then c is
76. Acetic anhydride reacts with diethyl ether in the (a) 6 (b) 5
presence of anhydrous AlCl3 to give (c) 4 (d) 3
(a) CH3COOCH3 (b) CH3CH2COOCH3
(c) CH3COOCH2CH3 (d) CH3CH2OH 84. Let a 2iˆ ˆj k,
ˆ b ˆi 2ˆj kˆ and a unit
77. Phenol is vector ĉ be coplanar. If ĉ is perpendicular to
(a) a base stronger than ammonia â , then ĉ =
(b) an acid stronger than carbonic acid
(c) an acid weaker than carbonic acid 1 1
(a) ( ĵ k̂) (b) ( î ĵ k̂ )
(d) a neutral compound 2 3
CH 2 O 1 1
(c) (î 2ˆj) (d) (î ĵ k̂ )
78. O CH 2 5 3
CH 2 O 85. Let X = {1, 2, 3, 4} A function is defined from X
The above shown polymer is obtained when a to N as R ( x, f (x )) : x X, f (x ) xPx 1 .
carbonyl compound is allowed to stand. It is a The range of f is
white solid. The polymer is (a) {1, 2, 3, 4} (b) {1, 4, 9, 16}
(a) Trioxane (b) Formose (c) {1, 2, 6, 24} (d) {1}
(c) Paraformaldehyde (d) Metaldehyde. 86. If f(x) be a quadratic polynomial such that
79. The compound formed when malonic ester is f(0) = 2, f’(0) = – 3 and f’’ (0) = 4 then
heated with urea is 1
(a) Cinnamic acid (b) Butyric acid f (x ) dx is equal of
(c) Barbituric acid (d) Crotonic acid. 1
80. Schotten-Baumann reaction is a reaction of
phenols with 16
(a) –3 (b)
(a) Benzoyl chloride and sodium hydroxide 3
(b) Acetyl chloride and sodium hydroxide (c) 0 (d) none of these.
(c) Salicylic acid and conc. H2SO4 87. The tangent at a point P on the hyperbola
(d) Acetyl chloride and conc H2SO4 x2 y2
1 meets one of its directrices in F. If
PART - III : MATHEMATICS a 2 b2
PF subtends an angle at the corresponding
81. The equation k sin + cos 2 = 2k – 7 possesses a focus, then equals
solution if : 3
(a) 2 k 6 (b) k > 2 (a) (b) (c) (d)
(c) k > 6 (d) k < 2 4 2 4 12
88. In a right angled triangle one side is thrice the
82. If the tangents are drawn from the point (x1, y1)
other side in length. The is suspended by a
to the parabola y2 = 4ax, then the length of their string attached at the right angle. The angle that
chord of contact is the hypotenuse of the will make with the
vertical is
www.jeeneetbooks.in

WWW.IIT-NEET.XYZ

Mock Test 5 MT-53

(a) sin–1 (3/5) (b) sin–1 (4/5) 1


(c) 60º (d) None of these (a) –2 (b) (c) 0 (d) 2
89. | (a × b).c| = |a| |b||c| , if 2
(a) a.b = b. c = 0 (b) b.c = c. a = 0 97. The equation of two curves are x – 3xy2 = a2
3

(c) c.a = a.b = 0 (d) a.b = b.c = c.a = 0 and 3x2y – y3 = b2. The angle of intersection of
these curves is
3 1
90. If, P( B) , P(A B C )
4 3 (a) (b) (c) (d) 0
3 2 4
1 98. For binary operation * defined on R – {1} such
and P( A B C ) , then P (B C) is
3 a
that a * b is
1 1 1 1 b 1
(a) (b) (c) (d) (a) not associative (b) not commutative
12 6 15 9 (c) commutative (d) both (a) and (b)
91. If m sin = n sin ( + 2 ), then tan ( + ) . cot
is equal to 99. The value of the boolean expression [a.b' c].a '
if a = 0, b = 0, c = 1
m n m–n
(a) (b) (a) 1 (b) 0
m–n m n (c) 2 (d) none
m n m–n 100. Suppose , , R are such that
(c) (d)
mn mn sin , sin , sin 0 and
2 3 1
92. If P A , P B and P A B , sin 2 sin cos cos 2
5 10 5 2
sin sin cos cos 2 then
then P A | B . P B | A is equal to
sin 2 sin cos cos 2
5 5 25
(a) (b) (c) (d) 1 cannot exceed
6 7 42 (a) 1 (b) 0
93. Solution of differential equation xdy – ydx = 0
represents: 1
(c) (d) None of these
(a) rectangular hyperbola. 2
(b) parabola whose vertex is at origin. 101. Area of the region between th e curves
(c) circle whose centre is at origin. x2 + y2 = 2, y = sin x and y-axis in first quadrant
(d) straight line passing through origin. is
94. If and ( < ) are the roots of the equation 3 3
8 4
x 2 + bx + c = 0 , where, c < 0 < b, then (a) sq. units (b) sq. units
4 8
(a) 0 < < (b) 0 | | 2 2
8 4
(c) 0 (d) 0 | | (c) sq. units (d) sq. units
4 8
3
x x2 6
x 3
102. If the roots of z iz
2
2i 0 represent the
95. dx is equal to
x (1 3
x) vertices of a ABC in the Argand plane, then
the area of the triangle is
3 2/3
(a) x 6 tan 1 x1 / 6 C 3 7 3 7
2 (a) (b)
2 4
3 2/3
(b) x 6 tan 1 x1 / 6 C (c) 2 (d) None
2 103. If n + 2C8 : n – 2P4 = 57 : 16, then the value of n is:
3 2/3 (a) 20 (b) 19 (c) 18 (d) 17
(c) x 6 tan 1 x1 / 6 C 104. The sides of a triangle are three consecutive
2
(d) None of these. natural numbers and its largest angle is twice the
96. If the system of equations x + 2y – 3z = 1, (p + 2) smallest one. The length of the greatest side is
z = 3(2p + 1) y + z = 2 is inconsistent, then the (a) 4 (b) 5
value of p is (c) 6 (d) none of these
EBD_7443
www.jeeneetbooks.in

WWW.IIT-NEET.XYZ

MT-54 Target VITEEE


105. A family of curves is given by the equation 3 1
(c) a ,c ,b 0
x2 y2 2 2
= 1. The differential equation (d) none of these
a 2 b2
representing this family of curves is given by 1 5
2 109. The value of tan cos 1 is
d2 y dy dy 2 3
xy Ax – y = 0. The value of A
dx 2 dx dx
is 3 5 3 5
(a) (b)
(a) 0 (b) 1 (c) 3 (d) 5 2 2
106. A brick manufacture has two depots A and B,
with stocks of 30000 and 20000 bricks respectively. 5
(c) (d) none of these
He receive orders from three builders P, Q and R 6
for 15000, 20,000 and 15000 bricks respectively. 110. If 3f (cos x) + 2 f (sin x) = 5x, then f’(cos x) =
The cost (in `) of transporting 1000 bricks to the 5 5
builders from the depots as given in the table. (a) (b)
cos x cos x
To Transportation cost
From per 1000 bricks (in `) 5
(c) (d) none of these
P Q R sin x
A 40 20 20 111. Let A {x : 1 x 1} = B and S be the subset
B 20 60 40 of A×B defined by S = {(x,y) : x2 + y2 = 1}. This
The manufacturer wishes to find how to fulfill defines
the order so that transportation cost is minimum. (a) an one-one function from A into B
Formulation of the L.P.P., is given as (b) a many - one function from A into B
(a) Minimize Z = 40x – 20y
Subject to, x + y 15, x + y 30, x 15, y 20, (c) a bijective function from A into B
x 0, y 0 (d) Not a function
(b) Minimize Z = 40x – 20y 112. A variable plane moves so that the sum of
Subject to, x + y 15, x + y 30, x 15, y 20, reciprocals of its intercepts on the three
x 0, y 0 coordinate axes is constant . It passes through
(c) Minimize Z = 40x – 20y a fixed point, which has coordinates
Subject to, x + y 15, x + y 30, x 15, y 20,
x 0, y 0 1 1 1
(d) Minimize Z = 40x – 20y (a) ( , , ) (b) , ,
Subject to, x + y 15, x + y 30, x 15, y 20,
x 0, y 0 1 1 1
(c) , , (d) , ,
107. Total number of ways of selecting five letters from
letters of the word INDEPENDENT is
113. The probability of a man hitting a target is 1/4.
(a) 70 (b) 72 (c) 75 (d) 80
The number of times he must shoot so that the
108. The values of a, b and c which make the function
probability he hits the target, at least once is
sin(a 1)x sin x more than 0.9, is
, x 0 [use log 4 = 0.602 and log 3 = 0.477]
x
f (x) c ,x 0 (a) 7 (b) 8 (c) 6 (d) 5
114. The coordinates of a point on the line
x bx 2 x 6
,x 0 x 2 y 1 z 3
bx 3/ 2 at a distance of from
3 2 2 2
continuous at x = 0 are the point (1, 2, 3) is
3 1
(a) a ,c , b= 0 56 43 111
2 2 (a) (56,,43, 111) (b) , ,
17 17 17
3 1
(b) a ,c ,b 0 (c) (2, 1, 3) (d) (–2, –1, –3)
2 2
www.jeeneetbooks.in

WWW.IIT-NEET.XYZ

Mock Test 5 MT-55

115. Suppose that X has a Poisson distribution. If souls. However, there were an unpleasant number of
P(X 1) 0.3 , and P(x 2) 0.2 then questions about why the Estonia sank and why so
many survivors were men in the prime of life, while
P(X = 0) = most of the dead were women, children and the elderly.
(a) e 1/3 (b) e –3/2 (c) e –4/3 (d) 0.5
121. One can understand from the reading that ----.
116. The value of x which satisfy the equation
(a) the lifesaving equipment did not work well
1 + sin2ax = cos x, where a is irrational is/are
and lifeboats could not be lowered
, (b) design faults an d incompetent crew
(a) 0 (b) 0, (c) (d) 0,
2 2 contributed to the sinking of the Estonia
117. A curve passes through the point (5, 3) and at ferry
any point (x, y) on it, the product of its slope (c) 139 people managed to leave the vessel but
and the ordinate is equal to its abscissa. The died in freezing water
curve is (d) most victims were trapped inside the boat
(a) parabola (b) ellipse as they were in their cabins
(c) hyperbola (d) circle 122. It is clear from the passage that the survivors of
118. The lines whose vector equations are the accident ----.
r 2iˆ 3jˆ 7kˆ (2iˆ pjˆ 5k)ˆ (a) helped one another to overcome the tragedy
that had affected them all
and r iˆ 2jˆ 3kˆ (3iˆ pjˆ pk)ˆ
(b) were mostly young men but women,
are perpendicular for all values of and if p = children and the elderly stood little chance
(a) 1 (b) –1 (c) – 6 (d) 6 (c) helped save hundreds of lives
sin( cos 2 x ) (d) are still suffering from severe post-traumatic
119. lim equals stress disorder
x 0 x2
123. According to the passage, when the Estonia
sank, ----.
(a) (b) (c) (d) 1 (a) there were only 139 passengers on board
2
120. Which one of the following relations on the set of (b) few of the passengers were asleep
real numbers R is an equivalence relation ? (c) there were enough lifeboats for the number
(a) aR1b |a| |b| (b) aR 2 b a b of people on board
(d) faster reaction by the crew could have
(c) aR 3b a divides b (d) aR 4 b a b increased the Estonia's chances of survival
PART - IV : ENGLISH 124. Fill in the blanks with the appropriate words from
the option to complete the sentence.
Direction (Qs. 121 -123) : Read the passage carefully A rising China and the anti-India _________of
and answer the questions given below. Kathmandu's hill elite, however, have the
Naval architects never claim that a ship is unsinkable, potential to neutralize, over the longer term, some
but the sinking of the passenger-and-car ferry Estonia of Delhi's natural _________advantages in
in the Baltic surely should have never have happened. Nepal.
It was well designed and carefully maintained. It carried (a) happiness, unnecessary
the proper number of lifeboats. It had been thoroughly
(b) good nature, unimportant
inspected the day of its fatal voyage. Yet hours later,
the Estonia rolled over and sank in a cold, stormy (c) resentments, strategic
night. It went down so quickly that most of those on (d) enjoyment, common
board, caught in their dark, flooding cabins, had no 125. Choose the best pronunciation of the word,
chance to save themselves: Of those who managed to Debris, from the following options.
scramble overboard, only 139 survived. The rest died (a) deb-rees (b) de-bree
of hypothermia before the rescuers could pluck them (c) deb-ris (d) deb-re
from the cold sea. The final death toll amounted to 912
EBD_7443
www.jeeneetbooks.in

WWW.IIT-NEET.XYZ

MOCK
VITEEE Mock Test Paper
6
Max. Marks : 125 Time : 2½ hrs
PART - I : PHYSICS
1 q2
(a)
1. Two particles A and B, move with constant 4 0 d2
velocities v1 and v2 . At the initial moment their
1 q2
(b) more than
position vectors are r1 and r2 respectively. The 4 0 d2
condition for particles A and B for their collision
is 1 q2
(c) Less than
(a) r1.v1 r2 .v2 4 0 d2
(b) r1 v1 r2 v2 (d) zero
4. There capacitors C1, C2 and C3 are connected
(c) r1 r2 v1 v 2 as shown in the figure. A potential difference of
r1 r2 v2 v1 14 volts is applied to the input terminals. What
(d) is the charge on C3 (in µC) ?
| r1 r2 | | v2 v1 |
2. In the figure, a proton moves a distance d in a
C1= 4µF
uniform electric field E as shown in the figure.
Does the electric field do a positive or negative 14 volt C2 C3
work on the proton? Does the electric potential 4F
energy of the proton increase or decrease? 2µF

E
(a) 8 (b) 4
(c) 42 (d) 100
+
p 5. The radii of two concentric spherical conducting
d shells are r 1 and r2 (> r1). The charge on the
(a) Negative, increase outer shell is q. What will be the charge on the
inner shell which is connected to the earth ?
(b) Positive, decrease
(c) Negative, increase r1 r2
(d) Positive, increase (a) q (b) q
r2 r1
Here, distance increases so, potential energy
increases. (c) – q (d) zero
3. Two charged spherical conductors (each of 6. Figure shows three points A, B and C in a region
radius R) are at a distance d part (d > 2R). The of uniform electric field E . The line AB is
charge on one is +q and that on the other is – q. perpendicular and BC is parallel to the field lines.
The magnitude of the force between them will Then which of the following holds good?
be :
www.jeeneetbooks.in

WWW.IIT-NEET.XYZ

Mock Test 6 MT-57

E 3 D 3
A

B C 3
3 3
C E
F
(a) VA = VB = VC (b) VA = VB > VC A B
3
(c) VA = VB < VC (d) VA > VB = VC
A cell of emf 3V is connected across AB. The
Where VA, VB and VC represent the electric effective resistance across AB and the current
potentials at the points A, B and C respectively. through the arm AB will be
7. The work done in carrying a charge q once (a) 0.6 , 1 A (b) 1.5 , 2 A
around a circle or radius r with a charge Q placed (c) 0.6 , 2 A (d) 1.5 , 1 A
at the centre will be 12. In a given network, each resistance has value of
6 . The point X is connected to point A by a
(a) 2 (b) Qq /(4
Qq(4 0r ) 0r) copper wire of negligible resistance and point Y
is connected to point B by the same wire. The
(c) Zero (d) Qq 2 /(4 0r) effective resistance between X and Y will be
8. An object A has a charge of +2 µC and the object
B has charge of +6 µC. Which statement is true? 6 6 A6
X Y
B
(a) FAB 3FBA (b) FAB FBA
(a) 18 (b) 6
(c) 3 FAB FBA (d) FAB 4FBA (c) 3 (d) 2
9. An aluminium (Al) rod with area of cross-section 13. The figure shows three situations when an
4 × 10–6 m2 has a current of 5 ampere, flowing electron with velocity v travels through a
through it. Find the drift velocity of electron in
the rod. Density of Al = 2.7 × 103 kg/m3 and uniform magnetic field B . In each case, what is
the direction of magnetic force on the electron?
Atomic wt. = 27. Assume that each Al atom
provides one electron y y y
(a) 8.6 × 10–4 m/s (b) 1.29 × 10–4 m/s
(c) 2.8 × 10–2 m/s (d) 3.8 × 10–3 m/s v B B
x x x
10. Taking the internal resistance of the battery as B v v
negligible, the steady state current in the 2 z z z
resistor shown in the figure will be 1 2 3
2 (a) + ve z-axis, – ve x-axis, + ve y-axis
(b) – ve z-axis, – ve x-axis and zero
(c) + ve z-axis, + ve y-axis and zero
A B (d) – ve z-axis, + ve x-axis and zero
3 14. The magnetic field at P on the axis of a solenoid
having 100 turns/metre and carrying a current of
0.2 µF 5 A is:
6

6V 2.8
(a) 1.8 A (b) 2.9 A 45° 45°
(c) 0.9 A (d) 2.8 A
P
11. Six resistors, each of value 3 area connected
as shown in the fig.
EBD_7443
www.jeeneetbooks.in

WWW.IIT-NEET.XYZ

MT-58 Target VITEEE


20. In a series LCR circuit, if the applied voltage V
(a) 250 0 (b) 500 2 0 and the current in the circuit I at any instant t are
(c) 500 0 (d) 250 2 0 given as :
15. In an A.C. circuit resistance R, inductance L and V= V0 sin t and I = I0 sin ( t – )
capacitance C are in parallel. The value of R is then which of the following holds good :
1 1
50 , inductive reactance X L = 40 and (a) L (b) L
C C
capacitive reactance XC = 25 . Then the reading
of ammeter will be: 1
(c) L (d) none
C
C 21. A particle tied to a string describes a vertical
L circular motion of radius r continually. If it has a
velocity 3 gr at the highest point, then the
A ratio of the respective tensions in the string
R holding it at the highest and lowest points is
(a) 4 : 3 (b) 5 : 4
200V (c) 1 : 4 (d) 3 : 2
22. A generator at a utility company produces 100 A
(a) 10.2 A (b) 5 A
of current at 4000 V. The voltage is stepped up
(c) 17 A (d) 7 A
to 2,40,000V by a transformer before it is sent on
16. The current i passed in any instrument in an a high voltage transmission line. The current in
A.C. circuit is i = 2 sin t amp and potential transmission line is
difference applied is given by V = 5 cos t volt. (a) 3.67 A (b) 2.67 A
Power loss in the instrument is: (c) 1.67 A (d) 2.40 A
(a) 10 watt (b) 5 watt 23. A parallel monochromatic beam of light is incident
(c) zero watt (d) 20 watt normally on a narrow slit. A diffraction pattern is
17 In A.C. circuit in which inductance and formed on a screen placed perpendicular to the
capacitance are joined in series. Current is found direction of the incident beam. At the first
to be maximum when the value of inductance is minimum of the diffraction pattern, the phase
0.5 H and the value of capacitance is 8 F. The difference between the rays coming from the two
angular frequency of applied alternating voltage edges of slit is
(a) 0 (b)
will be:
(a) 400 Hz (b) 5000 Hz (c) (d) 2
(c) 2 × 105 Hz (d) 500 Hz 2
18. In a series LCR circuit the frequency of a 10 V 24. The electric field of a plane electromagnetic wave
a.c. voltage source is adjusted in such a fashion in free space travelling along x-direction is given
by : Ez = 60 sin (kx + t) V/m.
that the reactance of the inductor measures 15 The magnetic field component will be given as :
and that of the capacitor 11 . If R = 3 , the 60
potential difference across the series (a) Bz sin (kz t)T
c
combination of L and C will be:
(a) 8 V (b) 10 V 60
(b) Bz sin (kx t)T
(c) 22 V (d) 52 V c
19. What direct current will produce the same 60
amount of thermal energy in a resistance R = (c) By sin (ky t)T
c
2 as an alternating current that a peak value
of 4.24 A and frequency 50 Hz? 60
(d) By sin (kx t)T
(a) 3 A (b) 2 A c
(c) 5 A (d) 4 A Here c represents the speed of light in free space.
www.jeeneetbooks.in

WWW.IIT-NEET.XYZ

Mock Test 6 MT-59

2 32. For the given case which figure is correctly


25. The binding energy of deuteron ( 1 H ) is 1.15 showing the after inelastic collision situation?
4 m1
MeV per nucleon and an alpha particle ( 2He ) m2
u 1
has a binding energy of 7.1 MeV per nucleon. Before collision
Then in the reaction u 2= 0
2 2 2 v1
1H 1
H 2 He Q v2
the energy released Q is : (a) m2
(a) 5.95 MeV (b) 26.1 MeV m1
(c) 23.8 MeV (d) 289.4 MeV m2
26. When a slow neutron is captured by U235 (b) m1
nucleus. A fission results, which release 200 MeV v
energy. If the output power of a nuclear reactor
m1 m2
is 1.6 Mwatt. The rate of nuclei undergoing (c)
fission is: –v1 +v1
(a) 4 × 1016 (b) 5 × 1016
(c) 3 × 1016 (d) none of these (d) m1 m2
27. Light of frequency 5 v0 falls on a photosensitive –v
surface of the threshold frequency v 0 . The 33. Intrinsic semiconductor is electrically neutral,
velocity of emitted electrons is 4 × 105 m/s. If extrinsic semiconductor having large number of
now light of frequency 9 v0 is incident on the current carriers would be:
surface, then velocity of electron will become: (a) electrically neutral
(a) 5.64 × 105 m/s (b) 7.2 × 105 m/s (b) positively charged or negatively charged
(c) 8 × 105 m/s (d) 2 × 105 m/s depending upon the type of impurity that
28. What is the maximum wavelength of photon that has been added
would excite an electron in the valence band of (c) positively charged
diamond to the conduction band ? The energy (d) negatively charged
gap for diamond is 5.5 eV : 34. A transistor has an = 0.95. It has change in
(a) 169 nm (b) 205 nm emitter current of 100 mA then the change in
(c) 226 nm (d) 350 nm collector current is:
29. If the electron in hydrogen atom jumps from third (a) 95 mA (b) 99.05 mA
orbit to second orbit, the wavelength of the (c) 100.96 mA (d) 100 mA
emitted radiation is given by: 35. In a p-n junction diode acting as a half-wave
36 5R rectifier, which of the following statements is not
(a) (b) true?
5R 36
(a) The average output voltage over a cycle is
5 R non-zero
(c) (d)
R 6 (b) The drift current depends on biasing
30. For Balmer series that lies in the visible region, (c) The depletion zone decreases in forward
the shortest wavelength corresponds to biasing
quantum number: (d) The diffusion current increases due to
(a) n = 1 (b) n = 2 forward biasing
(c) n = 3 (d) n = 36. A transistor with = 0.98 is operated as an
31. A p type semiconductor has acceptor levels 57 amplifier in a common base circuit with a load
meV above the valance band. The maximum resistance of 5 k . If the dynamic resistance of
wavelength of light required to create hole is the emitter base junction is 70, the voltage gain
(Planck constant h = 6.6 × 10–34Js) and power gain of the circuit will be:
(a) 11.61 × 10–33 Å (b) 217105Å (a) 70, 68.6 (b) 72, 70.6
(c) 57 × 10–3 Å (d) 57 Å (c) 74, 72.6 (d) none of these
EBD_7443
www.jeeneetbooks.in

WWW.IIT-NEET.XYZ

MT-60 Target VITEEE


37. A transmitter radiates 10 kW of power with the 42. Pick out the correct statement of the following
carrier unmodulated and 11.8 kW with the carrier (a) The stability of either of HgCl2 and SnCl2
sinusoidally modulated. The modulation factor is not affected when present
is simultaneously in aqueous solution
(a) 56% (b) 60 % (b) Both Cu(OH)2 and Fe(OH)2 are soluble in
(c) 72 % (d) 84% aqueous NH3
38. The graph given is a stress-strain curve for (c) Copper (I) salts are not known in aqueous
solution
(d) White precipitate of Zn(OH)2 is obtained
1.0 on adding excess of NaOH to aqueous
ZnSO4.
Stress (N/ m2)

43. The E.A.N. of iron in [Fe(CN)6]3– is :


0.5 (a) 32 (b) 35
(c) 38 (d) 41
44. The complex [ Fe ( H 2 O) 5 NO ] 2 ion has a
0
0.5 1.0 magnetic moment of 3.87 B.M. This is indicative
Strain of the fact that
(a) Fe in this complex exists in + 1 oxidation
(a) elastic objects (b) plastics
(c) elastomers (d) None of these state and nitrosyl as NO ( nitrosonium ion)
39. In Bernoulli’s theorem which of the following is (b) The complex is octahedral in geometry as
conserved? attained by sp 3d 2 hybridization.
(a) Mass
(b) Linear momentum (c) There are three unpaired electrons in the
(c) Energy central atom which is due to transfer of odd
(d) Angular momentum electron of NO to Fe 2
(d) The complex is an octahedral low spin
40. A bubble of n mole of helium is submerged at a complex with Fe in + 2 oxidation state.
certain depth in water. The temperature of water
45. Ethylenediaminetetraacetic acid (EDTA) is the
increases by t°C. How much heat is added antidote for lead poisoning. It is administered in
approximately to helium during expansion? the form of
nc p (a) free acid
(a) ncv t (b)
t (b) sodium dihydrogen salt
(c) Calcium dihydrogen salt
n 2c v
(c) (d) ncp t (d) none of these
t
46. Which of the following coordination compounds
PART - II : CHEMISTRY would exhibit optical isomerism?
(a) pentamminenitrocobalt(III) iodide
41. When d8 arrangement in nickel ion changes from (b) diamminedichloroplatinum(II)
a weak octahedral field in [Ni (H2O)6 ]2 to a (c) trans-dicyanobis (ethylenediamine)
chromium (III) chloride
2
strong field in [Ni(CN)4 ] , which orbitals get (d) tris-(ethylendiamine) cobalt (III) bromide
lowered in energy ? Assume tetragonal distortion 47. When electrons are trapped into the crystal in
along the Z-axis. anion vacancy, the defect is known as :
(a) d (b) d (a) Schottky defect
x 2 y2 z2 (b) Frenkel defect
(c) d xy , d (d) d xz , d yz , d (c) Stoichiometric defect
z2 z2
(d) F-centres
www.jeeneetbooks.in

WWW.IIT-NEET.XYZ

Mock Test 6 MT-61

48. Copper metal has a face-centred cubic structure (c) Intercept of CH3COOH plot
with the unit cell length equal to 0.361 nm. (d) Kohlrausch’s law
Picturing copper ions in contact along the face 54. For the reaction,
diagonal. The apparent radius of a copper ion H
CH3COCH3 + I2 products
is-
The rate is governed by expression
(a) 0.128 (b) 1.42
(c) 3.22 (d) 4.22 dx
k[acetone][H ]
49. For a reaction to be spontaneous at all dt
temperatures The order w.r.t. I2 is:
(a) G and H should be negative (a) 1 (b) 0
(c) 3 (d) 2
(b) G and H should be positive
55. 1.08 g of pure silver was converted into silver
(c) G= S= 0
nitrate and its solution was taken in a beaker. It
(d) H < G and was electrolysed using platinum cathode and
50. The standard entropies of CO2 (g), C(s) and silver anode. 0.01 Faraday of electricity was
O2(g) are 213.5, 5.740 and 205 JK–1 respectively. passed using 0.15 volt above the decomposition
The standard entropy of formation of CO2 is: potential of silver. The silver content of the
(a) 2.76 JK–1 (b) 2.12 JK–1 beaker after the above shall be
(c) 1.12 JK –1 (d) 1.40 JK–1 (a) 0 g (b) 0.108 g
51. In a closed insulated container, a liquid is stirred (c) 0.108 g (d) 1.08 g
with a paddle to increase the temperature, which 56. In a fuel cell, CH3OH is used as fuel and oxygen
of the following is true? gas is used as an oxidiser. The reaction is
(a) E W 0, q 0 3
CH3OH( ) O 2 (g) CO2 (g) H 2O( )
2
(b) E W q 0 If standard enthalpy of combustion for methanol
(c) E 0, W q 0 is – 726 kJ/mol, what will be the cell efficiency.
Given that
(d) W 0, E q 0
G f values (kJ/mol) :
52. Le-Chatelier’s principle is applicable only to a :
CH3OH(l) = – 166.3 CO2(g) = – 394.4
(a) system in equilibrium
H2O (l) = – 237.1
(b) irreversible reaction (a) 70.23% (b) 96.7%
(c) homogeneous reaction (c) 80% (d) none of these
(d) heterogeneous reaction 57. Specific conductance of a N/10 KCl solution at
53. The plot shown below represents the variation 23°C is 0.0112 ohm–1 cm–1.The resistance of cell
of molar conductivity of NaOH and CH3OH containing the solution at the same temperature
was found to be 55 ohm. The cell constant will
NaOH be :
(a) 0.316 cm–1 (b) 0.616 cm–1
(c) 6.16 cm–1 (d) 616 cm–1
m 58. 1-Phenylethanol can be prepared by the reaction
CH3COOH
of benzaldehyde with
(a) ethyl iodide and magnesium
(b) methyl iodide and magnesium
(c) methyl bromide and aluminium bromide
c
(d) methyl bromide
0
m for CH3COOH is obtained from the : 59. When phenol is treated with excess bromine
water. It gives
1000 K (a) m-Bromophenol
(a) Relation m
c (b) o-and p-Bromophenols
(b) Intersection of NaOH and CH3 COOH (c) 2,4-Dibromophenol
curves (d) 2,4, 6-Tribomophenol.
EBD_7443
www.jeeneetbooks.in

WWW.IIT-NEET.XYZ

MT-62 Target VITEEE


60. Ether in air forms : 69. What is product of the following sequence of
(a) Oxide (b) Peroxide reactions
(c) Alkenes (d) Alkanes
NaBH 4 HBr
61. Which one of the following will most readily be O
CH 3OH
dehydrated in acidic conditions ?
OH ( i) Mg, Et 2O PCC
O OH
( ii ) H 2C O CH 2Cl 2
(a) (b) (iii ) H 3O

O O
(a) CHO (b) COOH
(c) (d)

OH OH CH 3
OH (c) (d)
62. Prestone is a mixture of: CHO OH
(a) Glycol + H2O (b) Glycerol + H2O 70. With Tollen’s reagent a silver mirror will be formed
(c) Acetone + H2O (d) propanal + H2O by:
63. Benzyl ethyl ether can be prepared by which (a) acetic acid (b) acetaldehyde
combination of components ? (c) acetyl chloride (d) acetone
(a) C6H5CH2Br + C2H5ONa
(b) C6H5CH2ONa + C2H5Br 71. Identify the respective final products in the
(c) both (a) and (b) following two reactions.
(d) C6H5CH2OH + C2H5OH Br2 / PCl 3 Br2
(i) CH3CH2COOH [A] [B]
64. Which is not true about acetophenone ?
H2O
(a) It reacts with 2,4-dinitrophenylhydrazine [C]
to form 2, 4-dinitrophenylhydrazone Cl 2 / PBr3 Cl 2
(b) It reacts with Tollen’s reagent to form silver (ii) CH3CH2COOH [D] [E]
mirror H2O
(c) It reacts with I2/NaOH to form iodoform [F]
(d) On oxidation with alkaline KMnO4 followed (a) CH3CHOHCOOH and CH3CHOHCOOH
by hydrolysis it gives benzoic acid (b) CH3CHOHCOCl and CH3CHOHCOBr
65. Which of the following does not turn Schiff’s
(c) CH3CHBrCOOH and CH3CHClCOOH
reagent to pink ?
(a) Formaldehyde (b) Benzaldehyde (d) CH3CHClCOOH and CH3CHBrCOOH.
(c) Acetone (d) Acetaldehyde
O
66. When acetic acid is heated with P 2 O5 the
compound formed will be :
(a) (CH3CO)2O (b) CH3COCH3 72. Product .
(c) CH3CHO (d) CH4
67. Acetone on distillation with conc. H2SO4 forms :
Here the product is
(a) phorone (b) acrolein
(c) mesitylene (d) mesityl oxide O O
68. The major product in the reaction of -bromo
cyclohexanone with potassium ethoxide is
(a) O (b) O
O
(a) (b)
OC2H2 COOH COOH
CO2C2H5 (c) (d)
(c) (d) O COOH
www.jeeneetbooks.in

WWW.IIT-NEET.XYZ

Mock Test 6 MT-63

73. What is the main product when 77. Identify (C) and (D) in the following series of
reactions
HOOC COOH is heated ?
excess of AgOH
COOH CH3 NH 2 [A] [B]
CH 3I

COOH heat
(a) [C] [D]
COOH
(a) (CH3)3COH, CH3NH2
(b) (CH3)2C = CH2, CH3NH2
(b) O
(c) (CH3)3N, CH3OH
(d) (CH3)2C=CH2, CH3OH

(c) 78. Consider the following sequence of reactions :


HOOC O
Reduction HNO 2
[A] [B] CH 3 CH 2 OH
O the compound [A] is
(d) C (a) CH3CN (b) CH3NC
C O
(c) CH3CH2CN (d) CH3NO2
74. Acetyl chloride does not react with 79. The bad smelling substance formed by the
action of alcoholic caustic potash on chloroform
(a) Water
and aniline is
(b) Sodium acetate
(a) nitrobenzene
(c) 2-methylpropene (b) phenyl isocyanide
(d) It reacts with all the three (c) phenyl cyanide
75. Formic acid is obtained when (d) phenyl isocyanate
(a) Calcium acetate is heated with conc. H2SO4 80. When aniline is treated with benzene diazonium
(b) Calcium formate is heated with calcium chloride at low temperature in weakly acidic
acetate medium, the final product obtained is

(c) Glycerol is heated with oxalic acid at 373 K NH2


(d) Acetaldehyde is oxidised with K2Cr2O7 and
H2SO4. (a) N N
76. Which statement is true regarding the following
structure?
(b) N N NH2
N
CH3 C3H7
C2H5
(c) N N NH
(a) It is a chiral molecule
(b) It exists in two resolvable optically active NH2
forms
(c) Both (a) and (b) (d)
N N
(d) Neither a) nor b)
EBD_7443
www.jeeneetbooks.in

WWW.IIT-NEET.XYZ

MT-64 Target VITEEE


PART - III : MATHEMATICS 86. The region represented by the inequalities
x 6, y 2, 2x + y 10, x 0, y 0 is
81. For the set A = {1, 2, 3}, define a relation R in the (a) unbounded (b) a polygon
set A as follows (c) exterior of a triangle
R = {(1, 1), (2, 2), (3, 3), (1, 3)}
(d) None of these
Then, the ordered pair to be added to R to make
it the smallest equivalence relation is 2 2 2
87. 1+ + + + ............=
(a) (1, 3) (b) (3, 1) (c) (2, 1) (d) (1, 2) 1.2.3 3.4.5 5.6.7
82. The matrices (a) 2 log e 4 (b) 2 log e 2
u1 v1 w1 2 2 1 (c) 2 log e 3 (d) None
1
P= u2 v2 w2 ; Q = 13 –5 m 88. A variable plane moves so that the sum of
9 reciprocals of its intercepts on the three
u3 v3 w3 –8 1 5
coordinate axes is constant . It passes through
are such that PQ = I, an identity matrix. Solving the a fixed point, which has coordinates
u1 v1 w1 x 1 1 1 1
(a) ( , , ) (b) , ,
equation u2 v2 w2 y = 1 , the
u3 v3 w3 z 5
1 1 1
value of y comes out to be –3. Then the value of m is (c) , , (d) , ,
equal to
(a) 27 (b) 7 89. The perpendicular distance of A(1, 4, –2) from
(c) – 27 (d) – 7 BC, wher e coordinates of B and C ar e
83. Let 1 , 2 and 1 , 2 be the roots of respectively (2, 1, –2) and (0, –5, 1) is
ax2 + bx + c = 0 and px2 + qx + r = 0 respectively.
3 26
If the system of equations 1y + 2z = 0 and 1y (a) (b)
+ 2z = 0 has a non-trivial solution, then 7 7

b2 ac c2 ab 3 26
(a) 2 (b) 2
(c) (d) 26
q pr r pq 7
152
a2 bc 90. The value of cot cosec tan 1 is
(c) (d) None of these 3 3
p2 qr
5 6 3 4
(a) (b) (c) (d)
84. Let function f :R R be defined by 17 17 17 17
91. If cos–1x + cos–1y + cos–1z = 3 then xy + yz + zx is
f (x) = 2x + sin x for x R , then f is
equal to :
(a) one-one and onto
(a) 1 (b) 0
(b) one-one but NOT onto
(c) onto but NOT one-one (c) –3 (d) 3
(d) neither one-one nor onto 92. Let f : R R be a function such that
85. Eighteen guests are to be seated, half on each
side of a long table. Four particular guests x y f ( x) f ( y)
f , f (0) 0 and
desire to sit on one particular side and three 3 3
others on the other side of the table. The
f ' (0) 3 . Then
number of ways in which the seating
arrangement can be done equals (a) f(x) is a quadratic function
(a) 11C4 (9!)2 (b) 11C6 (9!)2 (b) f(x) is continuous but not differentiable
6 5 (c) f(x) is differentiable in R
(c) P0 × P0 (d) None of these
(d) f(x) is bounded in R
www.jeeneetbooks.in

WWW.IIT-NEET.XYZ

Mock Test 6 MT-65

3 100. The point (2a, a) lies inside the region bounded


93. If x + iy = then 4x – x2 – y2 by the parabola x2 = 4y and its latus rectum.
cos i sin 2 Then,
reduces to (a) 0 a 1 (b) 0 < a < 1
(a) 2 (b) 3 (c) a > 1 (d) a < 0
(c) 4 (d) 5 101. If the normal at the end of a latus rectum of an
94. The locus of the centre of a circle which touches ellipse passes through one extermity of the minor
the circle | z – z1 | = a and | z – z2 | = b externally axis, then
(z, z1 & z2 are complex numbers) will be
(a) an ellipse (b) a hyperbola (a) e4 e2 1 0 (b) e4 e2 1 0
(c) a circle (d) none of these (c) e 4 e 2 1 0 (d) None of these
95. In a parallelogram ABCD, | AB | a,| AD | b 102. The asymptotes of a hyperbola, having centre
at the point (1, –1), are parallel to the lines
and | AC | c , the value of DB . AB is x + 2y – 3 = 0 and 2x + y + 5 = 0. If the hyperbola
passes through the point (2, 1), then its equation
3a 2 b2 c 2 a 2 3b 2 c 2 is
(a) (b)
2 2 (a) 2x 2 2 y 2 5xy x y 21 0
2 2 2 2 2 2
a b 3c a 3b c
(c) (d) (b) 2x 2 2 y 2 5xy x y 21 0
2 2
96. Solve for x : (c) 2x 2 2 y 2 5xy x y 21 0
51 (d) None of these
{ x cos(cot–1x) + sin(cot–1 x)}2 = ,
50
103. If a , b , c are vectors such that [ a b c ] = 4,
1 1
(a) (b)
2 5 2 then [ a b b c c a ]
(c) 2 2 (d) 5 2 (a) 16 (b) 64
97. The number of solution of the equation (c) 4 (d) 8
1 104. If f (x) is differentiable and strictly increasing
| cot x | cot x (0 x 2 ) is :
sin x f (x 2 ) f ( x)
(a) 0 (b) 1 function, then the value of lim is
x 0 f ( x ) f ( 0)
(c) 2 (d) 3
98. If a group contains only four elements, then it is (a) 1 (b) 0
(a) non-abelian (b) abelian (c) –1 (d) 2
(c) cyclic (d) none of these 105. The line y = mx bisects the area enclosed by
99. If S1, S2, S3, ...., Sn are the sum of infinite geometric lines x =0, y = 0 and x = 3/2 and the curve
series whose first terms are 1, 2, 3, ..., n and whose y = 1 + 4x – x2. Then the value of m is
1 1 1 1 13 13
, , ,..., (a) (b)
comon ratios are 6 2
2 3 4 (n 1)
respectively, then the value of 13 13
(c) (d)
S12 S 22 S32 ... S22 n is equal to 5 7
1
106. If angle is divided into two parts such that
1 the tangent of one part is K times the tangent
(a) [ n ( 2 n 1)( 4 n 1) 3] to other and is their difference, then sin is
3
equal to
1
(b) [n (2n 1)(4n 1) 3] K 1 K 1
3 (a) sin (b) sin
K –1 2 K –1 2
1
(c) [ n ( 2 n 1)( 4n 1) 3] K 1 K –1
3 (c) sin (d) sin
(d) None of these K –1 K 1
EBD_7443
www.jeeneetbooks.in

WWW.IIT-NEET.XYZ

MT-66 Target VITEEE


107. The expansion of cos x by Maclaurin’s series is
112. Evaluate : x 3e 2x dx
2 4 6
x x x
(a) cos x 1 ..... [Use Bernoulli's formula]
2! 4! 6!
1 2x 4 3 2
xn (a) e x x
( 1)n / 2 . ..... where n is even 2 3 4
n!

x 2 x4 x6 xn e 2x x
3 2
x
(b) cos x 1 ..... ..... (b)
2! 4! 6! n! 2
where n is even
1 2x 3 3 2 3x 3
x x3 x5 (c) e x x
(c) cos x 1 ..... 2 2 2 4
1! 3! 5!
2 2
x 2n 1 (d) e 2x x 4 x3 x
( 1)2n 1
..... 3
(2n 1)!
113. The solution of differential equation
x x 2 x3 xn
(d) cos x 1 ..... dy
1! 2! 3! n! = cos (x + y) is –
108. A particle is moving along a straight line path dx
according to the relation x y
s2 = at2 + 2bt + c (a) tan x C
s represents the distance travelled in t seconds 2
and a, b, c are constants. Then the acceleration
of the particle varies as x y
(b) cosec x C
(a) s – 3 (b) s 3/2 2
(c) s –2/3 (d) s 2
x y
2 (c) cot x C
2 2 2
109. With the usual notation x x dx is
1 (d) None of these
equal to-
dy
(a) 4 + 2 – 3 (b) 4 – 2 + 3 114. The solution of x 3 4 x 2 tan y e x sec y
dx
(c) 4 – 2 – 3 (d) None of these satisfying y (1) = 0 is :
110. Let and be the distinct roots of
ax2 + bx = c = 0, then (a) tan y (x 2) e x log x

1 cos(ax 2 bx c) (b) sin y e x ( x 1) x 4


lim is equal to
x (x )2
(c) tan y ( x 1)e x x 3

a2
(a) ( )2 (b) 0 x
(d) sin y e (x 1) x
3
2
115. G is a set of all rational numbers except –1 and *
a2 1 is defined by a* b = a + b + ab for all a, b G .
(c) ( )2 (d) ( )2
2 2
In the group (G, *) the solution of
111. If f (x) = (x + 1)cot x be continuous at x = 0 then
f (0) is equal to: 2 1 * x *3 1
5 is
(a) 0 (b) – e
(a) 71 (b) 68
(c) e (d) None
(c) 63/5 (d) 72/5
www.jeeneetbooks.in

WWW.IIT-NEET.XYZ

Mock Test 6 MT-67

116. Which of the following is a contradiction? conservation efforts, soil erosion has accelerated due
(a) (p q) ~ (p q) to new demands placed on the land by heavy crop
production. In the years ahead, soil erosion and the
(b) p ( p q) pollution problems it causes are likely to replace
(c) (p q) p petroleum scarcity as the nation's most critical natural
(d) None of these resource problem.
117. If P = n(n 2 – 12) (n2 – 22) (n2 – 32) ...... (n2 – r2),
121. As we understand from the reading, today, soil
n > r, n N then P is necessarily divisible by erosion in America-
(a) (2r + 2) ! (b) (2r + 4) !
(a) causes humans to place new demands on
(c) (2r + 1) ! (d) None of these
the land
118. For a biased dice, the probability for the different
(b) is worse than it was in the nineteenth
faces to turn up are
century
Face 1 2 3 4 5 6 (c) happens so slowly that it is hardly noticed
P 0.10 0.32 0.21 0.15 0.05 0.17 (d) is the most critical problem that the nation
faces
The dice is tossed and it is told that either the 122. The author points out in the passage that erosion
face 1 or face 2 has shown up, then the probability in America-
that it is face 1, is (a) has damaged 282 million acres ever since
16 1 5 5 settlers first put the prairies and grasslands
(a) (b) (c) (d) under the plow
21 10 16 21
(b) has been so severe that it has forced people
119. The domain of the function
to abandon their settlements
3 (c) occurs only in areas with no vegetation
f(x) = + log10 (x3 – x), is
4 – x2 (d) can become a more serious problem in the
(a) (– 1, 0) (1, 2) future
(b) (1, 2) (2, ) 123. It is pointed out in the reading that in America-
(c) (– 1, 0) (1, 2) (2, ) (b) petroleum is causing heavy soil erosion and
(d) (1, 2) pollution problems
120. If E and F are the complementary events of (b) heavy crop production is necessary to meet
events E and F respectively and if 0 < P(F) < 1, the demands and to prevent a disaster
then (c) soil erosion has been hastened due to the
overuse of farming lands
(a) P(E \ F) + P( E \ F) = 1 (d) water is undoubtedly the largest cause of
erosion
(b) P(E \ F) + P(E \ F ) = 1
124. Realists in India can't object to a good neighborly
(c) P( E \ F) + P(E \ F ) = 1 relationship between Nepal and China
_________in Beijing know the dangers of
1 moving too far and too fast and __________an
(d) P(E \ F ) + P( E \ F ) =
2 Indian reaction.
PART - IV : ENGLISH (a) Pragmatists, provoking
(b) escapist, delighting
Direction (Qs. 121 -123) : Read the passage carefully
and answer the questions given below (c) evader, make happy
(d) idealist, rude
Erosion of America's farmland by wind and water has 125. Choose the best pronunciation of the word,
been a problem since settlers first put the prairies and Bowl, from the following options.
grasslands under the plow in the nineteenth century.
(a) bohl (b) ba-owl
By the 1930s, more than 282 million acres of farmland
were damaged by erosion. After 40 years of (c) bo-ul (d) ba-oul
EBD_7443
www.jeeneetbooks.in

WWW.IIT-NEET.XYZ

MOCK
VITEEE Mock Test Paper
7
Max. Marks : 125 Time : 2½ hrs
PART - I : PHYSICS 5. An electric dipole consists of two opposite
charges each 0.05 C separated by 30 mm. The
1. The force ‘F’ acting on a particle of mass ‘m’ is dipole is placed in an uniform external electric
field of 106 NC–l. The maximum torque exerted by
indicated by the force-time graph shown below.
the field on the dipole is
The change in momentum of the particle over (a) 6 × l0–3 N-m (b) 3 × 10–3 N- m
the time interval from zero to 8 s is : (c) 15 × l0–3 N- m (d) 1.5 × l0–3 N- m
6. A particle A has a charge q and particle B has
6 charge + 4q with each of them having the mass
3 m. When they are allowed to fall from rest through
same potential difference, the ratio of their speeds
0 VA : VB will be :
F(N)

2 4 6 8 (a) 4 : 1 (b) 1 : 4
–3
(c) 1 : 2 (d) 2 : 1
t(s) 7. The inward and outward electric flux from a
closed surface are respectively 8 × 103 and
4 × 103 units. Then the net charge inside the
(a) 24 Ns (b) 20 Ns
closed surface is :
(c) 12 Ns (d) 6 Ns (a) – 4 × 103 coulomb
2. An air filled parallel plate condenser has a (b) 4 × 103 coulomb
capacity of 2 pF. The separation of the plates is
4 103
doubled and the interspace between the plates (c) coulomb
is filled with wax. If the capacity is increased to 0
6pF, the dielectric constant of wax is : (d) – 4 × 103 0 coulomb
(a) 2 (b) 3 8. The adjacent graph shows the extension ( l) of
(c) 4 (d) 6 a wire of length 1m suspended from the top of a
3. Four metal conductors having different shapes : roof at one end with a load W connected to the
1. a sphere other end. if the corss-sectional area of the wire
2. cylinder is 10–6m2, calculate the Young’s modulus of the
3. pear material of the wire
4. lightning conductor
are mounted on insulating stands and charged.
The one which is best suited to retain the charges
for a longer time is : 4
(l ×10 )m

(a) 1 (b) 2 3
–4

(c) 3 (d) 4 2
1
4. After terminal velocity is reached, the
acceleration of a body falling through a fluid is
(a) equal to g (b) zero 20 40 60 80 W(N)
(c) less than g (d) greater than g
www.jeeneetbooks.in

WWW.IIT-NEET.XYZ

Mock Test 7 MT-69

(a) 2 × 1011 N/m2 (b) 2 × 10–11 N/m2 two turns and carries the same current, the new
(c) 2 × 10 N/m–12 2 (d) 2 × 10–13 N/m2 value of magnetic induction at the centre is :
9. In a potentiometer experiment the balancing with (a) 3 B (b) 5 B
a cell is at length 240 cm. On shunting the cell (c) 4 B (d) 2 B
with a resistance of 2 , the balancing length
14. How many calories of heat will be produced
becomes 120 cm. The internal resistance of the
cell is approximately in a 210 watt electric bulb in
(a) 0.5 (b) 1 5 minutes?
(c) 2 (d) 4 (a) 80000 cal (b) 63000 cal
10. A material 'B' has twice the specific resistance of (c) 1050 cal (d) 15000 cal
'A'. A circular wire made of 'B' has twice the 15. As the temperature of a liquid is raised, the
diameter of a wire made of 'A'. then for the two coefficient of viscosity
wires to have the same resistance, the ratio lB/lA (a) decreases (b) increases
of their respective lengths must be
(c) remains the same
1 (d) may increase or decrease depending on the
(a) 1 (b)
2 nature of liquid
1 16. On a linear temperature scale Y, water freezes at –
(c) (d) 2 160° Y and boils at – 50° Y. On this Y scale, a
4
temperature of 340 K would be read as : (water
11. A current of 6 A enters one corner P of an
freezes at 273 K and boils at 373 K)
equilateral triangle PQR having 3 wires of
resistances 2 each and leaves by the comer R. (a) – 73.7° Y (b) – 233.7° Y
Then the current I1 and I2 are (c) – 86.3° Y (d) – 106.3° Y
6A 17. The upper half of an inclined plane of inclination
is perfectly smooth while lower half is rough. A
block starting from rest at the top of the plane
P will again come to rest at the bottom, if the coef-
I1 I2
ficient of friction between the block and lower
2 2 half of the plane is given by
2
Q R (a) = (b) = 2 tan
2 tan
(a) 2 A, 4 A (b) 4A, 2A
1
(c) 1 A, 2A (d) 2A, 3A (c) = tan (d) =
tan
12. The electron drift speed is small and the
charge of the electron is also small but still, we 18. The tank circuit used in a radio transmitter should
obtain large current in a conductor. This is due have
to the (a) high effective Q
(a) conducting property of the conductor (b) low effective Q
(b) resistance of the conductor is small (c) loosely coupled load
(c) electron number density of the conductor (d) Both (a) and (c)
is small 19. A short-circuited coil is placed in a time-varying
(d) electron number density of the conductor magnetic field. Electrical power is dissipated due
is enormous to the current induced in the coil. If the number
13. A wire in the form of a circular loop of one turn of turns were to be quadrupled and the wire
carrying a current produces a magnetic field B at radius halved, the electrical power dissipated
the centre. If the same wire is looped into a coil of would be
EBD_7443
www.jeeneetbooks.in

WWW.IIT-NEET.XYZ

MT-70 Target VITEEE


(a) halved (b) the same × × × × ×
× × × × ×
(c) doubled (d) quadrupled × × I × ×
I
20. A coil of wire having inductance and resistance
has a conducting ring placed coaxially within it.
I D
Thecoil is connected to abattery at timet = 0, so
that a time-dependent current l1(t) starts flowing (1) (2)
through the coil. If I2(t) is the current induced in × ×
× × ×
the ring, and B(t) is the magnetic field at the axis × ×
× × ×
of the coil due to I1(t), then as a function of time D × ×
I × ×
(t > 0), the product I2(t) B(t) × × × ×
× ×
× × ×
(a) increases with time I × D × ×
×
(b) decreases with time × ×
× × ×
(3) (4)
(c) does not vary with time (a) i1 > i2 > i3 > i4 (b) i1 = > i2 > i3 > i4
(d) passes through a maximum (c) i1 > i2 > i3 = i4 (d) i1 = i2 > i3 = i4
21. In a LCR circuit capacitance is changed from C 25. A conducting circular loop is placed in a uniform
to 2 C. For the resonant frequency to remain magnetic field of 0.02 tesla with its plane
unchanged, the inductance should be changed perpendicular to the magnetic field. If the radius
from L to of the loop starts striking at a constant rate of 1
mm/s. Then the emf induced in the loop at the
(a) L/2 (b) 2 L instant when the radius is 4 cm will be:
(c) 4 L (d) L/4 (a) 8 V (b) 5.0 V
22. One conducting U tube can slide inside another (c) 2.0 V (d) 2.5 V
as shown in figure, maintaining electrical contacts 26. Two inductors 0.4 H and 0.6 H are connected in
between the tubes. The magnetic field B is parallel. If this combination is connected in series
perpendicular to the plane of the figure . If each with an inductor of inductance 0.76 H. The
tube moves towards the other at a constant equivalent inductance of the circuit will be:
speed v, then the emf induced in the circuit in (a) 0.1 H (b) 0.2 H
(c) 1 H (d) 2 H
terms of B, l and v where l is the width of each
27. Which one of the following is not true ?
tube, will be
(a) Ampere's law is : B.d µ0iin
X A X X X
BX
d
X v (b) Faraday's law is : C 0
X v X dt
X X X
X
X µ0 i r
X X C (c) Biot-Sawart law is dB
4
.
3
r
(a) – Blv (b) Blv
(d) Gauss's law is 0 E .d A qin
(c) 2 Blv (d) zero
23. If the number of turns in a coil becomes doubled 28. Two slits separated by a distance of 1 mm are
then its self inductances will be: illuminated with red light of wavelength
6.5 × 10–7 m. The interference fringes are
(a) unchanged (b) four times observed on a screen placed 1 m from the silts.
(c) halved (d) doubled The distance of the third dark fringe from the
24. Four identical circular conducting loops are central fringe will be equal to :
placed in uniform magnetic fields that are either (a) 0.65 mm (b) 1.30 mm
increasing (I) or decreasing (D) in magnitude at (c) 1.62 mm (d) 1.95 mm
identical rates. Arrange the magnitude of the
currents induced (i) in the loops.
www.jeeneetbooks.in

WWW.IIT-NEET.XYZ

Mock Test 7 MT-71

29. In an interference pattern produced by the If the average power radiated by the star is 10 16
identical coherent sources of monochromatic watt, the deuteron supply of the star is exhausted
light, the intensity at the site of central maxima is in the time of the order of the mass of the nuclei
follows:
I. The intensity at the same spot when either of
M(1H2) = 2.014 amu, M(p) = 1.007 amu,
the two slits is closed is I0 then we must have
M(n) = 1.008 amu, M(He4) = 4.001 amu
(a) I and I0 are not related (a) 1016 sec (b) 1012 sec
(b) I = 4I0 (c) 106 sec (d) 108 sec
(c) I = I0 35. Radiation from a hydrogen discharge tube is
(d) I = 2I0 incident on the cathode of a photocell. The work
30. Energy E and frequency v of a photon is correctly function of the cathode surface is 3.2 eV. To
represented by: reduce the photocurrent to zero, the voltage (in
E E volts) of the anode relative to the cathode must
be made :
(a) – 3.2 (b) + 3.2
(a) (c) – 10.4 (d) – 13.6
(b)
36. The circuit has two oppositively connected ideal
diodes in parallel. What is the current flowing in
the circuit?
E E
4

D1 D2
(c) (d)
12V
3 2
31. A freshly prepared radioactive source of half life
2 hours emits radiation of intensity of which is
64 times the permissible safe level. The minimum (a) 1.71 A (b) 2.00 A
time after which is would be possible to work
(c) 2.31 A (d) 1.33 A
safely with this source is:
(a) 64 hrs (b) 24 hrs 37. If the lattice constant of this semiconductor is
(c) 6 hrs (d) 12 hrs decreased, then which of the following is correct?
32. An -particle of 10 MeV collides head on with conduction
Ec
band width
copper nucleus (Z = 29) and is deflected back.
What is the maximum limit of radius of copper band gap Eg
nucleus? valence
(a) 10.4 × 10–15 m (b) 6.4 × 10–15 m band width Ev
(c) 4.2 × 10–15 m (d) 8.4 × 10–15 m (a) All Ec, Eg, Ev increase
33. Ultra violet light of wavelength 300 nm and (b) Ec and Ev increase, but Eg decreases
intensity 1.0 W/m2 falls on the surface of a
(c) Ec and Ev decrease, but Eg increases
photoelectric material. If one percent of incident
photons produce photoelectrons, then the (d) All Ec, Eg, Ev decrease
number of photoelectron emitted from an area of 38. The logic circuit shown below has the input
1.0 cm2 of the surface is nearly: waveforms ‘A’ and ‘B’ as shown. Pick out the
(a) 2.13 × 1011/sec (b) 1.52 × 1012/sec correct output waveform.
12
(c) 3.02 × 10 /sec (d) None of these
34. A star initially has 1040 deuterons. It produces
A
energy via the process
2 2 3 Y
1H + 1H 1H + p B
2 3 4
1H + 1H 2He + n
EBD_7443
www.jeeneetbooks.in

WWW.IIT-NEET.XYZ

MT-72 Target VITEEE

Input A PART - II : CHEMISTRY


41. Which of the following is diamagnetic ?
Input B (a) d 5 arrangement in Fe3 is strong
octahedral field.
Output is (b) d7 arrangement in Co 2 is weak octahedral
field.
(a)
(c) d 5 arrangement in Mn 2 is weak
octahedral field.
(b)
(d) d10 arrangement of Ni is tetrahedral
Ni(CO)4.
(c) 42. The covalent character of the following chlorides
follow the order
(d) (a) HgCl2 < CdCl2 < ZnCl2
(b) ZnCl2 < CdCl2 < HgCl2
39. In the circuit below, A and B represent two inputs (c) CdCl2 < ZnCl2 < HgCl2
and C represents the output. (d) HgCl2 < ZnCl2 < CdCl2
A 43. Which particle contains 2 neutrons and
1 proton ?
C (a) 1H2 (b) 2He4
(c) 1T 3 (d) 1D2
B 44. An aqueous solution of [Ti(H2O)6]3+ ion has a
mild violet colour of low intensity. Which of the
following statements is incorrect ?
The circuit represents (a) The ion absorbs visible light in the region
(a) NOR gate (b) AND gate of ~ 5000 Å.
(c) NAND gate (d) OR gate (b) The colour results from an electronic
40. A conductor and a semi-conductor are connected transition of one electron from the t2g to an
in parallel as shown in the figure. At a certain eg orbital ?
voltage both ammeters registers the same current. (c) The low colour-intensity is because of a
If the voltage of the DC source is increased then low probability of transition.
Conductor (d) The transition is the result of metal-ligand
A1 back bonding.
45. The IUPAC name of the red coloured complex
Semiconductor
A2 [Fe(C 4 H 7 O 2 N 2 ) 2 ] obtained from the reaction
of Fe 2 and dimethyl glyoxime
(a) bis (dimethyl oxime) ferrate (II)
(a) the ammeter connected to the (b) bis (dimethyl oxime) iron (II)
semiconductor will register higher current (c) bis (2, 3- butanediol dioximato) iron (II)
than the ammeter connected to the (d) bis (2, 3- butanedione dioximato) iron (II)
conductor 46. [Co(NH3)4 (NO2)2] Cl exhibits
(b) the ammeter connected to the conductor (a) linkage isomerism, ionization isomerism and
will register higher current than the ammeter geometrical isomerism
connected to the semiconductor (b) ionization isomerism, geometrical isomerism
(c) the ammeters connected to both and optical isomerism
semiconductor and conductor will register (c) linkage isomerism, geometrical isomerism
the same current and optical isomerism
(d) the ammeter connected to both (d) linkage isomerism, ionization isomerism and
semiconductor and conductor will register
optical isomerism
no change in the current
www.jeeneetbooks.in

WWW.IIT-NEET.XYZ

Mock Test 7 MT-73

47. The appearance of colour in solid alkali metal 53. The order of a reaction, with respect to one of
halides is generally due to the reacting component Y, is zero. It implies that:
(a) Schottky defect (a) the reaction is going on at a constant rate
(b) Frenkel defect (b) the rate of reaction does not vary with
(c) Interstitial positions temperature
(d) F-centres (c) the reaction rate is independent of the
48. The second order Bragg’s diffraction of X-rays concentration of Y
of wavelength 100 pm from a set of parallel lattice (d) the rate of formation of the activated
planes in a metal occurs at a grazing angle of complex is zero
30º. The spacing between the successive 54. The specific rate constant of a first order depends
scattering planes in the cystal is on the :
(a) 50 pm (b) 100 pm (a) concentration of the product
(c) 150 pm (d) 200 pm (b) concentration of the reactant
49. The decomposition of limestone (c) temperature
CaCO3 (s) CaO(s) + CO 2 (g) (d) all of these
55. The resistance of 1 N solution of acetic acid is
is non-spontaneous at 298 K. The H and S 250 ohm, when measured in a cell of cell constant
value for the reaction are 176.0 kJ and 160 JK–1 1.15 cm –1 . The equivalent conductance
respectively. At what temperature the (in ohm–1 cm2 equiv–1) of 1 N acetic acid will be
decomposition becomes spontaneous? (a) 4.6 (b) 9.2
(a) At 1000 K (c) 18.4 (d) 0.023
(b) below 500 C 56. Zn amalgam is prepared by electrolysis of
(c) Between 500 C and 600 C aqueous ZnCl2 using Hg cathode (9gm). How
much current is to be passed through ZnCl2
(d) above 827 C solution for 1000 seconds to prepare a Zn
50. Calculate the entropy change for Amalgam with 25% Zn by wt. (Zn = 65.4)
CH4(g) + H2O(g) 3H2(g) + CO(g), using (a) 5.6 amp (b) 7.2 amp
(c) 8.85 amp (d) 11.2 amp
the following data :
57. Rusting of iron is prevented by zinc coating over
Substance CH4(g) H2O(g) H2(g) CO(g)
its surface. This process is called :
S°/JK–1 186.2 188.7 130.6 197.6
(a) Electrolysis
mole–1
(b) Galvanization
The entropy change is:
(a) – 46 JK–1 mole–1 (c) Photoelectrolysis
(b) + 46 JK–1 mole–1 (d) Cathodic protection
(c) – 214.5 JK–1 mole–1 58. Ethanol and dimethyl ether form a pair of
(d) + 214.5 JK–1 mole–1 functional isomers. The boiling point of ethanol
51. According to the third law of thermodynamics is higher than that of dimethyl ether, due to the
which one of the following quantities for a presence of
perfectly crystalline solid is zero at absolute (a) H-bonding in ethanol
zero? (b) H-bonding in dimethyl ether
(a) Free energy (b) Entropy (c) CH3 group in ethanol
(c) Enthalpy (d) Internal energy (d) CH3 group in dimethyl ether
52. Which reaction is not affected by change in 59. Reaction of CH — CH with RMgX leads to
2 2
pressure?
O
(a) H 2 (g) + I 2 (g) 2HI(g) formation of
(b) N 2 (g) + 3H 2 (g) 2NH3 (g) (a) RCHOHR (b) RCHOHCH3
R
(c) 2C(s) + O 2 (g) 2CO(g)
(c) RCH2CH2OH (d) CHCH2OH
(d) SO 2 (g) + 2O 2 (g) 2SO3 (g) R
EBD_7443
www.jeeneetbooks.in

WWW.IIT-NEET.XYZ

MT-74 Target VITEEE


60. Acetylene, on reacting with formaldehyde under 67. Which of the following is formed, when calcium
high pressure, gives acetate is heated ?
(a) propanol (b) propyl alcohol (a) CH3CHO (b) CH3COCH3
(c) allyl aldehyde (d) butyndiol. (c) CH3CH2OH (d) CH3CHO
61. In the following reaction : 68. The reaction of an organic compound with
Conc. H 2SO 4
ammonia followed by nitration of the product
C2H5OH Z. gives a powerful explosive, called RDX. The
443 K
organic compound is:
Identify Z : (a) phenol (b) toluene
(a) CH2 = CH2 (c) glycerine (d) formaldehyde
(b) CH3 – CH2 – O – CH2 69. Which carbonyl compound will be purified with
| sodium bisulphite-
CH3
(c) CH3 – CH2 – C (a) 2-Pentanone (b) 3-Hexanone
| (c) 4-Heptanone (d) All of these
SO4 70. Which of the following will not give HVZ
(d) (CH3CH2)2 – SO4
reaction?
62. The most suitable reagent for the conversion of
(a) Trichloroethanoic acid
RCH2OH RCHO is :
(a) KMnO4 (b) 2-methyl butanoic acid
(b) K2Cr2O7 (c) Ethanoic acid
(c) CrO3 (d) Propanoic acid
(d) PCC (pyridine chloro chromate) 71. When propionic acid is treated with aqueous
63. Arrange the following in decreasing order of NaHCO3, CO2 is liberated. The C of CO2 comes
solubility in water from
(a) NaHCO3 (b) – CH3 group
(c) – COOH group (d) – CH2 group
O O O 72. Both propanoic acid, CH3CH2COOH as well as
I II III 2-chloropropanoic acid give precipitate with aq.
(a) I > III > II AgNO3. How you distinguish the precepitates
(b) III > II > I in the two ?
(c) II > III > I (a) By aq. NH4OH
(d) All are equally soluble (b) By aq. HNO3
64. Which of the following reaction can produce (c) By either of the two
(d) By none of the two.
R – CO – Ar ?
(a) AlCl3 O
ArCOCl H Ar
(b) RCOCl ArMgX 73. CH3CH2CH2COOH + NCl
HCl
Product is

(c) ArCOCl RMgX


O
AlCl3
(d) RCOCl H Ar
65. The compound which reacts with Fehling Cl
|
solution is (a) CH 3CH 2 C HCOOH
(a) C6H5COOH (b) HCOOH
(c) C6H5CHO (d) CH2ClCH3 (b) ClCH2CH2CH2COOH
66. Among the following which one does not act as
Cl
an intermediate in hofmann rearrangement? |
(a) RNCO (b) RCON (c) CH 3 C HCH 2 COOH
(c) RCONHBr (d) RNC (d) All the three
www.jeeneetbooks.in

WWW.IIT-NEET.XYZ

Mock Test 7 MT-75

74. Which of the following is correct ?


O
(a) heat CH3
(COOH) 2 HCOOH CO 2
CH 2 N
heat CH 3
(b) (COOH) 2 CO 2 CO H 2 O 77. In the reaction

heat
(c) HOOCCH CHCOOH
(A) , product (A) is :
CH 2 CHCOOH CO2
CH2
COOH (a) (b) RCONH2
O O
(d) heat

(c) CH3CH2NH2 (d)


75. Introduction of a methyl group in ammonia
markedly increases the basic strength of ammonia 78. In the reaction sequence
in aq. solution, introduction of the second
methyl group increases only marginally the basic NH2
strength of methyl amine in water. This is due to NaNO 2 ,HCl CuCN
(a) different type of hybridisation in the two A B
0 C
amines
(b) protonated dimethyl amine is more solvated LiAlH 4
than methyl amine C, the product ‘C’ is:
(c) protonated dimethyl amine is more solvated (a) benzonitrile (b) benzaldehyde
than the protonated methyl amine (c) benzoic acid (d) benzylamine
(d) protonated dimethyl amine is less stable 79. The end product (Z) of the following reaction is
than the protonated methyl amine Cu/KCN (X)
N 2Cl

H /H2 O NaOH
76.
(i ) CHN 2COOC 2 H 5
[Y] (Y) CaO, (Z)
(ii ) Heat with Cu powder (a) A cyanide (b) A carboxylic acid
(c) An amine (d) An arene
oxidation
[Z] 80. Consider the following ions
Compound Z should be +
(I) Me2N N N
O +
(a) (II) O 2N N N
COOH
+
HOOC (III) H3C O N N
(b) COOH
+
HOOC (IV) H3C N N
HOOC The reactivities of these ions in azo-coupling
(c) COOH reactions (under similar conditions) will be such
HOOC that
(a) (I) < (IV) < (II) < (III)
COOH (b) (I) < (III) < (IV) < (II)
(d) (c) (III) < (I) < (II) < (IV)
COOH (d) (III) < (I) < (IV) < (II)
EBD_7443
www.jeeneetbooks.in

WWW.IIT-NEET.XYZ

MT-76 Target VITEEE


PART - III : MATHEMATICS 88. a , b and c are perpendicular to b c , c a
1 2 5
an d a b respectively an d if a b 6,
81. The rank of the matrix 2 4 a 4 is
1 2 a 1 b c 8 and c a 10 , then a b c is
(a) 1 if a = 6 (b) 2 if a = 1 equal to
(c) 3 if a = 2 (d) none of these
(a) 5 2 (b) 50
x x , if 0 x 2
82. The function f x is (c) 10 2 (d) 10
x 1 x, if 2 x 3
89. The acute angle that the vector 2î 2ˆj k̂
(a) differentiable at x = 2
(b) not differentiable at x = 2 makes with the plane contained by the two
(c) continuous at x = 2 vectors 2î 3ˆj k̂ and î ĵ 2k̂ is given by
(d) None of these
83. Suppose p, q, r 0 and system of equation 1 1
1 1
(p + a)x + by + cz = 0, (a) cos (b) sin
3 5
ax (q b) y cz 0 ,
(c) 1 (d) 1
ax by (r c)z 0 has a non-trivial solution, tan 2 cot ( 2)
a b c x 1 y 1 z 3
then value of p q 90. Given the line L : and the
r is 3 2 1
(a) –1 (b) 0 (c) 1 (d) 2 plane : x 2y z . Of the following
1 cos{2( x 2)} assertions, the only one that is always true is
84. lim is equal to (a) L is to (b) L lies in
x 2 x 2
(c) L is paralel to (d) None of these
(a) equals (b) equals – 91. If f (x) = sin x + cos x, g (x) = x2 – 1, then g (f (x))
2 2
is invertible in the domain
1
(c) equals (d) does not exist 0,
2 (a) (b) ,
2 4 4
a a
85. If x, a R, where x , then
3 3 (c) , (d) [0, ]
2 2
3a 2 x – x 3 92. Shamli wants to invest `50,000 in saving
tan – 1 is equal to
a 3 – 3ax 2 certificates and PPE. She wants to invest atleast
`15,000 in saving certificates and at least
x x `20,000 in PPF. The rate of interest on saving
(a) 3 tan – 1 (b) –3 tan – 1 certificates is 8% p.a. and that on PPF is 9% p.a.
a a
Formulation of the above problem as LPP to
–1 a determine maximum yearly income, is
(c) 3 tan (d) None of these (a) Maximize Z = 0.08x + 0.09y
x
Subject to, x + y 50,000, x 15000, y 20,000
x 5 (b) Maximize Z = 0.08x + 0.09y
86. If sin 1 cosec 1 , then the value Subject to, x + y 50,000, x 15000,
5 4 2
of x is y 20,000
(a) 4 (b) 5 (c) 1 (d) 3 (c) Maximize Z = 0.08x + 0.09y
87. The sum of the series Subject to, x + y 50,000, x 15000,
y 20,000
log 4 2 – log 8 2 + log16 2 + ..................to is (d) Maximize Z = 0.08x + 0.09y
(a) 1 – log e 2 (b) 1 + log e 2 Subject to, x + y 50,000, x 15000,
y 20,000
(c) log e 2 – 1 (d) log 2 e
www.jeeneetbooks.in

WWW.IIT-NEET.XYZ

Mock Test 7 MT-77

93. If and are the roots of x2 – x + 1 = 0, then the the axis of x is always a constant c. The locus of
equation whose roots are 100 and 100 are P is
(a) x2 – x + 1 = 0 (b) x2 + x – 1 = 0 (a) a straight line having equal intercepts c on
2
(c) x – x – 1 = 0 (d) x2 + x + 1 = 0 the axes
94. If ( 1) is a cube root of unity, then
c
2 (b) a circle having its centre at 0, and
1 1 1 2
2
1 1 1
2 c
1 1 1
passing through c 2 ,
2
(a) 0 (b) 1
(c) – 4 (d) 2 c
95. If vector equation of the line (c) a parabola with its vertex at 0, and
2
x 2 2y 5
z 1, is c
2 3 passing through c 2 ,
2
5ˆ ˆ 3ˆ
r 2iˆ j k 2iˆ j pkˆ then p is (d) None of these
2 2
101. The projections of a line segment on the
equal to
coordinate axes are 12, 4, 3. The direction cosine
(a) 0 (b) 1 (c) 2 (d) 3
of the line are:
96. If A = sin8 + cos14 , then for all values of :
(a) A 1 (b) 0 < A 1 12 4 3 12 4 3
(a) - ,- , (b) ,- ,
(c) 1< 2A 3 (d) none of these 13 13 13 13 13 13
97. A circular wire of radius 7 cm is cut and bent
again into an arc of a circle of radius 12 cm. 12 4 3
(c) , , (d) None of these
The angle subtended by the arc at the centre 13 13 13
is 102. A parabola is drawn with its vertex at (0, – 3), the
(a) 50° (b) 210° (c) 100° (d) 60° axis of symmetry along the conjugate axis of the
98. The locus of the mid points of chords of the
x 2 y2
2 hyperbola 1 and passing through
parabola y 4ax, which pass through a fixed 49 9
point (h, k), is given by the two foci of the hyperbola. The co-ordinates
of the focus of the parabola are
(a) y 2 2ax ky 2ah 0
11 11
(a) 0, (b) 0,
(b) y 2 2ax ky 2ah 0 6 6

(c) y 2 2ax ky 2ah 0 11 11


(c) 0, (d) 0,
(d) None of these 12 12
99. If the tangent to the parabola y2 = 4ax meets the 103. A value of c for which conclusion of Mean Value
axis in T and tangent at the vertex A in Y and the Theorem holds for the function f (x) = loge x on
rectangle TAYG is completed, then the locus of the interval [1, 3] is
G is (a) log3 e (b) loge3
(a) y2 + 2ax = 0 (b) y2 + ax = 0
2
1
(c) x + ay = 0 (d) x2 + 4ay = 0 (c) 2 log3e (d)
log3 e
2
100. A point P moves such that the difference
104. Which of the following function is an odd
between its distances from the origin and from
function ?
EBD_7443
www.jeeneetbooks.in

WWW.IIT-NEET.XYZ

MT-78 Target VITEEE


(a) /3 (b) /2
(a) f (x) 1 x x2 1 x x2
(c) /4 (d) /6
ax 1 110. The area of the plane region bounded by the
(b) f (x) x
ax 1 curves x + 2y2 = 0 and x + 3y2 = 1is equal to

1 x2 5 1
(c) f ( x) log (a) (b)
1 x 2 3 3
(d) f(x) = k, k is a constant 2 4
105. Let p : I am brave, (c) (d)
3 3
q : I will climb the Mount Everest.
111. The area between the curve y = x (x –1) (x – 2)
The symbolic form of a statement,
'I am neither brave nor I will climb the mount and x-axis is
Everest' is (a) 1/4 (b) 1/2
(a) p q (b) ~ (p q) (c) 1 (d) 0
(c) ~ p ~ q (d) ~ p q /6
106. The expansion of log cosh x in powers of x by cos7 3x dx
112. Evaluate :
Maclaurin’s theorem is 0

x x3 x5 x6 15 16
(a) ...... (a) (b)
1 5 25 125 150 105

x2 x4 x6 105 16
(b) ........... (c) (d)
2 12 45 150 150
113. The differential equations of all conics whose
x2 x4 x6 axes coincide with the co-ordinate axis is
(c) ...........
2 12 45 d2 y dy
2
dy
(a) xy 2
x y 0
dx dx dx
x x3 x 5 x 6
(d) ........... 2
1 5 25 125 d2 y dy dy
(b) xy 2
x x 0
107. The current flowing through a tangent dx dx dx
galvanomenter is obtained from the relation i = k 2
tan where is the deflection and k is a constant. d2 y dy dy
(c) xy 2
x y 0
The relative error in the value of i due to error in dx dx dx
the observation of is 2
d2 y dy dy
d 2d (d) xy 2
x y 0
(a) (b) dx dx dx
sin sin 114. The degree and order of the differential equation
d 2d of the family of all parabolas whose axis is X - axis,
(c) (d) are respectively.
sin 2 sin 2
108. If the letters of the word SACHIN are arranged (a) 2, 3 (b) 2, 1
(c) 1, 2 (d) 3, 2
in all possible ways and these words are written
out as in dictionary, then the word SACHIN 115. In the group of non-zero rational numbers under
appears at serial number the binary operation * given by a* b = ab/5 the
(a) 601 (b) 600 identity element and ht inverse of 8 are
respectively.
(c) 603 (d) 602
(a) 5 and 5/8 (b) 5 and 25/8
/2
sin x (c) 5 and 8/25 (d) none of these
109. Evaluate dx
0 sin x cos x
www.jeeneetbooks.in

WWW.IIT-NEET.XYZ

Mock Test 7 MT-79

imagined. They look after other dolphins when they


( x 1) ( x 3)
116. f ( x) is a real valued function in are ill, care for pregnant mothers and protect the
( x 2) weakest in the community, as we do. Some scientists
the domain have suggested that dolphins have a language but it
(a) ( , 1] [3, ) (b) ( , 1] (2, 3] is much more probable that they communicate with
each other without needing words. Could any of these
(c) [ 1, 2) [3, ) (d) none of these mammals be more intelligent than man? Certainly the
117. In a chess tournament, where the participants most common argument in favor of man's superiority
were to play one game with another, two chess over them that we can kill them more easily than they
players fell ill, having played 3 games each. If can kill us is the least satisfactory. On the contrary, the
the total number of games played is 84, the more we discover about these remarkable creatures,
number of participants at the beginning was the less we appear superior when we destroy them.
(a) 15 (b) 16 121. It is clear from the passage that dolphins-
(c) 20 (d) 21 (a) don't want to be with us as much as we
want to be with them
3 (b) are proven to be less intelligent than once
118. The equation sin –1x – cos–1x = cos–1 has thought
2
(a) unique solution (c) have a reputation for being friendly to
(b) no solution humans
(c) infinitely many solutions (d) are the most powerful creatures that live in
(d) None of these the oceans
119. There is a five-volume dictionary among 50 books 122. The fact that the writer of the passage thinks
arranged on a shelf in random order. If the that we can kill dolphins more easily than they
volumes are not necessarily kept side by side, can kill us-
the probability that they occur in increasing order (a) means that they are better adapted to their
from left to right is : environment than we are
(b) shows that dolphins have a very
1 1 sophisticated form of communication
(a) (b)
5 550 (c) proves that dolphins are not the most
intelligent species at sea
1 (d) does not mean that we are superior to them
(c) (d) None
505 123. One can infer from the reading that-
120. If A and B play a series of games in each of which (a) dolphins are quite abundant in some areas
probability that A wins is p and that B wins is of the world
q = 1 – p. Therefore the chance that A wins two (b) communication is the most fascinating
games before B wins three is aspect of the dolphins
(c) dolphins have skills that no other living
p2 p2 creatures have such as the ability to think
(a) (b) (d) dolphins have some social traits that are
1 3q 1 3q 2
similar to those of humans
p 2 (1+2q+3 q 2 ) (d) None of these 124. Choose the word opposite in meaning to the word
(c)
given in bold.
PART - IV : ENGLISH Tremulous
(a) Steady (b) Obese
Direction (Qs. 121 - 123) : Read the passage carefully (c) Young (d) Healthy
and answer the questions given below 125. Choose the best pronunciation of the word,
Dolphins are regarded as the friendliest creatures in Asthma, from the following options.
the sea and stories of them helping drowning sailors (a) ash-thama (b) as-tma
have been common since Roman times. The more we (c) as-tha-maa (d) azma
learn about dolphins, the more we realize that their
society is more complex than people previously
EBD_7443
www.jeeneetbooks.in

WWW.IIT-NEET.XYZ

MOCK
VITEEE Mock Test Paper
8
Max. Marks : 125 Time : 2½ hrs
PART - I (PHYSICS) 4. In a half wave rectifier, the r.m.s. value of the
A.C. component of the wave is
1. Two spheres of the same material have radii 1 (a) equal to d.c. value
m and 4 m and temperatures 4000 K and 2000 (b) more than d.c. value
K respectively. The ratio of the energy radiated (c) less than d.c. value
per second by the first sphere to that by the (d) zero
second is 5. In a Wheatstone's bridge, three resistances P, Q
(a) 1 : 1 (b) 16 : 1 and R connected in the three arms and the fourth
arm is formed by two resistances S1 and S2
(c) 4 : 1 (d) 1 : 9.
connected in parallel. The condition for the
2. A current is flowing in a circular conductor of bridge to be balanced will be
radius r. It is lying in a uniform magnetic field B
such that its plane is normal to it. The magnetic P 2R
(a)
force acting on the loop will be Q S1 S2
(a) 0 (b) /rB
P R (S1 S2 )
(c) 2 /rB (d) irB (b) Q S1S2
3. A particle, which is constrained to move along
the x-axis, is subjected to a force in the same P R (S1 S2 )
direction which varies with the distance x of the (c) Q 2S1S2
particle from the origin as F(x) = -kx + ax3. Here
k and a are positive constants. For x 0 , the P R
(d)
functional form of the potential energy U(x) of Q S1 S2
the particle is 6. A 10 resistance, 5 mH coil and 10 µF capacitor
are joined in series. When a variable frequency
U(x) alternating current source is joined to this
(a) X combination, the circuit resonates. If the
resistance is halved, the resonance frequency
(a) is halved
(b) is doubled
U(x) (c) remains unchanged
(b) X (d) is quadrupled
7. An object at rest in space suddenly explodes into
three parts of same mass. The momentum of the

U(x)
two parts are 2pî and pĵ . The momentum of
(c) X
the third part
(a) will have a magnitude p 3

(b) will have a magnitude p 5


U(x)
(c) will have a magnitude p
(d) X (d) will have a magnitude 2p.
www.jeeneetbooks.in

WWW.IIT-NEET.XYZ

Mock Test 8 MT-81

8. If vs, vx and vm are the speed of soft gamma of photoelectrons would be about
rays, X-rays and microwaves respectively in (a) 1.49 eV (b) 2.2 eV
vacuum, then (c) 3.0 eV (d) 5.0 eV
(a) vs > vx > vm (b) vs < vx < vm 15. The load versus elongation graph for four wires
(c) vs > vx < vm (d) vs = vx = vm has been shown in the figure. The thinest wire
9. The resistance of a wire is R. If the length of the is Load a
b
wire is doubled by stretching, then the new (a) a c
resistance will be
(b) b
(a) 2R (b) 4R d
(c) R (d) R/4 (c) c
10. Which of the following curves correctly Elongation
(d) d
represents the variation of capacitive reactance
16. A charged particle shows an acceleration of
(Xc) with frequency n?
4.2 × 1010 ms–2 under an electric field at low speed.
The acceleration of the particle under the same
XC XC field when its speed becomes 2.88 × 108 ms–1 will
(a) (b) be
(a) 2.88 × 108 ms–2 (b) 1.176 × 1010 ms–2
n n (c) 4.2 × 1010 ms–2 (d) None of these
17. The real time variation of input signals A and B
are as shown below. If the inputs are fed into
XC XC

(c) (d) NAND gate, then select the output signal from
the following.
n n

11. The half-life of radioactive Radon is 3.8 days. A


The time at the end of which (1/20)th of the A
B Y
Radon sample will remain undecayed is (given B
log10e = 0.4343) t (s)
(a) 13.8 days (b) 16.5 days
(c) 33 days (d) 76 days
12. Find equivalent resistance between the points Y
(a)
A and B.
t (s)
2R
0 2 4 6 8
1 2R 3 R
A B
2 4
Y
(b)
(a) R/2 (b) R/4
(c) R (d) R/8 t (s)
0 2 4 6 8
13. Water boils in the electric kettle in 15 minutes
after switching on. If the length of heating wire
is decreased to 2/3 of its initial value, then the
(c) Y
same amount of water will boil with the same
supply voltage in t (s)
0 2 4 6 8
(a) 8 minutes (b) 10 minutes
(c) 12 minutes (d) 15 minutes
14. The threshold wavelength of the tungsten is 2300 (d)
Å. If ultraviolet light of wavelength 1800 Å is Y
incident on it, then the maximum kinetic energy t (s)
0 2 4 6 8
EBD_7443
www.jeeneetbooks.in

WWW.IIT-NEET.XYZ

MT-82 Target VITEEE


18. When a plastic thin film of refractive index 1.45
500
is placed in the path of one of the interfering G
waves then the central fringe is displaced
2V
through width of five fringes. The thickness of
12V R A
the film will be, if the wavelength of light is B
5890Å.
(a) 6.544 × 10–4 cm (b) 6.544 × 10–4 m (a) 100 (b) 200
(c) 6.54 × 10–4 cm (d) 6.5 × 10–4 cm (c) 1000 (d) 500
19. A ball moving with velocity 2 m/s collides head 25. When light is incident on a metal surface the
on with another stationary ball of double the
maximum kinetic energy of emitted electrons
mass. If the coefficient of restitution is 0.5, then
their velocities (in m/s) after collision will be (a) vary with intensity of light
(a) 0, 1 (b) 1, 1 (b) vary with frequency of light
(c) 1, 0.5 (d) 0, 2 (c) vary with speed of light
20. In the circuit given below, the charge in C, on (d) vary irregularly
the capacitor having 5 F is 26. If the electric flux entering and leaving an
2 F enclosed surface respectively is 1 and 2, the
3 F
d electric charge inside the surface will be
e
(a) ( 2 + 2) × o (b) ( 2 – 2) × o
5 F (c) ( 1 + 2) × o (d) ( 2 – 1) × o
f c
4 F 27. Total internal reflection can take place only if
a + b (a) light goes from optically rarer medium
6V (smaller refractive index) to optically
(a) 4.5 (b) 9 denser medium
(c) 7 (d) 15 (b) light goes from optically denser medium
21. A nuclear reaction is given by to rarer medium
A A 0 (c) the refractive indices of the two media are
ZX Z 1Y 1e , represents
close to different
(a) fission (b) -decay (d) the refractive indices of the two media are
(c) -decay (d) fusion widely different
22. A small current element of length d and carrying 28. Doubly ionised helium atom and hydrogen ions
current is placed at (1, 1, 0) and is carrying current are accelerated, from rest, through the same
in ‘+z’ direction. If magnetic field at origin be potential difference. The ratio of final velocities
B1 and at point (2, 2, 0) be B2 then of helium and hydrogen is

(a) B1 B2 (b) | B1 | | 2B2 | (a) 1 : 2 (b) 2 :1


(c) 1 : 2 (d) 2 : 1
(c) B1 B2 (d) B1 2B2
29. Figure here shows the frictional force versus
23. According to Maxwell’s equation the velocity displacement for a particle in motion. The loss
of light in any medium is expressed as of kinetic energy in travelling over s = 0 to 20 m
1 1 will be
(a) (b) f(N)
0 o
15
0
(c) / (d)
10
24. In the circuit , the galvanometer G shows zero
deflection. If the batteries A and B have 5
negligible internal resistance, the value of the
resistor R will be 0 x(m)
0 5 10 20
www.jeeneetbooks.in

WWW.IIT-NEET.XYZ

Mock Test 8 MT-83

(a) 250 J (b) 200 J 35. A nucleus ruptures into two nuclear parts which
(c) 150 J (d) 10 J have their velocity ratio equal to 2 : 1. What
30. In cyclotron the resonance condition is will be the ratio of their nuclear radius ?
(a) the frequency of revolution of charged (a) 21/3 : 1 (b) 1 : 21/3
1/2
(c) 3 : 1 (d) 1 : 31/2
particle is equal to the frequency of A.C.
voltage sources 36. An electron with velocity V along the axis
approaches a circular current carrying loop as
(b) the frequency of revolution of charged
shown in the figure. The magnitude of magnetic
particle is equal to the frequency of applied
i
magnetic field
(c) the frequency of revolution of charged V
–e
particle is equal to the frequency of rotation R
x
of earth
(d) the frequency of revolution of charged
force on electron at this instant is:
part icle, frequency of A.C. source an d
frequency of magnetic field are equal 0 eviR 2 x eviR 2 x
31. The length of a potentiometer wire is . A cell of (a) 2 (x 2 (b) 0
R 2 )3 / 2 (x 2 R 2 )3 / 2
emf E is balanced at a length /3 from the
positive end of the wire. If the length of the wire eviR 2 x
0
is increased by /2. At what distance will be the (c) (d) zero
4 (x 2 R 2 )3 / 2
same cell give a balance point?
(a) 2 /3 (b) /2 37. In a photoelectric experiment anode potential
(c) /6 (d) 4 /3 is plotted against plate current
32. Two drops of the same radius are falling through
air with a steady velocity of 5 cm per sec. If the I
two drops coalesce, the terminal velocity would
be C B
(a) 10 cm per sec A
(b) 2.5 cm per sec
(c) 5 × (4)1/3 cm per sec V

(d) 5 3 cm per sec (a) A and B will have different intensities


while B and C will have different frequencies
33. In Young’s double slit expt. the distance between (b) B and C will have different intensities
two sources is 0.1 mm. The distance of the screen while A and C will have different frequencies
from the source is 20 cm. Wavelength of light (c) A and B will have different intensities
used is 5460 Å. The angular position of the first while A and C will have equal frequencies
dark fringe is (d) A and B will have equal intensities while
(a) 0.08º (b) 0.16º B and C will have different frequencies
(c) 0.20º (d) 0.32º 38. A Gaussian surface in the figure is shown by
34. The forward biasd diode is dotted line. The electric field on the surface will
–2V be
(a) +2V

(b) –4V –3V q1 q2


(c) 3V 5V –q1

(d) 0V –2V (a) due to q1 and q2 only (b) due to q2 only


(c) zero (d) due to all
EBD_7443
www.jeeneetbooks.in

WWW.IIT-NEET.XYZ

MT-84 Target VITEEE


39. Two identical batteries, each of e.m.f. 2 volt and (a) Fehling solution (b) C6H5NHNH2
internal resistance 1.0 ohm are available to (c) Ammonia (d) HCl
produce heat in a resistance R = 0.5 , by passing 45. The IUPAC name for th e coordination
a current through it. The maximum power that compound Ba[BrF4 ]2 is
can be developed across R using these batteries (a) Barium tetrafluorobromate (V)
is (b) Barium tetrafluorobromate (III)
1 2V
(c) Barium bis (tetrafluorobromate) (III)
(d) none of these
46. The rate constant of the reaction : A 2 B, is 1.0
1 2V × 10–3 mol L–1 min–1. If the initial concentration
of A is 1.0 mol L –1, what would be the
concentration of B after 100 minutes:
(a) 0.1 mol L–1 (b) 0.2 mol L–1
(c) 0.9 mol L –1 (d) 1.8 mol L–1
0.5
47. Which of the following statements is correct for
(a) 1.28 W (b) 2.0 W a strong electrolyte :
(c) 8/9 W (d) 3.2 W (a) ½
m increases linearly with C
40. To use a transistor as an amplifier
(a) emitter-base junction is forward biased and (b) m increases linearly with C2
collector base junction is reverse biased (c) m decreases linearly with C2
(b) both junctions are forward biased ½
(d) m decreases linearly with C
(c) both junctions are reverse biased
(d) it does not matter how the transistor is 48. If all the reactants and the products in a reaction
biased, it always works as an amplifier are in their standard states of unit activity, then
which is true of the following ?
(a) Gº = 0 (b) G = 0
PART - II (CHEMISTRY) (c) G = Gº (d) Hº – T Sº = 0
41. Paramagnetism of Cr (Z = 24), Mn2+ (Z = 25) 49. EMF of an H2 – O2 fuel cell
and Fe3+ (Z = 26) are x, y and z respectively. (a) is independent of partial pressures of H2
and O2
They are in the order
(a) x = y = z (b) x > y > z (b) decreases on increasing PH 2 and PO2
(c) x = y > z (d) x > y = z (c) increases on increasing PH 2 and PO2
42. Nitrobenzene can be reduced to aniline by
(d) varies with the concentration of OH– ions
H 2 / Ni Sn / HCl in the cathodic and anodic compartments.
I II 50. When K2 Cr 2O7 is heated with concentrated
H2SO4 and soluble chloride such as KCl
Zn / NaOH LiAlH4 (a) Cl– ion is oxidised to Cl2 gas
III IV
(a) I, II and III (b) I and II (b) C 2 O 72 ion is reduced to green Cr3+ ion
(c) I, II and IV (d) only II (c) red vapour of CrO2Cl2 is evolved
43. Which reaction is used for converting a lower (d) CrCl3 is formed
carboxylic acid into its next higher homologue? 51. The energy of a photon is given as E/atom
(a) Curtius reaction 3.03 10 19 J atom 1 . Then the wavelength ( )
(b) Baeyer – Villiger reaction of the photon is
(c) Darzen glycidic ester synthesis (a) 65.6 nm (b) 656 nm
(d) Arndt – Eistert synthesis (c) 0.656 nm (d) 6.56 nm
44. Which of the following reagent reacts in 52. The unit cell of diamond is made up of
different ways with CH3 CHO, HCHO and (a) 6 carbon atoms, 4 atoms constitute ccp and
C6H5CHO ? two atoms occupy half of octahedral voids
www.jeeneetbooks.in

WWW.IIT-NEET.XYZ

Mock Test 8 MT-85

(b) 8 carbon atoms, 4 atoms constitute ccp and 57. In some fission process, the mass number of
4 atoms occupy all the octahedral voids fission fragments are 144 and 90 respectively.
(c) 8 carbon atoms, 4 atoms form fcc lattice If the K.E. of heavy fragment is 70 MeV, the
and 4 atoms occupy half of the tetrahedral total fission energy is
voids alternately (a) 200 MeV (b) 182 MeV
(c) 190 MeV (d) 170 MeV
(d) 12 carbon atoms, 4 atoms form fcc lattice
58. At constant pressure, addition of helium to the
and 8 atoms occupy all the tetrahedral
reaction system :
holes.
N 2 ( g ) 3H 2 ( g ) 2 NH 3 ( g )
OH
(a) favours the formation of ammonia
NaOH CH 2=CHCH 2Cl (b) reduces the formation of ammonia
53. [X] [Y] . (c) reduces the dissociation of ammonia
(d) does not affect the position of equilibrium
Here [Y] is a 59. Fehling solution can be used for distinguishing
(a) single compound between
(b) mixture of two compounds (a) CH3CHO and C6H5CHO
(c) mixture of three compounds (b) CH3CHO and CH3COCH2OH
(d) no reaction is possible (c) both (a) & (b)
54. In the reaction A + B Products, initial (d) none
concentration of both A and B equal to 0.1 60. Which one does not belong to the same
M,each is reduced to 1.0 × 10–2 M. The half-life compound ?
is increased to ten fold. The rate of the reaction (a) Paraformaldehyde
is : (b) Paraldehyde
(a) proportional to first power of concentration (c) Trioxane
(b) proportional to second power of (d) Formalin
concentration 61. An iron piece treated with acidified KMnO4
(c) independent of concentration (a) displaces copper from a dilute H2SO4
(d) insufficient information (b) liberates hydrogen from dilute H2SO4
55. Aomic size of Zr and Hf is almost the same (c) displaces copper from CuSO4 solution but
because of does not liberate H2 from dilute H2SO4
(a) both have (n – 1)d2ns2 configurations (d) neither displaces copper from CuSO4
(b) both have the same number of d-electrons solution nor liberates H2 from dilute
in penultimate energy level H2SO4 .
(c) lanthanide contraction 62. A B, k1 1011 e 3000 / T
(d) large screening effect of 4f-electrons over
the valence shell. X Y , k2 1010 e 2000 / T
56. Ethanol can be prepared more easily by which If the two reactions have all the reactants at unit
reaction? molarity each, at what temperature their initial
(i) rates will be equal?
CH 3 CH 2 Br H 2 O CH 3 CH 2 OH
(a) 2000 K (b) 3000 K
(ii) CH3CH 2 Br Ag2 O (in boiling water) (c) 1000 K (d) 1000/2.303 K
63. Molar conductances of BaCl2, H2SO4 and HCl
CH 3CH 2 OH at infinite dilutions are x1, x2 and x3 respectively.
(a) by (i) reaction Equivalent conductance of BaSO4 at infinite
(b) by (ii) reaction dilution will be :
(c) both reactions proceed at same rate (a) (x1 + x2 – x3) /2 (b) x1 + x2 – 2x3
(d) by none (c) (x1 – x2 – x3) /2 (d) (x1 + x2 – 2x3) /2
EBD_7443
www.jeeneetbooks.in

WWW.IIT-NEET.XYZ

MT-86 Target VITEEE


64. Geometrical isomerism can be shown by (a) o - aminoacetophenone
(a) [Ag( NH 3 )(CN)] (b) p-aminoacetophenone
(c) both (a) and (b)
(b) Na 2 [Cd( NO 2 ) 4 ] (d) m-aminoacetophenone
(c) [Pt Cl 4 I 2 ] 72. A coordination complex having which one of
the following descr iptions would be
(d) [Pt ( NH 3 )3 Cl][Au(CN) 4 ] paramagnetic to the maximum extent?
65. Which of the following statement is not true ?
(a) At room temperature, formyl chloride is (a) d 6 (octahedral, low-spin)
present in the form of CO and HCl
(b) d 8 (octahedral)
(b) Acetamide behaves as a weak base as well
as a weak acid. (c) d 6 (octahedral, outer orbital)
LiAlH
(c) CH 3 CONH 2 4 CH 3CH 2 NH 2 (d) d 4 (octahedral,low-spin)
(d) None of the three 73. Which of the following processes takes place
66. A carboxylic acid can best be converted into acid with decrease of entropy?
chloride by using (a) Solid gas
(a) PCl5 (b) SOCl2 (b) sugar + water solution
(c) HCl (d) ClCOCOCl (c) NH3(g) + HCl(g) NH4Cl(s)
67. In the Mac-Arther Cyanide process for (d) A(g) + B(g) mixture
extraction of silver, a small amount of KNO3 is 74. Identify (C) and (D) in the following series of
added. The function of KNO3 is reactions
(a) to oxidise the sulphur present in argentite excess of AgOH
ore to SO2 CH 3 NH 2 [A ] [B]
CH3I
(b) to form anions with Ag+ which is then
heat
reduced to metallic silver by zinc [ C] [ D ]
(c) to oxidise Ag in natural form to Ag+
(d) to oxidise the impurities of lead and zinc (a) (CH3)3COH, CH3NH2
present in it (b) (CH3)2C = CH2, CH3NH2
68. Dehydration of an alcohol in presence of (c) (CH3)3N, CH3OH
sulphuric acid gives alkene (d) (CH3)2C=CH2, CH3OH
H SO 75. Which one of the following is not an oxidation
CH 3CH 2 OH 2 4 CH 2 CH 2 H 2 O product of a primary amine?
Here sulphuric acid acts as (a) A hydroxylamine
(a) an acid (b) a base (b) A nitroso compound
(c) a catalyst (d) all the three (c) A nitro compound
69. Dopping of AgCl crystals with CdCl2 results in (d) None of these
(a) Schottky defect 76. Which method can be used to distinguish
(b) Frenkel defect
(c) Substitutional cation vacancy [Co(NH3 )6 ][Cr(NO 2 ) 6 ] and
(d) Formation of F- centres [Cr(NH3)6][Co(NO2)6]
70. An organic compound A on heating with ethanol (a) by measurement of their conductivity
gives compounds B and C, of which compound (b) by titration method
C is again a derivative of the compound B. The
(c) by precipitation method with AgNO3
compound A is
(d) by electrolysis of their aqueous solutions
(a) CH3COOH (b) (CH3CO)2O
(c) CH3COOC2H5 (d) CH3 CH2OH 77. Which electrphile is likely to be formed as an
71. When aniline is treated with acetyl chloride in intermediate in the following electrophilic
presence of anhydrous aluminium chloride, the substitution reaction ?
main product is
www.jeeneetbooks.in

WWW.IIT-NEET.XYZ

Mock Test 8 MT-87

OH OH 82. Which of the following statement is correct?


CHO (a) Every L.P.P. admits an optimal solution
CHCl3 (b) A L.P.P. admits a unique optimal solution
NaOH (c) If a L.P.P. admits two optimal solutions, it
has an infinite number of optimal solutions
(d) The set of all feasible solutions of a L.P.P.
(a) C HCl 2 (b) C HO is not a convex set.
83. The period of tan 3 is
(c) : C Cl 2 (d) : CCl 2 (a) (b) 3 /4
(c) /2 (d) None of these
78. In p-type semiconductor, the added impurity to
x x
silicon is ----- and conduction of electric current 84. The function f (x) 1 is –
x
is due to the movement of ----- e 1 2
(a) As, electrons (a) an odd function
(b) an even function
(b) P, holes (c) neither an odd nor an even function
(c) Ga, holes (d) a periodic function
(d) Ga, electrons and holes 85. The equation of the right bisector plane of the
79. In the radioactive decay segment joining (2, 3, 4) and (6, 7, 8) is
(a) x + y + z + 15 = 0
A A 4 A 4 A 4
zA z 2B z 1C zA (b) x + y + z – 15 = 0
(c) x – y + z – 15 = 0
the sequence of radiation emitted is
(d) None of these
(a) (b) 86. What is the angle between the line 6x = 4y = 3z
(c) (d) and the plane 3x + 2y – 3z = 4 ?
80. In the following reaction, the reagent X should (a) 0 (b) /6(c) /3 (d)
be /2
87. An infinite G.P. has first term ‘x’ and sum ‘5’,
conc . H 2SO 4 then x belongs to
RCOOH [ X ] RNH 2
(a) x < – 10 (b) – 10 < x < 0
(a) NH3 (b) HN3 (c) 0 < x < 10 (d) None of these
(c) Either of the two (d) None of the two 88. By examining the chest X-ray, the probability
that TB is detected when a person is actually
PART - III (MATHEMATICS) suffering is 0.99. The probability of an healthy
person diagnosed to have TB is 0.001. In a
2 certain city, 1 in 1000 people suffers from TB,
81. Let f : , [0, 4] be a function
3 3 A person is selected at random and is diagnosed
to have TB. Then, the probability that the person
defined as
actually has TB is
f (x) = 3 sin x cos x 2 . Then, f–1(x) is given 110 2
by (a) (b)
221 223
1 x 2 110 1
(a) sin (c) (d)
2 6 223 221

1 x 2 7 z
(b) sin 89. If f(z) = , where z = 1+ 2i, then |f(z)| is
2 6 1 z2
equal to :
2 x 2
(c) cos 1 |z|
3 2 (a) (b) | z |
2
(d) None of these (c) 2 | z | (d) None
EBD_7443
www.jeeneetbooks.in

WWW.IIT-NEET.XYZ

MT-88 Target VITEEE

90. If arg(z1 ) arg(z 2 ) , then (a) sin – cos (b) sin + cos
(c) tan + cot (d) tan – cot
(a) z2 = kz1–1 (k > 0) 98. If the eccentricity of the hyperbola x2 – y2 sec2 = 4
(b) z2 = kz1(k > 0)
is 3 times the eccentricity of the ellipse
(c) | z2 | = | z1 | x2sec2 + y2 = 16, then the value of equals
(d) None of these 3
91. If nPr = nPr + 1 and nCr = nCr – 1, then the values (a) (b)
6 4
of n and r are
(a) 4, 3 (b) 3, 2
(c) (d)
(c) 4, 2 (d) None of these 3 2
92. If (G, *) is a group such that (a * b)2 = (a * a) *
99. In a triangle ABC, a cos A b cos B c cos C is
( b * b) for all a, b, G , then G is a b c
(a) abelian (b) finite equal to
(c) infinite (d) None
r R
93. Let f : R R be a function defined by (a) (b)
R r
f (x) = min {x 1, x 1} ,Then which of the 2r R
(c) (d)
following is true ? R 2r
100. The value of sin (cot–1 (cos (tan –1x)) is
(a) f (x) is differentiable everywhere
(b) f (x) is not differentiable at x = 0
(a) x2 2 (b) x2 1
(c) f (x) 1 for all x R
x2 1 x2 2
(d) f (x) is not differentiable at x = 1
C1 C3 C5 x 1
94. The value of ....... is equal to (c) 2 (d) 2
2 4 6 x 2 x 2

2n 1 2n 1 5 1 12
(a) (b) 101. The equation sin sin has
x x 2
n 1 n 1
(a) no solution
2n 1 2n 1 (b) only one solution, x = 13
(c) (d) (c) only one solution, x = – 13
n 1 n 1
(d) two solutions, x = ± 13
32 k 42 32 3 k 102. A point P moves such that the sum of twice its
2
52 42 4 k = 0, then the value distance from the origin and its distance from
95. If 42 k
5 k 62 52 5 k the y-axis is a constant equal to 3. P describes
(a) A circle with its centre at (–1, 0) and radius
of k is
(a) 2 (b) 1 2 3
(c) –1 (d) 0 (b) An ellipse centred at (–1, 0) and of
96. If f(x) = x 2 – 4x – 5, then f(A), where 1
eccentricity
1 2 2 2
2 1 2 (c) A hyperbola centred at (1, 0) and of
A= equals eccentricity 2
2 2 1 (d) None of these
(a) O (b) I 103. The equation of the tangent to the parabola
(c) –I (d) 2I y 2 6x at the point whose ordinate is 6, is
cos sin (a) x + 2y + 6 = 0 (b) 2x – y + 6 = 0
97. is equal to (c) x – 2y + 6 = 0 (d) x – y + 6 = 0
1 tan 1 cot
www.jeeneetbooks.in

WWW.IIT-NEET.XYZ

Mock Test 8 MT-89

x2 y 2 1
104. If e1 is the eccentricity of the ellipse 1 111. If f(x) is a quadratic in x, then f (x) dx is
16 25
and e2 is the eccentricity of the hyperbola passing 0
through the foci of the ellipse and e1e2 = 1, then equal to
equation of the hyperbola is :
1 1
(a) f (0) 4f f (1)
x2 y2 x2 y2 6 2
(a) 1 (b) 1
9 16 16 9
1 1
x2 y 2 x2 y 2 (b) 4f (0) f f (1)
(c) 1 (d) 1 6 2
9 25 9 36
3 2
105. The two curves x – 3xy + 2 = 0 and 1 1
(c) f (0) f 4(1)
3x2y – y3 – 2 = 0 6 2
(a) Cut at right angles
(b) Touch each other 1 1
(c) Cut at an angle /3 (d) f (0) f f (1)
6 2
(d) Cut at an angle /4
106. For the curve yn = an – 1 x if the subnormal at 1
any point is a constant then n is equal to 112. If f’(x) =f (x) + f (x ) dx and given f (0) = 1.
(a) 1 (b) 2 0
(c) –2 (d) –1 Then f (x) =
107. The coordinates of a moving particle at time t
are given by x = c t2 and y = b t2. The speed of 2e x 1 e 2e x 1 e
the particle is given by (a) (b)
2 e 1 e 3 e 3 e
(a) 2 t (c + b) (b) 2 t (c 2 b2) 2e x 2e x
(c) (d)
(c) t (c 2 b2 ) (d) 2 t (c 2 b2 ) 2 e 3 e

1 1 113. If p (u v) (v w) (w u)
108. lim equals to
h 0 h3 8 h 2h 1
and [ u v w ] , then is equal to
5
1 1
(a) (b) (a) 5 (b) 10
8 8 (c) 15 (d) None of these
1 1
(c) (d) 114. Solve : (D2 + 6D + 8)y = e–2x.
48 48
109. Five balls of different colours are to be placed 1
(a) y Ae 4x Be 2x xe 2x
in three boxes of different sizes. Each box can 2
hold all five balls. In how many ways can we
place the balls so that no box remains empty? 4x 2x 1
(b) y Ae Be xe 2x
(a) 50 (b) 100 2
(c) 150 (d) 200 1
4x
110. In a hurdle race, a player has to cross 10 hurdles. (c) y Ae Be 2x xe 2x

The probability that he will clear each hurdle is 2


1
5
. Then, the probability that he will knock (d) y Ae 4x Be 2x
xe 2x
6 2
down fewer than 2 hurdles is 115. The area of the triangle formed by the tangent

5 510 and normal at the point 1, 3 on the circle x2


(a) (b) + y2 = 4 and the x-axis is
2 69 2 610
(a) 3 sq. units (b) 2 3 sq. units
59 510
(c) (d) (c) 3 2 sq. units (d) 4 sq. units
2 610 2 69
EBD_7443
www.jeeneetbooks.in

WWW.IIT-NEET.XYZ

MT-90 Target VITEEE


116. The temperature T of a cooling object drops at a In the world today we make health and end in itself.
rate proportional to the difference T – S, where We have forgotten that health is really means to enable
S is constant temperature of surrounding a person to do his work and do it well. a lot of modern
medium. If initially T = 150°C, find the
medicine and this includes many patients as well as
temperature of the cooling object at any time t.
(a) S – (S + 150) ekt (b) S + (150 – S)ekt many physicians pays very little attention to health
(c) S + (150 + S)et (d) S – (150 – S)ekt but very much attention to those who imagine that
117. The acute angle between the medians drawn they are ill. Our great concern with health is shown
through the acute angle of an isosceles right by the medical columns in newspapers. the health
angled triangle is articles in popular magazines and the popularity of
1 2 1 3 television programmes and all those books on
(a) cos (b) cos medicine. We talk about health all the time. Yet for
3 4
the most part the only result is more people with
4 1 1 5 imaginary illness. The healthy man should not be
(c) cos (d) cos
5 6 wasting time talking about health: he should be using
118. A girls walks 4 km towards West. Then, she health for work. The work does the work that good
walks 3 km in a direction 30° East to North and health possible.
stops. The girls displacement from her initial
point of departure is 121. Modern medicine is primarily concerned with
3ˆ 3 3ˆ 5ˆ 3ˆ (a) promotion of good health
(a) i j (b) i j
2 2 2 2 (b) people suffering from imaginary illness
5ˆ 3 3ˆ (c) people suffering from real illness
(c) i j (d) None of these (d) increased efficiency in work
2 2
119. For the probability density function 122. The passage suggests that
2x
2e , x 0 (a) health is an end in itself
f (x)
0 ,x 0 (b) health is blessing
F(2) is equal to (c) health is only means to an end
4 3
e 1 e 1 (d) we should not talk about health
(a) 4 (b) 3
e e 123. Talking about the health all time makes people
5/ 2 5 (a) always suffer from imaginary illness
e 1 e 1
(c) (d) 5
e e (b) sometimes suffer from imaginary illness
120. Consider the switching circuit (c) rarely suffer from imaginary illness
a' (d) often suffer from imaginary illness
124. Select the word or group of words that is most
b' similar in meaning to the word in capital letters.
RUMINATE
c
(a) to run fast (b) to reprimand
Logical expression corresponding to the
complimentary to the above circuit is (c) to think deeply (d) to spend lavishly
(a) a . b . c ' (b) a b c ' 125. Choose the best pronunciation of the word,
(c) a '. b '.c (d) none Video, from the following options.
(a) wee-diyo (b) vid-i-yoh
PART - IV (ENGLISH) (c) vehi-dyoh (d) vee-dio
Direction (Qs. 121 - 123) : Read the passage
carefully and answer the questions given below.
www.jeeneetbooks.in

WWW.IIT-NEET.XYZ

MOCK
VITEEE Mock Test Paper
9
Max. Marks : 125 Time : 2½ hrs
4. The figure shows electric potential V as a
PART - I (PHYSICS)
function x. Rank the four regions according to
1. If the temperature of the sun were to increase the magnitude of x-component of the electric field
from T to 2T and its radius from R to 2R, then the E within them, greatest first:
ratio of the radiant energy received on earth to
what it was previously will be V
(a) 32 (b) 16
(c) 4 (d) 64
2. Two thin flat metal plates having large surface area
are charged separately to acquire charge densities 1 2 3 4
+ and – . The plates are then brought near to x
(a) E4 > E2 > E3 > E1 (b) E2 > E4 > E1 = E3
each other and held parallel to each other (Fig.):
(c) E1 > E2 > E3 > E4 (d) E1 > E3 > E2 > E4
5. Identify the WRONG statement :
A B C
(a) In an electric field two equipotential surfaces
can never intersect.
(b) A charged particle free to move in an electric
field shall always move in the direction of
–vely charged
E.
+vely charged
(c) Electric field at the surface of a charged
If EA, EB and EC denote the electric fields at the conductor is always normal to the surface.
points A, B and C respectively, then which of (d) The electric potential decreases along a line
the following will be true : of force in an electric field.
6. In figure, a carriage P is pulled up from A to B.
(a) EA EC The relevant coefficient of friction is 0.40. The
0 work done will be B
0 kg
(b) EA EB EC P 5
(a) 10 kJ
0
m 30 m
(b) 23 kJ 50
(c) EA EC 0, E B
0
(c) 25 kJ A C
2
(d) EA EC 0, E B
0
(d) 28 kJ
3. An application of Bernoulli’s equation for fluid 7. A metallic spherical shell of radius R has a charge
flow is found in –Q on it. A point charge +Q is placed at the centre
(a) dynamic lift of an aeroplane of the shell. Which of the graphs shown below
(b) viscosity meter may correctly represent the variation of the
(c) capillary rise electric field E with distance r from the centre of
(d) hydraulic press the shell ?
EBD_7443
www.jeeneetbooks.in

WWW.IIT-NEET.XYZ

MT-92 Target VITEEE

E E

(a) (b) 2
1
0 R r 0 R r

E E
3
(c) (d) (a) i2 > i3 > i1 (b) i2 > i1 > i3
(c) i1 > i2 > i3 (d) i1 > i3 > i2
0 R r 0 R r 11. The variable point B of a 80 rheostat AC has
been set exactly in the midway such that the
8. A solid sphere of radius R1 and volume resistance of the part AB is equal to the resistance
of the part BC. The rheostat is connected with a
0 resistance of 20 and a battery of 8.0 V as shown
charge density is enclosed by a hollow
r in the figure. The current supplied by the battery
sphere of radius R2 with negative surface charge is :
density , such that the total charge in the
system is zero. 0 is a positive constant and r is A
the distance from the centre of the sphere. The B
R2
ratio R is C
1

(a) (b) 2 / 0
0
8.0V
1 1
(c) (d)
0 (a) A (b) A
0/ 2 2 5
9. A battery has an emf of 15 V and internal 2 1
(c) A (d) A
resistance of 1 . Is the terminal to terminal 15 3
potential difference less than, equal to or greater 12. Two cells, having the same e.m.f., are connected
in series through an external resistance R. Cells
than 15 V if the current in the battery is (1) from
have internal resistances r1 and r 2 (r1 > r 2 )
negative to positive terminal, (2) from positive
respectively. When the circuit is closed, the
to negative terminal (3) zero current?
potential difference across the first cell is zero.
(a) Less, greater, equal The value of R is
(b) Less, less, equal
(c) Greater, greater, equal r1 r2 r1 r2
(a) (b)
(d) Greater, less, equal 2 2
10. The figure shows three circuits with identical (c) r1 r2 (d) r1 r2
batteries, inductors and resistances. Rank the
13. An electron is moving in an orbit of radius R
circuits according to the currents through the with a time-period T as shown in the figure. The
battery just after the switch is closed, greatest magnetic moment produced may be given by :
first:
www.jeeneetbooks.in

WWW.IIT-NEET.XYZ

Mock Test 9 MT-93

2 e A 18. A conducting square loop is placed in a magnetic


(a) m field B with its plane perpendicular to the field.
T
The sides of the loop start shrinking at a constant
2 e A R
(b) m rate . The induced emf in the loop at an instant
T e when its side is ‘a’ is
e A (a) 2a B (b) a2 B
(c) m
T (c) 2a B 2 (d) a B
e A 19. A rectangular loop of wire with dimensions
(d) m A R2
T shown is coplanar with a long wire carrying
| e | represents the magnitude of the electron current I. The distance between the wire and the
charge. left side of the loop is r. The loop is pulled to the
14. A galvanometer has a resistance of 50 . If a right as indicated. What are the directions of the
resistan ce of 1 is connected across its
terminals, the total current flow through the induced current in the loop and the magnetic
galvanometer is [Ig represents the maximum forces on the left and right sides of the loop as
current that can be passed through the the loop is pulled?
galvanometer] r
(a) 42 Ig (b) 53 Ig
(c) 46 Ig (d) 51 Ig
15. An LC circuit contains a 20 mH inductor and a 50 I b
F capacitor with an initial charge of 10 mC. The
resistance of the circuit is negligible. Let the a
instant the circuit is closed be t = 0. At what time
is the energy stored completely magnetic ? Induced current Force on Force on
(a) t = 0 (b) t = 1.54 ms left side right side
(d) t = 3.14 ms (d) t = 6.28 ms (a) Counterclockwise To the left To the left
16. A uniformly wound solenoidal coil of self- (b) Counterclockwise To the right To the left
inductance 1.8 × 10–4 H and resistance 6 is (c) Clockwise To the right To the left
broken up into two identical coils. These identical (d) Clockwise To the left To the right
coils are then connected in parallel across a 12 V 20. An equilateral triangular loop ABC made of
battery of negligible resistance. The time constant uniform thin wires is being pulled out of a region
for the current in the circuit is: with a uniform speed v, where a uniform magnetic
(a) 0.1 × 10–4 s (b) 0.2 × 104 s field B perpendicular to the plane of the loop
(c) 0.3 × 10 s –4 (d) 0.4 × 10–4 s exists. At time t = 0, the point A is at the edge of
17. A 3 kg ball strikes a heavy rigid wall with a speed the magnetic field. The induced current (I) vs
of 10 m/s at an angle of 60º. It gets reflected with time (t) graph will be as
the same speed and angle as shown here. If the
ball is in contact with the wall for 0.20s, what is
the average force exerted on the ball by the wall? A v
×××××××××××××
×××××××××××××
×××××××××××××
×××××××××××××
×××××××××××××
60º × × × ×B× × × × ×C× × × ×
×××××××××××××

60º I

(a)
(a) 150N (b) Zero
t
(c) 150 3N (d) 300N
EBD_7443
www.jeeneetbooks.in

WWW.IIT-NEET.XYZ

MT-94 Target VITEEE


I (a) + ve y-axis, x-axis (b) – ve z-axis, y-axis
(c) + ve z-axis, x-axis (d) + ve x-axis, x-axis
25. Fpe represents electrical force on proton due to
(b)
electron and Fep on electron due to proton in a
t
hydrogen atom. Similarly, F'pe represents the
I
gravitational force on proton due to electron and
F'ep the corresponding force on electron due to
(c) proton. Which of the following is NOT true ?
t
(a) Fpe Fep 0
I
(b) F'pe ' 0
Fep

(d) (c) Fpe Fpe


' Fep Fep
' 0
t (d) Fpe Fep and Fpe
' Fep
'
21. The magnitude of flux linked with the circuit of 26. The photo electric work function for a metal
resistance 2 k varies with t second according surface is 4.125 eV. The cut-off wavelength for
to equation: 5t 2 4t 12 this surface is
The induced current in the circuit at t = 0.2 second is (a) 4125 Å (b) 3000 Å
(a) 2 A (b) 1 A (c) 6000 Å (d) 2062 Å
(c) 2 mA (d) 1 mA
22. Two bulbs mar ked 200 V–100 W an d 27. Let , e and p represent the wavelengths
e
200 V–200 W are joined in series and connected of electrons, positrons and proton, respectively,
to a power supply of 200 V. The total power of the same momentum then :
consumed by the two will be near to :
(a) 33 watt (a) p (b) p
e e e e

(b) 66 watt (c) (d)


e e p e e p
(c) 100 watt 28. Figure represents a graph of kinetic energy of
most energetic photoelectrons, Kmax (in eV), and
(d) 300 watt 200 V frequency ( ) for a metal used as cathode in
23. An unpolarised beam of light is incident on a photoelectric experiment. The threshold
place surface separating air and glass at an angle frequency of light for the photoelectric emission
equal to the Brewster angle. Then : from the metal is :
(a) the reflected light has electric component
only perpendicular to the incident plane.
(b) the reflected light has electric component
only in the plane of incidence. 3
Kmax

(c) the electric component parallel to the plane


of incidence in refracted ray completely 2
disappear. 1
(d) the magnetic component of the refracted 15
10 Hz
light completely disappear. 0
24. If the magnetic field B of a polarised
electromagnetic wave oscillates parallel to y-axis
and is given by : By = Bm sin (kz – t). What is
the direction of propagation of the em-wave and (a) 1 × 1014 Hz (b) 1.5 × 1014 Hz
parallel to which axis does the associated electric 14
(c) 2.1 × 10 Hz(d) 2.7 × 1014 Hz
field oscillates ? 29. Using the following data,
www.jeeneetbooks.in

WWW.IIT-NEET.XYZ

Mock Test 9 MT-95

mass of hydrogen atom = 1.00783 u (c) (Eg)C < (Eg)Ge > (Eg)Si
mass of neutron = 1.00867 u (d) (Eg)Si < (Eg)Ge > (Eg)C
34. In semiconductors at a room temperature
mass of nitrogen atom 7 N 4 =14.00307 u (a) the conduction band is completely empty
the calculated value of the binding energy of the (b) the valence band is partially empty and the
conduction band is partially filled
nucleus of the nitrogen atom 7 N 4 is close to (c) the valence band is completely filled and
(a) 56 MeV (b) 98 MeV the conduction band is partially filled
(c) 104 MeV (d) 112 MeV (d) the valence band is completely filled
30. The peak voltage in the output of a half-wave 35. The following figure shows a logic gate circuit
diode rectifier fed with a sinusoidal signal without with two inputs A and B and the output C. The
filter is 10V. The d.c. component of the output voltage waveforms of A, B and C are as shown
voltage is below
(a) 20/ V (b) 10/ 2 V A
Logic gate
(c) 10/ V (d) 10V B circuit C
31. The diagram of a logic circuit is given below.
1
W t
A
X 1
F
B t
1
W t
C
X
The logic circuit gate is
The output F of the circuit is given by : (a) NAND gate (b) NOR gate
(a) W. (X + Y) (b) W. (X. Y) (c) OR gate (d) AND gate
(c) W + (X. Y) (d) W + (X + Y) 36. Two bodies A and B having masses in the ratio
32. The graph given below represents the I-V of 3 : 1 possess the same kinetic energy. The
characteristic of a Zener diode. Which part of ratio of linear momentum of B to A is
the characteristic curve is most relevant for its (a) 1 : 3 (b) 3 : 1
operation as a voltage regulator ? (c) 1: 3 (d) 3 :1
I (mA) 37. Main function of the RF amplifiers in a
superheterodyne receiver is to
(a) amplify signal
(b) reject unwanted signal
Reverse bias a (c) discriminate against image frequency signal
Vz Forward bias and IF-signal
d c b V (V) (d) all the above
38. Optical fibre are used for long distance
communication because
e (a) it amplifies signals to be transmitted
(b) it transfer signals faster than electrical
I ( A) cables
(a) a b (b) b c (c) it pre-emphasise weak signals
(c) c d (d) d e (d) it provide little attenuation as compared to
33. Carbon, silicon and germanium have four valence electrical cable for light propagation
electrons each. These are characterized by 39. A super heterodyne receiver is designed to
valence and conduction bands separated by receive transmitted signals between 5 and 10
energy band gap respectively equal to (Eg)C , MHz. The tuning range of the local oscillate for
(Eg) Si and (Eg) Ge. Which of the following IF frequency 600 kHz for high-side tuning would
statements is true? be
(a) (Eg)C = (Eg)Si = (Eg)Ge (a) 4.6 to 9.6 MHz (b) 5.6 to 10.6 MHz
(b) (Eg)C > (Eg)Si > (Eg)Ge (c) 4.6 to 10.6 MHz (d) 5.6 to 9.6 MHz
EBD_7443
www.jeeneetbooks.in

WWW.IIT-NEET.XYZ

MT-96 Target VITEEE


40. Two straight metallic strips each of thickness t 46. A transition metal ion exists in its highest
and length are rivetted together. Their oxidation state. It is expected to behave as
coefficients of linear expansions are 1 and 2 . (a) a chelating agent
If they are heated through temperature T, the (b) a central metal in a coordination compound
bimetallic strip will bend to form an arc of radius (c) an oxidising agent
(a) t /{ 1 T} (d) a reducing agent
2
47. Non-stoichiometr ic compound have the
(b) t /{( 2 1 T} properties of:
(c) t( 1 2 T (a) electrical conductance
(b) isolation
(d) t( 2 1 T (c) insulation
(d) none of these
PART - II (CHEMISTRY) 48. The lattice parameters of a crystal are a = 5.62 Å,
b = 7.41 Å C = 9.48 Å. The three co-ordinates are
41. Colour of cupric chloride is blue due to _____.
mutually perpendicular to each other. The crystal
(a) p -d electronic transition causes emission is –
of energy (a) tetragonal (b) orthorhombic
(b) d-d electronic transition causes emission (c) monoclinic (d) trigonal
of energy 49. Enthalpy is equal to
(c) d-d electronic transition causes absorption
of energy at red visible wave length G G/T
(a) –T 2 (b) –T 2
(d) p-d electronic transition of Cu2+ state. T T
V P
42. Stainless steel does not rust because:
(a) chromium and nickel combine with iron G/T G
(c) T2 (d) T2
(b) chromium forms an oxide layer and protects T T
V P
iron from rusting
(c) nickel present in it, does not rust 50. Given that S oH 1
131 JK mol 1
,
(d) iron forms a hard chemical compound with 2

chromium present in it. SoCl 223 JK 1mol 1 and


43. Excited state configuration of Mn2+ is 2

(a) t 42g (b) t 32g e2g SoHCl 187 JK 1mol 1 . The standard entropy
change in the formation of 1 mole of HCl(g) from
(d) t 42g e2g (d) t 52g e0g H 2 (g ) and Cl 2 (g ) will be
44. [Ti(H2O)6]3+ shows purple colour due to (a) 20 JK–1 (b) 10 JK–1
(c) 187 JK –1 (d) 374 JK–1
(a) d yz d 2 2 electronic transition
x y 51. What is the entropy change (in JK–1 mol–1) when
(b) d xz d 2 2 electronic transition one mole of ice is converted into water at 0º C?
x y (The enthalpy change for the conversion of ice
(c) d xy eg [d ,d ] electronic transition to liquid water is 6.0 kJ mol–1 at 0ºC)
x2 y2 z2 (a) 21.98 (b) 20.13
(d) d xz eg [d 2 2 , d 2 ] electronic transition (c) 2.013 (d) 2.198
x y z 52. Two substances R and S decompose in solution
45. Amongst the following which are not true ? independently, both following first order kinetics.
(a) EAN of iron in Fe(C5 H 5 ) 2 is 36. The rate constant of R is twice that of S. In an
experiment, the solution initially contained 0.5
(b) [Fe(H 2 O) 6 ]2 has paramagnetism due to millimoles of R and 0.25 of S. The molarities of R
4 unpaired electrons. and S will be equal just at the end of time equal to
(a) twice the half life of R
(c) [Cr ( NH 3 ) 6 ]3 is paramagnetic. (b) twice the half life of S
(c) the half life of S
(d) [Co I 4 ]2 has square planar geometry.. (d) the half life of R
www.jeeneetbooks.in

WWW.IIT-NEET.XYZ

Mock Test 9 MT-97

53. The first order rate constant for a certain reaction (a) CH 3CH 2 – O O CH 2 CH 3 and
increases from 1.667 × 10 6 s 1 at 727ºC to 1.667 CH3
× 10–4 s–1 at 1571ºC. The rate constant at 1150ºC,
assuming constancy of activation energy over C6H5 C — O — OH respectively
the given temperature range is
(a) 3.911 × 10–5 s–1 (b) 1.139 × 10–5 s–1 CH3
(c) 3.318 × 10–5 s–1 (d) 1.193 × 10–5 s–1 (b) CH3CH2– O – O– CH2CH3 and
54. Law of mass action can be applied to aqueous CH3
solutions of weak electrolyte since-
(a) Reaction is always carried out in closed vessel. C6H5 CH respectively
(b) Pressure, temperature of surroundings re-
main constant. CH2 – O – OH
(c) State of equilibrium exist between the
ionised and unionised molecules OOH
|
(d) Nothing can be predicted (c) CH 3 C H – O – CH 2 CH 3 and
55. A 0.5 M NaOH solution offers a resistance of
31.6 ohm in a conductivity cell at room CH3
temperature. What shall be the approximate molar
conductance of this NaOH solution if cell C6H5 C — O — OH respectively
constant of the cell is 0.367 cm–1 . CH3
(a) 234 S cm2 mole–1 (b) 232 S cm2 mole–1
(c) 4645 S cm2 mole–1 (d) 5464 S cm2 mole–1 CH3
56. An electrochemical cell is set up as: Pt; H2 |
(1atm)|HCl(0.1 M) || CH3COOH (0.1 M)| H2 (1atm); (d) No reaction and C 6 H 5 C – OOH
|
Pt. The e.m.f of this cell will not be zero, because CH3
(a) the temperature is constant
(b) e.m.f depends on molarities of acids used respectively
(c) acids used in two compartments are different 59. Which of the following diols would cleave into
two fragments with HIO4
(d) pH of 0.1 M HCl and 0.1 M CH 3COOH is (a) 1, 3-hexanediol (b) 2, 4-hexanediol
not same (c) 1, 6-hexanediol (d) 3, 4-hexanediol
57. The half cell potential for the quinhydrone 60. One organic alcohol is irradiated with IR source
electrode and it produces three streching frequency such
O OH as 3391 cm–1, 2981 cm–1 and 1055 cm–1. Name of
that organic alcohol is
+ –
(a) Methanol (b) Ethanol
+ 2H + 2e (c) Isobutanol (d) Isopropanol
61. Which of the following reactions will not result
in the formation of anisole?
O (a) Phenol + dimethyl sulphate in presence of
OH
set up at pH = 4 will be (quinhydrone = 1 : 1 a base
molecular compound of quinone (Q) and (b) Sodium phenoxide is treated with methyl
hydroquinone (QH2), iodide
Eº = 0.699 V ) (c) Reaction of diazomethane with phenol
(a) 0.699 V (b) 0.463 V (d) Reaction of methylmagnesium iodide with
(c) 0.935 V (d) 0.817 V phenol
58. Predict the compounds A and B in the
OH
following reactions
h
CH 3CH 2 – O CH 2 CH 3 O2 A ; CHCl3 50% KOH
62. X
CH3 KOH
O2, 95 – 135º
C6H5CH B
CH3 The final product of this reaction is/are:
EBD_7443
www.jeeneetbooks.in

WWW.IIT-NEET.XYZ

MT-98 Target VITEEE


OH OH
(b) X is
CH
(a)
and Y is

OH OH
(c) Both are
CH2OH COOK
(b) +
(d) Both are
COOH COOK
(c) + 67. The condensation product of benzaldehyde and
acetone is
OH O O
||
C COOK (a) C 6 H 5 CH CH C CH 3
(d) (b) C 6 H 5CH C(CH 3 ) 2
63. In the reaction, O
||
C6 H5 OH
NaOH
(A)
CO 2
(B)
HCl
(C) : (c) C 6 H 5 CH 2 C CH CH 2
140 C,(4–7 atm)

The compound(C) is: O


||
(a) benzoic acid (b) salicylaldehyde (d) C H C CH CH.CH 3
6 5
(c) chlorobenzene (d) salicylic acid
68. During reduction of aldehydes with hydrazine
64. An ester (A) with molecular fomula, C9H10O2 was
and potassium hydroxide, the first is the
treated with excess of CH3MgBr and the complex formation of :
so formed was treated with H2SO4 to give an
olefin (B). Ozonolysis of (B) gave a ketone with (a) R — CH — — N — NH 2
molecular formula C8H8O which shows +ve (b) R —C N
iodoform test. The structure of (A) is
(a) C6H5COOC2H5 (c) R — C — NH 2
||
(b) C2H5COOC6H5 O
(c) H3COCH2COC6H5 R — CH—
(d) — NH
(d) p — H 3CO — C 6 H 4 — COCH 3
69. Which one of the following has maximum acid
65. The reagent (s) which can be used to distinguish strength?
acetophenone from benzophenone is (are) (a) p-nitrophenol
(a) 2,4- Dinitrophenylhydrazine (b) p-nitrobenzoic acid
(b) Aqueous solution of NaHSO3 (c) m-nitrobenzoic acid
(c) Benedict reagent (d) o-nitrobenzoic acid
(d) I2and Na2CO3. 70. Which of the following does not give benzoic
66. In the following reactions, X and Y are acid on hydrolysis?
(a) Phenyl cyanide (b) Benzoyl chloride
Y
(c) Benzyl chloride (d) Methyl benzoate
71. When succinic acid is heated, product formed is:
(a) X is (a) succinic anhydride
(b) acetic acid
(c) CO2 and methane
and Y is
(d) propionic acid
www.jeeneetbooks.in

WWW.IIT-NEET.XYZ

Mock Test 9 MT-99

72. Which of the following carboxylic acids 79. During the preparation of arenediazonium salts,
undergoes decarboxylation easily ? the excess of nitrous acid, if any is destroyed by
(a) C6H5CHOHCOOH adding.
(b) C6H5COCOOH (a) Aq NaOH (b) Aq Na2CO3
(c) C6H5COCH2COOH
(c) Aq NH2CONH2 (d) Aq KI
(d) C6H5COCH2CH2COOH
73. The reaction of HCOOH with conc. H2SO4 gives 80. A nitrogen containing organic compound gave
(a) CO2 (b) CO an oily liquid on heating with bromine and
(c) Oxalic acid (d) Acetic acid potassium hydroxide solution. On shaking the
74. If acetylchloride is reduced in the presence of product with acetic anhydride, an antipyretic
BaSO 4 and Pd, then drug was obtained. The reactions indicate that
the starting compound is :
(a) CH 3CHO is formed (a) aniline (b) benzamide
(b) CH 3COOH is formed (c) acetamide (d) nitrobenzene
(c) CH 3CH 2OH is formed PART - III (MATHEMATICS)
(d) CH 3COCH 3 is formed
81. If A is a non-zero column matrix of order m × 1
75. The treatment of acylazide (RCON3) with acidic and B is a non-zero row matrix of order 1× n, then
or alkaline medium gives rank of AB is equal to
(a) RCONH2 (b) R – NH2 (a) n (b) m
(c) RCH2 NH2 (d) RCOCHNH
(c) 1 (d) None of these
76. Which of the following compounds will form
alcohol on treatment with NaNO2, HCl/H2O at 82. The number of solutions of equation x 2 – x 3 = 1,
0°C? – x1 + 2 x 3 = 2, x1 – 2x 2 = 3 is
(a) (CH3)2CHNH2
(a) zero (b) one
(b) C6H5NH2 (b) two (d) infinite
(c) CH3 NH 2 83. Shaded region in the following figure is
represented by
Y
(d) H2N NH2
77. The refluxing of (CH3)2NCOCH3 with acid gives (0,20) x + y = 20
(a) 2 CH3NH2 + CH3COOH 20 , 40
C(10,16) B
3 3
(b) 2 CH3OH + CH3CONH2
(c) (CH3)2NH + CH3COOH 2x+5y = 80
(d) (CH3)2NCOOH + CH4
78. Which of the following reactions will not give N, X
A(20,0) (40,0)
N- dimethyl benzamide ?
(a) 2x 5y 80, x y 20, x 0, y 0
(a) CO.O.C 2 H5 (CH3 ) 2 NH
(b) 2x 5y 80, x y 20, x 0, y 0
(c) 2x 5y 80, x y 20, x 0, y 0
(b) CONH 2 CH 3Mg I (d) 2x 5y 80, x y 20, x 0, y 0

84. If | a b | |a b | then the vectors a


(c) COCl (CH 3 ) 2 NH
and b are adjacent sides of
(a) a rectangle (b) a square
(c) a rhombus (d) None of these
(d) CO.O.CO. 85. If three positive numbers a, b, c, are in H.P. then
(a) b 2 ac (b) a n c n 2 b n
+ (CH 3 ) 2 NH (c) Both correct (d) none correct
EBD_7443
www.jeeneetbooks.in

WWW.IIT-NEET.XYZ

MT-100 Target VITEEE


2 sin (c) a = 3 , b = 3
86. If = y,
1 cos sin (d) None of these
1 cos sin 92. The length intercepted by a line with direction
then = ratios 2, 7, –5 between the lines
1 sin
x 5 y 7 z 2 x 3 y 3 z 6
1 and is
(a) (b) y 3 1 1 3 2 4
y
(c) 1 – y (d) 1 + y (a) 75 (b) 78
87. If
(c) 83 (d) None of these
1 1
sin 1 sec 1(2) 2tan 1 sec 1 (5) 4 2 (cos x sin x)5
5 3 93. If value of lim is a 2 ,
1
x
4
1 sin 2x
1
sin 2 tan 1( 3) = k , then k = then the value of ‘a’ is
2
(a) 1 (b) 2 (a) 2 (b) 3
(c) 4 (d) 5 (c) 4 (d) 5
88. If the vectors 94. If z and are two non - zero complex numbers

aî ĵ k̂ , î bˆj k̂ , î ˆj ck̂ such that z 1 and Arg ( z ) Arg ( ) ,


2
are coplanar, where a 1, b 1, c 1 , then the then z is equal to
1 1 1 (a) –i (b) 1
value is (c) –1 (d) i
1 a 1 c 1 b
95. If z, z ane z are the vertices of a triangle,
(a) 1 (b) –1
(c) 2 (d) None then the area of the triangle will be (where is
89. A plane meets the coordinate axes in points A, cube root of unity) :
B, C and the centroid of the triangle ABC is 3 | z |2 3 3 | z |2
(a) (b)
( , , ) . The equation of the plane is 2 2

(a)
x y z
3 3 | z |2
(c) (d) None of these
2
(b) x y z 3 96. If sin x+ sin 2 x = 1, then the value of
x y z 1 cos12 x + 3 cos 10 x + 3 cos8 x + cos6 x – 1 is equal to
(c) (d) None of these (a) 0 (b) 1
2
(c) –1 (d) 2
90. A sphere is inscribed in a cube of side 2a. If the
cube is bounded by the coordinate planes as its 97. If cos 1 x cos 1
y cos 1
z then
three faces, then the equation of the sphere is x + y + z2 + 2xyz =
2 2

(a) 0 (b) 1
(a) x2 y2 z 2 2ax 2ay 2az a 2 0
3
(b) x2 y2 z 2 2ax 2ay 2az 2a 2 0 (c) (d) 2
2
(c) x2 y2 z 2 ax ay az a 2 0 98. A student is to answer 10 out of 13 questions
in an examination such that he must choose at
(d) x2 y2 z2 4a 2 least 4 from the first five questions. The
number of choices available to him is
91. Let a , b R , such that 0 <a < 1, 0 < b < 1. If the (a) 140 (b) 196
complex numbers z1 = – a + i, z2 = –1 + bi and (c) 280 (d) 346
z3 = 0 form an equilateral triangle, then values of 99. If a = 0 and b = 1, where a and b are two boolean
a and b are variables, then a + b =
(a) 1 (b) 0
(a) a = b = 2 3
(c) 1 or 0 (d) none
(b) a = 2 3 ,b= 2 3
www.jeeneetbooks.in

WWW.IIT-NEET.XYZ

Mock Test 9 MT-101

100. A ray of light is coming along the line which is 1 2 tan x 1


1
parallel to y-axis and strikes a concave mirror (a) tan C
22 2
whose intersection with the x – y plane is
parabola (x – 4)2 = 4 (y + 2). After reflection, the 1 1 2 tan x 1
ray must pass through the point (b) tan C
22 2
(a) (4, –1) (b) (0, 1)
(c) (–4, 1) (d) None of these 1 1 tan x 2
(c) tan C
101. The locus of the foot of the perpendicular drawn 22 2
from either focus upon any tangent to the ellipse
(d) None of these
2 2 108. The largest open intervals on which the given
x y
1 is
2
a a2 3
function f (x) x 3x 5 is concave upward
(a) x2 y2 a2 (b) x 2 y2 b2 or concave down, and also the location of a
points of inflection are
(c) x 2 y 2 a 2 b 2 (d) a 2 x 2 b 2 y 2 1 (a) (0, ), ( – ) and (0, –5)
102. The transverse axis of a hyperbola is of length (b) (– , ), ( , 0) and (–5, 10)
2a and a vertex divides the segment of the axis (c) (0, ), (0, 5) and (10, 20)
between the centre and the corresponding focus (d) (– , 0), (–5, ) and (10, 20)
in the ratio 2 : 1. If the transverse axis of the 109. The value of integral
hyperbola lies along x-axis and the centre is at /2
the origin then the equation of the hyperbola is (x)
dx is
(a) 5x2 – 4y2 = 5a2 (b) 5x2 – 4y2 = 4a2 0
(x) ( / 2 x)
2 2 2
(c) 4x – 5y = 5a (d) 4x2 – 5y2 = 4a2
(a) /4 (b) /2
103. A running track of 440 ft is to be laid out
enclosing a football field, the shape of which is a (c) (d) none of these
rectangle with a semi -circle at each end. If the 110. Area bounded by the curves y = |x| – 2 and
area of the rectangular portion is to be maximum y = 1 – |x–1| is equal to
then find the length of its sides. (a) 4 sq. units (b) 6 sq. units
(a) 44 ft, 70ft (b) 70 ft, 110 ft (c) 2 sq. units (d) 8 sq. units
(c) 88 ft, 44 ft (d) 135 ft, 70 ft 111. The relation R defined on set A = {x : |x| < 3,
104. On which of the following intervals is the x I} by R = {(x, y) : y = |x|} is
100 (a) {(–2, 2), (–1, 1), (0, 0), (1, 1), (2, 2)}
function x sin x 1 decreasing?
(a) (0, /2) (b) (0, 1) (b) {(–2, –2), (–2, 2), (–1, 1), (0, 0), (1, –2),
(c) ( /2, ) (d) None (1, 2), (2, –1), (2, –2)}
105. If the tangent at any point on the curve x4 + y4 = a4 (c) {(0, 0), (1, 1), (2, 2)}
cuts off intercepts p and q on the co-ordinate (d) None of these
axes then the value of p–4/3+q–4/3 is 112. Let f : R R be a function defined by
(a) a –4/3 (b) a –1/2 f (x) = max {x, x3}. The set of all points where f (x)
(c) a 1/2 (d) None is NOT differentiable is
(a) { 1, 1} (b) { 1, 0}
x2 3x 5
106. The domain of the function f(x) = (c) {0, 1} (d) { 1, 0, 1}
2
x – 5x 4 113. The order of the differential equation whose
is general solution is given by y = (c1 + c2) cos
(a) R (b) R – {1, 4} x c
(c) R – {1} (d) (1, 4) (x + c3) – c4 e 5 where c1, c2, c3, c4 and c5 are
arbitrary constants is
dx
107. Value of is (a) 5 (b) 4
4sin x 4sin x cos x 5cos2 x
2
(c) 3 (d) 2
EBD_7443
www.jeeneetbooks.in

WWW.IIT-NEET.XYZ

MT-102 Target VITEEE


114. The particular solution of the differential equation
27 5
(a) (b)
d2y dy 65 6
sin 1 1 x , where y 0 when
dx 2 dx 1
(c) (d) None
x = 0, is 6
(a) y x2 x sin x
PART - IV (ENGLISH)
2
x The strength of the electronics industry in Japan is
(b) y x sin x
2 the Japanese ability to organize production and
x2 x marketing rather than their achievements in original
(c) y sin x research. The British are generally recognized as a far
2 2
more inventive collection of individuals, but never seem
(d) 2y x 2 x sin x able to exploit what they invent. There are many
115. The composition table for a binary operation * examples, from the TSR Z hovercraft, high speed train
defined on a set S is symmetric about the leading and Sinclair scooter to the Triumph, BSA and Norton
diagonal, then Motorcycle which all prove this sad rule. The Japanese
(a) * is associative on S were able to exploits their strengths in marketing and
(b) * is commutative on S
(c) S has the identity element for * development many years ago, and their success was
(d) none of these at first either not understood in the West or was
dismissed as something which could have been
116. If X {8n 7n 1 : n N} produced only at their low price. They were sold
and Y {49 (n 1):n N} then: because they were cheap copies of other people's ideas
(a) X Y (b) Y X churned out of a workhouse which was dedicated to
(c) X = Y (d) X Y = N hard grind above all else.
117. There are 4 letters and 4 directed envelopes. The
number of ways in which all the letters can be put 121. It is evident from the passage that the strength
in wrong envelope is of a country's industry depends upon
(a) 9 (b) 4 (a) original research
(c) 5 (d) 12 (b) international cooperation
118. Suppose that X has a Poisson distribution. If (c) dedicated workforce
P(X 1) 0.3 , and P(x 2) 0.2 then (d) electronic development
P(X = 0) = 122. The sad rule mentioned in this passage refers to
(a) e 1/3 (b) e –3/2 (a) the inability of the Japanese to be inventive
(c) e –4/3 (d) 0.5 like the British
119. A fair coin is tossed a fixed number of times. If (b) the inability of the British to be industrious
the probability of getting seven heads is equal like the Japanese
to that of getting nine heads, the probability of (c) the lack of variety in Japanese inventions
getting two heads is
(d) the poorer marketing ability of British
15 2 123. According to the passage, prosperity in industry
(a) (b)
28 15 depends upon
15 (a) productivity (b) inventiveness
(c) (d) None (c) marketing ability (d) official patronage
213 124. Select the word which is most nearly the opposite
120. The probabilities of four cricketers A, B, C and D
in meaning to the word in capital letters.
1 1 PERTINACIOUS
scoring more than 50 runs in a match are , ,
2 3 (a) Irretrievable (b) Insipid
1 1 (c) Irresolute (d) Reproof
and . It is known that exactly two of the 125. Choose the best pronunciation of the word,
4 10
players scored more than 50 runs in a particular Violin, from the following options.
match. The probability that these players were A (a) vo-ee-lin (b) vaeo-lin
and B is (c) vaio-line (d) va-yuh-lin
www.jeeneetbooks.in

WWW.IIT-NEET.XYZ

MOCK
VITEEE Mock Test Paper
10
Max. Marks : 125 Time : 2½ hrs

PART - I (PHYSICS) 2q
(a) 2q, 4q (b) , 4q
3
1. The plate of a parallel plate capacitor are
separated by d cm. A plate of thickness t cm. 2q 4q 4q
(c) , (d) 2q,
with dielectric constant k1 is inserted and the 3 3 3
remaining space is field with a plate of dielectric 4. An uncharged capacitor with a solid dielectric is
constant k2. If Q is the charge on the capacitor connected to a similar air capacitor charged to a
and area of plates is A cm2 each, then potential potential of V0. If the common potential after
difference between the plates is – sharing of charges becomes V, then the dielectric
constant of the dielectric must be –
Q t d t V0 V
(a) A k k2 (a) (b)
0 1 V V0

V0 V V0 V
4 Q t d t (c) (d)
(b) V V0
A k1 k2
5. Two long coaxial and conducting cylinders of
radius a and b are separated by a material of
4 Q k1 k2 conductivity and a constant potential
(c)
A t d t difference V is maintained between them, by a
battery. Then the current, per unit length of the
Q k1 d t cylinder flowing from one cylinder the other is –
(d) 4 4
0A t k2
V
(a) ln (b / a) V (b)
(b a)
2. A ball of mass m moving with a constant velocity
strikes against a ball of same mass at rest. If e = 2 2
(c) V (d) V
coefficient of restitution, then what will be the ln (b / a) (b a)
ratio of velocity of two balls after collision? 6. A cell of emf E having an internal resistance ‘r’ is
1 e e 1 connected to an external resistance R. The
(a) (b) potential difference ‘v’ across the resistance R
1 e e 1
varies with R as shown by the curve :
1 e 2 e
(c) (d) V
1 e e 1
3. An infinite number of charges, each equal to q E A
B
coulomb, are placed along the x-axis at x (in
metres) = 1, 2, 4, 8, ...... and so on. The potential E/2
and field in SI units at x = 0 due to this set of
D
1 R
charges are respectively times (a) A (b) B
4 0 (c) C (d) D
EBD_7443
www.jeeneetbooks.in

WWW.IIT-NEET.XYZ

MT-104 Target VITEEE


7. Figur e shows a
wire of uniform
resistance between r
A and B with a total I
resistance of R0. A A
The contact C can
(0, b)
A fR0 (1-f)R0 B
divide the wire into
resistances f R 0 C
and (1 – f ) R0. (0,0) (a,0)
The batteries are identical and have zero internal
resistances. 0I b
(a) 1
The value of f for which ammeter reads maximum 4 a a 2
b2
is:
(a) 0 (b) 1
0I b
(c) 0 or 1 (d) cannot be found (b) 1
4 a a 2
b2
8. A battery of internal resistance 2 is connected
to a variable resistor whose value can vary from
4 to 10 . The resistance is initially set at 4 . If 0I b
(c) 1
the resistance is now increased then – 4 a a 2
b2
(a) power consumed by it will decrease
(d) None of these
(b) power consumed by it will increase 12. A straight wire B
(c) power consumed by it may increase or may current element is
decrease carrying current
(d) power consumed will first increase then 100A, as shown in i
decrease. figure.
9. Two electric bulbs rated P1 watt V volts and P2 The magnitude
watt V volts are connected in parallel and applied field at point P A
across V volts. The total power (in watts) will be which is at
(a) P1 + P2 (b) P1 P2 per p en di cu la r 60°
d i s t a n c e
30°
P1 P2 P1 P2 ( 3 1) m from the
(c) (d) P
P1 P2 P1 P2 3 1m
current element if
10. If a ball of steel (density = 7.8 g cm–3) attains a end A and end B of the element subtend angle
terminal velocity of 10 cms–1 when falling in a 30° and 60° at point P, as shown is
tank of water (coefficient of viscosity (a) 2.5 × 10–6 T (b) 5 × 10–6 T
4 (c) 2.5 × 10–5 T (d) 8 × 10–5 T
water 8.5 10 Pa s ) then its terminal
13. In the figure shown a
velocity in glycerine (r = 12 gcm–3 , water = 8.5 × conducting ring of radius
10–4 Pa-s) would be nearly a is placed in a uniform
(a) 1.6 × 10–5 cms–1 and constant magnetic
(b) 6.25 × 10–4 cms–1 field of induction B, with
(c) 6.45 × 10–4 cms–1 its plane perpendicular to
(d) 1.5 × 10–5 cms–1
B . The ring is made to
11. An infinitely long wire carrying current I is along
rotate with constant angular speed about the
Y axis such that its one end is at point A (0, b)
diameter YY'. The emf induced in the ring is :
while the wire extends upto + The magnitude
(a) a2B sin t (b) a2B cos t
of magnetic field strength at point (a, 0) is 2
(c) a B (d) zero
www.jeeneetbooks.in

WWW.IIT-NEET.XYZ

Mock Test 10 MT-105

14. A coil of area 7 cm2 and of 50 turns is kept with 18. The equation of alternating current is :
its plane normal to a magnetic field B. A
I 50 2 sin 400 t amp. Then the frequency
resistance of 30 ohm is connected to the
resistance-less coil. B is 75 exp (– 200t) gauss. and root mean square of current are respectively
(a) 200 Hz, 50 amp (b) 400 Hz, 50 2 amp
The current passing through the resistance at
(c) 200 Hz, 50 2 amp (d) 50 Hz, 200 amp
t = 5 ms will be-
19. In an L-R circuit, the value of L is (0.4/ ) henry
(a) 0.64 mA (b) 1.05 mA
and the value of R is 30 ohm. If in the circuit, an
(c) 1.75 mA (d) 2.60 mA alternating emf of 200 volt at 50 cycles per second
15. An air-plane with 20 m wing spread is flying at is connected, the impedance of the circuit and
250 ms-1 straight south parallel to the earth’s current will be :
surface. The earth’s magnetic field has a (a) 11.4 ohm, 17.5 ampere
horizontal component of 2 × 10–5 Wb m–2 and (b) 30.7 ohm, 6.5 ampere
the dip angle is 60º. Calculate the induced emf (c) 40.4 ohm, 5 ampere
between the plane tips is: (d) 50 ohm, 4 ampere.
(a) 0.174 V (b) 0.173 V 20. In the LCR series circuit, the voltmeter and
(c) 1.173 V (d) 0.163 V ammeter readings are –
16. A conducting rod is rotated in a plane 400V 400V
perpendicular to a uniform magnetic field with V
constant angular velocity. The correct graph
between the induced emf (e) across the rod and
time (t) is : L
C
e

t
(a)
100V,50Hz
A ~
e A.C. generator
(a) E = 100 volts, I = 2 amp
t (b) E = 100 volts, I = 5 amp
(b)
(c) E = 300 volts, I = 2 amp
(d) E = 300 volts, I = 5 amp
e 21. The energy of electromagnetic wave in vacuum
is given by the relation
(c) t
E2 B2 1 2 1
(a) (b) 0E µ0B2
2 0 2µ 0 2 2
e
E2 B2 1 2 B2
(c) (d) 0E
(d) t c 2 2µ0
22. Interference fringes were produced using white
17. An alternating voltage is given by: light in a double slit arrangement. When a mica
sheet of uniform thickness of refractive index 1.6
e e1 sin t e 2 cos t . Then the root mean (relative to air) is placed in the path of light from
square value of voltage is given by one of the slits, the central fringe moves through
some a distance. This distance is equal to the
(a) e12 e 22 (b) e1e 2 width of 30 interfernece bands if light of
wavelength 4800 Å is used. The thickness (in
e1e 2 e12 e22 µm) of mica is –
(c) (d) (a) 90 (b) 12
2 2
(c) 14 (d) 24
EBD_7443
www.jeeneetbooks.in

WWW.IIT-NEET.XYZ

MT-106 Target VITEEE


23. The first diffraction minima due to a single slit 28. The energies of energy levels A, B and C for a
diffraction is at =30° for a light of wavelength given ato are in the sequence EA < EB < EC. If the
5000 Å. The width of the slit is – radiations of wavelengths and are
(a) 5 × 10–5 cm. (b) 10 × 10–5 cm. emitted due to the atomic transitions C to B, B to
–5 A and C to A respectively then which of the
(c) 2.5 × 10 cm. (d) 1.25 × 105 cm.
following relations is correct ?
24. Two Nicol prisms are first crossed and then one
(a) + = 0 (b) =
of them is rotated through 60°. The percentage
of incident light transmitted is – 1 2
(c) (d) = .
(a) 1.25 (b) 25.0 1 2
(c) 37.5 (d) 50 29. In a photoelectric effect measurement, the
25. If radiation corresponding to first line of “Balmer stopping potential for a given metal is found to
series” of He+ ion knocked out electron from 1st be V0 volt when radiation of wavelength 0 is
excited state of H atom, the kinetic energy of used. If radiation of wavelength 2 0 is used with
ejected electron from H atom would be (eV) – the same metal then the stopping potential (in
volt) will be
Z2
(Given E n (13.6 eV) V0
n2 (a) (b) 2 V0
2
(a) 4.155 eV (b) 8.310 eV
(c) 2.515 eV (d) 5.550 eV hc hc
(c) V0 (d) V0
26. Cathode rays moving with same velocity v describe 2e 0 2e 0
an approximate circular path of radius r metre in an 30. The electrons can not exist inside the nucleus
electric field of strength x volt metre–1. If the speed because
of the cathode rays is doubled to 2v, the value of (a) de-Broglie wavelength associated with
electric field needed so that the rays describe the electron in -decay is much less than the
same approximate circular path (volt metre–1) is size of nucleus
(a) 2 x (b) 3 x (b) de-Broglie wavelength associated with
(c) 4 x (d) 6 x electron in -decay is much greater than
27. Two identical photocathodes receive light of the size of nucleus
frequencies f1 and f2 . If the velocites of the photo (c) de-Broglie wavelength associated with
electrons (of mass m ) coming out are electron in -decay is equal to the size of
nucleus
respectively v1 and v 2 , then
(d) negative charge can not exist in the
2h nucleus
(a) v12 v 22 (f1 f 2 )
m 31. A force F is applied to a block of mass 2 3 kg
as shown in the diagram. What should be the
1/ 2 maximum value of force so that the block does
2h
(b) v1 v 2 ( f1 f 2 ) not move ?
m
F
(c) 2 2 2h
v1 v2 (f1 f 2 )
m 60° =
1
2 3
1/ 2
2h
(d) v1 v 2 (f1 f 2 ) (a) 10 N (b) 20 N
m
(c) 30 N (d) 40 N
www.jeeneetbooks.in

WWW.IIT-NEET.XYZ

Mock Test 10 MT-107

32. Horizontal tube of non-uniform cross-section has (a) – 487.5 Vin (b) – 417.5 Vin
radii of 0.1 m and 0.05 m respectively at M and N (c) – 237.5 Vin (d) – 127.5 Vin
for a streamline flow of liquid the rate of liquid 38. By adding indium at the rate of one indium atom
flow is per 4 × 108 germanium atoms, a sample of
N germanium atom is made p material. The donor
density is assumed to be zero and
ni = 2.5 × 1019 per cubic metre at 300K. If there
are 4.4× 1028 germanium atoms per cubic metre,
M find the ratio np : ni.
(a) continuously changes with time (a) 0.22 (b) 0.51
(b) greater at M than at N (c) 0.31 (d) 0.78
(c) greater at N than at M 39. An n-type silicon sample of width 4 × 10–3 m,
(d) same at M and N thickness 25 × 10–5 m and length 6 × 10–2 m
33. A radioactive nuclide is produced at the constant carries a current of 4.8mA when the voltage is
rate of n per second (say, by bombarding a target applied across the length of the sample. What is
with neutrons). The expected number N of nuclei the current density ? If the free electron density
in existence t seconds after the number is N0 is
is 1022/m3 then find how much time does it take
given by
for the electron to travel the full length of the
(a) t
N N0e sample ?
(a) 4.8×103 A/m2, 2×10–2s
n t
(b) N N 0e (b) 1.8×103 A/m2, 2×10–2s
(c) 3.3×103 A/m2, 2.2×10–2s
n n t (d) 5.2×103 A/m2, 4 × 10–2s
(c) N N0 e
40. A material has poisson’s ratio 0.50. If a uniform
rod of it suffers a longitudinal strain of 2 × 10–3,
n n t then the percentage change in volume is
(d) N N0 e
(a) 0.6 (b) 0.4 (c) 0.2 (d) Zero
where is the decay constant of the sample
34. A force of 10 N acts on a body of mass 20 kg for
PART - II (CHEMISTRY)
10 seconds. Change in its momentum is 41. The variation of equivalent conductance of
(a) 5 kg m/s (b) 100 kg m/s strong electrolyte with (concentration) ½ is
(c) 200 kg m/s (d) 1000 kg m/s represented by
35. In the nuclear reaction
198
80 Hg X 79 Au198 1H1 . X-stands for
(a) proton (b) electron (a) (b)
(c) neutron (d) deutron
c c
36. A transistor is connected in CE configuration.
The voltage drop across the load resistance (RC)
3 k is 6V. Find the base current. The current
gain of the transistor is 0.97 is
(a) 61.86 µA (b) 51.86 µA (c) (d)
(c) 42.16 µA (d) 70.16 µA c c
37. Find the output of the circuit given below 42. The reaction of C6H5CH = CHCHO with NaBH4
740 k
650 k
gives
10 k
(a) C6H5CH2CH2CH2OH
– 100 k

(b) C6H5CH = CHCH2OH
Vin +
+
Vo (c) C6H5CH2CH2CHO
(d) C6H5CH2CHOHCH3
EBD_7443
www.jeeneetbooks.in

WWW.IIT-NEET.XYZ

MT-108 Target VITEEE


43. Which of the following compound will undergo 49. Which of the following gives primary amine on
self aldol condensation in the presence of cold reduction?
dilute alkali ? (a) CH 3CH 2 NO 2
(a) CH 2 CH CHO (b) CH 3CH 2 O N O
(b) CH C CHO
(c) C6H 5 N NC 6 H 5
(c) C 6 H 5CHO
(d) CH 3CH 2 NC
(d) CH 3CH 2CHO. 50. Conductivity (unit Siemen's S) is directly
44. CsBr crystallises in a body centered cubic lattice. proportional to area of the vessel and the
The unit cell length is 436.6 pm. Given that the concentration of the solution in it and is
atomic mass of Cs = 133 and that of Br = 80 amu inversely proportional to the length of the
and Avogadro number being 6.02 × 1023 mol–1, vessel, then th e unit of constant of
the density of CsBr is proportionality is
(a) 0.425 g/cm3 (b) 8.50 g/cm3 (a) S m mol–1 (b) S m2 mol–1
–2
(c) S m mol2 (d) S2 m2 mol–2
(c) 4.25 g/cm 3 (d) 42.5 g/cm3
45. In the anion HCOO– the two carbon-oxygen 51. In an irreversible process taking place at constant
bonds are found to be equal length. What is the T and P and in which only pressure-volume work
reason for it? is being done, the change in Gibbs free energy
(a) electronic orbitals of carbon atom are (dG) and change in entropy (dS), satisfy the
hybridised criteria
(b) the C=O bond is weaker than the C–C bond (a) (dS)V, E > 0, (dG)T, P < 0
(c) the anion HCOO– has two reasonating (b) (dS)V, E = 0, (dG)T, P = 0
structures (c) (dS)V, E = 0, (dG)T, P > 0
(d) the anion is obtained by removal of a proton (d) (dS)V, E < 0, (dG)T, P < 0
from the acid molecule. 52. Bordeaux used as fungicide is a mixture of
46. Identify X, (a) CuSO 4 Ca OH 2
H3C (b) CaSO 4 Cu OH
CH3MgI 2
C=O
H3C dry ether (c) CuCO3 Cu OH 2
(d) CuO CaO
H 2O
Intermediate X 53. The conductivity of a saturated solution of
(a) CH3OH (b) Ethyl alcohol BaSO4 is 3.06 × 10–6 ohm –1 cm –1 and its
equivalent conductance is 1.53 ohm –1 cm 2
(c) Methyl cyanide (d) tert-Butyl alcohol equiv–1. The Ksp for BaSO4 will be
47. The half-life of Th 232 is 1.4 × 1010 years and that (a) 4 × 10–12 (b) 2.5 × 10–9
of its daughter element Ra228 is 7 years. Calculate (c) 2.5 × 10 –13 (d) 4 × 10–6
the weight of radium228 in equilibrium with 1 gram 54. In a uranium mineral, the atomic ratio
of Th232. NU–238/NPb–206 is nearly equal to one. The age
(a) 6 × 10–12 (b) 8.4 × 10–12 (in years) of the mineral is nearly (given that the
–10
(c) 5 × 10 gm (d) 3 × 10–10 gm half-life of U-238 is 4.5 × 109 years)
48. When 1 mol of a gas is heated at constant (a) 3.0 × 108 (b) 4.5 × 108
volume, temperature is raised from 298 to 308 K. (c) 3.0 × 10 9 (d) 4.5 × 109
If heat supplied to the gas is 500 J, then which 55. The reaction L M is started with 10.0 g of
statement is correct ? L. After 30 and 90 minutes 5.0 g and 1.25 g of
(a) q = w = 500 J, U = 0 L respectively are left. The order of the reaction
(b) q = U = 500 J, w = 0 is
(c) q = –w = 500 J, U = 0 (a) 0 (b) 1
(d) U = 0, q = w = –500 J (c) 2 (d) 3
www.jeeneetbooks.in

WWW.IIT-NEET.XYZ

Mock Test 10 MT-109

56. Metallothermic processes involving Lanthanides 63. For the reaction


are called as CH 4 (g) 2O 2 (g) CO2 (g) + 2H2O(l),
(a) Aluminothermic process –1
(b) Lanthanido-thermic process rH = –170.8 k J mol
(c) Reduction process Which of the following statements is not true ?
(d) Oxidation process (a) The equilibrium constant for the reaction is
57. Ethanol when reacted with PCl5 gives A, POCl3 [CO 2 ]
and HCl. A reacts with silver nitrite to form B given by K p [CH 4 ][O 2 ]
(major product) and AgCl. A and B respectively
are (b) Addition of CH4 (g) or O2 (g) at equilibrium
(a) C2H5Cl and C2H5OC2H5 will cause a shift to the right
(b) C2H6 and C2H5OC2H5 (c) The reaction is exothermic
(c) C2H5Cl and C2H5NO2 (d) At equilibrium, the concentrations of CO2
(d) C2H6 and C2H5NO2 (g) and H2O(l) are not equal
58. A compound of the formula C4H10O reacts with
sodium and undergoes oxidation to give a 64. CH 3
carbonyl compound which does not reduce
Tollen’s reagent, the original compound is
(a) Diethyl ether (b) n-Butyl alcohol
(c) Isobutyl alcohol (d) sec-Butyl alcohol CH 3 CH 3
59. If Germanium crystallises in the same way as
diamond, then which of the following statement The above compound describes a condensation
is not correct polymer which can be obtained in two ways :
(a) Every atom in the structure is tetrahedrally either treating 3 molecules of acetone
bonded to 4 atoms. (CH3 COCH3) with conc. H2SO4 or passing
(b) Unit cell consists of 8 Ge atoms and co- propyne (CH3 C CH) through a red hot tube.
ordination number is 4. The polymer is
(c) All the octahedral voids are occupied. (a) Phorone
(d) All the octahedral voids and 50% tetrahedral (b) Mesityl oxide
voids remain unoccupied. (c) Deacetonyl alcohol
60. Which of the following cannot be made by using (d) Mesitylene.
Williamson’s synthesis? 65. The pair in which both species have same
(a) Methoxybenzene magnetic moment (spin only value) is :
(b) Benzyl p-nitrophenyl ether (a) [Cr(H2O)6]2+ , [CoCI4]2–
(c) Methyl tertiary butyl ether
2 3
(d) Di-tert-butyl ether (b) [Cr(H 2O)6 )] , [Fe(H 2O)6 ]
61. The atomic numbers of vanadium (V), chromium
(Cr), manganese (Mn) and iron (Fe) are (c) [Mn (H 2 O) 6 ) 2 , [Cr(H 2 O) 6 ]2
respectively 23, 24, 25, and 26. Which one of
(d) [CoCl 4 ) 2 , [Fe(H 2 O) 6 ]2
these may be expected to have the highest
second ionization enthalpy ? 66. Which one of the following pairs is not correctly
(a) Cr (b) Mn matched ?
(c) Fe (d) V (a) C O CH 2
62. The following reaction is known as : (Clemmensen 's reduction)
OH OH
anhy. (b) C O CHOH
HCl HCN
ZnCl 2
CHO (Wolf -Kishner 's reduction)
(a) Perkin reaction (c) COCl CHO
(b) Gattermann reaction
(Rosenmund 's reduction)
(c) Kolbe reaction
(d) Gattermann-aldehyde reaction (d) C N HO (Stephen 's reduction)
EBD_7443
www.jeeneetbooks.in

WWW.IIT-NEET.XYZ

MT-110 Target VITEEE


67. A drop of a solution (volume = 0.05 ml) contains 73. Which of the following has Frenkel defects?
6 10 7 mol of H+. If rate of disapperanace of (a) Sodium chloride (b) Graphite
H+ is 6.0 × 105 mol lit–1 s–1, how long will it take (c) Silver bromide (d) Diamond
for the H+ in the drop to disappear ?
74. Which of the following will give maximum
2 8
(a) 2.0 10 s (b) 2 .0 10 s number of isomers?
(a) [Co(NH3)4Cl2] (b) [Ni(en)(NH3)4]2+
(c) 8.0 10 8 s (d)
6.0 10 s 6
(c) [Ni(C2O4)(en)2] (d [Cr(SCN)2(NH3)4]2+
68. The weakest acid among the following is
75. 232 is the starting material of decay of chain
(a) CHCl2COOH (b) BrCH2COOH 90 Th
(c) ClCH2COOH (d) FCH2COOH series. This series of successive decays
69. A compound of a metal ion M x Z 24 has a terminates at 82208
Pb . The number of and
particles emitted in this process is
spin only magnetic moment of 15 Bohr
(a) 6 and 4 (b) 4 and 6
Magnetons. The number of unpaired electrons
in the compound are (c) 10 and 6 (d) 6 and 10
(a) 2 (b) 4 76. Which of the following shows oxidation state
(c) 5 (d) 3 of +8 ?
C N (a) Ir (b) Pt
+ (c) Ru (d) Os
H3 O
70. + CH3MgBr Q P 77. An electrochemical cell is set up as follows :
Pt (H2, 1 atm)/0.1 M HCl/0.1 M acetic acid/(H2, 1
OCH3
atm) Pt EMF of this cell will not be zero because
The product ‘P’ in the above reaction is
(a) the temperature is constant
OH (b) the pH of 0.1 M HCl and 0.1 M acetic acid is
CH CO CH3
not the same
CH3
(c) acids used in the two compartments are
(a) (b) different
OCH3 (d) EMF of a cell depends on molarities of the
acids used
CHO COOH
78. An organic compound ‘A’ has the molecular
formula C3H6O. It undergoes iodoform test.
(c) (d) When staturated with HCl it gives ‘B’ of
OCH3 OCH3 molecular formula C 9 H 14 O. ‘A’ an d ‘B’
71. The reaction of acetyl chloride on benzene in respectively are
the presence of AlCl3 gives (a) Propanal and mesitylene
(a) C6 H5Cl (b) C6 H5COCH3 (b) Propanone and mesityl oxide
(c) (c) Propan one and 2,6-dimeth yl-2,
CH3 CO CH3 (d) C 6 H 4 Cl2
5-heptadien-4-one
72. Aniline reacts with phosgene and KOH to form
(d) Propanone and mesitylene oxide
OH O
79. Which of the following is not formed when
C Cl glycerol reacts with HI?
(a) (b)
(a) CH2 = CH – CH2I
(b) CH2OH–CHI–CH2OH
NHCOCl NCO (c) CH3–CH = CH2
(c) (d)
(d) CH3–CHI–CH3
www.jeeneetbooks.in

WWW.IIT-NEET.XYZ

Mock Test 10 MT-111

80. The factor of G values is important in a 2x


metallurgy. The G values for the following 85. 51+x + 51–x, , 5 + 5–2x are in A.P., then the
2
reactions at 800ºC are given as :
value of a is:
S 2 (s ) 2O 2 ( g ) 2SO 2 ( g ) ; G =–544 kJ (a) a < 12 (b) a 12
(c) a 12 (d) None of these
2 Zn (s ) S 2(s) 2 ZnS (s ) ; G = –293 kJ
86. Number of different 3-letter words can be formed
with the letter of word 'JAIPUR' when A and I are
2 Zn (s ) O 2( g ) 2 ZnO (s ) ; G = – 480 kJ
always to be excluded is
Then G for the reaction : (a) 24 (b) 4
2 ZnS (s ) 3O 2 ( g ) 2 ZnO (s ) 2SO 2( g ) (c) 20 (d) 10
87. If A and B are two matrices and (A + B) (A – B) =
will be : A2 – B2, then
(a) –357 kJ (b) –731 kJ (a) AB = BA (b) A2 + B2 = A2 – B2
(c) –773 kJ (d) –229 kJ (c) A'B' = AB (d) none of these
88. If A and B are any two square matrices of the
PART - III (MATHEMATICS) same order, then
(a) (AB) A B
81. If z = rei , then |eiz| equals (b) adj(AB) = adj(A) adj(B)
(a) er sin (b) e–r sin (c) (AB) B A
(c) e –r cos (d) er cos
(d) AB = 0 A = O or B = O
82. The equation of two lines through the origin,
x 3 y 3 z cos (90 ) sec ( ) tan (180 )
which intersect the line at 89. sin (360 )sec (180 ) cot (90 )
2 1 1
equals
angles of each, are
3 (a) 2 (b) 1
x y z x y z (c) –1 (d) 0
(a) ; 90. Let the angles A, B, C of ABC be in A.P. and let
1 2 1 1 1 2
x y z x y z b : c = 3 : 2 . Then angle A is
(b) ;
1 2 1 1 1 2 (a) 75° (b) 45°
x y z x y z (c) 60° (d) none of these
(c) ;
1 2 1 1 1 2 91. Solution of y cos 1 ( ( 3 / 2)) for y, where
(d) None of the above
0 y is
1 i (a) 5 /4 (b) /6
83. If = A + iB, then A2 + B2 equals to (c) /2 (d) 5 /6
1 i
(a) 1 (b) 2 4 1
1 1
92. If sin sin and
(c) –1 (d) – 2 1
5 3
84. Which of the function defined below is one to
1 4 11
one? 2 cos cos then
5 3
(a) f : (0, ) R , f(x) = x2 – 4x + 3
(a) 1 2 (b) 1 2
(b) f : [0, ) R , f (x) = x2 + 4x – 5
(c) 1 2 (d) none of these
x 1 93. Value of for which the line y = x + touches the
(c) f : R R , f (x) = e
ex ellipse 9x2 + 16y2 =144, is
(a) ± 5 (b) ± 2
(d) f : R R, f (x) n(x 2 x 1) (c) ± 8 (d) ± 7
EBD_7443
www.jeeneetbooks.in

WWW.IIT-NEET.XYZ

MT-112 Target VITEEE


94. The point (at2 , 2bt) lies on the hyperbola, 100. Ten persons, amongst whom are A, B and C to
2 2 speak at a function. The number of ways in
x y
1 for which it can be done if A wants to speak before
2
a b2 B and B wants to speak before C is
(a) all values of t (b) t2 2 5 10!
(a) (b) 3! 7!
(c) t 2 2 (d) no real value of t. 6
5 10 .
(c) P3 7! (d) None of these
95. The area of the rectangle formed by the 101. The inverse of the function
perpendiculars from the centre of the ellipse
ex e x
x2 y2 f (x) 2 is
+ = 1 to the tangent and normal at a ex e x
a2 b2
1/ 2 1/2
x 3 x 1
point whose eccentric angle is , is (a) loge (b) loge
4 x 1 3 x
1/ 2 1/ 2
a2 b 2 ab a2 b2 ab x 2 x 1
(c) loge (d) loge
(a) (b) x 3 x 2
a2 b2 a2 b2 102. Area bounded by the circle x2 + y2 = 1 and the
curve | x | + | y | = 1 is
a2 b2 a2 b2
(c) (d) (a) 2 (b) – 2
ab a 2 b2 ab a 2 b2 (c) (d) + 3
96. For all values of x, function f (x) = 2x3 + 6x2 cosax cos bx m
+ 7x – 19 is 103. lim
x 0 cos cx 1 is equal to , where m and
n
(a) monotonic increasing n are respectively
(b) monotonic decreasing (a) a2 + b2, c2 (b) c2, a2 + b2
(c) not monotonic (c) a – b , c
2 2 2
(d) c2, a2 – b2
(d) none of these
x 3 z 2 (z x) 2
x 104. lim is equal to
sin ,0 x 1 x 0 3 4
97. f (x) = 2 , then 8xz 4x 2 3
8xz
3 2x, x 1
(a) f (x) has a local minimum at x = 1 z 1
(b) f (x) has a local maximum at x = 1 (a) 11 / 3 (b) 23 / 3
2 2 .z
(c) f (x) does not have any local maximum or
minimum at x = 1 (c) 2 21 / 3
z (d) None of these
(d) f (x) has a global minimum at x = 1 dx
98. If the points (1, 1, p) and (–3, 0, 1) be equidistant 105. The intergral is equal to
x 1 x3
from the plane r. 3iˆ 4j 12kˆ 13 0 , then the
value of p is 1 1 x3 1
(a) log C
3 7 3 1 x3 1
(a) (b)
7 3
1 1 x3 1
4 3 (b) log C
(c) (d) 3 1 x3 1
3 4
99. Let f '(x) < 0 and g' (x) > 0 for all real x, then
2 1
(a) f (g(x + 1)) > f (g (x + 5)) (c) log C
3 1 x3
(b) f (g(x)) < g (f(x +2))
(c) g (f(x)) < g (f (x +2)) 1
(d) log | 1 x 3 | C
(d) g (f (x)) > g(f (x – 2)) 3
www.jeeneetbooks.in

WWW.IIT-NEET.XYZ

Mock Test 10 MT-113

106. A function f (x) which satisfies the relation


1 113. If a , b and c are unit coplanar vectors, then
f (x) e x x
e f (x) dt , then f (x) is the scalar triple product
0
2a b, 2 b c, 2 c a
ex
(a) (b) (e – 2) ex
2 e (a) 0 (b) 1
(c) 2ex (d) ex/2 (c) (d)
3 3
107. The circle x2 + y2 = a2 is revolved about x-axis. 114. If p, q, r are statement with truth values
Then the volume of the sphere so formed is F, T, F respectively then the truth value of
(~ p ~ q) r is
4 2 4 (a) true (b) false
(a) a cub. units (b) a cub. units
3 3 (c) false if r is true (d) false if q is false
4 3 115. The mean and the variance of a binomial
(c) a cub. units (d) none of these distribution are 4 and 2 respectively. Then the
3
probability of 2 successes is
108. The solution of differential equation
28 219
dy (a) (b)
2x – y = 3 represents a family of 256 256
dx
(a) circles (b) straight lines 128 37
(c) ellipses (d) parabola (c) (d)
256 256
109. The general solution of the differential equation 116. A random variable X has Poisson distribution
dy with mean 2. Then P (X > 1.5) equals
(ex +e–x) = (ex – e–x) is
dx 2
(a) y = log |ex + e–x| + c (a) (b) 0
e2
(b) y = log | ex – e–x | – c
(c) y = – log | ex –e–x| +c 3 3
(d) none of these (c) 1 (d) 2
2 e
e
110. If vectors a iˆ ˆj kˆ , b iˆ 2 ˆj kˆ and 117. The normal at the point (at 12, 2at 1) on the
parabola, cuts the parabola again at the point
c 3iˆ pjˆ 5kˆ are coplanar, then the value of whose parameter is
p is 2 2
(a) t2 t1 (b) t2 t1
(a) 2 (b) 6 t1 t1
(c) – 2 (d) – 6
2
111. The angle between the lines 2x = 3y = –z and (c) t2 t1 (d) None of these
t1
6x = –y = –4z, is
(a) 0° (b) 30° 118. For real numbers x and y, wewrite x R y x – y + 2
(c) 45° (d) 90° is an irrational number. Then, the relation R is
112. Six dice are thrown. The probability that different (a) Reflexive (b) Symmetric
numbers will turn up is (c) Transitive (d) None of these
119. The set Z' of all non-negative integers under
129 1
(a) (b) addition is
1296 54 (a) a monoid (b) not a monoid
5 5 (c) semi-group (d) none
(c) (d)
324 54
EBD_7443
www.jeeneetbooks.in

WWW.IIT-NEET.XYZ

MT-114 Target VITEEE


120. Consider Max. z = – 2x – 3y subject to 122. We did not mind the hardness of road because
x y x y (a) we had nailed boots on
1, 1 , x, y 0
2 3 3 2 (b) it was good walking on the road
The max value of z is : (c) the walk was refreshing
(a) 0 (b) 4 (c) 9 (d) 6 (d) the nails bit on the frozen roads
123. Sometimes we walked through the pine forest as
PART - III (ENGLISH) (a) the path was unaffected by the frost
(b) it was good walking with nails in the boots
Direction (Qs 121 - 123) : Read the passage carefully
and answer the questions given below (c) the walks was invigorating
(d) it was sheer joy to walk in the forest
Sometimes we went off the road and on a path through
the pine forest. The floor of the forest was soft to walk 124. To drive home
on; the frost did not happen it as it did the road. But (a) To find one's roots
we did not mind the hardness of the road because we (b) To return to place of rest
had nails in the soles and heels nails bit on the frozen
(c) Back to original position
ruts and with nailed boots it was good walking on the
road and invigorating. It was lovely walking in the (d) To emphasise
woods. 125. Choose the best pronunciation of the word,
Extempore, from the following options.
121. 'Frozen ruts' means
(a) very cold roads (a) ex-tem-pour
(b) wheel marks in which frost had become hard (b) ex-tem-purree
(c) the road covered with frost (c) ex-tem-poray
(d) hard roads covered with snow
(d) ex-tem-pure
www.jeeneetbooks.in

WWW.IIT-NEET.XYZ

Solutions MT-115

SOLUTIONS
MOCK TEST 1
1. (c) s> k> r r3
C D
2. (d) E (ds) cos E ( r 2 ) cos 0º r2 E .
0.5Y r2 r4 r6
3. (d) Y [Alt : r5
2 (1 )

4. (a) Charge Q=C1V


A r1 O r7 B
Total capacity of combination (parallel)
C = C1+C2 Between C & D, the equivalent resistance is
given by
Q C1V
P.D. 1 1 1 3
C C1 C 2 1/ r 1
r3 (r4 r5 ) 2 2
5. (d) At A current is distributed and at B currents
are collected. Between A and B, the Equivalent resistance along
distribution is symmetrical. It has been 2 8
shown in the figure. It appears that current ACDB 1 1
3 3
in AO and OB remains same. At O, current i4 Effective resistance between A and B is
returns back without any change. If we
1 3 1 7 8
detach O from AB there will not be any or R
change in distribution. R 8 2 8 7
Now, CO & OD will be in series hence its 6. (b) The first law concerns the conservation of
total resistance = 2 charge.
7. (d) Since, wheatstone's bridge is balanced, then
It is in parallel with CD, so, equivalent
resistance of galvanometer will be
2 1 2 uneffective.
resistance =
2 1 3 R R 2R
This equivalent resistance is in series with
AC & DB, so, total resistance R

2 8 R R 2R
= 1 1
3 3 8. (b) We know that U = – W
i3 Therefore
C D
x x
i2 i4 U= Fdx or U kxdx
i4
0 0
i2
kx 2
A i1 O i1 B U ( x) U ( 0) given U(0) = 0,
2
8 kx 2
Now is parallel to AB, that is, 2 so U(x) =
3 2
total resistance 9. (b) The current is same in both the wire, hence
magnetic field induced will be same.
8/3 2 16 / 3 16 8 10. (b) In a perpendicular magnetic field, the path of
8/3 2 14 / 3 14 7 a charged particle is a circle, and the magnetic
field does not cause any change in energy.
EBD_7443
www.jeeneetbooks.in

WWW.IIT-NEET.XYZ

MT-116 Target VITEEE


11. (c) Power = V . I = I2R
medium is (v) 1 .
Power 2 1 1
i2
R 8 4 2
Therefore refractive index of the medium
1
Potential over 8 = Ri 2 8 4V Vel. of E.M.wave in vacuum (v 0 )
2 ( )
This is the potential over parallel branch. So, Vel. of E.M. wave in medium (v)
4
i1 1 1/ 0 0
4 = =
Power of 3 = i12R = 1 × 1 × 3 = 3W 1/ 0 0

output D
12. (a) Efficiency is given by 18. (c) Distance of n th maxima, x n
input d
5 15 14 As b g
0.875 or 87.5 %
10 8 15
x blue x green
13. (b) Time constant is L/R
Given, L = 40H & R = 8 19. (b) Material expands outward and so x, r
= 40/8 = 5 sec. increases. Due to linear expansion diameter
14. (c) L = 2mH, i = t2e–t of rod will increase.
20. (c)
di 21. (a) For Bragg’s scattering of a beam of electron,
E= L L[ t 2 e t 2 te t ]
dt the inter-planar distance ‘d’ is given by
when E = 0, 2d sin = n
–e–t t2 + 2te–t = 0 Here = 30°
or, 2t e–t = e–t t2 1
t = 2 sec. sin 30 ; n 1
2
15. (c) Initial current (I1) = 10 A; Final current
(I2)= 0; Time (t) = 0.5 sec and induced e.m.f. n 1
d
( ) = 220 V. 2 sin 1
Induced e.m.f. ( ) 2
2
(I I1 )
= L dI L 2 L
(0 10)
20L h
dt t 0.5 or d
mv
220 (According to de-Broglie, wavelength
or, L 11H
20 h
associated with an electron )
[where L = Self inductance of coil] mv
E 0 nBA mm
16. (c) I0 22. (b) For positronium reduced mass
R R m m
Given, n = 1, B = 10–2 T,
A = (0.3)2m2, R = 2 m2 m
f= (200/60) and = 2 (200/60) 2m 2
Substituting these values and solving, we Hence Rydberg’s constant for positronium
get is half of that for hydrogen atom.
I0 = 6 × 10–3 A = 6mA Hence, ground state energy for positronium
17. (d) We know that velocity of electromagnetic 13.6
6.8 eV
wave in vacuum (v0 ) 1 and 2
0 0 hc hc
23. (b) eV or
velocity of electromagnetic wave in min
min
eV
www.jeeneetbooks.in

WWW.IIT-NEET.XYZ

Solutions MT-117

24. (b) Using formula 33. (a) Applying formula


2d sin n 364 10 3
3 Maximum current 0.04 A
n 1 20 10 9.1
d
2sin 2sin 30 34. (c) Peak value Vm = P.D. across R – (– Vm)
= – (Vm + Vm) = –2Vm
3
20 10 35. (c) When the collector is positive and emitter
d 20 nm
1 is negative w.r.t. base it causes the forward
2
2 biasing for each junction, which causes
conduction of current.
mo mo 36. (c) (W + X) . (W + Y) = W + (X . Y)
25. (c) m 2m o
1/ 2 1 34 1/ 2 37. (b) The process of changing the frequency of
v2 a carrier wave (modulated wave) in
1
c2 accordance with the audio frequency signal
(modulating wave) is known as frequency
26. (c) modulation (FM).
27. (a) From the graph it is clear that A and B
38. (a)
have the same stopping potential and
therefore the same frequency. Also B and 39. (c) As velocities are exchanged on perfectly
C have the same intensity. elastic collision, therefore masses of two
28. (b) If a body slides down, then the force of objects must be equal.
friction acts upwards along the plane ma
1 or m a m b .
weight(mg) act vertically downwards. mb
29. (c) Energy released per fission is 200 MeV 40. (a) LED is generally and most suitably used in
picture tubes of TVs.
80
30. (c) No. of half lives n1 4; 41. (d) d-d transition of Cr3+.
20
Ruby is Alumina (Al2O3), contains about
80 0.5 to 1.5% Cr3+ ions which are randomly
n2 2
40 distributes in place of Al3+ ions. As Cr3+
n1 4 has d 3 configuration, it behaves like
N1 1 1 1 octahedral complex. At the points wherever
for A
N 0A 2 2 16 they are present within the lattice, they
undergo d-d transitions and thus produces
n2 2
N2 1 1 1 colour.
for B
N 0B 2 2 4 Ground state configuration of Cr 3+
Cr3+ 3d3 ( t2g 3 e0 )
g
N1 1 4 1
( N 0A N 0B )
N2 16 1 4 d xz , d xy , d yz , d ,d
x 2 y2 z2
31. (a) From the graph of binding energies of the
nuclei, the curve indicates that binding d x2–y2
energy increases to a maximum of about eg d z2
8.8 MeV per nucleon around mass number dxy
56 and then decreases slowly to about dxz e–
t2g dyz
7.6 MeV per nucleon at A = 238. Thus
among the options, the binding energy per
nucleon is largest for 56Fe. Any one of the electron from t2g moves into
32. (a) 9 × (2)3 = 9 × 8 = 72 eg state which is associated with some
Hence number of nucleons in the Ge specific amount of energy, gives rise to
nucleus = 72. colour.
EBD_7443
www.jeeneetbooks.in

WWW.IIT-NEET.XYZ

MT-118 Target VITEEE


42. (c) We know that regular decrease in the size colour is due to the hydrated chromium (III)
of the atoms and ions is called lanthanide in [Cr(H 2 O) 6 ]3 .
contraction. In vertical column of transition
51. (d) When electrons are trapped in anion
elements there is a very small change in
vacancies, these are called F-centres.
size and some times size is found same from
second member to third member.The
similarity in size of the atoms of Zr and Hf is +ve –ve
evident due to the object of lanthanide ion ion
contraction. Therefore Zr and Hf both have
same radius 160 pm. F- centre in crystal
43. (a) G° = –2.303 RT logKpwhen Kp = 1, G° = 0since 52. (c) The no. of atoms is a unit cell may be
log 1 = 0 calculated by the the formula
44. (c) By carbon dating method n n n n
Z= c + b + f + e
2.303 N0 8 1 2 4
Age of wood t½ log Where nc = no. of atom at the corner n b =
0.693 N no. of atoms at body centre nf = no. of atoms
at face centre ne = no. of atoms at edge
ratio of C14 / C12 in living wood centre.
ratio of C14 / C12 in dead wood An Fcc crystal contains
Hence it is based upon the ratio of C14 and 8 6
C12. = + = 4 atoms in a unit cell.
8 2
45. (c) 1 atom of 235 53. (d) Given : Order of Bragg diffraction (n) = 2 ;
92 U on fission gives energy
Wavelength ( ) = 1 Å and angle ( ) = 60º.
= 3.2 × 10–11 J
We know from the Bragg’s equation n = 2d
6.023 × 1023 atom (1 mole) on fission gives sin
energy = 3.2 × 10–11 × 6.023 × 1023 J or 2 × 1 = 2d sin 60º
235 gm of 235 on fission gives energy
92 U 3 2
2 1 2.d. d 1.15Å
6.023 2 3
= 3.2 1012 J = 8.2 × 107 kJ
235 (where d = Difference between the
46. (b) IUPAC name is potassium scattering planes)
trioxalatoaluminate (III). 54. (a) We know from the third law of
47. (c) As it forms two moles of silver chloride thus thermodynamics, the entropy of a perfectly
it has two moles of ionisable Cl. crystalline substance at absolute zero
temperature is taken to be zero.
[Co( NH3 ) 5 NO 2 ]Cl 2 [Co( NH 3 ) 5 NO 2 ] 2Cl 55. (a) If more trans-2-pentene is added, then its
2Cl 2AgNO 3 2AgCl 2 NO 3 concentration in right hand side will
48. (a) Triethoxyaluminium has no Al – C linkage increase. But in order to maintain the K
constant, concentration of cis-2-pentene
O CH 2 CH 3 will also increase. Therefore more cis-2-
Al O CH 2 CH 3 pentene will be formed.
56. (d) Rate of reaction
O CH 2 CH 3
1 d[Br ] 1 d[Br2 ]
49. (a) MA3 B3 – 2 geometrical isomers –
MA2 B4 – 2 geometrical isomers 5 dt 3 dt
MA4 B2 – 2 geometrical isomers d[Br2 ] 3 d[Br – ]
The complexes of general formula Ma6 and –
dt 5 dt
Ma5b octahedral geometry do not show
geometrical isomerism.
57. (b) RateI = k [A]x[B]y ... (1)
50. (d) We know that chromium (III) salts dissolve Rate I
in water to give violet solution. The violet = k [A]x [2B]y ... (2)
4
www.jeeneetbooks.in

WWW.IIT-NEET.XYZ

Solutions MT-119

or Rate1 = 4k[A]x[2B]y 63. (b) 2 Eº = –0.44


Fe / Fe
From (1) and (2) we get
Fe 3 / Fe 2 E º 0.77
k[A]x [B]y
= k[A]x[2B]y The metals having higher negative electrode
4
potential can displaced metals having lower
[ B]y 1 2B
y 1 values of negative electrode potential from
= [2B]y or 4 =2
y their salt solutions.
4 4 B
64. (d) This is because zinc has higher oxidation
or 2–2 = 2y or y = –2. potential than Ni, Cu and Sn. The process
58. (d) Solid Liquid of coating of iron surface with zinc is known
It is an endothermic process. So when as galvanization. Galvanized iron sheets
temperature is raised, more liquid is formed. maintain their lustrue due to the formation
Hence adding heat will shift the equilbrium of protective layer of basic zinc carbonate.
in the forward direction. 65. (c) We know that
59. (d) 3Fe(s) + 4H2O (steam)
H2C – CH2+RMgX CH2 – CH2
Fe3O4 (s) + 4 H2(g)
O
OMgX R
(pH 2 ) 4
Kp = H 2O CH2 – CH2
( pH 2 O) 4
–Mg(OH)X
only gaseous products and reactants. OH R
0.693 66. (d) Electron-donating groups (– OCH3, – CH3
60. (d) K etc.) tend to decrease an d electron
45
withdrawing groups (– NO2, – OCH3 etc.)
2.303 100 tend to increase the acidic character of
further t 45 log
0.693 100 99.9 phenols. Since – OCH3 is a more powerful
electron-donating group than – CH3 group,
2.303 45 3 1 therefore, p-methylphenol is slightly more
= 7 hours .
0.693 2 acidic that p-methoxyphenol while p-
61. (a) The equivalent conductance of BaCl2 at nitrophenol is the strongest acid. Thus,
infinite dilution option (d), i .e. p-methoxyphenol, p-
methylphenol, p-nitrophenol is correct.
1
of BaCl2 of Ba of Cl – 67. (d) With Br 2 water, phenol gives 2, 4, 6-
2
tribromophenol.
127
76 139.5 ohm –1 cm 2 OH
2
+ 3Br2 (excess)
W q
62. (a) By Faraday's Ist Law,
E 96500
OH
(where q it = charge of ion ) Br Br
we know that no of equivalent H 2O
+3HBr
W it 1 965 1
Br
E 96500 96500 100 2, 4, 6 Tribromphenol
(where i= 1 A, t = 16×60+5 = 965 sec.) 68. (c) In the cleavage of mixed ethers with two
Since, we know that different alkyl groups, the alcohol and alkyl
1 iodide that form depend on the nature of
no. of equivalent 100 alkyl group. When primary or secondary
Normality alkyl groups are present, it is the lower alkyl
Volume (in litre) 1
group that forms alkyl iodide therefore
= 0.01 N
EBD_7443
www.jeeneetbooks.in

WWW.IIT-NEET.XYZ

MT-120 Target VITEEE

CH 3 CH CH 2 O CH 2 CH 3 HI CH2OH HOOC CH 2OOC.COOH


| | | |
CH3 100 110 C
CHOH HOOC CHOH
| oxalic H 2O |
CH3 CH2OH acid CH 2OH
|
CH3 CH CH2 OH CH3CH 2 I
69. (c) Primary alcohol on oxidation give aldehyde CH2OOCH CH 2OH
| |
which on further oxidation give carboxylic CO2 H 2O
CHOH CHOH HCOOH
| | Formic acid
acid whereas secondary alcohols give
CH 2OH CH 2OH
ketone.
76. (c) Benzaldehyde reacts with primary aromatic
[O]
CH 3CH 2 CH 2 OH
n propyl alcohal amine to form schiff's base
[O]
CH 3 CH 2 CHO CH 3 CH 2 COOH
C6 H 5CH O C 6 H 5 NH 2
H3C [O] H3C Benzaldehyde Aniline
CH – OH C=O
isopropyl alcohal Ketone C 6 H 5C H NC 6 H 5 H 2O
Benzyliden e aniline
70. (c) Thermodynamic efficiency is given by
This is know as Schiff’s base reaction.
nFE
77. (d) RDX is prepared by treating urotropine with
H fuming nitric acid. When the inner bridge
71. (b) Pinacolone is 3,3-dimethyl-2 butanone. system is destroyed by oxidation and the
CH3 peripheral nitrogen atom are nitrated, it
| forms cyclonitrite (or RDX).
CH3 C C CH 3
| ||
78. (d)
CH 3 O

72. (d) Methyl acetate and ethyl acetate on


hydrolysis give CH3COOH which is a liquid.
Similarly ethyl formate on hydrolysis will
give formic acid which is also a liquid. Only
ethyl benzoate on hydrolysis will give
benzoic acid which is a solid.
73. (a)
OH OCOCH3
COOH COOH
Pyridine
+ ClCOCH3
O-hydroxy benzoic acid acetyl Aspirin
(Salicylic acid) chloride

74. (c) Tollen's reagent is used to detect of


aldehydes. Tollen's reagent is an ammonical
Benzyl alcohol
solution of silver nitrate. When aldehyde
C N
is added to Tollen's reagent, silver oxide is
reduced to metallic silver which deposits 79. (b) + CH3MgBr
as mirror. OCH3
RCHO + Ag2O RCOOH + 2Ag H3C –C = NMgBr H3C –C = NH
+
75. (c) When glycerol is heated with oxalic acid H 3O
following reaction occurs. OCH3
OCH3
www.jeeneetbooks.in

WWW.IIT-NEET.XYZ

Solutions MT-121

COCH3
+
84. (a) Since, b, c and b c are mutually
H3 O
perpendicular vectors, therefore any vector
OCH3 can be expressed in ter ms of
a
80. (d) All aliphatic amines are stronger bases than
NH3 and among different ethylamines order b, c and b c.
of basictity is 2° > 3° > 1°. Thus, the correct
order is (d) i.e., Let a x b y c z ( b c) ....(i)
(C 2 H 5 ) 2 NH (C 2 H 5 )3 N Taking dot product with b c in eq. (i),
C 2 H 5 NH 2 NH 3 .
This anomolous behaviour of tertiary ethyl
we get, a. ( b c) 0 0 z | b c |2
amine is due to steric factors i.e., crowding a . ( b c)
of alkyl groups cover nitrogen atom from z
all sides and thus makes the approach and | b c |2
bonding by a lewis acid relatively difficult
which results the maximum steric strain in Taking dot product with b in eq. (i), we get
tirtiary amines. The electrons are there but
a. b x b . b y c . b z .0 x
the path is blocked resulting the reduction
in its basicity. Taking dot product with c in eq. (i), we get
81. (c) R A × B under given condition a < b is
given by a. c 0 y 0 y a. c
R = {(1, 3), (1,5), (2,3) (2,5), (3,5) (4,5)}
R–1 = {(3,1), (5, 1), (3,2), (5,2), (5, 3), (5,4)} a . ( b c)
a ( a . b) b ( a . c ) c ( b c)
RoR–1 = {(3, 3), (3, 5), (5,3), (5, 5)}. | b c |2
1 i 1 i 1 i 1 i2 2i 2i 85. (c) Since y 1 | x | 1 x R, therefore,
82. (a) i
1– i 1– i 1 i 2 2
1– i graph of the given function must lie on or
n above the line y = 1.
1 i
1 in 1 hence only (c) option is correct.
1– i 86. (b) Let S x2 – 4y
Clearly the smallest value of n is 4. Since the point (2a, a) lies inside the
83. (c) The given equation can be written as parabola,
tan x tan / 3 tan x tan 2 / 3 S (2a, a) = 4a2 – 4a < 0
tan x 3 or a (a – 1) < 0 ...(1)
1 tan x tan / 3 1 tan x tan 2 / 3
Also, the vertex A (0, 0) and the point (2a, a)
tan x 3 tan x 3 are on the same side of the line y = 1 (the
tan x 3 equation of latus rectum)
1 3 tan x 1 3 tan x
So, a – 1 < 0 i.e. a < 1 ...(2)
(tan x 3)(1 3 tan x)
Y
(1 3 tan x) (tan x 3)
tan x 3
S (0, 1)

1 3tan 2 x
8 tan x
tan x 3 y=1
1 3 tan 2 x (2a, a)
tan x (1 3 tan 2 x ) 8 tan x
3
1 3 tan 2 x X
A
3(3 tan x tan 3 x )
3 From (1) and (2), we have a (a – 1) < 0
1 3 tan 2 x
or 0 < a < 1
3 tan 3x 3 tan 3x 1
EBD_7443
www.jeeneetbooks.in

WWW.IIT-NEET.XYZ

MT-122 Target VITEEE


87. (c) Let L1 : x + 2y = 8; P(E3/A) =
L2 2x + y = 2;
P E3 P A / E3
L3 : x – y = 1
Since the shaded area is below the line L1, P E1 P A / E1 P E2 P A / E2 +P E3 P A / E3
we have + P(E4)P(A/E4)
x + 2y 8. Since the shaded area is above
1 1
the line L2, we have 2x + y 2. Since the
= 4 7 = 0.165
common region is to the left of the line L3,
1 3 1 1 1 1 1 4
we have x – y 1
4 18 4 4 4 7 4 13
88. (a) We have,
(3 – x)4 + (2 – x)4 = (5 – 2x)4 = (3 – x + 2 – x)4 p q y r z
(3 – x)4 + (2 – x)4 = (3 – x)4 + (2 – x)4 + 4
90. (c). p x q r z 0
(3 – x) (2 – x)3 + 4 (3 – x)3 (2 – x) + 6 (3 – x)2
p x q y r
(2 – x)2
2 (3 – x) (2 – x) [2 (3 – x)2 + 3 (3 – x) (2 – x) Apply R1 R1 – R3 and R2 R2 – R3,
+ 2 (2 – x2)] = 0
(3 – x) (2 – x) (18 + 2x2 – 12x + 18 – 9x – 6x x 0 z
+ 3x2 + 8 + 2x2 – 8x) = 0 we get 0 y z 0
(3 – x) (2 – x) (7x2 – 35x + 44) = 0 p x q y r
x = 2, 3 and 7x2 – 35x + 44 = 0
Since discriminant of 7x2 – 35x + 44 = 0 is x[ yr z(q y)] z[0 y(p x)] 0
negative, [Expansion along first row]
it has no real roots. xyr xzq xzy yzp zyx 0
Hence the gievn equation has two real
p q r
roots and two imaginary roots. xyr zxq yzp 2 xyz 2
x y z
89. (c) Let A, E1, E2, E3 and E4 be the events as
defined below 91. (a) Let l, m, n be the d.c.’s of the normal to the
A : a black ball is selected plane which contains the two concurrent
lines with d.c’s l1, m1, n1 and l2, m2, n2.
E1 : box I is selected
The line whose d.c.,’s are l, m, n is
E2 : box II is selected perpendicular to the line whosed d.c.,’s are
E3 : box III is selected l1, m1, n1 and l2, m2, n2.
E4 : box IV is selected l1 l + m1 m + n 1 n = 0 ...(1)
Since, the boxes are chosen at random, and l2 l + m2 m + n2 n = 0 ...(2)
Therefore, P(E1) = P(E2) = P(E3) If the third line whose d.c’s are l3, m3, n3
also lies in this plane, then it is at right angles
1 to the normal to this plane.
= P(E4) = l3 l + m3 m + n 3 n = 0 ...(3)
4
Eliminating l, m, n from (1), (2) and (3), we
3 2 get
Also, P(A/E1) = , P(A/E2) = , P(A/
18 8
l1 m1 n1
1 4 l2 m2 n2 0,
E3) = and P(A/E4) =
7 13 l3 m3 n3
P(box III is selected, given that the drawn
ball is black) = P(E3/A) By Bayes’ theorem, 92. (b) If F be the resultant of the three given
forces then
www.jeeneetbooks.in

WWW.IIT-NEET.XYZ

Solutions MT-123

F (iˆ 2jˆ 3k)


ˆ (2iˆ 3jˆ 4k)
ˆ ( ˆi ˆj k)
ˆ
46 4
P BG ,P G
2iˆ 4jˆ 2kˆ 80 5
If O be the origin, then 10 5
P BG / G
OP = p.v. of P = ˆj 2k̂ 16 8
5 4 1
OA = p.v. of A = î 2ˆj P BG G
8 5 2
AP OP OA (ˆj 2k)
ˆ (iˆ 2j)
ˆ
P BG G
ˆi 3jˆ 2kˆ
B
P G P G
Vector moment of the given forces about G
A = vector moment of F about A = AP F Now, P(G/BG) =
P BG G 1 80 20
( ˆi 3jˆ 2k)
ˆ (2iˆ 4jˆ 2k)
ˆ
P BG 2 40 23
2iˆ 6jˆ 10kˆ
The magnitude of the moment 95. (d) U
= 4 36 100 140
93. (c) Let the G.P. be A, AR, AR2,............
Then x p th term AR p 1 ,

y q th term AR q 1 , A B'
A B A' B

z r th term AR r 1 At most one of two events occurs if the


event A' B' occurs. The probability of
Now, x q r
yr p zp q
this is P(A' B' ) 1 P(A B),
p 1 q r q 1 r p r 1 p q Now,
AR AR AR
P(A' B' ) P (A' ) P(B' ) P(A' B' )
q r (p 1)(q r) r p (q 1)(r p)
A R A R P (A ' ) P(B' ) [1 P(A B)]
p q (r 1)(p q)
A R P(A ' ) P(B' ) P(A B) 1.
q r r p p q Finally, since
A
P(A' B' ) P[( A' )' B' ] P[ A' ( B' )' ] P( A' B' )
R pq pr q r qr qp r p rp rq p q
A R 1 = P (A B' ) P (A ' B) P (A' B' )
94. (c) From the tree diagram, it follows that P( A B) P(A' B) P( A B' ) P(A B)
S
1 [See the Venn diagram].
4
5 5 96. (b) rank = 2

G R 4 5 x 4 5 x
3 1 3 1 5 6 y 0 5 6 y 0 k 7
4 4 4 4 6 k z 0 k 7 0
AG AR AR AG
4 5
3 13 1 1 3 1 3 Also 0 rank 2, if k 7
4 44 4 4 4 4 4 5 6
BG BR BG BR BG independent of the value of d
EBD_7443
www.jeeneetbooks.in

WWW.IIT-NEET.XYZ

MT-124 Target VITEEE


101. (a) Let A = Cr + 2 Cr–1 + Cr–2
n n n

97. (d) Given, sin–1 x – sin–1 2x = ±


3 n! 2n!
3 r!(n r)! (r 1)!(n r 1)!
sin–1 x – sin–1 2x = sin–1
2 n!
+
3 (r 2)!(n r 2)!
sin–1 x – sin–1 = sin–1 2x n![(n r 2.n r 1) 2(n r 2)r r(r 1)]
2
r!(n r 2)!
3 3
sin–1 x 1 1 x2 n![(n 2 nr n nr r 2 r 2n 2r 2
4 2 2 2
2nr 2r 4r r r]
= sin–1 2x
r!(n r 2)!
x 3
1 x2 2x (n 2 3n 2)n! (n 1)(n 2)n!
2 2
r!(n r 2)! r!(n r 2)!
( 3 1 x 2 ) 3x (n 2)!
On squaring both sides, we get = n+2Cr.
r!(n 2 r)!
3 (1– x2) = 9x2
1 102. (a) sin 1 x 1
1 – x2 = 3x2 4x2 = 1 x=± 2 2
2
1 1 1 x 1 1
1 1
98. (c) y tan (2 tan u) (2 tan u) 0 x–1<1
2 2
1 x 2
tan(tan 1 u tan 1 u ) tan(2 tan 1 u ) Domain of f (x) is [1, 2]
103. (b) We have,
1 2u 2u
tan tan x a
1 u2 1 u2 a2 a
f (x) dx sin a cos a
2 2 2
dy 0
1
dx Differentiating w.r.t. a, we get
99. (c) Since sides of a triangle are in A.P., i.e., a, b,
c, are in A.P. and let a < b < c, 2b = a + c 1
f(a) = a + (sin a + a cos a) – sin a
2 2
b2 c2 a 2
Now, cos A
2bc 1 1
Put a = ;f = + – =
b 2
c 2
( 2 b c) 2 2 2 2 2 2 2
=
2bc 104. (b) The number formed will be divisible by 4 if
the number formed by the two digits on
b 2 c 2 4b 2 c 2 4bc the extreme right is divisible by 4, i.e. it
2bc should be 04, 12, 20, 24, 32, 40.
The number of numbers ending in 04 = 3! = 6
4bc 3b 2 4c 3b
The number of numbers ending in 12
2bc 2c = 2.2!= 4 [ 0 cannot occur at first place]
100. (a) Replacing each octal digit by the The number of numbers ending in
corresponding 3-digit binary number, we 20 = 3! = 6
have
The number of numbers ending in
(23450)8 = (010 011 100 101 000)2 24 = 2.2!= 4
= (10011100101000)2.
www.jeeneetbooks.in

WWW.IIT-NEET.XYZ

Solutions MT-125

The number of numbers ending in 107. (b) Rewrite the equation of straight line in the
32 = 2.2!= 4 following form
The number of numbers ending in y x cot p cos ec ....(i)
40 = 3! = 6
So, the required number = 6 + 4 + 6 + 4 + 4 x2 y2
Now y = mx + c is a tangent to 1
+ 6= 30 a2 b2
105. (a) Solving the two equation x2 = ay and if c2 = a2m2 + b2
y – 2x = 1, we get Applying this condition at (i), we get
x2 = a (2x + 1) or x2 – 2ax – a = 0
(p cos ec ) 2 a 2 ( cot ) 2 b2
x1 + x2 = 2a and x1x2 = –a
So, the given line cuts the parabola at two p 2 cos ec 2 a 2 cot 2 b2
poins (x1, y1) and (x2, y2)
p 2 a 2 cos 2 b 2 sin 2 .
2
Now, 40 = (x1 – x2)2 + (y1 – y2)2 [Given] 108. (b) We have,

2
3x 4
7 4
x2 x 22 3x 4 5x 7
40 = (x1 – x2)2 + 1
a a gof (x) = g
5x 7 3x 4
5 3
5x 7
( x1 x 2 ) 2
= (x1 – x2)2 1 21x 28 20x 28 41x
a2 = x
15x 20 15x 21 41
7x 4
4a 2 Similarly, fog (x) = f
= [(x1 + x2)2 – 4x1x2] 1 5x 3
a2
7x 4
= 5(4a2 + 4a) 3 4
5x 3
a2 + a – 2 = 0 =
7x 4
a = 1, –2 5 7
5x 3
1
106. (a) I = sin x .1 dx 21x 12 20x 12 41x
= x
Integrating by parts and taking 1 as second 35x 20 35x 21 41
function. Thus, gof (x) = x, x B and fog (x) = x,
d x A, which implies that gof = IB and
I = sin 1 x 1dx sin 1
x 1dx dx
dx fog = IA.
109. (a) The function defined in (a) is not
1 x
sin x ( x) dx differentiable at x = 1/2.
2 110. (a) Construct the composition table by taking
1 x
elements 1,2, 3, 4 of G in this order in a
1 dt horizonltal row and a vertical column.
x sin 1 x , substituting1 x 2 t
2 t Composition table
x5 1 2 3 4
2xdx dt
1 1 2 3 4
1 1 t1 / 2 2 2 4 1 3
x sin x C 3 3 1 4 2
2 1
2 4 4 3 2 1
Computing as
1
x sin x 1 x2 C 1 ×5 1 = 1, 1 ×5 2 = 2, 1 ×5 3 = 3, 1 ×5 4 = 4
2 ×5 1= 2, 2 ×5 2 = 4, 2 ×5 3 = 1, 2 ×5 4= 2
EBD_7443
www.jeeneetbooks.in

WWW.IIT-NEET.XYZ

MT-126 Target VITEEE


3 ×5 1= 3, 3 ×5 2 = 1, 3 ×5 3 = 4, 3 ×5 4 = 2 … (i)
4 ×5 1= 4, 4 ×5 2 = 3, 4 ×5 3 = 2, 4 ×5 4 = 1. Clearly c > 0 as c = e2a
(i) Closure propety : As all the entries of Hence, the equation (i) gives a family of
composition table are clements of G, hyperbolas with eccentricity
therefore, G is closed under multiplication c 1
modulo 5. c 1
c c2 1
(ii) Associative Law : For all a, b, c G c 1 if c 1
(a ×5 b) ×5 c = a ×5 (b ×5 c) c 1 c 1
as the least non negative remainder Thus ecentricity varies from member to
obtained by dividing ( a × b) × c by 5 is member of the family as it depends on c.
same as obtained on dividing a × ( b × c) 113. (c) Put 1 x z2 dx 4z x dz
by 5.
×5 is associative in G.
(iii) Existence of Identity : For any a G, 1 G 1 x z
dx (4z x )dz
such that 1 ×5 a = a ×5 1 = a x x
1 G is identity element in G for × 5.
(iv) Existence of Inverse : From composition z2 1
4 dz 4 1 2
dz
table, z 2
1 z 1
1 ×5 1= 1, 2 ×5 3 = 3 ×5 2 = 1, 4 ×5 4 = 1
1, 3, 2, 4 G are respectively inverses 1 z 1
of 1, 2, 3, 4 under ×5 . 4 z log C
2 z 1
(v) Commutative Law : For all a, b G.
a ×5 b = b ×5 a
As the least non-negative remainder 1 x 1
4 1 x 2 log C
obtained by dividing a × b by 5 is same as
1 x 1
obtained on dividing b× a.
×5 is commutative in G.
114. (c) Let the variable point be ( , , ) then
Hence ( G, ×5 ) is a finite abelian group of
order 4. according to question
f ( x h ) f (x ) 2 2 2
111. (a) We have f ( x ) Lim | | | | | 2 |
h 0 h 9
3 2 6
f ( x ) f (h ) f ( x )
f ' (x ) Lim 2
9. 2 2
h 0 h
[by given property] So, the locus of the point is
f (h ) (2h 2 3h ) g(h ) x2 y2 z2 9
Lim Lim
h 0 h h 0 h 115. (a) Given equation of line is
Lim (2h 3)g (h ) 3 g (0) 3 3 9
h 0 r 2iˆ ˆj 2kˆ 3iˆ 4jˆ 2kˆ
x dx y dy
112. (d) Given 2
xiˆ yjˆ zkˆ 2 3 ˆi 1 4 ˆj 2 2 kˆ
1 x 1 y2
Integrating we get, Any point on the line is
1 1 (2 + 3 , – 1 + 4 , 2 + 2 )
log(1 x 2 ) log(1 y 2 ) a
2 2 Since it also lie on the plane r · ˆi ˆj kˆ
2 2
1 x c (1 y ),
So,
Where c e 2a
2 3 iˆ 1 4 ˆj 2 2 kˆ . ˆi ˆj kˆ 5
2 2 x2 y2
x cy c 1 1 2+3 +1–4 +2+2 =5
c 1 c 1
=0
c
www.jeeneetbooks.in

WWW.IIT-NEET.XYZ

Solutions MT-127

Therefore, coordinate of the point of


118. (a) y cx c 2 3c 3 / 2 2 ...(i)
intersection of line and plane is (2,–1, 2).
Distance Differentiating above with respect to x, we
dy
2 2 2 get c.
d 2 1 1 5 2 10 dx
13 Put this value of c in (i), we get
116. (a) Since the equation of a hyperbola and its 2 3/ 2
asymptotes differ in constant terms only, dy dy dy
y x 3 2
therefore the equations of asymptotes of dx dx dx
the given hyperbola are given by Clearly its order is ONE and after removing
xy 3x 4 y k 0 the fractional power we get the degree
Where k is a constant to be determined FOUR.
by the condition that 4 3 , dv 4 d 3 4 dr
119. (c) v r r 3r 2 .
abc 2fgh af 2
bg 2
ch 2
0 3 dt 3 dt 3 dt

2 dr
i.e 0 2 2
3 1
0 0 k
1
0 4 r2 .
2 2 2 dt
k = –12 when r = 10cm
Asymptotes of the given hyperbola are dv
xy 3x 4 y 12 0 or 4 (10)2 .(0.02) = 8 cm3/s.
dt
( x 4)( y 3) 0 i.e x = –4 and y = 3 120. (d) a . (a + b) = (a + b) . a
(Commutative law) = (a + b) . (a + 0)
(cos x 1)(cos x e x ) (Identity law) = a + (b . 0)
117. (c) 5 lim
x 0 xn (Distributive law) = a + 0 = a.

2sin 2 (x / 2)(e x 1 1 cos x) 121. (d)


lim
x 0 xn 122 (c)
123 (a)
2 sin 2 ( x / 2)(e x 1 2 sin 2 x / 2)
lim
x 0 xn 124 (a) Perplexed means cause to feel completely
baffled. Example: Rahul was at his wit's
2 end to find that his sister has failed.
1 sin x / 2 (e x 1 2 sin 2 x / 2)
lim
x 02 x/2 xn 2 125 (b)
which is finite and non zero if n = 3
ALTERNATE :

(cos x 1)(cos x e x )
lim
x 0 xn

x2 x4
...... x x 2 ......
2! 4!
lim
x 0 xn
is non-zero n = 3.
EBD_7443
www.jeeneetbooks.in

WWW.IIT-NEET.XYZ

MT-128 Target VITEEE

MOCK TEST 2
1. (b) In an inelastic collision, momentum remains 8. (b) E = V + ir
conserved, but K.E is changed.
2. (c) C2 and C3 are parallel so V2 = V3 1.8 10
2.2 1.8 r r
C1 and combination of C2 & C3 is in series. 5 9
So, V = V2 + V1 or V = V3 + V1 9. (c) In the electrolysis of CuSO4 , oxygen is
and also Q1 = Q2 + Q3 liberated at anode and copper is deposited
3. (d) When oil is placed between space of plates at cathode.
2A 0
C ... (1) V2 200 200
d 10. (b) R1 = 1000
P1 40
KA 0
C , if K 2 V2 200 200
d R2 = 400
P2 100
A 0
When oil is removed C' ........(2) R1 (for 40W) > R2 (for 100 W)
d
on comparison both equation, weget
C' = C/2 11. (c)
4. (d) According to Gauss’ Law
Q enclosed by closed surface For a beam, the depression at the centre is
E.ds flux fL
o given by,
so total flux = Q/ o 4Ybd3
Since cube has six face so flux coming out [f, L, b, d are constants for a particular beam]
through one wall or one face is Q/6 o.
1
10 O 10 F 10 i.e.
5. (c) A B Y
12. (b) For circular path in magnetic field,
10 10
mr 2 = qvB
G E
C D
10 10 10 2 qvB
As v r
Equivalent Circuit mr
F q(r )B qB
2
10
mr m
A
10 E 10 D
O If is frequency of roatation, then
10
10 10 qB
G 2 2 m
Equivalent Resistance of circuit
N 0 Ni
20 20 13. (c) L ; BA; B
= 10 10 10 10 10 30 i 2R
20 20
2
6. (c) According to Stoke's law , F= 6 vTR N 0 Ni 0N
7. (a) We know from the Kirchhoff's first law that L A A L N2
i 2R 2R
the algebraic sum of the current meeting at
any junction in the circuit is zero (i.e. i = 0) 14. (a) Power dissipated = Erms. Irms = (Erms) (Irms)
or the total charge remains constant. cos
Therefore, Kirchhoff's first law at a junction Hence, power dissipated depends upon
deals with the conservation of charge. phase difference.
www.jeeneetbooks.in

WWW.IIT-NEET.XYZ

Solutions MT-129

21. (b) Energy in first excited state


( 2 1) ( 0 NBA ) NBA
15. (b) e E1 13.6
t t t 3.4 eV
22 4
NBA 50 2 10 –2 10 –2 Hence, energy required to ionize atom from
t 0.1 s
e 0. 1 its first excited state will be 3.4 eV.
16. (b) Mutual Inductance of two coils 22. (b) For Balmer series, electron jumps from
higher orbit to secondary orbit, thus for
M M1 M 2 2mH 8mH 4mH
1 1
17. (c) Rays Wavelength series v RH 2
[Range in m] 2 n2
X-rays 1 × 10–11 to 3 × 10–8 For Brackett series, the electron jumps from
-rays 6 × 10–14 to 1 × 10–11 higher orbits to fourth stationary orbit
Microwaves 10–3 to 0.3 (nf = 4)
Radiowaves 10 to 104 1 1
Wavelength of U.V. rays ranges from v RH 2
4 n2
6 × 10–8 to 4 × 10–7.
18. (d) Huygen's construction of wavefront does For Lyman series, electron jumps from
not apply to origin of spectra which is higher orbit to the first stationary orbit i.e.,
explained by quantum theory. nf = 1
19. (d) Let the amplitude of the waves be a1, a2 1 1
and the intensities I1 and I2. The ratio of v RH 2
1 n2
this intensities is 16 : 9
For Paschen series, electron jumps from
I1 a12 16 higher orbits to third stationary orbit
Then I (nf = 3)
2 a 22 9
1 1
a1 4 v RH 2
....(1) 3 n2
a2 3
23. (c) W = area of F – x graph
In interference, the maximum and minimum = area of + area of rectangle + area
resultant amplitudes are (a1 + a 2) and of
(a1 – a2) respectively
5 3 5 3
= 10 3 = 45 J
Maximum intensity I max (a1 a 2 )2 2 2
Minimum intensity I min (a1 a 2 ) 2 24. (c) A Febry Perot interferometer consists of two
plane parallel glass plates. With this
But from equation (1),
interferometer fringes of constant inclination
4a 2 are obtained by transmitted light after
a1
3 multiple reflection between the glass plate.
2 hc 1 1
4a 2 25. (b) Ek (in eV)
a2 e 0
Imax 3
I min 2 34
4a 2
a2 6.6 10 3 108 1010 1010
3 = 19
1800 2300 = 1.5 eV
1.6 10
I max 49 1
49 :1 2
Imin 1 26. (b) m 1 2 W0 W0 W0 and
2
20. (c) The optical path length between A and B is
1 2
3 m 2 10 W0 W0 9W0
x + (t × ) + y = x + (1.5t) + y x t y 2
2
EBD_7443
www.jeeneetbooks.in

WWW.IIT-NEET.XYZ

MT-130 Target VITEEE

1 W0 1 32. (b) W L1 (W + X)
9 W0 3 X
2 W + (X.Y)
27. (b) Energy of electron = mec2 W L3
Y L1 (W + Y)
Energy of positron= mpc2
me mp c 33. (b)
speed of light. 34. (d) Positive terminal is at lower potential
Thus according to conservation of energy (–10V) and negative terminal is at higher
released potential 0V.
2 35. (b)
2m e c 2 2 9.1 10 31
3 108 36. (c) We know that for common base
1.6 10 13 Joules . i c collector current
0.98
28. (b) For a monoatomic gas, the average energy i e emmiter current
3 & for common emitter
of a molecule at temperature T is k BT .
2 ic collector current
3 ib base current
Internal energy U RT
2
0.98 0.98
dU 3 49.
CV = R 1 1 0.98 0.02
dT 2 37. (c) Since specific heat = 0.6 kcal/g × °C = 0.6
For an ideal gas, CP – CV = R cal/g × °C
5 CP 5 From graph it is clear that in a minute, the
CP =R and temperature is raised from 0°C to 50°C.
2 CV 3
29. (c) Forces between nucleons inside the Heat required for a minute = 50 × 0.6 × 50
nucleus are = 1500 cal.
(i) attractive in nature Also from graph, Boiling point of wax is
(ii) extremely short range 200°C.
(iii) strongest forces in nature 38. (c) Optical fibres are not subjected to
30. (d) 2 2 3 1 electromagnetic interference from outside.
1H 1H 2 He 0n 13 MeV
39. (b) Core of acceptance angle
2
The binding energy of is 2.21H 1
sin n12 n 22
(2.2) × 2 = 13 MeV + Binding energy
(4.4) = – 13 MeV + Binding energy 40. (c) If is the original length of wire, then
Binding energy of He = 13 + 4.4 = 17.4 change in length of first wire,
31. (a) Half live of given radioactive nucleus = 100 ( 1 )
1
years
Number of half lives is 300 years Change in length of second wire,

300 2 ( 2 )
3
100 T1 T2
Now the number of atoms left after n half- Now, Y
A 1 A 2
lives is
n 3
1 1 N0 T1 T2 T1 T2
N N0 N0 or or
2 2 8 1 2 1 2
Hence the number of nuclei which have
or T1 – T1 = T2 – T2
decayed in 300 years is 2 1

7N0 T2 1 T1 2
N0 N0 8 or
8 T2 T1
www.jeeneetbooks.in

WWW.IIT-NEET.XYZ

Solutions MT-131

41. (a) If Gsystem = 0 the system has attained N


3+ en 3+
equilibrium is right choice. en – N
N N
In it alternative (d) is most confusing as N N Co
en
when G > 0, the process may be Co en
spontaneous when it is coupled with a N N
N N
reaction which has G < 0 and total G is N en
en
negative, so right answer is (a). N
42. (b) XeF4 is planar d form l form
Enantiomers
43. (a) For a spontaneous process, S total is
always positive The two optically active isomers are
44. (a) Since MgO has a rock salt structure. In this collectivity called enantiomers.
structure each cation is surrounded by six 53. (c) Lanthanides are 4 f-series elements starting
anions and vice versa. from cerium (Z= 58) to lutetium ( Z = 71).
45. (b) Out of the given substances, only Li has These are placed in the sixth period and in
high electrical and thermal conductivity. third group.
46. (c) For third ionization enthalpy last
configuration of 54. (c) [X ] 10 n 60 Co 1 H
27 1
V – 4s0 3d3 Balancing the mass and atomic numbers
3d 4s
60 1 60
on both sides 28 X 0 n 27 Co 11 H

Cr – 4s0 3d4 Thus X should be 60


28 Ni
55. (d) Age of geological formations (i.e. predicting
Mn – 4s0 3d5 the age of the earth and rocks) is estimated
by U– Pb method, also known as helium
Fe – 4s0 3d6 dating.
Note : C-14 dating method is used to predict
the age of fossils or dead animals or a fallen
For third Ionization enthalpy Mn has stable tree.
configuration due to half filled d-orbital.
56. (d) The activation energy of reverse reaction
will depend upon whether the forward
47. (c) reaction is exothermic or endothermic.
As H = Ea (forward reaction) – Ea(backward reaction)
For exothermic reaction
H = –ve
– H = Ea(f) – Ea(b)
or Ea(f) = Ea(b) – H
Ea(f) < Ea(b)
Transform of [M(AA) 2 a 2 ]n± does not
shows optical isomerism. for endothermic reaction
H = + ve
48. (b) For orthorhombic system 90
–2 H = Ea(f) + Ea(b) or Ea(f) = H + Ea(b)
49. (b) As in [NiCl4] Chloride ion being a weak
ligand is not able to paired the electron in d Ea(f) > Ea(b).
orbital. 57. (a) From data 1 and 3, it is clear that keeping
50. (b) Relation is Kp = Kc (RT) n (B) const, [A] is doubled, rate remains
51. (d) Reduction occurs at cathode. unaffected. Hence rate is independent of
52. (d) The optical isomers are pair of molecules [A]. from 1 and 4, keeping [A] constant, [B]
which are non super imposable mirror is doubled, rate become 8 times. Hence rate
images of each other [ B]3 .
EBD_7443
www.jeeneetbooks.in

WWW.IIT-NEET.XYZ

MT-132 Target VITEEE

dx Copper being more electropositive readily


58. (b) For a first reaction = k(a - x) precipitate silver from their salt solution
dt
Cu + 2AgNO3 ¾¾ ® Cu(NO 3 ) 2 + Ag
ò ò
dx
on intergration , = k dt whereas in K[Ag (CN)2] solution a complex
(a - x)
anion [Ag(CN)2]– is formed and hence Ag+
i.e – n (a –x) = kt + c or kt = n a – n (a–x) are less available in the solution and
or kt = 2.303 [log a – log (a – x)] therefore copper cannot displace Ag from
Thus if we plot a graph between log a & t its complex ion.
we get 64. (b) Given current (i) = 0.5 amp;
Time (t) = 100 minutes × 60 = 6000 sec
Equivalent weight of silver nitrate (E) = 108.
st Accordin g to Far aday's fir st law of
1 order
log[a] electrolysis
Eit 108 0.5 6000
W 3.3575 g.
96500 96500
time t 65. (b) Order of reaction is independent of given
59. (a) Given initial concentration (a) = 2.00 m; factors.
Time taken (t) = 200 min and final 66. (b) Acetamide and ethylamine can be
concentration (a – x) = 0.15 m. For a first distinguished by heating with NaOH
order reation rate constant, solution.Acetamide evolves NH3 but
60. (d) According to Le-chatelier's principle" ethylamine does not.
whenever a constraint is applied to a system
in equilibrium, the system tends to readjust CH 3CONH 2 NaOH
so as to nullify the effect of the constraint.
CH 3COONa NH 3
0
61. (c) By Kohlraush's law, eq NaCl 126.45
CH 3CH 2 NH 2 NaOH
0 0
126.45 ....(1) No reaction.
Na Cl
0 0
426.16 ...(2) 67. (c) NaNO 2 ,H Cl
H Cl NH 2
0 5 C
0 0
91 ....(3)
CH 3OO – Na
NMe2
on adding (2) and (3) then subtract (1) from
it N 2 Cl +
H
0 0
517.16 126.45
CH 3COO Na

1 N=N NMe 2
Ù0(CH 390.71ohm cm 2
3COOH)
68. (c) Electrolytic reduction of Nitroalkane in
0.0591
62. (d) E log 10 K weakly acidic medium give aniline
n
Here, n 2, E 0.295 NO2 NH2

2 0.295 Electrolytic reduction


log 10 K = 10 or K 1010 (weakly acidic medium)
0.0591
63. (d) In th e silver platin g of copper, Aniline

K[Ag(CN ) 2 ] is used instead of AgNO3. Whereas in strongly acidic medium it gives


p-hydroxyaniline
www.jeeneetbooks.in

WWW.IIT-NEET.XYZ

Solutions MT-133

NO2 NHOH 4[O]


CH3COCH3+CO2+2H2O
electrolytic
(CH3)3 COH
reduction in presence 8[O]
of conc. H2SO4 CH3COOH+2CO2+3H2O
(Strongly acidic medium)
O
NH2 C–CH3 CH2–CH3

rearrangement
73. (b) Zn-Hg/HCl

Phenyl methyl ethyl


OH ketone benzene
p-Hydroxyaniline
This reaction is known as clammensen's
69. (a) An aliphatic aldehyde. reduction.
Generally H – C = O streching frequency is
74. (a) Due to H-bonding, the boiling point of
about 2830 – 2695 cm–1 but for aliphatic
ethanol is much higher than that of the
aldehyde is 1740 – 1720 cm–1.
isomeric diethyl ether.
70. (a) Nitrobenzene is reduced to aniline in the
B 2H 6
presence of Sn/HCl. 75. (c) CH3–CH=CH2
NaOH / H 2 O 2
NO 2 NH 2
B 2H 6
CH3–CH2–CH2–OH
Sn / HCl aOH / H 2 O 2 Propanol

COONa
Nitrobenzene Aniline
76. (b) 2HCOONa | + H2
Hence, option (a) is correct. 360
SOCl 2 COONa
71. (d) CH 3COOH CH 3 COCl sod.formate
77. (a)
OMgBr
|
CH3MgBr
C6 H5 COOC2 H5 C6 H5 C OC 2 H 5
|
CH3

O
||
Mg(OC2 H5 )Br Excess
C6 H5 C CH3
CH3MgBr

OMgBr CH3
H2 O |
C6 H5 C CH3 C6 H5 — C — CH3
| |
CH3 OH

CH3
|
Con. H 2SO 4 Ozonolysis
C 6 H 5 — C CH 2

'B'
C 6 H 5COCH 3 HCHO

72. (c) By oxidation of tertiary alcohol with stronger C 6 H 5 COCH 3 2 3I 4 NaOH


CHI 3
oxidising agents ketones may be formed
along with carboxylic acid. 78. (a) It is an example of Claisen condensation.
The product is acetoacetic ester.
EBD_7443
www.jeeneetbooks.in

WWW.IIT-NEET.XYZ

MT-134 Target VITEEE


O O It has real roots, So
|| ||
CH 3 C OC 2 H 5 H CH 2 C OC 2 H 5 D 0 (2anc) 2 4 2a 2 (n 2 2)c 2 0

O O n2 2(n 2 2) 0
|| ||
CH 3 C CH 2 C OC 2 H 5 [ 4a 2 c 2 is positive]
(ethyl acetoacetate )
79. (d) Water gas is mixed with half its volume of n2 4 0 (n 2)(n 2) 0
hydrogen.The mixture is compressed to
2 n 2 n [ 2, 2]
approximately 200 – 300 atmospheres. It is
then passed over a catalyst |ZnO + Cr 2O3| +
+
at 300°C. Methyl alcohol vapours are –2 – 2
formed which are condensed
ZnO Cr2O3 –2 n 2
CO 2H 2 CH3OH
Compressed gas
300 C
Methyl alcohol 84. (c) The equation of the tangent at P (at2, 2at)
to y2 = 4ax is ty = x + at2 ...(1)
80. (c) Br2 / KOH It meets the directrix x = –a
R CONH 2 RNH 2
This is known as Hoffmann’s Bromamide a ( t 2 1)
ty a at 2 y
reaction. t
81. (a) For the given A.P., we have
sin (z + x – y) – sin (y + z – x) = sin 2 at)
(x + y – z)– sin (z + x – y) P (a t1 , 2
2 cos z sin (x – y) = 2 cos x sin (y – z) Q

sin x cos y cos x sin y


X
cos x cos y O S
x+a=0

sin y cos z cos y sin z


cos y cos z
tan x – tan y = tan y – tan z, showing that Thus, (1) meets the directrix at
tan x, tan y and tan z are in A.P. a ( t 2 1)
Q a,
82. (b) tan 1
2x tan 1
3x t
4
2at 0 2t
1 2 x 3x 1
Now, slope of PS is m1 = 2 2
tan tan 1 at a t 1
1 2 x.3x and slope of QS is m2
5x
1 6x 2 5x 1 0 a ( t 2 1)
0
1 6x 2 t ( t 2 1)
=
a a 2t
1
(6 x 1)( x 1) or – 1
0 x Since m1m2 = –1, therefore PQ subtends a
6 right angle at the focus.
Now for x = – 1, LHS of equation becomes 85. (c) Obviously the well (W) must be on one
side of the trees T1, T2,......., T25,
1
negative, so x .
6
W 5m 5m
83. (a) The given equation is T1 T2 T3...................T24 T25
2 2 2 2
2a x 2ancx (n 2 )c 0
www.jeeneetbooks.in

WWW.IIT-NEET.XYZ

Solutions MT-135

The total distance covered by the gardener


B 1
WT1 (2WT1 T1T2 ) (2WT2 T2T3) ..... (2WT24 T24T25] 2 tan
2 2 8
10 ( 2 10 5) (2 15 5) ............ to 25 terms tan B
2 B 15 15
10 (25 35 45 ..........to 24 terms) 1 tan
2 16
24
10 [ 2 25 ( 24 1) 10] 10 12 [50 230 ] 3370
2 loge x
86. (c) n n n 90. (c) If (x) log 5 log 3 x log 5
Cr 1 Cr 1 2 Cr log e 3
n n n n
Cr 1 Cr Cr Cr 1 log5 log e x log5 log e 3
n 1 n 1 n 2
Cr Cr 1 Cr 1 log e (log e x )
log 5 (log e 3)
87. (c) Constraints are 2x + 3y 36; 5x + 2y 50; 2x log e 5
+ 6y 60, x 0, y 0
1 1 1
The number of constraints are 5. ' ( x) . . 0
log e 5 log e x x
88. (a) I = sin 3 x. cos 5 x dx 1 1 1 1
' (e ). .
log e 5 log e e e e log e 5
Put sin x = t cos x dx dt
91. (d) For Rolle’s theorem in [a, b], f(a) = f(b), In
2
I = sin 3 x . cos 4 x . cos x dx t 3 1 t 2 dt [0, 1] f(0) = f(1) = 0
the function has to be continuous in
t 3 2t 5 t 7 dt
1 4 2 6 1 8 [0, 1]
= t t t c
4 6 8
f (0) lim f ( x ) 0 lim x log x 0
x 0 x 0
1 4 1 1
= sin x sin 6 x sin 8 x c
4 3 8 log x
lim 0
ALTERNATE : x 0 x
I= sin 3 x . cos5 x dx sin 2 x . cos5 x . sin x dx 1/ x
Applyin g L.H. Rule lim 0
1
x 0 x
Put cos x = t – sin x dx = dt
x
t8 t6 lim 0 0
2 5 7 5
I= 1 t t dt t t dt c x 0
8 6 92. (c)
1 1 93. (b) In the function f (x) = (x – 1) (x – 2) (x – 3) for
= cos 8 x cos 6 x c more than one value of x, i.e. x = 1, x = 2 and
8 6
x = 3, value of the function is zero.
[Not that this result is in terms of powers
So, the function is not one-one.
of cos x]
Range of the function is the set of all real
89. (b) s(s a )(s b)(s c) 4(s a )(s c) number i.e. R.
Since Range = Co-domain = R, the function
[b 2 (c a ) 2 (b c a ) (b c a ) is onto.
Thus the given function f(x) is onto but not
2(s a )2(s c)]
one-one.
Squaring and simplifying we, get
94. (a) f ( x) is onto
s(s b) 16(s a )(s c) S = range of f (x)
(s a )(s c) 1 B 1 Now f (x) =
tan 2
s(s b) 16 2 16 sin x 3 cos x 1 2sin x 1
3
B 1
tan
2 4 1 sin x 1
3
EBD_7443
www.jeeneetbooks.in

WWW.IIT-NEET.XYZ

MT-136 Target VITEEE


(k 1)(k 1)k! 2k!
1 2sin x 1 3
3 Tk (k 1)(k 1)! 2(k )!
f ( x) [ 1, 3] S
= (k 2 3)(k 1)! 2(k)!
95. (a) The equation of the parabola is y2 = 4ax...(1)
Let (x1, y1) be the mid point of a focal chord (k 2)! 3(k 1)! 2(k)!
of the parabola. T1 (3)! 3(2)! 2(1)! ;
Equation of chord having (x1, y1) as its mid
point is T2 (4)! 3(3)! 2(2)! ;
2 T3 (5)! 3(4)! 2(3)!
T = S1 i.e. yy1 – 2a (x + x1) = y1 4ax 1
..... ........ ......... ........
2 ..... ........ ......... ........
or yy1 – 2ax = y1 2ax 1
Since it passes through the focus (a, 0) Tm (m 2)! 3(m 1)! 2(m)!
2 m
0 – 2a2 = y1 2ax1 or Tk T1 T2 T3 ........... ..... Tm
2 k 1
y1 2a ( x 1 a ) = 0
= 3(2)! 2(1)! 2(2)! (m 1)! (m 2)! 3(m 1)!
Thus, the locus of the mid point (x1, y1) is = (m 2)! 2(m 1)! (m 1)!(m 2 2)
y2 = 2a (x – a), which is a parabola whose
vertex is (a, 0). m(m 1)!
dy dz Now, m(m 1)! 1999 2000! m 1999
96. (a) Put x y z 1
dx dx 98. (c) 2 sin 1 x sin 1 (1 x ) 2
The equation then becomes 2 2
where, 1
dz dz a2 z2 a2 sin x x sin
z2 1 a2 1 2 2 1
dx dx z z and sin (1 x ) sin 1 x

z2 Now, cos 2 cos sin


2
dz dx 2
z a2
Integrating we get 1 2 sin 2 sin 1 2x 2 1 x
2
z 1
dz dx D x (2x 1) 0 x 0 or x
2 2 2
z a
Only x = 0 satisfies the given equation.
a2 99. (d) We have
[1 ] dz x D
2 2
z a a 2a 3a
a2 z [ ] 2a 1 2a 3 a 1
1
z tan x D 3a 5 a 5 a 2
a a
1 z a{(2a 3)(a 2) (a 5)(a 1)}
a tan z x D
a 2a{(2a 1)(a 2) (3a 5)(a 1)}
z z x D 3a{(2a 1)(a 5) (3a 5)(2a 3)}
tan
a a a (15a 2 31a 37) , on simplification
y c
x y a tan , where c D 2
a 31 1259
15a a 0
97. (a) The general term 30 900
Tk (k 2 1)k! (k 2 1 2)(k )! [for all non-zero a]
www.jeeneetbooks.in

WWW.IIT-NEET.XYZ

Solutions MT-137

Hence, the given vectors are non-coplanar, 1 1 1


for all a 0 . 2 2
a 2a p2
100. (c) 1 1
det(B AB) det(B ) det A det B
p 2 2a 2 p 2a
1
det( B ). det B. det A det( B 1B). det A The equation of the chord is
det(1). det A 1. det A det A.
x cos y sin 2a
4 /3 7 1/3 21 6 Which is the equation of straight line always
101. (a) Put 1 x z x dx z dz
4 touching the circle x 2 y2 ( 2a) 2 , a
8
7 3 21 6 21 z
3
x 4 dx fixed circle of radius
x 1 z
4
z dz
4 8
C
2a .

21 3 4
8/7 104. (d) I = x 2 cos x dx
1 x C
32 d 2
2
102. (a) We have, (2 * 3) * 4 = 1 * 4 = 1 = x cos x dx x cos x dx dx
dx
and 2 * (3 * 4) = 2 * 1 = 1 [Integrating by parts taking x2 as first
Since, each row is same as their function]
corresponding column.
Therefore, the operation * is commutative. x 2 sin x 2 x sin xdx
We have (2 * 3) = 1 and (4 * 5) = 1 x 2 sin x 2 x sin x dx
d
x sin x dx dx
Therefore, (2 * 3) * (4 * 5) = 1 * 1 = 1 dx
103. (a) Equation of the pair of straight lines joining [Integrating by parts again taking x as first
the centre (origin) and the intersection function]
points of the variable chord and the
hyperbola is x 2 sin x 2 x ( cos x ) cos x dx

Y x 2 sin x 2 x cos x 2 sin x c


P (x 2 2) sin x 2x cos x c
F f (x) x2 2, g( x ) 2x
S (ae, 0) X 105. (b) Let P (a cos , b sin )
C
P(acos , bsin )

a
x
e F2 (-ae, 0) F1(ae, 0)

2 Then, A = area of PF1F2


x2 y2 x cos y sin
0 ....(i)
2 2 p a cos b sin 1
a 2a
1 1
[By homogenising the equation of ae 0 1 2ae b sin
2 2
hyperbola with the help of the equation of ae 0 1
the straight lines. See the pair of straight
abe | sin |
lines].
The equation (i) represents a pair of Clearly, A is maximum when | sin | 1
perpendicular lines, if coefficient of x2 + Maximum value of A = abe
coefficient of y2 = 0 106. (c) Let W stand for the winning of a game and
i.e., L for losing it. Then there are 4 mutually
exclusive possibilities
1 cos 2 1 sin 2 (i) W, W, W (ii) W, W, L, W
0
a2 p2 2a 2 p2 (iii) W, L, W, W (iv) L, W, W, W.
EBD_7443
www.jeeneetbooks.in

WWW.IIT-NEET.XYZ

MT-138 Target VITEEE


[Note that case (i) includes both the cases happens. We can express this fact as
whether he losses or wins the fourth game.] If a tumbler is half empty, then it is half full.
By the given conditions of the question, If a tumbler is half full, then it is half empty.
the probabilities for (i), (ii), (iii) and (iv) We combine these two statements and get
respectively are the following. A tumbler is half empty, if
2 2 2 2 2 1 1 2 1 1 2 and only if it is half full.
. . ; . . . ; . . .
3 3 3 3 3 3 3 3 3 3 3 109. (d) Since 2 persons can drive the car, therefore
1 1 2 2 we have to select 1 from these two. This
and . . . . can be done in 2C1 ways. Now from the
3 3 3 3
Hence the required probability remaining 5 persons we have to select 2
8 4 4 4 36 4 which can be done in 5C2 ways. But the
= . front seat and the rear seat person can
27 81 81 81 81 9
[ The probability of winning the game if interchange among themselves. Therefore,
the required number of ways in which the
2 2
previous game was also won is car can be filled is 5C2 × 2C1 × 2!
1 2 3
and the probability of winning the game if = 20 × 2 = 40.

1 1 2
previous game was a loss is ].
1 2 3 110. (b) log sin x dx ...(i)
107. (d) We have 0

dx 2 2
a ( sin sin cos ) a cos
d log sin x dx log cos x dx ...(ii)
2
and 0 0
From (i) and (ii)
dy
a (cos cos sin ) a sin ,
d 2 2
2 log sin x dx log cos x dx
dy dy / d
tan 0 0
dx dx / d /2
So, the slope of the normal is sin 2 x
log dx
2
1 0
cot , which varies as .
dy / dx
2 2
The equation of the normal at any point is log sin 2x dx log 2 dx ;
y a(sin cos ) 0 0
cot [x a(cos sin )]
2
x cos y sin a. log sin 2x dx log 2
Clearly, it is a line at a constant distance 2
0
| a | from the origin.
Put 2x t 2dx dt
Further, the equation x cos + y sin = a
represents the equation of a tangent to the 1
circle x2 + y2 = a2 for any value of , which 2 log sin t dt log 2
2 2
is a fixed circle. 0
108. (b) The given statements are
p : A tumbler is half empty. 2
1
q : A tumbler is half full. . 2 log sin t dt log 2
2 2
We know that, if the first statement 0
happens, then the second happens and [Since sin ( – t) = sin t, see property No. 8]
also if the second happens, then the first 2 log 2 log e 2
2 2
www.jeeneetbooks.in

WWW.IIT-NEET.XYZ

Solutions MT-139

function is continuous at x = 0
sin 2A sin C sin B
For function to be differentiable :
111. (b) Let sin C sin 2B sin A f (0 + h)= f (0 – h)
sin B sin A sin 2C
f (0 h) f (0)
f (0 + h) =
2 sin A cos A sin C sin B h
sin C 2 sin B cos B sin A 1
h sin
0 1
sin B sin A 2 sin C cos C lim h lim sin
h 0 h h 0 h
The above determinant is the product of
two determinants, which does not exist.
sin A cos A 0 cos A sin A 0 1
( h)sin 0
sin B cos B 0 cos B sin B 0 0 h
f (0 – h) = lim
sin C cos C 0 cos C sin C 0 h 0 h
112. (b) Given plane is 2x y 2z 4 0. 1
= lim sin
P h 0 h
which does not exist. So function is not
differentiable at x = 0
Q R 114. (c) y = sin2x ...(i) and y = cos2x ...(ii)
Solving (i) and (ii), we get
sin2x = cos2x
tan2x = 1
N M x
tan x 1
4
From points P and Q draw PM and QN Y
perpendiculars on the given plane and
QR MP .
,y
4
| 2( 1) 2 0 4 | 8,
| MP |
22 ( 1) 2 ( 2) 2 3
X
| 2.1 ( 1) 2.2 4 | 5 O /4 /2
| NQ |
2 2 ( 1) 2 ( 2) 2 3
Required area = area of shaded region
Also /4 /2
2 2 2 = sin 2 x dx cos 2 x dx
| PQ | (1 1) ( 1 2) ( 2 0) 17
0 /4
2 2
| NM | | QR | PQ RP 17 1 4. /4 /2
1 cos 2x 1 cos 2x
113. (c) For function to be continuous : = dx dx
f(0 + h) = f(0 – h) = f(0) 2 2
0 /4

f(0 + h) = lim h sin 1/h


h 0 1 sin 2x
/4
1 sin 2x
/2
= x x
= 0 × (a finite quantity) = 0 2 2 0 2 2 /4
f (0 – h) = lim – h sin (1/–h)
h 0 1 1 1 1
= 0
= 0 × (a finite quantity) = 0 2 4 2 2 2 4 2
Also, lim x sin 1/x = 0 × (a finite quantity) 1 1 1 1 1
x 0 = 2 sq. units.
=0 2 4 2 4 2 4 2 4
EBD_7443
www.jeeneetbooks.in

WWW.IIT-NEET.XYZ

MT-140 Target VITEEE


115. (c) E D
a
1 [ Coeff. of x2 = Coeff. of y2]
2
F C
9 f2
and, – C > 0[Using : g2 + f 2 – c > 0]
4 4
A B a = – 2 and 9 + f 2 – 4C > 0
AB AC AD AE AF
118. (d) In case of R1 , f ( x) 6x 7
ED AC AD AE CD clearly every element of A has a unique
= (AC CD) AD (AE ED) image hence, R1 represents a function.
AD AD AD 3AD Similarly, R2 and R3 also represent
n 3. functions.
116. (a) Let CM be the altitude through C. In case of R4, f (x) = ± 4x
C(c)
every element of A has two unequal
images for example f(1) = ± 4, f(2) = ± 8, etc.
R4 is not a function

1/ x 1 f (1 x) f (1)
lim
f (1 x) x 0 x f (1)
B(b) 119. (c) lim e
A(a) M x 0 f (1)
Then, area of triangle ABC
1 1 f '(1)
(AB)(CM) | b a | CM ....(i)
2 2 e f (1)
e2
Again, area of triangle ABC
120. (a) 10 = 9.
= , 11 = 9. 2 = 2
1 1
| AB AC | | (b a ) ( c a ) | (2 – ) (2 – 2) (2 – 10) (2 – 11)
2 2
1 = (2 – )2 (2 – 2)2
|b c c a a b | ...(ii)
2 Now x3 – 1 = (x – 1) (x – ) (x – 2),
From (i) and (ii), dividing by x – 1
|b c c a a b| x2 + x + 1 = (x – ) (x – 2)
CM
|b a| or (x2 + x + 1)2 = (x – )2 (x – 2)2.
117. (b) We have, Put x = 2, we get (2 – )2 (2 – 2)2 = (7)2 = 49
dy ax 3 121. (c)
(ax + 3) dx = (2y + f) dy
dx 2y f
122 (c)

x2 123 (d)
a + 3x = y2 + fy + C (Integrating)
2 124 (b) The second part of the sentence explains
or amplifies the part before the semicolon.
a 2 2 Since his recordings range widely, his tastes
x + y – 3x + fy + C = 0
2 could be described as wide-ranging, or
This will represent a circle, if eclectic.
125 (d)
www.jeeneetbooks.in

WWW.IIT-NEET.XYZ

Solutions MT-141

MOCK TEST 3
1. (a) Also 3 > 1 otherwise the ball would have
floated in liquid 1. From the above
1 q2 (8 10 18 ) 2
2. (c) Work done = = discussion we conclude that
2 c 2 100 10 8
1 < 3 < 2.
= 32 × 10–32 J 9. (a) In mass spectrometer, when ions are
3. (d) Unit positive charge at O will be repelled accelerated through potential V
equally by three charges at the three corners
of triangle. 1
mv 2 qV ..........(i)
By symmetry, resultant E at O would be 2
zero. As the magnetic field curves the path of
4. (d) the ions in a semicircular orbit
5. (a) If m1, m2 are masses and u1, u2 are velocity
mv 2 BqR
then by conservation of momentum m1u1 Bqv v .......... (ii)
R m
+ m2u2 = 0 or | m1u1 | | m2 u 2 | Substituting (ii) in (i)
R/2 2
1 BqR q 2V
m qV or
6. (a) 2 m m B2 R 2
Since V and B are constants,
R/2
q 1
R R
. m R2
R1 R 2 2 2
Re q R 10. (d) Magnetic moment, m = IA
R1 R 2 R R R eq
4 qv qvR
2 2 ( R2 )
7. (a) A balanced wheatstone bridge simply 2 R 2
requires
q 2 R
P R 2 2 I and T
T v
Q S 2 S
11. (a) Electron moves undeflected if force exerted
Therefore, S should be 2 . due to electric field is equal to force due to
A resistance of 6 is connected in parallel.
magnetic field.
In parallel combination,
1 1 1 |E|
q | v || B | q | E | |v|
R R1 R2 | B|
12. (a) Conservative force is negative gradient
1 1 1
S 3 of potential
2 6 S
8. (d) From the figure it is clear that liquid 1 floats –dV(x)
F(x) =
on liquid 2. The lighter liquid floats over dx
heavier liquid. Therefore we can conclude 13. (b) Efficiency of the transformer
that 1 2 Poutput 100
100 100 90.9%
Also 3 < 2 otherwise the ball would have Pinput 220 0.5
sink to the bottom of the jar. 14. (d) Condition for which the current is maximum
in a series LCR circuit is,
EBD_7443
www.jeeneetbooks.in

WWW.IIT-NEET.XYZ

MT-142 Target VITEEE

1 En 21.76 10 19
Thus U n
LC 2 2
1 = – 10.88 × 10–19 J
1000 22. (a)
L(10 10 6 ) 23. (c) The stimulating radiation is an electro-
= L = 100 mH magnetic wave of suitable frequency.
34
1 1 hc 6.6 10 3 108
24. (d) Energy J
15. (d) At resonance L = , = 0.5 10 10
C LC
E 6.6 10 34 3 108
Current through circuit i = = eV = 24.82 keV
R 5 10 11 1.6 10 19
Power dissipated at Resonance = i2R
h
d(i 0 sin t ) 25. (a) Moment of photon p
di
16. (b) e M = 0.005
dt dt
E mc 2 But, p mc
= –0.005 × i0 ×( cos t)
E mc.c so, E pc
e max . 0.005 i 0 (when cos t = –1)
= 0.005 × 10 × 100 = 5 V hc hc
E pc
17. (c) Variation in magnetic field causes electric
field and vice versa.
h c hf
In electromagnetic waves, E B. Both p and p
f c
E and B are in the same phase.
1
In electromagnetic waves 26. (d) qV m v 2 or m v 2q Vm ;
E E0 sin( t kx) 2

B B0 sin( t kx) h h 1
so i.e. ;
The electromagnetic waves travel in the mv 2q V m qm
direction of (E B). p q m
so 2 4 2 2
0. 4 q pm p
18. (a) ' 0.3 mm
4
hc hc
3 27. (d) eV0 W0 and eV W0
19. (a) As the floor exerts a force on the ball along 0 2 0
the normal, & no force parallel to the Subtracting them we have
surface, therefore the velocity component
hc 1 hc
along the parallel to the floor remains e ( V0 V) 1
constant. Hence V sin = V1 sin 1. 0 2 2 0
20. (b) From Brewster’s law
or V hc
= tan i V0
i = tan–1 = tan–1(1.5) = 57° 2e 0
21. (c) The total energy of the electron in n th orbit
stress
is given by 28. (a) tan(90 )
En = Tn + Un strain
29. (b) Bason group
1 ze2 1 ze 2 photon, pion, graviton, kaon
4 0 2rn 4 0 2rn 30. (c) The half life of the sample is
or En = 2Un
N0
Here En = –21.76 × 10–19 J 0.693
N
www.jeeneetbooks.in

WWW.IIT-NEET.XYZ

Solutions MT-143

7 1 8 4 4
39. (b)
31. (a) 3 Li 1H 4 Be 2 He 2 He 40. (c) here, m = 4 , g = 4 × 10–3 kg
This nuclear reaction is not possible x 4t 2 t
32. (c) A A 4 A 4 dx d2 x
ZX Z 2X Z 2 1X 8t 1 8
no. of neutron = A – 4 – (Z – 2 + 1) dt dt 2
= A – 4 – (Z – 1) d 2 x dx
Work done, W fdx m dt
=A– Z– 4 + 1 =A– Z– 3 dt 2 dt
33. (d) During the formation of a junction diode. 2
Holes from p-region diffuse into n-region (4 10 3 )(8)(8 t 1) dt
and electrons from n-region diffuse into p- 0
region. In both cases, when an electrons 2
meets a hole, they cancel the effect to each 32 10 3 (8t 1)dt
other and as a result, a thin layer at the 0
junction becomes free from any of charges 2
carriers. This is called depletion layer there 8t 2
3
32 10 t
is a potential gradient in the depletion layer, 2
negative on the p-side, and positive on the 0
= 32 × 10–3 [4(2)2 + 2 – 0] = 576 mJ
n-side. The potential difference thus
developed across the junction is called 41. (b) For reaction 3A B C
potential barrier. If it is zero order reaction r = k [A]0, i.e the
34. (b) Output of upper AND gate = AB rate remains same at any concentration of
'A'. i.e independent upon concentration of
Output of lower AND gate = AB A.
Output of OR gate, Y A B BA 42. (b) For a first order reaction, A products
This is boolean expression for XOR gate. r
r k[A] or k
35. (c) As we know that I Ic Ib [ A]
Divide by I c both side
1.5 10 2
k = 3 × 10–2
Ie Ib 1 1 0.5
1 1
Ic Ic 1
0.693 0.693
36. (a) In reverse bias the current through a p-n Further, t1/ 2 23.1
junction is almost zero. 3 10 2k
37. (a) The critical frequency of a sky wave for 43. (c) We know that the activation energy of
reflection from a layer of atmosphere is given chemical reaction is given by formula
by fc = 9(Nmax)1/2
k2 E a é T2 - T1 ù
10 × 106 = 9(Nmax)1/2 = k = 2.303 êê T T úú , where k 1 is the
1 ë 1 2 û
2
10 106 rate constant at temperature T1 and k2 is
Nmax 1.2 1012 m 3
9 the rate constant at temperature T2 and Ea
is the activation energy. Therefore
38. (c) Frictional force is always opposite to the activation energy of chemical reaction is
direction of motion determined by evaluating rate constant at
N two different temperatures.
44. (b) r k[A]n if n = 0
W E
r k[A]0
S
EBD_7443
www.jeeneetbooks.in

WWW.IIT-NEET.XYZ

MT-144 Target VITEEE


or r = k thus for zero order reactions rate is 27
equal to the rate constant. Eq. wt of Al3+ = =9
3
45. (b) H Ea(f ) Ea(b) No. of gm eq. of H+ = no. of mole of H+
Thus energy of activation for reverse Hence Mass of H+ = 0.5 × 1g = 0.5 g
reaction depend upon whether reaction is We know that, 2g H2 at STP = 22.4 L
exothermic or endothermic 22.4
If reaction is exothermic, H ve 0.5 g H2 at STP = = ´ 0.5 = 5.6 L
2
Ea(b) Ea(f ) 49. (a) Thermodynamic is based on conservation
of energy.
If reaction is endothermic H ve According to Ist law of thermodynamics
Ea(b) Ea(f ) Supplied heat to any system is used as
46. (d) Given molarity = 0.01 M work and change in internal energy ( E )
Resistance = 40 ohm; Supplied heat to the system, Q = w + E
l 1
E = Q –w
Cell constant 0.4 cm . 50. (c) (CH3)2NH is most basic because two electron
A
releasing groups are present on Nitrogen.
Specific conductivity ( ) Also aromatic amines are less basic then
aliphatic amines. The basic character of
cell constant 0.4
= amines follow the order
resistance 40 R2NH > RNH2 > C6H5NHCH3 > NH3
1 51. (c) The mecahnism of Hoffmann bromide
= 0.01 ohm cm 1
reaction is
1000 (i) RCONH 2 Br2 RCONHBr HBr
Molar conductance ( m)
Molarity
OH
(ii) RCONHBr HBr R — CO N Br H 2O
1000 .01
10 3 ohm 1
cm mol 2 1
.01 K
+
(iii) R — C — N —Br R — C — N + KBr
47. (d) Given for the reaction
O O
Zn (s) Cu (2aq ) Cu (s ) Zn 2(aq ) , R
Eº = +1.10 V.
(iv) O = C N•• R—N=C=O
At equilibrium
••

(rearrangement)
0.0591
E cell log10 K eq
n (v) R N C O 2KOH RNH 2 K 2 CO 3
here (n number of exchan ge of
electrons) 52. (d) NH2 NHCOCH3
0.0591 (CH 3CO)2O Br2
or 1.10 log10 K eq
2 CH3COOH
2.20
log10 k eq = 37.22 CH3 CH3
0.059 (A)
or Keq = 1.66 × 1037 NHCOCH3 NH2
48. (d) No. of gram equivalent of H+ = No. of gm eq. of Br
+
H /H2O Br
4.5
Al3+ = 0.5
9 CH3 CH3
(B) (C)
www.jeeneetbooks.in

WWW.IIT-NEET.XYZ

Solutions MT-145

53. (c) Due to greater electron-withdrawing effect 59. (a) Fraction of unoccupied sites in NaCl crystal
of NO2, and also due to additional resonance
x-ray density–pyknometric density
due to –NO2 group in the phenoxide ion,
nitrophenols are stronger acids than x-ray density
chlorophenol. Further since resonance in
p-nitrophenoxide ion is more important than 2.178´103 - 2.165´103 0.13´103
= =
that in m-nitrophenoxide ion, former is more 2.178´103 2.178 ´103
stable, hence p-nitrophenol is more acidic
than m-nitrophenol. o-Nitrophenol is least 130
= = 5.96 × 10–3
acidic among the three isomers due to ortho- 2178
effect. 60. (c) For a spontaneous reaction
54. (b) Only amides (but not acids and esters) G(–ve), which is possible if S = +ve,
undergo hydrolysis in presence of soda- H = +ve
lime to form sodium salt of a carboxylic acid and T S > H [As G = H – T S]
and ammonia gas. Further, since the given 61. (c) [Co(NH3)6]3+ Co3+(27 –3 = 24)
compound is a liquid, it must be formamide,
because propanamide is a solid
55. (b) RCONH 2 NaOH RCOOH 2 3
NH 3 d sp
59 g (1 mole ) 17 g (1 mole )
(inner octahedral
complex & diamagnetic)
Thus wt. of 1 mole of RCONH2 = 59
In other words, R + 12 + 16 + 14 + 2 = 59 [Cr(NH3)6]3+ Cr3+(24 –3 = 21)

R = 59 – 44 = 15 2 3
Molecular mass of R as 15 corresponds to d sp (inner octahedral
CH3 group, hence RCOOH should be complex & paramagnetic)
CH3COOH
[Ni(NH3)6]2+ Ni2+ (28 – 2 = 26)
56. (a) The relation between free energy change
and equilibrium constant is given by
Nernst equation 3 2
sp d (outer octahedral
RT complex & paramagnetic)
E Eº ln Q
nF
[Zn(NH3)6]2+ Zn 2+(30 – 2 = 28)
At equilibrium, E c = 0 and Q = KC

RT 3 2
Eº ln K …(i) sp d (outer octahedral
nF complex &diamagnetic)
Again Gº = nFEº …(ii)
put in (i) 62. (a) The given compound may have linkage
G º RT
isomerism due to presence of NO2 group
lnK ; G º RT lnK which may be in the form –NO2 or –ONO.
nF nF
It may have ionisation isomerism due to
57. (c) A body-centred cubic system consists of
presence of two ionisable group –NO2 & –
all eight corners plus one atom at the centre
Cl. It may have geometrical isomerism in
of cube.
58. (d) If in an ionic crystal of the type A+, B–, the form of cis-trans form as follows :
equal number of cations and anions are [Co(NH3 ) 4 Cl(NO 2 )]NO 2 & [Co(NH3 )
missing from their lattice sites so that the (NO2)2]Cl ––– ionisation isomers.
electrical neutrality is maintained. The [Co(NH3)5(NO2)2]Cl & [Co(NH3)5(ONO)2Cl
defect is called Schottky defect. ––– Linkage isomers
EBD_7443
www.jeeneetbooks.in

WWW.IIT-NEET.XYZ

MT-146 Target VITEEE


66. (a) Chloro diaquatriammine cobalt (III) chloride
NO2 NO2
is [CoCl( NH 3 )3 (H 2O) 2 ]Cl 2
H3N NH3 H3N NO2 67. (a) The basic character of the transition metal
monoxide is TiO > VO > CrO > FeO because
CO CO
basic character of oxides decrease with
NH3 NH3 NH3 NH3 increase in atomic number. Oxides of
transitional metals in low oxidation state
NO2 NH3 i.e., + 2 and + 3 are generally basic except
Trans-form cis-form Cr2O3.
68. (c) According to VSEPR theory, a molecule
Geometrical isomers with six bond pairs must be octahedral.
63. (b) Lesser is the number of unpaired electrons 69. (b) Due to –I effect of the –COOH group, H-
smaller will be the paramagnetic behaviour. bonds in acids are much stronger than in
As Cr++, Mn++, Fe++ & Ni++ contains. alcohols; while aldehydes do not exhibit
H-bonding.
Cr++ (3d4) = 70. (d) Since acetic acid freezes at 16.6ºC while
water freezes at 0ºC, glacial acetic acid is
= 4 unpaired e–. obtained by crystallising, separating and
melting acetic acid.
Mn++ (3d5) =
1 2
= 5 unpaired e–. 71. (d) 2-Phenylethanol, C H 2OH C H 2C 6 H 5 , is
a 1º alcohol which can be prepared from
Fe++ (3d6)= C6H5MgBr by treating with ethylene oxide
= 4 unpaired e–. (note that HCHO will introduce only one
carbon atom, i.e. it will give C6H5CH2OH
Ni++ (3d8) = and not C6H5CH2CH2OH).
= 2 unpaired e–. C H MgBr O
6 5
As Ni++has minimum no. of unpaired e–
thus this is least paramagnetic. C 6 H 5 CH 2 CH 2 OH ;
64. (c) Given t1/2 = 12.3 years
C 6 H 5 MgBr HCHO C 6 H 5 CH 2 OH
Initial amount (N0) = 32 mg
Total time = 49.2 years 72. (a) Among acetic acid, phenol and n-hexanol
only CH 3 COOH reacts with NaHCO3 to
T 49.2
No. of half lives ( n ) 4 evolve CO2 gas.
t1 / 2 12.3
CH3COOH + NaHCO3
n 4 CH3COONa + CO2 + H2O
1 1 32
now N t N0 = 32 = 2 mg
2 2 16 73. (d) Iodoform test is exhibited by ethyl alcohol
acetaldehyde, acetone methyl ketones and
Hence 32 mg becomes 2 mg in 49.2 years
those alcohols which possess CH3CH(OH)-
65. (d) Remaining activity = 0.01M group. As 3-pentanone does not contain
after 24 hrs CH3CO-group as therefore it does not give
n iodoform test.
1
Remaining activity = Initial activity 74. (b) Acetaldehyde reacts only with
2 nucleophiles.
Total time 24
Since the mobile p electrons of carbon–
Used half life time (n) = 4 oxygen double bond are strongly pulled
T1 / 2 6
towards oxygen, carbonyl carbon is
4 electron-deficient and carbonyl oxygen is
1 electron-rich. the electron deficient (acidic)
So, 0.01 = Initial activity
2 carbonyl carbon is most susceptible to
Initial activity = 0.01 × 16 = 0.16M attack by electron rich nucleophilic
www.jeeneetbooks.in

WWW.IIT-NEET.XYZ

Solutions MT-147

reagents, that is, by base. Hence the typical 80. (a) 725 - 720 cm–1.
reaction of aidehydes and ketones is Generally in case of long chain alkanes
nucleophilic addition. C – H rocking frequency classically
75. (d) Dihydrogen sodium phosphate (NaH2PO4) characterized in the region of 725 - 720 cm–1.
does not have a lone pair of electrons on
81. (b) We have 2y cos = x sin
the P atom. As such it can not act as a
nucleophile and hence does not react with cos sin
aldehydes and ketones. or k (say)
x 2y
OH
C CH HO C CH 3 Then cos = kx and sin = 2ky ...(1)
H 2SO 4 /HgSO 4 Again 2x sec – y cosec = 3
76. (a)
O or 2 x y
3
C–CH3 cos sin
–H2O 2x y
or 3 [from (1)]
Acetophenone
kx 2ky
77. (d) Out of given compound only acetaldehyde
can form optical active hydroxy acid as it 2 1 1
or 3 , giving k =
contains one asymmetric carbon atom as k 2k 2
marked below :
x
O OH We now get cos and sin =y
|| | 2
HCN
CH3 C H CH3 C CN Squaring and adding we get
|
CH
x2
O cos2 + sin2 =
| 4 y2
hydrolysis
CH3 C* H
|
x2
COOH y2 1 , or x2 + 4y2 =4
78. (c) Chlorine is electron withdrawing group. 4
Further Inductive effect is stronger at 82. (c) We have
position than -position. i.e.,
x2 x 2 ( x2 x 1) 1
f (x)
2 2
CH 2 ClCOOH CH 2 Cl CH 2 OOH x x 1 x x 1
79. (c) d xy eg [d 2 2 , d 2 ] 1
x y z 1 2
electronic transition 1 3
x
If we see the d elctronic configuration of 2 4
Ti, it has d 1 configuration. Since
We can see here that as x , f (x) 1
[Ti(H2O)6]3+ is an octahedral compound which is the min value of f (x). i.e. fmin = 1.
hence the d-orbital splitting will be
2
d x 2–y2 , d z2
1 3
Energy Also f (x) is max when x is min
2 4
eg
electron transition which is so when x = – 1/2
t2g e 2
dxy dxz dyz 1 3 3
i.e. when x .
2 4 4
Here an electron which is present in t2g
excites and transition occuring from 1
fmax 1 7/3 Rf = (1, 7/3]
d xy eg[d 2 2 , d 2 ] . 3/4
x y z
EBD_7443
www.jeeneetbooks.in

WWW.IIT-NEET.XYZ

MT-148 Target VITEEE


83. (d)
86. (b) We have PQ AP PB PC
2 2
2 + 2x + x 1+x
1 x PQ AB PC

AB PQ PC PQ CP
1+x 1
1 = CP PQ CQ .
sin cot 1 1 x
2 2x x 2
A

1
cos(tan–1x) =
1 x2 B
P
C

1 1
So,
2
2 2x x 1 x2
Q
1
1 + 2x = 0 x=
2 AB CQ and AB || CQ
84. (d)
ABQC is a parallelogram.
2 2
sin 1 sin 1 2 tan 1 x Q is a fixed point.
2 2
1 1 87. (c) Since, A is obtuse angle 90 A 180
2tan–1 + 2tan–1 = 2tan–1 x
90 180 (B C) 180
2x
[ 2 tan–1 x = sin–1 ] 90 B C 0 B C 90
1 x2
tan–1 + tan–1 = 2 tan –1 x B 90 C tan B tan(90 C)
2x tan B cot C tan B tan C 1
2 tan 1 x sin 1
1 x2 88. (c) Chord PQ is normal to the parabola at Q
tan–1 + tan–1 = tan–1 x and is equation is
y = mx – 2am – am3 ...(1)
tan 1 tan 1 x
1 P

x y
[ tan–1 x + tan–1 y = tan–1 1 xy ]
90º X
O

x
1 Q
85. (b) We have The slope of normal PQ is m = tan . The
(gof) (1) = g (f (1)) = g (3) = 1 joint equation of OP and OQ is obtained by
(fog) (1) = f (g (1)) = f (4) = 1 making the equation of parabola y2 = 4ax
(gof) (2) = g (f (2)) = g (5) = 3 homogeneous with the help of (1). Thus
(fog) (2) = f (g (2)) = f (1) = 3 joint equation of lines OP, OQ is
(gof) (3) = g (f (3)) = g (3) = 1
mx y
(fog) (3) = f (g (3)) = f (1) = 3 y2 = 4ax
(gof) (4) = g (f (4)) = g (1) = 4 2am am3
(fog) (4) = f (g (4)) = f (2) = 5 or m (2 + m2) y2 + 4xy – 4mx2 = 0 ...(2)
(gof) (5) = g (f (5)) = g (2) = 1
(fog) (5) = f (g (5)) = f (3) = 3 Since POQ = , th e sum of the
2
gof = {(1, 1), (2, 3), (3, 1), (4, 4), (5, 1)} coefficients of x2 and y2 in (2) must be zero.
and fog = {(1, 1), (2, 3), (3, 3), (4, 5), (5, 3)}. m (2 + m2) – 4m = 0 or m (m2 – 2) = 0
www.jeeneetbooks.in

WWW.IIT-NEET.XYZ

Solutions MT-149

As m 0, we get m2 = 2 or m = ± 2 . 6
having the first throw = and that of the
Thus, m = 2 . 11
89. (b) Let (x1, y1)be the mid point of the variable 5
chord AP (where A is fixed while P varies) other = .
11
x2 y2 Hence the ratio of chances of their winning
of the hyperbola 1 = 6:5.
2
a b2
91. (c) The m men can be seated in m! ways
Equation of the chord having (x1, y1) as its
mid points is When they are seated, there are (m + 1)
places, shown by × where n women can
xx1 yy1 x12 y12 sit. Then no two women would be together
2 2 2 (T = S1)
a b a b2 as shown below
As it passes through the fixed point A (a, 0) × M × M × M × ...... × M ×
ax1 x12 y12 4x12 4 x1 4y12 Then n women can arrange themselves in
or 0
2 2 2 2
a a b a a b2 these (m + 1) places in m 1
Pn ways.
4x12 4ax1 a 2
4y12 Hence the required no. of ways
1
a2 2
b
m 1 m!(m 1)!
( 2x 1 a ) 2
4y12
= m! Pn
1 (m n 1)!
a2 b 2
92. (c) Put x sin 2 and a sin 2 . then
(2x a) 2 4y 2
Hence locus of (x1,y1) is 1
a2 b2 y sin 1 (sin cos cos sin )
90. (a) Let the persons be A and B.
Suppose that A has the first throw. We know 1 1
sin x sin a
that the probability of getting ‘a three’ in a
1 dy 1 1
single throw = and that of ‘not getting a dx 1 x 2 x
6
1 5 93. (c) z 3 iz 2 2i (z i )(z 2 2iz 2) 0
three’ = 1 .
6 6
z i, 1 i, – 1 – i
2 4
1 5 1 5 1 Let A = (0,1), B = (1,–1), C=(–1, –1),
P (A wins ) .....to
6 6 6 6 6
[Since, A can win at the 1st, 3rd, 5th, ..... 0 1 1
1
mutually exclusive trials with respective ABC 1 1 1
2
1 5
2
1 5
4
1 1 1 1
probabilities , , ...... ]
6 6 6 6 6
1
| 2 2| 2
1 5
2
5
2 2
1 .........
6 6 6 35
94. (b) Given mean = np = ...(i)
6
1 1 1 36 6
6 2 . 35
1
5 6 36 25 11 and variance = npq = ...(ii)
36
6
Solving (i) and (ii), we get
6 5 1 1 5
P(B wins) . 1 q= , p= 1
11 11 6 6 6
i.e. the probability of winning of the person we get n = 7
EBD_7443
www.jeeneetbooks.in

WWW.IIT-NEET.XYZ

MT-150 Target VITEEE


P (x = r) = nC pr qn – r for Binomial Thus R is equivalence relation.
r
distribution where r = 7
1
7 7 7 7 98. (a) It is given that a (b c) b
5 1 5 2
P (x > 6) = 7C7 . =
6 6 6 Note: a (b c) (a.c)b (a .b)c
95. (b) From the graph of f(x) = |sinx|, it is clear that 1
f(x) is continuous everywhere but not b (a .c)b (a .b)c
2
differentiable at x = n , n Z.
1
Y (a .c ) b (a .b)c 0 ...(i)
2
f(x) = y = |sin x|
Since b and c are non-parallel, therefore
for the existence of relation (i), the coeff. of
X b and c should vanish separately..
– 3 – 3 /2 – 2 – 3 /2 – – /2 O /2 3 /2 2 5 /2 3 .
Therefore, we get
96. (c) We have 1 1
a .c 0 i.e., a . c
and a . b = 0
x 2 2
y2 4a x a sin ........( i)
a Since a , b , c are the un it vectors
Differentiating w.r. t x, therefore
dy x 1 1
2y 4a 1 cos a.b . ( cos–1 = )
dx a 2 3 2
Since, the tangent is parallel to 3 2 1
99. (a) |A| 3 0 3 0 A exists
dy 0 1
x axis , 0
dx
1 0 1 2
This gives Adj. A
2 3 0 3
x x x
4a 1 cos 0 cos 1 sin 0
a a a 1 1 1 2
A
3 0 3
From (i) y 2 4a x 0 y 2 4ax,
which is the required locus. 1 1 2 1 2 1 2
(A 1 ) 3
97. (b) The relation is defined on N N . since 27 1 3 0 3 0 3
the elements of N N are ordered pairs 1 1 8 1 2 1 1 26
hence the relation on N N relates two 27 0 9 0 3 27 0 27
ordered pairs. Now,
(i) a , b N ab ba 100. (a) Given curves are y x .... (i)
and 2y – x + 3 = 0 .... (ii)
(a , b) R (a , b) (a , b) N N . Thus R is
Y
refiexive
(ii) Let y= x
(a, b) R (c, d) ad bc (a, b) and (c, d) N N
bc ad cb da (c, d) R (a , b) . 2y – x + 3 = 0
X X
Thus R is symmetric O
3
(iii) Again let (a,b) R (c,d) and (c,d) R (e,f) –3/2
ad bc and cf de
adcf bcde af de (a , b) R (e, f )
Thus R is transitive. Y
www.jeeneetbooks.in

WWW.IIT-NEET.XYZ

Solutions MT-151

On solving eqs. (i) and (ii), we get


x 2 y 2 z 2 ax by cz 0
2
2 x x 3 0 Radius of this sphere is equal to 2k
2 a 2 b 2 c 2 4(2k) 2 16k 2
x 2 x 3 0
Also let (z, y, z) be the co-ordinates of the
x 3 x 1 0 centroid of the tetrahedron OABC, then
1 1 1
x 3 ( x = –1 is not possible) x a, y b, z c
y=3 4 4 4
Required area so that a = 4x, b = 4y, c = 4z
Eliminating a, b, c from (1) and (2), the
required locus is
3 3
= x x1 dy 2y 3 y 2 dy 16x 2 16y 2 16z 2 16k 2
0 2 0

3 or x 2 y2 z2 k2 .
y3
= y 2 3y 9 9 9 9
3 0 2
104. (a) I = cos sin3 d
101. (c) Given equations is cos x sin x 2 0

Since cos x sin x 1 1 2,


2
We must have cos x sin x 2 cos .sin 2 .sin d
0
Put cos t sin d dt
cos x 1 cos x 1, 1
4 4 [Note that power of sin is odd]
also, when

x+ = 0, 2 , 4 6 , ..... or , 3 , 5 , ..... 0, t 1, When , t 0


4 2
0 1 1 5
3 7
x= , I t1 t 2
. dt t2 t2 dt
4 4
102. (c) Let the roots of the equation be and 1 0
2 , then 3 7
1
2 2 2 2 2 2 8
2
l and 2
1 = 3t 7
t
3 7 21
m l l m 0
105. (a) Since each number consisting of 6 digits
l 2 1 starts with 35, 3 and 5 are fixed in the first
and .
3(m l ) 2(l m) and second places.
Eliminating , we get The other four places can be filled up with
remaining 8 digits in 8P4 ways = 8 . 7 . 6 . 5
l2 1 = 1680 ways.
2l 2 9l 9m 0
9( m l ) 2 2(l m) Hence the required number of telephone
Since l is real, so (–9)2 – 4 . 2 . 9m 0 numbers is 1680.
106. (b) The given equation can be converted to
9 linear form by dividing both the sides by
m
8 cos2y. We get
103. (c) Let co-ordinate of the points A, B, C be dy 1
(a, 0, 0), (0, b, 0) and (0, 0, c) respectively. sec 2 y 2 tan y x 3 ;
dx x
Then equation of the sphere OABC is
EBD_7443
www.jeeneetbooks.in

WWW.IIT-NEET.XYZ

MT-152 Target VITEEE

dy dz 1 1 1
Put tan y z sec 2 y 3
dx dx 1 1 1 40
3* * 3* 5 2 3*
dz 2 5 2 4 40 4
The equation becomes z x3 ,
dx x 3
which is linear in z =
160
The integrating factor is
P
2
dx
110 (a)
x 2 log x log x 2 2
I.F. e e e x
Hence, the solution is
Q
x6
z( x 2 ) x 3 ( x 2 ) dx a zx 2 a,
6 R S
a is constant of integration.
The four points are P (–2, 3, 4), Q (–4, 4, 6),
(6tan y)x 2 x 6 6a 6x 2 tan y x 6 c, R (4, 3, 5), S (0, 1, 2).
[c = 6a] direction ratios of RS are
107. (a) The system of equations will be consistent 0 – 4, 1 – 3, 2 – 5, i.e., 4, 2, 3.
if
Dividing each by ( 4) 2 ( 2) 2 (3) 2 , i.e.,
1 2
1 2 0
29 , the actual direction cosines' of RS
3
4 2 3
To evaluate we use R1 R 1 R 2 are , , .
29 29 29
followed by C 2 C 2 C1 to obtain
Projection of PQ on RS
1 1 0 1 0 0 4 2 3
= ( 4 2) ( 4 3) (6 4 )
1 2 1 2 29 29 29
3 0 3
1
( 8 2 6) 0
= 3( 1)(1 ) 3(1 2 ) 29
For the system to be consistent, we must which implies that PQ RS
have 1 2
0 or 1. 111. (c) As given,
Mean = np = 3 and Variance = npq = 2
dv
108. (d) The retardation at time t . Hence, npq 2 2
dt
np 3
q=
the differential equation is 3
dv dv 2 1
v dt ...(i) and p = ( 1– q) = 1 –
dt v 3 3
Integrating, we get log v t c ...(ii) 1
n. 3 n=9
When t 0, v V C log V 3
The equation (ii) becomes log v = – t + P(X > 8) = P(X = 8) + P(X = 9)
logV 8 9
9 1 2 9 1
v v t t = C8 C9
log t e v Ve 3 3 3
V V
8 9
ab 1 2 1 18 1 19
109. (a) Given that a * b a, b Q =9 1 = 9
4 3 3 3 3 39
www.jeeneetbooks.in

WWW.IIT-NEET.XYZ

Solutions MT-153

dx dy
n2 x y 0 ....(2)
112. (c) Tn dt dt
(2n 1) . 2n . (2n 1) . (2n 2)
When OA = 12, i.e. x = 12 then from (1),
n 132 = 122 + y2
2(2 n 1)(2 n 1)(2 n 2)
or y 13 2 12 2 5.
1 1 1 1 dx
= 2 12 (2n 1) Putting the values of y and in (2),
4 (2n 1) 3 (2n 2) dt

1 1 3 4 5 dy
we get 12 5 0
= 2 dt
24 2n 1 2n 1 2n 2
dy dy
1 1 4 1 4 or 30 5 0; 6ft / sec .
= dt dt
24 2n 1 2n 1 2n 2 Hence the end B is moving at the rate of
6ft/sec. The negative sign shows the
1 1 1 1 1 downward tendency of B i.e.y decreases
= 2 2n 1 2n 1 4 2 n 1 2 n 2
when t increases)
Putting n = 1, 2, 3, 4 and adding, we get 114. (b) Any tangent to the parabola y2 = 8ax is

1 1 1 1 1 1 2a
S 1 .... y = mx+ ...(i)
m
24 3 3 5 5 7
If (i) is a tangent to the circle, x2 + y2 = 2a2
1 1 1 1 2a
4 ... then, 2a
3 4 5 6
m m2 1
1 1 m2(1 + m2) = 2 ( m2 + 2)(m2 – 1) = 0;
= 24 1 4 log e 2 1
2 m = 1.
So from (i), y = (x + 2a).
1 1 1 115. (c) Let the no. of pencils in a bag be x
= (4 log e 2 1) log e 2 Let the no. of pens in a bag be y. There
24 6 24
should be at least 5 items in a bag
Note that sum of n terms of the first series
we have x + y 5
1 cost of pencils in a bag = `5x
is 1 which tends to 1 as n .
2n 1 cost of pens in bag `10y
Thus the sum of the first infinite series is 1. Total cost of a bag = 5x + 10y,
113. (b) At time t, let OA = x and OB = y. The total cost has to minimized
dx 1 Objective function is minimize
Then 2 ft./sec. C = 5x + 10y subject to x + y 5, x 0, y 0
dt 2
Y sin 2 x sin x 1
116. (c) Let I 2, 4 dx I 0, 2
B 4 3 3
13ft cos x 3 . cos x
sin x 1
O A X tan x c
3 3
3 cos x
dy
and we have to find .
dt 1 1
tan x 1 tan 2 x tan x c
From right angled AOB, 3 3
we have AB2 = OA2+OB2
1
132 = x2 + y2 ....(1) tan 3 x c
Differentiating with respect to t, we get 3
EBD_7443
www.jeeneetbooks.in

WWW.IIT-NEET.XYZ

MT-154 Target VITEEE


ALTERNATE : of eq. (ii) and the given line are
perpendicular.
sin 2 x
dx tan 2 x . sec 2 x dx ˆi ˆj 2kˆ .
cos x4 b1ˆi b 2 ˆj b3 kˆ 0
put tan x t b1 b 2 2b3 0
1 3 1
t 2 . dt
t c tan 3 x c b1 b2 2b3 0 ...(iv)
3 3
Similarly,
117. (b) We have ae = 5 [Since focus is (±ae, 0)]
a 3iˆ ˆj kˆ . b1ˆi b 2 ˆj b3 kˆ 0
a 36
and since directrix is x
e 5 e (3b1 + b2 + b3) = 0 ...(v)
From eqs. (iv) and (v), we obtain
5
On solving we get a = 6 and e = b1 b2 b3
6
1 1 1 2 2 3 1 1 1 1 3 1
2 2 25 2
b a (1 e ) 36 1 11 b1 b 2 b3
36
3 5 4
Thus, the required equation of the ellipse
Therefore, the direction ratios of b are –3, 5
x2 y2 and 4.
is 1.
36 11
b b 1ˆi b2ˆj b3kˆ 3iˆ 5jˆ 4kˆ
sin nx tan x
118. (d) lim n . lim (a n )n 0 Substituting the value of b in eq. (i), we
x 0 nx x 0 x obtain
n((a n)n 1) 0 ˆi 2jˆ 3kˆ
r 3iˆ 5jˆ 4kˆ
1 This is the equation of the required line.
(a n)n 1 a n .
n 120. (b)
119. (c) The equation of line passing through (1, 2,
3) and parallel to b is given by 121 (d)

ˆi 2ˆj 3kˆ 122 (b)


r b1iˆ b 2 ˆj b3 kˆ ...(i)
The equations of the given planes are 123 (b)
124 (d) Foment means to instigate or stir up (an
r. ˆi ˆj 2kˆ 5 ...(ii)
undesirable or violent sentiment or course
and r. 3iˆ ˆj kˆ 6 ...(iii) of action). Therefore, Incite is the synonym
of Foment.
The line in eq. (i) and plane in eq. (ii) are
parallel. Therefore, the normal to the plane 125 (c)
www.jeeneetbooks.in

WWW.IIT-NEET.XYZ

Solutions MT-155

MOCK TEST 4
1. (b) In shell , q charge is uniformly distributed 7. (d) It is a balanced Wheatstone bridge. Hence
over its surface it behaves as a conductor. resistance 4 can be eliminated.
X 6 9 18
X q R eq
6 9 5
R X V 5V
X i
X X R eq 18
X 8. (d) Current in the circuit
V = potential at surface E E 2E
q q r1 r2 R r1 r2 R
and inside V P.D. across first cell = E – ir 1
4 0R 4 0R
Because of this it behaves as an E r1
E
equipotential surface. (r1 r2 ) R
2. (b) Net field along,
2Er1
Now, E 0
kQ 2 kQ kq (r1 r2 ) R
AB =0
a2 ( 2a ) 2 a
2
2Er1
E 2r1 r1 r2 R
2 r1 r2 R
R = r 1 – r2
–Q –Q
9. (a) As for an electric appliance R (Vs2 / W ) ,
so for same specified voltage Vs
R 25 100
4
R 100 25
–Q –Q
i.e, R 25 4R with R100 R
Q 2 Q Q Now in series potential divides in
2q q (2 2 1)
1 2 4 proportion to resistance.
1 1 2 R1
CV 2 q /C So, V1 V
3. (c) W= U = (R 1 R 2 )
2 2
1 q1q 2 4
i.e., V25 440 352V
4. (a) In air Fair = 2 5
4 0 r
R2
1 q1q 2 and V2 V
In medium Fm = ( R1 R 2 )
4 0 Kr 2
1
i.e., V100 440 88V
Fm 1 Fair 5
Fm
Fair K K From this, it is clear that voltage across 100
W bulb (= 88 V) is lesser than specified
(decreases K-times)
(220 V) while across 25 W bulb (= 352 V) is
5. (c) By using m = ZIt
greater than specified (220 V), so, 25 W bulb
Where, Z is the electrochemical equivalent will fuse.
of copper.
Change in length
m 30 10 5 1.5 10 60 = 0.27 gm. 10. (a) strain =
6. (a) Kirchoff's first law deals with conservation Original length
of electrical charge & the second law deals 11. (d) When the deflection produced by electric
with conservation of electrical energy. field is equal to the deflection produced by
EBD_7443
www.jeeneetbooks.in

WWW.IIT-NEET.XYZ

MT-156 Target VITEEE


magnetic field, then the electron can go x 1
undeflected. or = .
y 3
0i
B and so it is independent of 18. (c) Interference is a wave phenomenon shown
2 r by both the light waves and sound waves.
thickness. 19. (c) Two sources of light are said to be coherent
12. (a) Let I be current and l be the length of the if they emit light having a constant phase
wire. relationship.
0 In 0I dU
For Ist case : B where 20. (d) | F | = , which is greatest in the reagion
2r l dx
2 r l and n = 1 CD.
l 21. (d) For hydrogen atom
For IInd Case : l 2(2 r' ) r'
4
1 1 1
R
0 nI 0 2I 4 0I n12 n 22
B' 4B
2r ' l l
2 For first condition
4
1
R v0 ; n1 2, n 2 1
13. (b) Power factor,
2
1
L R2 1 1 1 1
C v0 R R
2 2 4 1
(2) (1)
8 8
3 4
(31 25)2 82 6 2 82 v0 R or R v0
4 3
For second condition
0.8
10 4
R (constant) n1 = 4, n2 = 1
14. (d) When there is change of flux in the core of 3
a transformer due to change in current
around it, eddy current is produced. The 1 4 1 1
v0
direction of this current is opposite to the 3 (4)2 (1)2
current which produces it, so it will reduce
the main current. We laminate the core so 4 1 1 4 15 5
that flux is reduced resulting in the reduced v0 v0 v0
3 16 1 3 16 4
production of eddy current.
22. (a)
Np Ep 1
15. (b) QV 4 3
Ns Es 25 23. (b) r g and
3
Es = 25 Ep
But EsIs = EpIp QV 4 QV
(2 r )3 g so 8
25Ep × 2 = Ep × Ip Ip = 50 A 3 QV
16. (d) Quality factor
or Q 8 QV 800
Potential drop across capacitor or inductor 8Q 2Q
= V 3200
Potential drop across R
24. (c)
I L L 25. (b) Since work function for a metal surface is
=
IR R hc
17. (c) The heat lost in condensation = x × 540 cal. W
0
x 540 = y 80 y 1 (100 – 0) where is threshold wavelength or cut-
0
www.jeeneetbooks.in

WWW.IIT-NEET.XYZ

Solutions MT-157

off wavelength for a metal surface. can cross over to conduction band at 0 K.
here W = 4.125, But at room temperature some electrons in
eV = 4.125 × 1.6 × 10–19 Joule the valence band jump over to the
34 conduction band due to the small forbidden
6.6 10 3 108
so 0 3000A º gap, i.e., 1 eV.
4.125 1.6 10 19
26. (b) As per Einstein’s photoelectric equation : V0 10
34. (c) V V
E= h wF KE max
i.e. till a certain valve of , KE remains 0, it 35. (a)
only starts increasing once the Work 36. (b) A Y1
function (WF) of the metal surface is B
achieved. Y
27. (d) By Newton’s second law of motion
F = n(mv) = nmv Y2

h
28. (c) de Broglie wavelength, = Y1 A B, Y2 A.B
2mEK.E
Y (A B).AB AB BA
p m 4m p
This expression is for XOR
mp mp 37. (c) Change in momentum of the ball
= mv sin – (– mv sin )
[ E K.E ( ) E K.E ( p) ]
= 2 mv sin
p 2 2vsin
=
1 = mg g
29. (a) = weight of the ball × total time of flight
30. (a) Binding energy 38. (d) Let the velocity of the particle be v m/s.
E = [Zmp + (A – Z)mn – M]c2
Momentum of the particle (p) = mv
It can be shown as
Mass defect M Kinetic energy of the particle
= Mass of nucleons – Mass of nucleus. 1 1 (mv )2
From Einstein mass energy relation (E) = mv 2 .
E = Mc2 2 2 m
E = {Zmp + Nmn) – M}c2
p2
If A is the mass number, then N = A – Z E
Binding energy 2m
E = [Zmp + (A – Z)mn – M]c2 39. (a) Let the readings of two thermometers
31. (d) -rays are electrically neutral. The -rays coincide at C = F = x
are electromagnetic waves like X-rays. They C F 32
are called -photons. Thus, there will be no As
5 9
change in either mass number or atomic
number of the element. x x 32
32. (a) Let no. of -particles emitted be x and no. 5 9
of particles emitted be y. or 9x = 5x – 160
Diff. in mass no. 4x = 238 – 206 = 32 x = 8 4x = – 160, x = – 40°C
Diff. in charge no. 2x – 1y = 92 – 82 = 10 40. (d)
16 – y = 10, y = 6
33. (c) In semiconductors, the conduction is P h g 200 103 10
K 2 109
empty and the valence band is completely V /V V /V 0.1/100
filled at 0 K. No electron from valence band
EBD_7443
www.jeeneetbooks.in

WWW.IIT-NEET.XYZ

MT-158 Target VITEEE


41. (b) Dehydration of CH3 OH gives carbene 49. (a) Galvanization
(methylene), an unstable intermediate. 50. (c) Br2 / KOH
R CONH 2 RNH 2
H 2SO 4 This is known as Hoffmann’s Bromamide
CH 3OH [: CH 2 ] H 2O
Carbene reaction.
51. (c) Br2 / KOH
42. (a) The streching frequency for C = O in R CONH 2 RNH 2
case of aliphatic ketones is 1715 cm –1. This is known as Hoffmann’s Bromamide
43. (a) Due of chelation (intramolecular H- reaction.
bonding), o-nitrophenol has less b.p. than 52. (b) When benzaldehyde is refluxed with
the p-nitrophenol in which association of aqueous alcoholic potassium cyanide, two
molecules is possible due to intermolecular molecules of benzaldehyde condense
H-bonding. Hence the two isomers have together to form benzain
considerably different boiling points. H O
KCN (alc)
44 (c) Ethers react with Cl2 in presence of light to —C +C—
form fully chlorimated product (all hydrogen O H
atoms of the alkyl groups are replaced by H O
chlorine). —C +C—
45. (b) Only –OCOCH3 is electron-releasing due
OH
to presence of lone pair of electrons on the Benzoin
key atom. 53. (d) Aldehydes which contain a -hydrogen
on a saturated carbon, i.e., CH3CH2CHO
O O
.. || || undergo aldol condensation.
O
.. C CH 3 C OCH 3
H CH3
| |
CH3CH 2 – C O H H C CHO
O
|| propanol
C N C OH
CH3 CH3
2.303RT .0591 OH
| |
46. (c) E cell log K eq log K eq CH3CH2 C CHCHO
nF n |
OH
.0591
= log 10 6 = .0591 × 3 = 0.1773 3-hydroxy-2-methyl pentanal
2 54. (a) When 3 molecules are combined in
47. (a) The equivalent conductance of BaCl2 at presence of dry HCl gas they condense to
infinite dilution form phorone.
1 CH3 CH3
of BaCl2 of Ba of Cl –
2 C = O + H2 CHCOCH H2 + O = C
CH3 CH3
127 –1 2
76 139.5 ohm cm CH 3 CH 3
2 HCl
48. (c) d-d electronic transition from t2g eg state C CH.CO.CH C
–2H 2O CH 3
associated with an amount of energy which CH 3
comes under visible green region. Note : Two molecules of acetone condense
Here in the given compound Ni2+ is d8 to form mesityl oxide.
system so it is an octahedral compound CH3
hence splitting pattern will be C = O + H2 CHCOCH3
eg CH3

electronic CH 3
transition C = CH.CO.CH3
t2g CH 3
www.jeeneetbooks.in

WWW.IIT-NEET.XYZ

Solutions MT-159

55. (c) When glycerol is heated with oxalic acid


3
following reaction occurs. 60. (c) Number of t 1 / 2 3 . The rate of
CH 2OH HOOC CH2OOC.COOH
1
| | |
CHOH HOOC
100 110 C
CHOH 1000
| oxalic H 2O | dissociation after 3t1/2 = 125 .
CH 2OH acid CH 2OH ( 2) 3
CH 2OOCH CH 2OH 61. (a) t1/2 = 4 s T = 16
| |
CO 2 H 2O
CHOH CHOH HCOOH T 16
| | Formic acid n 4 ( T = n × t½)
CH2OH CH2OH t1 / 2 4
O
æ 1 ön æ 1 ö4 0.12
56. (d) R COCl + AlCl3 A = A o çç ÷÷÷ = 0.12 ´çç ÷÷÷ = = 0.0075 M
R C çè 2 ø èç 2 ø

+ AlCl4 16
Electrophile
57. (d) The total number of isomers for the complex Where Å = initial concentration & A =
compound concentration left after time t
62. (b) The presence of enzyme (catalyst)
[Cu II ( NH 3 ) 4 ][Pt II Cl 4 ] is four.. increases the speed of reaction by lowering
These four isomers are the energy barrier, i.e. a new path is followed
[Cu ( NH 3 )3 Cl] [Pt ( NH 3 )Cl 3 ], with lower activation energy.
[Cu ( NH 3 )Cl 3 ] [Pt ( NH 3 ) 3 Cl] ,
ET
[CuCl 4 ][Pt ( NH 3 ) 4 ] E'T

and Cu( NH 3 ) 4 PtCl 4 . Ea


Products
Energy

The isomer [Cu (NH3)2 Cl2][Pt (NH3)2 Cl2] Ea


1
does not exist due to both parts being
neutral. Reactants + catalyst
58. (b) There is a steady decrease in the radii as
Progress of reaction
the atomic number of the lanthanide
elements increases. For every additional Here ET is the threshold energy.
proton added in nucleus the corresponding Ea and Ea is energy of activation of reaction
electron goes to 4f subshell. 1
in absence and presence of catalyst
The shape of f -orbitals is very much
respectively.
diffused and they have poor shielding
63. (d) k = Ae–Ea/RT
effect. The effective nuclear charge
increases which causes the contraction in Ea
the size of electron charge cloud. This or log k = log A -
2.303 RT
contraction in size is quite regular and
known as Lanthanoid contraction. Comparing the above equation with
Since the change in the ionic radii in the y = mx + c
lanthanide series is very small, thus their 1
chemical properties are similar. y = log k, x
59. (a) Age of artifact t = T
Thus A plot of log10k vs 1/T should be a
2.303 Activity of fresh wood
log straight line, with slope equal to – Ea/2.303
Activity of dead wood RT and intercept equal to log A
0.693
Ea
t1 / 2 Slope =
log k 2.303RT

2.303 15.2
t 5760 log 5760 years
0.693 7.2
1/T
EBD_7443
www.jeeneetbooks.in

WWW.IIT-NEET.XYZ

MT-160 Target VITEEE

Ea O
Slope
2.303R MgBr C OMgBr
or E a –2.303R Slope (i) CO2 H3O
+

64. (d) The activation energy of reverse reaction


will depend upon whether the forward
reaction is exothermic or endothermic. O
As H = Ea (forward reaction) – Ea(backward reaction) C O H
For exothermic reaction H = –ve
– H = Ea(f) – Ea(b) or Ea(f) = Ea(b) – H + Mg(OH)Br
Ea(f) < Ea(b) 'P'
for endothermic reaction H = + ve Benzoic acid
H = Ea(f) + Ea(b) or Ea(f) = H + Ea(b) 71. (a)
Ea(f) > Ea(b).
65. (b) According to equation COCH3
2HI H2 + I2 C6H6
At t = 0 (2 moles) 0 0 CH3COOH +PCl5 CH3COCl AnhydAlCl3
At equilibrium (2 – 2 ) moles mole mole [A]
Friedle Craft
Total moles at equilibrium reaction
= 2 – 2 + + = 2 mole
66. (d) Aniline when treated with acetic anhydride OH OMgBr
+
forms acetanilide (nucleophilic substitution) H MgBrC2H5
C2H5 – C – CH3 ether C2H5 – C – CH3
O
hydrolysis
NH2 NH–C–CH3
O O
O
CH 3–C–O –C –CH 3 (C)
+ CH3–C–OH
72. (c) Remember that alkyl group present on
+ – benzene ring when oxidised by strong
N2 Cl OH
oxidising agent is always oxidised to –
H 2O COOH group, ignoring its length.
+ N2+HCl
67. (b) heat
73. (c) CH 3COONH 4 CH 3 CONH 2
H 2O
CH3 CH3 X
p- cresol

NO2 P2O5 H3O


CH 3CN CH 3COOH
H 2O
Y Z
68. (c) Sn HCl
reduction
74. (a) Measure of disorder of a system is
Nitrobenzene (B) N C nothing but Entropy. For a spontaneous
(A)
reaction, G < 0. As per Gibbs Helmnoltz
C HC l 3 KOH
equation,
NH–CH3 G= H– T S
(C)
Thus G is –ve only
Re duction
Na/C2 H5OH
When H = –ve (exothermic)
and S = +ve (increasing disorder)
N-methylaniline
H
69. (c) See action of HONO on P, S and T nitro 75. (a) S
alkanes. T
70. (b) Grignard reagent forms addition product H per mole 6000
with bubbled carbondioxide which on S( per mole )
T 273
hydrolysis with HCl yields benzoic acid.
21.98 JK 1mol 1
www.jeeneetbooks.in

WWW.IIT-NEET.XYZ

Solutions MT-161

76. (b) G = – P V = Work done


1 1 1
V is the change in molar volume in the sin 2 cot sin 2 tan 2
conversion of graphite to diamond. 2

V
12 12
10 3 L 1.91 10 3 L
2 2 4
3.31 2.25 2 5
1 (2)
Work done = –(–1.91 × 10–3) × P × 101.3 J
1 7 4
1895J mol –1 Hence, given expression tan
P 9794 atm 25 5
1.91 10 3 101.3
1 atm = 105 × 1.013 Pa 1 27 1 2
tan tan 1
25 25 4
P 9.92 108 Pa
82. (c) Given equation is
77. (c) In fluorite structure each F – ion is
surrounded by four Ca++ ions whereas each 2 cot 2 2 3 cot 4 cos ec 8 0
Ca++ is surrounded by eight F– ions, giving 2 2
a body centred cubic arrangement. Thus (cot 3) cot 4 cos ec 5 0
the co-ordination number of Ca++ = F– are (cot 3 ) 2 (cos ec 2) 2 0
8 and 4.
78. (a) p-type of semiconductors are produced cot 2 cosec 2 1
(a) due to metal deficiency defects
cot 3 and cosec = – 2
(b)by adding impurity containing less
electrons (i.e. atoms of group 13)
Ge belongs to Group 14 and In to Group 13. lies is the 4th quadrant and
6
Hence on doping, p-type semicondutor is
obtained. General solution is 2n ,n I
79. (b) Pepsin and Trypsin are two enzymes 6
involved in the process (hydrolysis of 83. (b) We have f (x) = x2 + 7, x R.
proteins) f(–2) = (–2)2+ 7 =11, f (2) = (2)2 + 7 = 11
Pepsin/ HCl
The images of distinct elements –2 and
Pr oteins Proteases and Peptones 2 of R are equal.
Stomach

Trypsin
‘f’ cannot be one-one.
Peptides
Chemotrypsin 3
(Pancreatic juice Intestine) 84. (d) sin 2 cos 1
5
Peptidases
Amino acids
(Intistine)
3 3
80. (b) Peptide bonds are present in enzyme. 2 sin cos 1 cos cos 1
5 5
H
3 3
N C 2sin cos 1
5 5
O
6 3 6 9
2 tan sin cos 1 sin sin 1 1
81. (c) sin 2 5 5 5 25
1 tan 2
1 tan 2 6 4 24
and cos 2 5 5 25
2
1 tan
85. (c) n(A) = p, n(B) = q
9 n(A × B) = pq = 7
1
3 16 7
cos 2 tan 1 So, possible values of p, q are 7, 1
4 9 25 p2 + q2 = (7)2 + (1)2 = 50.
1
16
EBD_7443
www.jeeneetbooks.in

WWW.IIT-NEET.XYZ

MT-162 Target VITEEE


1 3 1 1 3
log e (1 x 2 tan x ) 1 ( 1) ( 2) 0
86. (b) f ( x) ; 1 1 1
3
sin x
1
( 1) (3 1) 2(1 3 ) 0 .
f(0) must be equal to Lt f ( x ) 5
x 0
A (1, 0, 4)
1
2
x 2 tan x log(1 x 2 tan x ) x tan x
Lt
x 0 sin x 3
1
tan x x 3 2
Lt log(1 x 2 tan x ) x tan x (0, –1, 5)B M
:1
C
x 0 x sin x 3 (2, –3, 1)

= (1) (1) loge e = 1 Putting this value of in (1), we get the


[Note that, for the functions , where limit can be desired foot M of the perpendicular from A
obtained directly, there is no need to find LHL as
and RHL as in the above example]
87. (b) Given: x2 – y2 sec2 = 4 1 1 1
2. 3. 1 5
and x2 – sec2 + y2 = 16 5 , 5 , 5 1 4 13
i.e., , , .
1 1 1 3 3 3
x2 y2 1 1 1
1 5 5 5
4 4 cos 2
2 x
x2 y2 89. (a) f(x) = log ,
and 1 2 x
16 cos2 16
1
According to problem 2 x 2 x
f ( x) log log
4 4cos 2 16 16 cos 2 2 x 2 x
3
4 16 2 x
log f (x)
1 + cos2 = 3(1 – cos2 ) 4 cos2 = 2 2 x
1 3 f(x) is an odd function
cos , 1
2 4 4 2 x
88. (d) Let M be the foot of the perpendicular drawn log dx 0
2 x
from the point A to the line BC. Since the 1
point M lies on the line BC, it must divide 90. (d) Since both the roots are less than 2, we
BC in some ratio, say : 1, ( 1) . Then have
coordinates of M are given by 22
(i) D>0 15p – 66 < 0 p<
0 2 1 .( 3) 5 .1 5
, , , (ii) 4f (2) > 0 4 (25p2 – 25p – 50) > 0
1 1 1 p2 – p – 2 > 0 p < –1 or p > 2
2 3 5 1 20p 4
i.e. , ,
...(1) (iii) – <2 p<
1 1 1 2.4 5
Now d.rs of line BC are 2 – 0, –3 –(–1), 1–5, Taking the intersection of above three
i.e., 2, –2, –4 i.e., 1, –1, –2. solutions, we get p < – 1 or p (– , –1)
Also d.rs. of perpendicular line AM are
91. (b) Let R = x î yˆj zk̂ . Then
2 3 1 5
1, 0, 4
1 1 1 R B C B (R C) B 0
1 3 1 1 3
i.e., , , . î ĵ k̂
1 1 1
Now AM is to BC. Using the condition x 4 y 3 z 7 0
of perpendicularity, we get 1 1 1
www.jeeneetbooks.in

WWW.IIT-NEET.XYZ

Solutions MT-163

( y z 10) î ( z x 3)ˆj ( x y 7) k̂ 0
a2 b2 c2
y z 10, z x 3, x y 7 4 a b c
Also ;
(a 1) 2 (b 1) 2 (c 1) 2
R .A 0 2x 0 . y z 0 z 2x .
Solving, we obtain Apply R 3 R 3 2R 2 R 1
x = –1, y = –8, z = 2. Hence
a2 b2 c2
R î 8 ĵ 2 k̂ . 4 a b c
= 4 (a – b) (b – c) (c – a).
92. (c) We have 1 1 1
fog = { (1, 1), (2, 4), (3, 2), (4, 2)}
gof = { (1, 2), (2, 3), (3, 2) (4, 4)} Now given
The identical pair in fog and gof is (3, 2). 0 4(a b)(b c)(c a ) 0
Hence, fog (x) = gof (x) x=3 Clearly, atleast one of the factors must be
93. (b) Let y be the population at an instant t. Now zero. Hence the triangle must be isosceles.
population increase at a rate no. of ALTERNATE :
inhabitants If a = b then 0 as C1 and C2 become
dy dy identical. Similarly if b = c or c = a, 0.
y or = ky 95. (c) Probability that exactly one event out of A
dt dt
dy dy and B occur is P(A) P(B) 2P(A B)
y
= kdt Integrating = kdt + c
y P(A) P(B) 2P(A B) 1 a ...(1)
or log y = kt + c ...(i) Similarly,
In 1999, t = 0, population = 20,000
log 20,000 = c Put the value of c in (i) P(B) P(C) 2P(B C) 1 2a .....(2)
log y = kt + log 20,000 or log y – log 20000 and P(C) P(A) 2P(C A) 1 a
= kt ......(3)
y Now, Probability that at least one out of A,
or log = kt ...(ii)
20000 B, C will occur is
In 2004, t = 5, y = 25000 P(A B C) P(A) P(B) P(C)
25000 1 5
log = k × 5 k = log P(A B) P(B C) P(C A)
20000 5 4
y 1 5 P(A B C)
Equ (ii) as log log t
20000 5 4 1
In 2009, t = 10 {P(A) P(B) 2P(A B)}
2
y 1 5 5 {P(B) P(C) 2P(B C)}
log log 10 2 log
20000 5 4 4 {P(C) P(A) 2P(C A)}
2
5 25 y 25
log log P(A B C)
4 16 20000 16
25 1
y= × 20000 = 25 × 1250 = 31250 (1 a ) (1 2a ) (1 a ) a2
16 2
3 1
a2 b2 c2 a 2 2a a2 2a 1
2 2
94. (c) Let (a 1) 2 (b 1) 2 (c 1) 2 ;
1 1
(a 1) 2 (b 1) 2 (c 1) 2 (a 1) 2 [ a 1]
2 2
Apply R 2 R2 R 3 and take 4 common 96. (b) The given equation can be rewritten as
|cos x| |cos x|2 ..... to
81 64
EBD_7443
www.jeeneetbooks.in

WWW.IIT-NEET.XYZ

MT-164 Target VITEEE

81 /(1 |cos x|)


82 or
dy n m n m n m dy y
dx y x y x y x dx x
[since, 1 | cos x | | cos x | 2 .....to is
100. (b) The given system of equations can be
an infinite G.P.]
written in a single matrix equation :
1 1 2 2 x 1
2 1 | cos x | ;
1 | cos x | 2 4 2 1 y 2
6 6 z 3
1
| cos x | The given system of equations will have a
2
unique solution if and only if the coefficient
1 matrix is non-singular i.e iff
cos x , but x ( , )
2 2 2
2 4 2 1 0
x ,
3 3 6 6
97. (b) (2 2
6) 2( 4 6) 2(24 12 ) 0
98. (b)
or 3
11 30 0
or ( 2)( 2
2 15) 0 or 2
(only real root).
101. (a) Let A (a, b, c) be the fixed point on the
variable plane
Using the result |z1+z2 | |z1| + |z2|, we get |z| O(0, 0, 0)
+ |z – i| = |z| + |i – z| [since |z| = | – z|]
| z + i – z | = |i| = 1
minimum value of |z| + |z–i| is 1
ALTERNATE : 90° A(a, b, c)
We may obtain the above answer using M
(x, y, z)
geometrical representation. Consider a
triangle which has vertices O (origin) P (z)
Now D. R ‘s of OM are x 0, y 0, z 0
and Q (z – i) then
OP = |z|. OQ = | z – i| and PQ | z + i–z| = |i| = 1 i.e x, y, z
Now in a triangle sum of two sides third D.R.’s of MA are x – a, y – b, z – c.
side Since OM perpendicular MA
That is OP + OQ PQ x ( x a ) y( y b) z (z c) 0
[The equality holds, when O, P and Q are
collinear] x 2 y 2 z 2 ax by cz 0
Thus | z | + | z – i| 1 which isthe required locus and represents
a sphere.
99. (c) Given x m y n ( x y) m n 102. (a) Tangent drawn at any point P (x, y) is
Taking logarithm with base e of both sides, dy
we get Y y (X x ) ...(i)
dx
m logx + n log y = (m + n) log (x + y).
Differentiating both the sides with respect The triangle , whose area is given is OPT
to x, we get (see the shaded region in the adjacent
figure) If coordinates of T are (X, 0) then
d d d
m log x n log y (m n) log(x y ) y
dx dx dx
m n dy m n dy P(x, y)
or . 1
x y dx x y dx
m n m n dy
.
x y x y dx x
O N T
www.jeeneetbooks.in

WWW.IIT-NEET.XYZ

Solutions MT-165

dx
X x y (Put Y = 0 in (i)) area
dy Similarly, ˆj a ˆj a a 2 ˆj
1
of OPT | X y | 2a 2 (given) and kˆ a kˆ a a 3 kˆ
2
1 dx dx ˆi a ˆi ˆj a ˆj kˆ a kˆ
| xy y 2 | 2a 2 xy y 2 2a 2
2 dy dy
= 3a a1ˆi a 2 ˆj a 3kˆ 3a a 2a
dx x 2a 2
, which is linear in x
dy y y2 106. (b) The general term is
The integrating factor is k k
1 xk 2 4 2
y
dy
log y 1 1 k k 1 2k k4 k2
I.F. = e e
y
k
2 2
The solution is x
1 2a 1
dy c (k 1)2 k2
y 2 y
y
1 2k
x 2a 2 y 2
a2 2 (k 2 1 k)(k 2 1 k)
c x cy
y 2 y
103. (b) The no. of ways of selecting 3 out of 7 1 1 1
consonants = 7C3 = 35 2 k 2 2
k 1 k k 1
The no. of ways of selecting 2 out of 4
vowels = 4C2 = 6 1
Now these five letters after selection can [f (k) f (k 1)]
be arranged among themselves in 5P5 i.e., 2
120 ways. 1
Hence the required no. of words = 35 × 6 × Where f (k)
2
120 = 25200. k k 1
104. (c) The given curve n n
y 1
x y x .........(i) Sn xk [f (k) f (k 1)]
2
k 1 k 1
It cuts the x-axis, when y 0 x 1
The required point is (1, 0) 1 1 1
Taking log of (i), we get log (x + y) = y log x [f (1) f (n 1)] 1
2 2 2
Differentiating w.r. t x, n n 1

1 dy dy y 3 7
1 log x 107. (d) P (r) = , so P( r )
x y dx dx x 10 10
When, x = 1, y = 0, we get The probability of at least one rainy day in 7
1 dy dy 0
days
1 0
1 0 dx dx 1 7
7
dy P (A) = 1
1. So, the slope of normal = 1 10
dx
Now the probability that at least two rainy
Equation of the normal is y – 0 = 1 (x – 1)
days in 7 days
y x 1 0
7 6
7 7 3 7
105. (b) Let a a1ˆi a 2ˆj a 3kˆ P (B) = 1 C1
10 10 10
Now, ˆi a ˆi ˆˆ a
i.i ˆi.a ˆi a a1ˆi Hence,
EBD_7443
www.jeeneetbooks.in

WWW.IIT-NEET.XYZ

MT-166 Target VITEEE

P (B A) ALTERNATE :
P (B / A) The equations are 3l m n 0 ...(1)
P(A)
and 6nm – 2nl + 5lm = 0 ....(2)
7 6 Divide first by n and second by n2, we get
7 7 3 7
1 C1 l m
10 10 10 3 1 0 and
7 n n
7
1 m l l m
10 6 2 5 0.
n n n n
6
14 7 l m
1 Put x, y , we get
15 10 n n

7
7 3x y 1 0 and 6 y 2x 5xy 0 .
1 Solving above equations for x, and y we
10
get x = –1, y = –2 or x = –2, y = 1.
108. (a) X = a . b
l m
109. (a) Closure property : For a, b N, a + b N. If 2, 1 l :m:n 2 :1: 1 .
n n
( sumof two natural numbers is a natural
number.) If l 1,
m
2 l : m : n 1: 2 : 1 .
Associative Law : a, b, c N m n
Now proceed as above.
(a + b) + c = a + (b + c).
< N, + > is a semi group. ( x 2 1) ( x 2 1)
111. (a) dx dx
Existence of Identity : , let e be the x x 4 3x 2 1 2 2 1
x x 3
additive identity, then for all a N. x2
a+e= e+ a=a e=0 N
Additive identity does not exist in N 1
1
< N, + > is not a monoid. x2 dz
dx
110. (b) The given equations are 2
1 z2 1
3l m 5n 0 ....(i) x 1
x
and 6mn 2nl 5lm 0 ...(ii)
From (i), we have m = –3l – 5n. Putting m 1 1
Putting x z 1 dx dz
= –3l – 5n in (ii), we get x x2
6( 3l 5n )n 2nl 5l ( 3l 5n) 0
log z z2 1 C
2 n 2 3ln l 2 0 (n l )(2n l ) 0
either l = –n or l = –2n. 1 1
If l = –n, then putting l = –n in (i), we obtain m log x x2 3 C
= –2n. If l = –2n, then putting l = –2n in (i), we
x x2
obtain m = n. 112. (b) (f ig)(z) f (z) ig(z ) sin z i cos z
Thus, the direction ratios of two lines are = – i(cos z + isin z)
–n, –2n, n and = – i eiz = (say)
–2n, n, n i.e., 1, 2, –1 and –2, 1, 1.
Hence, the direction cosines are Now (f ig) * (f ig)(z)
1 2 1 2 1 1 = (f ig)(f ig)(z )
, , or , , . The angle
6 6 6 6 6 6 = (f ig)( ) f ( ) ig( )
between the lines is given by = sin i cos i(cos i sin )
1 2 2 1 1 1 1
cos i e i( ie iz ) iz
6 6 6 6 6 6 6 = ie ie = ie
113. (c) We suppose the distribution to be Binomial
1
cos 1 . with n = 10, p = 0.2, q = 1 – p =0.8
6
www.jeeneetbooks.in

WWW.IIT-NEET.XYZ

Solutions MT-167

The probability that not more than one exists and


defective is found. for a > 0, lim f (x) = lim x – 1 = a – 1 exists.
x a x a
n Hence, lim f(x) exists for all a 0.
P( k 0) P (k 1) qn pqn 1
x a
1
116. (c) Let the drawn tangents be PA and PB. AB
(0.8)10 10(0.2)(0.8)9 . is clearly the chord of contact of point P.
(0.8)9 [0.8 2] 2.8(0.8)9 y = x2 – 2x + 3
This value is very small so the Binomial
probabilities are approximated by Poisson
probabilities then
m np 10 0.2 2 y=4
The probability that not more than one A B
defective is found.
P(k 0) P(k 1) e m me m
e 2 2e 2 3e 2
P(1, 0) X
114. (a) The equation of the circle through (1, 0), (0,
1) and (0, 0) is x2 + y2 – x – y = 0 Thus equation of AB is
It passes through (2k, 3k)
1
So, 4k2 + 9k2 – 2k – 3k = 0 or 13k2 – 5k = 0 . (y 0) = x.1 – (2 + 1) + 3 i.e., y = 4
2
5 x coordinates of points A and B will be
k(13k – 5) = 0 k = 0 or k = given by,
13
But k 0 [ all the four points are distinct) x2 – 2x + 3 = 4 i.e., x2 – 2x –1 = 0 x = 1
5 2
k .
13
Thus AB = 2 2 units.
115. (b) Given,
1
| x| 1 , x 0 x 1, x 0 Hence PAB (2 2).4 4 2 sq. units
f (x) = 0 , x 0 0 , x 0 2
| x| 1 , x 0 x 1 , x 0 Now area bounded by line AB and parabola
is equal to
Let us first check the existence of limit of
f (x) at x = 0. 1 2
At x = 0, (4 2 (x 2 2x 3)) dx
RHL = lim f (x) 1 2
x 0

= lim f (0 h) lim(0 h) 1 4 2
h 0 h 0 = sq. units.
3
= lim h – 1 = 0 – 1 = – 1 Thus required area
h 0

LHL = lim f (x) = limf (0 h) 4 2 8 2


x 0 h 0 = 4 2 sq. units.
= lim (0 h) 1 3 3
h 0 117. (c) Let any point P(h, k) will satisfy
= limh
h 0
1 0 1 1 y2 = 4ax i.e, k2 = 4ah ...(i)
RHL LHL Let a line OP makes an angle from the x-
At x = 0, limit does not exist. axis.
Note that for any a < 0 or a > 0, PA
limf (x) exists, In OAP, sin
x a OP
as for a < 0, limf (x) = lim – x + 1 = –a + 1 k
x a x a sin
l
EBD_7443
www.jeeneetbooks.in

WWW.IIT-NEET.XYZ

MT-168 Target VITEEE


k = l sin
1
OA where p(x) = and q(x) = 1 x 2
and cos 2x 1
OP
h Now, Domain of p(x) exist when 2x – 1 0
cos h = cos
l 1
Y x= and 2x – 1 > 0
2
P(h, k)
1 1
x= and x
l k 2 2

X X 1
O h A x ,
2
and domain of q(x) exists when

1 x2 0 x2 1 |x| 1
Y 1 x 1
Hence, from eq.(i), we get
l2sin2 = 4a × l cos (put k = l sin , h = 1
l cos ) Common domain is ,1
2
4a cos 120. (a) In the absence of any condition, no book
l
sin 2 need to be selected on a particular subject
118. (b) For the balloon, and all the selected books may be on a
981 g single subject. If xn stands for the selection
5 cm / sec 2
u 0, f 122 cm / sec 2 of n (0 n 8) books on one subject, the
8 8 8
and t = 30 seconds. total number of ways of selecting 8 books
Let h be the height and v the velocity of the out of the lot.
balloon after 30 seconds. Then = the coeff. of x8 in (x 0 x1 x 2 .... x 8 )4 ,
g 15 4 being the number of subjects
v 0 .30 g cm / sec 4
8 4 1 x9
and = the coeff. of x8 in
1 x
1 g 2 225
h 0 . .30 g = the coeff. of x8 in (1 x) 4 (1 x 9 ) 4
2 8 4
Now the initial velocity of the released body = the coeff of x8 in (1 x ) 4 11
C8 165
will be 15/4 g cm/sec in the upward direction.
Suppose the body reaches the ground in t 121. (a)
sec. 122 (d)
225 15
1 2
g gt
gt 123 (c)
4 4
2
or 2 t 2 15t 225 0 or ( t 15) ( 2t 15) 0 124. (a) Abate means to reduce in degree or
15 intensity; Augment means to increase;
t 15 sec . sin ce t Provoke means to stimulate or give rise to
2
(a reaction or emotion, typically a strong or
1 unwelcome one) in someone; Wane means
119. (a) Given, f(x) = 1 x2
2x 1 to decrease gradually in size, number,
strength, or intensity
= p(x) – q(x)
125 (a)
www.jeeneetbooks.in

WWW.IIT-NEET.XYZ

Solutions MT-169

MOCK TEST 5

1. (c) Let us place the charges as shown on the 5. (a) Let unknown resistance be X. Then
regular hexagon condition of Wheatstone's bridge gives

P Q X 20r
+ , where r is resistance of wire per
R 80r
cm.
U +R
X R=1

+T S
E due to each charge at O is same in magni-
tude, the arrow shows the direction of E
due to each charge
E R E u E 0 where E 0 is electric field P = 20r Q = 80r
20 cm
due to +ve charge placed at R (100 – 20) cm
2. (a) Let m g of steam get condensed into water
(By heat loss). This happens in following 20 1
X R 1 0.25
two steps. 80 4
6. (a) R1 = 3.1 at T = 30°C
100°C
100°C Water R2 = 4.5 at T = 100°C
Steam (H1= m × 540) We have, R = R0 (1 + t)
R1 = R0 [1 + (30)]
R2 = R0 [1 + (100)]
[H2= m ×1× (100× 90)]
R1 1 30
R2 1 100
90°C 3.1 1 30 1
Water 0.0064 C
4.5 1 100
Heat gained by water (20°C) to raise it's E1 E2
temperature upto 90°C = 22 × 1× (90 – 20) 18 12
r r2 2 1
Hence , in equilibrium, heat lost = Heat gain 7. (d) V = 11 1 = 1 1
14 V
m × 540 + m × 1 × (100 – 90) = 22 × 1 × (90 r1 r2 2 1
– 20)
(Since the cells are in parallel).
m = 2.8 gm 8. (c) A galvanometer can be converted into a
The net mass of the water present in the voltmeter by connecting the high
mixture = 22 +2.8 =24.8 gm. ressistance in series with the galvanometer
3. (b) In parallel, potential is same, say V so that only a small amount of current
passes through it.
Q1 C1V C1
mv v
Q2 C2V C2 9. (d) r r
qB B
4. (b)
EBD_7443
www.jeeneetbooks.in

WWW.IIT-NEET.XYZ

MT-170 Target VITEEE


10. (c) For a perfectly rigid body strain produced 1
is zero for the given force applied, so R L
C
Y = stress/strain =
11. (a) If the electric field is switched off, and the 1 1
C= =
same magnetic field is maintained, the (R L) 2 f (R 2 f L)
electrons move in a circular orbit and
c
electron will travel a magnetic field to its 17. (b) Size of particle
velocity.
12. (d) Power of heating coil = 100 W and voltage 3 108
(V) = 220 volts. When the heating coil is 3 10 6
cut into two equal parts and these parts
are joined in parallel, then the resistance of = 1014 Hertz
the coil is reduced to one-fourth of the However, when frequency is higher than
previous value. Therefore energy liberated this, wavelength is still smaller. Resolution
per second becomes 4 times i.e., 4 × 100 = becomes better.
400 J. 18. (c) For first minimum, a sin = n = 1
13. (a) Eddy currents are produced when a metal 5000 10 –10
is kept in a varying magnetic field. sin 0.5
a 0.001 10 – 3
n2 A = 30°
14. (d) Self inductance of a solenoid = 19. (c)
20. (d) According to law of conservation of
So, self induction n2 momentum the third piece has momentum
So, inductance becomes 4 times when n is
doubled. 1 –(3iˆ 4j)
ˆ kg ms–1
15. (c) The time constant for resonance circuit, Impulse = Average force × time
T CR
Growth of charge in a circuit containing Impulse
Average force
capacitance and resistance is given by the time
formula, q q 0 (1 e / CR )
Change in momentum
CR is known as time constant in this time
formula.
16. (d) From figure,
–(3iˆ 4ˆj)kg ms –1
10 –4 s
21. (d) K-electron capture is accompanied by the
characterstic X-rays emission.
22. (c) Among given atoms/species, doubly
ionised lithium will produce shortest
wavelength emission from the transition
n = 2 to n = 1.
1 23. (a) According the Moseley’s law
L
tan 45º = C f a(Z b) f a 2 (Z b) 2
R
C
1 a 2 (Z b) 2
L =R
C
for k line, b = 1
www.jeeneetbooks.in

WWW.IIT-NEET.XYZ

Solutions MT-171

From (i) n = 2 i.e. in two half lives


t = nT = 2 × 4 = 8 months
2 (Z1 1)2 4 (11 1)2 31. (b) Mass defect, M = 8.03170 – 8.00774
1 (Z 2 1)2 (Z2 1)2 = 0.02396
(3Li6 + 1H2 2 2He4)
10 6.017 + 2.0147 2(4.00387)
Z2 1 Z2 6 8.0317 8.00774
2
Mass defect = 8.0317 – 8.00774
24. (b) 2 d sin = n = 0.02396
n 1 20 and 1 a.m.u. = 931 MeV
or d 20 nm Thus is this case the mass defect 0.02396
2 sin 2 sin 30º
a.m.u. is equivalent to (0.02396 × 931 MeV)
E 22 MeV
25. (d) Momentum of a photon 32. (d) 7N14 + 2He4 8O16 + aXb
c
Balancing the eqation, we have, a = 1, b = 2
2
1X whichis deutron
19
10 1.6 10
= 33. (a) The given truth table is of
3 10 8 (OR gate + NOT gate) NOR gate
27 1
5.33 10 kg ms Vs 0.01
ic 6
34. (d) = 50 500 10 A
hc R in 1000
26. (d) eVs W0 . If decreases, V increases
s = 500 A
35. (d) (a) is a NAND gate so output is 1 1 1 0
h h 1 v2 / c2
27. (b) 0 ( c) (b) is a NOR gate so output is 0 1 1 0
mv m0 v
(c) is a NAND gate so output is 0 1 0 1
28. (a) F = –5iˆ 9cos 60 iˆ 9sin 60 ˆj 3 ˆj
(d) is a XOR gate so output is 0 0 0
9ˆ 9 3ˆ
= –5iˆ i j 3 ˆj
2 2 0
1
iˆ 9 3 1
= 3 ˆj
2 2
Following is NAND Gate
s = –3iˆ .
Y A.B
iˆ 9 3 36. (d) On the basis of given graph, following table
W = F .s = 3 ˆj .( 3iˆ)
2 2 is possible.
= 1.5 J. A B C
R t 975 5 0 0 0
29. (b) e e
R0 9750 1 1 1
5 log e 1 log e 10 log e 10 0 1 0
1 0 0
2.3026 log 10 10 1
0.461 min It is the truth table of AND gate.
5
30. (c) Substance left undecayed - 6
37. (a) a g [using v = u + at]
3 1 10
N0 N0 N0
4 4 6 6
= 0.06
n 10 g 10 10
N 1 1
38. (d) Few advantage of optical fibres are that the
N0 4 2 number of signals carried by optical fibres
EBD_7443
www.jeeneetbooks.in

WWW.IIT-NEET.XYZ

MT-172 Target VITEEE


is much more than that carried by the Cu presence of electron withdrawing – NO2
wire or radio waves. Optical fibres are group which makes o - and p - carbon
practically free from electromagnetic electron deficient and hence liable to be
interference and problem of cross talks attacked by OH– (a nucleophile)
whereas ordinary cables and microwave NH 2 Cl
links suffer a lot from it. Br Br Br Br
39. (a)
HONO
1 43. (c) CuCl
40. (c) Ki m1u12 , Sandomeye
2
NO 2 NO 2
1 m1 m 2 Cl
Kf m1v12 , v1 u1 Br Br
2 m1 m 2
Sn / HCl
Fractional loss

1 1 NH 2
m1u12 m1 v12
Ki Kf 2 2 Cl
Ki 1 Br Br
m1u12
2 HONO
H 3PO 2
elimenatio n of NH 2

v12 m1 m 2 2 4m1m 2 NO2


1 1
u12 m1 m 2 2
m1 m 2 2
nitration
44. (b)
m2 m ; m1 nm HNO3 , H 2SO 4

Kf 4n NO2
1
Ki 2
1 n
Br2
Energy transfer is maximum when K f 0 FeBr3
Br
4n
2
1 4n 1 n2 2n NH 2
1 n
Sn / HCl
n 2 1 2n 0 NaOH
Br
n 12 0 n 1 ie. m2 m,
NO2
m1 m HNO 3, H2SO 4 Cl2/Fe
Transfer will be maximum when both 45. (b)
333 K
masses are equal and one is at rest.
41. (b) As far as characterstic of IR spectrum for
CH3CH2OH is NO2 NH2
O–H streching frequency 3391 cm–1
Cl /Fe reduction
O–H streching frequency 2981 cm–1
O–H streching frequency 1055 cm–1
Cl Cl
42. (a) Although benzene does not undergo
nucleophilic substitution, nitrobenzene (a) In case chlorination is done earlier
undergoes such reaction due to the than nitration, chlorobenzene formed
www.jeeneetbooks.in

WWW.IIT-NEET.XYZ

Solutions MT-173

at first step would introduce –NO2 53. (b) Cobalt - 60 is used for the treatment of blood
group in ortho-position, not in m- cancer.
position 54. (d) Given [A] = 0.01 M
Rate = 2.0 × 10–5 mol L–1 S–1
(b) Again if –NO2 group is reduced earlier For a first order reaction
than the chlorination step, –NH2 group Rate = k[A]
formed on reduction will again
introduce –Cl in o-position 2.0 10 5
k= = 2 × 10–3
46. (a) The Lanthanides are transition metals from [0.01]
Atomic numbers 58 (Ce) to 71(Lu).
0.693
Hence the electron configulation becomes: t1/2 = = 347 sec.
(n –2) f 1– 14 (n – 1) s2p6 d0 – 1 ns2. 2 10 3
47. (a) IUPAC name of sodium nitroprusside K 310
55. (b) Temperature coefficient = = 2.3
Na 2 [Fe(CN) 5 NO] is sodium K 300
pentacyanonitrosoyl ferrate (III) because
or K 310 = 2.3 K 300
in it NO is neutral ligand. Hence
2×O.N. of Na + O.N. of Fe + 5×O.N. of CN 56. (a) The rate law equation contains [H ]2 ,
1×O.N. of NO = 0 term. The rate will charge with change in
2×(+1) + O.N. of Fe + 5 ×(–1) +1×0 = 0 pH and new rate will be
O.N. of Fe = 5 – 2 = +3, Hence ferrate (III) [2]2 = 4 times the old rate
48. (d) Co here is in +3 oxidation state 57. (c) It is bimolecular first order reaction since
Rate [N2O5]
Co
58. (d) A 2 (g) B 2(g ) 3C(g) D(g)
step-1 step-2
Co3+ since the steps 1 and 2 are exothermic hence
Unpaired electrons = 4 and sp 3 d 2 low temprature will favour both the
hybridisation and octahedral shape. reactions. In step - 1 moles are increasing
49. (c) Diaminodichloroplatinum (II) commonly hence low pressure will favour it. In step 2
known as cis - platin is found to have moles are decreasing, hence high pressure
anticancer property. will favour it.
50. (b) The cause of showing different oxidation 59. (a) Among the given options, only PCl5 can
is due to the fact that there is only a small convert an alcoholic group as well as a
difference between the energies of electron carboxyl group to chloride.
in the ns orbitals and (n – 1)d orbitals with PCl5
the result both ns as well (n – 1)d electrons RCH 2OH RCH 2Cl
can be used for compound formation. PCl5
Lesser energy difference between 5f and RCOOH RCOCl
6d orbitals than between 4f and 5d orbitals
result in larger no. of oxidation state. O OH
|| |
HCN
51. (c) d-d electronic transition causes absorption 60. (a) R – C– R R– C – CN
KCN |
of energy at red visible wave length.
R (A )
In cupric chloride CuCl 2, cupper is in +2
state and hence it is d9 system. In five d-
orbitals there is one unpaired electron OH
|
present which absorbs energy due to d-d Re duction by
R – C – CH 2 NH 2
transition. LiAlH 4 ( B) |
52. (b) Lead Apron since it absorbs harmful R
radiations.
EBD_7443
www.jeeneetbooks.in

WWW.IIT-NEET.XYZ

MT-174 Target VITEEE


61. (c) CH 3COOH CaCO3 (CH 3COO) 2 Ca 2 º
e Cu Cu ; E 2 0.15V; ...(ii)
CH 3
Heat
CO Cu e Cu; E3º ? ... (iii)
CH 3
Now, G1º nFE1º 2 0.34F
I 2 NaOH
CHI3 NaI CH 3COONa 3H 2 O
G º2 1 0.15F , G 3º 1 E 3º F
62. (c) Due to resonance in carboxylate ion, the
double bond character of C = O bond in Again, G1º G º2 G 3º
carboxylic acids is greatly reduced as
compared to that in aldehydes and ketones. 0.68F 0.15F E3º F
O O O E3º 0.68 0.15 0.53V
C C C
O O O E ºcell º
E cathode º
(Cu / Cu ) E anode (Cu 2 / Cu )
63. (d) We know that = 0.53 – 0.15 = 0.38 V.
G= H– T S 1000 1.06 10 2 1000
68. (a) m
When the reaction is in equilibrium, G = 0 M 0.1
H 1.06 10 2
0 H T S T
S 69. (b) H2 – O2 fuel cell supply power for pressure.
2 30 1000 70. (b) Corrosion is an electrochemical
T 285.7 K phenomenon.
2 105
64. (b) In graphite, the electrons are spread out Eq.wt i t
71. (b) Mass deposited =
between the sheets. 96500
65. (d) The appearance of colour in solid alkali
108 9650
metal halide is due to presence of F-centres 10.8 g
found as defect in the crystal structure. 96500
66. (d) In body centred cubic lattice one molecule 72. (b) Four primary alcohols of C5 H11 OH are
of CsBr is within one unit cell. possible. These are:
Atomic mass of unit cell (i) CH 3CH 2 CH 2 CH 2 CH 2 OH
= 133 + 80 = 213 a.m.u
(ii) CH 3CH 2 CH CH 2 OH
Volume of cell = (436.6 × 10–12)3 |
n ´ at.wt. CH 3
Density =
Av.no.´ vol.of unit all cell (iii) CH 3 CH CH 2CH 2 OH
213 |
Density = CH 3
23 12 3
6.02 10 ( 436.6 10 )
CH 3
ZM |
(iv) CH 3 C CH 2 OH
a 3 NA |
CH 3
213 107
42.5 gm / cm 3 73. (b) Greater the stability of the intermediate
6.02 (436.6) 3 carbocation, more reactive is the alcohol.
2 Since 2-methylpropan-2-ol generates 3°
67. (c) 2 Cu Cu Cu carbocation, therefore, it reacts fastest with
º HBr.
2e Cu 2
Cu ; E1 0.34V; ...(i)
www.jeeneetbooks.in

WWW.IIT-NEET.XYZ

Solutions MT-175

74. (b) Secondary alcohols on oxidation give 81. (a) Given, k sin + cos 2 = 2k – 7
ketones. k sin + 1 – 2 sin 2 = 2k – 7
Note : – Primary alcohols from aldehydes. 2sin2 – k sin + (2k – 8) = 0
For the existence of real roots, discriminant
R R 0.
[O]
CHOH C=O k2 – 4 × 2 (2k – 8) 0
R R (k – 8)2 0, which is always true.
Isopropyl Ketone
Roots of the quadr atic equation are
alcohol
k 4
, 2, but sin 2.
75. (b) Due to greater electronegativity of sp2 - 2
hybridized carbon atoms of the benzene
ring, diaryl ethers are not attacked by k 4
sin but 1 sin 1
nucleophiles like I–. 2
76. (c) (CH 3CH 2 ) 2 O (CH 3 CO) 2 O k 4
1 1
Ether Acid anhydride 2
anhy. AlCl3 2 k 4 2 2 k 6
CH 3COOCH 2 CH 3
Ester 82. (b) Given parabola is y 2 4ax
77. (c) Conjugate base of phenol, i.e. phenoxide
Let P ( x1, y1 ) ... (1)
ion is a weaker base than the HCO3– ,
)
conjugate base of H2CO3. This is due to ,2 2at 1
at 1
resonance, in HCO3– in which all resonating Q(
structures are equivalent.
CH2 P (x1,y 1)
On keeping O O
78. (a) 3 HCHO CH2 CH2 R (a
t2 2, 2
aq. solution O at2 )
Trioxane Let the tangents from P touch the parabola
(meta formaldehyde) at Q(at12, 2at1) and R (at22, 2at2), then P is
COOH H2N the point of intersection of tangents.
79. (c) CH 2 + CO x1 = at1 t2 and y1 = a (t1 + t2)
COOH H2 N or t1t2 = x1/a and t1 + t2 = y1 / a ...(2)
Malonic ester Now QR (at12 at 22 ) 2 ( 2at 1 2at 2 ) 2
CO NH
CH 2 CO a 2 ( t1 t 2 ) 2 [( t1 t 2 ) 2 4]
2 H 2O
CO NH
Barbituric acid | a || t1 t 2 | ( t1 t 2 )2 4
80. (a)
OH O.COC6H5 | a | ( t1 t 2 )2 4 t1t 2 . ( t1 t 2 )2 4
aq. NaOH
+ C6H5COCl
y12 4x1 y12
|a| . 4 [using (2)]
phenyl benzoate a2 a a2
The function of NaOH is
(i) To convert phenol to more stronger 1
( y12 4ax1 ).(y12 4a 2 )
nucleophile PhO– |a|
(ii) To neutralize the acid formed
EBD_7443
www.jeeneetbooks.in

WWW.IIT-NEET.XYZ

MT-176 Target VITEEE


83. (a) We have, f (1) 1! 1, f (2) 2! 2,
31
A B 1 cos( A B)
1 f (3) 3! 3 2 1 6 ,
32 1
tan
31
...(i) f (4) = 4! = 4 × 3 × 2 × 1 = 24
2 1 cos( A B) 1 63
32 Range of f = {1, 2, 6, 24}
Using Napier’s analogy, 86. (b) Let f (x) = ax 2 bx c . f (0 ) 2 c 2
A B a b C Also f’(x) = 2ax + b and f”(x) = 2a
tan cot ...(ii)
2 a b 2 f”(0) = 4 2a = 4 a = 2 and f’(0) = – 3
From (i) and (ii) we get, b = –3
f(x) = 2x2 – 3x + 2
a b C 1 5 4 C 1
cot cot 1 1
a b 2 63 5 4 2 63 2x 2 3x 2 dx 2 2x 2 2 dx
[given a = 5, b = 4] 1 0
C 9 C 7 [Separating even & odd parts]
cot tan
2 63 2 3 1 1
2 x3
4 x 1 dx 4 x
C 7 3
1 tan 21 0 0
2 9 1
cos C
C 7 8 1 16
1 tan 2 1 1 4
2 9 3 3
Now, using cosine rule, 87. (b) The equation of tangent at any pont
c2 a2 b2 2ab cos C x sec y tan
1
P (a sec , b tan ) is
1 a b
52 42 2 5 4 = 36
8 Y

c=6 P
84. (a) As c is coplanar with a and b, we take,
c= a+ b ...(1) F

where , are scalars. X


C S
As c is pependicular to a, c . a = 0
from (1) we get, 0= a. a+ b.a
0 6) 2 2 1) 3(2 ) a
x
e
2 .
Thus, c = (a – 2b) = (–3j + 3k) Its point of intersection with the directrix
= 3 (– j + k)
a a b(sec e)
| c |2 9 2
(1 1) 18 2
1 18 2 x= is F ,
e e e tan
1 1
c ( j k ). b(sec e)
3 2 2 Slope of SF, m1 2
,
a tan (e 1)
1 where S is the focus (ae, 0).
Thus, we may take c ( j k ).
2
b tan
Also the slope of PS, m 2
x! a (sec e)
85. (c) f (x) x Px 1 x!
(x x 1)! r
Clearly m1m 2 1 . That is SF PF .
n n!
Pr , see Permutatio ns .
(n r )! 2
www.jeeneetbooks.in

WWW.IIT-NEET.XYZ

Solutions MT-177

88. (a) In a right angled triangle the mid-point of


3 1 1 1
the hypotenuse is equidistant from the .
vertices 4 3 3 12
A 91. (a) We have, m sin = n sin( + 2 )
b
sin 2 m
/2 x
3b
sin n
–G
C Using componendo and dividendo, we get
x
x D sin 2 sin m n
/2
B sin 2 – sin m–n
W 2 2 –
DB = DC = DA 2 sin cos
2 2 m n
Hence, DAB is isosceles
2 2 – m–n
2 cos sin
2 2
A B
2 2 sin cos m n
where is the angle which BC makes with 2 cos sin m–n
the vertical. When the triangle is suspended m n
from right angle A by a string then line of tan( + ) . cot =
m–n
action of W, i.e., the vertical through G must
pass through A, so, that median AD is 2 3 1
92. (c) P A , P B , P A B
vertical. 5 10 5
Now sin = 2 sin ( /2) cos ( /2) 2 3
P A 1 P A 1
b 3b 6 3 5 5
2. .
b 10 b 10 10 5 3 7
P B 1 P B 1
3 10 10
1
sin 2 3 1 1
5 P A B P A P B P A B
5 10 5 2
89. (d) We have | (a × b).c | = | a || b || c |
| a || b || sin n.c.| = | a || b || c | 1 1
P A B P A B 1 P A B 1
| a || b | | c || sin cos | = | a || b || c | 2 2
P A B P A B
|sin | |cos | and =0 P A |B P B |A .
2 P B P A
a b and c | | n
a b and c is perpendicular to both a 1/ 2 1/ 2 1 50 25
= .
and b 7 /10 3 / 5 4 21 42
a, b, c are mutually perpendicular 93. (d) Given: xdy – ydx = 0
Hence, a.b. = b.c = c.a. = 0 Dividing by xy on both sides,
90. (a) From venn diagram, we can see that we get:
A C dy dx dy dx
- =0
y x y x
By integrating on both sides, we get,
A B C log y = log x + log c
B
A B C y
log = log c y = cx or y – cx = 0
x
P(B C) which represents a straight line passing
P(B) P(A B C) P(A B C) through origin.
EBD_7443
www.jeeneetbooks.in

WWW.IIT-NEET.XYZ

MT-178 Target VITEEE


94. (b) Given ,c 0, b 0, 1
We find that (Adj A)B = 0 if p and
b 0 and c 0 2
Clearly, and have opposite signs and 0 if p = –2
< 97. (b) Differentiating first
0 and 0 0 dy
3x 2 3y 2 6xy 0
Further 0 | | | | dx

| | ( 0 | | dy x2 y2
m1
Hence, 0 | | dx 2xy

95. (a) Put x = z6 dx = 6z5 dz Differentiating second


dy dy
x
3
x2 6
x 6xy 3x 2 3y 2 0
3
dx dx dx
x(1 x)
dy 2xy
6 4 5 m2
(z z z)6z dz dx y 2
x2
z 6 (1 z 2 )
We note that m1m2 = –1
The curves are orthogonal curves.
z5 z 3 1 1
6 dz 6 z3 dz a b
2 2
z 1 z 1 98. (d) Commutative: a * b = and b * a =
b 1 a 1
3 4 a *b b*a
z 6 tan 1 z C
2 * is not commutative.
3 2/3 Associative:
x 6 tan 1 x1 / 6 C Now,
2
a
1 3 2 x a b 1 a
a *b *c *c
96. (a) Here A 0 p 2 ,x0 y , b 1 c 1 b 1 c 1
0 2p 1 1 z and
b a a c 1
1 a * b*c a*
c 1 b b c 1
B 3 1
c 1
2 So, clearly (a * b) * c a * (b * c)
Now |A| = –(2p +1) (p + 2), | A | = 0 Hence, * is not associative.
1 99. (a) (a . b' c) . a ' [(0.1) 1].1
= -2 or
2 (a=0 a' 1
(p 2)(2p 1) 6p 5 2p 4 = (0 + 1).1 b=0 b' 1 )
adjA 0 1 p 2 = 1.1 = 1
0 2p 1 0 100. (a) We can write as,

1 cot cot 2
2p2 19p 9
sin 2 sin 2 sin 2 1 cot cot 2
So, (adjA)B 2p 1 1 cot cot 2
6p 3
www.jeeneetbooks.in

WWW.IIT-NEET.XYZ

Solutions MT-179

2
sin 2 sin 2 (cot (n 2)(n 1)n.(n 1) 57
= sin cot )
8.7.6.5.4.3.2.1 16
(cot cot )(cot cot ) (n + 2) (n + 1) n (n – 1) = 143640
(n2 + n – 2) (n 2 + n) = 143640
= sin( ) sin( ) sin( )
(n2 + n)2 – 2(n2 + n) + 1 = 143640 + 1
It is clear from here that cannot exceed 1. (n2 + n – 1)2 = (379)2
[ sin / 1, for any R] n2 + n – 1 = 379 [ n2 + n – 1 > 0]
101. (a) 2 2 2
We have x + y = ...(i) is a circle with n2 + n – 1 – 379 = 0
vertex (0, 0) and radius . n2 + n – 380 = 0 (n + 20) (n – 19) = 0
and y = sinx ...(ii) n = – 20, n = 19
Y n is not negative.
2 2 2
x+y = n = 19
y = sinx 104. (c) Let the sides of the ABC be a = n, b = n + 1,
c=n +2
( ,0) Where n is a natural number
X Clearly, C is the greatest and A is the least
(0, 0) angle
As given, C = 2 A
sin C = sin 2 A
i.e. sin C = 2in A cos A
Required area = area of shaded region
= Area of the circle in 1st quadrant – b2 c2 a 2
i.e. k c = 2 k a ×
2 2bc
sin xdx = cos x
0 4 0 or b c2 = a (b2 + c2 – a2)
3 3 Substituting the values of a, b, c we get
8
1 1 sq. units (n + 1) (n + 2)2 = n [(n + 1)2 + (n + 2)2 – n2]
4 4 or (n + 1) (n + 2)2 = n (n2 + 6 n + 5)
102. (c) z 3 iz 2 2i (z i )(z 2 2iz 2) 0 or (n + 1) (n + 2)2 = n (n + 1) (n + 5)
or (n + 2)2 = n (n + 5), [Since n –1]
z i, 1 i, – 1 – i
or n = 4
Let A = (0,1), B = (1,–1), C=(–1, –1), Hence the sides are 4, 5 and 6.
0 1 1 x2 y2
1 105. (b) Differentiating the equation =1
ABC 1 1 1 a2 b2
2
1 1 1 w.r.t. x, we get

1 2x 2y dy
| 2 2| 2 2 =0
2 a b2 dx
n 2
103. (b) Given C8 : n 2 P4 57 :16 x2 xy dy
or 2 =0
n n! a b 2 dx
n 2
C8 Cr
57 r!(n r)!
n 2 n! y2 xy dy x2 y2
P4 16 n
and Pr or 1 =0 1
(n r)! b2 b2 dx a2 b2
(n 2)! (n 2 4)! 57 Differentiating again w.r.t. x, we get
8!(n 2 8)! (n 2)! 16
EBD_7443
www.jeeneetbooks.in

WWW.IIT-NEET.XYZ

MT-180 Target VITEEE


x 0, y 0
2y dy y dy x dy dy xy d 2 y 107. (b) There are 11 letters in the given word which
. =0
b2 dx b2 dx b 2 dx dx b 2 dx 2 are as follows (NNN) (EEE) (DD)IPT
Five letters can be selected in the following
d2 y dy
2
dy manners :
or xy 2
x y =0 (i) All letters different : 6C5 = 6
dx dx dx (ii) Two similar and three different : 3C1.
5C = 30
comparing with the given differential 3
equation, we get A = 1. (iii) Three similar and two different : 2C1.
5C = 20
106. (c) The given information can be expressed as 2
given in the diagram: (iv) Three similar and two similar : 2C1. 2C1
In order to simply, we assume that 1 unit = =4
1000 bricks (v) Two similar, two similar and one
Suppose that depot A supplies x units to P different :
3C . 4C = 12
and y units to Q, so that depot A supplies 2 1
(30 – x – y) bricks to builder R. Total selections = 6 + 30 + 20 + 4 + 12 = 72
Now, as P requires a total of 15000 bricks, it 108. (c) In the definition of the function, b 0, for
requires (15 – x) units from depot B. then f(x) will be undefined in x > 0.
Similarly, Q requires (20 – y) units from B f(x) is continuous at x = 0,
and R requires 15 – (30 – x – y) = x + y – 15 LHL = RHL = f(0)
units from B.
Using the transportation cost given in sin(a 1)x sin x
lim
table, total transportation cost. x 0 x
x 0
Z = 40x + 20y + 20(30 – x – y) + 20(15 – x)
+ 60(20 – y) + 40(x + y – 15) x bx 2 x
= 40x – 20y + 1500 lim 3/ 2
c
Obviously the constraints are that all x 0 bx
x 0
quantities of bricks supplied from A and B
sin(a 1)x sin x
to P, Q, R are non-negative. lim
x 0 x x
A
30 units 1 bx 1
lim c
x 30 x 0 bx
y – x–
0
` 20

`4 `2 y (1 bx ) 1
0 (a 1) 1 lim c
P Q R x 0 bx ( 1 bx 1)
15 units 20 units 15 units
1
15 ` 40 ) a+2 lim c
60

–y
y


`2 x +6 5 x 0 1 bx 1

`

x
20

1
0
B (7 – y –
8– =x+ 1
20 units a+2 c
2
x 0, y 0, 30 – x – y 0, 15 – x 0, 20 – y 0, 3 1
x + y – 15 0. a ,c ,b 0
2 2
Since, 1500 is a constant, hence instead
of minimizing Z = 40x – 20y + 1500, we 1 5
109. (b) Let cos , then 0 and
can minimize Z = 40x – 20y. 3 2
Hence, mathematical formulation of the 5
given LPP is Minimize Z = 40x – 20y, cos
3
subject to the constraints:
x + y 15, 2
sin 1 cos 2
x + y 30, 3
x 15, y 20,
www.jeeneetbooks.in

WWW.IIT-NEET.XYZ

Solutions MT-181

Hence, the variable plane (1) always passes


1 1 5
So, tan cos tan
2 3 2 1 1 1
through the fixed point , , .

1 cos 3 5 113. (b) Let n denote the required number of shots


sin 2 and X the number of shots that hit the
110. (c) Given 3f (cos x) + 2f (sin x) = 5x .....(i) target. Then X ~ B(n, p), with p = 1/4. Now,
P(X 1) 0.9 1 P( X 0) 0.9
Replace x by x we get
2 3
n
3
n
1
n
1 C0 0.9
4 4 10
3f cos x 2f sin x
2 2 n
4
10 n(log 4 log 3) 1
3
5 x
2 1
n(0.602 0.477) 1 8 n
0.125
or 3f (sin x ) 2f (cos x ) 5 x ...(ii) Therefore the least number of trials required
2 is 8.
Solving (i) and (ii) simultaneously, we get 114. (b) Given equation of line is
f(cos x) = 5x – x 2 y 1 z 3
Differentiating we get (let)
3 2 2
( sin x)f '(cos x) 5 x = 3 – 2y = 2 – 1, z = 2 + 3 ...(i)
Coordinates of any point on the line
5
f '(cos x) are (3 – 2, 2 – 1, 2 + 3)
sin x The distance between this point and
111. (d) The relation S is defined by,
6
S {( x, y) : x 2 y2 1} A B (1, 2, 3) is
2
Clearly to each x A , there exists two
2 2 2 6
2 3 2 1 2 1 2 2 3 3
values of y given by y 1 x 2
S cannot represent a function.
112. (b) Let equation of the variable plane be 36
(3 – 3)2 + (2 – 3)2 + (2 )2 =
2
x y z
1 ...(1) Squaring on both sides
a b c
9 2 + 9 – 18 + 4 2 + 9 – 12 + 4 2 = 18
The intercepts on the coordinate axes are
a, b, c. The sum of reciprocals of intercepts (17 – 30) = 0
is constant , therefore 30
0,
1 1 1 17
a b c Substituting the values of in eq. (i) we
get the required point (–2, –1, 3) and
(1 / ) (1 / ) (1 / )
or 1 56 43 111
a b c , ,
17 17 17
1 1 1
, , lies on the plane (1)
EBD_7443
www.jeeneetbooks.in

WWW.IIT-NEET.XYZ

MT-182 Target VITEEE

e m
m2 118. (d) 2iˆ pjˆ 5kˆ and 3iˆ pjˆ pkˆ are
115. (c) We are given that e–m . m = 0.3 and
2! perpendicular
2 × 3 + p (–p) + 5 (p) = 0
= 0.2
p = –1 or p = 6
4 Hence for p = 6, the lines are perpendicular.
Dividing, m 0.4 0.3
3
sin( cos 2 x)
P (X = 0) = e–m = e–4/3 119. (b) lim
x 0 x2
116. (a) The given equation can be written as
sin{ sin 2 x}
1 cos x sin 2 ax 0 lim
x 0 x2
x
or sin 2 sin 2 ax 0 sin( sin 2 x)
2 lim
x 0 x2
x
sin 0 and sin ax 0
2 sin( sin 2 x) sin 2 x
lim
x 0 sin 2 x x2
x
n and
2 sin 2 x
lim
ax m where m, n are integers x2
x 0

m 120. (a) The relation R1 is an equivalence relation


x 2n and x a R, | a | | a |, i.e. aR1a a R
a

m R1 is reflexive.
2n m 2an ...(1)
a; Again a, b R, | a | | b | |b| |a|
Now m and n are integers but a is irrational aR1b bR 1a . Therefore R is symmetric.
hence the result (1) will hold good if m = 0
and n = 0 and in that case x = 0 is the only Also, a, b,c R, | a | | b | and | b | | c |
solution. |a| |c|
dy aR1b and bR 1c aR1c
117. (c) We have, .y x (Given)
dx R1 is transitive
ydy xdx R2 and R3 are not symmetric.
R4 is neither reflexive nor symmetric.
y2 x2 C
Integrating. i.e y 2 x 2 C 121 (d)
2 2 2
122 (b)
Since the curve passes through the point
(5, 3), 123 (c)
9 = 25 + C C = – 16 124 (c)
So the curve is y2 = x2 – 16 i.e x2 – y2 = 16 125 (b)
which is clearly a rectangular hyperbola.
www.jeeneetbooks.in

WWW.IIT-NEET.XYZ

Solutions MT-183

MOCK TEST 6

1. (d) For collision V B/A should be along V2 = 3V1 ...(ii)


From (i) and (ii),
B A rA/B V1 = 3.5 V and V2 = 10.5 V
Thus charge of C3,
V 2 V1 r1 r2 q3= C3V2 = 4 × 10.5 = 42 V.
So, V2 V1 r1 r2 5. (a) r2 q
V1 V2 r1

q1
A B
A B
2. (a) Force = eE Let q1 be the charge on the inner shell. On
Workdone = force × distance being earth its potential will be zero.
Force and distance are in opposite 1 q1 q2
direction, so work is negative. or V=0 =
W=–eE×d 4 0 r1 r2
Here, distance increases so, potential
r1
energy increases. q' = q
q –q r2
– + – + 6. (b) Here the electric field at A and B is same but
3. (b) – + – + at C it will decreases,
– + – + s o VA = VB ( V = – Edr)
and VC is less than VA or VB
d VA = VB > VC
Because of induction, the effective force 7. (c) In a round trip, displacement is zero. Hence,
between them becomes equal to greater work done is zero.
than
+q
1 q2
F= . r
4 0 d2
4. (c) +Q
C1 = 4µF

14V 8. (b) The magnitude of force on A due to B and


4µF on B due to A is same but both will act in
2µF opposite directions as the electrostatic
force between them is repulsive.
+2 C +6 C
The capacitance of parallel capacitors
2 4 4
F AB A d B FBA
= µF
2 4 3 FAB = FBA
If V1 and V2 are the p.d. s across C1 and
parallel capacitors, then N d
V1 + V2 = 14 ...(i) 9. (b) Electron density, n
M
4 where d = density, N = Avogadro number,
and 4 V1 = V2
3 M = atomic weight
EBD_7443
www.jeeneetbooks.in

WWW.IIT-NEET.XYZ

MT-184 Target VITEEE


When a cell of emf 3V is connected across
2.7 103 6.02 10 26 A & B,
so, n (atom/kg)
27 3
= 6.02 × 10 electrons/m3
28 C D
I A B
Drift velocity, vd 3
neA
5 3V
or vd 28 19 6 The current through the circuit is given by
6.02 10 1.6 10 4 10
= 1.29 × 10–4 m/s 3V
i 2A
10. (c) No current flows through 6 resistor as a 1.5
capacitor offers infinite resistance to a D.C. This current is equally distributed along
The equivalent circuit is thus as shown arms AB and CD. Hence, current along AB
below in which 2 and 3 are in parallel is 1A.
2 12. (d) One end of all the three resistors are
connected at the point X and the other at
A B the point Y. Therefore, the equivalent circuit
3 is given below :
6

6
6V 2.8
1 1 1 6 X Y
or R ' 1.2
2 3 R' 5
Current drawn from the battery 6
The equivalent resistance is given by
E 6
I 1.5 Ampere 1 1 1 1 3 1
R 4
Potential difference between A and B R 6 6 6 6 2
V IR ' 1.5 1.2 1.8V Req 2
Current through 2 resistor 13. (b) According to Fleming's Right hand Rule
V 1.8 In (1) force is in – z axis
0.9 A In (2) force is in – x-axis
2 2
11. (d) Rearranging the resistors CDEF, we get a In (3) force is zero
balanced wheatstone bridge as shown in because
the figure. F = q (v B) = qvB sin 180
D i.e. F = 0
3 3 14. (d) Given: n = 100 turns/m
I = 5A
C 3 E
0 nI
3 B= (sin 1 sin 2 )
3 3 2
A B
F 0 100 5
3 B= (sin 45 sin 45 )
A B 2
3
effective resistance between C and E 0 500 1 1
=
6 6 2 2 2
=3
6 6 500
0 2
Now, effective resistance between A and B = = 0 250 2
2 2
3 3
1.5
3 3 = 250 2 0
www.jeeneetbooks.in

WWW.IIT-NEET.XYZ

Solutions MT-185

15. (b) I = I2R (IC IL )2 æ 4.24 ö÷2 4.24


Id = çç ÷ = = 2.99A 3A
çè 2 ø÷ 2
E 200
IR 4A
R 50 20. (b) Given V = V0 sin t; i = i0sin( t – )
Here current lays the potential. This can be
E 200
IC 8A 1
XC 25 possible when XL > XC or L > .
C
E 200 21. (c) Tension at the highest point
IL 5A
XL 40
mv2
Ttop – mg 2mg ( vtop = 3gr )
r
I (4)2 (8 5)2
Tension at the lowest point
I 16 9 25 Tbottom = 2mg + 6mg = 8mg
I = 5A
16. (a) i = 2 sin t Ttop 2mg 1
.
V = 5 cos t Tbottom 8mg 4
I (max) = 2 amp, V = 5V
P = VI = 5 × 2 = 10 watt. 22. (c) For a transformer,
17 (d) Current is max, when at resonance condition Vp Ip = Vs Is

1 1 Vp I p4000 100
So, = Is A = 1.67 A
LC 0.5 8 10 6 Vs 240000
23. (d) In a single slit diffraction pattern, the path
1
= difference ( x) for nth minima is given by
6
4 10 x = n ,, where n 1, 2, 3,...
1 1000 or phase difference, n(2 )
=
2 For 1st minimum, n = 1.
= 500 Hz
18. (a) XL = 15 ; XC = 11 ; R = 3 2
24. (d) Given Ez= 60sin (kx + t) V/m. The magnetic
Impedance, Z = R2 (X L X C )2 field component must be perpendicular to
Ez and x.
= 9 16 5
E0 60
E rms 10 Also B0 = . Thus
Irms = = 2A c c
Z 5
VL = Irms × XL = 2 × 15 = 30 V 60
By = sin (kx + t).
VC = Irms × XC = 2 × 11 = 22 V c
VL – VC = 8V 26. (c) Given, 1H2 + 1H2 4
2He + Q
19. (a) Peak current, I0 = 4.24 A The total binding energy of the deutrons
= 4 × 1.15 = 4.60 MeV
I0 4.24
I rms The total binding energy of alpha particle
2 2 = 4 × 7.1 = 28.4 MeV
Thermal energy produced, The energy released in the process
2 2 = 28.4 – 4.60 = 23.8 MeV.
H = Id RT I rms RT
26. (b) Number of fissions per second
2 Total energy produced / sec.
Id I rms
=
Direct current, Energy released per fission
EBD_7443
www.jeeneetbooks.in

WWW.IIT-NEET.XYZ

MT-186 Target VITEEE


31. (b) Given:
1.6 106 J / s 1019 E = 57 meV = 57× 10–3 × 1.6 × 10–19 J
= 13
200 1.6 10 J 200 hc
E=
1000
= 1016
34
200 hc 6.6 10 3 108
= 5 ×1016 fissions per second. = 22
E 57 1.6 10
27. (a) We have 1 mv 2 h h 0 6.6 3 10 26
2 =
1 57 1.6 10 22
mv12 h(5 0 0) ....(1) = 0.2171052 × 10–4 m
2 = 217105.2 × 10–10 m
1 = 217105 Å (Approx.)
mv2 2 h(9 0 0) ....(2) 32. (b) When m1 > m2 & m2 at rest then the bodies
2
(2)/(1) collide in elastically and move together as
one body without changing the direction.
v22 8 0 33. (b) Extrinsic semiconductor is due to the type
2 of impurity atom's valency. If impurity atoms
v12 4 0
are trivalent, then carriers are hole. And if
impurity atoms are pentavalent then carriers
v2 = 2v1 2 4 105
are electron.
= 5.64 × 105m/sec 34. (a) = 0.95
h ie = 100 mA
28. (a) = ic = ?
2mK
34 ic
6.63 10 = i
2 9.1 10 31 5.5 1.6 10 16 e
= 169 nm ic
0.95 =
100
1 1 1
29. (a) We have, = R ic 0.95 100mA
n12 n 22
i c 95mA
1 1 1 35. (d) In forward biasing, the value of diffusion
=R current decreases.
22 32
36. (a) = 0.95; RL = 5k
1 1 1 Ri = 70
=R
4 9 RL 5 103
AV = 0.98 70
1 9 4 Ri 70
=R RL
36 Power gain = 2
. 0.98 70 68.6
Ri
1 5
=R
36 m2
37. (b) Pc Pt 1
36 2
5R
30. (d) For shortest wavelength m2
11.8 10 1
n1 = 2, n2 = 2
1 1 1 m 0.6 % modulation = 60%.
R
2 38. (c) The given graph does not obey Hooke's
1 law and there is no well defined plastic
R(0.5) , shows visible region. region. So the graph represents elastomers.
www.jeeneetbooks.in

WWW.IIT-NEET.XYZ

Solutions MT-187

39. (c) In Bernoulli’s theorem only law of 48. (a) For a face-centred cube, we have.
conservation of energy is obeyed. 2a 2 0.361
40. (d) H = ncp t radius = = nm = 0.128
4 4
41. (d) In a tetragonal distortion, the ligands along 49. (a) For a reaction to be spontaneous G should
the z-axis move away. Their repulsion be negative.
significantly decreases for the orbitals We know G = H – T S
involving z-direction. These are lowered in Thus, for a reaction to be spontaneous at
energy. all temperatures G and H should be
42. (c) Cu + ion in aqueous solution negative.
disproportionates to Cu(s) and Cu 2(aq) . 50. (a) C(s) O 2 (g) CO 2 (g)
43. (b) In [Fe(CN)6]3– Fe is in +3 state. Thus entropy of formation of CO2
Fe (Z = 26) in Fe3+ we have 23 electrons. SCO2 (SC SO2 )
Thus, EAN of Fe = 26 – 3 + 12 = 35 = [213.5 – (5.740 + 205)] JK–1
[12 electrons gained from six ligands] = 2.76 JK–1.
44. (d) Magnetic moment 51. (a) Internal energy is dependent upon
temperature and according to first law of
n ( n 2) 3.87 B.M n 3
thermodynamics total energy of an isolated
(unpaired electrons). The comlex may be system remains same, i.e., in a system of
regarded as a high spin d7 complex of Fe+ constant mass, energy can neither be
and No+ created nor destroyed by any physical or
45. (c) EDTA forms a more stable coordination chemical change but can be transformed
complex with lead than with calcium or from one form to another
sodium and in this form passes out from E q w
the body without harmful effects. The For closed insulated container, q = 0, so,
calcium salt is used so that any excess of E w , as work is done by the system
EDTA will not remove Ca 2 ions from the 52. (a) Le-Chatelier’s principle is applicable only
body. to a system in equilibrium.
53. (d) We can obtain for NaOH at infinite dilution
46. (d) The optical isomers are pair of molecules
by extrapolating the line of NaOH but we
which are non super imposable mirror cannot obtain for CH3COOH because on
images of each other extrapolating the curve of weak electrolyte
3+ N en 3+ it does not approach a limiting value. But
en – N molar concluctivity at infinite dilution for a
N N
N N Co weak electrolyte can be calculated by using
en Kohlrausch's law. According to this law
Co en
N N
N N Ù°m for AxBy = x y
N en A B
en The relation of molar conductivity and
N 1000K
d form l form specific conductivity i.e., Ùm = ,
Enantiomers C
The two optically active isomers are is not useful because when C 0,
collectivity called enantiomers. Ùm
47. (d) When electrons are trapped in anion 54. (b) The order w.r.t. I2 is zero because the rate is
vacancies, these are called F-centres. not depends on the concentration of I2.
55. (a) Ag e Ag
+ve –ve 1F 108 g
ion ion 1 F = 1 mole of electrons = 96500 C
0.01F = 1.08 g Ag; Ag left = 1.08 – 1.08 = 0
F- centre in crystal
EBD_7443
www.jeeneetbooks.in

WWW.IIT-NEET.XYZ

MT-188 Target VITEEE


61. (c) Option (c) is the answer. Dehydration will
56. (b) G reaction Products Reactants be most easier when –OH group is trans to
C = O group.
G (CO2 ) G (H 2O) 62. (a) Prestone is a mixtrue of glycol & H2O. It
has freezing point much below 0°C, hence
3 presence of glycol (CH2OHCH2OH) in H2O
G (CH3OH) G (O2 ) reduce its freezing point. Therefore, it is
2
used as an antifreeze for automobiles
= [– 394.4 + (– 237.1)] – [(–166.3) + (0)]
radiators.
= – 702.3 kJ mol–1
63. (c) C6H5CH2Br + NaOC2H5 or
G 702.3 C6H5CH2ONa + C2H5Br
% Efficiency = 100 = 100 C6H5CH2OC2H5
n 726
64. (b) Acetophenone (C6 H5 COCH3 ) being a
= 96.7%
ketone, does not reduce Tollen’s reagent.
Specific conductance 65. (c) Aldehydes restore the pink colour of
57. (b) Cell constant =
Conductance Schiff's reagent (Schiff's reagent is a dilute
solution of rosaniline hydrochloride in
0.0112 water whose red colour has been
= or 0.0112 × 55
1/55 discharged by passing sulphur dioxide).
= 0.616 cm–1. Ketones do not restore Schiff's reagent
58. (b) Dry ether colour.
CH 3 I Mg CH 3 MgI
66. (a) P2O5 is a dehydrating agent. When it is

C6H5 heated with CH 3COOH, it eliminates a


CH3MgI + C O water molecule from two CH 3COOH and
H
thus, acetic anhydride is formed.
C6H5
CH O MgI CH 3CO OH CH 3CO
CH3 P2O 5
+ O + H2O
Heat
C6H5 CH 3COOH CH3CO
H 2O
CHOH 2 mole of acetic acid Acetic anhydride
CH3 Mg ( OH ) I 67. (c) Mesitylene is formed.
(1-Phenyl ethanol)
CH3
O
59. (d) With Br 2 water, ph enol gives 2, 4, ||
H 2SO 4
6- tribromophenol. 3CH3 C CH3
3H 2O
OH CH3 CH3

Trimethyl benzene or Mesitylene


+ 3Br2 (excess) 68. (d) It is Favorskii rearrangement – ring
contraction.
The transformation of -haloketones to
OH
Br Br esters with rearranged carbon skeleton by
H2O the treatment with alkoxide ions is called
+3HBr
Favorskii rearrangement
Br
2, 4, 6 Tribromophenol
RO
60. (b) Ether exists as peroxide in air.
www.jeeneetbooks.in

WWW.IIT-NEET.XYZ

Solutions MT-189

NaBH 4 CH2OOC.COOH
69. (a) O OH |
CH 3OH
CHOH CO2
|
HBr (i) Mg. Et 2O CH2OH
Br
(ii ) H 2C O
CH2OOCH CH 2OH
| |
CH 2 OH CHOH
H 2O
CHOH HCOOH
| | Formic acid
PCC
CH2OH CH2OH
CHO 76. (a) 3
In amines, N is sp hybridised and thus has
CH 2Cl 2
pyramidal shape. In the given structure,
70. (b) Acetaldehyde is the only compound among since the three alkyl groups are different,
the given compounds which can reduce and the fourth corner of the pyramid is
Tollen’s reagent to silver mirror. occupied by lone pair of electrons, the
molecule is chiral. However, the two
CH3CHO + 2[Ag(NH3)2]+ + 3OH– enantiomers of the amine are not resolvable
Acetaldehyde Tollen's reagent because of their rapid interconversion
CH3COO– + 2H2O + 2Ag + 4NH3 through a transition state having planar
structure (sp2 hybridised nitrogen)
Silver
CH3I
mirror 77. (c) CH3 NH 2 (CH3 )4 N I
excess (A)
Br2
71. (c) (i) CH3CH2COOH PCl3
CH3CH2COCl AgOH heat
(CH3 )4 N OH
Br2 (B)
CH3CH(Br) COCl
H2 O
CH3CH(Br) COOH. (CH3 )N CH 3OH
(C) and (D)
Cl2
(ii) CH3CH2COOH 78. (a) Consider the given sequence
PBr3
Reduction HNO 2
Cl2 [A] [B] CH3 CH 2 OH
CH3CH2COBr CH2CHClCOBr Since, [B] on reaction with HNO2 form a
H2O primary alcohol (CH3CH2OH) so [B] is a
CH3CH(Cl)COOH.
72. (a) This is an example of Baeyer-Villiger primary aliphatic amine. i.e., [B] is
oxidation CH3CH2NH2.
73. (c) 1, 6- and 1, 7-dicarboxylic acids on heating Since, [A] on reduction forms CH3CH2NH2
form cyclic ketones (Blanc rule) so [A] is CH3CN.
Reduction
CH3 CH 3 CN CH 3CH 2 NH 2
|
74. (d) CH 3 C CH 2
CH3COCl 79. (b) When alcoholic KOH reacts with chloroform
ZnCl2 or AlCl 3 and aniline the following reaction take
place:
CH3 NH2
|
CH3 C CH 2COCH3
| + CHCl3 + 3KOH
Cl
75. (c) When glycerol is heated with oxalic acid
following reaction occurs. N=C
CH2OH HOOC
| | + 3KCl + 3H2O
100 110 C
CHOH HOOC
| oxalic H 2O
CH 2OH acid Phenyl isocyanide
EBD_7443
www.jeeneetbooks.in

WWW.IIT-NEET.XYZ

MT-190 Target VITEEE


This is also called Carbylarmine reaction. It 84. (a) Given that
is used for detection of 1° amines. The f (x) = 2x + sin x , x R
isocyanide formed has a very offensive
smell which is used to detect its presence. f ' (x) = 2 + cos x
80. (c) Diazonium cation reacts with aniline in But – 1 cos x 1
weakly acidic medium resulting in N, N- 1 2 + cos x 3
coupling rather than C-coupling.
1 2 + cos x 3
f ' (x) > 0, x R
N N HN f (x) is strictly increasing and hence one-
one
H Also as x , f (x) and x –
–H f (x) –
N N NH

81. (b) The given relation is R = {(1, 1), (2, 2),(3, 3), Range of f (x) = R = domain of f (x) f (x)
(1, 3)} on the set A = {1, 2, 3}. is onto.
Clearly, R is reflexive and transitive. Thus, f (x) is one-one and onto.
To make R symmetric, we need (3, 1) as (1, 85. (b) After sending 4 to one side and 3 to other
3) R. side. We have to select 5 for one side and
If (3, 1) R, then R will be an equivalence 6 for other side from remaining.
relation. Hence, (3, 1) is the single ordered This can be done in 11C5 × 6C6 ways
pair which needs to be added to R to make
= 11C5
it the smallest equivalence relation.
82. (d) PQ = I P–1 = Q Now, there are 9 on each side of the long
Now the system in matrix notation is PX = B table and each can be arranged in 9! ways.
X = P–1 B = QB Required number of ways
x 2 2 1 1 = 11C5 × 9! × 9!
1 = 11C6 × (9!)2 [ nCr = nCn – r]
y 13 5 m 1
9 86. (d) The graph of the inequalities 2x + y 10,
z 8 1 5 5 x 0, y 0 is the region bounded by AOB.
1 This region has no point common with the
y (13 5 5m) region {(x, y) : x 6, y 2} as is clear from
9
– 27 = 8 + 5m (Given y = – 3) the figure . Hence, the region of the given
m=–7 inequalities is the empty set.
83. (a) Since 1, 2 and 1, 2 are the roots of ax2
+ bx + c = 0 and px2 + qx + r = 0 respectively, Y
therefore
b c
1+ 2= a , 1 2= a ...(1)
B(0, 10)
q r
and 1 + 2 = p , 1 2 = p ...(2)
9=x

Since the given system of equation has a


2x +

non-trivial solution
y=

1 2
10

= 0 i.e. 2– 2 1=0
1 2
y=2
or 1 2 1 2 1 2
A
1 2 1 2 1 2 X
2 (5, 0) (6, 0)
pb pc b ac
qa ra q2 pr
www.jeeneetbooks.in

WWW.IIT-NEET.XYZ

Solutions MT-191

87. (b) The general term of the series


[ | BA | BA AB]
2 2 2
.......
1.2.3 3.4.5 5.6.7 Now BC 2 î 6ˆj 3k̂

tn =
2
, n = 1, 2, 3,...... and BA î 3ˆj
(2n 1) 2n (2n 1)
ˆi ˆj kˆ
1 2 1
= BC BA 2 6 3 9iˆ 3jˆ 12kˆ
2n 1 2 n 2n 1
1 3 0
1 1 1 1
=
2n 1 2n 2n 2n 1 | BC BA | 92 32 (12) 2 3 26
2 2 2 and | BC | 4 36 9 7
1+ ........
1.2.3 3.4.5 5.6.7
3 26
1 1 1 1 AD
=1 .... 7
1 2 3 4
90. (b)
1 1 1 1 5
....
2 3 4 5 3
= 1 + loge2 + loge2 – 1= 2 loge2
88. (b) Let equation of the variable plane be 4
x y z 5 2
a b c
1 ...(1) Let cosec 1 , tan 1
3 3
The intercepts on the coordinate axes are
a, b, c. The sum of reciprocals of intercepts 5 2
cosec , tan
is constant , therefore 3 3
1 1 1 4 3
cot , cot
a b c 3 2
(1 / ) (1 / ) (1 / ) cot cot 1
or 1 Now, cot
a b c cot cot
1 1 1
, , lies on the plane (1) 13
Hence, the variable plane (1) always passes 2

1 1 1
through the fixed point , , . 3
4 3
A (1, 4, –2) 1
3 2 6 2 1 6
= =
4 3 9 8 17
3 2
89. (c) 91. (d) As given: cos–1x + cos–1y + cos–1 z = 3
and we know that 0 cos –1 x
B (2, 1, –2) D C (0, –5, 1)
cos 1 x , cos 1 y , cos 1 z
AD =AB sin x = y = z = cos = –1.
| BC BA | | BC BA | xy + yz + zx = (–1) (–1) + (–1) (–1) + (–1) (–1)
AB. = 1+ 1 + 1 = 3
| BC | . | BA | | BC |
EBD_7443
www.jeeneetbooks.in

WWW.IIT-NEET.XYZ

MT-192 Target VITEEE

x y f ( x ) f ( y) 94. (b) Let the the variable circle be | z z 0 | r .


92. (c) We have f 3 3
,

f(0) =0 and f (0) = 3 z0 r a z1


f (x h) f (x)
f (x ) lim r
h 0 h
3x 3h
f f (x) z2 b
3
lim
h 0 h Then | z 0 z1 | a r and
f (3x ) f (3h ) f (3x ) f (0)
| z0 z2 | b r
lim 3 3
h 0 h Eliminating r, we get,
f (3h ) f (0) | z 0 z1 | | z 0 z 2 | a b
lim 3 Hence locus of z0 is a hyperbola
h 0 3h
f(x) = 3x + c, f(0) = 0 c=0 95. (a) Let AB a , AD b and AC c
f(x) = 3x
3 when a , b and c are non-collinear
93. (b) x + iy = coplanar vectors.
cos i sin 2
DB AB AC a b
1 cos i sin 2 Now, D B. A B (a b ).( a ) a .a b .a
x iy 3
c2 a2 b2
a2 ab cos a2
x iy 2
(x iy) (x iy)
3a 2 b2 c2
1 2
[(cos 2) i sin ]
3 a 2 b2 c 2
In ABC , cos( )
x 1 2ab
(cos 2)
2 2 3 D
x y C

x 2 1
cos
2 2 3 3
x y b
c

y 1 –
and sin
2 2 3
x y A
a
B

Squaring and adding, we get 51


96. (b) {x cos(cot–1x) + sin(cot–1x)}2 =
2 2 50
x 2 y 1
2
x 2
y 2 3 x 2
y 2 9 1 1 51
x cos tan 1 sin tan 1
x x 50
1
(3 4 x ) 1 0 2
2
x y2 1
1 x
xcos cos 1 sin sin 1
3 4x x2 y2 1
1
1
1
x2 x2
4x x 2 y2 3
www.jeeneetbooks.in

WWW.IIT-NEET.XYZ

Solutions MT-193

51 Note that ac is either b or e.


= If ac = c, then a = e, which is not so.
50
So, ac = b.
2 Similarly bc = a and ca = b.
x2 1 51
Next, complete the table keeping in mind
1 x 2
1 x 2 50 that each element in a row (or a column)
must be distinct. Then the composition table
2 shows that G is abelian.
x2 1 51
x 2
1 50 n n (n 1)
99. (a) We have Sn n 1
1 n 1 1
51 1 1
x2 1 x n 1
50 5 2
97. (c) If cot x 0, then |cot x| = cot x, the equation S2n (n 1) 2

1 So, S12 S22 ... S2n


2
1
becomes 0 , which is not posible
sin x
22 32 ... (2n)2
Now, if cot x < 0, then | cot x| = – cot x, the
equation becomes 1
1 (2n )(2n 1)(4n 1) 1
cot x cot x 6
sin x
cos x cos x 1 1
[n (2n 1)(4 n 1) 3]
sin x sin x sin x 3
2 cos x 1
0
1 100. (b) Let S x2 – 4y
cos x
sin x 2 Since the point (2a, a) lies inside the
parabola,
2 S (2a, a) = 4a2 – 4a < 0
x 2n ,n I and 0 x 2 .
3 or a (a – 1) < 0 ...(1)
Also, the vertex A (0, 0) and the point (2a, a)
2 4
Hen ce x , i.e., number of are on the same side of the line y = 1 (the
3 3 equation of latus rectum)
solution is 2. So, a – 1 < 0 i.e. a < 1 ...(2)
98. (b) A group containing exactly four elements
is always abelian. Y
Let G = {e, a, b, c} be a finite group of order
4 with identity element e, then e–1 = e.
S (0, 1)

Case I : If a–1 = a, b–1 = b, c–1 = c, then each


element is its own inverse and hence G is y=1
abelian. (2a, a)
Case II : If a–1 a, say a–1 = b then b–1 = a
and hence c–1 = c. So, in this case ab = e,
ba = e. X
A
The composition table becomes :

e a b c From (1) and (2), we have a (a – 1) < 0


or 0 < a < 1
e e a b c
101. (a) Let the equation of the ellipse be
a a c c b
b b e c a x2 y2
1.
c c b a e a2 b2
EBD_7443
www.jeeneetbooks.in

WWW.IIT-NEET.XYZ

MT-194 Target VITEEE


y
= (a b ). (b c) (c a)
B L

=( a b ). (m a ) c (m c ) a )
S x
A C A
(where m b c )
B L
{( a b ). c }.{( a ( b c )}
Let the normal at the extremity L of the latus [ a b c ]2 42 16
rectum passes through the extremity B of
the normal axis f ( x 2 ) f (x )
104. (c) lim
b2 x 0 f (x ) f (0)
Coordinates of L are ae, and
a Using L.H. Rule
coordinates of B are (0, –b). f (x 2 ).2 x f (x )
lim
Equation of the normal at L is x 0 f ( x)
ax 2 2
ay a b 2 xf ( x 2 )
e lim 1 0 1 1
(see previous illustration) x 0 f ( x)
If it passes through B’ (0, – b), then
105. (a) y 1 4x x 2 5 ( x 2) 2
0 + ab = a2 – b2
a 2b 2 (a 2 b2 ) 2 Y
y = mx
But b 2 a 2 (1 e 2 ).
y = 1+ 4x – x2
a 2 a 2 (1 e 2 ) [a 2 a 2 (1 e 2 )]2
1 e 2 e 4 or e 4 e 2 1 0
102. (b) Let the asymptotes be X
x + 2y + k1 = 0 …(1) 3 2
and 2x + y + k2 = 0 …(2) 2
Since (1) and (2) pass through the centre of 3/ 2 3/ 2
the hyperbola i.e (1, –1), (1 4x x )dx 2
2 mx dx
We have
k1 = 1 and k2 = –1
So, the asymptotes are x + 2y + 1 = 0 0 0
and 2x + y – 1 = 0 3 9 1 27 9
Since the equation of a hyperbola and its 2 m
2 4 3 8 4
asymptotes differ in constant terms only,
therefore equation of hyperbola is 13
On solving, we get m
( x 2 y 1)(2x y 1) 0 6
Since it passes through the point (2, 1), 106. (c) Let = + . Then, tan = K tan
(2 2 1)(4 1 1) 0 20 tan K
or
Thus the equation of hyperbola is tan 1
( x 2 y 1)(2x y 1) 20 0 Applying componendo and dividendo,
we have
i.e 2x 2 2 y 2 5xy x y 21 0
tan tan K 1
103. (a) We have, [ a b b c c a] tan – tan K –1
www.jeeneetbooks.in

WWW.IIT-NEET.XYZ

Solutions MT-195

or
sin cos cos sin K 1 d 2s as 2 ( at b) 2
sin cos – cos sin K –1 dt 2 s3
sin K 1 d 2s
i.e.,
sin – K –1
a s 3.
dt 2
Given that, – = and + = .
2 2
Therefore, 2
109 (c) I = [x 2 ] dx – [x ] dx
sin K 1 K 1 0 0
sin K –1 or sin = sin .
K –1 2 2 2 2
107. (a) We have = 1 dx + 2 dx + 3 dx – 1 dx
1 1 3 1
y cos x, y1 sin x cos x
2 =4– 2 – 3
2 1 cos a ( x )( x )
y2 sin x cos x 110. (a) Given limit = lim 2
2 2 x (x )
n (x )( x )
then yn cos x 2sin 2 a
2 = lim 2
Now put x = 0; then x (x )2
y0 1, (y1 )0 0, (y 2 )0 1 =
3 (x ) (x )
(y3 )0 cos 0 sin 2 a
2 2 2
lim
n x (x )2 a2 ( x )2 ( x )2
In general (y n )0 cos 4
2
Now, put n = 4; then (y4)0 = cos 2 1 a2 ( x )2 ( x )2
n 4
Hence (y n )0 cos = 0, if n is odd
2
a2 ( )2
= ( 1) n/2 if n is even. = .
2
x2 111. (c) Given f (x) = (x + 1) cot x is continuous at x
Then f (x) f (0) xf '(0) f ''(0) ... =0
2!
lim f (x) f (0)
x 0
x2 x4 x6 xn
cos x 1 ... ( 1) n / 2 ... x cot x
2! 4! 6! n! 1
when n is even. cot x
Now, lim (1 x) lim (1 x) x
108. (a) s2 = at2 + 2bt + c x 0 x 0

ds ds at b 1
2s
2 at 2 b or , x cot x
= lim e
x
dt dt s lim 1 x e
x 0
again differentiating x 0
x
d 2s a.s (at b) ds lim
x 0 tan x lim
x
1
. = e = e
dt 2
s2 dt x 0 tan x
f (0) = e
at b
as (at b) 112. (c) x 3e 2x dx
s
2 dv = e2x dx
s
u = x3 v = 1/2 e2x
EBD_7443
www.jeeneetbooks.in

WWW.IIT-NEET.XYZ

MT-196 Target VITEEE


u' = 3x2 v1 = 1/4 e2x 116. (a)
u'' = 6x v2 = 1/8 e2x
u''' = 6 v3 = 1/16 e2x p q p q p q ~ (p q) (p q) ~ (p q)
Using Bernoulli's formula T T T T F F
T F F T F F
u dv uv u ' v1 u "v 2 ....
F T F T F F
We get F F F F T F
1 2x 1 2x
x 3 e 2x dx (x 3 ) e (3x 2 ) e (p q) (~ (p q)) is a contradiction.
2 4
117. (c) P = n (n + 1) (n – 1) (n + 2) (n – 2).........
1 2x 1 2x (n + r) (n – r)
(6x) e (6) e
8 16 = {n (n + 1) (n 2).......(n + r)} {(n – 1)
(n – 2).......(n – r)}
1 2x 3 3 2 3x 3 = (n + r) (n + r – 1)......(n + 1) (n) (n – 1)......
e x x
2 2 2 4 (n – r)
Clearly P is product of (2r + 1) consecutive
dy
113. (a) We are given that cos (x y) integers, so divisible by (2r + 1) !
dx
Put x + y = v, 118. (d) Let E : ‘face 1 comes up’ and F: ‘face 1 or 2
dy dv dy dy comes up’
so that 1+ 1
dx dx dx dx E F E E F
So, the given equation becomes P(E) = 0.10 and P(F) = P(1) + P(2)
dv dv = 0.10 + 0.32 = 0.42
1 cos v 1 cos v Hence, required probability
dx dx
1 2v P E F P E 0.10 5
1 sec dv dx = P E/F
dv dx P F P F 0.42 21
1 cos v 2 2
Integrating both sides, we get 3
119. (c) Let g(x) = x ±2
1 v
sec 2 dv 1.dx 4 – x2
2 2 D (g(x)) = R – {–2, 2}
h (x) = log10 (x3 – x)
v x y x3 – x > 0
tan x C tan x C
2 2 x (x + 1) (x – 1) > 0
which is the required solution. – + – +
114. (b) Rewriting the given equation in the form –1 0 1
4 dy 3 x x (–1, 0) (1, )
x cos y 4 x sin y xe Domain of f(x) is (–1, 0) (1, 2) (2, ).
dx
120. (a) P (E / F) + P ( E / F)
d 4
(x sin y ) xe x
dx P(E F) P(E F)
=
P(F) P(F)
x 4 sin y xe x dx c ( x 1) e x c
Since, y (1) 0 so, c = 0. P( E F) P(E F) P(F)
1
P(F) P(F)
4
Hence, sin y x ( x 1) e x 121 (b)
115. (a) 1
2 * x *3 5 1 122 (d)
123. (c)
x 2*(5*3) 2*(5 3 15) 124. (a)
2*(23) 2 23 46 71 125 (a)
www.jeeneetbooks.in

WWW.IIT-NEET.XYZ

Solutions MT-197

MOCK TEST 7

1. (c) Change in momentum, 2vq


Thus, vA =
p = Fdt m
2v 4q
= Area of F-t graph and, vB =
= ar of – ar of + ar of m
vA 1 1
1
= 2 6 3 2 4 3 Hence, v 4 2
2 B
= 12 N-s or, vA : vB = 1 : 2.
2. (d) When air is filled in between the plates, then 7. (d) Net electric flux
capacitance is given by = (4 × 103 – 8 × 103) = – 4 × 103 Nm2c–1
A q
C= 0 =
d 0
When interspace between the plates is filled q = 0 = – 4 × 103 0 coulomb.
with wax, then capacitance is 8. (a) From the graph l = 10–4m, F = 20 N
K 0A A = 10–6m2, L = 1m
C =
2d FL 20 1
Y
0
A K Al 10 6
10 4
or C = d 2
20 1010 2 1011 N/m 2
K 9. (c) The internal resistance of the cell,
or C =C
2
1 2 240 120
r= R 2=2
K 120
6=2 K=6 2
2
10. (d) B=2 A
3. (a) In case of a spherical metal conductor, the dB = 2dA
charge spreads uniformly over the entire
surface. Therefore, the sphere will retain the RB = RA B B A A
charge for longer time. AB AA
4. (b) When terminal velocity is reached then
body moves with constant velocity hence, B A d B2 A 4d A2
2
accelesation is zero. d A2 2 A d A2
A B
5. (d) The maximum torque experienced by a
dipole of dipole moment p placed in an 11. (a) From Kirchhoff’s junction law at junction P,
uniform external electric field E is given by I1 + I 2 = 6 ...(1)
From Kirchhoff’s voltage law to the closed
max = pE
Here, ciruit PQRP,
p = qa = 0.05 × 10–6 × 30 × 10–3 –2I1 – 2I1 + 2I2 = 0
= 1.5 × 10–9 cm –4I1 + 2I2 =0
2I1 – I2 = 0 ...(2)
E = 106 NC–1
–9 6 Adding (1) and (2), we get
max = (1.5 × 10 × 10 ) Nm
3I1 = 6
= 1.5 × 10–3 Nm.
I1 = 2A
1 From (1), I2 = 6 – 2 = 4A.
6. (c) As, mv2 = vq
2 12. (d) The number of electrons per unit volume,
2vq i.e., the electron number density in a
or, v = conductor is very large ( 10 28 m –3).
m
EBD_7443
www.jeeneetbooks.in

WWW.IIT-NEET.XYZ

MT-198 Target VITEEE


Therefore, a large current is obtained in a v2 = u2 + 2a S/2 = 2 (g sin ) S/2
conductor irrespective of small drift speed = gS sin
of electron and small charge.
13. (c) Magnetic field at the centre of a current For lower half of inclined plane
carrying loop is given by 0 = u2 + 2 g (sin – cos ) S/2
ni – gS sin = gS ( sin – cos )
B= 0
2r 2 sin = cos
For n = 1 turn
2 sin
i = = 2 tan
B= 0 ... (1) cos
2r
When n = 2 turns and radius 18. (d) Harmonics and fundamental frequency
r voltages cross-modulate with each other
r2 = , i2 = i producing fundamental frequency currents
2
2 i whose relation with the normal fundamental
B= 0 current vary with amount of harmonics in
r
2 the oscillator circuit. The phase angle of
2 the resultant fundamental frequency
2 0i 2 current thus varies resulting in frequency
or B = ...(2)
2r variation. Such frequency variation can be
Now, from eqn. (1) and (2) minimized by using a tank circuit having
B high effective Q having loosely coupled
=4
B load.
Hence, B = 4B
2
14. (d) Heat developed by the electric bulb, E2 r2 d r2 d
19. (b) P ( NBA)2
W P 210 5 60 R dt dt
H= =
J J 4.2 2
= 15000 Cal. r2 dB
= N 2 A2
15. (a) As the temperature rises the atoms of the dt
liquid become more mobile and the
coefficient of viscosity falls. N 2r 2
P
Reading on any scale – LFP
16. (c)
UFP LFP
= constant for all scales N 2r 2
Case 1 : P1
340 273 y ( 160)
373 273 50 ( 160) (4 N )2 ( r / 2) 2
Case 2 : P2
67 y 160 4
100 110 When we decrease the radius of the wire,
y = – 86.3° y its length increases but volume remains the
17. (b) same]
S/2 P1 1
h
oot S/2 sin
Sm P2 1

S/2 Power remains the same.


ugh S/2 sin 20. (b) The magnetic field at the centre of the
Ro coil
B(t) = µ0nI1.
As the current increases, B will also increase
For upper half of inclined plane with time till it reaches a maximum value
www.jeeneetbooks.in

WWW.IIT-NEET.XYZ

Solutions MT-199

(when the current becomes steady). where is magnetic flux


The induced emf in the ring
dr
d d d = B.2r
e ( B. A) A (µ0 nI1 ) dt
dt dt dt
The induced current in the ring 22 4 1
= 0.02 2
7 100 1000
|e| µ0 nA dI1
I 2 (t ) = 5.02 × 10–6V
R R dt
= 5 V (approx.)
dI1 26. (c) The equivalent inductance can calculated
decreases with time and hence I2 also
dt as the combination of resistances
decreases with time.] L1 = 0.4 H; L2 = 0.6 H
Where I1 = Imax (1 – e –t/ ) both are in parallel
The relevant graphs are
L1 L2
ie, L' = L L 0.24 H
1 2
Now, L' and L3 = 0.76 H are connected in
series
ie, L = L' + L3 = 0.24 + 0.76 = 1H.
27. (b) Ampere's law is : B.d µ0i in
Faraday law is :
d d
e= not e 0
21. (a) For resonant frequency to remain same dt dt
LC should be const. LC = const Biot-Savart's law is :

L µ0 id r
LC = L' × 2C L' dB .
2 4 r3
22. (c) Relative velocity = v + v = 2v
Gauss's law is : 0 E.dA q.
emf. = B.l (2v)
23. (b) Self inductance of a small circular coil of 28. (c) Given d = 1mm, = 6.5 × 10–7m.
D = 1m.
2 2
0N A 0N ( r2 )
radius r, L = 2n 1 D
2r 2r For n th dark fringe yn=
2 d
i.e. L N 2 For third fringe,
If no. of turns are doubled then self 2 3 1 1 6.5 10 7
inductance will be four times. y3 = 3
2 1 10
24. (d) The change in flux in both 1 and 2 are same
while in (3) and (4) is zero. = 1.625 mm.
25. (b) B = 0.02T ; = 90° 29. (b) I = I0 + I0 + 2 I0 I0 4I0
dr 1 I/I0= 4
1mm / sec m / sec
dt 1000 30. (c) E = h
(Since radius is decreasing) E
= ? ; when r = 4 cm = 0.04 m It is a straight line graph (c)
31. (d) Given N0 = 64 N
d d
=– (B r 2 ) 1
n
dt dt
N = N0
2
EBD_7443
www.jeeneetbooks.in

WWW.IIT-NEET.XYZ

MT-200 Target VITEEE

n n
1 1 1 1040
N 64N total number of reactions, n =
2 64 2 3
6 n Etotal = n × E
1 1
i.e.,
2 2 1040 12
= 3.875 10 J
n=6 3
but n = T/T1/2; T1/2 = 2 hrs = 1.29 × 1028 J
T = T1/2 × n We have Etotal = Pt
T = 2 × 6 = 12 hrs.
32. (d) The distance of closest approach of - E total 1.29 10 28
particle t=
P 1016
2
1 2Ze = 1.29 × 1012 s.
r=
4 0 E 35. (a) Given W = 3.2 eV
9 10 9
2 29 1.6 10 19
1.6 10 19 Vs = – 3.2 V.
= 13 36. (b) D2 is forward biased whereas D1 is reverse
10 1.6 10
= 8.35 ×10–15 m = 8.4 × 10–15 m. biased.
So effective resistance of the circuit
hc
33. (b) Energy of a photon = R= 4+2 =6

34 12
6.62 10 3 108 i 2A
= 6
300 10 9
37. (c) A crystal structure is composed of a unit
= 6.62 × 10–19 J
No. of photons emitted/sec/m2 cell, a set of atoms arranged in a particular
way; which is periodically repeated in three
IA 1 10 4
1019 10 4 dimensions on a lattice. The spacing
= =
(hc / ) 19 6.62
6.62 10 between unit cells in various directions is
called its lattice parameters or constants.
1015
= Increasing these lattice constants will
6.62
increase or widen the band-gap (E g), which
If one percent of incident photons produce
photo electrons, th en the no. of means more energy would be required by
photoelectrons emitted electrons to reach the conduction band
from the valence band. Automatically Ec and
1 1015
= = 1.52 × 1012/sec Ev decreases.
100 6.62
38. (d) Here y ( A B ) A.B A B . Thus it is
34. (b) 2 2 3 an AND gate for which truth table is
1H 1H 1H P
2 3 4 A B y
1H 1H 2 He n
0 0 0
31 H 2 2 He
4
P n 0 1 0
M = M(2He4) + M(p) + M(n) – 3M (1H2) 1 0 0
= (4.001 + 1.007 + 1.008) – (3 × 2.014) 1 1 1
= (6.016 – 6.042) = – 0.026 amu
| E | = | M | c2 = 0.026 × 931.5 MeV
= 0.026 × 931.5 × 1.6 × 10–13J
= 3.875 × 10–12J
As each reaction involves 3 deutrons, the
www.jeeneetbooks.in

WWW.IIT-NEET.XYZ

Solutions MT-201

39. (d) A NO2 NO2

H3N NH3 H3N NO2


C
CO CO
B NH3 NH3
NH3 NH3
The truth table for the above logic gate is :
NO2 NH3
A B C
Trans-form cis-form
1 1 1
1 0 1 Geometrical isomers
0 1 1 47. (d) The appearance of colour in solid alkali metal
halide is due to presence of F-centres found
0 0 0
as defect in the crystal structure.
This truth table follows the boolean algebra 48. (d) d-spacing
C = A + B which is for OR gate.
n 2 100
40. (c) Since conductor and semiconductor are = 200pm
connected in parallel hence voltage across 2 sin 2 sin 30 º
them is same. If ammeters show same 49. (d) Using the relation,
G = H – T S, we get
V For a reaction to be spontaneous the value
reading hence their resistances R are
I of G should be negative. When G = 0,
same. If voltage is increased by small value the equilibrium exists.
then following the same relation V I for Substituting the given values, we get
constant R, both conductor and G = 176.0 × 103 – T × 160
semiconductor show same current. or 0 = 176.0 × 103 – T × 160
41. (d) In d10 arrangement of Ni in Ni(CO)4 all (at equilibrium)
electrons are paired. or 160 T = 176.0 × 103
42. (b) ZnCl2 and CdCl2 are ionic, but HgCl2 is
176.0 103
covalent. or T = 1100 K.
43. (c) For a particle that contains 2 neutrons and 160
1 proton. Z = 1, A = 2 + 1 = 3. Thus it is 1T3. The reaction will be spontaneous above
1100 K at which it is in equilibrium.
44. (d) Ti(H2O )36 has one unpaired electron in Since, 1100 K = (1100 – 273)°C = 827°C.
its d-subshell which gives rise to d-d tran- So, the reaction will be spontaneous above
sition to impart colour. 827°C.
45. (d) 50. (d) S° = S (products) S (reactants)
46. (a) The given compound may have linkage
= [3 S (H 2 ) S (CO)] [S (CH 4 )
isomerism due to presence of NO2 group
which may be in the form –NO2 or –ONO. S (H 2O)]
It may have ionisation isomerism due to = [3 130.6 197.6] [186.2 188.7]
presence of two ionisable group –NO2 & –
= [391.8 197.6] [374.9]
Cl. It may have geometrical isomerism in the
= 214.5 JK–1 mol–1
form of cis-trans form as follows :
51. (b) Entropy states the randomness or
[Co(NH3 ) 4 Cl(NO 2 )]NO 2 & [Co(NH3 ) disorderness of the system. At absolute
(NO2)2]Cl ––– ionisation isomers. zero, the movement of molecules of the
[Co(NH3)5(NO2)2]Cl & [Co(NH3)5(ONO)2Cl system or randomenss of the system is
––– Linkage isomers zero, hence entropy is also zero.
EBD_7443
www.jeeneetbooks.in

WWW.IIT-NEET.XYZ

MT-202 Target VITEEE


52. (a) A reaction in which ng is zero is not
affected by change in pressure, considering H2C – CH2+RMgX CH2 – CH2
various given reactions O
OMgX R
(a) ng = 2 – 2 = 0 CH2 – CH2
H 2O
(b) ng = 2 – (3 + 1) = –2
–Mg(OH)X
(c) ng = 2 – 1 = + 1 OH R
[Consider only gaseous substances]
(d) ng = 2 – (2 + 1) = –1 High pressure
60. (d) HC CH HCHO
Thus, (a) is the correct option. acetylene formaldehyde
53. (c) Let us consider a reaction,
x X + yY aA+bB HC C CH 2 OH
rate = [X]x [Y]y
It is given that order of reaction w.r.t. H
|
component Y is zero. HC C CH 2 OH H C O
Hence, rate = [X]x
i.e., rate becomes independent of the
concentration of Y. HOCH 2 CCH 2 OH
54. (c) When concentration of each of the reactant But 2 yn, 4 diol
is taken as unity, the rate of a reaction is Therefore, acetylene with formaldehyde
called specific rate constant. Specific rate
under high pressure forms by butyndiol.
constant is depend on time as illustrated in
equ. (i) and also on temperature as illustrated 61. (a) If ethyl alcohol is dehydrolysed by conc.
in Arrhenius equation (ii) H 2SO 4 at 443°K, then ethylene is formed.
2.303 [A ]
k= log 0 ........ (i) C 2 H 5OH conc . H 2SO 4
C2H 4
t [A] H 2O
k = Ae–Ea/RT ........ (ii)
Cell constant 1.15 62. (d) The most suitable reagent for converting
55. (a) k alcohol to acetaldehyde is PCC. Other
R 250
reagent will converted into acid.
1.15 1000
eq 4. 6 63. (a) Higher the electron density on O, stronger
250 1 is the H-bond with water and thus more is
56. (c) Let x gm of Zn deposit on 9 gm of Hg the solubility. Thus solubility of the three
x ethers follow the order
% of Zn in Amalgam = × 100 = 25
9 x
x = 3gm
3 2 ..
> ..
> ..
Eq. of Zn = .O. .O. .O.
65.4
I III II
6 96500 electron pair e pair e pair more
Current = × = 8.85 amp. localised delocalised delocalised
65.4 1000
57. (b) The process of prevention of rusting of iron 64. (a) ArCOR’ can be prepared by the
by coating zinc over its surface is called combination of ArH + R’COCl and not by
Galvanization.
ArCOCl + RMgX because here the ArCOR
58. (a) Due to H-bonding, the boiling point of
ethanol is much higher than that of the formed will further react with RMgX to form
isomeric diethyl ether. 3º alcohol, ArC(OH) R2 as the final
59. (c) We know that products.
65. (b) Like aldehydes but unlike other acids, formic
acid reduces Tollen's reagent, Fehling
www.jeeneetbooks.in

WWW.IIT-NEET.XYZ

Solutions MT-203

solution, mercuric chloride and potassium 71. (a) Carboxylic acids decompose bicarbonates
permanganate into CO2 gas and H2O to form sodium salts
of carboxylic acids.
HCOOH 2CuO CO 2 H 2 O Cu 2 O -
Fehling sol. Cuprous oxide So, CO2 comes from HCO3 of NaHCO3
(reddish brown)
72. (b) Precipitates in two cases is due to different
66. (d) Hoffman Rearrangement compounds.
O O CH 3CH 2 CH 2 COO Ag + (Soluble in HNO3)
|| Br2 || From propanoic acid
R C NH 2 R C NHBr
AgCl (Insoluble in HNO 3 )
O From chloropropanoic acid
|| 73. (a) N-Chloro- or N-bromo-succinimide is the
KOH latest reagent used for -halogenation.
R C N Br
Br 74. (a)
O O 75. (d) The basic character of an amine in water is
|| || determined by (i) electron availability on the
: :

R–C–N C=N–R N atom and (ii) the extent of stabilization of


the cation (protonated amine) due to
Alkyl siocyanate solvation by hydrogen bonding
O H OH2
+
|| OH H +
C N R RNH 2 CO32 . CH3 N H CH3 N H OH2 ;
H 2O
So (d) is the answer as this intermediate is H H 2OH
+
not present in the reaction. Methyl amine Protonated methyl amine, CH 3NH 3
67. (b) When calcium acetate is heated, then (Highly stabilized)
acetone is formed. H OH2
+
CH 3 H +
CH 3COO CH3 N H CH3 N H OH2
Ca CO + CaCO 3
CH 3COO CH 3 CH3 CH3
Calcium acetate Acetone Dimethyl amine Protonated dimethyl amine
+
(Lesser stabilized than CH 3NH 3)
68. (d) 6HCHO 4NH 3 (CH 2 )6 N 4 6H 2 O
(urotropine) COOC 2 H 5
[Urotropine is a drug for treatment of urinary |
infection.] C N
|
(CH2)6N4 or C6H12N4 + 3HNO3 NH Cu powder
C3H 6O6 N6 3HCHO NH3 76. (c) heat ( N 2 )
RDX
(or cyclonite) [Y]
[HNO3 is a nitrating agent]. Pyrazoline derivative
69. (a) The reaction is used in purification of all CHCOOC 2 H 5
types of aldehydes and methyl ketones only oxidation
because the ketones having bulky alkyl
form unstable adduct.
70. (a) In H.V.Z reaction, in presence of small CHCOOH
amount of phosphorus, aliphatic carboxylic
acids containing -hydrogen atom reacts HOOC
with Cl2, Br2 to yield an acid halide in which
-hydrogen has been replaced by halogen. COOH
So, trichloroethanoic acid, among the given [Z]
options which do not contains -hydrogen
atom, will not give HVZ reaction.
EBD_7443
www.jeeneetbooks.in

WWW.IIT-NEET.XYZ

MT-204 Target VITEEE


77. (a) Completing the reaction, we get
x x , if 0 x 2
82. (b) Here, f x
O– x 1 x, if 2 x 3
CH3 at x = 2
CH2 – N
+ CH3 f 2 h f 2
LHD = Lf 2 lim
h 0 h
CH3 2 h 2 h 2 1 2
CH3 = lim
+ N h 0 h
CH3
'A' HO 2 h 1 2
= lim 2 h 1
+
h 0 h
78. (d) NH2 N NCl
2 h 2 h
= lim lim 1
NaNO2 ,HCl CuCN h 0 h h 0 h
O°C f 2 h f 2
Aniline (diazotisation) (A) RHD = Rf (2) = lim
h 0 h
benzene
diazonium 2 h 1 2 h 2 1 2
chloride = lim
h 0 h
h 1 2 h 2
= lim
h 0 h
2
h 3h 2 2
= lim
h 0 h
h 2 3h h h 3
= lim lim =3
h 0 h h 0 h
Cu/KCN
79. (d) N 2Cl CN LHD RHD
f(x) is not differentiable at x = 2.
H /H2O Sodalime 83. (a) Since the system is homogeneous and has
COOH
a non-trivial solution
80. (b)
p a b c p q 0
81. (b) Let 0 a q b c 0 q r
1 2 5 1 2 5 a b r c a b r c
A 2 4 a 4 ~ 0 0 a 6
using R1 R 1 R 2 and R 2 R 2 R 3.
1 2 a 1 0 0 a 6 Expanding along C1 we get
[R 2 R 2 2R1, R 3 R 3 R1 ] p[q(r c) br ] aqr 0
Clearly rank of A is 1 if a = –6
pqr pqc prb qra 0
1 2 5
Also, for a 1, | A | 2 4 3 0 a b c
1.
1 2 2 p q r
2 5 1 cos{2( x 2)} 2 sin( x 2)
and 6 20 14 0 84.(d) lim lim
4 3 x 2 x 2 x 2 x 2
rank of A is 2 if a = 1
2 sin( x 2)
L.H.L = lim 1
(at x = 2) x 2 ( x 2)
www.jeeneetbooks.in

WWW.IIT-NEET.XYZ

Solutions MT-205

87. (a) log42 – log82 + log162........


2 sin( x 2)
R.H.L = lim 1
(at x = 2) x 2 ( x 2) = log 2 2 2 log 23 2 log 2 4 2 .........

Thus L.H.L R.H.L 1 1 1


(at x = 2) (at x = 2) ......
=
2 3 4
1 cos{2( x 2)}
Hence, lim does not
x 2 x 2 1 1 1
= 1 ........ 1 = 1 – loge2
exist. 2 3 4
x x3
2 3 3 88. (d) a b 6
85. (a) y tan 1 3a x x
tan 1 a a3
3 2
a 3ax x2 2 2
1 3 a b 2a.b 36 ...(i)
a2
x 1 2 2
Let tan , then y tan tan 3 Similarly, b c 2b.c 64 ...(ii)
a
2 2
If 3 , then and c a 2c.a 100 ...(iii)
2 2 6 6
On adding eqs.(i), (ii) and (iii), we get
1 x
y 3 3 tan 2 2 2
a a b c a.b b.c c.a 100 ...(iv)
a a 1 x
x y 3 tan 2 2 2
3 3 a a b c 100 a.b b.c c.a 0
x 5 Now,
86. (d) Let sin 1 cosec 1
5 4 2 2 2 2 2
a b c a b c 2 a.b b.c c.a
1 x 1 5
sin cosec
5 2 4 2
a b c 100 [using (iv)]
1 x 1 4
sin sin
5 2 5 a b c 10

[ sin 1 x cos 1 x / 2] 89. (d) Let a 2 î 3ˆj k̂ and b î ĵ 2 k̂ , then


1 x 1 4 î ˆj k̂
sin cos … (i)
5 5 a b 2 3 1 5î 5ˆj 5k̂
4 4
Let cos 1 A cos A 1 1 2
5 5
C
unit vector perpendicular to the plane of a and
3
sin A = 5 b is
5 3
1
(î ĵ k̂ ) . If is the required angle, then
3 A 3
A = sin–1 4 B
5 2î 2 ĵ k̂ 1
cos–1(4/5) = sin –1 (3/5) cos . (î ĵ k̂ ) = 1
2 3 3 3
equation (i) become,
1 1 1 1
sin 1
x
sin 1
3 x 3 sin tan cot 1 ( 2 )
x=3 3 2
5 5 5 5
EBD_7443
www.jeeneetbooks.in

WWW.IIT-NEET.XYZ

MT-206 Target VITEEE


90. (b) Since 3 (1) + 2 (–2) + (–1)(–1) = 3 – 4 + 1 = 0 95. (a) The given line is
given line is to the normal tothe plane x 2 2y 5
i.e., given line is parallel to the given plane. z 1,
2 3
Also (1, –1, 3) lies on the plane
x 2 y z 0 if 1 – 2 (–1) –3 = 0 5
y
i.e. 1 + 2 – 3 = 0 x 2 2 z 1
which is true L lies in plane . 2 3 0
91. (b) f (x) = sin x + cos x, g (x) = x2 – 1 2
g (f (x)) = (sin x + cos x)2 – 1 = sin 2x This shows that the given line passes
5
Clearly g (f (x)) is invertible in – 2x through the point 2, , 1 and has
2 2 2
[ sin is invertible when – /2 /2]
3
direction ratios 2,
, 0 . Thus, given
– x 2
4 4 line passes through the point having
92. (a) Let Shamali invest ` x in saving certificate
and ` y in PPF. 5ˆ ˆ
position vector a 2iˆ j k and is
x + y 50000, x 15000 and y 20000 2
8 9 parallel to the vector
Total income = x y
100 100

Given problem can be formulated as b 2iˆ j 0kˆ . So, its vector equation
Maximize Z = 0.08x + 0.09y 2
Subject to, x + y 50000, x 15000, y 20000. is
93. (d) We have x 3 1 ( x 1) ( x 2 x 1) 5ˆ ˆ 3ˆ
r 2iˆ j k 2iˆ j 0kˆ .
Therefore, and are the complex cube 2 2
roots of –1 so that we may take =– and Hence, p = 0
=– 2, where 96. (b) We have, sin 8 0 and cos14 0
1 is a cube root of unity..
A = sin + cos14
8 0
100
Thus ( )100 But sin8 + cos14 = 0 is possible only if
100
sin = 0 and cos = 0 simultaneously which
and ( 2 )100 2
is not true for any value of
so that the required equation is A 0 or A > 0
x2 + x + 1 = 0. Also, 0 sin2 1
94. (c) Apply c1 c1 c 2 c 3 , then determinant 14
is sin 8 sin 2 and cos cos 2
2
2 1 1 sin 8 cos14 sin 2 cos 2 1
2 1 1 A 1
2
2 1 1
Hence, we get 0 A 1
2 1 1 2 97. (b) Circumference of a circular wire of radius
2 7 cm is
2 0
= 2 × 7 = 14
2 2
0
l
As we know, =
2( 3 2 3 3 4
) r
14 7 180
2( 2 1 ) 4 = = 210°.
12 6
www.jeeneetbooks.in

WWW.IIT-NEET.XYZ

Solutions MT-207

98. (c) The given parabola is y2 = 4ax.


c
Let (x1, y1) be the midpoint of the chord at 0, and passing th rough
2
passing through the point (h, k).
Equation of the chord having (x1, y1) as its c
mid point is c 2, .
2
T = S1 i.e yy1 – 2a(x + x1) = y12 – 4ax1
101. (c) Let direction cosine of the line be l, m, n
or yy1 – 2ax = y12 – 2ax1.
Since it passes through (h, k), DC 's
where given, cos =
ky1 – 2ah = y12 – 2ax1 r
The locus of the mid point (x1, y1) is DC's = r cos = rl
ky – 2ah = y2 – 2ax, rl = 12 … (i)
similarly r m = 4 … (ii)
or y 2 2ax ky 2ah 0 and r n = 3 … (iii)
99. (b) Let P (at2, 2at) be any point on the parabola Squaring and adding equations (i), (ii) and
y2 = 4ax. The equation of the tangent at P is (iii), we get
ty = x + at2 r2(l2 + m2 + n2) = 122 + 42 + 32
Since the tangent meets the axis of parabola r2 = 169 ( l2 + m2 + n2 = 1)
in T and tangent at the vertex A in Y, r = 13
coordinates of T and Y are (–at2, 0) and projection on x axis 12
(0, at) respectively. Now, l =
length of line segment 13

4 3
2
2at) Similarly, m = ,n
P (at , 13 13

G Y 12 4 3
Hence, direction cosines are , , .
13 13 13
X
T A x 2 y2
102. (a) Eqn. of hyperbola is 1
49 9
Its conjugate axis is y-axis.

Let the coordinates of G be (x1, y1) b2 9 58


Since TAYG is a rectangle. Also e 1
2
1
a 49 7
So, x1 = –at2 and y1 = at
Eliminating t, we get Foci of hyperbola ( ae,0),
2 i.e. ( 58 ,0) .
y1
x1 a y12 ax 1 0 Now equation of parabola with vertex at
a
The locus of G (x1, y1) is y2 + ax = 0 (0,–3) and axis along y-axis is x 2 ( y 3)
100. (c) Let (x, y) be the co-ordinates of point P.
It passes through ( 58 ,0) .
Given,
58
| x2 y2 y| c 58 (0 3)
3
Taking only positive sign and squaring
58
c Parabola is x 2 ( y 3)
x 2
c 2
2cy 2c y 3
,
2
58 11
which represents a parabola with its vertex Its focus is 0, 3 or 0,
4.3 6
EBD_7443
www.jeeneetbooks.in

WWW.IIT-NEET.XYZ

MT-208 Target VITEEE


103. (c) Using Lagrange's Mean Value Theorem y3 2y1y2
Let f(x) be a function defined on [a, b]
f (b) f (a) y4 2(y1 y3 y 22 )
then, f '(c) ....(i)
b a y5 2(y1y4 y 2 y3 2y 2 y3 )
c [a, b]
Given f(x) = logex y6 2(y1 y5 4y2 y4 3y32 )
1 Now putting x = 0, we obtain
f '(x) =
x (y)0 0, (y1 )0 0,(y2 )0 1, (y3 )0 0,
equation (i) become
(y 4 )0 2, (y5 )0 0 and (y6 ) 0 16 etc.
1 f (3) f (1)
Substitute the values in maclaurin’s series
c 3 1
1 log e 3 log e 1 log e3 x2
y (y) 0 x(y1 )0 (y ) ....
c 2 2 2! 2 0
2 We obtained
c c = 2 log3e
log e 3 x2 x4 x6
y log cosh x – ....
1 x x2 1 x x2
2 12 45
104. (a) (a) f ( x )
di
f ( x) 1 x x2 1 x x2 f ( x) 107. (d) Given i k tan k.sec 2
d
f(x) is an odd function
Let i be the error in i, corresponding to
x
a 1 error in
(b) f ( x ) x x
a 1 di
i k sec 2 . The relative
a x
1 d
f (–x) = ( x) x
a 1 i
error in i is
i

1 ax ax 1 1
( x) x f (x) i k sec2 .
1 a x
a x
1 cos 2
i k tan sin
It is an even function
cos
1 x2
(c) f ( x) log 1 cos
1 x2 2
. .
cos sin
2
1 x i 2 2
f ( x) log f ( x)
1 x2 i sin .cos 2sin .cos sin 2
It is an even function 108. (a) Alphabetical order is
(d) f(x) = k f (–x) = k = f(x) A, C, H, I, N, S
It is an even function No. of words starting with A = 5!
105. (c) No. of words starting with C = 5!
106. (b) We have
No. of words starting with H = 5!
y log cosh x No. of words starting with I = 5!
y1 tanh x No. of words starting with N = 5!
SACHIN – 1
y2 sech 2 x 1 tanh 2 x 1 y12 Sachin appears at serial no. 601
www.jeeneetbooks.in

WWW.IIT-NEET.XYZ

Solutions MT-209

/2 The required area is ACBDA, given by


sin x
109. (c) Let I = dx ... (i) 1 1
0 sin x cos x 2 2 5 y3
= (1 – 3 y – 2 y )dy y–
Then, 3
–1 –1
/2
sin( / 2 x) 5 5
I= dx 2 4
sin( / 2 x) cos( / 2 x) = 1 – 3 – –1 3 =2 sq. units.
0 3 3
/2
111. (b) Given curve meets x-axis at x = 0,1, 2
cos x
= dx ... (ii)
0 cos x sin x Y
Adding (i) and (ii), we get

/2 x=1 X
sin x
2I = dx O x=2
0
cos x sin x

/2
cos x
dx
sin x cos x
0 The required area is symmetrical about the
/2
point x = 1 as shown in the diagram.
sin x cos x So Reqd. area
= dx
0
sin x cos x 1 1
= 2 y dx 2 (x 3 3x 2 2x) dx
/2
0 0
1.dx [x]0 / 2 0
0
2 1
x4 3 2 1
/2 = 2 x x =2 1 1 = 1/2
sin x 4 4
dx 0
= 4 4
0 sin x cos x
/6

x 112. (b) cos7 3x dx


110. (d) x 2 y2 0 y2 – 0
t 3x
2
Put 3x = t x 0 /6
[Left handed parabola with vertex at (0, 0)]
3dx = dt t 0 /2
1 dx = 1/3 dt
x 3y2 1– ( x – 1) y2
3 /6 /2
[Left handed parabola with vertex at (1, 0)] 7 1
cos 3x dx cos7 t dt.
Solving the two equations we get the 0
3 0
points of intersection as (–2, 1), (–2, –1)
Y 1 6 4 2 16
. .
A (–2, 1) 3 7 5 3 105
113. (c) Any conic whose axes coincide with co-
D
ordinate axis is
X C (1, 0)
X ax2 + by2 = 1 .....(i)
B Diff. both sides w.r.t. 'x', we get
(–2, –1) dy
Y
2ax + 2by =0
dx
EBD_7443
www.jeeneetbooks.in

WWW.IIT-NEET.XYZ

MT-210 Target VITEEE


Then the number of games played by (n –
dy
i.e. ax + by =0 .....(ii) 2) players
dx
n 2
Differential again, = n – 2C2 C 2 6 84

2
(Two players played three games each)
d2 y dy n 2
a+b y =0 .....(iii) C2 78 (n 2)(n 3) 156
dx 2 dx
n 2 5n 150 0 n 15 .
a ydy / dx
From (2), 118. (a) Given, sin–1x – cos–1x = …(i)
b x 6
2
a d2 y dy sin–1x + cos–1 x= …(ii)
From (3), b y 2
2
dx dx Adding equations (i) and (ii), we get
2
dy 2sin–1 x = sin 1 x
y 2 3 3
dx d2 y dy
y 3
x dx 2 dx x sin
3 2
2 Given equation has unique solution.
d2 y dy dy 119. (d) The number of ways of arranging 50 books
xy 2
x y =0
dx dx dx = 50 P 50 = 50!. The number of ways of
choosing places for the five volume
114. (c) y2 4a(x h), 2yy1 4a dictionary is 50C5 and the number of ways
yy1 2a y12 yy1 0 of arranging the remaining 45 books =
45P = (45)! Thus the number of favourable
Degree = 1, order = 2. 45
ways is ( 50C5) (45 !). Hence the probability
115. (b) Let e be the identity element. Then
of the required event
ae
a *e a for all a a e 5 50
5 C5 45! 50! 45!
Let x be the inverse of 8. Then 50! 5! 45! 50!
8x 25
x *8 e x *8 5 5 x 1 1
5 8
5! 120
116. (c) {g ( x )} is real if g(x) 0 120. (c) With respect to A the following results are
in favour of the required event .
(x 1)(x 3) WW; WLW; WLLW; LWW; LWLW;
or 0
(x 2) LLWW
So, the desired probability
(x 1) (x 2) (x 3)
or 0 = p.p + pqp + pqqp + qpp + qpqp + qqpp
(x 2) 2 = p2(1+ 2q + 3q2)
or [ x ( 1)](x 2)(x 3) 0, x 2 121 (c)
122. (d)
+ + 123 (d)
– –1 2 — 3 124 (a) Tremulous means shaking or quivering
1 x 2 or x 3 slightly. Therefore, Steady is the opposite
of Tremulous.
Domain = [–1, 2) [3 )
117. (a) Let there be n participants in the beginning. 125: (d)
www.jeeneetbooks.in

WWW.IIT-NEET.XYZ

Solutions MT-211

MOCK TEST 8

1. (a) Energy radiated R2T4.


V2 V2
2. (a) 13. (b) H 15 60 t
3. (d) dU(x) = – Fdx R (2/ 3) R
x kx 2 ax 4 2
Ux Fdx or t 15 60 600 s 10 minutes.
0 2 4 3

2k 1 1
14. (a) K max h h hc
U = 0 at x = 0 and at x ; also at 0
a 0

x = 0, F is zero. Use hc 1.24 10 6 (eV) m


4. (b) The r.m.s. value of a.c. component of wave
is more than d.c. value due to barrier voltage 108 108
6
of p-n junction used as rectifier. hc 1.24 10
18 23
P R S1S 2
5. (b) where S 1.24 100 (23 18)
Q S S1 S 2
1.49 eV
6. (c) Resonance frequency is independent on 18 23
resistance. 15. (b)
7. (b) Total momentum = 2pî pˆj m0 m0 m0
16. (b) m*
Magnitude of total momentum v 2
1 (.96) 2 0.28
1 2
= 2 2 2
( 2 p) p 5p 5p c
This must be equal to the momentum of anew = 0.28 (4.2 × 1010) =1.176 × 1010 ms–2
the third part. 17. (b) From input signals, we have,
8. (d)
9. (b) Rnew = n2R A B Output NAND gate
1 1 1 0 0 1
10. (c) XC or X C
C 2 nC n 1 0 1
n 0 0 1
1 N 1
11. (b) gives n = 4.32 1 1 0
2 N0 20
0 0 1
t = nT = 4.3 × 3.8 = 16.5 days
12. (a) Point 1 and 3 are at same potential. Similarly The output signal is shown at B.
point 2 and 4 are at same potential. Joining
(0.45)t
resistance between 1 and 2, 2 and 3, 3 and 4 18. (a) X0 ( 1)t 5 10
we find that they all are in parallel. 5890 10
10
5 5890 10
t = 6.544 × 10–4 cm
0.45
1,3 2,4
19. (a) Clearly v1 = 2 ms –1, v2 = 0
m1 = m (say), m2 = 2m
So equivalent resistance v1' = ?, v'2 = ?
v1 ' v2 '
1 1 1 1 2 e= v v ....(i)
Rp 2R 2R R R or Rp = R/2 2 1
EBD_7443
www.jeeneetbooks.in

WWW.IIT-NEET.XYZ

MT-212 Target VITEEE


By conservation of momentum, 12 1
2m = mv1' + 2mv2' ... (ii) Again, i =
500 R 50
v2 ' v1 '
From (i), 0.5 = 500 + R = 600 R = 100
2
v2' = 1 + v1'
From (ii), 2 = v1'+ 2 + 2 v1' 25. (b) Max. K.E. = h – W0 ; so Max. K.E.
v1 = 0 and v2 = 1 ms–1 26. (d)
20. (b) Potential difference across the branch de is sin i 2
6 V. Net capacitance of de branch is 2.1 µF 27. (b) According to Snell’s Law,
sin r 1
So, q = CV
q = 2.1 × 6 µC where r = 90º for particular incidence angle
q = 12.6 µ C called critical angle. When the incidence
Potential across 3 µF capacitance is angle is equal to or greater than i c, then total
internal reflection occurs. It takes place
12.6 when ray of light travels from optically
V 4.2 volt
3 denser medium ( 1 > 2) to optically rarer
Potential across 2 and 5 combination in medium.
parallel is 6 – 4.2 = 1.8 V
1 2q V q
So, q' = (1.8) (5) = 9 µC 28. (a) q V m v 2 or v i.e. v
0 2 m m
21. (b) 1e is known as – particle & is
known as antineutrino. Since in this reaction v He q He mH
is emitted with 1 e 0( –
particle or vH qH m He
electron), so it is known as -decay.
2e m 1
0 id r =
22. (c) B for B1 , r ( ˆi ˆj) e 4m
2
4 r3
i ˆ 29. (a) Loss in K.E = Area under the curve
B1 0
k ( ˆi ˆj) ..... (1) 30. (a)
4 2 2
31. (b) Let x be the desired length
for B , r ˆi ˆj
2 E0 E0

0 ikˆ (iˆ ˆj)


B2 ......... (2) /3 x
4 2 2
From (1) and (2)
E E
B1 B2 and | B1 | | B2 |
Potential gradient in the first case E0
23. (b) Velocity of light in a medium,
1 1
c E0
E0
0 o r r E . …(i)
3 3
Potential gradient in second case
24. (a) 500
A E0 2E 0
i 3 /2 3
2V
R 2E 0
12V E (x) …(ii)
3
From equations (i) and (ii),
E0 2E 0
10 1 x x
12 – 2 = (500 ) i i 3 3 2
500 50
www.jeeneetbooks.in

WWW.IIT-NEET.XYZ

Solutions MT-213

32. (c) If R is radius of bigger drop formed, then 41. (d) Number of unpaired electrons in Cr, Mn2+
4 4 3 and Fe3+ are 6, 5 and 5 respectively.
R3 2 r or R = 21/3 r 42. (b) LiAlH4 does not reduce –NO2 group to
3 3
–NH2 group.
As v0 r2 SOCl 2
43. (d) R COOH RCOCl
2 1/ 3 2
v 01 R (2 r)
22 / 3 CH 2 N 2 Ag 2O
v0 2 2
r r RCOCHN 2 RCH 2 COOH
H 2O
or v 01 v 0 2 2 / 3 5 (4)1 / 3 Diazoketone
33. (b) The position of nth dark fringe. So position 44. (c) With ammonia, HCHO forms
of first dark fringe in x1 D / 2d . hexamethylenetetramine, CH3CHO gives
d = 20 cm, D = 0.1mm, = 5460 Å, acetaldehydeammonia addition product,
x1 = 0.16 while C6H5CHO gives hydrobenzamide.
45. (b)
34. (d) 0V –2V It can be sketched as
46. (c) Units of k show zero order;
rate = k = constant
Immobile ions
[A] reacted after 100 min = 100 × k
– + = 100 ×1.0 × 10–3 = 0.1 mol L–1.
P – + N [B] after 100 min = 2 × 0.1 = 0.2 mol L–1
0V – +
depletion
47. (c) A plot of m vs C is linear with negative
layer –2V slope.
35. (b) As momentum is conserved, therefore, 48. (c)
49. (c) In H2 – O2 fuel cell, anode :
m1 A1 v2 1
m2 A2 v1 2 H 2 (g) 2OH (aq) 2H 2 O( ) 2e
Cathode :
1/ 3 1/ 3
R1 A1 1
1 : 21 / 3 H 2O
1
O 2 (g) 2e 2OH (aq)
R2 A2 2 2
ˆ ˆ 0 Hence, net reaction :
36. (d) F q [v ( i)] B (i)
37. (a) From the graph it is clear that A and B have 1
H 2 (g) O2 (g) H 2 O( )
the same stopping potential and therefore 2
the same frequency. Also B and C have the
º 0.0592 1/ 2
same intensity. Ecell Ecell log P PO
2 H2 2
38. (d) The electric field deduced by Gauss’ law is
due to all the charges. 50. (c)
39. (b) Total internal resistance of two cells
K 2 Cr2 O 7 6H 2 SO 4 4KCl 2KHSO 4
1 1
0.5 . Since internal resistance of 4NaHSO4 2CrO 2Cl 2 3H 2O
1 1
coil is equal to external resistance (= 0.5 ), hc
hence power developed is maximum by 51. (b) E h ;
cells in circuit.
2 hc 6.63 10 34 (3 108 )
Current through R 2A 656 nm .
0.5 0.5 E 3.03 10 19
Power (2) 2 0.5 2 W. 52. (c)
40. (a)
EBD_7443
www.jeeneetbooks.in

WWW.IIT-NEET.XYZ

MT-214 Target VITEEE


53. (c) 59. (a) Aromatic aldehydes are weak reducing

agents, hence these do not reduce Fehling
OH O O O solution. On the other hand, aliphatic
aldehydes and –hydroxyketones are easily
:– oxidised by Fehling solution, i.e., they
reduce Fehling solution.
.–.
60. (b) Paraldehyde is a solid trimer of
acetaldehyde, while others are different
CH 2 = CHCH2Cl
forms of formaldehyde.
61. (d) Iron becomes passive on treatment with
OCH2CH = CH2 OH conc. HNO3 or acidified KMnO4, H2O2 or
CH2CH = CH2 H2CrO4.
+ + 62. (d) Under the given condition, k1 = k2
1011 e–3000/T = 1010 e–2000/T ; 10 = e1000/T
ln 10 = 1000/T, 2.303 log 10 = 1000/T;
OH T= 1000/2.303 K
63. (d) m ( BaSO4 ) Ba 2 SO42

m ( BaCl2 ) m ( H 2 SO4 ) 2 ( HCl )


CH2CH = CH2
º 1 º
x1 x2 2 x3 ; e m ( BaSO 4 )
n 1 n 1 2
(t1/ 2 )2 a1 0.1 64. (c)
54. (b) ; 10
(t1/ 2 )1 a2 0.01 65. (d)
O
n 2 ||
(a) H C Cl CO HCl
Rate = k [A] [B ]
55. (c) (b) CH 3CONH 2 HCl CH 3CO N H 3Cl
56. (b) Heavy metal ions, particularly Ag+, catalyse
(Acetamide as a weak base)
S reaction because of presence of empty 2 CH3CONH2 HgO (CH3CONH) 2 Hg
N1
orbital. (Acetamide as a weak acid)
LiAlH 4
(c) CH 3CONH 2 CH 3CH 2 NH 2 H 2O
CH3CH 2 X Ag [CH3CH 2 .... X .... Ag] or Na / C2H 5OH

+ – 66. (d) Use of SOCl 2 and ClCOCOCl forms


slow
CH3CH 2 OH CH 3CH 2OH gaseous by-products which can be easily
(–AgX) from H 2O removed, giving better yield of RCOCl.
57. (b) The momenta of fission fragments are equal. Further, oxalyl chaloride is particularly easy
to use becasue any excess of it can be easily
1 2 m2 v 2 p2 evaporated due to its low b.p. (62ºC)
K .E. mv
2 2m 2m O O O
|| || ||
(p = momentum) R C OH Cl C C Cl R

K.E. of lighter fragment E 144 O


||
K.E. of heavier fragment 70 90 R C Cl HCl CO CO 2
E 112 MeV 67. (d) KNO3 is added to oxidise the lead and zinc
impurities.
Thus, total fission energy = 70 + 112
H
= 182 MeV 68. (d) CH 3CH 2 OH CH 3CH 2 O H 2
(from H 2SO 4 )
58. (b)
www.jeeneetbooks.in

WWW.IIT-NEET.XYZ

Solutions MT-215

On electrolysis of aqueous solution of the


CH3 C H 2 HSO4
CH 2 CH2 H 2SO4 complex ion of chromium [i.e.,
(from H 2SO 4 )
{Cr (NH 3 ) 6 } 3+ ] of the complex
Thus note that H2SO4 is acting as an acid in [Cr(NH3 ) 6 ][Co(NO 2 ) 6 ] moves towards
step (i), as a base in step (iii). Since it is cathode (i.e., negative electrode) and on this
regenerated back as such it also acts as a electrode chromium would finally be
catalyst. deposited.
69. (c) Since two Ag+ ions are replaced by one Cd2+ –

ion to maintain electrical neutrality, there 77. (d) HCCl3 OH H 2 O : CCl3


are cation vacancies.
: CCl 2 Cl
C2 H5OH
70. (b) (CH 3CO) 2 O Dichlorocarbene
( A) (a neutral electrophile)

O O
CH 3COOH CH 3COOC 2 H 5 – H
( B) ( C) : :CCl2
CCl2
71. (d) In presence of AlCl3 (a Lewis acid), aniline
is converted into anilinium cation, which – –
being m-dir ecting gives aminoaceto- O O
phenone. CHCl2 – CHO
6 OH
72. (c) (a) d : d sp (n = 0)2 3

(b) d 8 : sp3 d 2 (n = 2) 78. (c)


6 3 2 79. (b)
(c) d : sp d (n = 4) 80. (b) Reaction is an example of Schmidt reaction
(d) d 4 : (d 2 sp3 ) (n = 2) in which carboxylic acids are heated with
73. (c) hydrazoic acid in presence of a mineral acid
74. (c) to form primary amines.
conc . H 2SO 4
CH 3 NH 2
CH3I
(CH 3 ) 4 N I
AgOH RCOOH HN 3 RNH 2 CO 2
excess ( A) 2
81. (b) f : , [0, 4], ,
(CH 3 ) 4 N OH
heat
(CH 3 ) N CH 3OH 3 3
( B) (C) and (D) f (x) 3 sin x cos x 2
75. (d) All of the three are oxidation products of a
1º amine. f(x) = 2sin x 2
H OH 6
[O] [O]
R – .N. – H R – .N. – H x 2
(f 1 (x)) sin 1
6 2
1º Amine Hydroxylamine
82. (c)
O 83. (d) tan is of period so that tan 3 is of period
[O] [O] .. [O] + /3.
H R – .N. – H R–N=O R–N
O– x x
Nitroso Nitro 84. (b) f (x) x
1
76. (d) The given compounds are isomeric. They e 1 2
can be distinguished by electrolysis of their 2x xe x x x xe x
aqueous solution. = 1 1
On electrolysis of its aqueous solution the 2 (e x 1) 2 (e x 1)
complex ions of cobalt [i.e., {Co(NH3)6}3+] x xe x x xe x
of the complex [Co(NH3 )6 ][Cr(NO 2 )6 ] f ( x) x
1 1
moves towards cathode (i.e., negative 2 (e 1) 2 (e x 1)
electrode) and on this electrode finally f (–x) = f (x) for all x
cobalt would be deposited. f (x) is an even function.
EBD_7443
www.jeeneetbooks.in

WWW.IIT-NEET.XYZ

MT-216 Target VITEEE


85. (b) If the given points be A (2, 3, 4) and B (6, Let E denote the event that the person is
7, 8), then their mid-point N(4, 5, 6) must diagnosed to have TB.
lie on the plane. The direction ratios of AB 1 999
are 4, 4, 4, i.e. 1, 1, 1. P(A) = ,P B
1000 1000
A (2, 3, 4)
E E
P 0.99, P 0.001
A B
The required probability is given by
N
E
P A
A A
P
E E E
P A P P B P
B (6, 7, 8) A B
The required plane passes through N (4, 1
5, 6) and is normal to AB. Thus its equation 0.99
1000
is =
1 999
1(x 4) 1( y 5) 1(z 6) 0 x y z 15 0.99 0.001
1000 1000
86. (a) The equation of the given line is
6x = 4y = 3z 0.99 0.99
=
which is written in symmetric form as 0.99 0.001 0.999 0.99 0.999
x 0 y 0 z 0
990 990 110
1/ 6 1/ 4 1/ 3 =
990 999 1989 221
1 1 1
Direction ratios of this line are , , 89. (a) z 1 2i |z| 1 4 5
6 4 3
and equation of the plane is 3x + 2y –3z – 4 7 z 7 1 2i
=0
f ( z)
1 z 2 1 (1 2i ) 2
If be the angle between line and plane,
then direction ratios of the normal to this 6 2i 6 2i 3 i
plane is (3, 2, – 3) 1 (1 4 4i ) 4 4i 2 2i
3 i |3 i|
a1a 2 b1b2 c1c 2 | f ( z) |
sin 2 2i | 2 2i |
a12 b12 c12 a 22 b 22 c 22
9 1 5 |z|
1 1 1 4 4 2 2
3 2 ( 3)
6 4 3 0 z1 z1
1 1 1 90. (a) z1 | z1 |2 z1 1
9 4 9 z1
36 16 9
= 0° arg(z1–1) = arg (z1 ) = arg (z2)
z2 = kz1–1 (k > 0)
x x 91. (b) We have, nPr = nPr + 1
87. (c) 5 r 1
1 r 5
n! n! 1
Since, G.P. contains infinite terms 1
–1 < r < 1 (n r )! (n r 1)! (n r)
x or n–r=1 ...(1)
1 1 1 0 x 10, x 5 Also, nCr = nCr – 1 r + r – 1 = n
5 2r – n = 1 ...(2)
88. (a) Let A denote the event that the person has Solving (1) and (2), we get r = 2 and n = 3
TB
Let B denote the event that the person has 92. (a) (a * b)2 (a *a)*(b * b) for all a, b G
not TB. (a *b)*(a *b) (a *a)*(b * b) for all
www.jeeneetbooks.in

WWW.IIT-NEET.XYZ

Solutions MT-217

a, b G [Applying R3 – R2 and R2 – R1 in second det.]


a *(b *a) *b a *(a * b) *b for all 9 k 16 3
a, b G 7 9 1 = 0 [Applying R3 – R2]
b * a a * b for all a, b G (by can- 2 2 0
cellation laws)
G is abelian 9 k 7 k 3
93. (a) f (x) = min {x + 1, | x | + 1} f (x) 7 2 1 =0
=x+1 x R 2 0 0
Y [Applying C2 – C1]
2 (7 – k – 6) = 0 k=1
y=–x+1 y=x+1 96. (a) Given f(x) = x2 – 4x – 5

(0, 1) f (A ) A2 4A 5I
1 2 2 1 2 2 9 8 8
X' Now, A 2
X 2 1 2 2 1 2 8 9 8
(–1, 0)
2 2 1 2 2 1 8 8 9
Y'
9 8 8
Hence, f (x) is differentiable everywhere for
2
all x R. A 4A 5I 8 9 8
C1 C 3 C 5 8 8 9
94. (d) .....
2 4 6
1 2 2 1 0 0
n n (n 1)(n 2)
4 2 1 2 5 0 1 0
2 4!
2 2 1 0 0 1
n (n 1)(n 2)(n 3)(n 4)
+ ......
6! 0 0 0
0 0 0 O
1
(n 1)n (n 1)n (n 1)(n 2)
0 0 0
n 1 2! 4!

(n 1)n (n 1)(n 2)(n 3)(n 4) cos sin


..... 97. (b)
6! 1 tan 1 cot

1 cos sin
[ n 1C2 n 1
C4 n 1
C6 .....] =
n 1 sin cos
1 1
1 2n 1 cos sin
2n 1 1 n 1
C0
n 1 n 1
95. (b) Breaking the given determinant into two cos 2 sin 2
=
determinants, we get cos sin cos sin
32 k 42 32 k 32 k 42 3
cos2 sin 2
4 2
k 5 2
4 2
k + 4 2
k 5 2
4 =0 = = cos + sin
cos sin
2 2 2 2 2
5 k 6 5 k 5 k 6 5 98. (b) Given: x2 – y2 sec2
= 4 and x2 – sec2 + y2 = 16
9 k 16 3
x2 y2
0+ 7 9 1 =0 1
4 4 cos 2
9 11 1
EBD_7443
www.jeeneetbooks.in

WWW.IIT-NEET.XYZ

MT-218 Target VITEEE


This represents an ellipse centred at (–1, 0)
x2 y2
and 1 1
16 cos2 16 4 3
and of eccentricity = = .
According to problem 4 2
103. (c) Let x1 be the abscissa of the point whose
4 4cos 2 16 16 cos 2 ordinate is 6.
3
4 16 Since the point (x1, 6) lies on y2 = 6x,
36 = 6x1 or x1 = 6
1 + cos2 = 3(1 – cos2 ) 4 cos2 = 2 The point is (6, 6)
1 3 Equation of tangent at (6, 6) is
cos ,
2 4 4 3
y 6 2
( x 6) ( yy1 = 2a (x + x1))
2
99. (a) We have, a cos A b cos B c cos C
a b c or x 2 y 6 0
R (sin 2A sin 2B sin 2C) x2 y2
104. (b) The eccentricity of 1 is
2R (sin A sin B sin C) 16 25
4 sin A sin B sin C 16 3
e1 1
A B C 25 5
2 4 cos cos cos
2 2 2 5
e2 ( e1e2 = 1)
A B C r 3
4 sin sin sin foci of ellipse = (0, 3)
2 2 2 R
Equation of hyperbola is
1 1
100. (b) We have, sin [cot (cos (tan x ))]
x2 y 2
1
16 9
1 1
= sin cot 105. (a) We have x3 – 3xy2 + 2 = 0 ......... (1)
1 tan tan 1 x2 and 3x2y – y3 – 2 = 0 ......... (2)
Diff. (1) and (2) w.r.t. x, we obtain

1 1 dy x2 y2
sin cot = and
dx 2xy
1 x2 C1

dy 2xy
1 =
dx C2 x y2 2

1
1 cot 2 cot 1 Since m1 × m2 = – 1, therefore the two
1 x2 curves cut at right angles.
106. (b) For the curve yn = an – 1 x
On putting n = 2, we get y2 = ax
1 1 x2 Which is curve of the form of parabola,
1 2 x2 where the subnormal at any point is a con-
1 stant.
1 x2 n = 2.
101. (b) 5 and 12 are two sides of a right angled
triangle and dx dy
107. (d) 2 c t and 2bt
dt dt
x = hypotenuse = 52 122 13
2
2 x2 dx 2 dy
102. (b) y2 + x = 3 v
dt dt
(x 1) 2 y2
+ = 1.
4 3 { 2ct 2 2bt 2 } 2t c 2 b2
www.jeeneetbooks.in

WWW.IIT-NEET.XYZ

Solutions MT-219

3 9
lim
2 8 h 5 5 10 510
108. (d) =
h 0 2h.3 8 h 6 6 6 2 69
111. (a) 2
If f (x) = ax + bx + c,
8 (8 h)
lim 1
h 0 2h.3 8 h{82/3 81/3. 1
f (x) dx = (2a + 3b + 6c)
(8 h)1/3 (8 h)2/3} 0 6
1 1 a b
48 f (0) = c, f = + + c, f (1) = a + b + c
2 4 2
109. (c) Let the boxes be marked as A, B and C. We
have to ensure that no box remains empty 1
f (0) + 4f + f (1) = 2a + 3b + 6c
and all five balls have to put in. There will 2
be two possibilities : 1
(i) Any two box containing one ball each
and 3rd box containing 3 balls. Number 112. (b) Given f’(x) = f(x) + f (x ) dx ...(1)
0
of ways
= A(1) B(1) C(3) Differentiating we get,
= 5C1 . 4C1 . 3C3 = 5 . 4 . 1 = 20 f " (x )
1
f " (x ) f ' (x ) 0
(ii) Any two box containing 2 balls each f ' (x )
and third containing 1 ball, the number
of ways f " (x)
On integrating dx dx
= A(2) B(2) C(1) = 5C2 . 3C2 . 1C1 f ' (x)
= 10 × 3 × 1 = 30
ln f ' ( x ) x c f ' (x ) ex c
ke x
Since, the box containing 1 ball could be
any of the three boxes A, B, C. Hence, the where k = ec, a constant
required number of ways = 30 × 3 = 90. Again integrate
Hence, total number of ways = 60 + 90 = f ' (x) dx kex D f ( x) kex D ...(2)
150.
Put x = 0 in (1), f(0) = ke°+D k+ D= 1
110. (d) It is a case of Bernoulli trials, where success
is not crossing a hurdle successfully. Here, (given f(0) = 1) ...(3)
n = 10. 1
Also from (1), f’(x) = f(x) + f ( x ) dx
5 1 5
p = P (success) = 1 q= 0
6 6 6
1
let X be the random variable that represents
the number of times the player will knock ke x ke x D (ke x D) dx
down the hurdle. 0
Clearly, X has a binomial distribution with 1
ke x Dx D 0 ke D k D 0
1 0
n = 10 and p =
6 k (e 1) 2D 0
P(X = r) = nCrqn–r. pr Solving (3) and (4) , we get
1 r 5 10 r k
2
and D
1 e
= 10Cr 3 e 3 e
6 6
P (player knocking down less than 2 2e x 1 e
hurdles) f (x )
3 e 3 e
= P(X < 2) = P(X = 0) + P(X = 1)
113. (d) p (u v) (v w ) v( w u)
10 1 0 5 10 0 10 1 1 5 9
= C0 C1 p.w (u v ). w (v w ). w
6 6 6 6
EBD_7443
www.jeeneetbooks.in

WWW.IIT-NEET.XYZ

MT-220 Target VITEEE

v( w u ). w 1 4
x2 1 x2
= 3 4x
2 3 2
[u v w] 0 0 0 1
5
1 1 1
5( p . w ) = 3 4 4 1 16 1
2 3 2
Similarly, 5 ( p . u ) and v 5 ( p . v )
3 1 15 3 9
= 12
v 5( p . w ) 5( p . u ) 5( p . v ) 2 3 2 2 2 3

5 p.(u v w) 3 3 3
= 2 3 sq. units
Hence, depends on the vectors 2 2
116. (b) Let T be the temperature of the cooling
114. (b) The characteristic equation is p2 + 6p + 8 = 0 object at any time t.
(p + 4) (p + 2) = 0 p = – 4 and – 2
The C.F. is Ae–4x + Be–2x dT dT
(T S) k(T S) T S ce kt ,
Particular integral, dt dt
1 where k is negative.
2x
P.I. = e
D 2
6D 8 T S ce kt
When t = 0, T = 150 150 = S + c
1 c = 150 – S
e 2x
(D 4)(D 2) The temperature of the cooling object at
Since, f(D) = (D + 2) q (D) any time is
T = S + (150 – S)ekt
1 1
xe 2x xe 2x 117. (c) Without loss of generality,
( 2) 2 let the right angled OAB be
Hence the general solution is such that OA = OB = a units.
1 Along OA take unit vector as
4x 2x 2x
y Ae Be xe B
2
115. (b) The tangent on x2 + y2 = 4 at 1, 3 is

x 3y 4 and equation of normal at D

1, 3 is y x 3
A
Y O E

î and along OB taken unit vector


(1, 3 ) as ĵ , so that

ˆ OE aˆ ˆ OD aˆ
OA ai; i; OB aj; j;
2 2
X a
(0, 0) (4, 0) AD OD OA ĵ a î ;
2
a
BE OE OB î aˆj ;
1 4 2
4 x
Required area = x 3 dx dx a a
3 AD . BE ĵ a î . î aĵ
0 1 2 2
www.jeeneetbooks.in

WWW.IIT-NEET.XYZ

Solutions MT-221

5a 2 5a 2 3 ˆ 3 3ˆ
cos a2 = 4 i j
4 4 2 2

4 4 8 3 ˆ 3 3ˆ 5ˆ 3 3ˆ
cos ; cos 1 = i j i j
5 5 2 2 2 2
Hence the girl’s displacement from her
118. (c) Let O and B be the initial and final positions
of the girl respectively. 5ˆ 3 3 ˆ
initial point of departure is i j
Then, the girl’s position can be shown as 2 2
in the figure. 2

Now, we have OA = 4iˆ 119. (a) F(2) F(X 2) f (x)dx

AB ˆi AB cos 60 ˆj AB sin60

2 2x 2
B N 2x e
3 km 2e dx 2.
0
2
0

30° 4 4 e4 1
[e 1] 1 e
60° e4
W E
A 4 km O 120. (a) Logical expression corresponding to above
circuit is = a ' b' c
Logical expression corresponding to the
complimentary to the above circuit is
S
(AB cos 60° is component of AB along X- (a ' b' c)' (a ' b' )' . c'
axis and AB sin 60° is component of AB
along Y-axis) = (a ' )' . (b' )'. c'

ˆ 1 ˆ 3 3ˆ 3 3 ˆ = a . b . c' [(a ' )' a ; (b ') ' b]


= i 3 2 j 3
2 2
i
2
j
121. (b)
By the triangle law of vector addition, we 122. (c)
have 123. (d)
124. (c) Ruminate means to think deeply about
3ˆ 3 3ˆ
OB = OA + AB = 4iˆ i j something.
2 2 125. (b)
EBD_7443
www.jeeneetbooks.in

WWW.IIT-NEET.XYZ

MT-222 Target VITEEE

MOCK TEST 9

Total work done


1. (d) E AT 4
Wg Wf 15 kJ 8 kJ 23 kJ
A R2 E R 2T 4
1 q
E2 R 22 T24 7. (b) For r R: E = 4
0 r2
E1 R12 T14
For r R: R
put R 2 2R, R1 R –Q
1
T2 2T, T1 T E.dA qin Q
0
E2 (2R) 2 (2T) 4 or E × 4 r2 = 0
64
E1 R 2T 4 E=0
8. (c) Let us first calculate the total charge on the
2. (c) –
soild sphere.
Let us consider a concentric sphere of
A. .B .C
radius r and thickness dr.
Then volume of the sphere, dV = 4 r2dr
Given, the volume charge density of the
R1
EA = 0, EC = 0, sphere =
0 r dr
2 0 2 0 r
EB = .
2 0 2 0 0
3. (a) Charge on this sphere,
dV 0 2
4. (b) E = slope of V x curve dQ = .dV .4 r dr = 4 .rdr..
dx r
dV dV Total charge on the whole solid sphere,
R1 R1
dx (2) dx (4) Qs dQ 4
0 0 0 rdr
Since for region–2, V = +ve
E2 = –Ve R12
and for region-4, V = –ve = 4 0 2 0 R12
2
E4 = +ve QS = 2 0R12 ...(1)
dV dV Now, the total charge on the hollow sphere,
dx (1) dx (3) (for region 1 and 3, potential Qh = – (4 R22) ... (2)
By question, Qs + Qh = 0
remains constant so E = 0) 2 0R12 = 4 R22
So, E2 > E4 > E1 = E3
2
5. (b) The negative charge will move opposite to R2 0 R2 0
the direction of field. R1 2 R1 2
6. (b) Work done against gravity
Wg = 50 × 10 × 30 = 15 kJ 9. (a) Terminal potential difference,
Work done against friction V = E – Ir
(i) This is the case when cell is discharged
Wf mg cos s
so, V < E.
4 (ii) This is the case when cell is charged
= 0.4 50 10 50 8 kJ so, V > E
5
(iii) When I = 0 so, V = E
www.jeeneetbooks.in

WWW.IIT-NEET.XYZ

Solutions MT-223

10. (a) In circuit 1 , on closing the switch, the 2Er1


current through the inductor is zero. (i1 = 0) Now, E 0
(r1 r2 ) R
i1 R1
2Er1
E 2r1 r1 r2 R
r1 r2 R
R = r 1 – r2
R2
13. (d)
In circuit 2 , on closing the switch, the
current through the inductor is zero and i 2 R

E i
=
R2 e
The equivalent current, i
T
In circuit 3, on closing the switch , the Magnetic moment is given by
current through the inductor is zero and e
E M iA A
i3 = T
R1 R 2 14. (d) In the galvanometer, Ig = max. current
th rough galvan ometer, S = shunt
resistance, G = galvanometer resistance
then
S
Ig(galvanometer) = I
G S
G S 50 1
Thus i2 > i3 > i1 I Ig Ig 51Ig
S 1
A
40 15. (b) The time period of oscillation
B
11. (d) 20 T =2 LC
40 8V
C = 2 20 10 3 50 10 6
= 6.28 × 10–3 s.
The required time will be
T
60 40 t 1.54 ms
=
4
16. (c) The self induction and resistance of
The equivalent resistance of the circuit each part become 0.9 × 10–4 H and 3
60 40 respectively. The effective value of self
R= 24 inductance
60 40
4
V 8 1 0.9 10
Current, i = A L 0.45 10 4 H
R 24 3 2
12. (d) Current in the circuit 3
and R 1.5
E E 2E 2
r1 r2 R r1 r2 R Thus time constant
P.D. across first cell = E – ir 1 L 0.45 10 4
= 0.3 × 10–4
E r1 R 1.5
E
(r1 r2 ) R
EBD_7443
www.jeeneetbooks.in

WWW.IIT-NEET.XYZ

MT-224 Target VITEEE


17. (c) Change in momentum along the wall 22. (b) The power consumed in series is given by
= mv cos60º – mv cos 60º = 0 1 1 1
Change in momentum perpendicular to the
wall P P1 P2
= mv sin60º – (– mv sin60º) = 2mv sin60º P1 P2 100 200
Applied force = P 66 W
P1 P2 100 200
Change in momentum
23. (a) The related light becomes plane polarised.
Time 24. (c) The direction of propagation of em-wave is
2 mv sin 60º 2 3 10 3 z. As electric field and magnetic field must
= = be perpendicular and so direction of
0.20 2 0.20
oscillations of E will be x.
= 50 3 3 = 150 3 newton
18. (a) At any time t, the side of the square 25. (d) Fpe Fep and F pe
' F ep
' .
a = (a0 – t), where a0 = side at t = 0. Thus (d) option is not correct.
At this instant, flux through the square : 26. (b) Since work function for a metal surface is
= BA cos 0° = B (a0 – t)2 hc
d W
emf induced E = – 0
dt where 0 is threshold wavelength or cut-off
E = – B.2 (a0 – t) (0 – ) = +2 aB wavelength for a metal surface.
r here W = 4.125,
19. (d) A B eV = 4.125 × 1.6 × 10–19 Joule
34
I v 6.6 10 3 108
so 0 19
3000Å
4.125 1.6 10
D C h
As flux decreases the flux current in loop is 27. (b) Wavelength, = . As each of them has
p
clockwise. Force on DA due to long wire
and BC is towards left while on wire BC is same momentum, so
towards right. e p.
e
20. (b)
A v
28. (d) We have
h
h = h 0 + Kmax
××× × × ×b× × × × ××× h K max
××× ××××××× ×××
×××
×××
×××××××
×B× × × × ×C×
×××
××× 0= h
××× ××××××× ×××
××× ××××××× ××× 34
××× ××××××× ××× 6.63 10 1015 3 1.6 10 19
=
dA 6.63 10 34
B.
| d / dt | dt = 2.7 × 1014 Hz
I
R R R 29. (c) Mass of nucleons
d 1 dh 7 1.00783 7 1.00867 14.1155 u
B h.b Bb The mass defect, m = 14.1155 – 14.00307
dt 2 dt Bbv = 0.11243 u
R R Thus binding energy = 0.11243 × 931
b t I t = 104 MeV.
21. (d) R = 2k = 5t2 – 4t + 12, t = 0.2 sec.
V0 10
d 30. (c) V V
Induced emf e =
dt
e = – 10t + 4 = –10 × 0.2 + 4 = + 2 V 31. (a) F (W X ). (W Y ) W .( X Y )
32. (d) Voltage regulator needed constant voltage
e 2 and so de part is most relevant for its
I= = 10–3 A= 1 mA
R 2 103 operation.
www.jeeneetbooks.in

WWW.IIT-NEET.XYZ

Solutions MT-225

33. (b) Because atomic mass M of carbon,


41. (c) d-d electronic transition causes absorption
MC < MSi < MGe
of energy at red visible wave length.
So energies are inversely proportional to
In cupric chloride CuCl2, cupper is in +2
that of atomic mass.
34. (c) In semiconductors, the conduction is state and hence it is d9 system. In five d-
orbitals there is one unpaired electron
empty and the valence band is completely
present which absorbs energy due to d-d
filled at 0 K. No electron from valence band
transition.
can cross over to conduction band at 0 K.
42. (b) Stainless steel is an iron-carbon alloy with
But at room temperature some electrons in
a minimum of 10.5% chromium content.
the valence band jump over to the
conduction band due to the small forbidden Stainless steel has higher resistance to
oxidation (rust) and corrosion in many
gap, i.e., 1 eV.
natural and man made environments
35. (d) On the basis of given graph, following table
because the chromium forms a passivation
is possible.
layer of chromium (III) oxide (Cr2O3) when
A B C exposed to oxygen. The layer is too thin to
0 0 0 be visible, which means that the metal
remains lustrous. It is, however, impervious
1 1 1 to water and air, protecting the metal
0 1 0 beneath. Also, this layer quickly reforms
when the surface is scratched.
1 0 0
43. (b) After crystal field spliting, the five d-
It is the truth table of AND gate. orbitals. get separated as three t2g and two
1 1 orbitals. For Mn 2 +, last shell has 5 e–s i.e.,
2 2
36. (c) As mA v A mB v B 3d5 4s0 . So according to Hund's rule of
2 2 maximum multiplicity the excited state
mB PB mB vB configuration will be t2g3 eg2.
vA
; P mA vA 44. (c) Colour is due d xy eg [d 2 2 , d 2 ]
vB mA A x y z
electronic transition.
mB mA mB 1 If we see the d elctronic configuration of Ti,
mA mB mA 3 it has d1 configuration. Since [Ti(H2O)6]3+
is an octahedral compound hence the d-
37. (d) All options (a), (b) and (c), are correct. orbital splitting will be
38. (d) d x 2–y2 , d z2
39. (b) For high-side tunning Energy
eg
fLO = fm + fIF , PIF 600 kHz 0.6 MHz electron transition
f LOL 5 0.6 5.6 MHz t2g e
dxy dxz dyz
f LOU 10 0.6 10.6 MHz
Here an electron which is present in t2g
40. (b) Let the angle subtended by the arc formed excites and transition occuring from
be . Then
d xy eg[d 2 2 , d 2 ] .
x y z
2 1
or
r r r1 r2 45. ( d) (a) In [Fe(C5 H 5 ) 2 , EAN of Fe 26
electron of Fe 10 electron from two
( 2 1) T
t C5 H 5 ions
6

or
r
( 2
t
1) T
(b) [Fe(H 2 O) 6 ]2 :
3d
..
4s

4p
So, r
t .. .. .. .. .. 4d
sp3 d 2
( 2 1) T
( H 2O a weak field ligand)
EBD_7443
www.jeeneetbooks.in

WWW.IIT-NEET.XYZ

MT-226 Target VITEEE


6

(c) [Cr( NH 3 ) 6 ] 3
:
3d
..
4s G
2
H
2
1 G
T T T T
P
4p
.. .. .. d 2 sp3 (3 unpaired electrons)
on rearrangement, we get
7 G H

(d) [CoI 4 ]2 :
3d 4s
.. T
P
T2
4p
.. .. .. sp3 (tetrahedral) H –T 2
T
G

46. (c) A transition metal ion that exists in its P


highest oxidation state can be expected to 1 1
behave as a oxidising agent as it can not be 50. (b) H (g ) Cl 2( g ) HCl (g )
2 2
further oxidised but can be reduced.
S° = S° (Products) – S° (reactants)
47. (a) Non-stoichiometr ic compounds are
chemical compounds with an elemental 1 1
S HCl S H2 S Cl2
composition that cannot be represented by 2 2
a ratio of well-defined natural numbers, and
are therefore in violation of the law of 1
187 (131 223) = 10 JK-1
definite proportions. They are most often 2
solids that contain random defects,
H
resulting in the deficiency of one element. 51. (a) S
So, none of given option is correct. T
48. (b) In orthorhombic geometry, H per mole 6000
y S(per mole )
T 273
c 21.98 JK 1mol 1
52. (a) Substance R Substance S
2k k rate constant
a
b t½ 2 t½ Half life period
x
z T = n × t1/2
Here, a b c; = = = 90º where n = number of half life period
49. (d) Gibbs Helmholtz Equation–
0. 5
G H–T S .......(1) Amount of R left = ;
differentiate this equation w.r.t. temperature ( 2) T / t ½
at constant pressure
0 .25
G Gy Gx Amount of S left
– ( 2)T / 2 t ½
T T T ....(2)
P P P T/ t1 / 2
0.5 ( 2)
= – Sy – ( – Sx) Equating both
0.25 T/ 2 t1 / 2
= – ( Sy – Sx) = S ......(3) ( 2)
where S change in entropy T /t
on combining equation (1) & (3) we get or 2 (2) 1/ 2

G T 2 t 1 / 2 . 2t1/2 is half life of S and


G H T ....(4) twice the half -life of R
T
P 4
Equation (4) is on alternative form of Gibbs 1.667 10
53. (c) 2.303 log 6
Helmholtz equation 1.667 10
Dividing equ. (4) by T2, we get
Ea 1 1
=–
R 1844 1000
www.jeeneetbooks.in

WWW.IIT-NEET.XYZ

Solutions MT-227

844 0.1M CH3 COOH is (weak acid) not same,


E therefore the cell voltage is not zero.
2.303 × 2 = a × .......(1)
R 1844 1000
0.0592 [QH 2 ]
4.606 1844 1000 57. (b) E Eº log
Ea 2 [Q][H ]2
=
R 844
E º 0.0592pH
K 0.699 0.0592 4 0.463V
2.303 log 6 58. (c) Ethers, when exposed to air in presence of
1.667 10
light, form ether peroxides. Cumene
Ea 1423 1000 (isopropylbenzene), when heated in
= × presence of oxygen also forms peroxide,
R 1423 1000
here oxidation occurs at benzylic carbon.
Ea 423 59. (d) CH 3 CH 2 CH CH CH 2 CH 3
= × .......(2)
R 1423 1000 | |
Dividing equation (2) by equation (1) OH OH
HIO4
K 2 CH 3 CH 2 CHO
log 6 60. (b) As far as characterstic of IR spectrum for
1.667 10
CH3CH2OH is
2 O–H streching frequency 3391 cm–1
423 1844 1000
O–H streching frequency 2981 cm–1
= 1423 1000 × O–H streching frequency 1055 cm–1
844 61. (d) Phenol has active (acidic) hydrogen so it
reacts with CH3MgI to give CH4, and not
K anisole
log 6
1.667 10 C 6 H 5 OH CH 3 MgI CH 4 C 6 H 5 OMgI
423 1844
= 2× = 1.299
1423 844
K = 19.9 × 1.667 × 10–6 62. (b)
= 3.318 × 10–5 s–1
54. (c)
55. (b) Here, R = 31.6 ohm
1 1
C= ohm 1 = 0.0316 ohm–1
R 31.6 OH OH
Specific conductance = conductance × cell 50%KOH CH2OH COOK
constant.
Cannizzaro +
= 0.0316 ohm–1 × 0.367 cm–1 reaction
= 0.0116 ohm–1 cm–1 63. (d) Completing the given equation
Now, molar concentration = 0.5M (given)
= 0.5 × 10–3 mole cm–3 C6 H5OH NaOH C6H5ONa
H2 O
'A '
k 0.0116
Molar conductance = OCOONa
C 0.5 10 3
= 23.2 s cm2 mol–1 CO 2
56. (d) For a concentration cell having different 140 C
concentrations of ions. 4 7 atm

0.0591 c OH
E= log 1
n c2 COONa
Rearrangement
If all the concentrations are identical then
obviously the cell voltage is zero. But as
the pH of 0.1 M HCl (strong acid) & pH of 'B'
EBD_7443
www.jeeneetbooks.in

WWW.IIT-NEET.XYZ

MT-228 Target VITEEE

OH 69. (d) Carboxylic acids are more stronger acids


than phenols due to greater resonance
COOH stablization of carboxylate ion than
+HCl
phenoxide ion. So we have to choose
NaCl
strongest acid from ortho, para or meta –
'C' nitrobenzoic acid. And nitrobenzoic acid is
‘C’ is 2-hydroxybenzoic acid (or salicylic strongest among three due to ortho–effect.
acid)
CH2Cl
Note: If the reaction is carried out at a higher
temperature p-hydroxybenzoic acid is
produced. 70. (c) on hydrolysis will not yield an
64. (a)
OMgBr acid.
| CH 2 COOH
CH3MgBr 71. (a) CH2CO
C6 H5 COOC2 H5 C6 H 5 C OC2 H5 |
heat (235°C)
O
| H2 O CH2CO
CH3 CH 2 COOH
Succinic acid Succinic
anhydride
O
|| 72. (c) -Keto carboxylic acids undergo
Mg(OC2 H5 )Br Excess decarboxylation very easily.
C6 H 5 C CH3
CH3MgBr H 2SO4
73. (b) HCOOH H 2O + CO
OMgBr CH3
H2 O | O H+ O H2O
C6 H5 C CH3 C6 H5 — C — CH3 H C H C
| | OH OH 2
CH3 OH
CH3
Con. H 2SO 4
|
Ozonolysis H C O CO H
C 6 H 5 — C CH 2
74. (a) Pd / BaSO 4
CH 3 COCl 2 H
'B'
C 6 H 5COCH 3 HCHO CH 3CHO HCl
3I 4 NaOH
(Rosenmund's reduction)
C 6 H 5 COCH 3 2 CHI 3 75. (b) This reaction is known as curtius
65. (d) I2 and Na2CO3 react with acetophenone rearrangement.
(C6H5COCH3) to give yellow ppt. of CHI3 N2 2NaOH
RCON3 RNCO
but benzophenone (C 6H5COC6H5 ) does
not and hence can be used to distinguish Na2CO3 + RNH2
between them. 1° amine is formed.
66. (a) 76. (a) NaNO2 with HCl at 0 °C gives nitrous acid
(HNO2) which on reaction with primary
aliphatic amines gives alcohol with
quantitative evolution of N2 gas. Since
(CH3)2CHNH2 is the only primary aliphatic
amine among the given options, hence it

OCH3 gives alcohol on reaction with NaNO2, HCl/
CH 3OH H2O at 0°C. The reactions are as follows:
NaNO2 + HCl HNO2 + NaCl
67. (a) (a) is th e product of cross aldol (CH3)2CHNH2 + HNO2 (CH3)2CHOH
Condensation + N2 + H2O
O 77. (c) (CH3 )2 NCOCH3 is dimethylacetamide,
hence on refluxing with acid, it undergoes
68. (a) R C H + NH2 NH2 R C N NH2 hydrolysis in the following way
H
Aldehyde Hydrazine Aldehyde hydrazone H ,reflux
(CH 3 ) 2 NCOCH 3 ( CH 3 ) 2 NH HOOCCH 3
www.jeeneetbooks.in

WWW.IIT-NEET.XYZ

Solutions MT-229

78. (b) (a) C 6 H 5COOC 2 H 5 + (CH 3 ) 2 NH


a11
C 6 H 5CON (CH 3 ) 2 + C 2 H 5OH
a 21
81. (c) Let A
(b) C 6 H 5CONH 2 + CH 3 Mg I ....
C 6 H 5CO NH MgI + CH 4 a m1

(c) C 6 H 5COCl + (CH 3 ) 2 NH and B [b11 b12 b13 .....b1n ]


B two non-zero column and row matrices
C 6 H 5CON (CH 3 ) 2 + HCl
respectively.
(d) C 6 H 5CO.O.COC 6 H 5 + (CH 3 ) 2 NH a 11b11 a 11b12 ... a11b1n
C 6 H 5CON (CH 3 ) 2 + C 6 H 5COOH a 21b11 a 21b12 ... a 21b1n
AB
79. (c) Urea decompose HNO2 to N2, CO2 and H2O ... ... ... ...
NH2CONH2 + 2HONO a m1b11 a m1b12 ... a m1b1n
2N2 + CO2+ 3H2O Since A, B are non-zero matrices, matrix
80. (b) Let X be the nitrogen containing AB will be a non zero matrix. The matrix AB
compound. As we get an oily liquid on will have atleast one non-zero element
treating X with Br2 and KOH, it indicates obtained by multiplying corresponding
toward Hoffman's Bromamide degradation non-zero elements of A and B . All the two-
in which a carbon atom is degraded from rowed minors of AB clearly vanish.
acid amide and amine is formed as product. rank of AB = 1
All this information indicates that the 82. (a) The system is 0x1 + x2 – x3 = 1
starting compound must be one from – x1 + 0x2 + 2x3 = 2
x1 – 2x2 + 0x3 = 3
benzamide and acetamide.
0 1 1 x1 1
CONH2 NH2
1 0 2 x2 2 or AX B
....(i) 1 2 0 x3 3
+ Br2 + KOH
Clearly | A | = 0
Aniline Now Adj A = 4 2 2
CH3CONH2 + Br2 + KOH ¾¾
® 2 1 1
CH 3 NH 2 .....(ii) 2 1 1
Methyl amine (Adj A) B 0 system is inconsistent
When benzamide is treated with (CH3CO)2O 83. (c) In all the given equations, the origin is
an antipyretic, paracetamol is formed which present in shaded area, answer (c) satisfies
confirms that starting compound is this condition.
benzamide. 84. (a) B C
NH2 NH2 a +b
b
(CH3CO)2O
a –b
O a A
COCH3
Paracetamol Let OA a and OB b . Complete the
As an oily product is formed when parallelogram OACB.
benzamide is treated with Br2 and KOH, it
clearly indicated that final product is aniline a b OA OB OC |a b | OC
which is oily in nature. Hence starting Again
compound is benzamide.
a b OA OB BA |a b | BA
EBD_7443
www.jeeneetbooks.in

WWW.IIT-NEET.XYZ

MT-230 Target VITEEE

Given | a b | |a b | OC BA 1 1 1 1
sin cos sec 1 (2) cosec 1
2
Diagonals of the parallelogram OACB 5 5
are equal.
OACB is a rectangle. 1 1 1 1
2tan 2cot k
3 3
a and b are adjacent sides of a
rectangle. 1 1
85. (b) Clearly b is H.M. of a and c or 2 tan 1 cot 1 k
2 2 3 3
Also the G.M. of a and c is ac
Since or 2 k
2
2
G.M. H.M ac b b ac .......(i ) or + =k
Again, for the positive numbers an, cn or 2 =k
or k=2
an cn
G.M. a n c n and A.M. 88. (a) Since are coplanar vectors,
2 , ,
therefore
an cn
Since A.M. > G.M. a n cn a 1 1
2
[ ] 0 1 b 1 0
an cn n n
ac b (Using (i)) 1 1 c
2
an cn 2b n a (bc 1) 1(c 1) 1(1 b) 0
abc a c 1 1 b 0 ...(1)
2 sin
86. (b) We have, = y abc (a b c) 2
1 cos sin
1 1 1
Now,
4 sin cos 1 a 1 b 1 c
2 2
Then, =y (1 b)(1 c) (1 a)(1 c) (1 a)(1 b)
2 cos 2 2 sin cos
2 2 2 (1 a)(1 b)(1 c)
3 2(a b c) (ab bc ca )
2 sin sin cos
2 2 2 1 (a b c) (ab bc ca ) (abc)
=y
cos sin sin cos 3 2(a b c) (ab bc ca )
2 2 2 2 1 (a b c) (ab bc ca ) (a b c) 2
1 – cos sin [From (1)]
= y..
1 sin 3 2(a b c) (ab bc ca )
1.
Trick: Put value of = 30° and check. 3 2(a b c) (ab bc ca )
87. (b) The given question can be written as 89. (a) Let the equation of the required plane be
x y z
1 ...(i)
1 1 1 a b c
sin sec 1(5) sec 1(2) sin 1
5 2 It meets co-ordinate axes in points
A (a, 0, 0), B(0, b, 0), C(0, 0, c).
1 1
2 tan 2 tan 1 ( 3) k a b c
3 The centroid of ABC is , ,
3 3 3
or
a b c
, ,
3 3 3
www.jeeneetbooks.in

WWW.IIT-NEET.XYZ

Solutions MT-231

Equating real and imaginary parts,


a 3 ,b 3 ,c 3
Hence the required plane is a 3
1 a 2 3
2 2
x y z x y z
1 i.e., 3.
3 3 3 1 3
a b b 2 3
90. (b) Since the sphere is inscribed in the cube 2 2
whose faces are x = 0, x = 2a, y = 0, y = 2a, 92. (b) The general points on the given lines are
z = 0 and z = 2a, therefore, the centre of the respectively P(5 3t , 7 t , 2 t ) and
sphere is the centre of the cube i.e., mid-
point of any of its diagonals and radius is Q( 3 3s, 3 2s, 6 4s) .
half the distance between the parallel faces. Direction ratios of PQ are
Hence the radius of the sphere is 3 3s 5 3t, 3 2s 7 t, 6 4s 2 t
1 i.e., 8 3s 3t, 4 2s t, 8 4s t
(2a ) a and the centre is mid-point of
2 If PQ is the desired line then direction ratios
the segment joining (0, 0, 0) and (2a, 2a, 2a) of PQ should be proportional to < 2, 7, –5>,
which are the corners of a diagonal. therefore,
So the centre is
8 3s 3t 4 2s t 8 4s t
0 2a 0 2a 0 2a 2 7 5
, , (a , a , a ) .
2 2 2 Taking first and second numbers, we get
Hence, the equation of the sphere is 56 21s 21t 8 4s 2t
25s 23t 48 … (i)
(x a)2 (y a)2 (z a ) 2 a2
Taking second and third members, we get
x2 y2 z2 2ax 2ay 2az 2a 2 0 20 10s 5t 56 28s 7 t
91. (a) 38s 2t 36 … (ii)
Solving (i) and (ii) for t and s, we get
s = –1 and t = –1.
The coordinates of P and Q are respectively
(5 3( 1), 7 ( 1), 2 1) (2, 8, 3)
and
( 3 3( 1), 3 2( 1), 6 4( 1)) (0, 1, 2)
The required line intersects the given
lines in the points (2, 8, –3) and (0, 1, 2)
respectively.
Length of the line intercepted between the
z2 0 given lines
Clearly from the figure
z1 0 = | PQ | = ( 0 2) 2 (1 8) 2 ( 2 3) 2 78 .
93. (d) We have
= cos i sin
3 3 4 2 (cos x sin x)5
lim
1 3 x 1 sin 2x
z2 z1 i 4
2 2 5 5
2 2 [(cos x sin x) 2 ]2
1 i 3 = lim
or, 1 bi ( a i) x 2 (1 sin 2x)
4
2
5 5
(1 sin 2x) 2 2 2
=
a 3
i
1 3
a = lim
x (1 sin 2x) 2
2 2 2 2 4
EBD_7443
www.jeeneetbooks.in

WWW.IIT-NEET.XYZ

MT-232 Target VITEEE


5 5 97. (b) It is given that
t 2 22 cos 1
x cos 1
y cos 1
z
= lim , where t = 1 + sin 2x
t 2 t 2
1 1 1
5 cos x cos y cos z
5 1
= (2) 2 5 2
2 i.e cos 1[xy 1 x 2 1 y2 ]
94. (c) | z | | z || | | z || | 1
cos 1 ( z)
Arg(z ) arg(z) arg( ) 1 1
2 [Since cos ( z) cos z]
arg(z) arg i.e xy 1 x2 1 y2 z
z 1
95. (b) Let the point A represents the complex i.e xy z 1 x 2 1 y2
number z, B represents z and C represents
Squaring both sides, we get
z.
& are complex cube roots of unity x 2 y2 z2 2 xyz

clearly z means rotation of z by


2
and (1 x 2 )(1 y 2 ) 1 x 2 y2 x 2 y2
3
x 2 y 2 z 2 2 xyz 1
2
2
z ( z) means rotation of z by . 98. (b) The number of choices available to him
3 = 5C4 × 8C6 + 5C5 × 8C5
y 5! 8! 5! 8!
B (w, z) =
4! 1! 6! 2! 5! 0! 5! 3!
A (z)
7 8 8 7 6
x
=5× +1×
O 2 3 2
D =5×4×7+8×7
= 140 + 56 = 196
99. (a) a = 0, b = 1
C (w, z)
a + b = Max (0, 1) = 1
2 100. (a) The equation of axis of the parabola is
AOB = BOC =
also COA = x – 4 = 0 which is parallel to y-axis. So the
3 ray of light is parallel to the axis of the
OA = OB = OC = |z|. That is the ABC is parabola. We know that any ray parallel to
equilateral. the axis of a parabola passes through the
focus after reflection.
3
Now AC = 2AD = 2 (OA cos 30°) = 2 |z| The ray must pass through the point
2 (4, –1).
= 3 |z| 101. (a) The equation of a tangent to the given circle
is
3 3 3 2
Area of ABC = (side)2 = |z| y mx a 2 m2 b2
2 2
96. (a) From sin x + sin2 x = 1, we get y mx a 2 m 2 b2 ...(i)
sin x = cos2 x ...(1)
Now the given expression is equal to Equation of a line perpendicular to the
above tangent and passing through any of
cos6 x (cos6 x + 3cos4 x + 3 cos2 x + 1) – 1
the foci (±ae, 0) is
= cos6 x (cos2 x + 1)3 –1
my + x = ± ae ...(ii)
= sin 3 x (sin x + 1)3 –1 [From (1)] The locus of point of intersection of the
= (sin 2 x + sin x)3 – 1 = 1 – 1 = 0 lines represented by equations (i) and (ii)
www.jeeneetbooks.in

WWW.IIT-NEET.XYZ

Solutions MT-233

can be found by eliminating m from the two


equations.
104. (d) f (x) x100 sin x 1.
Squaring and adding (i) and (ii), we have f '(x) 100x 99 cos x
(1 m 2 )y2 (1 m 2 )x 2 If 0 < x < /2, then f’(x) > 0,
therefore f(x) is increasing on (0, /2).
a 2 m2 b2 a 2 e2 If 0 < x < 1, then 100x99 > 0 and cos x > 0
a 2 m 2 a 2 (1 e 2 ) a 2 e 2 a 2 (1 m 2 ) [ x lies between 0 and 1 radian]
f’(x) = 100x99 + cos x > 0
x 2 y 2 a 2 , which is the equation of f(x) is increasing on (0, 1).
the desired locus. If /2 < x < , then 100 x99 > 100
[ x > 1, x99 > 1]
x2 y2 99
100x + cos x > 0
102. (a) Let eq. of hyperbola be 1 centre
a 2 b2 [ cos x 1, 100x 99 cos x 99]
is (0, 0), vertex (a, 0), focus (ae, 0). f’(x) > 0 f(x) is increasing on ( /2, ).
1 2 105. (a) 4x3 + 4y3 (dy/dx) = 0 dy/dx = –x3/y3
e = eccentricity = a b 2 . It is given Equation of tangent, Y – y = – x3/y3 (X–x)
a
y3 Y + x 3 X = x 4 + y4 = a 4
2ae X Y
that a 1
3 4 3
a /x a / y3
4
4 2 2 4 2
a2 a e (a b2 ) Here, p = a4/x3, q = a4/y3
9 9
a 16 / 3 a 16 / 3
2 2 2 2 5 2 p 4/3 q 4/3
9a 4a 4b b a x 4 y 4
4
16 / 3
x2 y2 a x4 y4
Eq. of hyperbola is 1
a2 5a 2
a 16 / 3 (a 4 ) a 4 / 3
4
106. (b) Since x2 – 5x + 4 = (x – 4) (x – 1), the
5x 2 4y 2 5a 2 function f is defined for all real numbers
103. (b) Perimeter = 440 ft. except x = 4 and x = 1. Hence, the domain
2x + r+ r = 440 of f is R – {1, 4}.
or 2x + 2 r = 440 ..........(1)
A = Area of rectangular portion = x. 2r 107. (b) After dividing by cos2 x to numerator and
denominator of integration
(400 2x) 1 sec 2 x dx
A= x (440 x – 2x2) I
4 tan 2 x 4 tan x 5

D x C sec2 x dx
(2 tan x 1)2 4
r r
1 2 tan x 1
A tan 1 C
x B 22 2
108. (a) The given function is f(x) = x3 + 3x – 5
dA 1 d 2A The first derivative is f '(x) = 3x2 + 3
(440 – 4x) x = 110 ; = – ve The second derivative is f"(x) = 6x .
dx dx 2
A is max. when x = 110. The function is concave upward when
f"(x) > 0 , that is, when x > 0.
440 2x 440 220 The function is concave downward when
2r = = = 70 ft
22 / 7 f"(x) < 0 , that is, when x < 0.
EBD_7443
www.jeeneetbooks.in

WWW.IIT-NEET.XYZ

MT-234 Target VITEEE


At point of inflection, f"(x) = 0 , that is, x = 0.
1; x 1
At x = 0, f(x) = – 5
2
Hence, f is concave upward in the interval 3x ; 1 x 0
(0, ), concave downward in the interval f ' (x) =
1; 0 x 1
(– , 0) a nd (0, –5) is the point of inflection.
2
/2
3x ; x 1
(x)
109. (a) Let I dx then Clearly f is not differentiable at – 1, 0 and 1.
0 (x) x 113. (c) The given equation can be rewritten as
2
y = A cos (x + B) – Cex,
/2 x where A = c1 + c2, B = c3, C = c4e c5
2 As the minimum number of parameters is 3,
I dx
0 x (x) order of the differential equation = 3.
2 114. (b) The differential equation is
Adding
d2y
1 sin x ....(i)
(x) x dx
/2
2
2I dx dy
Integrating we get x cos x c ....(ii)
0 x (x) dx
2
dy
/2 When x 0, 0 c 1
/2 dx
1.dx x 0 = /2
0 dy
Equation (ii) is x cos x 1
I = /4 dx
110. (a) Bounded figure ABCD is a rectangle. Integrating again we get
AB 1 1 2 , BC 4 4 2 2 x2
y sin x x D ...(iii)
2
When x 0, y 0 D 0
A(1, 1) The particular solution is
B y = |x| –2 x2
y x sin x
–2 –1 0 1 2 y = 1 – |x –1|
2
115. (b)
D
116. (a) 8n 7n 1 (1 7) n 7 n 1
–2 C
Which is a multiple of 49 of n 2
Thus, bounded area = ( 2) (2 2) = 4 sq. For n 1, 8n 7n 1 8 7 1 0
units.
111. (a) Given, A = {x : |x| < 3, x I} For n 2, 8 n 7n 1 64 14 1 49,
A = {x : –3 < x < 3, x I} = {–2, –1, 0, 1, 2} a multiple of 49. So, 8n 7n 1 is a multiple
Also, R = {(x, y) : y = |x|}
of 49 for all n N .
R = {(–2, 2), (–1, 1), (0, 0), (1, 1), (2, 2)}
Hence X contains all elements which are
112. (d) f (x) = max. {x, x3}
multiples of 49 for n N . Also Y contains all
x ; x 1 elements which are multiples of 49.
3 X Y
x ; 1 x 0
= Note that Y contains all multiples of 49 but X
x ; 0 x 1 does not contain all multiples of 49 but some,
3 i.e. Y / X .
x ; x 1
www.jeeneetbooks.in

WWW.IIT-NEET.XYZ

Solutions MT-235

117. (a) Let us first consider 2 letters and 2 119. (c) Let n be the number of tosses and X the
envelopes, then there is only one way to number of times heads occurs. Then X ~ B
place both the letters in wrong envelope. (n, p), with p = 1/2. Since P(X = 7) = P(X = 9),
Next, we consider 3 letters and 3 directed we have
envelopes. 7 n 7 9 n 9
The number of ways of putting all letters in n 1 1 n 1 1
C7 C9
wrong envelopes 2 2 2 2
= Total number of possible arrangements –
Number of ways in which all letters are in n n
n 1 n 1
correct envelopes C7 C9
2 2
– Number of ways in which 1 letter in
correct envelope That is, nC7 = nC9 = nCn–9, yielding 7 = n – 9
= 3 ! – 1 – 3C1 × 1 = 2 or n = 16. Hence
[ The case of two letters in correct 1
16
16 15 1
16
15
envelope and one in wrong envelope is not P( X 2) 16 C 2
2 2 2 213
possible]
120. (a) Let E1 be the event that exactly two players
Further, we consider 4 letters and 4 directed scored more than 50 runs then
envelopes.
The number of ways of putting all letters in 1 1 3 9
P(E1) =
wrong envelopes 2 3 4 10
= Total number of possible arrangements –
1 2 1 9 1 2 3 1
number of ways in which all letters are in +
correct envelope – Number of ways in 2 3 4 10 2 3 4 10
which 1 letter is in correct envelopes (3 in 1 1 1 9 1 1 3 1
wrong envelope)
2 3 4 10 2 3 4 10
– Number of ways in which 2 letters are in
correct envelope (2 in wrong envelope) 1 2 1 1 65
= 4 ! – 1 – 4C1 × 1 = 9. 2 3 4 10 240
NOTE : Such problems are called problems Let E2 be the event that A and B scored
of deragement. Hence, using the formula more than 50 runs, then
of deragement.
The required number of ways of placing all 1 1 3 9 27
P(E1 E2 )
letters in wrong envelope 2 3 4 10 240
Desired probability
1 1 1 1
= 4! 1 1! 2! 2! 4!
P ( E1 E 2 ) 27
= P ( E 2 / E1 )
P ( E1 ) 65
4! 4! 4! 121 (c)
= 12 4 1 9
2 ! 3! 4 ! 122 (d)
118. (c) We are given that e–m . m = 0.3 and 123.(c)
124. (b) Pertinacious means holding firmly to an
m
e m2 opinion or a course of action. Insipid means
= 0.2 lacking vigour or interest.
2!
125 (d)
4
Dividing, m 0.4 0.3
3
P (X = 0) = e–m = e–4/3
EBD_7443
www.jeeneetbooks.in

WWW.IIT-NEET.XYZ

MT-236 Target VITEEE

MOCK TEST 10

t d t 5. (c) E , where is the linear charge


1. (b) V 4 2 0r
k1 k2
density on the inner cylinder.
4 Q t d t b
V b
A k1 k2 and V E.d ln ....... (1)
a
2 0r a
2. (a) As u2 = 0 and m1 = m2, therefore from
m1 u1 + m2 u2 = m1 v1 + m2 v2 we get u1 = Now,
v1 + v2
I J.dA E.dA .2 dr
Also, 2 0r
v2 v1 v2 v1 1 v1 / v 2 Current per unit length will be
e ,
u1 v2 v1 1 v1 / v 2
I ....... (2)
v 1 e 0
which gives 1
v2 1 e 2 0 2
From (1) : I v
0 ln (b / a) ln (b / a)
q 1 1 1
3. (d) V 1 ...... 6. (b) V = E – ir
4 0 2 4 8
Er
V
(R r)
q 1 V = 0 at R = 0
V
4 0
1 R= , V=E
1
2 7. (c)
f R0
1
V (2q) f R0(1–f)
4 0
r
q 1 1 A
E 1 ......
4 0 4 16 A
r
q 1 (1–f)R0
E
4 0 1
1 E
4 i
r f R 0 (1 f )
1 4q i max if f R0 (1 – f) = 0 f = 0 or 1
E
4 0 3 8. (a) Power maximum
4. (c) Let K be the dielectric constant of solid when r = R.
capacitor and if C0 is capacitance of air So, power consumed by it
capacitor then solid capacitor will have P
capacitance KC0.
After charge sharing the common potential
becomes V.
C1V1 C2 V2
V
C1 C2
R
CV0 KC(0) V0 V r=R
V K
C KC V will decrease for R > r.
www.jeeneetbooks.in

WWW.IIT-NEET.XYZ

Solutions MT-237

9. (a) In parallel combination, total power So the emf induced between its tips,
P = P1 + P2. e = BV v
But as by definition of angle of dip,
0
10. (b) v BV
tan i.e., BV BH tan
BH
v2 02 1 So,
v1 01 2 e (BH tan )v 2 10 5
3 250 20
4
7.8 –1.2 8.5 10 10 i.e., e ( 3) 10 V 1
0.173V
=
7.8 –1 13.2 16. (c)
= 6.25 ×10–4 cms–1 17. (d)
0 I 18. (a) 2 nt 400 t n = 200
11. (b) B [sin 90 sin( )]
4 a I0 50 2 amp.
I
= 0 (1 sin ) r.m.s. current = I0 / 2 = 50 amp.
4 a
19. (d) Here, X L L 2 fL
0.4
2 50 40 R = 30
I
Z R2 X L2 302 40 2 50
A
(0, b) Vrms 200
I rms 4A
Z 50
20. (a) VL and VC will be in opposite phase so they
(0,0) will cancel each other being equal in
(a,0) magnitude.
Resultant potential difference = applied
0 I b
p.d. = 100 volts
= 1
4 a a 2
b2 Hence, Z = R
Vrms 100
0i I rms 2 amp.
12. (b) B (sin 1 sin 2 ) Z 50
4 d
1 2
10 –7 100 3 1 21. (d) 0E 0 is electric energy density..
2
= = 5 × 10–6 T
3 1 2 2
B2
13. (d) 2 0
is magnetic energy density..
FG dB IJ B20
14. (a) E = – NA H dt K So, total energy =
1
2
2
0E 0
2 0
E NA d tD
i= = (75e –200t) × 10–4 22. (d) Shift of fringe pattern = ( 1)
R R dt d
NA 10
= [75 × (–200)e–200t] × 10–4 30D (4800 10 ) D
R (0.6)t
d d
50 7 10 8
=+ [15000e–1] 30 4800 10 10
0.6t
30
10 5
175 10 5 175 10 5 30 4800 10 1.44 10
= = t
e 2.73 0.6 0.6
= 0.64 × 10–3 A = 0.64 mA 24 10 6
15. (b) As the plane is flying horizontally it will cut 23. (b) 24. (d)
the vertical component of earth’s field BV .
EBD_7443
www.jeeneetbooks.in

WWW.IIT-NEET.XYZ

MT-238 Target VITEEE


25. (a) Energy of photon corresponding to first line dN
33. (c) n N
2 1 1 dt
of Balmer series = (13.6) (2)
4 9 dN (n N)dt
Energy need to eject electron from n = 2 N t N
level in H atom dN 1 dN
dt t
1 4 1 n N n N
N0 0 N0
= (13.6) = (13.6) 1
9 4
eV
4 1 N
log e (n N) N0 t
11
= 13.6 4.155 eV
36 1 n N
26. (a) According to relation, log e t
n N0
Mv2 Mv
Bqv r n N0
r Bq t log e
n N
m2v
r B 2B' 2x n N0
B'q t
e
27. (a) 28. (d) n N
t
hc hc n N (n N0 ) e
29. (d) eV0 W0 and eV W0
0 2 0 n n t
N0 e N
Subtracting them, we have
hc 1 hc 34. (b) Change in momentum = F × t
e ( V0 V) 1 = 10 × 10 = 100 Ns or 100 kg m/s
0 2 2 0
35. (c)
hc 36. (a) Voltage across the collector load resistance
or V V0 (RC) = 6V, = 0.97; RC = 3 k .
2e 0 The voltage across the collector resistance
30. (b) is,
31. (d) Balancing vertical forces, we have RC = ICRC = 6V
N Fsin 60 mg 6 6
Hence, IC = R 2mA
For the block not to move, we must have C 3 103
Fcos 60 N 0.97
Current gain 32.33
i.e., Fcos 60 (Fsin 60 mg) 1 1 0.97

1 1 3 2 10 3
IC
F F (2 3)(10)N IB 61.86 A
32.33
2 2 3 2
37. (a) Rf = 740 k , R'f = 650 k , R1 = 10 k ,
1 1 R'1 = 100 k .
or F 10N Let V1 be the output voltage of the first
2 4 amplifier
or F 4 10N 40N
32. (d) According to principle of continuity, for a Rf 740 103
V1 1 V 1 Vin
streamline flow of fluid through a tube of R1 in 10 103
non-uniform cross-section the rate of flow 75Vin
of fluid (Q) is same at every point in the
tube. Rf 650 103
i.e., Av = constant A1v1 = A2v2 V0 V1 75Vin
R in 100 103
Therefore, the rate of flow of fluid is same
at M and N. 487.5 Vin
www.jeeneetbooks.in

WWW.IIT-NEET.XYZ

Solutions MT-239

4.4 10 28 H CH3
38. (a) Na 8
1.1 10 20 atom / m3 , | |
4 10 CH3CH 2 – C O H H C CHO
propanol
ni2 6.25 1038 ,
CH3 CH3
pp Na 1.1 1020 atom / m3 | |
OH
CH3CH 2 C CHCHO
ni2 6.25 1038 |
np 20 OH
Na 1.1 10 3.hydroxy,2 methyl pentanal
18
5.68 10 atom / m 3 44. (d) In body centred cubic lattice one molecule
of CsBr is within one unit cell.
np 5.68 1018 Atomic mass of unit cell
0.22 = 133 + 80 = 213 a.m.u
ni 2.5 1020
Volume of cell = (436.6 × 10–10)3
39. (a) Current I = 4.8 mA = 4.8 × 10–3 A
Width b = 4 × 10–3 m, Thickness n ´ at.wt.
Density =
t = 25 × 10–5 m Av.no.´ vol.of unit cell
Length = 6 × 10–2 m 213
Free electron density n = 1022/m3. Density =
The current density J = I/A 6.02 10 23 (436.6 10 10 )3
Area = b × t = 4 × 10–3 × 25 × 10–5 = 10–6 m2 For body centered cubic crystal n = 2
I 4.8 10 3 ZM
So, current density J
A 10 6 a3NA
= 4.8 × 10 A/m2.
3
Suppose the drift velocity is vd then 2 213 107
I = neAvd. 8.50 gm / cm3
6.02 (436.6)3
3
4.8 10 45. (c)
vd 3 m / sec
10 22 1.6 10 19 10 6 46. (d) H3 C
Now the time taken by the electron to travel
C = O + CH 3 MgI
distance
full length of the sample t H3 C
speed vd
6 10 2 H3 C
= 2 10 2 sec H OH H2 O
3 C O MgI
40. (d) H3 C
41. (a) The equivalent conductance of strong CH3
electrolyte little increases with dilution.
42. (a) Normally NaBH4 as well as LiBH4 reduce
only–CHO group without effecting carbon- CH3
|
carbon double bond, however when it is C C OH Mg(OH)I
present in conjugation with benzene ring |
and aldehydic group it is also reduced CH3
along with the reduction of – CHO group. Tert butyl alcohol
LiAlH 4
C6 H5CH CH CHO C6 H5CH 2CH 2CH 2 OH
NA NB
C6 H5CH CH CHO C6 H5CH 2 CH 2CH 2OH 47. (c) At equilibrium
43. (d) Aldehydes which contain a -hydrogen t1 / 2 of A t1 / 2 of B
on a saturated carbon, i.e., CH3CH2CHO
N0 N0
undergo aldol condensation. 1 x
232 228
1.4 1010 7
EBD_7443
www.jeeneetbooks.in

WWW.IIT-NEET.XYZ

MT-240 Target VITEEE

where N0 Avogadro no. 55. (b) After every 30 minutes the amount is
and x g is wt. of radium 1
reduced to and t1/2 is independent of
on solving x = 5 × 10–10g 2
48. (b) As volume is constant hence work done in initial concentration for 1st order, hence
this proces is zero hence heat supplied is reaction is 1 order.
equal to change in internal energy. 56. (b)
49. (a) Nitroethane on reduction in acidic medium, AgNO 2 PCl 5
57. (c) C 2 H 5 NO 2 C 2 H 5 OH
like Sn + HCl, Fe + CH3COOH or H2 – Ni B
gives.
Primary amine. C 2 H5C l POCl3 HCl
[ A]
Sn HCl
CH 3CH 2 NO 2 6H CH 3CH 2 NH58.
2 2H(d)2 O Since the compound (C4H10O) react with
Sn HCl sodium, it must be alcohol (option b, c, or d).
CH3CH2 NO2 6H CH3CH2 NH2 2H2O
whereas alkyl nitrite on reduction in neutral As it is oxidised to carbonyl compound
medium gives alcohol. which does not reduce Tollen’s reagent, the
carbonyl compound should be a ketone and
Area Concentration thus C 4 H10 O should be a secondary
50. (a) Conductivity
length alcohol, i.e., sec-butyl alcohol; other two
given alcohols are 1º.
k Area Concentration 59. (c) The statement (c) is incorrect.
length 60. (d) The two components should be
(CH3)3CONa + (CH3)3CBr. However, tert-
Conductivity length
k alkyl halides tend to undergo elimination
Area concentration reaction rather than substitution leading to
Sm 1
m the formation of an alkene, Me2C = CH2
= S m mol–1 61. (a) Electronic configuration of Cr is
m 2 mol m 3 3d 4s
51. (a) For spontaneous reaction, dS > 0 and G
and dG should be negative, i.e. < 0.
52. (a) CuSO 4 Ca(OH) 2 is Bordeaux mixture. So, due to half filled orbital I.P. is high of Cr.
6 62. (d) The reaction is known as Gatterman
K 1000 3.06 10 1000 aldehyde reaction.
53. (d) Solubility
eq 1.53 63. (a) First option is incorrect as the value of KP
= 2 × 10–3 given is wrong. It should have been
Ksp = S2 = 4 × 10–6 PCO 2
2.303 N KP
54. (d) t log 0
PCH 4 [PO2 ]2
Nt
64. (d) Acetone forms mesitylene (1,3,5-trimethyl
0.693 N0 benzene) on distillation with conc. H2SO4.
where log
t1 2 Nt 65. (b) [Cr(H2O)6]2+ Cr is in Cr2+ form
where N0 = original amount
Nt = amount left after time t Cr 2
24 3d 4s
2.303 U 238 Pb206
t log
U 238 Fe 3
26 3d 4s
2.303 Pb206 In [Fe(H2O)]2+ Fe2+ form. Both will have 4
or t log 1
U 238 unpaired electrons.
66. (b) Like Clemmensen reduction, Wolf-Kishner
Given Pb206 : U238 = 1
reduction involves reduction of >C=O to
2.303 >CH2 , of course by different reagent.
4.5 109 log 2 4.5 10 9 years
0.693
www.jeeneetbooks.in

WWW.IIT-NEET.XYZ

Solutions MT-241

7 exhibit geometrical and linkage isomerism.


6 10 2
67. (b) [H+] = = 1.2 10 M 75. (a) 90 Th 232 6 208 4 208
0.05 10 3 78 X 82 Pb
76. (d) Os shows + 8 oxidation state.
[H ] 77. (b) It is an example of concentration cell, Ecell
Rate of disappearance of H+ r = ;
t cannot be zero since [H+] are different (HCl
is strong and CH3COOH weak acid).
2 78. (c) Since, compound A (C3H6O) undergoes
[H ] 1.2 10 M
t = = iodoform test, it must be CH3 COCH3
r 6 105 M s 1 (propanone). Further the compound B
T = 2 × 10–8 s obtained from A has three times the number
68. (b) Acidity increases with increases with of carbon atoms in A (propanone), B must
increasing electronegativity of elements. be phorone, i.e., 2, 6-dimethyl 2, 5- heptadien
Fluorine is highly electronegative so -4- one.
produces weaker + I effect and stabilize the
carboxylate ion. So, (d) is the strongest acid. (CH 3 ) 2 C O H 3 CCOCH 3 O C(CH 3 ) 2
A , propanone ( 3 molecules )
Order of electronegativity is F Cl Br .
Bromoacetic acid is the weakest acid. HCl
(CH3 )2 C CHCOCH C(CH3 )2
2, 6 dimethyl 2,5 heptadiene 4 one
69. (d) Magetic moment n n 2 where
79. (b) Since, 1º alcohol is more reactive than 2º
n = number of unpaired electrons
alcohol toward HI, latter can react only
15 nn 2 when both 1º alcoholic groups have
n=3 reacted.
70. (b) 80. (b) For the reaction
2 ZnS 2 Zn + S2 ; G1º = 293 kJ ..........(1)
C N H3C – C = NH 2 Zn + O2 ZnO ; G2º = – 480 kJ ..........(2)
S2 + 2 O2 2 SO2 ; G3º = –544 kJ .........(3)
Gº for the reaction
CH3 MgX
2 ZnS + 3 O2 2 ZnO + 2 SO2 can be
OCH3 OCH3 obtained by adding eqn. (1), (2) and (3)
Q Gº = 293 – 480 – 544 = – 731 kJ
H3C – C = O 81. (b) If z = rei = r (cos + i sin )
Then iz = ir (cos + i sin ) = – r sin + ir
cos
H 3O
or eiz = e(–r sin +ir cos )) = e–r sin ei r cos
or |ei z| = |e– r sin | | er i cos |
P = e–r sin [cos2 (r cos ) + sin2 (r cos )]1/2
= e–r sin
O 82. (b) Given equation of line is
||
71. (b) + CH3 – C– Cl Anhyd.AlCl3 x 3 y 3 z
Benzene
Acetyl chloride 2 1 1
DR’s of the given line are 2, 1, 1
COCH3 + HCl 2 1 1
DC’s of the given line are , ,
Acetophenone 6 6 6
This reaction is known as Friedel Craft’s
acetycation. Since, required lines make an angle with
72. (d) C6 H 5 NH 2 COCl 2 KOH C 6 H 5 NH.COCl HCl 3
the given line
C6 H5 NH 2 COCl2 KOH C6 H5 NH.COCl HCl The DC’s of the required lines are
C 6 H 5 NCO HCl 1 2 1 1 1 2
73. (c) AgBr exhibit Frenkel defect. , , and , ,
6 6 6 6 6 6
74. (d) The complex ion [Cr(SCN)2(NH3)4]2+ can
respectively.
EBD_7443
www.jeeneetbooks.in

WWW.IIT-NEET.XYZ

MT-242 Target VITEEE


Also, both the required lines pass through 87. (a) (A + B) (A – B) = A2 – B2
the origin. By matrix distribution law,
Equation of required lines are A2 – AB + BA – B2 = A2 – B2
x y z x y z BA – AB = 0 BA = AB
and
1 2 1 1 1 2 88. (c) It is a known fact that (AB) BA
1 i 1 i 89. (c) Given expression
83. (a) A + iB = A – iB =
1 i 1 i ( sin ) (sec ) ( tan )
1
(1 i ) (1 i ) (sin )( sec ) tan
(A iB) (A iB) 1 90. (a) Since A, B, C are in A. P.
(1 i ) (1 i )
A + C = 2B A + B + C = 3B
A2 + B2 =1 180° = 3B B = 60°
84. (b) By definition only f (x) = x2 + 4x – 5 with A + C = 120°
domain [0, ) is one to one.
Now sin B : sin C = 3: 2
a
85. (d) Given : 51 x 51 x , ,52x 5 2x are in A.P.. [ b: c= 3: 2]
2
We know that if a, b, c are in A.P. then 2b = 2 2 3 1
a+c sin C sin 60 .
3 3 2 2
a 1 x 1 x 2x 2x C = 45° A = 120° – 45° = 75°
2. 5 5 5 5
2
91. (d) The expression cos 1 ( ( 3 / 2)) can be
a = 5.5 x x 1 x 2 x 2
5(5 ) (5 ) (5 ) interpreted as the angle whose cosine is
Let 5x = t
( 3 / 2) .
5 2 1
a = 5t t We know that y 5 /6.
t t2
1 4 1 1
2 1 1 92. (c) 1 tan tan
a= t 5 t 3 2 2
t2 t
2 1 8 2 3
1 1 tan
a= t 2 5 t 6 2 4 2
t t
4 1
2 sin 1 sin 1
1 2 5 2 3
Put t A
t
a = A2 + 5A – 2 1
2
2
b 93. (a) If the line y = x + touches the ellipse
[ add & subtract ] 9x2 + 16y2 = 144, then c2 = a2m2 + b2
2a 2 = 16 × 12 + 9
2 2 2 = 25, =±5
a = A2 5 5
5A 2
2 2 94. (b) We have t 4 4t 2 1
4 16 4
5 17
2
17 t2 2 5
a= A a . 2
2 4 4
t2 > 0 t2 2 5
86. (a) After leaving A and I, we are remained with
4 different letters which are to be used for 95. (a) Equation of the tangent at /4 is
forming 3-letter words. Hence the required x y
number + – 2 =0 .......(1)
a b
= 4P3 = 4.3.2 = 24 Equation of the normal at /4 is
www.jeeneetbooks.in

WWW.IIT-NEET.XYZ

Solutions MT-243

x y a b 20 12p 8
– = – ........(2) 13 13
b a b 2 a 2
p1 = length of the perpendicular from the Taking +ve sign, we get
centre to the tangent 20 12p 8
p=1
2 13 13
2 ab Taking –ve sign, we get
= 1 1 =
a 2 b2 20 12p 8 28 7
a 2 b2 p
13 13 12 3
p2 = length of the perpendicular from the 99. (a) Given, f (x) < 0 and g (x) > 0 therefore g (x)
centre to the normal is an increasing function and f (x) is a
a b decreasing function
b 2 a 2 a2 b2 x + 1 < x + 5 g (x + 1) < g (x + 5)
= 1 1 = f (g (x + 1)) > f(g (x + 5))
2 a2 b2 Again x < x + 1 g (x + 1)
a2 b2 f (g (x)) > f (g (x + 1)) x < x + 2
f (x) > f (x + 2)
ab a 2 b2 g (f (x)) > g (f (x + 1)) x > x – 2
Area of the rectangle = p1 p2 = f (x) < f (x – 2)
a 2 b2
g (f (x)) < g (f (x – 2))
96. (a) f ' (x) = 6x2 + 12x + 7 = 6 (x2 +2x) + 7 = 6
(x +1)2 + 1 100. (a) For A, B, C to speak in order of alphabets,
which is positive for all value of x. Hence 3 places out of 10 may be chosen first in
f(x) is monotonic increasing function. 10
C3 ways.
The remaining 7 persons can speak in 7!
97. (b) f (x) = cos x, 0 x < 1 ways. Hence, the number of ways in which
2 2
= – 2, x 1 all the 10 persons can speak is 10C3 .
f (x) changes sign from positive to nega- 10! 10!
tive from the left side of x = 1 to the right 7! = .
3! 6
side of x = 1. f (x) changes from an increas-
ing function to a decreasing function. y 2 ex e x
f (x) has a local maximum at x = 1. 101. (b)
1 ex e x
98. (b) The distance of point (1, 1, p) from the plane
Applying comp. and dividendo.
r . 3iˆ 4ˆj 12kˆ 13 0
y 1 2e x
or (in cartesian form) 3x + 4y – 12z + 13 = 0, is
x
e2x
3 y 2e
3 1 4 1 12 p 13
d1 1/2
2 1 y 1 y 1
32 42 12 x log = log
2 3 y 3 y
3 4 12p 13 20 12p
(i) Hence, the inverse of the function
169 13
1/2
The distance of the point (–3, 0, 1) from the ex e x
x 1
f (x) 2 is loge
plane x x 3 x
e e
3x + 4y – 12z + 13 = 0 is 102. (b) By changing x as –x and y as–y, both the
3 3 4 0 12 1 13 8 given equation remains unchanged so
d2 required area will be symmetric with respect
2 13
32 42 12 to both the axis, which is shown in the
figure. So,
According to the given condition,
d1 = d2 1
Required area = 4 1 x2 (1 x) dx
20 12p 8
0
13 13
EBD_7443
www.jeeneetbooks.in

WWW.IIT-NEET.XYZ

MT-244 Target VITEEE


1 x 4/3 3 2z x
x 1 x2 lim
=4 1 x2 sin 1 x x x 0 x 4/3 [ 3 8z 4x 3
8z ]4
2 2 2
0
3
2z 1
1 1 4 23/3
= 4 0 . 1 = –2 2 3 8z 2 .z
2 2 2
dx x 2 dx
Y 105. (b)
x 1 x3 x3 1 x3
3
Put 1 – x = t 2 –3x2dx = 2t dt
2
tdt 1 1 t
3 2 dt log
integral =
O (1 t 2 ) t 3 1 t2 3 1 t
X

1 1 x3 1
log C
3 1 x3 1
1
106. (a) f (x) ex ex f (x) dt = ex + kex, where
0
cos ax cos bx
103. (c) We have, lim 1
x 0 cos cx 1
k f (t) dt
(a b) (a b)x 0
2 sin x sin 1
2 2
= lim k (e t ke t ) dt e ke 1 k ,
x 0 cx
2 sin 2 0
2
e 1
k
(a b)x (a b)x 2 e
sin .sin 2
2 2 . x e 1 ex
= lim 2
cx Thus f (x) ex 1
x 0
x sin 2 2 e 2 e
2 107. (c) The sphere is the solid of revolution
2 generated by the revolution of a semi
(a b)x (a b)x cx 4 circular area about its diameter.
sin sin 2
2 2 2 c
= lim
x 0 (a b)x 2
.
(a b)x 2
.
cx
y
. . sin 2
2 a b 2 a b 2

a b a b 4 a 2 b2
= 2 . Hence m
2 2 c c2
and n are
a2 – b2 and c2 respectively. x
B(–a,0) O A(a,0)
3 2
x z (z x) 2
104. (b) lim
x 0 3 4
8xz 4x 2 3
8xz Let the equation of the circle be x2 + y2 = a2
for the semi circle about the x-axis the
3 variable varies form x = –a to x = a.
x 2xz x 2 Volume of the sphere
lim
x 0 (3 x 3 8z 4x 3
8z 3 x ) 4
www.jeeneetbooks.in

WWW.IIT-NEET.XYZ

Solutions MT-245

a a Angle between two lines is given by


2 2 2
= y .dy (a x ) dx a1a 2 b1b 2 c1c 2
a a
cos
a12 b12 c12 a 22 b 22 c 22
a
2 (a 2 x 2 )dx 1 1 1 1
0 ( 1) ( 1)
2 6 3 4
( a2 – x2 is an even function) cos 0
1 1 1 1
a 1 1
2 x3 a3 4 9 36 16
2 a x 2 a3
3 0 3 90
112. (c) The number of ways of getting the different
4 3 numbers 1, 2, ....., 6 in six dice = 6 !.
a cub. units.
3 Total number of ways = 66
dy ax h Hence, required probability
108. (c) (by + k) dy = (ax + h) dx 6! 1 2 3 4 5 6 5
dx by k = 6 6
6 6 324
a 2
by2 + ky = x + hx + C
2 113. (a) a , b , c are coplanar vectors,
For this to represent a parabola, one of the
two terms x2 or y2 is zero. 2 a b , 2 b c , and 2 c a are also
Therefore, either a = 0, b 0 or a 0 , b= 0 coplanar vectors.
dy
109. (a) We have (ex +e–x) = (ex – e–x)
dx Thus 2 a b 2b c 2c a 0
dy ex e x
114. (b)
dx ex e x
115. (a) mean = np 4 and variance = npq = 2
x x
e e 1
Integrating y = dx + c p q and n 8
ex e x 2
Put e + e = t so that (e –e–x) dx = dt
x –x x
6 2
8 1 1 28
dt P(2 success) C2
y= + c = log | t | + c 2 2 28
t
k
Hence y = log | ex + e–x | + c
which is the reqd. general solution. 116. (c) P(x k) e
k!
110. (d) a, b, c are coplanar
[a b c] = 0 P(x 2) 1 P ( x 0) P ( x 1)

1 1 1 3
1 e e 1 .
1 2 1 =0 1! e2
3 p 5 117. (c) Let the normal at ‘t1’ cuts the parabola again
at the point ‘t2’. the equation of the normal
(10 + p + 3) – (6 – 5 – p) = 0 p=–6 at (at12, 2at1) is y + t1x = 2at1 + at13
111. (d) The given lines may be written as Since it passes through the point ‘t2’ i.e
x y z x y z (at22, 2at2)
and 2at2 + at1t22 = 2at1 + at13
1/ 2 1/ 3 1 1/ 6 1 1/ 4
2a(t1 – t2) + at1(t12 – t22) = 0
1 1 2 + t1(t1 + t2) = 0 ( t1 t 2 0)
Direction ratios of two lines are , , 1
2 3 2 + t12 + t1t2 = 0
1 1 2
and , 1, t2 t1
6 4 t1t2 = –(t12 + 2) t1
EBD_7443
www.jeeneetbooks.in

WWW.IIT-NEET.XYZ

MT-246 Target VITEEE


118. (a) Reflexive: For any x R, we have Y
x x 2 2 an irrational number..
x R x for all x. So, R is reflexive. (0, 3)
Symmetric: R is not symmetric, because 2
(0, 2) (6/5, 6/5)
R 1 but 1R 2 ,
Transitive: R is not transitive also because X' X
(2, 0) (3, 0)
2x +
2 R 1 and 1 R 2 2 but 2 R 2 2 . 3x 3y =
6
119. (a) For a, b Z' a + b Z' +
2y
Z' is closed under addition =
6
Also, a + b = b + a Y'
Addition on Z' is commutative Shaded portion shows the feasible region.
a b c a b c , a, b, c Z '. Now, the corner points are
Addition is associative on Z'. 6 6
(0, 2), (2,0), , , (0, 0).
a 0 0 a a, a Z ' 5 5
0 Z'is an additive identity At (0, 2),
< Z', + > is a monoid. value of z = – 2(0) – 3(2) = – 6
120. (a) Given problem is max z = – 2x – 3y At (2, 0),
x y x y value of z = – 2(2) – 3(0) = – 4
Subject to 1, 1 , x, y 0
2 3 3 2 6 6 6 6
First convert these inequations into At , , Value of z = – 2 –3
5 5 5 5
equations we get
3x + 2y = 6 ...(i) 30
2x + 3y = 6 ...(ii) = =–6
5
on solving these two equation, we get point At (0, 0), value of z = – 2(0) – 3(0) = 0
6 6 The max value of z is 0.
of intersection is , . 121 (c)
5 5 122 (a)
Now, we draw the graph of these lines. 123. (a)
124. (d)
125 (b)

You might also like